Download as pdf or txt
Download as pdf or txt
You are on page 1of 550

INTERMEDIATE

Paper 7

DIRECT TAXATION
Study Notes
SYLLABUS 2022

The Institute of Cost Accountants of India


CMA Bhawan, 12, Sudder Street, Kolkata - 700 016
www.icmai.in
First Edition : August 2022
Reprint : November 2022
Reprint : January 2023
Reprint : March 2023
Reprint : June 2023
Reprint : August 2023
Revised Edition : February 2024

Price: ` 600.00 (Direct and Indirect Taxation - 2 Books)

Published by :

Directorate of Studies
The Institute of Cost Accountants of India
CMA Bhawan, 12, Sudder Street, Kolkata - 700 016
[email protected]

Printed at :

M/s. Infinity Advertising Services Pvt. Ltd.


Plot No. 171 & 172, Sector - 58, Faridabad,
Haryana - 121 004

Copyright of these Study Notes is reserved by the Institute of Cost Accountants of India and prior permission
from the Institute is necessary for reproduction of the whole or any part thereof.
Copyright © 2022 by The Institute of Cost Accountants of India
PAPER 7 : DIRECT TAXATION
SECTION - A
Syllabus Structure:

The syllabus in this paper comprises the following topics and study weightage:

Module No. Module Description Weight

Section A: Direct Taxation 50%

1 Basics of Income Tax Act 10%

2 Heads of Income 25%

3 Total Income and Tax Liability of Individuals & HUF 15%


Learning Environment
Subject Title DIRECT TAXATION (Section - A)
Subject Code DITX
Paper No. 7
Course This section deals with the provisions of Income Tax Act, 1961 relating to computation of income under
Description various heads and determination of total income and tax liability of individuals and HUFs. It also covers
relevant provisions for filing of return and assessment procedure.
CMA Course 1. Interpret and appreciate emerging national and global concerns affecting organizations and be in a state
Learning of readiness for business management.
Objectives
a. Identify emerging national and global forces responsible for enhanced/varied business challenges.
(CMLOs)
b. Assess how far these forces pose threats to the status-quo and creating new opportunities.
c. Find out ways and means to convert challenges into opportunities
2. Acquire skill sets for critical thinking, analyses and evaluations, comprehension, syntheses, and
applications for optimization of sustainable goals.
a. Be equipped with the appropriate tools for analyses of business risks and hurdles.
b. Learn to apply tools and systems for evaluation of decision alternatives with a 360-degree
approach.
c. Develop solutions through critical thinking to optimize sustainable goals.
3. Develop an understanding of strategic, financial, cost and risk-enabled performance management in a
dynamic business environment.
a. Study the impacts of dynamic business environment on existing business strategies.
b. Learn to adopt, adapt and innovate financial, cost and operating strategies to cope up with the
dynamic business environment.
c. Come up with strategies and tactics that create sustainable competitive advantages.
4. Learn to design the optimal approach for management of legal, institutional, regulatory and ESG
frameworks, stakeholders’ dynamics; monitoring, control, and reporting with application-oriented
knowledge.
a. Develop an understanding of the legal, institutional and regulatory and ESG frameworks within
which a firm operates.
b. Learn to articulate optimal responses to the changes in the above frameworks.
c. Appreciate stakeholders’ dynamics and expectations, and develop appropriate reporting
mechanisms to address their concerns.
5. Prepare to adopt an integrated cross functional approach for decision management and execution with
cost leadership, optimized value creations and deliveries.
a. Acquire knowledge of cross functional tools for decision management.
b. Take an industry specific approach towards cost optimization, and control to achieve sustainable
cost leadership.
c. Attain exclusive knowledge of data science and engineering to analyze and create value.
Subject Direct Taxation
Learning 1. To acquire application-oriented knowledge and skill for appreciating various provisions of the Income
Objectives
Tax Act, 1961 and attain abilities to solve problems while computing tax liabilities. (CMLO 4 a)
[SLOB(s)]
2. To attain abilities to apply various provisions of direct taxation laws, including assessment to identify
the impacts thereof on business decisions. (CMLO 3a and b)
3. To gather knowledge of various compliance related provisions of direct taxation laws and attain
abilities to ensure due compliance to avoid any eventual risks. (CMLO 4 c)
Subject SLOCs:
Learning 1. Students will be able to appropriately apply various provisions of direct taxation laws and perform tax
Outcome
computation of taxable income and tax liability for different types of assessees.
[SLOC(s)]
and 2. They will be able to ensure compliance of legal provisions related to direct taxes.
Application APSs:
Skill [APS]
1. Students will acquire skill to compute taxable income and tax liabilities of various person like
individual, Hindu Undivided Family, Firm, Co-operative Society, Political Parties and Association of
Persons & Body of Individuals.
2. Students will attain skill sets for solving computation related issues of direct taxes.
3. Students will be able to ensure compliances of direct taxation related provisions.

Module wise Mapping of SLOB(s)

Module Additional Resources (Research


Topics and Sub-topics SLOB Mapped
No. articles, case studies, blogs)
1 Basics of Income tax Act Common for Modules of Direct 1. To acquire application-oriented
Taxation 1,2, & 3: knowledge and skill for appreciating
various provisions of the Income
Income-tax Act, 1961 (Bare Act) and the Tax Act, 1961 and attain abilities to
Income-tax Rules solve problems while computing tax
www.incometaxindia.gov.in liabilities.
2. To attain abilities to apply various
For this Module: provisions of direct taxation laws,
Refer Chapter I, II & III of the Income including assessment to identify the
Tax Act. impacts thereof on business decisions.
2 Heads of income Refer Chapter IV of the Income-tax Act, 3. To gather knowledge of various
compliance related provisions of
1961 (Bare Act)
direct taxation laws and attain abilities
3 Total income and tax liability Refer Chapter V, VI & VIA of the to ensure due compliance to avoid any
of individuals and HUF Income-tax Act, 1961 (Bare Act) eventual risks.
Contents as per Syllabus
SECTION A : DIRECT TAXATION 01 - 542
Module 1. Basics of Income Tax Act 01 - 64
1.1 Basic Concepts, Basis of Charge and Capital & Revenue Receipts
1.2 Residential Status and Scope of Total Income
1.3 Agricultural Income
1.4 Income which do not form part of Total Income
Module 2. Heads of Income 65 - 356
2.1 Salaries
2.2 Income from House Property
2.3 Profits and Gains of Business or Profession including Tax Audit u/s 44AB;
and Provisions u/s 43A, 43B, 43AA, 44AD, 44ADA and 44AE (excluding
Sections 42 to 44DB)
2.4 Capital Gains
2.5 Income from Other Sources
Module 3. Total Income and Tax Liability of Individuals & HUF 357 - 542
3.1 Income of Other Person included in Assesses Total Income (Clubbing of
Income)
3.2 Set off and Carry Forward of Losses
3.3 Deductions, Rebate and Relief
3.4 Taxation of Individual (including AMT but excluding Non-resident) & HUF
3.5 Advance Tax
3.6 Tax Deducted at Source & Tax Collected at Source (excluding Non-resident)
3.7 Filing of Return of Income
3.8 PAN
3.9 Self-Assessment & Intimation
SECTION - A
DIRECT TAXATION
Basics of Income Tax Act 1
This Module includes

1.1 Basic Concepts, Basis of Charge and Capital & Revenue Receipts

1.2 Residential Status and Scope of Total Income

1.3 Agricultural Income

1.4 Income which do not form part of Total Income

The Institute of Cost Accountants of India 1


Basics of Income Tax Act
SLOB Mapped against the Module:
1. To acquire application-oriented knowledge and skill for appreciating various provisions of the Income
Tax Act, 1961 and attain abilities to solve problems while computing tax liabilities.
2. To attain abilities to apply various provisions of direct taxation laws, including assessment to identify
the impacts thereof on business decisions.
3. To gather knowledge of various compliance related provisions of direct taxation laws and attain
abilities to ensure due compliance to avoid any eventual risks.

Module Learning Objectives:


After studying this module, the students will be able to –
 Appreciate the various type of taxations
 Appreciate the source of tax laws
 Understand the meaning of various terms like previous year, assessment year, etc.
 Appreciate the provisions relating to residential status
 Apply the knowledge to ascertain the taxability of income in India
 Appreciate various income which are not liable to be taxed in India

2 The Institute of Cost Accountants of India


Basics of Income Tax Act

Basic Concepts, Basis of Charge and


1.1
Capital & Revenue Receipts

T
ax is the compulsory levy by the government on income, commodity, services, activities or transaction.
The word ‘tax’ derived from the Latin word ‘Taxo’. Taxes are the basic source of revenue for the
government, which are utilized for the welfare of the people of the country through government policies,
provisions and practices. Income Tax is levied on the total income of the previous year of every person,
subject to residential status of that person. Further, few of the income of the person are not subject to income tax,
those income are termed as exempted income.
Basic Reasons to impose taxation
 To provide basic facilities for every citizen of the country: Whatever money is received by the government
from taxation is spent by it for the welfare of the citizens of the country. Some of the services provided by
the government are: health care, electricity, roads, education system, free houses for the poor, water supply,
police, firefighters, judiciary system, disaster relief, taking care of bridges and other things of public welfare.
 To finance multiple governments: All the local governments of the state like village panchayats, block
panchayats and municipal corporations receive funds from the finance commission.
 Protection of the life: Taxpayers receive the protection of life and wealth from the government in case of
external aggression, internal armed rebellion or any other situation.

1.1.1 Constitutional Validity of taxes


The Constitution of India is the supreme law of India. It consists of a Preamble, 22 parts containing 444 articles and
12 schedules. Any tax law, which is not in conformity with the Constitution, is called ultra vires the Constitution
and held as illegal and void. Some of the provisions of the Constitution are given below:
Article 265 of the Constitution lays down that no tax shall be levied or collected except by the authority of law. It
means the tax proposed to be levied must be within the legislative competence of the legislature imposing the tax1.

Article 246 read with Schedule VII divides the subject matter of law made
by the legislature into three categories:
¾¾ Union list (only the Central Government has the power of legislation on
subject matters covered in the list)
¾¾ State list (only the State Government has the power of legislation on
subject matters covered in the list)
¾¾ Concurrent list (both Central & State governments can pass legislation on
subject matters). If a state law relating to an entry in List III is repugnant
to a Union law relating to that entry, the Union law will prevail, and the
state law shall, to the extent of such repugnancy, be void. (Article 254).

1 Kunnathat Thathunni Moopil Nair –vs.- The State of Kerala 1961 AIR 552 (SC)

The Institute of Cost Accountants of India 3


Direct Taxation

1.1.2 Administration of tax laws


The administrative hierarchy of tax law is as follows:
Taxpoint:

 Both of the Boards have been constituted


under the Central Board of Revenue
Act, 1963.
 CBDT deals with levy and collection of
all direct tax whereas matters relating to
levy and collection of Central indirect
tax are dealt by CBIC.

1.1.3 Sources of Income Tax Law in India


1. Income tax Act, 1961 (Amended up to date)

The provisions of income tax extend to the


whole of India and became effective from
1/4/1962 (Sec. 1). The Act contains provisions
for:
(a) determination of taxable income;
Source of Income Tax
(b) determination of tax liability;
(c) procedure for assessment, appeals, penalties and prosecutions; and
(d) powers and duties of Income-tax authorities.
Taxpoint: The Income-tax Act, 1961 has been divided into 23 chapters (covering 298 sections) and 14
schedules. Few of them are further sub-divided.
2. Annual Amendments
(a) Income-tax tax Act has undergone several amendments from the time it was originally enacted through
the Union Budget2. Every year, a Finance Bill (a part of the Union Budget) is presented before the
Parliament by the Finance Minister. The Bill contains various amendments which are sought to be made
in the areas of direct and indirect taxes levied by the Central Government.
(b) When the Finance Bill is approved by both the Houses of Parliament and receives the assent of the
President, it becomes the Finance Act. The provisions of such Finance Act are thereafter incorporated in
the Income Tax Act.
(c) If on the 1st day of April of the Assessment Year, the new Finance Act has not been enacted, the provisions
in force in the preceding Assessment Year or the provisions proposed in the Finance Bill before the
Parliament, whichever is more beneficial to the assessee, will apply until the new provisions become
effective [Sec. 294]

2 According to Article 112 of the Indian Constitution, the Union Budget of a year is a statement of the estimated receipts and expenditure of
the government for that particular year. Union Budget is classified into Revenue Budget and Capital Budget.
Revenue budget includes the government’s revenue receipts and expenditure. There are 2 kinds of revenue receipts - tax and non-tax revenue.
If revenue expenditure exceeds revenue receipts, the government incurs a revenue deficit. Capital Budget includes capital receipts and
payments of the government. Loans from public, foreign governments and RBI form a major part of the government’s capital receipts. Fiscal
deficit is incurred when the government’s total expenditure exceeds its total revenue.

4 The Institute of Cost Accountants of India


Basics of Income Tax Act

Note: Besides these amendments, whenever it is found necessary, the Government introduces amendments in
the form of various Amendment Acts and Ordinances.
3. Income tax Rules, 1962 (Amended up to date)
(a) As per Sec. 295, the Board may, subject to the control of the Central Government, make rules for the
whole or any part of India for carrying out the purposes of the Act.
(b) Such rules are made applicable by notification in the Gazette of India.
(c) These rules were first made in 1962 and are known as Income tax Rules, 1962.
Since then, many new rules have been framed or existing rules have been amended from time to time and the
same has been incorporated in the aforesaid rules.
4. Circulars and Clarifications by CBDT
(a) U/s 119, the Board may issue certain circulars and clarifications from time to time, which have to be
followed and applied by the Income tax authorities.
(b) Effect of circulars: These circulars or clarifications are binding upon the Income tax authorities, but the
same are not binding on the assessee. However, assessee can claim benefit under such circulars.
Note: These circulars are not binding on the Income Tax Appellate Tribunal or on the Courts.
5. Judicial decision
(a) Decision of the Supreme Court: Any decision given by the Supreme Court shall be applicable as law till
there is any change in law by the Parliament. Such decision shall be binding on all the Courts, Tribunals,
Income tax authorities, assessee, etc.
(b) Contradiction in the decisions of the Supreme Court: In case, there is apparently contradiction in two
decisions, the decision of larger bench, whether earlier or later, shall always prevail. However, where
decisions are given by benches having equal number of judges, the decision of the recent case shall be
applicable.
(c) Decisions given by a High Court or ITAT: Decisions given by a High Court or ITAT are binding on all
assessees and Income tax authorities, which fall under their jurisdiction, unless it is over ruled by a higher
authority.

1.1.4 Basic principles for charging Income Tax [Sec. 4]


1. Income of the previous year of a person is charged to tax in the immediately following assessment year.
2. Rate of tax is applicable as specified by the Annual Finance Act of that year. Further, though the Finance
Act prescribes the rates of tax, in respect of certain income, the Income Tax Act itself has prescribed specific
rates, e.g. Lottery income is to be taxed @ 30% (Sec.115BB), Long term capital gain is to be taxed @ 20%
(Sec.112), short term capital gain on listed shares u/s 111A is to be taxed @ 15%, etc.
3. In respect of income chargeable to tax, tax shall be deducted at source, or paid in advance (wherever applicable).
Sec. 4 is a charging section and it is the backbone of the Income Tax Act. The tax liability arises by virtue of this
section and it arises at the close of a previous year. However, the finalisation of amount of tax liability is postponed
to the assessment year. It follows the rule that the liability to tax is not dependent upon assessment.

The Institute of Cost Accountants of India 5


Direct Taxation

1.1.5 Assessment Year (A.Y.) [Sec. 2(9)]


Assessment year means the period of 12 months commencing on the 1st day of April every year. It is the year (just
after the previous year) in which income earned in the previous year is charged to tax. E.g., A.Y.2024-25 is a year,
which commences on April 1, 2024 and ends on March 31, 2025. Income of an assessee earned in the previous year
2023-24 is assessed in the A.Y. 2024-25.

1.1.6 Previous Year or Uniform Previous Year [Sec.3]


Previous Year means the financial year immediately preceding the Assessment Year. Income earned in a year is
assessed in the next year. The year in which income is earned is known as Previous Year and the next year in which
income is assessed is known as Assessment Year. It is mandatory for all assessee to follow financial year (from 1st
April to 31st March) as previous year for Income-Tax purpose.
Financial Year
According to sec. 2(21) of the General Clauses Act, 1897, a Financial Year means the year commencing on the 1st
day of April. Hence, it is a period of 12 months starting from 1st April and ending on 31st March of the next year. It
plays a dual role i.e. Assessment Year as well as Previous Year.

Example 1: Financial year 2023-24 is -


 Assessment year for the Previous Year 2022-23; and
 Previous Year for the Assessment Year 2024-25.

Determination of the first previous year in case of a newly set-up business or profession or for a new source
of income
In case of Previous year is the period
Business or profession being Beginning with the date of setting up of the business & ending on 31st March
newly set-up of that financial year.
A source of income newly Beginning with the date on which the new source of income comes into
coming into existence existence & ending on 31st March of that financial year.
Note: Calender year cannot be considered as previous year though calender year is followed as accounting
year by a person

Exceptions to the general rule that income of a Previous Year is taxed in its Assessment Year
This is the general rule that income of the
previous year of an assessee is charged to
tax in the immediately following assessment
year. However, in the following cases, income
of the previous year is assessed in the same
year in order to ensure smooth collection of
income tax from the taxpayer who may not
be traceable, if assessment is postponed till
the commencement of the Assessment Year:

6 The Institute of Cost Accountants of India


Basics of Income Tax Act

1. Income of a non-resident assessee from shipping business (Sec. 172)


2. Income of a person who is leaving India either permanently or for a long period (Sec. 174)
3. Income of bodies, formed for a short duration (Sec. 174A)
4. Income of a person who is likely to transfer property to avoid tax (Sec. 175)
5. Income of a discontinued business (Sec. 176). In this case, the Assessing Officer has the discretionary power
i.e. he may assess the income in the same previous year or may wait till the Assessment year.

1.1.7 Assessee [Sec. 2(7)]


“Assessee” means,
a. a person by whom any tax or any other sum of money (i.e., penalty or interest) is payable under this Act
(irrespective of the fact whether any proceeding under the Act has been taken against him or not);
b. every person in respect of whom any proceeding under this Act has been taken (whether or not he is liable for
any tax, interest or penalty) for the assessment of his income or loss or the amount of refund due to him;
c. a person who is assessable in respect of income or loss of another person;
d. every person who is deemed to be an assessee under any provision of this Act; and
e. a person who is deemed to be an ‘assessee in default’ under any provision of this Act. E.g. A person, who was
liable to deduct tax but has failed to do so, shall be treated as an ‘assessee in default’.

1.1.8 Person [Sec. 2 (31)]


The term person includes the following:

¾¾ an Individual;
¾¾ a Hindu Undivided Family (includes Jain and Sikh
family);
¾¾ a Company;
¾¾ a Firm (includes LLP);
¾¾ an Association of Persons (AOP) or a Body of
Individuals (BOI), whether incorporated or not;
¾¾ a Local authority; &
¾¾ every artificial juridical person not falling within
any of the preceding categories.

Illustration 1:
Determine the status of the following:

Case Status
(a) Howrah Municipal Corporation Local authority
(b) Corporation Bank Ltd. Company
(c) Mr. Amitabh Bachchan Individual

The Institute of Cost Accountants of India 7


Direct Taxation

Case Status
(d) Amitabh Bachchan Corporation Ltd. Company
(e) A joint family of Sri Ram, Smt. Ram and their son Lav and Kush HUF
(f) Calcutta University Artificial juridical person
(g) X and Y who are legal heirs of Z BOI
(h) Sole proprietorship business Individual
(i) Partnership Business Firm
(j) Reserve Bank of India Artificial juridical person

1.1.9 Income [Sec. 2(24)]


To consider any receipt as income, following points should be kept in mind:-

Cash vs. Kind Income may be received in cash or in kind. Income received in kind is to be valued as per
the rules prescribed and if there is no specific direction regarding valuation in the Act or
Rules, it may be valued at market price.
Significance Method of accounting is irrelevant In case of income under the head “Salaries”,
of method of “Income from house property” and “Capital gains”
accounting method of accounting is irrelevant.
Method of accounting is relevant In case of income under the head “Profits & gains
of business or profession” and “Income from other
sources” (other than Dividend) income shall be
taxable on cash or accrual basis as per the method
of accountancy regularly followed by the assessee.
Notional income A person cannot make profit out of transaction with himself. Hence, goods transferred
from one department to another department at a profit, shall not be treated as income of
the business.
Source of income Income may be from a temporary source or from a permanent source.
Capital vs. A capital receipt is not liable to tax, unless specifically provided in the Act, whereas, a
Revenue receipt revenue receipt is not exempted, unless specifically provided in the Act. (Further refer
following heading)
Loss Income also includes negative income.
Disputed income In case of dispute regarding the title of income, assessment of income cannot be withheld
and such income, normally, be taxed in the hands of recipient.
Lump-sum receipt There is no difference between income received in lump sum or in installment.
Reimbursement Mere reimbursement of expenses is not an income.
Legality The Act does not make any difference between legal or illegal income.

8 The Institute of Cost Accountants of India


Basics of Income Tax Act

Double taxation Same income cannot be taxed twice.


Income by mutual In this regard it is to be noted that in case of mutual activities, where some people contribute
activity to the common fund and are entitled to participate in the fund and the surplus arises which
is distributed among the contributors of the fund, such surplus cannot be termed as income.
Exceptions:
¾¾ Income derived by a trade, professional or similar association from rendering specific
services to its members shall be taxable u/s 28(iii).
¾¾ Profits and gains of any insurance business carried on by a mutual insurance company
or by a co-operative society.
¾¾ Profits and gains of any business of banking (including providing credit facilities)
carried on by a co-operative society with its members.
Pin money Pin money is money received by wife for her personal expenses & small savings made by
a woman from money received from her husband for meeting household expenses. Such
receipt is not treated as income.
Note: Income on investment out of pin money shall be treated as income.
Award Award received, by a person related to his business or profession, shall be treated as income
incidental to such business or profession. However, award received by a non-professional
person is in nature of gift and/or personal testimonial, the taxability thereof is subject to
other provisions of the Act
Embezzlement Money embezzled is a gain to the embezzler and, therefore, falls within the wider definition
of income
Contingent A contingent or anticipated income is not taxable.
income
Subsidy Assistance in the form of a subsidy or grant or cash incentive or duty drawback or waiver
or concession or reimbursement (by whatever name called) by the Central Government or
a State Government or any authority or body or agency in cash or kind to the assesse, e.g.
LPG Subsidy3, Subsidy for establishing manufacturing unit in backward area, etc.

However,
a. subsidy or grant or reimbursement which is taken into account for determination of the
actual cost of the asset as per Explanation 10 to sec. 43(1) is not taxable separately.
b. the subsidy or grant by the Central Government for the purpose of the corpus of a trust
or institution established by the Central Government or a State Government
- shall not be taxable.

3 Finance Ministry has clarified that LPG subsidy received by an individuals in their bank accounts will continue to be exempt from income tax.

The Institute of Cost Accountants of India 9


Direct Taxation

1.1.10 Heads of Income [Sec. 14]


According to Sec.14 of the Act, all income of a person shall be classified under the following five heads:

1. Salaries;
2. Income from house property;
3. Profits and gains of business or profession;
4. Capital gains;
5. Income from other sources.
For computation of income, all taxable income
should fall under any of the five heads of income
as mentioned above. If any type of income does not
become part of any one of the above mentioned first
four heads, it should be part of the fifth head, i.e.
Income from other sources, which may be termed as
the residual head.

Computation of Total Income for the A.Y.


Particulars Amount
Salaries ***
Income from house property ***
Profits and gains of business or profession ***
Capital gains ***
Income from other sources ***
Gross Total Income ****
Less: Deduction under chapter VIA (Sec 80C to 80U) ****
Total Income ****

1.1.11 Rounding-off of total income [SEC. 288A]


The total income so computed will have to be rounded off to the nearest multiple of ₹ 10, i.e., if the last figure in
the ‘rupee element’ is ₹ 5 or more, it should be rounded off to the next higher amount, which is a multiple of ₹ 10.
The ‘paise’ element should be ignored.
Thus, if the total income works out to ₹ 41,645, it should be rounded off to ₹ 41,650, but if it works out to
₹ 41,644.98, it should be rounded off to ₹ 41,640.

1.1.12 Rounding-off of Tax [SEC. 288B]


The tax calculated on the total income should be rounded off to the nearest ₹ 10. Amount of tax (including TDS or
advance tax), interest, penalty, etc. and refund shall be rounded off to the nearest ₹ 10.
Provision illustrated

Tax liability actually worked out (₹) 4,876.49 6,452.50 8,738.92 5,132.75

Tax liability as rounded off (₹) 4,880 6,450 8,740 5,130

10 The Institute of Cost Accountants of India


Basics of Income Tax Act

1.1.13 Capital -vs.- Revenue


Receipts
A capital receipt is not liable to tax, unless specifically provided in the Act, whereas, a revenue receipt is not
exempted, unless specifically provided in the Act. Further, capital receipts are to be charged to tax under the head
“Capital Gains” and revenue receipts are taxable under other heads. The Act does not provide exhaustive definition
of the income, thus, distinction between capital receipts and revenue receipts is not easily made. However, based
on a number of judicial pronouncements, the following principles are worthwhile to note:
1. Receipt in lump sum or in Instalments: Whether any income is received in lump sum or in instalments, it
will not make any difference as regards its nature, e.g., an employee is to get a salary of ₹ 10,000 p.m. Instead
of this he enters into an agreement to get a sum of ₹ 3,60,000 in lump sum to serve for a period of 3 years. The
receipt where it is monthly remuneration or lump sum for 3 years is a revenue receipt.
2. Nature of receipt in the hands of recipient: Whether a receipt is capital or revenue will be determined in
the hands of the persons receiving such income. No attention will be paid towards the source from which the
amount is coming. Salary even if paid out of capital by a new business will be it revenue receipt in the hands
of employee.
3. Accounting treatment: The name given to the transaction by the parties involved or its treatment in the books
of account may not alter its character as capital or revenue.
4. Income from wasting assets: Profits from capital which is consumed and exhausted in the process of
realization, e.g. royalties from mines and quarries, is taxable as income regardless of the consumption of
capital involved in the process.
5. Magnitude of receipt: The magnitude of the receipt, whether big or small, cannot decide the nature of the
receipt.
6. Time of receipt: The nature of the receipt has to be determined at the time when it is received and not
afterwards when it has been appropriated by the recipient.
7. Quality of receipt: Whether the income is received voluntarily or under a legal obligation, it will not make
any difference as regards its nature.
8. Tests as to the purpose of keeping an article: If a person purchases a piece of sculpture to keep as decoration
piece in his house, if sold later on, will bring capital receipt but if the same sculpture is sold by an art dealer it
will be his revenue receipt.

Instances of transactions which are capital in nature but specifically taxable:


1. Capital gains arising from sale of capital assets being defined u/s 2(14). [Sec. 45]
2. Compensation for termination of service or modification in the terms of service [Sec. 17(3)]
3. Compensation or other payments due to or received by the persons specified u/s 28(ii)/28(va).

Expenses
Similarly, a capital expenditure is not allowable as expenses, unless specifically allowed in the Act, whereas, a
revenue expenditure is allowable as expenses, unless specifically disallowed in the Act. Based on a number of
judicial pronouncements, the following principles are worthwhile to note:
1. Acquiring asset or advantage of enduring nature: Bringing into existence an asset or advantage of enduring
nature4 would lead to the inference that the expenditure disbursed is of a capital nature.

4 ‘enduring’ does not mean ‘everlasting’ or ‘perpetual’.

The Institute of Cost Accountants of India 11


Direct Taxation

2. Capital assets belonging to third parties: Even though a expenditure results in the creation of a capital asset,
if the capital asset belongs to a third party, such expenses will be treated as revenue expenditure.
3. Profit-earning process: Where the outgoing expenditure is so related to the carrying on or the conduct of
the business that it may be regarded as an integral part of the profit-earning process and not for acquisition of
an asset or a right of a permanent character, the possession of which is a condition of the carrying on of the
business, the expenditure may be regarded as revenue expenditure
4. Object of the transaction: The object of the transaction which has impact on the business, the nature of trade
for which the expenditure is incurred and the purpose thereof, etc.
5. Fixed capital -vs.- Circulating capital: An item of disbursement may be regarded as of a capital nature when
it is relatable to a fixed capital, whereas if it is related to circulating capital or stock-in-trade it would be treated
as revenue expenditure.
6. Expenditure on removing restriction: Where the assessee has an existing right to carry on a business, any
expenditure made by it during the course of business for the purpose of removal of any restriction or obstruction
or disability would be on revenue account, provided the expenditure does not result in the acquisition of any
capital asset.
7. Payment made to rival dealer to ward off competition in business would constitute capital expenditure
8. If the expenditure is a part of the working expenses in ordinary commercial trading, it is not capital but revenue
expenditure.
9. If the expenditure is incurred for the initial outlay or for extension of business or substantial replacement of
equipment, it is capital expenditure but if it is incurred for running the business or is laid out as part of the
process of profit making, it is revenue in character.
10. If expenditure is incurred for ensuring the regular supply of raw material, maybe for period extending over
several years, it is on revenue account
11. When an owner incurs expenditure on additions in a building which enhances its value the expenditure can be
of a capital nature. But, if a tenant incurs an expenditure on a rented building for its renovation, he does not
acquire any capital asset, because the building does not belong to him and, ordinarily, such an expenditure will
be of a revenue nature.
12. Acquisition of the goodwill of the business is acquisition of a capital asset, and, therefore, its purchase price
would be capital expenditure. It would not make any difference whether it is paid in a lump sum at one time
or in instalments distributed over a definite period. Where, however, the transaction is not one for acquisition
of the goodwill, but for the right to use it, the expenditure would be revenue expenditure

13. Expenses incurred by the assessee for the purpose of creating, curing or completing the title is capital
expenditure and on the other hand if such expenses are incurred for the purpose of protecting the same, it is
revenue expenditure.

Illustration 2

Birla Ltd., a cement manufacturing company, entered into an agreement with a supplier for purchase of additional
cement plant. One of the conditions in the agreement was that if the supplier failed to supply the machinery
within the stipulated time, the company would be compensated at 5% of the price of the respective portion of
the machinery without proof of actual loss. The company received ₹ 8.50 lakhs from the supplier by way of
liquidated damages on account of his failure to supply the machinery within the stipulated time. What is the nature

12 The Institute of Cost Accountants of India


Basics of Income Tax Act

of liquidated damages received by Birla Ltd. from the supplier of plant for failure to supply machinery to the
company within the stipulated time — a capital receipt or a revenue receipt? [CMA – Inter Dec. 2011]

Answer:

In the case of CIT -vs.- Saurashtra Cement Ltd. (2010) 325 ITR 422, the Apex Court has held that the damages
were directly and intimately linked with the procurement of a capital asset, which lead to delay in coming into
existence of the profit-making apparatus. It was not a receipt in the course of profit earning process. Therefore,
the amount received by the assessee towards compensation for sterilization of the profit earning source, not in the
ordinary course of business, is a capital receipt in the hands of the assessee.

1.1.14 Diversion & Application of Income


There is a very thin line of difference between Diversion of income & Application of income.
Diversion of income: Where by virtue of an obligation, income is diverted before it reaches to the assessee, it is
known as diversion of income & it is not taxable (i.e. even if the assessee were to collect the income he does so on
behalf of the person to whom it is payable).
Example 2. A, B and C are co-authors of a book. The publisher of the book gave the whole royalty of Rs.6,00,000
to A. A paid Rs.2,00,000 to B and C each. Such payment is not application of income but diversion of income.
Application of income: Whereas, application of income means to discharge an obligation (which is gratuitous or
self-imposed) after such income reaches the assessee & hence it is taxable.

Annexure
TAX RATES FOR THE A. Y. 2024-25
Default Tax Regime for Individual / HUF / AOP / BOI / AJP [Sec. 115BAC]
Applicable to
Individual / HUF / AOP (other than co-operative society) / BOI / AJP
Rate of Tax
Under this tax regime, income tax shall be computed at the option of the assessee considering the following rate:
Total income Rate of tax
Upto ₹ 3,00,000 Nil
From ₹ 3,00,001 to ₹ 6,00,000 5%
From ₹ 6,00,001 to ₹ 9,00,000 10%
From ₹ 9,00,001 to ₹ 12,00,000 15%
From ₹ 12,00,001 to ₹ 15,00,000 20%
Above ₹ 15,00,000 30%

Taxpoint: If a person opts for this regime, ₹ 3,00,000 shall be considered as basic exemption limit irrespective
of his age. In other words, for all category of individual i.e, senior citizen, super senior citizen and others, basic
exemption limit is ₹ 3,00,000

Rebate u/s 87A for tax computed as per sec. 115BAC


Applicable to: Resident Individual
The Institute of Cost Accountants of India 13
Direct Taxation

Conditions to be satisfied: Total income of the assessee does not exceed ₹ 7,00,000.
Quantum of Rebate: Lower of the following:
a. 100% of tax liability as computed above; or
b. ₹ 25,000/-

Marginal relief is available even total income exceeds ₹ 7,00,000 [available upto ₹ 7,27,770]
Marginal relief = Positive value of (Tax on income – Income in excess of ₹ 7,00,000)
Example 3

Particulars Case 1 Case 2 Case 3 Case 4


Assessee Individual Individual Senior Citizen Individual
Residential status Resident Resident Resident
Regime Default Default Default Default
Total Income (₹) 6,00,000 6,80,000 7,10,000 7,30,000
Tax on above 15,000 23,000 26,000 28,000
Rebate u/s 87A 15,000 23,000 16,000 Nil
Reason [₹ 26,000 – (₹ 7,10,000 - [₹ 28,000 – (₹ 7,30,000 -
₹ 7,00,000)], is positive ₹ 7,00,000)], is negative
Tax after rebate Nil Nil 10,000 28,000

Surcharge on tax after rebate u/s 87A


Surcharge at the following rate is also payable on tax as computed above after rebate u/s 87A

Total Income Rate of Surcharge


Total income does not exceed ₹ 50 lacs Nil
Total income exceeds ₹ 50 lacs but does not exceed ₹ 1 crore 10% of tax
Total income exceeds ₹ 1 crore but does not exceed ₹ 2 crores 15% of tax
Total income exceeds ₹ 2 crores 25% of tax*
Subject to Marginal Relief.
* Where the total income includes dividend, any income chargeable u/s 111A, 112 and 112A, the surcharge on the
amount of income-tax computed on that part of income shall not exceed 15%. In other words, surcharge higher than
15% is applicable only on tax on income other than dividend, income covered u/s 111A, 112 and 112A. Moreover,
in case of an AOP consisting of only companies as its members, the rate of surcharge on the amount of Income-tax
shall not exceed 15%.

Health & Education Cess


Applicable on: All assessee
Rate of cess: 4% of Tax liability after Surcharge.
Section 115BAC has been discussed in details later.

14 The Institute of Cost Accountants of India


Basics of Income Tax Act

Old Tax Regime


Individual/HUF/Association of Persons/Body of Individuals/Artificial Juridical Person
In case of Super Senior citizen

Total Income Range Rates of Income Tax


Up to ₹ 5,00,000 Nil
₹ 5,00,001 to ₹ 10,00,000 20% of (Total income – ₹ 5,00,000)
₹ 10,00,001 and above ₹ 1,00,000 + 30% of (Total income – ₹ 10,00,000)
Super Senior Citizen means an individual who is resident in India and is of at least 80 years of age at any time
during the relevant previous year (i.e. any resident person, male or female, born before 02-04-1943).
In case of Senior citizen

Total Income Range Rates of Income Tax


Up to ₹ 3,00,000 Nil
₹ 3,00,001 to ₹ 5,00,000 5% of (Total Income – ₹ 3,00,000)
₹ 5,00,001 to ₹ 10,00,000 ₹ 10,000 + 20% of (Total income – ₹ 5,00,000)
₹ 10,00,001 and above ₹ 1,10,000 + 30% of (Total income – ₹ 10,00,000)
Senior Citizen means an individual who is resident in India and is of at least 60 years of age at any time during the
relevant previous year. (i.e., a resident person, male or female, born on or after 02-04-1943 but before 02-04-1963)
In case of other Individual1 / HUF / Association of Persons / Body of Individuals / Artificial Juridical Person

Total Income Range Rates of Income Tax


Up to₹ 2,50,000 Nil
₹ 2,50,001 to ₹ 5,00,000 5% of (Total Income – ₹ 2,50,000)
₹ 5,00,001 to ₹ 10,00,000 ₹ 12,500 + 20% of (Total income – ₹ 5,00,000)
₹ 10,00,001 and above ₹ 1,12,500 + 30% of (Total income – ₹ 10,00,000)
1
. born on or after 02-04-1963 or non-resident individual
Rebate u/s 87A
Applicable to: Resident Individual
Conditions to be satisfied: Total income of the assessee does not exceed ₹ 5,00,000.
Quantum of Rebate: Lower of the following:
(a) 100% of tax liability as computed above; or
(b) ₹ 12,500/-
Surcharge on tax after rebate u/s 87A

Surcharge at the following rate is also payable on tax as computed above after rebate u/s 87A

The Institute of Cost Accountants of India 15


Direct Taxation

Total Income Rate of Surcharge


Total income does not exceed ₹ 50 lacs Nil
Total income exceeds ₹ 50 lacs but does not exceed ₹ 1 crore 10% of tax
Total income exceeds ₹ 1 crore but does not exceed ₹ 2 crores 15% of tax
Total income exceeds ₹ 2 crores but does not exceed ₹ 5 crores 25% of tax*
Total income exceeds ₹ 5 crores 37% of tax*

* Where the total income includes dividend, any income chargeable u/s 111A, 112 and 112A, the surcharge on the
amount of income-tax computed on that part of income shall not exceed 15%. In other words, surcharge higher than
15% is applicable only on tax on income other than dividend, income covered u/s 111A, 112 and 112A. Moreover,
in case of an AOP consisting of only companies as its member, the rate of surcharge shall not exceed 15%.

Health & Education Cess


Applicable on: All assessee
Rate of cess: 4% of Tax liability after Surcharge

Marginal Relief
Example 4: Compute tax liability of the assessee (52 years) whose total income is:
(Case 1) ₹ 49,90,000 (Case 2) ₹ 50,10,000; (Case 3) ₹ 60,00,000

Particulars Working Case 1 Case 2 Case 3


Tax liability before Rebate ₹ 2,50,000 × Nil Nil Nil Nil
₹ 2,50,000 × 5% 12,500 12,500 12,500
₹ 5,00,000 × 20% 1,00,000 1,00,000 1,00,000
Balance Income × 30% 11,97,000 12,03,000 15,00,000
Total 13,09,500 13,15,500 16,12,500
Less: Rebate u/s 87A As income exceeds ₹ 5,00,000 Nil Nil Nil
Liability [A] 13,09,500 13,15,500 16,12,500
Add: Surcharge B = [10% of (A)] Nil 1,31,550 1,61,250
Tax and surcharge payable 13,09,500 14,47,050 17,73,750

Analysis of case (1) and case (2)

Increase in income ₹ 20,000


Liability for surcharge increased ₹ 1,31,550

16 The Institute of Cost Accountants of India


Basics of Income Tax Act

To provide relaxation from levy of surcharge to a taxpayer where the total income exceeds marginally above ₹50
lakh or ₹ 1 crore or 2 crores or 5 crores, the concept of marginal relief is designed.
Condition: Total income exceeds ₹ 50,00,000 (or ₹ 1 crore or 2 crores or 5 crores)
Relief: Marginal relief is provided to ensure that the additional income tax payable including surcharge on excess
of income over ₹ 50,00,000 or ₹ 1,00,00,000 or ₹ 2,00,00,000 or ₹ 5,00,00,000 is limited to the amount by which
the income is more than ₹ 50,00,000 or ₹ 1,00,00,000 or ₹ 2,00,00,000 or ₹ 5,00,00,000
Marginal relief = Calculated Surcharge - 70% (Income – ₹ 50,00,000)] (if positive)
Or
Marginal relief = [(Income tax + surcharge) on income] – [(Income tax on ₹ 50,00,000) + (Income –
₹ 50,00,000)]
Similar relief shall also be provided where income exceeds marginally above ₹ 1 crore or ₹ 2 crores or ₹ 5 crores.
In that case, the aforesaid equation shall be changed accordingly.
Now, computation of tax liability is made after considering marginal relief:
Particulars Working Case 1 Case 2 Case 3
Liability [A] 13,09,500 13,15,500 16,12,500
Add: Surcharge B = [10% of (A)] Nil 1,31,550 1,61,250
Tax and surcharge 13,09,500 14,47,050 17,73,750
Less: Marginal relief [(B) –{70% (50,10,000 – 50,00,000)}] Nil 1,24,550 Nil
Effective Surcharge [C] Nil 7,000 1,61,250
Liability after surcharge [A + C] 13,09,500 13,22,500 17,73,750
Add: Health & Education 4% of above 52,380 52,900 70,950
cess
Total Rounded off u/s 288B 13,61,880 13,75,400 18,44,700
Taxpoint: The concept of marginal relief is not applicable in case of cess.

An Individual / HUF/AOP/BOI/AJP can opt for alternative tax regime u/s 115BAC. The provision relating to
sec. 115BAC will be discussed in subsequent chapter.

Quick MCQs:-

1. Income Tax Act extends to –


(a) Whole of India
(b) Whole of India except Jammu & Kashmir
(c) Whole of India except Sikkim
(d) Option (a) except Jammu & Kashmir and Sikkim

2. Which one of the following is not treated as Deemed Assessee.


(a) Legal Representative of deceased person
(b) Agent of a Non-Resident
(c) Trustee of a Trust
(d) None of the above

The Institute of Cost Accountants of India 17


Direct Taxation

3. Person u/s. 2(31) does not include


(a) Minor
(b) Local Authority
(c) Unsound Person
(d) None of the above

4. Which of the following are Revenue Receipts?


(a) Bonus Shares received by a dealer of shares.
(b) Money received by a tyre manufacturing company for sale of technical know-how regarding
manufacturing of tyres.
(c) Premium on issue of new shares
(d) All of the above

5. Which of the following is a Capital Receipt?


(a) Perquisites received by a professional during the course of carrying on profession.
(b) Compensation received in respect of permanent disablement due to an accident.
(c) Compensation received in respect of temporary disablement due to an accident.
(d) All of the above

6. Which of the following is not a Capital Expenditure?


(a) Expenditure incurred in connection with the acquisition or installation of a Fixed Asset.
(b) Expenditure incurred in raising capital.
(c) Expenditure incurred for improving the profit earning capacity of an asset.
(d) Expenditure incurred for repairing an asset.

7. Which of the following is a Revenue Expenditure?


(a) Lumpsum payment made by a employer as a gratuity to the employee.
(b) Legal expenses incurred by a person in defending or maintaining his right or title to the property
used for business.
(c) Expenditure incurred for the purchase of goods for resale.
(d) All of the above

8. Receipt of amount on maturity of LIC Policy is


(a) Capital Receipt
(b) Casual Receipt
(c) Revenue Receipt
(d) None of the above

18 The Institute of Cost Accountants of India


Basics of Income Tax Act

Residential Status and Scope of Total


1.2
Income

T
he taxability of a person depends upon his residential status in India for any particular previous year. The
term residential status must not be confused with an individual’s citizenship in India. An individual may be
a citizen of India but may not be a resident for a particular previous year. Similarly, a foreign citizen may
be a resident of India for the purpose of income tax for a particular previous year. Residential status of an assessee
determines the scope of chargeability of his income. Whether a person will be charged to a particular income or
not, depends on his residential status.
Sec. 6 provides the test for residential status for the persons which can be categorized as under:

1.2.1 General points to be kept in mind regarding residential status of a person


Different for each Residential status is determined in respect of each previous year. In other words,
previous year residential status of a person may vary from one previous year to another previous year.
Single Status for each A person can have only one residential status for a previous year i.e. he cannot be a
source of income resident for one source of income and non-resident for another source.
Impact of citizenship Citizenship and residential status are two different concepts. A citizen of India may not
be a resident in India for the purpose of income-tax.
Country Specific A person can have same residential status in more than one country.

The Institute of Cost Accountants of India 19


Direct Taxation

1.2.2 Determination of Residential Status


Individual [Sec. 6(1)]
First of all, an individual is classified as resident or non-resident and again a resident individual may further be
categorized as Ordinarily Resident or Not Ordinarily Resident in India.

Resident in India
An individual is said to be a resident in India, if he satisfies any one of the following conditions -
(i) He is in India in the previous year for a period of 182 days or more [Sec. 6(1)(a)]; or
(ii) He is in India for a period of 60 days or more during the previous year and for 365 or more days during 4
previous years immediately preceding the relevant previous year [Sec. 6(1)(c)]
Taxpoint: Given Conditions are alternative in nature i.e. assessee needs to satisfy any one condition.

Sec. 6(1)(a): P.Y: Stays 182 days or more


Or
Sec/ 6(1)(c): P.Y: Stays 60 days or more
+ 4 PPY: Stays 365 days or more

Non-Resident in India
An assessee who is not satisfying sec. 6(1) shall be treated as a non-resident in India for the relevant previous year.

Illustration 3
Sam came to India first time during the P.Y. 2023-24. During the previous year, he stayed in India for (i) 50 days;
(ii) 183 days; & (iii) 153 days. Determine his residential status for the A.Y. 2024-25.
Solution:
(i) Since Sam resides in India only for 50 days during the P.Y. 2023-24, he does not satisfy any of the conditions
specified in sec. 6(1). He is, therefore, a non-resident in India for the P.Y. 2023-24.
(ii) Since Sam resides in India for 183 days during the previous year 2023-24, he satisfies one of the conditions
specified in sec. 6(1). He is, therefore, a resident in India for the P.Y. 2023-24.
(iii) Sam resides in India only for 153 days during the previous year 2023-24. Though he resided for more than 60
days during the previous year but in 4 years immediately preceding the previous year (as he came India first
time), he did not reside in India. Hence, he does not satisfy any of the conditions specified in sec. 6(1). Thus,
he is a non-resident for the P.Y. 2023-24.

20 The Institute of Cost Accountants of India


Basics of Income Tax Act

Illustration 4
Andy, a British national, comes to India for the first time during 2019-20. During the financial years 2019-20,
2020-21, 2021-22, 2022-23 and 2023-24, he was in India for 55 days, 60 days, 80 days, 160 days and 70 days
respectively. Determine his residential status for the assessment year 2023-24.
Solution:
During the previous year 2023-24, Andy was in India for 70 days & during 4 years immediately preceding the
previous year, he was in India for 355 days as shown below:

Year 2019-20 2020-21 2021-22 2022-23 Total


No. of days stayed in India 55 60 80 160 355
Thus, he does not satisfy Sec.6(1) & consequently, he is a non-resident in India for the P.Y. 2023-24.

Exceptions to the above rule


A. In the following cases, condition (ii) of sec. 6(1) [i.e. sec. 6(1)(c)] is irrelevant:
1. An Indian citizen, who leaves India during the previous year for employment purpose.
2. An Indian citizen, who leaves India during the previous year as a member of crew of an Indian ship.
Taxpoint: Above assessee shall be treated as resident in India only if he resides in India for 182 days or more
in the relevant previous year.
B. In case of an Indian citizen or a person of Indian origin# comes on a visit to India during the previous year,
modified condition (ii) of sec. 6(1) is applicable:

Case Modified condition (ii) of sec. 6(1)


His total income, other than the income from He is in India for a period of 120 days or more (but
foreign sources!, exceeds ` 15 lakhs during the less than 182 days) during the previous year and for
previous year 365 or more days during 4 previous years immediately
preceding the relevant previous year
His total income, other than the income from He is in India for a period of 182 days or more during
foreign sources, does not exceed `15 lakhs during the previous year In short, sec. 6(1)(c) is not applicable.
the previous year
# P
 erson of Indian origin: A person is deemed to be of Indian origin if he or either of his parents or grand
parents were born in undivided India. Here, grand parents may be paternal or maternal.
!
“ Income from foreign sources” means income which accrues or arises outside India (except income derived
from a business controlled in or a profession set up in India) and which is not deemed to accrue or arise in
India.
C. An individual shall be deemed to be resident in India, if following conditions are satisfied
a. He is a citizen of India
b. His total income, other than the income from foreign sources, exceeds ₹ 15 lakhs during the previous
year;
c. He is not satisfying any of the basic conditions given u/s 6(1) [i.e., 182 days or 60 days + 365 days]; and
d. He is not liable to tax in any other country or territory by reason of his domicile or residence or any other
criteria of similar nature. [Sec. 6(1A)]

The Institute of Cost Accountants of India 21


Direct Taxation

Taxpoint:
¾¾ However, if such individual has satisfied either of the basic conditions, then he shall be treated as resident
in India u/s 6(1).
¾¾ Further note that the exception is not applicable in case of foreign citizen even if he is a person of Indian
origin.
¾¾ If these conditions are satisfied, then such individual shall be deemed as resident irrespective of number
of days of his stay in India.

In case of an individual, being a citizen of India and a member of the crew of a ship, the period or periods of
stay in India shall, in respect of an eligible voyage, not include the period beginning on the date entered into the
Continuous Discharge Certificate in respect of joining the ship by the said individual for the eligible voyage and
ending on the date entered into the Continuous Discharge Certificate in respect of signing off by that individual
from the ship in respect of such voyage. In simple words, in the Continuous Discharge Certificate the date of
joining is recorded as 1st January 2020 and the date of ending the voyage is recorded as 31st January 2020, then
the entire period of 31 days shall be excluded from his stay in India
“Eligible voyage” shall mean a voyage undertaken by a ship engaged in the carriage of passengers or freight in
international traffic where-
(i) for the voyage having originated from any port in India, has as its destination any port outside India; and
(ii) for the voyage having originated from any port outside India, has as its destination any port in India.’.

Illustration 5
Miss Pal, an Indian citizen, left India for first time on 1st April, 2023 for joining job in Tokyo. She came to India on
11th Jan, 2024 for only 170 days. Determine her residential status for P.Y. 2023-24.

Solution:
Number of days Miss Pal stayed in India can be calculated as under:

P.Y. Apr May June July Aug Sep Oct Nov Dec Jan Feb Mar Total
23-24 1 - - - - - - - - 21 29 31 82
24-25 30 31 29 - - - - - - - - - 89

Since she left India for employment purpose, hence for becoming resident she has to stay in India for at least 182
days. However, she is in India for only 82 days during the previous year, thus she is a non-resident for the P.Y.
2023-24.
Points to be kept in mind
(a) Stay at same place in India is not necessary.
(b) Continuous stay in India is not necessary.
(c) A person shall be deemed to reside in India, if he is on the territorial waters of India5. For instance, if an
individual stays on a ship, which is in the territorial waters of India, then it shall be treated as his presence in
India.

5 Territorial water extends to 12 nautical miles (1 nautical miles = 1.1515 miles = 1.853 km) into the sea from the base line on the coast of India and
include any bay, gulf, harbour, creek or tidal river

22 The Institute of Cost Accountants of India


Basics of Income Tax Act

Additional conditions to test whether resident individual is ‘Ordinarily resident or not’ [Sec. 6(6)]
A resident individual in India can further be categorised as -

(i) Resident and ordinarily resident in India (ii) Resident but not ordinarily resident in India

Resident and ordinarily resident

If a resident individual satisfies the following two


additional conditions, he will be treated as resident
& ordinarily resident in India -
(a) He has been resident in India [as per sec.
6(1)] in at least 2 out of 10 previous years
immediately preceding the relevant previous
year; and
(b) He has resided in India for a period of 730 days
or more during 7 previous years immediately
preceding the relevant previous year.
Taxpoint: To be a Resident & Ordinarily resident
in India, one has to satisfy at least one condition of
sec. 6(1) & both the additional conditions of sec.
6(6).

Resident but not ordinarily resident


If a resident individual does not satisfy both additional conditions as given u/s 6(6), he is “Resident but not
ordinarily resident in India”.
Exceptions
A. An individual shall be deemed to be resident but not ordinarily resident in India, if following conditions
are satisfied:
(a) He is a citizen of India
(b) His total income, other than the income from foreign sources, exceeds ₹ 15 lakhs during the previous
year; and
(c) He is not liable to tax in any other country or territory by reason of his domicile or residence or any other
criteria of similar nature.
(d) He is deemed to be resident in India u/s 6(1A).
B. An individual shall be deemed to be resident but not ordinarily resident in India, if following conditions
are satisfied:
(a) He is an Indian citizen or a person of Indian origin.
(b) He comes on a visit to India during the previous year
(c) His total income, other than the income from foreign sources, exceeds ₹ 15 lakhs during the previous
year
(d) He is in India for a period of 120 days or more (but less than 182 days) during the previous year and for
365 or more days during 4 previous years immediately preceding the relevant previous year.

The Institute of Cost Accountants of India 23


Direct Taxation

Taxpoint: If aforesaid conditions are satisfied, then such individual shall be deemed to be resident but not
ordinarily resident even though he has satisfied both conditions specified u/s 6(6).
Provision Illustrated
Determine the residential status in the following different cases:

Case A B C D E F G H
Citizenship Foreign India India India Foreign Foreign India Foreign
Is he person of Indian origin Yes Yes Yes Yes Yes Yes Yes No
Total income (excluding income from Yes No Yes Yes Yes Yes No No
foreign source) exceeds ` 15,00,000
Liable to pay tax in other country No No No Yes No No No No
Stay in India during the previous year 30 30 30 30 138 185 85 85
Stay in India during 4 years immediately 380 380 380 380 380 180 380 380
preceding previous year
Are dual conditions given u/s 6(6) Yes Yes Yes Yes Yes Yes Yes Yes
satisfied
Residential Status NR NR NOR NR NOR ROR NR ROR
Note 1 2 3 4 5 6 7 8
1. He is not an Indian citizen, hence sec. 6(1A) is not applicable. Further his stay in India during the previous year
does not exceed 120 days.
2. His total income does not exceed ₹ 15,00,000.
3. All conditions of sec. 6(1A) are satisfied.
4. He is liable to pay tax in other country.
5. His stay in India exceeds 120 days (but does not exceed 182 days)
6. He has satisfied one condition of sec. 6(1) [i.e. 182 days criteria] and dual conditions of sec. 6(6)
7. He is not satisfying any of the condition provided in sec. 6(1)
8. He has satisfied one condition of sec. 6(1) [i.e. 182 days criteria] and dual conditions of sec. 6(6)

Illustration 6
Mr. X, aged 19 years, left India for first time on May 31, 2023. Determine his residential status for the previous
year 2023-24 if:
(i) He left India for employment purpose
(ii) He left India on world tour.
Solution:
During the previous year 2023-24, Mr. X was in India for 61 days as shown below –

P.Y. Apr May June July Aug Sep Oct Nov Dec Jan Feb Mar Total
23-24 30 31 - - - - - - - - - - 61

24 The Institute of Cost Accountants of India


Basics of Income Tax Act

During the previous year 2023-24, X stayed in India for 61 days. Further, he was in India for more than 365 days
during 4 years immediately preceding the relevant previous year (as he left India for first time).
(i) Since he left India for employment purpose, condition of sec. 6(1)(c) shall not be applicable on such assessee.
He will be treated as resident in India, if and only if, he resided in India for at least 182 days during the
previous year. Hence, Mr. X is a non-resident in India for the previous year 2023-24.
(ii) Since he left India on world tour, which is not an exception of sec. 6(1), satisfaction of any one condition of
sec. 6(1) makes him resident in India for the previous year 2023-24. As he satisfies 2nd condition of sec. 6(1)
[shown above], he is resident in India. Further, he also satisfies dual conditions specified u/s 6(6) (since he left
India for first time). Therefore, he is an ordinarily resident for the previous year 2023-24.

Illustration 7
X came India for first time on July 24, 2019. From July 24, 2019 to December 25, 2020 he was in India. Again,
he came to India on August 5, 2023 for employment purpose & left India on November 25, 2023 permanently.
Determine his residential status for the previous year 2023-24 assuming -

(a) He is a foreign citizen (b) He is an Indian citizen


Solution:
During the previous year 2023-24, X was in India for 113 days as shown below:

Year Apr May June July Aug Sep Oct Nov Dec Jan Feb Mar Total
23-24 - - - - 27 30 31 25 - - - - 113
Further, he was in India for more than 365 days during 4 years immediately preceding the previous year as shown
below:

Year Apr May June July Aug Sep Oct Nov Dec Jan Feb Mar Total
19-20 - - - 8 31 30 31 30 31 31 29 31 252
20-21 30 31 30 31 31 30 31 30 25 - - - 269
21-22 - - - - - - - - - - - - -
22-23 - - - - - - - - - - - - -
As he satisfies condition given in sec. 6(1)(c), he is a resident in India.
Further, he was resident during 2 out of 10 years immediately preceding the relevant previous year but he was in
India only for 521 days in 7 years immediately preceding the relevant previous year. As he is not satisfying dual
conditions of sec. 6(6), he is a resident but not ordinarily resident in India for the previous year 2023-24.
Note: His status shall remain same in both the cases as -
(a) Foreign citizens are not covered by ‘exceptions to sec. 6(1)(c)’.
(b) Coming in India for employment purpose is not covered by ‘exceptions to sec. 6(1)(c)’.

Illustration 8
X, a foreign citizen, resides in India during the previous year 2023-24 for 83 days. Determine his residential status
for previous year 2023-24 assuming his stay in India during the last few previous years are as follows -

The Institute of Cost Accountants of India 25


Direct Taxation

Year Days Year Days Year Days Year Days


2008-09 220 days 2012-13 36 days 2016-17 137 days 2020-21 175 days
2009-10 15 days 2013-14 115 days 2017-18 265 days 2021-22 15 days
2010-11 257 days 2014-15 123 days 2018-19 310 days 2022-23 67 days
2011-12 110 days 2015-16 65 days 2019-20 121 days

Solution:
During previous year 2023-24, X was in India for 83 days & during 4 years immediately preceding the previous
year, he was in India for 378 days as shown below:

Year 2019-20 2020-21 2021-22 2022-23 Total


No. of days stayed in India 121 175 15 67 378

Thus, he satisfies one of the conditions specified u/s 6(1) & consequently, he becomes resident in India in the P.Y.
2023-24. Further, to determine whether X is an ordinarily resident or not, he needs to satisfy both conditions laid
down u/s 6(6).

Year Presence in India Resident or Non Condition satisfied to become a


(In Days) resident resident
2022-2023 67 Resident 6(1)(c)
2022-2022 15 Non Resident None
2020-2021 175 Resident 6(1)(c)
2019-2020 121 Resident 6(1)(c)
2018-2019 310 Resident Both
2017-2018 265 Resident Both
2016-2017 137 Non Resident None
2015-2016 65 Resident 6(1)(c)
2014-2015 123 Resident 6(1)(c)
2013-2014 115 Resident 6(1)(c)

Condition (i) of sec. 6(6) requires that an individual should be resident in India for at least 2 out of 10 years
preceding the relevant previous year. X was resident in India for 8 out of 10 years immediately preceding the
previous year. Thus, he satisfies this condition.
Condition (ii) of sec. 6(6) requires that an individual should be present in India for at least 730 days during 7 years
preceding to relevant previous year. X was in India for 1090 days during 2016-17 to 2022-23. Hence, he satisfies
this condition also.
X satisfies condition (ii) of sec. 6(1) as well as both the conditions of sec. 6(6). Thus, he is a resident and ordinarily
resident in India for the previous year 2023-24.

26 The Institute of Cost Accountants of India


Basics of Income Tax Act

Hindu Undivided Family (HUF) [Sec. 6(2)]


An HUF can be either a resident or non-resident in India. Again, a resident HUF can further be classified as
‘Ordinarily resident’ and ‘Not ordinarily resident’.
Resident HUF: When the control & management1 of affairs of HUF is wholly or partly situated in India during the
relevant previous year, then it is treated as resident in India.
1
Control & management means -
 controlling & directive power;
 actual control & management (mere right to control & manage is not enough);
 central control & management and not the carrying out of day to day affairs.
The place of central control & management is situated where the head, the seat & the directing power is situated.
Non-resident HUF: An HUF is non-resident in India if the control & management1 of its affairs is wholly situated
outside India.
Ordinarily resident in India: If the ‘karta’ or manager of a resident HUF satisfies both additional conditions
given u/s 6(6), HUF is said to be an ordinarily resident. If the ‘karta’ or manager of a resident HUF do not satisfies
both additional conditions given u/s 6(6), HUF is said to be a not-ordinarily resident.

1.2.3 Incidence of Tax [Sec. 5]


The following chart highlights the provisions of tax incidence in brief:

Tax incidence in the case of


Resident
Nature of Income Resident &
but not
ordinarily Non resident
ordinarily
resident
resident
Income accrued or deemed to be accrued and received or deemed Taxable Taxable Taxable
to be received in India
Income accrued outside India but received or deemed to be
Taxable Taxable Taxable
received in India.
Income accrued or deemed to be accrued in India but received
Taxable Taxable Taxable
outside India
Income accrued and received outside India from a business
Taxable Taxable Not taxable
controlled in or profession set-up in India.
Income accrued and received outside India from a business
Taxable Not taxable Not taxable
controlled or profession set-up outside India.
Income accrued and received outside India in the previous year
Taxable Not taxable Not taxable
(it makes no difference if the same is later remitted to India).
Income accrued and received outside India in any year preceding
the previous year and later on remitted to India in current Not taxable Not taxable Not taxable
financial year.
Note: In case of resident assessee like company, firm etc. (other than Individual and HUF) in which there is
no classification as ‘Resident but not ordinarily resident’, income accrued and received outside India from a
business controlled or profession setup outside India shall be taxable.

The Institute of Cost Accountants of India 27


Direct Taxation

Illustration 9
Ram provides following details of income, calculate the income which is liable to be taxed in India for the
A.Y.2024-25 assuming that –

(a) He is an ordinarily resident (b) He is not an ordinarily resident (c) He is a non-resident.


Particulars Amount
Salary received in India from a former employer of UK 1,40,000
Income from tea business in Nepal being controlled from India 10,000
Interest on company deposit in Canada (1/3rd received in India) 30,000
Profit from a business in Mumbai controlled from UK 1,00,000
Profit for the year 2012-13 from a business in Tokyo remitted to India 2,00,000
Income from a property in India but received in USA 45,000
Income from a property in London but received in Delhi 1,50,000
Income from a property in London but received in Canada 2,50,000
Income from a business in Jambia but controlled from Turkey 10,000
Solution:
Calculation of income liable to be taxed in India of Ram for the A.Y.2024-25
Resident & Resident but
Non-
Particulars Ordinarily not ordinarily
resident
resident resident
Salary received in India from a former employer of UK 1,40,000 1,40,000 1,40,000
Income from tea business in Nepal being controlled from India 10,000 10,000 Nil
Interest on company deposit in Canada -
- 1/3rd received in India 10,000 10,000 10,000
- 2/3rd received outside India 20,000 Nil Nil
Profit from a business in Mumbai controlled from UK 1,00,000 1,00,000 1,00,000
Past Profit from a business in Tokyo remitted to India Nil Nil Nil
Income from a property in India but received in USA 45,000 45,000 45,000
Income from a property in London but received in Delhi 1,50,000 1,50,000 1,50,000
Income from a property in London but received in Canada 2,50,000 Nil Nil
Income from a business in Jambia but controlled from Turkey 10,000 Nil Nil
Income liable to tax in India 7,35,000 4,55,000 4,45,000

Test Yourself
1. Mr. Rupankar Roy, an Indian Citizen, left India for the purpose of employment in USA for the first time
on 1st October, 2023. He came back to India on 30th March, 2024 for visit and returned back to USA after
staying 20 days in India. During the previous year 2023–24, he earned the following Income:
(i) Interest earned in USA ₹ 5,00,000 and credited in USA.
(ii) Interest received in India out of Fixed Deposit in Bank ₹ 1,20,000.
Determine his residential status and Tax Incidence in India for the A.Y. 2024-25

28 The Institute of Cost Accountants of India


Basics of Income Tax Act

2. Mr. Ajnabi provides following information regarding his income of P.Y. 2023-24. Compute income liable to
be charged in India in the following cases:
a) He is an ordinarily resident b) He is not an ordinarily resident. c) He is a non-resident
Particulars ₹
Business income from USSR received in India 10,000
Business income earned in India received in Pakistan 20,000
Salary income from a company of UK situated in India 15,000
Interest on German Development Bond (2/5th received in India) 60,000
Income from agriculture in Nepal received there but later on remitted to India 1,81,000
Income from property in Jakarta received outside India 86,000
Income earned from business in UAE which is controlled from Delhi (₹ 15,000 received in 65,000
India)
Past untaxed profit of 15-16 brought to India during previous year 10,43,000
Profit from a business in Madras and managed from outside India 27,000
Profit on a sale of a building in India but received in Sri Lanka 14,80,000
Pension from a former employer in India received in USSR 36,000
Hints
1. Resident and ordinarily resident; ₹ 6,20,000/- 2. ₹ 19,80,000; ₹ 16,77,000; ₹ 16,27,000

1.2.4 Income received in India


Income received in India is taxable in all cases (whether accrued in India or elsewhere) irrespective of residential
status of the assessee, therefore it is significant to know the meaning of income received in India. If the place,
where the recipient gets the money (on first occasion) under his control, is in India, it is said to be income received
in India.
Taxpoint: Receipt is different from remittance. The receipt of income refers to the first occasion when the recipient
gets the money under his control. Once the amount is received as income (at any place outside India), any subsequent
remittance or transmission of the amount to India does not result to receipt in India
Example 5: Mr. X, a non-resident, received dividend from an Italian company in Japan on 15/12/2022. On
17/12/2022, he remitted such income in India. Such income shall not be taxable in India as income has neither
received in India nor accrued in India.

Salary accrued to a non-resident seafarer for services rendered outside India on a foreign going ship (with Indian
flag or foreign flag) shall not be included in the total income merely because the said salary has been credited in
the NRE account maintained with an Indian bank by the seafarer.

1.2.5 Income deemed to be received in India


Following incomes shall be deemed to be received in India and taxable in hands of all assessee irrespective of their
residential status -
a) The annual accretion in the previous year to the balance at the credit of an employee participating in a
recognized provident fund, to the extent provided in Rule 6 of part A of the IV schedule i.e.-
i) Employer’s contribution to the recognised provident fund in excess of 12% of salary.

The Institute of Cost Accountants of India 29


Direct Taxation

ii) Interest credited on the above balance by a rate exceeding 9.5% [Sec. 7(i)]
b) The transferred balance in recognised provident fund, to the extent liable to income tax [Sec. 7(ii)]
c) The contribution made, by the employer in the previous year, to the account of an employee under a pension
scheme notified u/s 80CCD [Sec. 7(iii)]
d) Tax Deducted at source [Sec. 198]
e) Deemed profit.
f) Income from undisclosed sources

1.2.6 Income deemed to accrue or arise in India [Sec. 9]


Following incomes are deemed to accrue or arise in India:

Income Salary Salary from Income Income from Income Income Deemed
from earned Govt. by an from interest from from receipt
connection in India Indian citizen dividend payable by royalty technical of gift
in India for services paid by specified services by non-
rendered outside an Indian person resident
India company
Sec. Sec. 9(1) Sec. 9(1)(iii) Sec. 9(1) Sec. Sec. 9(1) Sec. 9(1) Sec. 9(1)
9(1)(i) (ii) (iv) 9(1)(v) (vi) (vii) (viii)

Quick MCQs:

1. A Person may be Resident of


(a) Only one country always
(b) More than one country for any previous year.
(c) Only one country for any previous year.
(d) No specific rule

2. Residential Status is to be determined for –


(a) Previous Year
(b) Assessment Year
(c) Financial Year
(d) Accounting Year

3. Incomes which accrue or arise outside India but are received directly into India are taxable in case of –
(a) Resident only
(b) Both Ordinarily Resident and NOR
(c) Non-Resident
(d) All Assesses

30 The Institute of Cost Accountants of India


Basics of Income Tax Act

4. Total Income of a person is determined on the basis of his –


(a) Residential Status in India
(b) Citizenship in India
(c) Residential Status and Citizenship in India
(d) None of the above

5. R was born in England, his parents were born in India in 1952. His grand parents were born in South
Africa. RB shall be a-
(a) Person of India Origin
(b) Foreign National
(c) Artificial Person
(d) Citizen of India

6. R a person of Indian Origin visited India on 03.10.2023 and plans to stay here for 185 days. During 4
years prior to previous year 2024-25, he was in India for 750 days. Earlier to that he was never in India.
For A.Y. 2024-25, RE shall be-
(a) Resident and Ordinarily Resident in India
(b) Resident but not Ordinarily Resident in India
(c) Non-Resident
(d) Deemed Resident

7. Asha leave India for the first time on 24-12-2023, Determine her Residential status for the AY 2024-2025.
(a) Resident
(d) Resident and Ordinary Resident
(c) Resident and Not Ordinary Resident
(d) Non-Resident

8. Foreign Income earned by Seema will be taxable in India if she is a


(a) Non-Resident
(b) Resident and Ordinary Resident
(c) Resident and Not Ordinary Resident
(d) All of the above

9. Interest paid by a Resident for any other purpose other than carrying on business or profession, outside
India is deemed to accrue or arise in India, if the receiver is a
(a) Non-Resident
(b) Resident and Ordinary Resident
(c) Resident and Not Ordinary Resident
(d) All of the above

The Institute of Cost Accountants of India 31


Direct Taxation

10. Determine residential status of Chidambaram which carries out its transactions in Malaysia. Its affairs are
partly controlled from India. The Karta of HUF, Mr. Chidambaram who is from Chennai visits India on
01-06-2022 and leaves to Malaysia on 10-02-2023. He has not visited India for the past 11Years.
(a) Non-resident
(b) Resident but not ordinarily resident
(c) Deemed resident
(d) Resident and ordinarily resident

11. Share of Profit of Mr. Vivek who is Partner in M/S.VIVA & Co. is-
(a) Exempt from tax
(b) Taxable as his Business Income
(c) Taxable as his Salary
(d) Taxable as Income from Other Sources

12. Casual Income received by the Assessee is –


(a) Fully Exempt
(b) Exempt upto ` 5,000
(c) Fully Taxable
(d) None of the above.

13. The Circulars issued by CBDT are binding on –


(a) Assessee
(b) Income Tax Authorities
(c) Both of the above
(d) Assessee and Court

14. Chennai Corporation falls under the category –


(a) Artificial Judicial Person
(b) Local Authority
(c) Association of Persons
(d) None of the above

15. Surcharge on Income Tax is payable by –


(a) Foreign Company
(b) Individual and HUF
(c) A Domestic Company
(d) All of the above

32 The Institute of Cost Accountants of India


Basics of Income Tax Act

16. The maximum amount on which Income – Tax is not chargeable in case of HUF for AY 2024-25 is –
(a) ` 2,50,000
(b) ` 5,00,000
(c) ` 3,00,000
(d) ` 2,00,000

17. Health and Education Cess is leviable on –


(a) Income Tax
(b) Income Tax + Surcharge
(c) Surcharge
(d) None of the above

18. The total income of the assesse has been computed as ` 2,53,494.90. For rounding off, the Total Income
will be taken as –
(a) ` 2,53,500
(b) ` 2,53,490
(c) ` 2,53,495
(d) ` 2,53,400

19. Undisclosed Income u/s. 68 to 69D are charged to tax at the effective rate of –
(a) 60% only
(b) 78% (Inclusive of surcharge @ 25% & HEC @ 4%)
(c) 75% (inclusive of surcharge @ 25%)
(d) 100%

The Institute of Cost Accountants of India 33


Direct Taxation

Agricultural Income 1.3

A
griculture income is exempt under the Indian Income Tax Act. The reason for exemption of agriculture
income from Central Taxation is that the Constitution gives exclusive power to make laws with respect to
taxes on agricultural income to the State Legislature.

1.3.1 Meaning
By virtue of sec. 2(1A), agricultural income means -

1. Rent or Revenue: Any rent or revenue derived


from a land, which is situated in India & is used
for agricultural purposes$;
Taxpoint:
¾¾ Rent may be in cash or in kind.
¾¾ Assessee may be the owner or tenant of
such land.

2. Cultivation of Land: Any income derived from such land by agriculture $


3. Income from Process: Any income derived from such land by the performance by –
(a) a cultivator;
(b) receiver of rent in kind;
- of any process ordinarily employed by a them to render the produce raised or received by him fit to be taken
to market.
4. Income from Sale of Produce: Any income derived from such land by the sale by
(a) a cultivator of the produce raised by him; or
(b) receiver of rent-in-kind of the produce received by him;
- in respect of which no process has been performed other than a process required to render it fit for the market.
Taxpoint: The process must be employed only to convert ‘the produce or rent in kind’ in marketable form.
If marketing process is performed on the ‘produce or rent in kind’, which can be sold in its raw form in
market, then income derived from such product is partly agricultural & partly non-agricultural income. (Detail
discussion is given later in this chapter)
5. Income from Let Out of Agricultural House Property: Any income derived from a building subject to
fulfillment of the following conditions -
(a) The building should be occupied by the cultivator or receiver of rent in kind.
(b) The building should be on or in the immediate vicinity of the land, being situated in India and used for
agricultural purposes.

34 The Institute of Cost Accountants of India


Basics of Income Tax Act

(c) The building should be used as dwelling house or store-house or other out building.
(d) The land is either situated in –
(i) Rural area; or
(ii) Urban area6 and assessed to land revenue / local rates.
Taxpoint:
¾¾ Where such land or building is used for non-agricultural purpose then any income derived from such land
or building shall not be treated as agricultural income.
¾¾ Income derived from land being let out for storing crop shall not be agricultural income.
¾¾ Building should be owned and occupied by the land-holder if he receives rent or revenue from the land.
On the other hand, in case of cultivator or receiver of rent in kind, it is enough that the building is
occupied by him.

(a) Profit on transfer of agricultural land: Profit on transfer of agricultural land shall not be treated as
agricultural income.
(b) Nexus between agro-activity and agro-income: There must be a close nexus between agro-activity
and agro-income. Income by way of sale of commodity, being different from what is raised and
processed, is not agricultural income. E.g. Assessee growing mulberry leaves to feed silkworms
and to obtain silk-cocoons, income on sale of such silk-cocoons shall not be treated as agricultural
income.
$
Agriculture or Agricultural operations or Agricultural purposes: The Act nowhere defines the term agricultural
operations or agricultural purposes. However, the Supreme Court laid down guidelines for the determination of the
scope of these terms in CIT -vs.- Raja Benoy Kumar Sahas Roy. Accordingly, for the purpose, agricultural activity
is divided into two parts:

(a) Basic Operation: It means application of human skill & labour


upon the land, prior to germination.
E.g. Tilling of land, sowing of seeds, planting, irrigation, etc.
Taxpoint: Any spontaneous growth from land itself (i.e. without
any human effort) cannot be termed as agricultural operation.
(b) Subsequent Operation: It means operations -

¾¾ which fosters the growth and preserves the produce;


6 Following is considered as urban area:
a. land which is situated within the jurisdiction of any Municipality (whether known as a municipality, municipal corporation, notified area
committee, town area committee, town committee, or by any other name) or Cantonment Board having population of 10,000 or more; or
b. in any area within the distance, measured aerially,—
Population of the municipality or cantonment board Area within the aerial distance from the local limits of such
municipality or cantonment board is treated as non-rural area
More than 10,000 but not exceeding 1,00,000 Upto 2 kilometres
More than 1,00,000 but not exceeding 10,00,000 Upto 6 kilometres
More than 10,00,000 Upto 8 kilometres

Population, according to the last preceding census of which the relevant figures have been published before the first day of the previous
year, shall be considered.

The Institute of Cost Accountants of India 35


Direct Taxation

¾¾ for rendering the produce fit for sale in market; and


¾¾ which are performed after the produce sprouts from the land.
E.g. Digging the soil around the growths, removal of undesirable undergrowths, weeding, tending, pruning,
cutting, harvesting, etc.
Taxpoint:

Activity Whether treated as agricultural activity?


Mere Basic Operation Agricultural activity
Mere Subsequent Operation Not an agricultural activity
Subsequent operation together with basic operation Agricultural activity

Agricultural Income, at a glance

Land must be in India

1.3.2 Instances of Agricultural (Agro) Income


1. Income from growing trade or commercial products like jute, cotton, etc. is an agro income.
2. Income from growing flowers and creepers is an agro income.
3. Plants sold in pots are an agro income provided basic operations are performed.
4. Remuneration and interest to partner: Any remuneration (salary, commission, etc.) received by a partner
from a firm engaged in agricultural operation is an agro income.
Interest on capital received by a partner from a firm, engaged in agricultural operation is an agro income.
5. Income arising by sale of trees grown on denuded parts of the forest after replanting and by carrying on
subsequent operations, is an agro income.
6. Compensation received from insurance company for damage caused by hail-storm to the green leaf of the
assessee’s tea garden is agricultural income. Further, no part of such compensation consists of manufacturing
income, as such compensation cannot be apportioned under Rule 8 between manufacturing income and
agricultural income.

36 The Institute of Cost Accountants of India


Basics of Income Tax Act

7. Any fee derived from land used for grazing of cattle, being used for agricultural operation, is an agro income.
8. Any income derived from saplings or seedlings grown in a nursery shall be deemed to be agricultural income

1.3.3 Instances of Non-agricultural (Non-agro) Income


1. Salary received by an employee from any business (having agricultural income) is non-agro income.
2. Dividend received from a company engaged in agricultural operation is non-agro income.
3. Income from salt produced by flooding the land with sea-water is non-agro income.
4. Income from fisheries, poultry farming, dairy farming, butter & cheese making, etc. is non-agro income.
5. Breeding & rearing of livestock is non-agro income.
6. Interest received by a moneylender in the form of agricultural produce is non-agro income.
7. Profit on sale of standing crops after harvest, where such crops were acquired through purchase is non-agro
income.
8. Royalty income from mines is non-agro income.
9. Remuneration to a Director or Managing Director from a company engaged in agricultural business is non-
agro income. The provision holds good even when such remuneration is on the basis of certain percentage of
net profit.
10. Income earned by a cultivator from conversion of sugarcane (raised on own land) to jaggery is non-agro
income to the extent to which income is related to such conversion only. This is because sugarcane itself is
marketable.
11. Interest on arrears of rent receivable in respect of agricultural land is non-agro income.
12. Income from a land situated outside India is non-agro income
13. Annuity received by a person in consideration of transfer of agricultural land, is non-agro income.
14. Income on supply of water for agricultural operation is non-agro income. The provision holds good even when
such income is received in the form of agro-produce.
15. Income from sale of trees and grasses grown spontaneously (without any human effort), is non-agro income.

1.3.4 Treatment of Partly Agricultural & Partly Non-Agricultural Income


In case assessee is engaged in an integrated activity, comprising of agricultural activity as well as non- agricultural
activity, then profit of such integrated activity shall be segregated into agricultural income and non-agricultural
income in the following manner –

Agricultural Non-Agricultural
Rule Case
Income Income
8 Assessee is engaged in the business of growing and 60% of income 40% of income
manufacturing tea in India
E.g., If an assessee earns ₹ 5 lakh (as per sec. 28) from the business of growing & manufacturing tea
in India, then his business income will be ₹ 2 lakh (i.e., 40% of ₹ 5 lakh) & agro income will be ` 3
lakh (i.e. 60% of ₹ 5 lakh)

The Institute of Cost Accountants of India 37


Direct Taxation

Agricultural Non-Agricultural
Rule Case
Income Income
7A Assessee is engaged in the business of growing and 65% of income 35% of income
manufacturing rubber in India
Assessee is engaged in the business of growing and manufacturing Coffee in India
7B(1) ¾¾ Coffee grown and cured by the seller in India 75% of income 25% of income
7B(1A) ¾¾ Coffee grown, cured, roasted and grounded by the 60% of income 40% of income
seller in India, with or without mixing chicory or
other flavouring ingredients
Salary and interest received by a partner from a firm growing and manufacturing tea, coffee or rubber:
Such remuneration or interest shall be treated as partly agricultural income and partly business income as stated
above.
Any other case
For computing agricultural income from a business having both agricultural as well as non-agricultural income,
1. Assessee is required to prepare two Profit or Loss statements, one for agro-business & another for non agro-
business
2. Agro expenses debited to Agro Profit or Loss and non agro expenses shall be debited to Non agro-business
Profit or Loss
Note: Non-apportionable expenditure, related to composite business of agriculture and non-agriculture, is
fully charged to non-agricultural business.
3. Market value of any agricultural produce, which is utilised as raw material in such business, is to be treated as
income for agro-business and expenditure for non agro-business.
Illustration 10
X Ltd. grows sugarcane to manufacture sugar. Details for the previous year 2023-24 are as follows:

Particulars ₹ in lacs.
Cost of cultivation of sugarcane (5,000 tons) 10
Sugarcane sold in market (1,000 tons) 3
Sugarcane used for sugar manufacturing (4,000 tons) -
Cost of conversion 5
Sugar produced & sold in market 25
Compute income of X Ltd.
Solution:
Computation of income of X Ltd. for the A.Y. 2024-25 ₹ in lacs

Particulars Manufacturing Agriculture


Sale of agro product in market - 3
Sale of manufactured product in market 25 -

38 The Institute of Cost Accountants of India


Basics of Income Tax Act

Particulars Manufacturing Agriculture


Notional sale of agro product used in the process of manufacturing - 12
(4,000 ton × ₹ 3 lacs per ‘000 ton)
Revenue [A] 25 15
Less: Expenses incurred
Cost of conversion 5 -
Market value of sugarcane used (4,000 ton × ₹ 3 lacs per ‘000 ton) 12 -
Cost of cultivation - 10
Expenditure [B] 17 10
Income [A – B] 8 5

1.3.5 Impact of agricultural income on tax computation


Sec. 10(1) of the Act exempts agricultural income from tax as our Constitution does not provide power to the
Parliament to levy tax on agro-income. However, since 1973 an indirect method7 has been found, to levy tax on
agro-income. According to this method, agricultural income is included in the total income of the assessee for
deciding the tax slab of the assessee.
The way to apply higher rate of tax-slab on non-agricultural income by including agricultural income in the total
income of the assessee are as under:
Conditions for including agricultural income in the total income of the assessee
1. The assessee is an individual, a Hindu-undivided family, a body of individual, an association of person or an
artificial juridical person.
2. The assessee has non-agricultural income exceeding the maximum amount of exemption (i.e. in case of Senior
citizen ₹ 3,00,000, Super Senior citizen ₹ 5,00,000 and in case of other individual/ HUF/AOP / BOI /artificial
juridical person ₹ 2,50,000)
3. The agricultural income of the assessee exceeds ₹ 5,000.

Treatment
Step 1: Compute income tax on total income of assessee including Agro-income.
Step 2: Compute income tax on (Agro-income + Maximum exempted limit)
Step 3: Tax liability before cess = (Tax as per step 1) - (Tax as per step 2)

Illustration 11
Mr. X aged 42 years has non-agro income of ₹ 3,25,000 and agro income of ₹ 2,55,000. Compute his tax liability
for the A.Y. 2024-25.

7 On the recommendation of the Committee on Taxation of Agricultural Wealth and Income headed by Dr. K. N. Raj

The Institute of Cost Accountants of India 39


Direct Taxation

Solution:
Computation of tax liability of Mr. X for the A.Y. 2024-25

Particulars ₹
Income Tax on ₹ 5,80,000 (i.e. agro income ₹ 2,55,000 + non agro ₹ 3,25,000) 28,500
Less: Tax on ₹ 5,05,000 (i.e. agro income ₹ 2,55,000 + maximum exempted limit ₹ 2,50,000) 13,500
Tax liability 15,000
Less: Rebate u/s 87A 12,500
2,500
Add: Health & Education Cess (4% of ₹ 2,500) 100
Tax and cess payable (Rounded off u/s 288B) 2,600

Test Yourself
1. Mr. X aged 62 years has non-agro income of ₹ 3,25,000 and agro income of ₹ 2,55,000. Compute his tax
liability for the A.Y. 2024-25.
2. Mr. Y aged 62 years has non-agro income of ₹ 3,75,000 and agro income of ₹ 2,55,000. Compute his tax
liability for the A.Y. 2024-25.
3. Mr. Z aged 82 years has non-agro income of ₹ 6,75,000 and agro income of ₹ 2,55,000. Compute his tax
liability for the A.Y. 2024-25.
4. Mr. A aged 32 years has non-agro income of ₹ 2,25,000 and agro income of ₹ 5,55,000. Compute his tax
liability for the A.Y. 2024-25.
Hints
1. Nil; 2. ₹ 2,600; 3. ₹ 36,400; 4. Nil
Illustration 12
Mr. Tony had estates in Rubber, Tea and Coffee. He derives income from them. He has also a nursery wherein he
grows plants and sells. For the previous year ending 31.3.2024, he furnishes the following particulars of his sources
of income from estates and sale of plants. Compute taxable income:
(a) Manufacture of Rubber ₹ 5,00,000.
(b) Manufacture of Coffee grown and cured ₹ 3,50,000.
(c) Manufacture of Tea ₹ 7,00,000.
(d) Sale of plants from nursery ₹ 1,00,000.
Solution:
Computation of income of Mr. Tony for the A.Y. 2024-25

Particulars Agricultural income Business income


Income from growing and manufacturing rubber [Rule 7A]
65% of ₹ 5,00,000 3,25,000
35% of ₹ 5,00,000 1,75,000

40 The Institute of Cost Accountants of India


Basics of Income Tax Act

Particulars Agricultural income Business income


Income from growing and curing coffee [Rule 7B(1)]
#

75% of ₹ 3,50,000 2,62,500


25% of ₹ 3,50,000 87,500
Income from growing and manufacturing tea [Rule 8]
60% of ₹ 7,00,000 4,20,000
40% of ₹ 7,00,000 2,80,000
Sale of plants from nursery 1,00,000
Total 11,07,500 5,42,500
#
Assume the coffee grown and cured is not roasted and grounded by the seller.

Test Yourself
1. Mr. Krishna Daripa, engaged in growing and manufacturing of tea, furnished the following information for
the previous year 2022-23
Sale of Tea ₹ 15,00,000
Growing and manufacturing expenses of tea ₹5,00,000
You are required to compute the taxable income of Ms. Krishna Daripa for the A. Y. 2024-25.
Hints
1. ₹ 4,00,000

Illustration 13
State the tax treatment of the following income -
(a) A is employed in an agricultural farm and entrusted with tilling of land, his remuneration being 50% of the net
profits earned by the farm.
(b) C receives a dividend of ₹ 12,000 from a company whose entire income is derived from agricultural operations
only.
(c) D of Kolkata earns an income of ₹ 12,000 from agricultural land owned by him and situated in Bangladesh.
Such income is received in Bangladesh.
(d) F receives ₹ 600 on account of interest on loan on the mortgage of land which is used for agricultural purposes.
(e) G earns an income of ₹ 1,200 from lease of land for grazing of cattle required for agricultural operations.
(f) H receives ₹ 400 on account of interest on arrears of rent in respect of land used by tenant for agricultural
operations.
(g) Income from the sale of replanted trees where the denuded parts of the forest are replanted and subsequent
operation in forestry are carried out.
(h) Income from sale of trees of forest which are of spontaneous growth and in relation to which forestry operations
alone are performed or Income from sale of wild grass of spontaneous growth

The Institute of Cost Accountants of India 41


Direct Taxation

(i) Income from sale of tea leaf from a tea garden.


(j) Income from sale of jute produced in land situated in Bangladesh.
(k) Income from poultry farming
(l) Income from growing flowers in gardens
(m) Income from sale of tobacco leaves after being dried to make it fit for sale.
(n) Income from fisheries or poultry or dairy
(o) Income of ₹ 50,000 from agricultural land, the land is situated in Bangladesh
(p) Income of ₹ 25,000 from the land used as stone quarries.

Solution:
(a) Since Mr. A is an employee of the concern, therefore his income shall be taxable under the head ‘Salaries’ and
shall not be treated as agricultural income. However, if Mr. A is a partner of the concern then such income shall
be treated as agricultural income.
(b) Dividend received from a company (engaged in agricultural business) cannot be treated as agricultural income.
Such dividend shall be taxable under the head “Income from other sources”.
(c) Any income from a land situated outside India is not an agro-income and taxable under the head “Income from
other sources”. It is to be noted that such income shall be taxable only if the assessee is an ordinarily resident
in India.
(d) Interest on loan on the mortgage of land used for agricultural purpose is not an agro-income.
(e) Any rent derived from land used for grazing of cattle, used for agricultural operation, is an agro-income.
(f) Interest on arrears of rent receivable in respect of agricultural land is non-agricultural income.
(g) Assume replantation of trees has been done with application of basic operation on land. Hence such income is
agro-income.
(h) Income from sale of trees, grass grown spontaneously and without any human effort is non-agricultural income.
(i) It will be treated as agricultural income.
(j) Income from sale of jute produced in land situated in Bangladesh is not treated as agricultural income. For the
purpose of this, land should be situated in India.
(k) Income from Poultry farming is not an agricultural income because such income is not derived from land.
(l) Income from growing flowers in garden is as an agricultural Income as the same is derived from a land by
performing agricultural operations on it.
(m) Income from sale of tobacco leaves after being dried to make it fit for sale is an agricultural income.
(n) Income from fisheries or poultry or dairy is not considered as agricultural income as the same is not derived
from land.
(o) Since the land in situated outside India, hence income is not considered as agricultural income.
(p) It is not an agricultural income as no agricultural operation has been carried on the land.

42 The Institute of Cost Accountants of India


Basics of Income Tax Act

Quick MCQs:

1. CG arising from compulsory acquisition of urban agricultural land is-


(a) Taxable
(b) Exempt
(c) Exempt if acquired by RBI or Central Government
(d) None of the above

2. An Assessee is engaged in the business of growing and manufacturing of rubber, the agricultural income
in that case shall be-
(a) 40% of the income from such business
(b) 60% of the income from such business
(c) 65% of the income for such business
(d) None of above

3. If a Firm earns Agricultural Income, it will be exempt –


(a) In the hand of Firm
(b) In the hands of Firm but taxable in Partner’s hands
(c) In the hands of Firm as well as Partners
(d) In the hands of the firm as well as its Partner but would be included for computation of tax on
partners’ incomes.

4. If a Company declares divided out of Agricultural Income, such dividend declared by the Company shall
be-
(a) Exempt in the hands of the Shareholder
(b) Not be subject to any Income Tax, either in the hand of Company or the Share holders
(c) Included in the Total Income of the Company
(d) Taxable in the hands of the Shareholders.

The Institute of Cost Accountants of India 43


Direct Taxation

Income which do not form part of Total 1.4


Income
1.4.1 Income Exempt from Tax
Sec. 10 enlists the various income which are exempt from tax i.e. does not form part of total income of the assessee.
These are –
Agricultural Income [Sec. 10(1)]
Refer chapter Agricultural income
Member’s Share in Income of HUF [Sec. 10(2)]
Any sum received by an individual as a member of a Hindu undivided family –
–– Where such sum has been received out of the income of the family; or
–– Where such sum has been received out of the income of an impartible estate belonging to the family.
Share of Profit from a Firm [Sec. 10(2A)]
Share in the total income of the firm is exempt in the hands of partner.
Interest Income of Non-resident [Sec. 10(4)/(4B)]
 Interest on specified securities or bonds, including premium on redemption of such bonds is exempted in the
hands of a non-resident [Sec. 10(4)(i)]
 Interest on Non-Resident (External) Account in any bank in India to a person who is a resident outside India
as per as defined in sec. 2(w) of the Foreign Exchange Management Act, 1999 or is a person who has been
permitted by the Reserve Bank of India to maintain the aforesaid Account
 Interest on notified savings certificates issued before 1-6-2002 by the Central Government to a non-resident,
being a citizen of India or a person of Indian origin [Sec. 10(4B)]
Interest on Rupee Denominated Bond [Sec. 10(4C)]
Interest payable to a non-resident, not being a company, or to a foreign company, is exempt if following conditions
are satisfied:
(a) Interest is payable by any Indian company or business trust.
(b) Such interest is payable in respect of monies borrowed from a source outside India by way of issue of rupee
denominated bond, as referred to in sec. 194LC(2)(ia).
(c) Such bond has been issued during 17-09-2018 and 31-03-2019.

44 The Institute of Cost Accountants of India


Basics of Income Tax Act

Income received by specified fund [Sec. 10(4D)]Amended


any income accrued or arisen to, or received by a specified fund as a result of transfer of capital asset referred to in
sec. 47(viiab), on a recognised stock exchange located in any International Financial Services Centre; and
Where the consideration for such transaction is paid or payable in convertible foreign exchange or as a result of
transfer of securities (other than shares in a company resident in India) or any income from securities issued by a
non-resident (not being a permanent establishment of a non-resident in India); and
Where such income otherwise does not accrue or arise in India or any income from a securitisation trust which is
chargeable under the head “Profits and gains of business or profession”,
–– to the extent such income accrued or arisen to, or is received, is attributable to units held by non-resident (not
being the permanent establishment of a non-resident in India) or is attributable to the investment division of
offshore banking unit, as the case may be, computed in the prescribed manner.
Income of IFSC [Sec. 10(4E)/(4F)]Amended
 Any income accrued or arisen to, or received by a non-resident as a result of transfer of non-deliverable
forward contracts or offshore derivative instruments or over-the-counter derivatives entered into with an
offshore banking unit of an International Financial Services Centre as referred to in sec. 80LA(1A), which
fulfils such conditions as may be prescribed;
 Any income of a non-resident by way of royalty or interest, on account of lease of an aircraft or a ship in
a previous year, paid by a unit of an International Financial Services Centre, if the unit has commenced its
operations on or before 31-03-2024.
¾¾ “Aircraft” means an aircraft or a helicopter, or an engine of an aircraft or a helicopter, or any part thereof;
¾¾ “Ship” means a ship or an ocean vessel, engine of a ship or ocean vessel, or any part thereof.
Income of IFSC [Sec. 10(4G)]New
Any income received by a non-resident from portfolio of securities or financial products or funds, managed or
administered by any portfolio manager on behalf of such non-resident, in an account maintained with an Offshore
Banking Unit in any International Financial Services Centre, as referred to in sec. 80LA(1A), to the extent such
income accrues or arises outside India and is not deemed to accrue or arise in India.
Capital gain from transfer of equity shares [Sec. 10(4H)]
Income in respect of capital gain from transfer of equity shares of a domestic company engaged in aircaraft leasing
business in IFSC is exempt.
Leave Travel Concession [Sec. 10(5)]
Refer chapter Salaries.
Remuneration to Person who is not a Citizen of India in certain cases [Sec. 10(6)]
Following remuneration to an individual who is not a citizen of India shall be exempt –
 Remuneration received by him as an official of an embassy, high commission, legation, commission, consulate,
or the trade representation of a foreign state or as a staff of any of these officials provided corresponding Indian
officials in that foreign country enjoy similar exemptions in their country - Sec. 10(6)(ii).
 Remuneration received as an employee of a foreign enterprise for services rendered by him during his stay in
India provided -

The Institute of Cost Accountants of India 45


Direct Taxation

(a) the foreign enterprise is not engaged in any business or profession in India;
(b) his stay in India does not exceed 90 days in aggregate; and
(c) such remuneration is not liable to be deducted from the income of the employer under this Act - Sec.
10(6)(vi)
 Remuneration for services rendered in connection with his employment on a foreign ship provided his total
stay in India does not exceed 90 days in the previous year - Sec. 10(6)(viii)
 Remuneration received as an employee of the Government of a foreign State during his stay in India in
connection with his training in any undertaking owned by Government, Government company, subsidiary of a
Government company, corporation established by any Central, State or Provincial Act and any society wholly
financed by the Central or State Government – Sec. 10(6)(xi)
Tax paid by Government on Royalty or Fees for Technical Service [Sec. 10(6A)]
Tax paid by Government on Income of a Non-resident or a Foreign Company [Sec. 10(6B)]
Tax paid on Income from Leasing of Aircraft [Sec. 10(6BB)]
Tax paid by an Indian company on income arising from leasing of aircraft, etc. to the Government of a foreign
state or foreign enterprise under an approved agreement entered into with such Indian company engaged in the
business of operation of aircraft, provided such agreement was entered into between 1-4-1997 and 31-3-1999 or
after 31-3-2007.
Taxpoint: Only tax paid on such income is exempt, however such income is taxable.
Fees for Technical Services in Project connected with Security of India [Sec. 10(6C)]
Any income arising to notified foreign company by way of royalty or fees for technical services received in
pursuance of an agreement entered into with Central Government for providing services in or outside India in
projects connected with security of India.
Income from service provided to National Technical Research Organisation [Sec. 10(6D)]
Any income arising to a non-resident or to a foreign company, by way of royalty from, or fees for technical services
rendered in or outside India to, the National Technical Research Organisation
Allowance or Perquisite paid Outside India [Sec. 10(7)]
Any allowance or perquisite paid outside India by the Government to a citizen of India for rendering services
outside India.
Death-cum-retirement-gratuity [Sec. 10(10)]
Refer chapter Salaries.
Commutation of Pension [Sec. 10(10A)]
Refer chapter Salaries.
Leave Encashment [Sec. 10(10AA)]
Refer chapter Salaries.
Workmen’s Retrenchment Compensation [Sec. 10(10B)]
Refer chapter Salaries.

46 The Institute of Cost Accountants of India


Basics of Income Tax Act

Compensation under Bhopal Gas Leak Disaster Act, 1985 [Sec. 10(10BB)]
Compensation for any Disaster [Sec. 10(10BC)]
Any amount received or receivable from the Central Government or a State Government or a local authority by
an individual or his legal heir by way of compensation on account of any disaster, except the amount received or
receivable to the extent such individual or his legal heir has been allowed a deduction under this Act on account of
any loss or damage caused by such disaster.
Payment under Voluntary Retirement Scheme [Sec. 10(10C)]
Refer chapter Salaries.
Tax paid by Employer on behalf of Employee on Non-monetary Perquisites u/s 17(2) [Sec. 10(10CC)]
Refer chapter Salaries.

Sum received under a Life Insurance Policy [Sec. 10(10D)]Amended


Any sum received under a life insurance policy including bonus on such policy is wholly exempt from tax.
However, exemption is not available on -
1. any sum received u/s 80DD(3) or u/s 80DDA(3); or
2. any sum received under a Keyman insurance policy; or
3. any sum received under an insurance policy issued on or after 1-4-20128 in respect of which the premium
payable for any of the years during the term of the policy exceeds 10%9 of the actual capital sum assured.
4. Where any Unit Linked Insurance Policy (ULIP), is issued on or after 01-02-2021 and the premium payable
for any of the previous year during the term of such policy exceeds ₹ 2,50,000
¾¾ Where the premium is payable, by a person, for more than one ULIP, issued on or after 01-02-2021, the
exemption shall apply only with respect to those ULIP, where the aggregate amount of premium does not
exceed the aforesaid limit in any of the previous year during the term of any of those policies.
Notes:
(a) Point (3) & (4) shall not apply to any sum received on the death of a person.
(b) Actual capital sum assured shall mean the minimum amount assured under the policy on happening of the
insured event at any time during the term of the policy.
(c) If the exemption u/s 10(10D) is not available in respect of ULIP due to point (4), income shall be taxable under
the head Capital Gains u/s 45(1B) and tax liability shall be computed as per sec. 112A.
(d) For calculating actual capital sum assured (for point 3), no account shall be taken for -
¾¾ the value of any premiums agreed to be returned; or
¾¾ any benefit by way of bonus or otherwise over and above the sum actually assured, which is to be or may
be received under the policy by any person.
Payment from Statutory or Public Provident Fund [Sec. 10(11)]Amended

8 If policy is issued between 01-04-2003 and 31-03-2012, premium payable for any of the years during the term of the policy exceeds 20% of
the actual capital sum assured
9 Where policy is issued on or after 01-04-2013 and Insured is disable or severe disable as per sec. 80U or suffering from disease specified u/s
80DDB – 15%

The Institute of Cost Accountants of India 47


Direct Taxation

Any payment from a provident fund to which the Provident Funds Act, 1925, applies or from any other notified
provident fund set up by the Central Government is exempt.
Exceptions
Interest accrued during the previous year in the account of an employee maintained by the fund shall not be
exempted to the extent it relates to the following amount:

Case Interest not exempted


Where employer is giving Interest on employee’s contribution (made on or after 01-04-2021) in
contribution excess of ₹ 2,50,000 per year
Where employer is not giving Interest on employee’s contribution (made on or after 01-04-2021) in
contribution excess of ₹ 5,00,000 per year
In such case, income shall be computed in such manner as may be prescribed.
Payment from Sukanya Samriddhi Account [Sec. 10(11A)]
Any payment from an account, opened in accordance with the Sukanya Samriddhi Account Rules, 2014 made
under the Government Savings Bank Act, 1873.
Payment from Recognised Provident Fund [Sec. 10(12)]
The accumulated balance due and becoming payable to an employee participating in a recognised provident fund,
to the extent provided in rule 8 of Part A of the Fourth Schedule is exempt
Exceptions
Interest accrued during the previous year in the account of an employee maintained by the fund shall not be
exempted to the extent it relates to the following amount:

Case Interest not exempted


Where employer is giving Interest on employee’s contribution (made on or after 01-04-2021) in
contribution excess of ₹ 2,50,000 per year
Where employer is not giving Interest on employee’s contribution (made on or after 01-04-2021) in
contribution excess of ₹ 5,00,000 per year
In such case, income shall be computed in such manner as may be prescribed.
Payment from National Pension Trust [Sec. 10(12A) & 10(12B)]
Any payment from the National Pension System Trust to an assessee on closure of his account or on his opting out
of the pension scheme referred to in sec. 80CCD, to the extent it does not exceed 60% of the total amount payable
to him at the time of such closure or his opting out of the scheme [Sec. 10(12A)]
Any payment from the National Pension System Trust to an employee under the pension scheme referred to in
sec. 80CCD, on partial withdrawal made out of his account in accordance with the terms and conditions, specified
under the Pension Fund Regulatory and Development Authority Act, 2013, to the extent it does not exceed 25% of
the amount of contributions made by him [Sec. 10(12B)]
Payment from ACF [Sec. 10(12C)]
Any payment from the “Agniveer Corpus Fund” by a person enrolled under the “Agnipath Scheme 2022” is
exempt.

48 The Institute of Cost Accountants of India


Basics of Income Tax Act

Payment from Approved Superannuation Fund [Sec. 10(13)]


Any payment from an approved superannuation fund made -
 on the death of a beneficiary; or
 to an employee in lieu of or in commutation of an annuity on his retirement at or after a specified age or on his
becoming incapacitated prior to such retirement; or
 by way of refund of contributions on the death of a beneficiary; or
 by way of refund of contributions to an employee on his leaving the service (otherwise than by retirement at
or after a specified age or on his becoming incapacitated prior to such retirement) to the extent to which such
payment does not exceed the contributions made prior to 1-4-1962 and any interest thereon.
 by way of transfer to the account of the employee under a pension scheme referred to in sec. 80CCD and
notified by the Central Government
House Rent Allowance [Sec. 10(13A)]
Refer chapter Salaries.
Notified Special Allowances [Sec. 10(14)]
Refer chapter Salaries.
Interest on Securities [Sec. 10(15)]
1. Interest, premium on redemption or other payment on notified securities, bonds or certificates
2. Interest in the hands of an individual and Hindu undivided family on Specified Capital Investment Bonds or
Specified Relief Bonds
3. Interest on specified bonds to non resident or his nominees if such bonds are purchased by a non-resident
Indian in foreign exchange; and
4. The interest and principal received in respect of such bonds, whether on their maturity or otherwise, is not
allowable to be taken out of India. Interest on securities held by the Issue Department of the Central Bank of
Ceylon;
5. Interest payable to any bank incorporated in a country outside India and authorised to perform central banking
functions in that country on any deposits made by it, with the approval of the RBI, with any scheduled bank;
6. Interest payable on a loan advanced by the Nordic Investment Bank for an approved project;
7. Interest payable to the European Investment Bank for financial co-operation agreement;
8. Interest payable by a Government, local authority, certain industrial undertakings or financial institution on
money borrowed before 1/6/2001
9. Interest on securities held by the Welfare Commissioner, Bhopal Gas Victims or deposits for the benefit of the
victims of the Bhopal gas leak disaster.
10. Interest on Gold Deposit Bonds issued under the Gold Deposit Scheme, 1999 or deposit certificates issued
under the Gold Monetisation Scheme, 2015
11. Interest on specified bonds issued by a local authority or by a State Pooled Finance Entity.
12. Interest received by a non-resident or a person who is not ordinarily resident, in India on a deposit made on or
after 1-4-2005 in an offshore banking unit referred in the Special Economic Zones Act, 2005

The Institute of Cost Accountants of India 49


Direct Taxation

13. Interest payable to a non-resident by a unit located in an International Financial Services Centre in respect of
monies borrowed by it on or after 01-09-2019

Income from Leasing of Aircraft [Sec. 10(15A)]


Any payment made, by an Indian company engaged in the business of operation of aircraft, to acquire an aircraft or
an aircraft engine (other than a payment for providing spares, facilities or services in connection with the operation
of leased aircraft) on lease from the foreign Government or a foreign enterprise under an approved agreement. The
agreement must not be entered into -
 between 1-4-1997 to 31-3-1999; and
 on or after 1-4-2007.
Note: “Foreign enterprise” means a person who is a non-resident.

Taxpoint: Tax paid on an agreement made between 1-4-1997 and 31-3-1999 is eligible for exemption u/s 10(6BB).

Scholarship [Sec. 10(16)]

Scholarships granted to meet the cost of education.


Notes:
(a) Cost of education also includes incidental expenses incurred for education.
(b) The exemption is irrespective of actual expenditure.

Daily Allowance, etc. to MP and MLA [Sec. 10(17)]


Any income by way of -
(a) Daily allowance received by any person by reason of his membership of Parliament or of any State Legislature
or of any Committee thereof;
(b) Any allowance received by any person by reason of his membership of Parliament;
(c) Constituency Allowance received by any person by reason of his membership of State legislature;

Awards and Rewards [Sec. 10(17A)]


Any payment made, whether in cash or in kind -
(a) in pursuance of any award instituted in the public interest by the Central Government or any State Government
or by any other approved body; or
(b) as a reward by the Central Government or any State Government for approved purposes.

Pension to receiver of Gallantry Awards [Sec. 10(18)]


Any income by way of -
(a) pension received by an individual who has been in the service of the Central or State Government and has been
awarded “Param Vir Chakra” or “Maha Vir Chakra” or “Vir Chakra” or such other notified gallantry award10;
or
(b) family pension received by any member of the family of such individual.
10 Asadharan Suraksha Seva Praman Patra

50 The Institute of Cost Accountants of India


Basics of Income Tax Act

Family Pension to Widow or Children of Armed Force [Sec. 10(19)]


Family pension received by the widow or children or nominated heirs, of a member of the armed forces (including
para-military forces) of the Union, where the death of such member has occurred in the course of operational
duties, in such circumstances and subject to such conditions, as may be prescribed.
Palace of Ex-ruler [Sec. 10(19A)]
The annual value in respect of any one palace, which is in the occupation of an ex-ruler
Income of Local Authority [Sec. 10(20)]
Following income of a local authority is exempt -
(a) Income chargeable under the head Income from House Property, Capital Gains or Income from other Sources
(b) Income from the supply of commodities (other than water or electricity) or services, within its own jurisdiction
(c) Income from the supply of water services or electricity within or outside its jurisdiction
Income of Scientific Research Association [Sec. 10(21)]
Any income of a scientific research association [being approved for the purpose of Sec. 35(1)(ii)] or research
association which has its object, undertaking research in social science or statistical research [being approved and
notified for the purpose of Sec. 35(1)(iii)], is exempt provided such association—
(a) applies its income, or accumulates it for application, wholly and exclusively to the objects for which it is
established; and
(b) invest or deposit its funds in specified investments.
Income of Professional Institutions [Sec. 10(23A)]
Any income (other than income chargeable under the head “Income from house property” or any income received
for rendering any specific services or income by way of interest or dividends derived from its investments) of
professional association shall be exempt provided -
(a) Such association or institution is established in India having as its object the control, supervision, regulation or
encouragement of the profession of law, medicine, accountancy, engineering or architecture or other specified
profession;
(b) Such association or institution applies its income, or accumulates it for application, solely to the objects for
which it is established; and
(c) The association or institution is approved by the Central Government.
Income of Regimental Fund [Sec. 10(23AA)]
Any income received by any person on behalf of any Regimental Fund or Non-public Fund established by the
armed forces of the Union for the welfare of the past and present members of such forces or their dependants is
exempt.
Income of specified Employee Welfare Fund [Sec. 10(23AAA)]
Income of specified Pension Fund [Sec. 10(23AAB)]
Income of trust for Development of Khadi and Village Industries [Sec. 10(23B)]
Income of Khadi and Village Industries Boards [Sec. 10(23BB)]
Income of body formed for Administration of Public Religious or Charitable Trusts [Sec. 10(23BBA)]

The Institute of Cost Accountants of India 51


Direct Taxation

Any income of any body established under any Central, State or Provincial Act which provides for the administration
of any public, religious or charitable trusts or endowments including Maths, Temples, Gurudwaras, Wakfs, Churches
or other places of public religious worship or societies for religious or charitable purposes.
Income of European Economic Community [Sec. 10(23BBB)]
Income of SAARC Fund [Sec. 10(23BBC)]
Income of ASOSAI-SECRETARIAT [Sec. 10(23BBD)]
Income of Insurance Regulatory Authority [Sec. 10(23BBE)]
Income of the Central Electricity Regulatory Commission [Sec. 10(23BBG)]
Income of the Prasar Bharati (Broadcasting Corporation of India) [Sec. 10(23BBH)]
Income of Certain Funds [Sec. 10(23C)]Amended
Any income received by any person on behalf of
1. The Prime Minister’s National Relief Fund or the Prime Minister’s Citizen Assistance and Relief in Emergency
Situations Fund (PM CARES FUND); [sec. 10(23C)(i)]
2. The Prime Minister’s Fund (Promotion of Folk Art); [sec. 10(23C)(ii)]
3. The Prime Minister’s Aid to Students Fund; [sec. 10(23C)(iii)]
4. The National Foundation for Communal Harmony; [sec. 10(23C)(iiia)]
5. The Swachh Bharat Kosh; [sec. 10(23C)(iiiaa)]
6. The Clean Ganga Fund; [sec. 10(23C)(iiiaaa)]
7. The Chief Minister’s Relief Fund or the Lieutenant Governor’s Relief Fund; [sec. 10(23C)(iiiaaaa)]
8. Any other charitable fund or institution notified by the prescribed authority (subject to condition) [sec. 10(23C)
(iv)]
9. Any trust or institution wholly for public religious purposes or wholly for public religious and charitable
purposes notified by the prescribed authority (subject to conditions) [sec. 10(23C)(v)]
10. Any university or other education institutions, (wholly or substantially financed by Government or having
annual receipt of prescribed limit upto ₹ 5 crores) existing solely for education purposes and not for profit.
[sec.10(23C)(iiiac), (iiiad) (vi)]
11. Any hospital or other institution (wholly or substantially financed by Government or having annual receipt
upto ` 5 crores) for treatment of person suffering from illness or mental defectiveness or during convalescence
or requiring medical attention or rehabilitation, existing solely for philanthropic purposes and not for profit.
[sec.10(23C)(iiiac), (iiiae) and (via)]

Income of Mutual Fund [Sec. 10(23D)]


Any income of -
(a) A Mutual Fund registered under the Securities and Exchange Board of India Act, 1992 or regulation made
thereunder;
(b) A Mutual Fund set up by a public sector bank or a public financial institution or authorised by the Reserve
Bank of India and subject to certain notified conditions.

52 The Institute of Cost Accountants of India


Basics of Income Tax Act

Income of Securitisation Trust [Sec. 10(23DA)]


Any income of a securitisation trust from the activity of securitisation.
 “Securitisation” shall have the same meaning as assigned to it,
(a) in regulation 2(1)(r) of the Securities and Exchange Board of India (Public Offer and Listing of Securitised
Debt Instruments) Regulations, 2008 made under the Securities and Exchange Board of India Act, 1992
and the Securities Contracts (Regulation) Act, 1956; or
(b) in clause (z) of sub-section (1) of section 2 of the Securitisation and Reconstruction of Financial Assets
and Enforcement of Security Interest Act, 2002; or
(c) under the guidelines on securitisation of standard assets issued by the Reserve Bank of India;
 “Securitisation trust” shall have the meaning assigned to it in the Explanation below sec. 115TCA
Income of Investor Protection Fund [Sec. 10(23EA)]
Income (by way of contribution received from recognized Stock exchange and members thereof) of Investor
Protection Fund set up by the recognised Stock Exchanges in India as the Central Government may by notification
in Official Gazette specify shall be exempt.

Income of Credit Guarantee Fund Trust for Small Industries [Sec. 10(23EB)]

Income of Investor Protection Fund set up by Commodity Exchange [Sec. 10(23EC)]

Income of Investor Protection Fund of Depositories [Sec. 10(23ED)]


Any income, by way of contributions received from a depository, of notified Investor Protection Fund set up in
accordance with the regulations by a depository.
However, where any amount standing to the credit of the Fund and not charged to income-tax during any previous
year is shared, either wholly or in part with a depository, the whole of the amount so shared shall be deemed to
be the income of the previous year in which such amount is so shared and shall, accordingly, be chargeable to
income-tax.
Income of Core Settlement Guarantee Fund [Sec. 10(23EE)]
Any specified income of such Core Settlement Guarantee Fund, set up by a recognised clearing corporation in
accordance with the regulations notified by the Central Government.
However where any amount standing to the credit of the Fund and not charged to income-tax during any previous
year is shared, either wholly or in part with the specified person, the whole of the amount so shared shall be deemed
to be the income of the previous year in which such amount is so shared.
Income of Ventures Capital Fund or Venture Capital Company [Sec 10(23FB)]
Any income of a venture capital company or venture capital fund from investment in a venture capital undertaking.
However, w.e.f. A.Y. 2016-17, the exemption is not applicable to any income of a venture capital company or
venture capital fund, being an investment fund specified in clause (a) of the Explanation 1 to sec. 115UB
Non-business income of Investment Fund [Sec. 10(23FBA)]
Any income of an investment fund other than the income chargeable under the head “Profits and gains of business
or profession”.

The Institute of Cost Accountants of India 53


Direct Taxation

Income of Unit holder [Sec. 10(23FBB)]


Any income, referred to in sec. 115UB, to a unit holder of an investment fund, being that proportion of income
which is of the same nature as income chargeable under the head “Profits and gains of business or profession”.
 For the purposes of sec. 10(23FBA) and (23FBB), “investment fund” shall have the meaning assigned to it in
clause (a) of the Explanation 1 to sec. 115UB.
Income from specified fund [Sec. 10(23FBC)]
Any income accruing or arising to, or received by, a unit holder from a specified fund or on transfer of units in a
specified fund is exempt.
 “Specified fund” shall have the same meaning as assigned to it sec. 10(4D)
 “Unit” means beneficial interest of an investor in the fund and shall include shares or partnership interests
Income of Business Trust [Sec 10(23FC)]
Any income of a business trust by way of
(a) interest received or receivable from a special purpose vehicle; or
(b) dividend received or receivable from a special purpose vehicle
 “Special purpose vehicle” means an Indian company in which the business trust holds controlling interest and
any specific percentage of shareholding or interest, as may be required by the regulations under which such
trust is granted registration
Income of Real Estate Investment Trust [Sec. 10(23FCA)]
Any income of a business trust, being a real estate investment trust, by way of renting or leasing or letting out any
real estate asset owned directly by such business trust.
Distributed Income to unit holder of a Business Trust [Sec 10(23FD)]
Any distributed income, referred to in section 115UA, received by a unit holder from the business trust, not
being that proportion of the income which is of the same nature as the income referred to in 10(23FC)(a) [i.e.,
proportionate interest income] or 10(23FC)(b) [i.e., proportionate dividend income where the special purpose
vehicle has exercised the option u/s 115BAA] or 10(23FCA) [i.e., proportionate rental income]
Taxpoint: Such income is taxable in hands of unitholders.
Income to wholly owned subsidiary of Abu Dhabi Investment Authority and Sovereign Wealth Fund [Sec
10(23FE)]
Any income of the specified person in the nature of dividend, interest or long-term capital gains arising from an
investment made by it in India, whether in the form of debt or share capital or unit, if the investment:
(i) is made on or after 01-04-2020 but on or before 31-03-2024;
(ii) is held for at least 3 years; and
(iii) is in:
(a) a business trust referred to in sec. 2(13A)(i); or
(b) a company or enterprise or an entity carrying on the business of developing, or operating and maintaining,
or developing, operating and maintaining any infrastructure facility or other specified business; or

54 The Institute of Cost Accountants of India


Basics of Income Tax Act

(c) a domestic company, set up and registered on or after 01-04-2021, having minimum 75% investments in
one or more of the companies or enterprises or entities referred to in item (b); or
(d) a non-banking financial company registered as an Infrastructure Finance Company as referred to in
notification number RBI/2009-10/316 issued by the Reserve Bank of India or in an Infrastructure Debt
Fund, a non-banking finance company, as referred to in the Infrastructure Debt Fund - Non-Banking
Financial Companies (Reserve Bank) Directions, 2011, issued by the Reserve Bank of India, having
minimum 90% lending to one or more of the companies or enterprises or entities referred to in item (b)
(e) a Category-I or Category-II Alternative Investment Fund regulated under the Securities and Exchange
Board of India (Alternative Investment Fund) Regulations, 2012, having 50% investment in one or more
of the company or enterprise or entity referred above or in an Infrastructure Investment Trust referred to
in sec. 2(13A)(i)
Capital Gains of Resultant Fund [Sec. 10(23FF)]
Any income of the nature of capital gains, arising or received by a non-resident or a specified fund, which is on
account of transfer of share of a company resident in India, by the resultant fund or a specified fund to the extent
attributable to units held by non-resident (not being a permanent establishment of a non-resident in India) in
such manner as may be prescribed, and such shares were transferred from the original fund, or from its wholly
owned special purpose vehicle, to the resultant fund in relocation, and where capital gains on such shares were not
chargeable to tax if that relocation had not taken place.
Income of Trade Union [Sec. 10(24)]
Any income chargeable under the heads “Income from house property” and “ Income from other sources” of -
(a) a registered union within the meaning of the Indian Trade Unions Act, 1926, formed primarily for the purpose
of regulating the relations between workmen and employers or between workmen and workmen.
(b) an association of registered unions
Income of specified Provident Funds, etc. (e.g. RPF, Superannuation fund, Approved gratuity fund) [Sec.
10(25)]
Income of Employees’ State Insurance Fund [Sec. 10(25A)]
Income of Scheduled Tribe [Sec. 10(26)]
Following income of member of a Scheduled Tribe is exempt –
(a) from any source in specified areas or States; or
(b) by way of dividend or interest on any securities.
–– provided he resides in specified area or States.

Income of Sikkimese [Sec. 10(26AAA)]


Following income of an individual, being a Sikkimese, is exempt:
(i) from any source in the State of Sikkim; or
(ii) by way of dividend or interest on securities:
Note: The exemption is not available to a Sikkimese woman who, on or after 1/4/2008, marries an individual who
is not a Sikkimese, except when non-sikkimese male or his ancestors were domiciled in Sikkim or when non-

The Institute of Cost Accountants of India 55


Direct Taxation

sikkimese male or his father or brother from same father or parental grand father were domiciled in Sikkim on or
before 26.04.1975.
Income of an Agricultural produce Market Committee [Sec. 10(26AAB)]
Income of an agricultural produce market committee or board constituted under any law for the time being in force
for the purpose of regulating the marketing of agricultural produce is exempt.
Income of Corporation for promoting the Interests of the Members of the Scheduled Castes or the Scheduled
Tribe or Backward Classes [Sec. 10(26B)]
Income of Corporation for promoting Interest of Members of a Minority Community [Sec. 10(26BB)]
Income of Corporation for the Welfare and Economic Upliftment of Ex-servicemen [Sec. 10(26BBB)]
Income of a Co-operative Society for promoting the Interests of the Members of Scheduled Castes or
Scheduled Tribes [Sec. 10(27)]
Income of specified Boards [Sec. 10(29A)]
Any income accruing or arising to The Coffee Board; The Rubber Board; The Tea Board; The Tobacco Board; The
Marine Products Export Development Authority; The Coir Board; The Agricultural and Processed Food Products
Export Development Authority and The Spices Board.
Subsidy received from Tea Board [Sec. 10(30)]
Any subsidy received from or through the Tea Board under any scheme for replantation or replacement of tea
bushes or for rejuvenation or consolidation of areas used for cultivation of tea as the Central Government may
specify, is exempt
Subsidy received from other Board [Sec. 10(31)]
Any subsidy received from or through the concerned Board (like Coffee Boards, Rubber Board, etc.) under any
such scheme for replantation or replacement of rubber plants, coffee plants, cardamom plants or plants for the
growing of such other commodity or for rejuvenation or consolidation of areas used for cultivation of rubber,
coffee, cardamom or such other specified commodity is exempt.
Income of Minor [Sec. 10(32)]
Income up to ₹ 1,500 is exempt in respect of each minor child whose income is clubbed u/s 64(1A).
Income on Transfer of Units of US 64 [Sec. 10(33)]
Any income arising from the transfer of a capital asset, being a unit of the Unit Scheme, 1964 where such transfer
takes place on or after the 1st day of April, 2002.
Income of Shareholder on Buy-back of Shares [Sec. 10(34A)]
Any income arising to an assessee, being a shareholder, on account of buy back of shares by the company,
which pay additional income-tax u/s 115QA.
Dividend of IFSC unit [Sec 10(34B)]
Dividend income of IFSC unit involved in aircraft leasing business from a company in IFSC engaged in aircraft
leasing business in IFSC is exempt.
Capital Gain on compulsory Acquisition of Urban Land [Sec. 10(37)]
Refer Chapter Capital Gains

56 The Institute of Cost Accountants of India


Basics of Income Tax Act

Capital Gain on transfer under Land Pooling Scheme for Andhra Pradesh [Sec. 10(37A)]
Refer Chapter Capital Gains
Specified Income, Arising from any International Sporting Event [Sec. 10(39)]
Any specified income, arising from any international sporting event held in India, to the person(s) notified by the
Central Government in Official Gazette, if such international sporting event –
(a) is approved by the International body regulating the international sport relating to such event;
(b) has participation by more than 2 countries;
(c) is notified by the Central Government in the Official Gazette for the purpose of this clause.
Note: For the purpose of this clause “the specified income” means the income, of the nature and to the extent,
arising from the international sporting event, which the Central Government may notify in this behalf.
Reconstruction or Revival of Power Generation Subsidiary Company [Sec. 10(40)]
Any income of any subsidiary company by way of grant or otherwise received from an Indian company, being its
holding company engaged in the business of generation, transmission or distribution of power, if such receipts is
for the settlement of dues in connection with reconstruction or revival of an existence business of power generation.
Note: The above clause is applicable if reconstruction or revival of any existing business of power generation is by
way of transfer of such business to the Indian company notified u/s 80-IA (4)(v)(a)
Income of a Non-profit Body or Authority specified by the Central Government [Sec. 10(42)]
Any specified income arising to a body or authority which -
 has been established or constituted or appointed under a treaty or an agreement enterted into by the Central
Government with tow or more countries or a convention signed by the Central Government;
 is established or constituted or appointed not for the purpose of profit;
 is notified by the Central Government.
Reverse Mortgage [Sec. 10(43)]
Any amount received by an individual as a loan, either in lump sum or in instalment, in a transaction of reverse
mortgage is exempt.
New Pension Trust [Sec. 10(44)]
Any income received by any person for, or on behalf of, the New Pension System Trust is exempt
Specified Income of notified body or authority or Board or Trust or Commission [Sec. 10(46)]]
Any specified income arising to a body or authority or Board or Trust or Commission (by whatever name called),
or a class thereof, which —
(a) has been established or constituted by or under a Central, State or Provincial Act, or constituted by the Central
Government or a State Government, with the object of regulating or administering any activity for the benefit
of the general public;
(b) is not engaged in any commercial activity; and
(c) is notified by the Central Government in the Official Gazette

The Institute of Cost Accountants of India 57


Direct Taxation

Credit Guarantee Trust/Fund [Sec 10(46B)]


Any income of specified Credit Guarantee Trust/Fund is exempt.
Infrastructure Debt Fund [Sec. 10(47)]
Any income of notified infrastructure debt fund is exempt.
Import of Crude Oil [Sec. 10(48)]
Any income received in India in Indian currency by a foreign company on account of sale of crude oil or other
notified goods or service to any person in India provided:
(a) receipt of such income in India by the foreign company is pursuant to an agreement or an arrangement entered
into by the Central Government or approved by the Central Government;
(b) having regard to the national interest, the foreign company and the agreement or arrangement are notified by
the Central Government in this behalf; and
(c) the foreign company is not engaged in any activity, other than receipt of such income, in India.
Storage of Crude Oil [Sec. 10(48A)]
Any income accruing or arising to a foreign company on account of storage of crude oil in a facility in India and
sale of crude oil therefrom to any person resident in India provided:
(i) the storage and sale by the foreign company is pursuant to an agreement or an arrangement entered into by the
Central Government or approved by the Central Government; and
(ii) having regard to the national interest, the foreign company and the agreement or arrangement are notified by
the Central Government in this behalf.
Sale of leftover stock of crude oil [Sec. 10(48B)]
Any income accruing or arising to a foreign company on account of sale of leftover stock of crude oil, if any, from
the facility in India after the expiry of the agreement or the arrangement referred to sec. 10(48A) or on termination
of the said agreement or the arrangement, in accordance with the terms mentioned therein, as the case may be.
Income of Indian Strategic Petroleum Reserves Limited [Sec. 10(48C)]
Any income accruing or arising to the Indian Strategic Petroleum Reserves Ltd., being a wholly owned subsidiary
of the Oil Industry Development Board under the Ministry of Petroleum and Natural Gas, as a result of arrangement
for replenishment of crude oil stored in its storage facility in pursuance of directions of the Central Government in
this behalf is exempt.
However, nothing contained in this clause shall apply to an arrangement, if the crude oil is not replenished in the
storage facility within 3 years from the end of the financial year in which the crude oil was removed from the
storage facility for the first time.
Income of certain institutions [Sec. 10(48D)/(48E)]
 Any income accruing or arising to an institution established for financing the infrastructure and development,
set up under an Act of Parliament and notified by the Central Government for the purposes of this clause, for
a period of 10 consecutive assessment years beginning from the assessment year relevant to the previous year
in which such institution is set up [Sec. 10(48D)]
 Any income accruing or arising to a developmental financing institution, licensed by the Reserve Bank of
India under an Act of the Parliament referred to in sec. 10(48D) and notified by the Central Government for
this purposes, for a period of 5 consecutive assessment years beginning from the assessment year relevant to
the previous year in which the developmental financing institution is set up

58 The Institute of Cost Accountants of India


Basics of Income Tax Act

However, the Central Government may, by issuing notification, extend the period of exemption for a further
period, not exceeding 5 more consecutive assessment years, subject to fulfilment of such conditions as may be
specified in the said notification;
Equalization Levy [Sec. 10(50)]
Any income arising from any specified service provided on or after the date on which the provisions of Chapter
VIII of the Finance Act, 2016 comes into force or arising from any e-commerce supply or services made or
provided or facilitated on or after 01-04-2020 and chargeable to equalisation levy under that Chapter.
However, the income shall not include and shall be deemed never to have been included any income which is
chargeable to tax as royalty or fees for technical services in India under this Act read with the agreement notified
by the Central Government u/s 90 or 90A.
Expenditure related to Exempted Income [Sec. 14A]
For the purposes of computing the total income, no deduction shall be allowed in respect of expenditure incurred
by the assessee in relation to income, which does not form part of the total income under this Act. Where the AO is
not satisfied with the correctness of the claim of such expenditure by assessee, he can determine the disallowable
expenditure in accordance with the method prescribed by the CBDT.
 The provisions of this section shall be applicable even in a case where the income, not forming part of the total
income, has not accrued or arisen or has not been received during the previous year relevant to an assessment
year and the expenditure has been incurred during the said previous year in relation to such income not
forming part of the total income.
Special Provision in respect of Newly established Units in SEZ [Sec. 10AA]
Applicable to: All assessee
Conditions to be satisfied
1. The assessee is an entrepreneur as defined in Sec.2(j) of SEZ Act, 2005.
2. The undertaking has begun or begins to manufacture or produce articles or things or provide services on or
after 01/04/2005 but not after 31/03/2021 in any SEZ.
3. New Business: Business should not be formed by splitting up or reconstruction of an existing business.
Exception:
However, this condition is not applicable when conditions given u/s 33B are satisfied, which are as follows -
(a) The business of an industrial undertaking carried on in India is discontinued in any previous year by
reason of extensive damage to, or destruction of, any building, machinery, plant or furniture owned by
the assessee being used for business purpose.
(b) Such damage was caused due to -
(i) flood, typhoon, hurricane, cyclone, earthquake or other convulsion of nature; or
(ii) riot or civil disturbance; or
(iii) accidental fire or explosion; or
(iv) action by an enemy or action taken in combating an enemy (whether with or without a declaration of
war),
(c) Such business is re-established, reconstructed or revived by the assessee at any time before the expiry of
3 years from the end of previous year in which such damage was caused.

The Institute of Cost Accountants of India 59


Direct Taxation

4. New Plant and Machinery: Such undertaking should not be formed by transfer of machinery or plant
previously used for any purpose.
Exception:
(a) A plant or machinery is deemed as a new asset if the following conditions are satisfied -
(i) Such plant or machinery is imported into India;
(ii) Depreciation on such asset has not been allowed under this Act to any person; and
(iii) The assessee was the first user of such asset in India.
(b) Where the total value of old plant and machinery transferred to the new business does not exceed 20% of
total value of plant and machinery used in such business, then this condition is deemed to be satisfied.
Taxpoint: Usage of old plant and machinery upto 20% of total value of plant and machinery is allowed.
5. A report of a chartered accountant in specified Form must be uploaded one month prior to the due date of filing
return of income.

Quantum of Deduction

Period Deduction
For first 5 years from Profits of the business of the undertaking × Export turnover
the commencement of Total turnover of the business carried on by the undertaking
operation
For next 5 years 50% of [Profits of the business of the undertaking × Export turnover]
Total turnover of the business carried on by the undertaking
For next 5 years 50% of [Profits of the business of the undertaking × Export turnover]
Total turnover of the business carried on by the undertaking
Conditions: Such profit must be credited in reserve account called “SEZ Re-
investment Allowance Reserve A/c”.
Utilisation of such Reserve:
¾¾ Such reserve shall be utilised for the purposes of acquiring new machinery
or plant, which is first put to use before the expiry of a period of next 3 years
following the previous year in which the reserve was created.
¾¾ Until the acquisition of new machinery or plant, such reserve can be utilised
for any purpose of the business of the undertaking other than for distribution by
way of dividends or profits or for remittance outside India as profits or for the
creation of any asset outside India.
Export turnover means -
It means the consideration in respect of export by the undertaking, being the Unit of articles or things or
services received in or brought into, India but does not include:
1. Freight, telecommunication charges and insurance attributable to the delivery of the articles or things
outside India;
2. Expenses incurred in foreign exchange in providing technical services outside India.

60 The Institute of Cost Accountants of India


Basics of Income Tax Act

Quick MCQs:-

1. Income that do not form part of the total Income are called
(a) Exempt Income
(b) Deduction
(c) Excluded Income
(d) None of the above

2. Compensation receive on account of disaster is exempt u/s 10(10BC) if it is received form


(a) Central Government
(b) State Government
(c) Local Authority
(d) All of the above

3. Which of the following income is not exempt under Section 10?


(a) Share in total income of firm
(b) Bonus on life insurance
(c) Income from agriculture in Lahore
(d) Income from mutual funds

4. Sec.10AA exemption is applicable for –


(a) Special Economic Zone only
(b) Special Economic Zone and Domestic Tariff Area
(c) SEZ and FTZ & ETP converted into SEZ
(d) None of the above

5. Which of the following income would be exempt in the hands of a Sikkimese Individual?
(a) Only income from any sources in the State so of Sikkim
(b) Only income by way of dividend
(c) Only income from interest on securities
(d) All of the above

The Institute of Cost Accountants of India 61


Direct Taxation

Exercise
A. Theoretical Questions:
¾¾ Multiple Choice Questions:
1. Out of the following which one is not a capital receipt?
a. Dividend on investment
b. Bonus Shares
c. Sale of know-how
d. Compensation received for vacating business place
2. Which of the following is Casual Income?
a. Dividend income
b. Winning from lotteries
c. Interest received
d. Pension received
3. Which of the following receipt is not included in the term ‘Income’ under the Income-tax Act,
1961?
a. Profits and gains of Business or Profession
b. Profit in lieu of salary
c. Dividend
d. Reimbursement of travelling expenses
4. A person is said to be a person of Indian origin if –
a. He or either of his parents were born in undivided India
b. He or either of his siblings were born in undivided India
c. He or either of his parents or either of his grandparents were born in undivided India
d. He was born in India
5. Income received in India in the previous year is taxable in the hands of –
a. Resident
b. Non-Resident
c. All assessee irrespective of residential status
d. Not Ordinarily resident
6. An individual is said to be resident in India if –
a. He has a house in India
b. He is in India in the previous year for a period of 182 days or more
c. He is in India for a period of 30 days or more during the previous year and for 365 or more
days during 4 previous years immediately preceding the relevant previous year
d. His parents are Indian citizen

62 The Institute of Cost Accountants of India


Basics of Income Tax Act

7. The incidence of taxation depends on the –


a. Residential status of the assessee
b. Accommodation of the assessee
c. Citizenship of the as assessee
d. Marital status of the assessee
8. Which of the following is an agriculture income?
a. Dividend paid by a company to its shareholders out of agricultural income
b. Share of Profit of a Partner from a firm engaged in an agriculture operation
c. Income from supply of water by an assessee from a tank in agriculture land
d. Interest received by a money lender in the form of agricultural produce
9. In case of an assessee engaged in the business of manufacturing of tea, his agricultural income is –
a. 60% of total receipt of the business
b. 60% of income of the business
c. Nil
d. 40% of income of the business
10. Income from saplings shall be considered as __________ .
a. Agricultural Income
b. Business Income
c. Partly agricultural income and partly business income
d. Income from other sources
[Answer : 1-a ; 2-b ; 3-d ; 4-c ; 5-c ; 6-b ; 7-a ; 8-b ; 9-c ; 10-a]

B. Numerical Questions
¾¾ Multiple Choice Questions
1. An individual (aged 28 Years) born in India left for first time for employment in France on
30.10.2023. His visit outside India is for the first time. His residential status for the assessment
year 2024-25 will be –
a. Resident and ordinarily resident
b. Resident but not ordinarily resident
c. Non-resident
d. Residential Status is not applicable
2. Income of ₹ 3,00,000 is received in Sri Lanka by an ordinarily resident of India. But later on
₹ 50,000 is remitted to India –

The Institute of Cost Accountants of India 63


Direct Taxation

a. ₹ 3,00,000 will be taxable


b. ₹ 3,50,000 will be taxable
c. It is not taxable at all
d. ₹ 50,000 will be taxable

3. An individual is said to be a resident in India in the previous year (in which the Feb month has 29
days) if he is in India in that year for a period of ___.
a. 182 days or more
b. 183 days or more
c. 70 days or more
d. 150 days or more
4. Mr. X is engaged in growing and manufacturing tea in India. His income from this activity is
₹ 1,40,000. His agriculture income will be –
a. ₹ 70,000
b. ₹ 84,000
c. ₹ 1,40,000
d. ₹ 56,000
[Answer : 1-a ; 2-a ; 3-a ; 4-b]
¾¾ Unsolved Case
1. Parikshit (aged 25 years) is engaged in growing and manufacturing tea in India. His profit for the
previous year 2023-24 amounts to ₹ 10,00,000 which includes profit of ₹ 2,00,000 from sale of
green leaves plucked in his own garden. He has no other income during the year.
On the basis of aforesaid information, you are requested to answer the following:
a. What would be his agricultural income?
b. What would be his tax liability for the relevant assessment year
[Answer : [Hints: (a) ₹ 6,80,000; (b) ₹ 1,560]]
¾¾ References:
https://www.incometaxindia.gov.in/
https://www.incometax.gov.in/
https://www.indiabudget.gov.in/

64 The Institute of Cost Accountants of India


Heads of Income 2
This Module includes

2.1 Salaries

2.2 Income from House Property

2.3 Profits and Gains of Business or Profession

2.4 Capital Gains

2.5 Income from Other Sources

The Institute of Cost Accountants of India 65


Heads of Income
SLOB Mapped against the Module:
1. To acquire application-oriented knowledge and skill for appreciating various provisions of the Income
Tax Act, 1961 and attain abilities to solve problems while computing tax liabilities.
2. To attain abilities to apply various provisions of direct taxation laws, including assessment to identify
the impacts thereof on business decisions.
3. To gather knowledge of various compliance related provisions of direct taxation laws and attain
abilities to ensure due compliance to avoid any eventual risks.

Module Learning Objectives:

After studying this module, the students will be able to –

� Appreciate the provisions of computing income under different heads of income

� Appreciate the point of time when income is taxable in respective heads

� Apply the knowledge to ascertain the gross total income of the person

� Understand the various compliances like tax audit, etc.

66 The Institute of Cost Accountants of India


Heads of Income

Salaries 2.1
Salary is the recompense or consideration given to a person for the pains he has bestowed upon another’s business”
– Stroud’s Judicial Dictionary

2.1.1 Basic Elements of Salary


 Payer and payee must have
employer and employee (or
Master & Servant) relationship;
and
 Payment must have been made by
the employer in such capacity.

Employer-employee relationship
A payment can be construed as salary only if the payer is the employer and the payee is the employee of the payer.
 Criteria for employer-employee relationship : The key criteria to hold this relationship is that, employee is
always bound to work as per the direction and supervision of the employer.
 Payment in employer’s capacity : To treat any payment as salary it is necessary that the payer, being the
employer, must have made the payment in such (employer’s) capacity.
 Contract of service vs contract for service : In “contract of service”, the employer can direct and control the
duties and the manner of performance of the employee hence employer-employee relationship exists in such
contract. However, in case of “contract for service” the contractee can simply decide and quote the object or
target to be achieved but cannot decide or direct the manner of performance.
 Agent and Principal : If a person is acting as an agent for his principal, any commission or remuneration
earned by the agent is not taxable under the head “Salaries”. This is because, an agent is not the employee of
his principal.
 Salary received by a partner from its firm shall not be taxable as salary, because there is no employer-
employee relationship between the firm and the partner. Such salary shall be taxable under the head “Profits &
gains of business or profession”.
 Salary received by proprietor from his proprietorship firm is not an income. As proprietor and proprietorship
firm are the same person and no one can earn from himself.
 Remuneration to director from his company can be treated as salary only if the director is employee of the
company, otherwise the same shall be taxable under the head “Income from other sources”.
Note : Directors’ sitting fee is taxable under the head “Income from other sources”.

The Institute of Cost Accountants of India 67


Direct Taxation

 Pension received by the widow or legal heir of deceased employee is not taxable as salary as no employer-
employee relationship exists between the payer and the payee. However, such amount shall be taxable under
the head “Income from other sources”.
 Remuneration received by Judges is taxable under the head “Salaries” even though they are not having any
employer.
Concluding the above discussions, a payment received for services rendered, from a person other than employer, is
not taxable under the head “Salaries” but may be taxed under the head “Profits & gains of business or profession”
or “Income from other sources”.

Illustration 1 :
State whether the following receipts should be treated as salary or not?
 A teacher receives emoluments in kind from school in which he teaches.
Yes, it is immaterial whether salary has been received in cash or in kind.
 A teacher of a college receives fees from an University for checking answer sheets.
No, as employer – employee relationship does not exist between payer and payee. (College-teacher is not the
employee of the University). Such receipt shall be taxable under the head ‘Income from other sources’.
 A payment made to the Member of the Parliament or the State legislature.
No, as employer-employee relationship does not exist.
A member of the Parliament or the State legislature is not treated as employee of the Government. Payment
received by them shall be taxable under the head “Income from other sources”.

2.1.2 Section-wise Scheme

2.1.3 Definition of Salary [Sec. 17(1)]


As per sec. 17(1) of the Income-tax Act, 1961, salary includes the following :
a. Wages;
b. Any annuity or pension;
c. Any gratuity;
d. Any fees, commission, perquisite or profits in lieu of or in addition to any salary or wages;

68 The Institute of Cost Accountants of India


Heads of Income

e. Any advance of salary;


f. Any payment received in respect of any period of leave not availed of by the assessee;
g. The portion of the annual accretion in any previous year to the balance at the credit of an employee, participating
in recognised provident fund, to the extent it is taxable;
h. Transferred balance in a Recognised Provident Fund to the extent it is taxable.
i. Contribution made by the employer in the previous year, to the account of an employee under a pension
scheme referred to in sec. 80CCD [National Pension Scheme and Atal Pension Yojana].

2.1.4 General Notes


 Salary & Wages are identical in the Income-tax Act
 Voluntary Payments : The Act does not make any difference between voluntary and contractual payment.
Both are taxable as salary.
 Remuneration for Extra Work : Where an employee gets extra payment from his employer (in such capacity)
for work performed outside the duties of his office and thus, such payment shall be taxable as salary.
 Salary from more than one source : If an individual receives salary from more than one employer during the
same previous year, salary from each employer shall be accumulated and taxable under the head “Salaries”.
 Salary from former, present or prospective employer is chargeable to tax under the head “Salaries”. E.g.
Pension from a former employer and advance salary from prospective employer shall be taxable under the
head “Salaries”.
 Foregoing of salary : Once salary has been earned by an employee, its subsequent waiver does not make it
exempt from tax liability. Such waiver shall be treated as application of the income.
Note : However, where an employee opts to surrender his salary to the Central Government u/s 2 of Voluntary
Surrender of Salaries (Exemption from Taxation) Act, 1961, the salary so surrendered shall not be taxable.

2.1.5 Basis of Charge [Sec. 15]


Salary is chargeable to tax either on ‘due’ basis or on ‘receipt’ basis, whichever is earlier. Hence, taxable salary
includes :
a. Advance salary (on ‘receipt’ basis): Salary paid in advance is taxable under the head ‘Salaries’ in the year of
receipt.
Note : Such advance salary shall not be included again in the total income when the salary becomes due.
b. Outstanding salary (on ‘due’ basis) : Salary falling due is taxable under the head ‘Salaries’ in the year in
which it falls due.
Note : Such due salary shall not be included again in the total income when it is received.
c. Arrear salary : Any increment in salary with retrospective effect which have not been taxed in the past, such
arrears will be taxed in the year in which it is allowed. Arrear salary are taxable on receipt basis
Provision illustrated
Mr. X joined A Ltd. for a salary of ₹ 25,000 p.m. on 1/4/2021. In the year 2022-23, his increment decision was
pending. On 1/12/2023, his increment was finalized as for 2022-23: ₹ 5,000 p.m. and for 2023-24 ₹ 7,500 p.m.
Such arrear salary received on 5/12/2023. Find Gross taxable salary. Further, salary of April 2024 has also been
received in advance on 15/03/2024.

The Institute of Cost Accountants of India 69


Direct Taxation

Solution :
Gross taxable salary for the previous year 2023-24 shall be calculated as under :

Particulars Workings Amount


Salary for 2023-24 (25,000 + 5,000 + 7,500) x 12 4,50,000
Arrear salary for 2022-23 (5,000) × 12 60,000
Advance salary for April 2024 37,500
Gross total salary 5,47,500
Taxpoint : Method of accounting followed by the employee is irrelevant
Salary due vs Salary accrued
Salary due is different from salary accrued.
Example 1 : Mr. X joined an organisation for ₹ 10,000 p.m. on 1st Dec. 2023, in which salary falls due on 1st
day of every next month. In such case taxable salary for the previous year 2023-24 shall be ₹ 30,000 calculated as
under :

Month Amount of Salary Due date of salary Taxable in the P.Y.


December 2023 10,000 1/1/2024 2023-24
January 2024 10,000 1/2/2024 2023-24
February 2024 10,000 1/3/2024 2023-24
March 2024 10,000 1/4/2024 2024-25
Advance salary vs Advance against salary
‘Advance salary’ is taxable u/s 17(1)(e) whereas ‘Advance against salary’ is treated as loan hence, not taxable
under the head “Salaries”.
Place of accrual of salary
Salary which is received in India or earned in India shall be taxable in hands of all assessee whether resident or non
resident in India. Salary is deemed to be earned in India provided -
a. The service is rendered in India;
b. The rest period or leave period, which is preceded and succeeded by the service rendered in India and forms
part of the service contract of employment.
Exceptions :
 Salary paid to a Government employee, being a citizen of India, is deemed to accrue in India, irrespective of
place of work [Sec. 9(1)(iii)].
 Pensions payable outside India to certain categories of Government employees and Judges who permanently
reside outside India, shall not be deemed to arise or accrue in India. [Sec. 9(2)]
Taxpoint : Salary is earned at the place where service is rendered.

Employee Employer Place of service Salary received Taxable


Any Any India Any where Yes

70 The Institute of Cost Accountants of India


Heads of Income

Employee Employer Place of service Salary received Taxable


Any Any Any where In India Yes
Ordinarily resident in India Any Any where Any where Yes
Indian citizen Any Government Outside India Any where Yes
Not ordinarily resident/Non resident Any Outside India Outside India No

 Basic Salary : It is the sum paid by employer


to employee as salary and shall be fully
taxable.
 Pay-Scale (Grade system) : It is a system of
payment where increment scale is pre-known
to employee. E.g. Basic salary is given as
5,000 – 1,000 – 8,000 – 2,000 – 12,000. The
above data indicates the increment schedule.
As per this schedule initial payment is ₹ 5,000
p.m. which will increased by ₹ 1,000 every
year until salary reaches to ₹ 8,000 p.m. Once
salary reaches to ₹ 8,000 then increment will
be ₹ 2,000 every year till salary reaches the
scale of ₹ 12,000. Accordingly, basic salary is
calculated.
 Dearness Allowance (DA) or Dearness
Pay (DP) : It is an extra amount given to an
employee to meet the burden of inflation or
increased cost of living. This is fully taxable.
Note : Sometimes, it is given that DA/DP is not forming a part of retirement benefit (Leave encashment,
Pension, Provident Fund, etc.). In such case, DA/DP itself shall be fully taxable. However, for calculating
taxable Leave encashment, Pension, HRA, etc., DA/DP will be included in ‘salary’ only if it forms a part of
retirement benefit.
 Fees : An employee may be given apart from basic salary, extra remuneration for doing specific job under the
terms of employment. Such extra remuneration is termed as fee and shall be fully taxable.
 Commission : It may be as a percentage of turnover or as a percentage of profit. In either case, it is taxable.
 Bonus : Bonus may be contractual or voluntary. In either case, it is fully taxable.
(i) Contractual bonus is taxable as bonus whereas voluntary bonus is taxable as perquisite.
(ii) It is taxable in the year of receipt.
(iii) If arrear bonus is received, assessee can claim relief u/s 89(1).

The Institute of Cost Accountants of India 71


Direct Taxation

Retirement Benefits

2.1.6 Gratuity
Gratuity is a retirement benefit given by the employer to the employee in consideration of past services. Sec. 10(10)
deals with the exemptions from gratuity income. Such exemption can be claimed by a salaried assessee. Gratuity
received by an assessee other than employee shall not be eligible for exemption u/s 10(10). E.g. Gratuity received
by an agent of LIC of India is not eligible for exemption u/s 10(10) as agents are not employees of LIC of India.
Treatment :

Case A: Gratuity received during continuation of service


Gratuity received during continuation of service is fully taxable in the hands of all employees (whether Government
or non-Government employee).
Case B: Gratuity received at the time of termination of service by Government employee
Gratuity received at the time of termination of service by Government employee is fully exempt from tax u/s
10(10)(i).
Taxpoint : Government employee, here, includes employee of the Central or the State Government or local
authority but does not include employee of statutory corporation.
Case C: Gratuity received at the time of termination of service by non–government (including foreign
government) employee, covered by the Payment of Gratuity Act
In such case, minimum of the following shall be exempted from tax u/s 10(10)(ii) :

72 The Institute of Cost Accountants of India


Heads of Income

1. Actual Gratuity received;


2. ₹ 20,00,000; or
3. 15 working days salary for every completed year of service

[Arithmetically, 15 × Completed year of service × Salary p.m.]


26
Notes :
a. Completed year of service includes any fraction in excess of 6 months. (e.g. 7 years 9 months will be treated
as 8 years; 7 years 5 months will be treated as 7 years and 7 years 6 months will be treated as 7 years).
b. Salary here means Basic + DA, last drawn

In case of an employee of a seasonal establishment: 15 days shall be replaced by 7 days. (i.e., 7 × Completed
26
year of service × Salary p.m.)
In case of a piece-rated employee: 15 days salary would be computed on the basis of average of total wages
(excluding wages paid for over time) received for a period of 3 months immediately preceding the termination
of his employment.

Illustration 2 :
Ashok, an employee of ABC Ltd., receives ₹ 8,05,000 as gratuity under the Payment of Gratuity Act, 1972. He
retires on 10th September, 2023 after rendering service for 35 years and 7 months. The last drawn salary was
₹ 32,700 per month. Calculate the amount of gratuity chargeable to tax.
Solution :
Computation of taxable gratuity of Mr. Ashok for the A.Y. 2024-25 :

Particulars Details Amount


Gratuity received 8,05,000
Less : Minimum of the following is exempted as per Sec 10(10)(ii) :
a) Actual gratuity received 8,05,000
b) Statutory Amount 20,00,000

c) 15 × completed year of service × salary p.m. [ 15 × 36 × ₹ 32,700] 6,79,154 6,79,154


26 26
Taxable Gratuity 1,25,846
Case D : Gratuity received at the time of termination of service by non-government employee (including
foreign government employee) not covered under the Payment of Gratuity Act
Gratuity received at the time of termination of service by non-government employee being not covered under the
Payment of Gratuity Act shall be exempted from tax u/s 10(10)(iii) to the extent of lower of the following :
1. Actual Gratuity received;
2. ₹ 20,00,000; and
3. ½ × Completed year of service x Average Salary p.m.

The Institute of Cost Accountants of India 73


Direct Taxation

Notes :
a. While calculating completed year of service ignore any fraction of the year. (e.g. 7 years 9 months will be
treated as 7 years only)
b. Average Salary here means, Basic + DA# + Commission (being a fixed percentage on turnover) being last
10 months average salary, immediately preceding the month of retirement. (E.g. If an employee retires on
18/11/2023 then 10 months average salary shall be a period starting from Jan’ 2023 and ending on Oct’ 2023).
#
If DA is not forming a part of retirement benefit then the same shall not be included in salary for above
purpose. However, DA itself shall be fully taxable.

Illustration 3 :
Mr. Oldman retired from his job after 29 years 6 months and 15 days of service on 17/12/2023 and received gratuity
amounting ₹ 4,00,000. His salary at the time of retirement was basic ₹ 6,000 p.m., dearness allowance ₹ 1,200 p.m.,
House rent allowance ₹ 2,000, Commission on turnover 1%, Commission on profit ₹ 5,000. He got an increment
on 1/4/2023 of ₹ 1,000 p.m. in Basic. Turnover achieved by assessee ₹ 1,00,000 p.m. Calculate his taxable gratuity
if he is a —
a. Government employee
b. Non-Government employee, covered by the Payment of Gratuity Act;
c. Non-Government employee not covered by the Payment of Gratuity Act
Solution :
a. Government employee: Taxable amount: Nil as per section 10(10)(i).
b. & c. Other cases :
Computation of taxable gratuity of Mr. Oldman for the A.Y. 2024-25 :

Case (b) Case (c)


Particulars
Details Amount Details Amount
Gratuity received 4,00,000 4,00,000
Less : Min. of the following is exempted u/s 10(10)
–– Actual gratuity received 4,00,000 4,00,000
–– Statutory Amount 20,00,000 20,00,000
15 × completed year of service × salary p.m. 1,24,615 1,24,615
––
26
[ 15 × 30 × 7,200]
26
–– ½ x completed year of service x salary p.m. 1,16,000 1,16,000
[½ x 29 x 8,000]
Taxable Gratuity 2,75,385 2,84,000
Workings for case (b) :
1. Completed year of service is 30 years.
2. Salary here means (Basic + Dearness Allowance) last drawn. i.e. (₹ 6,000 + ₹ 1,200) = ₹ 7,200

74 The Institute of Cost Accountants of India


Heads of Income

Workings for case (c) :


1. Completed year of service is 29 years.
2. Salary here means Basic + Dearness Allowance + Commission on turnover, being last 10 months average just
preceding the month of retirement, as shown below :

Particulars 1 2 3 4 5 6 7 8 9 10 Total
Feb’23 Mar Apr May June July Aug Sept Oct Nov
Basic 5,000 5,000 6,000 6,000 6,000 6,000 6,000 6,000 6,000 6,000 58,000
D.A. 1,200 1,200 1,200 1,200 1,200 1,200 1,200 1,200 1,200 1,200 12,000
Commission 1,000 1,000 1,000 1,000 1,000 1,000 1,000 1,000 1,000 1,000 10,000
Total 80,000
Average salary = ₹ 80,000 / 10 months 8,000

Note : Applicable in Case D and not in Case C


While claiming the statutory amount (i.e. ₹ 20,00,000) any amount earlier claimed as deduction u/s 10(10) shall
be reduced from ₹ 20,00,000.
Example 2 : An assessee left a job in the year 2001-02 and claimed a deduction of ₹ 40,000 for gratuity in that
year. He joined another organisation, left the same in the year 2023-24, and received a gratuity of ₹ 19,80,000.
While calculating exemption for gratuity for the assessment year 2024-25, statutory amount of ₹ 20,00,000 shall
be reduced by earlier deduction claimed i.e. ₹ 40,000. Hence, statutory deduction limit for the assessee in the
A.Y. 2024-25 will be ₹ 19,60,000 only.
Note : Applicable in Case C and Case D
Where gratuity is received from more than one employer : Where gratuity is received from more than one
employer in the same previous year, the aggregate amount exempt from tax shall not exceed statutory deduction.

Test Yourself
1. Mrs. Payal retired from Pajeb Ltd. on 29/8/2023 after completing 30 years 9 months of service and received
a gratuity of ₹ 2,00,000. Her last drawn salary: Basic ₹ 10,000 p.m. + DA ₹ 5,000 p.m. + Commission
being a fixed percentage on turnover ₹ 2,000 p.m. (turnover evenly accrued during the previous year) +
Commission on Profit ₹ 23,000 for this year. Her last increment was on 1/1/2023 in Basic ₹ 1,000 and in DA
₹ 500. Find taxable gratuity assuming that she is not covered by the Payment of Gratuity Act.
Hints
1. Nil;

Case E : Gratuity received after death of employee


The Act is silent on treatment of gratuity received after death of employee. However, on following grounds, it can
be concluded that gratuity received by a legal heir shall not be taxable in the hands of the recipient -
 A lump sum payment made gratuitously to widow or legal heir of employee, who dies while in service, by way
of compensation or otherwise is not taxable under the head “Salaries”. [Circular No.573, Dated 21.08.1990]
 Unutilised deposit under the capital gains deposit account scheme shall not be taxable in the hands of legal
heir. [Circular No.743 dated 6/5/1996]

The Institute of Cost Accountants of India 75


Direct Taxation

 Legal representative is not liable for payment of tax on income that has not accrued to the deceased till his
death.
 Leave salary paid to the legal heir of deceased employee is not taxable as salary. [Circulars Letter No.
F.35/1/65-IT(B), dated 5/11/1965]. Further, leave salary by a legal heir of the Government employee who died
in harness is not taxable in the hands of the recipient [Circulars No.309, dated 3/7/1981].
Taxpoint : If gratuity becomes due before the death of the assessee (no matter when and by whom received), it
shall be taxable in the hands of employee. Whereas if gratuity becomes due after the death of assessee, it shall
not be taxable (even in the hands of legal heir of the assessee).

Illustration 4 :
Mrs. X is working with ABC Ltd. since last 30 years 9 months. Her salary structure is as under :
Basic ₹ 5,000 p.m. Dearness allowance ₹ 3,000 p.m.
On 15/12/2023, she died. State the treatment of gratuity in following cases:
Case 1 : Mrs. X retired on 10/12/2023 & gratuity ₹ 4,00,000 received by her husband (legal heir) as on 18/12/2023.
Case 2 : Husband of Mrs. X received gratuity on 18/12/2023 falling due after death of Mrs. X.
Mrs. X is covered by the Payment of Gratuity Act.
Solution :
In Case 1, Computation of taxable gratuity in hands of Mrs. X for the A.Y. 2024-25 :

Particulars Details Amount


Total Gratuity received 4,00,000
Less : Minimum of the following is exempted as per Sec 10(10)(ii):
a) Actual gratuity received 4,00,000
b) Statutory Amount 20,00,000

c) 15 × completed year of service x salary p.m. [ 15 × 31 × ₹ 8,000] 1,43,077 1,43,077


26 26
Taxable Gratuity 2,56,923
In Case 2, Since gratuity falls due after the death of Mrs. X hence the same is not taxable in hands of Mrs. X. The
said gratuity is not taxable even in hands of husband of Mrs. X.

2.1.7 Leave Salary Encashment


As per service contract and discipline, normally, every employee is allowed certain period of leave (with pay) every
year. Such leave may be availed during the year or accumulated by the employee. The accumulated leave lying to
the credit of an employee may be availed subsequently or encashed. When an employee receives an amount for
waiving leave lying to his credit, such amount is known as leave salary encashment.

76 The Institute of Cost Accountants of India


Heads of Income

Treatment :

Case A : Leave salary received during continuation of service


Leave salary during continuation of service is fully taxable in the case of the Government employee as well as other
employees [Sec. 17(1)(va)].
Case B : Leave salary received by Government employee on termination of service
At the time of termination of service, leave salary received by the Central or State Government employee is fully
exempted u/s 10(10AA)(i).
Taxpoint : Government employee here does not include employee of local authority or public sector undertaking
or foreign Government employee.
Case C : Leave salary received by non-Government employee on termination of service
At the time of termination of service, leave salary received by a non-Government employee (including employee
of foreign Government, local authority, public sector undertaking) is exempted to the minimum of the following
u/s 10(10AA)(ii) :
a. Actual amount received as leave salary
b. ₹ 25,00,000/-
c. 10 × Average salary p.m.
d. To the maximum of 30 days (normally taken as 1 month) average salary1 for every completed year of service2,
subject to deduction for actual leave availed during the tenure of service.
Academically : [{(1 x completed year of service) – leave actually taken in terms of month} x average salary
p.m.]
1 Average salary means Basic + DA# + Commission (as a fixed percentage on turnover) being last 10
months average salary ending on the date of retirement or superannuation. (e.g. if an employee retires on
18/11/2022 then 10 months average salary shall be a period starting from 19th Jan’ 2022 and ending on
18th Nov’ 2022).
# If DA is not forming a part of retirement benefit then the same shall not be included in salary for the
above purpose. However, DA itself shall be fully taxable.
2 While calculating completed year of service, ignore any fraction of the year. E.g. 10 years 9 months shall
be taken as 10 years.

The Institute of Cost Accountants of India 77


Direct Taxation

Notes :
a. Leave encashment received from more than one employer: Where leave encashment is received from more
than one employer in the same previous year, the aggregate amount exempt from tax shall not exceed the
statutory deduction i.e. ₹ 25,00,000.
b. Earlier deduction claimed for leave encashment: While claiming the statutory amount (i.e. ₹ 25,00,000) any
deduction claimed earlier as leave encashment shall be reduced from ₹ 25,00,000.

Illustration 5 :
a. Mr. Bhanu is working in Zebra Ltd. since last 25 years 9 months. Company allows 2 months leave for every
completed year of service to its employees. During the job, he had availed 20 months leave. At the time of
retirement on 10/8/2023, he got ₹ 1,50,000 as leave encashment. As on that date, his basic salary was ₹ 5,000
p.m., D.A. was ₹ 2,000 p.m., Commission was 5% on turnover + ₹ 2,000 p.m. (Fixed p.m.). Turnover effected
by the assessee during last 12 months (evenly) ₹ 5,00,000. Bhanu got an increment of ₹ 1,000 p.m. from
1/1/2023 in basic and ₹ 500 p.m. in D.A. Compute his taxable leave encashment salary.
b. How shall your answer differ if the assessee had taken 2 months leave instead of 20 months, during his
continuation of job.
Solution :
Working :
1. Completed year of service: 25 years 9 months = 25 years
2. As per sec. 3(35) of the General Clauses Act, 1897, month shall mean a month reckoned according to the
British calendar e.g. the period commencing from 7th September & end on 6th October shall be a month.
3. Salary here means Basic + Dearness Allowance + Commission on turnover (last 10 months average from the
date of retirement)

Oct’ Aug
22 Jan’ 10
Particulars Nov Dec Feb Mar April May June July Total
(21 23 Days
days)
Basic 2,710 4,000 4,000 5,000 5,000 5,000 5,000 5,000 5,000 5,000 1,613 47,323
D.A. 1,016 1,500 1,500 2,000 2,000 2,000 2,000 2,000 2,000 2,000 645 18,661
Commission 500000 × 5% × 10/12 20,833
Total 86,817
Average salary i.e. ₹ 86,817 / 10 months 8,682
Monthly fixed commission is irrelevant. Commission as fixed percentage of turnover is to be considered.
Computation of taxable leave encashment salary of Mr. Bhanu for the A.Y. 2024-25

Case (b) Case (c)


Particulars
Details Amount Details Amount
Leave encashment received 1,50,000 1,50,000
Less: Min. of the following is exempted u/s 10(10AA)(ii):
a. Actual amount received 1,50,000 1,50,000

78 The Institute of Cost Accountants of India


Heads of Income

Case (b) Case (c)


Particulars
Details Amount Details Amount
b. Statutory Amount 25,00,000 25,00,000
c. 10 months × Av. Salary p.m. (10 × 8,682) 86,820 86,820
d. [{1x completed year of service - Leave taken} × 1,16,000 1,16,000
salary p.m.]
43,410^ 43,410 1,99,686# 86,820
Taxable Leave Encashment 1,06,590 63,180
^ [{1 x 25 – 20} × 8,682]
# [{1 x 25 – 2} × ₹ 8,682]
Illustration 6 :
Mr. Das retired on 31/3/2024. At the time of retirement, 18 months leave was lying to the credit of his account.
He received leave encashment equivalent to 18 months Basic salary ₹ 1,26,000. His employer allows him 1½
months leave for every completed year of service. During his tenure, he availed of 12 months leave. At the time of
retirement, he also gets D.A. ₹ 3,000. His last increment of ₹ 1,000 in basic was on 1/4/2023. Find taxable leave
encashment.
Solution :
Working :
1. Calculation of completed year of service: Employee has received 18 months leave encashment on termination
of service as well he had enjoyed leave of 12 months during his tenure. That means he had received a leave
benefit of 30 months. Since leave allowed by employer is 1½ months for every completed year of service, this
signifies that Mr. Das had completed 20 years (being 30 ÷ 1½) of service.
2. Salary here means, Basic + DA + Commission, being last 10 months average from the date of retirement.
There is no increment in last 10 months (last increment was on 1/4/2023) and there is no commission, hence
Average Salary = ₹ 7,000 (i.e. ₹ 1,26,000/18) + ₹ 3,000 = ₹ 10,000 p.m.
Computation of taxable leave encashment of Mr. Das for the A.Y. 2024-25 :
Particulars Details Amount
Leave Encashment received 1,26,000
Less : Minimum of the following is exempt u/s 10(10AA)(ii):
a) Actual amount received 1,26,000
b) Statutory Amount 25,00,000
c) 10 months × Avg. Salary p.m. (10 × 10,000) 1,00,000
d) {1 × completed year of service - Leave taken} × Avg. salary p.m. 80,000 80,000
[{1 × 20 – 12} × ₹ 10,000]
Taxable Leave Encashment 46,000
Case D: Leave salary paid to the legal heir
Leave salary paid to the legal heir of deceased employee is not taxable. [Circulars Letter No. F.35/1/65-IT(B),
dated 5/11/1965]. Further, leave salary received by a legal heir of the Government employee who died in harness
is not taxable in the hands of the recipient [Circulars No.309, dated 3/7/1981].

The Institute of Cost Accountants of India 79


Direct Taxation

Test Yourself
1. Miss Mamta has been working with X Ltd. for last 20 years and 11 months. Before that, she was employed
in Y Ltd. wherefrom, she received leave encashment ₹ 40,000 (fully claimed as deduction).
On 31/7/2023, she has taken voluntary retirement from X Ltd. and received leave encashment ₹ 3,00,000. At
the time of retirement, her monthly salary detail was as under:
Basic Salary ₹ 5,000, D.A. ₹ 2,000, Commission as a % on turnover ₹ 2,000 p.m. (Turnover accrued evenly
throughout the year). Her last increment was ₹ 1,000 in basic salary fell due on 1/1/2023.
X Ltd. allows 20 days leave to its employees for each completed year of service. During continuation of
service she availed 160 days leave. Find her taxable leave salary.
Hints
1. ₹ 2,30,400;

2.1.8 Pension [Sec. 17(1)(ii)]


Pension means a periodical payment received by an employee after his retirement. On certain occasions, employer
allows to withdraw a lump sum amount as the present value of periodical pension. When pension is received
periodically by employee, it is known as Uncommuted pension. On the other hand, pension received in lump sum
is known as Commuted pension. Such lump sum amount is determined considering factors like the age and health
of the recipient, rate of interest, etc.
Treatment :

Case A : Uncommuted pension


Uncommuted pension is fully taxable in the hands of all employees whether Government or Non – Government
employee.
Case B : Commuted pension received by a Government employee
Commuted pension received by a Government employee is fully exempt from tax u/s 10(10A)(i).
Note: Government employee here includes employee of the Central or State Government, Local authority as well
as employee of Statutory corporation. Judges of the High Court and the Supreme Court are also entitled to the
exemption [Circular No.623 dated 6/1/1992]
Case C : Commuted pension received by an employee who also received gratuity [Sec. 10(10A)(ii)]

80 The Institute of Cost Accountants of India


Heads of Income

One third of total pension (which assessee is normally entitled for) commuted is exempt.
Taxpoint: It is immaterial whether the employee is covered by the Payment of Gratuity Act or not.
Case D : Commuted pension received by an employee who does not receive gratuity [Sec. 10(10A)(ii)]
One half of total pension (which assessee is normally entitled for) commuted is exempt.
Notes :
a. Pension received by a widow or legal heir of a deceased employee shall not be taxable as salary but taxable u/s
56 as income from other sources (further refer chapter “Income from other sources”.)
b. Where commuted pension is taxable, relief u/s 89 is available.
c. Pension received from United Nations Organisation is not taxable. Further, pension received by a widow of the
United Nations ex-officials from UN Joint Staff Pension Fund is also exempt
Illustration 7 :
Mr. Amit has retired from his job on 31/3/2023. From 1/4/2023, he was entitled to a pension of ₹ 3,000 p.m. On
1/8/2023, he got 80% of his pension commuted and received ₹ 1,20,000. Compute taxable pension if he is:
Case a) Government employee; Case b) Non-Government employee & not receiving gratuity;
Case c) Non-Government employee (receiving gratuity, but not covered by the Payment of Gratuity Act)
Solution :
Computation of taxable pension of Mr. Amit for the A.Y. 2024-25 :
Particulars Case a Case b Case c
Details Amount Details Amount Details Amount
Uncommuted Pension
- 1/4/2023 to 31/7/2023 (₹ 3,000x4) 12,000 12,000 12,000
- 1/8/2023 to 31/3/2024 (₹ 600 x 8) 4,800 16,800 4,800 16,800 4,800 16,800
Commuted Pension 1,20,000 1,20,000 1,20,000
Fully exempted u/s 10(10A)(i) 1,20,000 Nil
Exempted u/s 10(10A)(ii) 75,000 45,000
(½ of ₹ 1,50,000#)
Exempted u/s 10(10A)(ii) 50,000 70,000
(1/3 of ₹ 1,50,000#)
Taxable Pension 16,800 61,800 86,800
# Commuted Amount for 80% of pension = ₹ 1,20,000. Commuted amount for 100% of pension = ₹ 1,50,000

Test Yourself
1. Mr. Narayan retired from service on 1/6/2023. As on that date, his monthly salary was Basic ₹ 5,000 p.m.,
Commission on turnover 5%. Total turnover achieved by him during last 10 months (occurred evenly)
₹ 5,00,000. On retirement, after 20 years 6 months of service, he received gratuity ₹ 5,00,000, leave salary
₹ 3,00,000. He is entitled to pension of ₹ 1,500 p.m. On 1/1/2024, he commuted 60% of his pension and
received ₹ 90,000. Compute gross salary assuming he is covered by the Payment of Gratuity Act.
Hints
1. ₹ 7,34,608;

The Institute of Cost Accountants of India 81


Direct Taxation

2.1.9 Retrenchment Compensation


Retrenchment means cancellation of contract of service by employer.
Tax Treatment [Sec. 10(10B)] : Any compensation received by a worker at the time of retrenchment is exempted
to the extent of minimum of the following :
a. Actual amount received;
b. ₹ 5,00,000; or
c. An amount calculated in accordance with the provisions of sec. 25F(b) of Industrial Dispute Act, 1947 (Under
the said Act a workman is entitled to retrenchment compensation equivalent to 15 days’ average pay, for every
completed year of service or any part thereof in excess of 6 months).
Notes :
a. In case, where the compensation is paid under any scheme approved by the Central Government nothing shall
be taxable.
b. Compensation received by a workman at the time of closing down of the undertaking in which he is employed
is treated as compensation received at the time of his retrenchment.

2.1.10 Compensation Received at the time of Voluntary Retirement [Sec. 10(10C)]


If an employee accepts retirement willingly in lieu of compensation then such retirement is known as Voluntary
Retirement. Voluntary retirement compensation received or receivable by an employee is eligible for exemption
subject to the following conditions -
Conditions for exemption
1. Compensation is received from specified employer#
2. Compensation is received as per Voluntary Retirement Scheme (VRS) framed in accordance with prescribed
guidelines*
Amount of exemption
Exemption shall be minimum of the following -
a. Actual amount received as per guidelines; or
b. ₹ 5,00,000.
*Guidelines [Rule 2BA]
1. Scheme (VRS) must be applicable to all employees (other than director) who have either completed age of
40 years or has completed 10 years of service. (This condition is, however, not applicable in the case of an
employee of a public sector company)
2. Such scheme must be framed to reduce the number of employees.
3. The vacancy caused by VRS is not to be filled up.
4. The retiring employee is not to be employed in another company or concern belonging to the same management.
5. The amount of compensation does not exceed
¾¾ the amount equivalent to 3 months salary for each completed year of service; or
¾¾ salary at the time of retirement multiplied by the balance month of service left.

82 The Institute of Cost Accountants of India


Heads of Income

Note: Salary here means [Basic + DA (if forms a part of retirement benefit) + fixed percentage of commission
on turnover], last drawn.
# Specified Employer
Any company; or An authority established under Central, State or Provincial Act; or A local authority; or A Co-
operative society; or A specified University; or An Indian Institute of Technology (IIT); or Any State Government;
or The Central Government; or Notified Institution of Management (IIM Ahmedabad, IIM Banglore, IIM Calcutta,
IIM Lucknow, and the Indian Institute of Foreign Trade New Delhi); or Notified Institution.
Taxpoint: Voluntary retirement compensation received from the employer being an individual, firm, HUF, AOP,
etc. is fully taxable in the hands of employee.
Note:
 Where exemption is allowed to an assessee under this section in any assessment year then no deduction is
allowed in any subsequent assessment years. It means deduction under this section is allowed once in life of
an assessee.
 Where any relief has been allowed to an assessee u/s 89 in respect of voluntary retirement, no exemption shall
be allowed under this section.

2.1.11 Annuity [Sec. 17(1)(ii)]


Annuity means a yearly allowance, income, grant of an annual sum, etc. for life or in perpetuity.
Treatment :
Case Treatment
Annuity payable by a present employer, whether Fully taxable as salary
voluntarily or contractual
Annuity received from an ex-employer Fully taxable as ‘profit in lieu of salary’ u/s 17(3)(ii).
Annuity received from a person other than employer Taxable as per provision of Sec. 56 as ‘Income from
e.g. from insurer, etc. other sources’.

2.1.12 Salary received in lieu of notice period


When an employer retrenches an employee then he has to give a proper notice. If an employer fails to do so then he
will have to pay salary equivalent to notice period, apart from retrenchment compensation. Such amount is known
as salary received in lieu of notice period and it is fully taxable.

2.1.13 Profits in lieu of salary [Sec. 17(3)]


Following receipts are taxable as profits in lieu of salary :
1. The amount of any compensation due to or received by an assessee from his employer or former employer at
or in connection with the (a) termination of his employment, (b) modification of the terms and conditions of
employment.
2. Any payment due to or received by an assessee from his employer or former employer except the following:
¾¾ Gratuity exempted u/s 10(10);
¾¾ House rent allowance exempted u/s 10(13A);
¾¾ Commuted pension exempted u/s 10(10A);
¾¾ Retrenchment compensation exempted u/s 10(10B);

The Institute of Cost Accountants of India 83


Direct Taxation

¾¾ Payment from an approved Superannuation Fund u/s 10(13);


¾¾ Payment from statutory provident fund or public provident fund;
¾¾ Payment from recognised provident fund to the extent it is exempt u/s 10(12).
3. Any payment from unrecognised provident fund or such other fund to the extent to which it does not consist
of contributions by the assessee or interest on such contributions.
4. Any sum received by the employee under the Keyman Insurance Policy including the sum allocated by way of
bonus on such policy.
5. Any amount due to or received by the employee (in lump sum or otherwise) prior to employment or after
cessation of employment.

2.1.14 Allowances
Allowance means fixed quantum of money given regularly in addition to salary to meet particular requirement. The
name of particular allowance may reveal the nature of requirement, e.g. House Rent Allowance, Tiffin Allowance,
Medical Allowance etc.
Allowances at a glance :

Tax treatment of various allowances


Following allowances are fully taxable :

Allowances Meaning
City Compensatory An allowance to meet personal expenses, which arise due to special circumstances,
Allowance or to compensate extra expenditure by reason of posting at a particular place.
Tiffin Allowance An allowance to meet the expenditure on tiffin, refreshment etc.
Medical Allowance An allowance to meet the expenditure on medical treatment etc.
Servant Allowance An allowance to meet the expenditure of servant for personal purpose.

84 The Institute of Cost Accountants of India


Heads of Income

Non-practicing Allowance given to professionals to compensate them for restriction on private


Allowance practice.
Warden or Proctor Allowances given to employees of educational institutions for working as warden of
Allowance the hostel or working as proctor in the institutions.
Allowances given to an employee, when he is sent on deputation for a temporary
Deputation Allowance
period from his permanent place of service.
Entertainment It is an allowance to meet expenditure on entertainment, by whatever name called.
Allowance Government employee can claim deduction u/s 16(ii) discussed later in this chapter.
House rent allowance (HRA) [Sec. 10(13A) and rule 2A]
An allowance to meet the expenses in connection with the rent of the house, by whatever name called.
Tax Treatment: Minimum of the following is exempted from tax :
a. Actual HRA received.
b. An amount equal to 50% of salary1 (when house is situated in a metro city) or 40% of salary1 (when house is
situated in any other place) for the relevant period
c. The excess of rent paid over 10% of salary1. [Arithmetically, (Rent Paid – 10% of Salary)]
1.
Salary here means: Basic + D.A. (if it forms a part of retirement benefit) + Commission as a fixed % on turnover.
Notes :
a. Salary shall be determined on due basis for the period for which the employee occupies rented accommodation
in the previous year and gets HRA.
b. Exemption is not available if employee lives in his own house, or in a house for which he does not pay any
rent.
c. For criteria of 50% or 40% of salary as deduction, place of employment is not significant but place where the
house is situated is important.
d. Deduction from HRA depends on Salary of the employee, Amount of HRA, place of residence (not place of
employment), rent paid by the employee.
Illustration 8:
X, a resident of Ajmer, receives ₹ 48,000 as basic salary during the previous year 2023-24. In addition, he gets
₹ 4,800 as dearness allowance forming part of basic salary, 7% commission on sales made by him (sale made by X
during the relevant previous year is ₹ 86,000) and ₹ 6,000 as house rent allowance. He, however, pays ₹ 5,800 as
house rent. Determine the quantum of exempted house rent allowance.
Solution :
Computation of taxable house rent allowance of X for the A.Y. 2024-25 :
Particulars Details Amount
House Rent Allowance Received 6,000
Less : Minimum of the following being exempted u/s 10(13A)
a) Actual Amount Received 6,000
b) 40% of Salary (Note) 23,528
c) Rent paid – 10% of salary [₹ 5,800 – ₹ 5,882] Nil Nil
Taxable House Rent Allowance 2,56,923

The Institute of Cost Accountants of India 85


Direct Taxation

Note : Salary for the purpose of HRA

Basic salary ₹ 48,000


Dearness Allowance ₹ 4,800
Commission (7% of ₹ 86,000) ₹ 6,020
Total ₹ 58,820

Hence, exemption u/s 10(13A) is Nil.

Illustration 9 :

Compute the taxable house rent allowance of Mr. Abhijeet from the following data :

 Basic Salary ₹ 5,000 p.m., D.A. ₹ 2,000 p.m., HRA ₹ 4,000 p.m., Rent paid ₹ 4,000 p.m. in Pune.

 On 1/07/2023, there is an increment in Basic salary by ₹ 1,000.

 On 1/10/2023, employee hired a new flat in Kolkata at the same rent as he was posted to Kolkata.

 On 1/01/2024, employee purchased his own flat and resides there.

Solution :

Computation of taxable house rent allowance of Mr. Abhijeet for the A.Y. 2024-25 :

Particulars Details Amount Amount


House Rent Allowance Received (from 1.4.2023 to 30.6.2023) 12,000
Less : Minimum of the following being exempted u/s 10(13A)
a) Actual Amount Received 12,000
b) 40% of Salary [(₹ 5,000 + ₹ 2,000) x 3] 8,400
c) Rent paid – 10% of salary (₹ 12,000 – ₹ 2,100) 9,900 8,400 3,600
House Rent Allowance Received (from 1.7.2023 to 30.9.2023) 12,000
Less : Minimum of the following being exempted u/s 10(13A)
a) Actual Amount Received 12,000
b) 40% of Salary [(₹ 6,000 + ₹ 2,000) x 3] 9,600
c) Rent paid – 10% of salary (₹ 12,000 – ₹ 2,400) 9,600 9,600 2,400
House Rent Allowance Received (from 1.10.2023 to 31.12.2023) 12,000
Less : Minimum of the following being exempted u/s 10(13A)
a) Actual Amount Received 12,000
b) 50% of Salary [(₹ 6,000 + ₹ 2,000) x 3] 12,000
c) Rent paid – 10% of salary (₹ 12,000 – ₹ 2,400) 9,600 9,600 2,400
House Rent Allowance Received (from 1.1.2024 to 31.3.2024)
(Fully taxable as assessee resides in his own house) 12,000
Taxable House Rent Allowance 20,400

86 The Institute of Cost Accountants of India


Heads of Income

Test Yourself

1. Mr. Mohit has following salary structure:

Basic salary ₹ 5,000 p.m., D.A. ₹ 2,000 p.m. (40% does not form a part of retirement benefit), HRA ₹ 5,000
p.m. Rent paid by assessee for a house in Kolkata ₹ 4,000 p.m. Find taxable HRA.

Hints

1. ₹ 22,800;

Special allowance exempt u/s 10(14)


Allowances, deduction from which depends on actual expenditure [Sec. 10(14)(i)] :

Allowance Meaning

An allowance, by whatever name called, to meet the cost of travel on tour. Cost of travel
Travel or transfer
includes any sum paid in connection with transfer, packing and transportation of personal
Allowance
effects on such transfer.
An allowance, by whatever name called, granted on tour (or for the period of journey in
Daily Allowance connection with transfer) to meet the ordinary daily charges incurred by employee on
account of absence from his normal place of duty.
Any allowance granted to meet the expenditure on conveyance in performance of duties of
the office, provided free conveyance is not provided by the employer.
Conveyance
Allowance Taxpoint : Expenditure for covering the journey between office and residence is not treated
as expenditure in performance of duties of office and consequently not covered under this
allowance. (Refer Transport allowance)
Any allowance (by whatever name called) to meet the expenditure of assistant or helper,
Helper / Assistant provided such helper is appointed for the performance of duties of an office.
Allowance
Taxpoint : Servant allowance is fully taxable.
Any allowance, by whatever name called, granted to encourage academic, research
Research
and other professional pursuits. This allowance may also be termed as Professional
Allowance
Development / Academic allowance
Any allowance, by whatever name called, to meet the expenditure on purchase or
Uniform maintenance of uniform wear, during the performance of duties of an office.
Allowance Taxpoint : Uniform allowance is different from Dress allowance. Dress allowance is fully
taxable.
Tax Treatment of aforesaid allowances :
Minimum of the following shall be exempted :
a. Actual amount received; or
b. Actual expenditure incurred for such purpose.

The Institute of Cost Accountants of India 87


Direct Taxation

Allowances, deduction from which do not depend on actual expenditure [Sec. 10(14)(ii)]
Children Education Allowance
An allowance to meet the expenses in connection with education of children, by whatever name called.
Treatment : Minimum of the following is exempted from tax -
a. ₹ 100 per month per child (to the maximum of two children)
b. Actual amount received for each child (to the maximum of two children)
Children Hostel Allowance
An allowance to meet the hostel expenses of children, by whatever name called.
Treatment : Minimum of the following is exempted from tax -
a. ₹ 300 per month per child (to the maximum of two children)
b. Actual amount received for each child (to the maximum of two children)
Notes for Children Education Allowance and Hostel Allowance :
a. Child includes adopted child, step-child but does not include illegitimate child and grandchild.
b. Child may be major or minor child.
c. Deduction is available irrespective of actual expenditure incurred on education of child.

Illustration 10 :
Mr. Laloo Singh, received education allowance of ₹ 80 p.m. for his 1st child, ₹ 90 p.m. for his 2nd child and ₹ 120
p.m. for his 3rd child. He also received hostel allowance of ₹ 1,000 p.m. None of his children are studying. Find
taxable Children Education Allowance and Hostel allowance.
Solution :
Computation of taxable children education allowance for Mr. Laloo Singh for the A.Y. 2024-25 :

Particulars Details Amount


Hostel allowance 12,000
Less : Exempted (₹ 300 x 2 x 12) 7,200 4,800
Children Education allowance [(₹ 80 x 12) + (₹ 90 x 12) + (₹ 120 x 12)] 3,480
Less : Exempted {(₹ 100 + ₹ 90) x 12} 2,280 1,200
Taxable Allowance 6000
Note : Education allowance is allowed for any two children of assessee therefore education allowance of first child
(which is the lowest one i.e. ₹ 80 only) is not considered, to avail higher deduction.

Illustration 11 :
Mr. & Mrs. X have three children and two of them are not studying. Both Mr. & Mrs. X are working in A Ltd. and
getting children education allowance ₹ 500 per month and hostel allowance ₹ 1,000 per month. Compute taxable
children education allowance and hostel allowance.

88 The Institute of Cost Accountants of India


Heads of Income

Solution :
Computation of taxable allowance of Mr. & Mrs. X for the A.Y. 2024-25 :
Mr. X Mrs. X
Particulars
Details Amount Details Amount
Education allowance (₹ 500 x 12) 6,000 6,000
Less : Exemption (₹ 100 x 12 x 2) 2,400 3,600 2,400 3,600
Hostel Allowance (₹ 1,000 x 12) 12,000 12,000
Less : Exemption (₹ 300 x 12 x 2) 7,200 4,800 7,200 4,800
Taxable Allowance 8,400 8,400

Test Yourself
1. Mr. Anand has six children. He receives Children education allowance ₹ 1,000 p.m. and Hostel allowance
₹ 1,500 p.m. None of his children are studying. Compute taxable allowances.
2. Mr. Sushank has only one child aged one year. His employer allows him Education allowance ₹ 80 p.m. and
Hostel allowance ₹ 1,000 p.m. His child has neither been sent to school nor to any hostel. Compute taxable
allowances.
Hints
1. ₹ 20,400; 2. ₹ 8,400

Truck Driver’s Allowance


Any allowance (by whatever name called) granted to an employee working in any transport system to meet his
personal expenditure during his duty performed in the course of running of such transport (from one place to
another place), provided such employee is not in receipt of daily allowance.
Treatment : Minimum of the following shall be exempted :
a. 70% of allowance.
b. ₹ 10,000 p.m.
Taxpoint : If assessee is in receipt of Daily allowance then above allowance shall be fully taxable.

Transport Allowance
An allowance, by whatever name called, to meet the expenditure for the purpose of travelling between the place of
residence and the place of duty.
Available to: Assessee is blind / deaf and dumb / orthopaedically handicapped.
Treatment : Minimum of the following shall be exempted :
a. Actual amount received; or
b. ₹ 3,200 p.m.
Taxpoint : No exemption is available to the assessee other than specified above.
Allowance to Government employees outside India
As per sec. 10(7), any allowance or perquisite allowed outside India by the Government to an Indian citizen for
rendering services outside India is wholly exempt from tax.

The Institute of Cost Accountants of India 89


Direct Taxation

Taxpoint :
1. Assessee must be -
a. Government employee b. Citizen of India; and c. Working outside India
2. Any allowance or perquisite to such employee shall be exempted u/s 10(7)
Allowance received from UNO (United Nations Organisation)
Basic salary or Allowance paid by the UNO to its employees are not taxable.
Compensatory allowance under Article 222(2) of the Constitution
It is fully exempt from tax.
Allowance to judges of the High Court or the Supreme Court
Any allowance paid to Judges of the High Court u/s 22A(2) and sumptuary allowance u/s 22C of the “High Court
Judges (Conditions of Service) Act, 1954” is not taxable. Allowance to the Supreme Court Judges u/s 23B of the
“Supreme Court Judges (Conditions of Service) Act, 1958” is also exempt.
Salary to teacher or professor from SAARC Member States [DTAA]
Salary including allowances and perquisites of a teacher or professor or research scholars from SAARC Member
States shall not be taxable if following conditions are satisfied :
1. Such professor, teacher or research scholar is a resident of other SAARC member State (i.e., Bangladesh,
Bhutan, India, Maldives, Nepal, Pakistan & Sri Lanka) prior to visiting another member State.
Taxpoint : An individual is deemed to be a resident of a member State if he/she is resident in that member
State in the fiscal year in which he visits the other member State or in the immediately preceding fiscal year.
2. Such visit is for the purposes of teaching or engaging in research or both at a university or college or similar
approved institution in that other Member State.
3. The remuneration from aforesaid activities in other Member State is exempt for a period of 2 years from the
date of arrival in the other member State.
Illustration 12 :
Mr. Mugal joined Star Ltd. on 1/4/2023. Details regarding his salary are as follows :
Particulars Amount (in `)
Basic 5,000 p.m.
Dearness Allowance 2,000 p.m. (50% considered for retirement benefit)
Education Allowance 1,000 p.m. (he has 1 son and 3 daughters)
Hostel Allowance 2,000 p.m. (none of the children is sent to hostel)
Medical Allowance 1,000 p.m. (total medical expenditure incurred ₹ 3,000)
Transport Allowance 1,800 p.m. (being used for office to residence & vice versa)
Servant Allowance 1,000 p.m.
City compensatory Allowance 2,000 p.m.
Entertainment Allowance 1,000 p.m.
Assistants Allowance 3,000 p.m. (paid to assistant ₹ 2,000 p.m.)
Professional Development Allowance 2,000 p.m. (actual expenses for the purpose ₹ 8,000 p.m.)
Bonus 24,000 p.a.
Commission 9,000 p.a.
Fees 5,000 p.a.
Compute his gross taxable salary for the assessment year 2024-25.

90 The Institute of Cost Accountants of India


Heads of Income

Solution :
Computation of gross taxable salary of Mr. Mugal for the A.Y. 2024-25 :
Particulars Details Amount Amount
Basic Salary 60,000
Bonus 24,000
Commission 9,000
Fees 5,000
Allowances
Dearness Allowance 24,000
Education Allowance 12,000
Less : Exemption (₹ 100 x 2 x 12) 2,400 9,600
Hostel Allowance 24,000
Less : Exemption (₹ 300 x 2 x 12) 7,200 16,800
Medical Allowance 12,000
Transport Allowance 21,600
Less : Exemption Nil 21,600
Servant Allowance 12,000
City Compensatory allowance 24,000
Entertainment Allowance 12,000
Assistance Allowance 36,000
Less : Exemption (Being actual expenditure) 24,000 12,000
Professional development allowance 24,000
Less : Exemption (Actual expenditure max. of amount received) 24,000 Nil 1,44,000
Gross Taxable Salary 2,42,000
Illustration 13 :
Miss Sonal, being a citizen of India and Government employee has following salary details :
(Amount in `)
Basic Salary 2,000 p.m.
Dearness Allowance 3,000 p.m.
Dearness Pay 1,000 p.m.
Fees 50,000 p.a.
House Rent Allowance 5,000 p.m. (Rent paid for Kolkata house ₹ 4,000 p.m.)
Children Education allowance 3,000 p.m. (She is having one adopted child)
Children allowance 1,000 p.m.
Hostel allowance 2,000 p.m.
Dress Allowance 5,000 p.m. (Actual expenditure ₹ 10,000 p.m.)
Uniform Allowance 2,000 p.m. (Actual expenditure ₹ 1,000 p.m.)
Tiffin Allowance 1,000 p.m.
Education Allowance for her own education 2,000 p.m. (Actual expenditure ₹ 1,500 p.m.)
Compute her gross salary for the assessment year 2024-25.

The Institute of Cost Accountants of India 91


Direct Taxation

Solution :
Computation of gross taxable salary of Miss Sonal for the A.Y. 2024-25 :

Particulars Details Amount Amount


Basic Salary 24,000
Fees 50,000
Allowances
Dearness Allowance 36,000
Dearness Pay 12,000
House Rent Allowance 60,000
Less : Minimum of the following u/s 10(13A)
a) Actual Amount Received ₹ 60,000
b) 50% of Salary, i.e. 50% of (24,000 + 36,000 + 12,000) ₹ 36,000
c) Rent Paid – 10% of Salary (48,000 – 7,200) ₹ 40,800 36,000 24,000
Children Education Allowance 36,000
Less : Exemption (₹ 100 x 1 x 12) 1,200 34,800
Children Allowance 12,000
Hostel Allowance 24,000
Less : Exemption (₹ 300 x 1 x 12) 3,600 20,400
Dress Allowance (fully taxable) 60,000
Uniform Allowance 24,000
Less : Exemption (₹ 1,000 x 12) 12,000 12,000
Tiffin Allowance 12,000
Education allowance for own study 24,000
Less : Exemption (₹ 1,500 x 12) 18,000 6000 2,29,200
Gross Taxable Salary 3,03,200

Illustration 14 :
In the above illustration, how shall your answer differ if Miss Sonal is working outside India and rent paid for the
house in Japan.

Solution :
Computation of gross taxable salary of Miss Sonal for A.Y. 2024-25 :

Particulars Amount
Basic Salary 24,000
Fees 50,000
Gross Taxable Salary 74,000
Note : Since, Miss Sonal, being Government-employee and citizen of India, is working outside India. Hence, all
allowances paid to her by the Government are exempted u/s 10(7).

92 The Institute of Cost Accountants of India


Heads of Income

2.1.15 Perquisite [Sec. 17(2)]


Meaning and Chargeability
In common parlance, perquisite means, any casual emoluments or benefits attached to an office or position, in
addition to salary or wages, which is availed by an employee. In other words, perquisites are the benefits in addition
to normal salary.
As per sec. 17(2) of the Income tax Act, Perquisite includes -
i. Value of rent-free accommodation provided by the employer.
ii. Value of concession in rent in respect of accommodation provided to the assessee by his employer.
iii. The value of any benefit or amenity granted or provided free of cost or at a concessional rate to a ‘specified
employees$.
iv. Amount paid by an employer in respect of any obligation which otherwise would have been payable by the
employee.
Taxpoint : Any obligation of the employee met by the employer shall be taxable on cash basis i.e. in the year
in which the amount is paid by the employer.
Example 3 : Employer has paid employees’ professional tax liability pertaining to period 2022-23 in April
2023, such perquisite shall be taxable in the previous year 2023-24.
v. Sum payable by an employer, whether directly or through a fund other than recognised provident fund or
approved superannuation fund or deposit-linked insurance fund, to effect an assurance on the life of the
assessee or to effect a contract for an annuity.
Taxpoint : Such sum shall be taxable on accrual basis.
vi. The value of any specified security or sweat equity shares allotted or transferred, directly or indirectly, by the
employer, or former employer, free of cost or at concessional rate to the assessee.
vii. The amount or the aggregate of amounts of any contribution made to the account of the assessee by the
employer :
a. in a Recognised Provident Fund (RPF);
b. in the scheme referred to in sec. 80CCD(1) [i.e., NPS]; and
c. in an approved superannuation fund,
- to the extent, it exceeds ₹ 7,50,000 in a previous year.
Taxpoint : There is combined upper limit of ₹ 7,50,000 in respect of employer’s contribution in a year to
NPS, superannuation fund and recognised provident fund and any excess contribution is taxable.
viii. The annual accretion (like interest, dividend, etc.) during the previous year to the balance at the credit of the
aforesaid fund or scheme to the extent it relates to the contribution referred above.
Taxpoint : Such accretion shall be included in the total income and shall be computed in such manner as may
be prescribed.
ix. the value of any other fringe benefit or amenity as may be prescribed.

The Institute of Cost Accountants of India 93


Direct Taxation

Notes :
a. Perquisites are taxable under the head “Salaries” only if, they are :
¾¾ Allowed by an employer to his employee or any member of his household.
¾¾ Resulting in the nature of personal advantage to the employee.
¾¾ Derived by virtue of employee’s authority.
b. Perquisite may be contractual or voluntary. In other words, it is not necessary that the benefit must have been
received under an enforceable right.
c. Perquisite may be received from the former, present or prospective employer
d. Member of household includes :
¾¾ Spouse (whether dependent or not) ¾¾ Parents (whether dependent or not); ¾¾ Servants; and
¾¾ Children and their spouse (whether dependent or not); ¾¾ Dependents.
$
Specified employees [Sec. 17(2)(iii)]
Specified employee means :
1. A director employee.
Note : It is immaterial -
a. whether he is a nominee of the workers, financial institutions, etc. on the board;
b. whether the employee is full time director or a part time; and
c. whether he was a director throughout the previous year or not.

94 The Institute of Cost Accountants of India


Heads of Income

Taxpoint :
¾¾ A director-employee shall be treated as specified employee of that company only.
Example 4 : If Manu is working with X Ltd. as director-employee and with Y Ltd. as employee only, she will
be treated as specified employee only for X Ltd. and not for Y Ltd.
¾¾ Director even for a day is construed as specified employee of such company.
2. An employee who has substantial interest in the employer company.
Substantial interest means the employee who beneficially holds 20% or more voting power in the employer
company.
Taxpoint :
¾¾ Such employee shall be treated as specified employee of that company only.
¾¾ The main criteria is beneficial ownership and not the legal ownership.
¾¾ Substantial interest must be held by the assessee individually, and not together with relative.
Example 5 : Mr. Mohan holds 18% equity share of X Ltd. and his wife holds 7% equity share of the same
company. In such case Mr. Mohan will not be treated as specified employee.
3. An employee whose aggregate salary from all employers together exceeds ₹ 50,000 p.a.
For computing the sum of ₹ 50,000, following are to be excluded/deducted :
a. All non-monetary benefits;
b. Non-taxable monetary benefits;
c. *Deduction u/s 16(ia), 16(ii) and 16(iii) [Discussed later in this chapter]; and
d. Employer’s contribution to Provident Fund.
Taxpoint :
¾¾ Where salary is received from two or more employers, the aggregate salary from all employers shall be
considered for calculation of above ceiling. And if aggregate salary exceeds ₹ 50,000 p.a. the employee
shall be treated as specified employee of all employers.
Example 6 : Mr. Rohan is working with X & Co. and Y Ltd. His taxable monetary salary from X & Co.
is ₹ 36,000 p.a. and from Y Ltd. is ₹ 45,000 p.a. Since the aggregate salary is more than ₹ 50,000 p.a. Mr.
Rohan will be treated as specified employee for both the employer i.e. X & Co. and Y Ltd.
¾¾ Even ‘DA not forming a part of salary for retirement benefit’ shall be included in salary, while determining
the above limit of ₹ 50,000 p.a.

Exempted Perquisites
Following perquisites are exempted in hands of employee :
1. Tea or snacks : Tea, similar non-alcoholic beverages and snacks provided during working hours.
2. Food : Food provided by employer in working place.
3. Recreational facilities : Recreational facilities extended to a group of employees.
4. Goods sold to employee at concessional rate : Goods manufactured by employer and sold by him to his
employees at concessional (not free) rates.

The Institute of Cost Accountants of India 95


Direct Taxation

5. Conveyance facility : Conveyance facility provided -


¾¾ to employees for journey between office and residence and vice versa.
¾¾ to the judges of High Court and Supreme Court
6. Training : Amount spent on training of employees including boarding & lodging expenses for such training.
7. Services rendered outside India : Any perquisite allowed outside India by the Government to a citizen of
India for rendering services outside India.
8. Contribution in some specified schemes
¾¾ Employer’s contribution to a pension or deferred annuity scheme.
¾¾ Employer’s contribution to staff group insurance scheme.
¾¾ Annual premium paid by the employer on personal accident policy affected by him in respect of his
employee.
9. *Loans
¾¾ Loan given at nil or at concessional rate of interest by the employer provided the aggregate amount of loan
does not exceed ₹ 20,000.
¾¾ Interest free loan for medical treatment of the diseases specified in Rule 3A.
10. *Medical facility : A provision of medical facility at office is exempt. Reimbursement of medical expenses for
treatment of Covid-19 is exempt
Note : However, medical allowance is fully taxable.
11. Periodicals and journals : Periodicals and journals required for discharge of work.
12. Telephone, mobile phones : Expenses for telephone, mobile phones actually incurred on behalf of employee
by the employer whether by way of direct payment or reimbursement.
13. *Free education facility : Free education facility to the children of employee in an institution owned or
maintained by the employer provided cost of such facility does not exceed ₹ 1,000 p.m. per child.
Note : Such facility is not restricted to two children as in case of Children Education allowance.
14. Computer or Laptop : Computer or Laptop provided whether to use at office or at home (provided ownership
is not transferred to the employee).
15. *Movable assets : Sale or gift of any movable asset (other than car and electronic items) to employee after
being used by the employer for 10 or more years.
16. *Leave Travel Concession : Leave Travel Concession (LTC) subject to few conditions.
17. Rent-free accommodation
¾¾ Rent-free official residence provided to a Judge of a High Court or the Supreme Court.
¾¾ Rent-free furnished residence (including maintenance thereof) to Official of Parliament, a Union Minister
or a Leader of opposition in Parliament.
18. *Accommodation : Accommodation provided -
¾¾ on transfer of an employee in a hotel for a period not exceeding 15 days in aggregate.

96 The Institute of Cost Accountants of India


Heads of Income

¾¾ in a remote area to an employee working at a mining site or an onshore exploration site or a project
execution site or a dam site or a power generation site or an offshore site.
19. Tax on non-monetary perquisite paid by employer on behalf of employee. With effect from A.Y. 2003-04 a
new sec. 10(10CC) has been inserted which provides that income tax paid by employer on behalf of employee
on income, being non-monetary perquisite, is not a taxable perquisite.
20. Health club, Sports club facility
* Discussed later in this chapter

2.1.16 Valuation of Accommodation


Valuation of Rent-free unfurnished accommodation (RFA) [Rule 3(1)]
Rent-free accommodation is taxable in the hands of all employees (except the Judges of High Court or Supreme
Court and Official of the Parliament or Union Minister and a leader of Opposition).
Accommodation here includes fixed as well as floating structure.
Fixed Structure A house, flat, farm house (or a part there of), accommodation in hotel, motel, service
apartment, a guest house, etc
Floating Structure A caravan, mobile home, ship etc.
For the purpose of valuation, employees are divided into two categories :
a. Employees of the Central or State Government or of any undertaking under the control of the Government;
b. Other employees
I) Central and State Government Employee (including military person)

Where the accommodation is provided


by the Central Government or any State
Government to the employees either
holding office or post in connection with
the affairs of the Union or of such State,
the value of perquisite in respect of such
accommodation is equal to the licence
fee, which would have been determined
by the Central or State Government in
accordance with the rules framed by the
Government.
{Academically, the taxable value of
the perquisite will be mentioned in the
problem}

Taxpoint : Employees of a local authority or a foreign government are not covered under this category.
II) Other Employees (residual category)
The value of perquisite is determined as per the following table:

The Institute of Cost Accountants of India 97


Direct Taxation

City in which accommodation is Accommodation is owned by the Accommodation is not owned by


provided employer the employer
Having population exceeding 25 15% of salary for the period during Rent paid or payable by the
lacs as per 2001 census which the employee occupied the employer or 15% of salary,
said accommodation. whichever is lower.
Having population exceeding 10 10% of salary for the period during
lacs but not exceeding 25 lacs as per which the employee occupied the
2001 census said accommodation.
Any other city 7.5% of salary for the period during
which the employee occupied the
said accommodation.
Notes :
a. Salary for the purpose of Rent free accommodation: Salary here means:
Basic + Dearness allowance/pay (if it forms a part of retirement benefit) + Bonus + Commission + Fees
+ All other taxable allowances (only taxable amount) + Any other monetary payment by whatever name
called (excluding perquisites and lump-sum payments received at the time of termination of service
or superannuation or voluntary retirement, like gratuity, severance pay leave encashment, voluntary
retrenchment benefits, commutation of pension and similar payments)
Taxpoint :
¾¾ Salary shall be determined on due basis.
¾¾ Where an assessee is receiving salary from two or more employers, the aggregate salary for the period
during which accommodation has been provided (by any of the employer) shall be considered.
¾¾ Monetary payments, which are not in the nature of perquisite, shall be considered. E.g. Leave encashment
received during the continuation of service shall be included in salary for this purpose. However, if such
pay leave is received at the time of retirement, then such receipt shall not be considered.
¾¾ Here salary does not include employer’s contribution to Provident Fund of the employee.
b. Exemption of 90 days in case of allotment of two houses: Where an employee is transferred from one place
to another and he is provided with an accommodation at new place also, the value of perquisite shall be taken
for only one such house having lower value for a period not exceeding 90 days. Thereafter, the values of both
such houses are taxable.
c. Any accommodation provided to an employee working at a mining site; or an on-shore oil exploration site; or
a project execution site; or a dam site; or a power generation site; or an off-shore site, which
a) being of a temporary nature and having plinth area not exceeding 800 sq.ft. is located not less than 8 kms
away from the local limits of any municipality or a cantonment board; or
b) is located in a remote area.
d. Remote area here means an area located at least 40 K.M. away from a town having population not
exceeding 20,000 as per latest published census.
e. Where the accommodation is provided by the Central Government or any State Government to an employee
who is serving on deputation with any body or undertaking under the control of such Government:
i. the employer of such an employee shall be deemed to be that body or undertaking where the employee
is serving on deputation; and

98 The Institute of Cost Accountants of India


Heads of Income

ii. the value of perquisite of such an accommodation shall be the amount calculated as per residual
category (II) considering as if the accommodation is owned by the employer.
Illustration 15 :
Mr. Chauhan has the following salary structure :
a) Basic Salary ₹ 5,000 p.m. b) Entertainment Allowance ₹ 1,000 p.m.
c) Education Allowance ₹ 500 p.m. (he has 3 children) d) DA ₹ 3,000 p.m.
e) Fees ₹ 5,000 p.a. f) Bonus ₹ 10,000 p.a.
g) Professional tax of employee paid by employer ₹ 2,000 for the year
h) He has been provided a rent-free accommodation in Mumbai.
i) 60% of DA only forms part of retirement benefits
Compute taxable value of accommodation in the hands of Mr. Chauhan in the following cases :
I) The employer owns such accommodation.
II) The employer hires such accommodation at a monthly rent of ₹ 900.
Solution :
Taxable value of rent-free accommodation for the A.Y. 2024-25 :

Particulars Basis of determination Taxable


Perquisite
i) Owned by employer 15% of Salary (Working) ₹ 16,830
ii) Hired by employer 15% of Salary or Actual rent paid by employer, whichever is lower ₹ 10,800
Working : Salary for the purpose of Rent-free accommodation :

Particulars Details Amount Amount


Basic Salary 60,000
Bonus 10,000
Fees 5,000
Allowances
Dearness allowance ₹ 36,000 x 60% 21,600
Entertainment Allowance 12,000
Education Allowance ₹ 6,000 – ₹ 2,400 3,600 37,200
Gross Taxable Salary 1,12,200
Note : Professional tax paid on behalf of employee is a perquisite; hence the same shall not be included in salary
for the aforesaid purpose.

Illustration 16 :
In above illustration, how shall answer differ if the property is situated in a city where population is only 14,60,000.
Solution :
Taxable value of rent free accommodation for the A.Y.2024-25 :

The Institute of Cost Accountants of India 99


Direct Taxation

Particulars Basis of determination Taxable value of Perquisite


Owned by employer 10% of Salary (as per the above working) ₹ 11,220
Hired by employer 15% of Salary or Actual rent paid by ₹ 10,800
employer, whichever is lower

Illustration 17 :
Miss Stuti has the following salary structure :

a) Basic salary 15,000 p.m.


b) Dearness Allowance 5,000 p.m. (not forming part of retirement benefit)
c) Hostel Allowance 1,000 p.m. (does not have any child)
d) Tiffin Allowance 500 p.m.
e) Transport Allowance 200 p.m.
f) Bonus 20,000 p.a.
g) Commission 15,000 p.a.
h) Free refreshment in office worth 5,000 p.a.
i) Mobile phone facility by employer 900 p.m.
j) Computer facility worth 10,000 p.a.
She has been provided a Rent-free Accommodation (owned by employer) in Kolkata. The house was allotted to
her with effect from 1/5/2023 but she could occupy the same only from 1/6/2023. Find her gross taxable salary.
Solution :
Computation of gross taxable salary of Miss Stuti for the A.Y. 2024-25 :

Particulars Details Amount Amount


Basic Salary 1,80,000
Bonus 20,000
Commission 15,000
Allowances :
Dearness Allowance 60,000
Hostel Allowance (Fully taxable as she has no child) 12,000
Tiffin Allowance 6,000
Transport Allowance 2,400 80,400
Perquisite u/s 17(2) :
Free Refreshment (not taxable) Nil
Mobile or telephone facility Nil
Computer facility Nil
Rent Free Accommodation Working 29,425 29,425
Gross Salary 3,24,825

100 The Institute of Cost Accountants of India


Heads of Income

Working : Salary for the purpose of rent-free accommodation :

Basic Salary 1,80,000


Bonus 20,000
Commission 15,000
Allowances
Dearness allowance Nil
Hostel Allowance 12,000
Tiffin Allowance 6,000
Transport Allowance 2,400
Total 2,35,400
Value of Rent-Free Accommodation (being 15% x ₹ 2,35,400 x 10/12) 29,425

Illustration 18 :

Miss Khushi has the following salary details :

i) Basic salary ₹ 6,000 p.m.

ii) DA ₹ 3,000 p.m.

iii) Academic development allowance ₹ 1,000 p.m., expenditure incurred ₹ 700 p.m.

iv) Entertainment allowance ₹ 500 p.m.

She has been provided with a rent-free accommodation in Purulia. On 1/7/2023, she was posted to Kolkata. A new
house further allotted to her on same date. But she surrendered her Purulia house only on 31/12/2023. Rent paid
by employer for Purulia House ₹ 500 p.m. while Kolkata house is owned by the employer. Find her gross taxable
salary.

Solution :

Computation of gross taxable salary of Miss Khushi for the A.Y. 2024-25 :

Particulars Details Amount Amount


Basic Salary 72,000
Allowances:
Dearness Allowance 36,000
Academic Development Allowance 12,000
Less : Exempted to the extent of actual expenditure 8,400 3,600 45,600
Entertainment Allowance 6,000
Perquisite u/s 17(2):
Rent Free Accommodation Working 13,320
Gross Taxable Salary 1,30,920
Working : Since Miss Khushi has been transferred from Purulia to Kolkata and she is provided with an
accommodation at Kolkata also, the value of perquisite shall be taken for only one such house having lower value
for a period not exceeding 90 days. Thereafter, the value of both such houses is taxable.

The Institute of Cost Accountants of India 101


Direct Taxation

Valuation of rent-free accommodation :

Period Particulars Purulia house Kolkata house Taxable Amount


1/4/2023 - She is having only Purulia 15% of (₹ 1,17,600 x 3/12) or Not applicable 1,500
30/6/2023 house Rent paid by employer (₹ 500
x 3) whichever is lower
1/7/2023 - She has both house but the ₹ 1,500 (as calculated above) 15% of Salary1 1,500
30/9/2023 house having lower value
i.e. ₹ 4,410
shall be taxable
1/10/2023 - She has both house and ₹ 1,500 (as calculated above) ₹ 4,410 5,910
31/12/2023 both shall be taxable
(as above) (Note b)
1/1/2024 - She has only Kolkata Not applicable ₹ 4,410 4,410
31/3/2024 house
(as above)
Taxable perquisite 13,320
Note :
a. For the sake of simplicity, 3 months have been taken as equivalent to 90 days.
b. After 90 days, value of both houses shall be considered.
1
. Salary for valuation of rent- free accommodation :
Basic Salary 72,000
Allowances
Dearness allowance 36,000
Entertainment allowance 6,000
Academic development Allowance 3,600
Total 1,17,600
Valuation of Rent-free furnished accommodation
Furnished accommodation means Accommodation + Furniture.
Value of Furnished accommodation = Value of accommodation + Value of furniture
Valuation of Accommodation: As discussed above.
Valuation of Furniture : As per the following table
Case Taxable value
Furniture owned by the employer 10% of original cost of furniture
Furniture hired by the employer Actual hire charges paid/payable by the employer
Notes :
1. “Furniture” here, includes refrigerator, television, radio, air-conditioner and other household appliances, etc.
2. The above rule is applicable to Government as well as Non-Government Employees.
Illustration 19 :
Sri Ashutosh has been provided with a furnished accommodation in a city having population of 14,00,000 as per
last census. Municipal Value of the house (owned by employer) is ₹ 80,000 whereas Fair rent of the house is ₹
1,00,000. His salary details are as under :

102 The Institute of Cost Accountants of India


Heads of Income

Basic 25,000 p.m.


Allowance for increased cost of living 5,000 p.m.
Children Education allowance 3,000 p.m. [He has one son and two married daughters]
Furniture details as under :
Particulars Hired by the employer Owned by the employer
(Hire charge) (Original Cost)
T.V. 2,000 p.a. -
Refrigerator - 10,000
Washing Machine - 5,000
Other furniture 1,000 p.m. 20,000
Calculate gross taxable salary of Sri Ashutosh for the A.Y. 2024-25.
Solution :
Computation of gross taxable salary of Sri Ashutosh for the A.Y. 2024-25 :
Particulars Amount Amount
Basic Salary 3,00,000
Dearness allowance (Allowance for increased cost of living) 60,000
Children Education Allowance 36,000
Less : Exemption (₹ 100 x 2 x 12) 2,400 33,600
Rent Free Furnished Accommodation
Value of Accommodation (10% of Salary1) 39,360
Value of furniture2 17,500 56,860
Gross Taxable Salary 4,50,460
1
Salary for valuation of rent- free accommodation :
Basic Salary 3,00,000
Dearness allowance 60,000
Education Allowance 33,600
Total 3,93,600
2Valuation of taxable perquisite for furniture :
Furniture Perquisite for hired Perquisite for owned Total Taxable value of
furniture furniture furniture
T.V. 2,000 - 2,000
Refrigerator - 10% of 10,000 1,000
Washing Machine - 10% of 5,000 500
Other furniture 12,000 10% of 20,000 14,000
Total 17,500
Municipal value and Fair rent are irrelevant.
The Institute of Cost Accountants of India 103
Direct Taxation

Test Yourself
1. Mr. Raja is the employee of an Indian company. He has been provided a rent-free accommodation in Mumbai
on 1/4/2023 for which the employer is to pay a monthly rent of ₹ 3,000. During the year, employer paid rent
of ₹ 30,000 (2 months rent is outstanding). On 1/9/2023, furniture of ₹ 4,00,000 owned by the employer is
also provided. His Basic salary is ₹ 10,000 p.m. and DA is ₹ 5,000 p.m. Find taxable value of perquisite.
Hints :
1. ₹ 50,333;
Valuation of accommodation provided at concessional rent

Valuation will be made as if the rent-free accommodation is provided and the amount so computed will be reduced
by the rent payable by the employee.

Value of Rent free accommodation as usual *****


Less : Rent payable by employee to employer for the above facility ****
Taxable value of perquisite ****

Taxpoint : The above rule of valuation shall be applicable in case of the Government employee also.

Test Yourself
1. Mr. Saket has been provided an accommodation in Patna (owned) with furniture (hire charges paid by
employer ₹ 10,000). His salary details are as under :
¾¾ Basic ₹ 10,000 p.m.
¾¾ D.A. ₹ 2,000 p.m.
¾¾ Transport allowance ₹ 3,000 p.m.
¾¾ Bonus ₹ 10,000 p.a.
¾¾ Fee ₹ 5,000 p.a.
¾¾ Rent paid by employee is ₹ 1,000 p.m. for such house.
Find taxable value of perquisite.
Hints
1. ₹ 27,250;

Accommodation provided in a hotel

In this case, value of perquisite shall be minimum of the following :

a. 24% of salary for the period such accommodation is provided; or

b. Actual charges paid or payable to such hotel.

However, if the following conditions are satisfied then nothing is taxable -

 Such accommodation is provided for a period not exceeding in aggregate 15 days; and

 Such accommodation is provided on transfer of employee from one place to another place.

104 The Institute of Cost Accountants of India


Heads of Income

Note : If the employee pays any rent, the value so determined shall be reduced by the rent actually paid or payable
by the employee

Taxpoint :
 Salary here has the same meaning as in the case of rent-free accommodation.
 Above rule shall be applicable whether the assessee is a Government or a Non-Government employee.
 If the facility is provided for more than 15 days, then the perquisite is exempt for first 15 days and thereafter
taxable. E.g. if facility has been provided for 45 days then taxable perquisite shall be only for last 30 days.
 Hotel includes licensed accommodation in the nature of motel, service apartment or guest house.

2.1.17 Insurance premium payable by employer


As per sec. 17(2)(v), following sums payable by an employer shall be taxable perquisite in the hands of all employees,
whether it is paid directly or through a fund (other than recognised provident fund or approved superannuation fund
or deposit-linked insurance fund),
 to effect an assurance on the life of the assessee; or
 to effect a contract for an annuity
Note : Employee can claim deduction u/s 80C for LIC premium paid by employer
 Sweat equity shares means equity shares issued by a company to its employees or directors at a discount or for
consideration other than cash for providing know-how or making available rights in the nature of intellectual
property rights or value additions, by whatever name called.
Taxpoint : If such shares are allotted or transferred not for above reasons (i.e, for providing know-how,
etc.), then it is not taxable as perquisite. E.g., if such option is granted to the employee against acquisition
of immovable property by the company, then such benefit shall not be considered as perquisite. However,
employee is liable to pay tax, if any, under the head ‘Capital Gain’
Perquisites :
Value of any specified security or sweat equity shares shall be considered as perquisites in hands of employee if
the following conditions are satisfied :
a. Such security or sweat equity shares are allotted or transferred on or after 01-04-2009
b. Such security or sweat equity shares are allotted or transferred by the employer (former or present) directly or
indirectly.
c. Such security or sweat equity shares are allotted or transferred free of cost or at concessional rate to the
assessee.
Valuation :
Value of such perquisite shall be computed as under :

Particulars Amount
The fair market value of the specified security or sweat equity shares, as the case may be, on the ***
date on which the option is exercised by the assessee

The Institute of Cost Accountants of India 105


Direct Taxation

Less : The amount actually paid by, or recovered from the assessee in respect of such security or ***
shares
Value of perquisite ***
Notes : Option means a right but not an obligation granted to an employee to apply for the specified security or
sweat equity shares at a predetermined price.

2.1.18 Valuation of perquisites in respect of Motor Car [Rule 3(2)]


Motor-car facility provided by an employer is taxable in the hands of employee on the following basis :

Car is Car is Used by Who is


Taxable value
owned by Maintained by employee for Chargeable
Office purpose Not a perquisite Not applicable
Employer Personal M1+D2
Specified
purpose
Employee
Both purpose ₹ 1800 or ₹ 2400 p.m.3
Office purpose Not a perquisite Not applicable
Employer Employee Personal D
Specified
purpose
employee
Both purpose ₹ 600 / ₹ 900 p.m.4
Office purpose Not a perquisite Not applicable
Personal M All employee
purpose
Both purpose Actual expenditure incurred by the employer
as reduced by ₹ 1800 / ₹ 2400 p.m.3 (further
deduction of ₹ 900 p.m. for driver) or a
higher deduction if prescribed conditions are
Employee Employer satisfied5
Employee Any purpose Not a perquisite Not applicable
1 M =Maintenance cost
2 D = Depreciation @ 10% of actual cost of the car. However, if the car is not owned by employer then actual hire
charge incurred by employer shall be considered.
3 ₹ 2400 p.m. in case of higher capacity car# and ₹ 1800 p.m. for lower capacity car.
4 ₹ 900 p.m. in case of higher capacity car# and ₹ 600 p.m. for lower capacity car.
#
Higher capacity car means a car whose cubic capacity of engine exceeds 1.6 litres.
5 Conditions to be fulfilled for claiming higher deductions :
 The employer has maintained complete details of journey undertaken for official purpose, which may include
date of journey, destination, mileage, and the amount of expenditure incurred thereon; and
 The employer gives the certificate to the effect that the expenditure was incurred wholly and exclusively for
the performance of official duties.

106 The Institute of Cost Accountants of India


Heads of Income

Chauffeur / Driver
If chauffeur is also provided, then salary of chauffeur is further to be added to the value of perquisite (as computed
above). However, if car is used for both i.e. official and personal purpose then ₹ 900 p.m. (irrespective of higher or
lower capacity of car) is to be taken as value of chauffeur perquisite.
Notes :
a. If motor car is provided at a concessional rate then charges paid by employee for such car, shall be reduced
from the value of perquisite.
b. The word “month” denotes completed month. Any part of the month shall be ignored.
c. When more than one car is provided to the employee, otherwise than wholly and exclusively for office purpose,
the value of perquisite for -
¾¾ One car shall be taken as car is provided partly for office and partly for private purpose i.e. ₹ 1,800 or ₹
2,400 p.m. (plus ₹ 900 p.m. for chauffeur, if provided); and
¾¾ For other car(s), value shall be calculated as car(s) are provided exclusively for private purpose.
d. Conveyance facility to the judges of High Court or Supreme Court is not taxable.
e. Use of any vehicle provided to an employee for journey from residence to work place or vice versa is not a
taxable perquisite.

Illustration 20:
Sonam, has been provided a car (1.7 ltr.) by his employer Vikash Ltd. The cost of car to the employer was ₹3,50,000
and maintenance cost incurred by the employer ₹ 30,000 p.a. Chauffeur salary paid by the employer ₹3,000 p.m.
Find value of perquisite for Sonam for the A.Y. 2024-25, if the car is used for:
a) Office purpose. b) Personal purpose. c) Both purposes.
In case (b) and (c), employee is being charged ₹ 15,000 p.a. for such facility.
Solution :
a. Nil, as car is used for office purpose.
b. Taxable value of car facility :

Particulars Details Amount


Depreciation of Car 10% of ₹ 3,50,000 35,000
Maintenance cost Actual 30,000
Driver’s salary Actual 36,000
Total 1,01,000
Less : Amount charged from employee 15,000
Taxable Perquisite 86,000
c. ₹ 2,400 p.m. for car facility + ₹ 900 p.m. for driver facility = ₹ 3,300 p.m.
Taxable value of perquisite ₹ 3,300 × 12 = ₹ 39,600.
Note : Whenever statutory value (₹ 1,800 or ₹ 2,400 and ₹ 600 or ₹ 900) is taken as taxable value of perquisite
then amount charged from employee shall not be subtracted.

The Institute of Cost Accountants of India 107


Direct Taxation

Illustration 21 :
Mr. Piyush has been provided a car (1.5 ltr.) on 15/7/2023. The cost of car to the employer was ₹ 6,00,000 and
maintenance cost incurred by employer ₹ 20,000 p.a. Chauffeur salary paid by employer (Mr. Ratan) ₹ 4,000 p.m.
The car is 40% used for office and 60% for personal purpose. Charges paid by employee for such facility ₹ 5,000
p.a. Find taxable value of perquisite.
Solution :
Taxable value of perquisite :

Particulars Details Amount


Car ₹ 1,800 × 8 14,400
Driver ₹ 900 × 8 7,200
Taxable Perquisite 21,600
1. A part of month shall not be considered for this purpose.
2. Whenever statutory value is taken as taxable value of perquisite then amount charged from employee shall not
be subtracted.

Illustration 22 :
Mr. Vikram being a Government employee has a car (1.7 ltr.) used for office as well as for personal purpose. During
the year, he incurred ₹ 40,000 on maintenance and ₹ 20,000 on driver’s salary. The entire cost is reimbursed by
employer. Find taxable perquisite.
Solution :
Taxable perquisite in the hands of Mr. Vikram :
As the car is owned by the assessee & maintained by the employer, taxable value of perquisite shall be -
Actual expenditure incurred by the employer as reduced by ₹ 2,400 p.m. (in case of 1.7 ltr.) and ₹ 900 p.m. for
driver’s salary. Hence, taxable amount shall be -

Amount reimbursed by employer (₹ 40,000 + ₹ 20,000) ₹ 60,000


Less: Deduction for the amount used for office purpose (₹ 2,400 + ₹ 900) × 12 ₹ 39,600
Taxable amount ₹ 20,400

Illustration 23 :
Wasim has a car (1.5 ltr.) used for office as well as for personal purpose. During the year car is used 80% for
business purpose being certified by the employer. During the year, he incurred ₹ 50,000 on maintenance and
running of such car. The entire cost is reimbursed by the employer. Find taxable perquisite if assessee wish to claim
higher deduction, when – (a) A proper log book is maintained; (b) A proper log book is not maintained
Solution :
a. When log book is maintained
Taxable perquisite in the hands of Wasim

108 The Institute of Cost Accountants of India


Heads of Income

Actual expenditure incurred by the employer is reduced to the extent it is used for office purpose, as a proper
record is kept and duly certified by employer.

Amount reimbursed by the employer ₹ 50,000


Less : Deduction (80% of ₹ 50,000) ₹ 40,000
Taxable amount ₹ 10,000
b. When log book is not maintained
Taxable perquisite in the hands of Wasim
Actual expenditure incurred by the employer is reduced to the extent of ₹ 1,800 p.m. even though it is used for
office purpose but a proper record is not kept.

Amount reimbursed by the employer ₹ 50,000


Less : Deduction (₹ 1,800 x 12) ₹ 21,600
Taxable amount ₹ 28,400

Illustration 24:
Amit is provided with two cars, to be used official & personal work, by his employer Raj. The following information
is available from the employer records for computing taxable value of perk (assuming car 1, is exclusively used
by Amit).

Particulars Car 1 Car 2


Cost of the car 6,00,000 4,00,000
Running and maintenance (borne by the company) 40,800 28,000
Salary of driver (borne by the company) 24,000 24,000
Solution :
Valuation of perquisite for Mr. Amit :

Particulars Workings Details Amount


Valuation of perquisite in respect of Car 1
- Depreciation of car 10% of ₹ 6,00,000 60,000
- Maintenance 40,800
- Driver salary 24,000 1,24,800
Valuation of perquisite in respect of Car 2 (₹ 1,800 + ₹ 900) × 12 32,400
(assumed capacity of engine does not exceed 1.6 cc)
Value of car perquisite 1,57,200

Illustration 25 :
Mr. Vijay, manager, has been provided the following car facilities by Kishan Ltd. (his employer) :

Particulars Car A Car B Car C


Owned by Employer Employer Employer
Used for Office as well as personal purpose Personal purpose
Cost of car 3,00,000 5,00,000 2,00,000

The Institute of Cost Accountants of India 109


Direct Taxation

Particulars Car A Car B Car C


Maintenance expenditure incurred by employer 50,000 60,000 -
Maintenance expenditure incurred by employee - - 40,000
Capacity of car 1.8 ltr. 1.4 ltr. 1.6 ltr.

Find taxable value of car facility.

Case a) Mr. Vijay holds 17% of equity share capital and 30% of preference share capital of Kishan Ltd. and his wife
holds 13% equity share capital of the same company. Assume his total salary during the year other than perquisite
is ₹ 40,000;

Case b) Mr. Vijay holds 25% equity share capital of the employer company.

Solution :

Case a) Since Mr. Vijay is not a specified employee & employer owns all cars therefore car facility shall not be
taxable.

Case b) Since Mr. Vijay holds substantial interest in employer-company hence he is a specified employee.

As employee has been provided 2 cars, used for office as well as for personal purpose, therefore he will have to opt
one car as for ‘office as well as personal purpose’ & the other car for personal purpose. In the given case, assessee
has two options -

Option 1) Car A is used for office as well as personal purpose and car B is used for personal purpose.

Option 2) Car A is used for personal purpose and car B is used for office as well as personal purpose.

In any case, Car C is used for Personal purpose.

Option 1 Option 2
Particulars Workings
Car A Car B Car C Car A Car B Car C
Car used for Both Personal Personal Personal Both Personal
₹ 2,400 x 12 28,800
10% of ₹ 5,00,000 + ₹ 60,000 1,10,000
10% of ₹ 2,00,000 20,000 20,000
10% of ₹ 3,00,000 + ₹ 50,000 80,000
₹ 1,800 x 12 21,600
Total 1,58,800 1,21,600
As option 2 has lesser taxable value, hence assessee will opt for option 2 & taxable value shall be ₹ 1,21,600.

2.1.19 Valuation of Perquisite in respect of Vehicle other than Motor Car


The facility provided by employer is taxable in the hands of employee on the following basis :

110 The Institute of Cost Accountants of India


Heads of Income

Who is
Owned by Maintained by Used for Taxable Value of perquisite
Chargeable
Not
Office purpose Nil
Applicable
Actual Maintenance + Depreciation @ 10% Specified
Personal purpose
of Original cost employee

Employer Reasonable proportion of (Maintenance +


Both purpose
Depreciation @ 10% of Original cost)
Office purpose Nil Not Applicable
Personal purpose Actual Maintenance
Employee Employee Actual expenditure incurred by the employer
Both as reduced by ₹ 900 p.m. or as reduced by
purpose higher sum if prescribed conditions (as
discussed in case of Car facility) are satisfied. All employee

2.1.20 Valuation of perquisite in respect of Free Domestic Servants [Rule 3(3)]


Value of perquisite is determined as under :

Servant appointed by Taxable value of perquisite Taxable in hands of


Employer Actual cost to the employer is Specified employee
Employee taxable as perquisite All employee
Notes :
a. If rent-free accommodation (owned by the employer) is provided with gardener then gardener’s salary and
maintenance cost of garden shall not be taxable. [Circular No.122 dated 19/101973]
b. Any amount charged from the employee for such facility shall be reduced from above value.
c. Domestic servant allowance given to employee is fully taxable.
d. Reimbursement of servant-salary by the employer shall be taxable in hands of all employee.
Illustration 26:
Sri Bhagawan, has been provided with the following servants by his employer :
Servant Appointed by Salary of Servant
Watchman Employer 2,000 p.m.
Cook Employee’s wife 3,000 p.m.
Maid servant Employer 1,000 p.m.
Sweeper Employee 500 p.m.
Gardener Employer 1,000 p.m.
Sri Bhagawan has also been provided a rent-free accommodation, which is owned by the employer. Find taxable
value of servant facility if - Case a) He is a specified employee. Case b) He is a non-specified employee.

The Institute of Cost Accountants of India 111


Direct Taxation

Solution :

Computation of taxable value of perquisite for A.Y. 2024-25 :

Taxable Amount
Servant
Case a Case a
Watchman 24,000 Nil
Cook 36,000 36,000
Maid servant 12,000 Nil
Sweeper 6,000 6,000
Gardener (since Rent free accommodation, owned by employer, is provided) Nil Nil
Taxable Perquisite 78,000 42,000

2.1.21 Gas, electricity or water facility [Rule 3(4)]


It is taxable on the following basis :
Taxable value of perquisite
Case Facility is provided Facility is provided Taxable in the hands of
from own sources from other agency
Facility is in name of employee Manufacturing cost to Prices paid to such All employees
Facility is in name of employer the employer agency Specified employees

Note : Where the employee is paying any amount for such facility, the amount so paid by employee shall be
reduced from the value determined above.

2.1.22 Valuation of perquisite in respect of free education [Rule 3(5)]


Taxable value of perquisite is as follows :

Case Taxable Value


Facility provided to employee Not taxable
Facility provided to family member
Facility provided in an institution owned by the Child of the assessee : Cost of such education in similar
employer institution subject to an exemption of ₹ 1,000 p.m. per child
Facility provided in any institution (not owned by shall be taxable*.
the employer) by reason of his being in employment.
Other family member : Cost of such education in similar
institution shall be taxable.
Reimbursement of education expenditure to Actual reimbursement shall be taxable. Such reimbursement
employee. of tuition fee shall also be taxable in the hands of Central
Government employee. (Circular letter No 35/7/65–IT(B) dt
12/2/1965)

* However, Hon’ble Punjab & Haryana High Court in the case of CIT –vs.- Director, Delhi Public School (2011) 202 Taxman 318 has held that if value
of perquisite exceeds ₹ 1,000/-, then entire amount shall be taxable.

112 The Institute of Cost Accountants of India


Heads of Income

Who is chargeable?

Case Taxability in the hands of


In case of reimbursement; or
School fee of family member of the employee paid by the employer directly to All employee
school
In any other case Specified employee

Notes :

a. ₹ 1,000 per month


per child shall be
exempted without
any restriction on
number of children.
b. Child includes
adopted child,
stepchild of the
assessee, but
does not include
grandchild or
illegitimate child.

c. Any amount charged from the employee for such facility shall be reduced from the above value.
d. Contribution made under an Educational Trust, created for the children of particular group of employees, is not
taxable.

2.1.23 Valuation of perquisite in respect of Free Transport [Rule 3(6)]


The facility provided by employer is taxable in the hands of employee on the following basis :

Case Treatment
If employer is engaged in transportation business. Amount charged from public for such facility is taxable
in the hands of specified employee.
In any other case Actual cost of employer for such facility is taxable in
the hands of all employees.
Notes :
a. In case above facility is provided to employees of Railways & Airlines, nothing shall be chargeable to tax.
b. Any amount charged from the employee for such facility shall be reduced from the above value.
c. Conveyance facility provided to the employee for journey between office and residence is not taxable.

The Institute of Cost Accountants of India 113


Direct Taxation

2.1.24 Valuation of perquisite in respect of interest free loan or concessional rate of interest
[Rule 3(7)(i)
Perquisite in respect of interest free loan or loan
at concessional rate of interest to th]e employee
or any member of his household by the employer
or any person on his behalf, is not taxable if
aggregate amount of loan given by the employer
(or any other person on his behalf) does not
exceed ₹ 20,000. The taxable value of such
perquisite shall be determined as per the rate
as on the 1st day of the relevant previous year
charged by the State Bank of India in respect of
loans for the same purpose advanced by it.
Notes :
a. Maximum outstanding monthly balance: Interest is calculated on the maximum outstanding monthly
balance. Maximum outstanding monthly balance means the aggregate outstanding balance for each loan as on
the last day of each month.
b. Loan for medical treatment: Nothing is taxable if loan is given for medical treatment of the employee or any
member of his household in respect of diseases specified in rule 3A. However, such exempted loan will not
include the amount that has been reimbursed by an insurance company under any medical insurance scheme.
c. Concessional interest: Any interest paid by the employee to the employer for such loan shall be reduced from
the above computed value. If rate of interest charged by the employer is higher than the above rate, nothing is
taxable as perquisite.
d. Amount on which interest shall be calculated: If loan amount is more than ₹ 20,000, interest shall be levied
on total loan amount, rather than the excess amount.
e. Treatment of outstanding loan taken earlier: Interest on loan, taken before insertion of this provision, shall
also be treated as taxable perquisite. [Circular No.15/2001dated 12/12/2001]

2.1.25 Travelling / Touring / Holiday Home expenditure on Holiday [Rule 3(7)(ii)]


Valuation of perquisite in respect of travelling, touring, holiday home or any other expenses paid for or borne or
reimbursed by the employer for any holiday availed of by the employee or any member of his household is taxable
in the hands of all employees as per the following table :

Case Taxable value of perquisite


Where such facility is maintained by employer and is Notional cost of such facility. In other words, value at
not available uniformly to all employee which such facilities are offered by other agencies to
the public.
Where the employee is on official tour and the expenses The amount of expenditure so incurred for the
are incurred in respect of any member of his household accompanying member of his household.
accompanying him
Where any official tour is extended as a vacation The value will be limited to the expenses incurred in
relation to such extended period of stay or vacation.
In any other case Amount incurred by the employer.

114 The Institute of Cost Accountants of India


Heads of Income

Notes :
a. Any amount charged from employee shall be reduced from the above determined value.
b. The above provisions are not applicable in case of Leave Travel Concession (discussed earlier)

2.1.26 Valuation of perquisite in respect of free meals [Rule 3(7)(iii)]


The facility provided by employer is taxable in the hands of employee on the following basis :

Case Tax Treatment


Tea, snacks or other non-alcoholic beverages in
the form of light refreshment provided during
Nil
office hours (including over-time)
Free meals provided during office hours in:
¾¾ Remote area; or Nil
¾¾ An offshore installation
Free meals provided by the employer during Expenditure on free meals in excess of ₹ 50 per meal shall
office hours: be taxable perquisite to the extent of excess amount in hands
of all employees.
¾¾ At office or business premises; or
E.g. Free meal given to employee worth ₹ 70 per meal
¾¾ Through paid vouchers which are not
through non-transferable coupon for 300 times in a year.
transferable and usable only at eating joints.
Taxable perquisite in such case shall be ₹ 6,000 {being ₹
(70 – 50) x 300}.
The actual expenditure incurred by employer as reduced
by amount charged from employee for such lunch or meal
In any other case
shall be taxable in the hands of all employees. i.e. [Actual
expenditure to employer – Amount charged from employee]

2.1.27 Gift, voucher or token given by employer [Rule 3(7)(iv)]


The value of any gift, voucher, or token (in lieu of which any gift may be received) given to the employee (or
any member of his household) on ceremonial occasion or otherwise by the employer shall be taxable in the hands
of all employees. However, gift, voucher or token upto ₹ 5,000, in aggregate, during the previous year, shall be
exempted.
Notes :
a. Where worth of gift is in excess of ₹ 5,000 then entire amount shall be taxable.
b. No such exemption (₹ 5,000) is available on gift made in cash or convertible into money.
Illustration 27 :
Determine taxable perquisite in the following cases :
1. Miss Shradha received a wrist-watch of ₹ 3,000 on 17/7/2023 and a golden chain worth ₹ 12,000 on 18/8/2023
from her employer, Mr. Raju.
2. Miss Rakhi received ₹ 11,000 cash–gift from her employer, Dipu Ltd.
3. Mr. Anirudha is working with X & Co. a partnership firm. During the year, the employer firm gifted a diamond
ring worth ₹ 80,000 to wife of Mr. Anirudha.

The Institute of Cost Accountants of India 115


Direct Taxation

Solution :
1. Taxable perquisite in the hands of Shradha shall be ₹ 15,000 (being ₹ 3,000 + ₹ 12,000)
2. Taxable perquisite in the hands of Rakhi shall be ₹ 11,000.
3. Taxable perquisite in the hands of Mr. Anirudha shall be ₹ 80,000.

2.1.28 Credit Card [Rule 3(7)(v)]


Expenditure incurred by an employer in respect of credit card facility to employee shall be treated as under :

Case Tax Treatment


Where such credit card is used wholly and exclusively Nil
for office purpose and specified conditions# are satisfied.

Where expenses (including membership and annual If directly paid by the employer
fees) are incurred by the employee or any member
Any amount incurred by the employer as reduced by
of his household, which is charged to a credit card
amount charged from the employee shall be taxable in
(including any add-on card) provided by the employer
the hands of all employees
or otherwise, are paid or reimbursed by the employer.
If amount reimbursed by the employer
Any amount reimbursed by the employer shall be
taxable in the hands of all employees.
#
Specified conditions to be fulfilled to claim that expenses have been incurred wholly and exclusively for office
purpose :
a. Complete details in respect of such expenditure are maintained by the employer which may, inter-alia, include
the date of expenditure and the nature of expenditure; and
b. The employer gives a certificate for such expenditure to the effect that the same was incurred wholly and
exclusively for the performance of official duty.

2.1.29 Club Expenditure [Rule 3(7)(vi)]


Expenditure incurred by employer in respect of club facility to employee shall be treated as under :

Case Tax Treatment


Where such expenses are incurred wholly and
exclusively for office purpose and specified conditions#
Nil
are satisfied.

Where health club, sports and similar facilities are Nil


provided uniformly to all employees by the employer.
Where the employer has obtained corporate Amount incurred by employer for such facility shall
membership of the club and the facility is enjoyed by be taxable perquisite in the hands of all employees.
the employee or any member of his household However, initial fees paid for obtaining corporate
membership shall not be a taxable perquisite.

116 The Institute of Cost Accountants of India


Heads of Income

Case Tax Treatment


Any payment or reimbursement by the employer of any If directly paid by the employer
expenditure incurred (including the amount of annual
Any amount incurred by the employer as reduced by
or periodical fee) in a club by employee or any member
amount charged from the employee shall be taxable in
of his household
the hands of all employees.
If amount reimbursed by the employer
Any amount reimbursed by the employer shall be
taxable in the hands of all employees.
#
Specified conditions to be fulfilled to claim that expenses have been incurred wholly and exclusively for office
purpose :
a. Complete details in respect of such expenditure is maintained by the employer which may, inter alia, include
the date of expenditure, the nature of expenditure and its business expediency; and
b. The employer gives a certificate for such expenditure to the effect that the same was incurred wholly and
exclusively for the performance of official duty;

2.1.30 Valuation of perquisite in respect of use of movable assets [Rule 3(7)(vii)]


If employee (or any member of his household) uses any movable asset (other than the assets for which provisions
have been made) belonging to employer, then such facility is taxable in the hands of all employees. The value of
such benefit is determined as per the following table :

If the asset is owned by the employer 10% of the original cost of such asset.
If the asset is hired by the employer Charges paid or payable by the employer
Notes :
a. Any sum charged from the employee shall be reduced from the value determined as above.
b. Use of computer, laptop, etc. (as discussed earlier) is exempted perquisite.
c. Here movable asset does not include car.

2.1.31 Valuation of the perquisite in respect of movable assets sold by an employer [Rule
3(7)(viii)]
If the sale price is less than
the written down value
(calculated as per method and
rate mentioned below) then the
difference would be treated as
perquisite and taxable in the
hands of all employees.
Rates and methods of
depreciation for different types
of assets are as follow :

The Institute of Cost Accountants of India 117


Direct Taxation

Types of asset Rate of depreciation Method of depreciation


Electronic items#/Computer 50% Reducing balance
Motor car 20% Reducing balance
Any other 10% Straight line
#
Electronic items here means data storage and handling devices like computer, digital diaries and printers. They do
not include household appliances like washing machines, microwave ovens, mixers, etc.
Mathematically, taxable perquisite is as under :

Original cost to the employer *****


Less : Accumulated depreciation for each completed year during which such asset is used by the ****
employer
Written down value ****
Less : Amount charged from employee ****
Value of Perquisite (if positive) *****
Taxpoint : No depreciation shall be charged for a part of the year.

Illustration 28:
X Ltd. has sold the following assets to its employee, Mr. Amit. Compute taxable perquisite.

Assets Date of purchase Purchase value Date of sale Sale price


Computer 1/7/2020 2,00,000 18/8/2023 20,000
Car 1/4/2021 3,00,000 1/3/2024 50,000
Television 1/4/2018 50,000 1/4/2023 2,000
Sofa set 1/4/2008 80,000 1/7/2023 5,000
Solution :
Computation of taxable value of perquisite in hands of Mr. Amit for the A.Y.2024-25 :

Assets Written down value Sale value Taxable perquisite


Computer 25,0001 20,000 5,000
Car 1,92,0002 50,000 1,42,000
Television 25,0003 2,000 23,000
Sofa set Nil4 5,000 Nil
Taxable Perquisite 1,70,000
1. Calculation of WDV of Computer :
Particulars Amount
Purchase value 2,00,000
Less : Depreciation from 1/7/2020 to 30/6/2021 @ 50% 1,00,000
WDV as on 1/7/2021 1,00,000
Less : Depreciation from 1/7/2021 to 30/6/2022 @ 50% 50,000
WDV as on 1/7/2022 50,000

118 The Institute of Cost Accountants of India


Heads of Income

Particulars Amount
Less : Depreciation from 1/7/2022 to 30/6/2023 @ 50% 25,000
WDV as on 1/7/2023 25,000
Less : Depreciation from 1/7/2023 to 18/8/2023 (as not being a complete year) Nil
WDV as on the date of sale 25,000
2. Calculation of WDV of Car :
Particulars Amount
Purchase value 3,00,000
Less : Depreciation from 1/4/2021 to 31/3/2022 @ 20% 60,000
WDV as on 1/4/2022 2,40,000
Less : Depreciation from 1/4/2022 to 31/3/2023 @ 20% 48,000
WDV as on 1/4/2023 1,92,000
Less : Depreciation from 1/4/2023 to 1/3/2024 (as not being a complete year) Nil
WDV as on date of sale 1,92,000
3. Calculation of WDV of television :
Particulars Amount
Purchase value 50,000
Less : Depreciation from 1/4/2018 to 31/3/2023 @ 10% 25,000
WDV as on the date of sale 25,000
4. Depreciation on sofa set is charged @ 10% as per straight-line method. Since the asset is used for more than
10 years, hence its WDV will be Nil.

Test Yourself
1. Mr. Lucky has been provided furniture for household use on 1/7/2023, original cost to employer being
₹ 5,00,000 on 17/8/2019. On 1/2/2024, such furniture being sold to the assessee for ₹ 60,000. Find taxable
perquisite for the previous year 2023-24.
Hints
1. ₹ 2,69,167;

2.1.32 Medical Facility [Proviso to Sec. 17(2)]Amended


Medical facility is taxable as under :
a. Medical facility provided in India

Case Treatment
1. Medical facility provided to the employee or his family in a hospital, clinic, dispensary or Fully Exempted
nursing home maintained by the employer.
2. Reimbursement of medical bill of the employee or his family of -
¾¾ Any hospital maintained by Government or Local Authority; or Fully Exempted
¾¾ Any hospital approved by the Government for its employee.

The Institute of Cost Accountants of India 119


Direct Taxation

Case Treatment
3. Payment/reimbursement by employer of medical expenses incurred by an employee
on himself/his family in a hospital, which is approved by the CCIT, for the prescribed
diseases (like Cancer, TB, AIDS, etc.)
Employee must attach with the return of income -
Fully Exempted
¾¾ a certificate from the approved hospital specifying the prescribed disease or ailment
for which hospitalisation was required; and
¾¾ a receipt for the amount paid to the hospital.
4. Group medical insurance (i.e. Mediclaim) obtained by the employer for his employees. Fully Exempted
5. Any reimbursement by employer of any insurance premium paid by the employee, for Fully Exempted
insurance of his health or the health of any member of his family.
6. Reimbursement of medical bill of the employee or his family in respect of any illness Fully Exempted
relating to Covid 19 subject to certain restrictions
b. Medical facility provided outside India

Case Treatment
Medical Expenditure Exempted to the extent permitted by RBI.
Cost of stay abroad (Patient + One Exempted to the extent permitted by RBI.
Attendant/Care taker)
Cost of travel (Patient + One Exempted only when gross total Income of the employee excluding this
Attendant/Care taker) (cost of travel) perquisite, does not exceed ₹ 2,00,000 p.a.
Taxpoint : In calculation of gross total income ceiling, taxable value of
medical treatment perquisite and cost of stay perquisite shall be included.
Notes :
a. Hospital includes a dispensary, a clinic or a nursing home.
b. For this purpose ‘family’ means :
¾¾ Spouse, children of the individual; and
¾¾ Parents, brothers, sisters of the individual, wholly or mainly dependent on him.
c. Fixed Medical Allowance is fully taxable.
d. The expenditure on medical treatment by the employer may be by way of payment or reimbursement.
e. The perquisite is taxable in the hands of specified employee, however if the bills are issued in the name of
employee and reimbursed by the employer, then it shall be taxable in the hands of all employees.

Illustration 29:
Find taxable amount of perquisite in the following cases :
1. Y has been allowed a fixed medical allowance of ₹ 2,000 p.m.
2. Apart from reimbursement of petty medical bill of ₹ 25,000, Z and his family get medical treatment in a
dispensary maintained by the employer. Value of facility provided to Z and his family members during the
previous year are as follows :

120 The Institute of Cost Accountants of India


Heads of Income

Particulars Amount
a. Z 2,000
b. Mrs. Z 5,000
c. Major son of Z (independent) 8,000
d. Minor daughter of Z 25,000
e. Dependent younger brother of Z 8,000
f. Independent younger sister of Z 10,000
g. Dependent sister-in-law 5,000
Solution :
1. Medical allowance is fully taxable, hence the taxable amount is ₹ 24,000
2. Taxable perquisite in hands of Mr. Z is as under :
Particulars Amount
a. Z Nil
b. Mrs. Z Nil
c. Major son of Z (independent) Nil
d. Minor daughter of Z Nil
e. Dependent younger brother of Z Nil
f. Independent younger sister of Z 10,000
g. Dependent sister-in-law 5,000
h. Reimbursement of medical bill 25,000
Taxable Perquisite 40,000
Illustration 30:
Himalaya Ltd. reimburses the following expenditure on medical treatment of the son of an employee Karan. The
treatment was done at UK :
1. Travelling expenses ₹ 1,15,000.
2. Stay expenses at UK permitted by RBI ₹ 45,000 (Actual expenses ₹ 70,000).
3. Medical expenses permitted by RBI ₹ 50,000 (Actual expenses ₹ 70,000).
Compute the taxable perquisites for the assessment year 2024-25 in the hands of Karan, if his annual income from
salary before considering medical facility perquisite was (i) ₹ 1,50,000; (ii) ₹ 2,00,000.
Solution :
Taxable value of perquisite in hands of Mr. Karan is as under :
Particulars Workings Details Case 1 Case 2
Medical expenditure Amount paid in excess of RBI permission
and actual expenditure shall not qualify
₹ 70,000 – ₹ 50,000 ₹ 20,000 ₹ 20,000
for exemption.
Stay cost Stay cost in excess of RBI permission
and actual expenditure shall not qualify
₹ 70,000 – ₹ 45,000 ₹ 25,000 ₹ 25,000
for exemption.
Travel cost Travel cost (Note) Nil ₹ 1,15,000
Total taxable perquisite ₹ 45,000 ₹ 1,60,000

The Institute of Cost Accountants of India 121


Direct Taxation

Note : Travel cost shall be eligible for exemption only if gross total income of the assessee does not exceed
₹ 2,00,000, which can be evaluated as under :

Particulars Case 1 Case 2


Salaries
Annual income from salary other than foreign medical perquisites 1,50,000 2,00,000
Add : Medical facility
Medical expenditure perquisite 20,000 20,000
Stay cost perquisite 25,000 25,000
Gross Total Income for the purpose of foreign travel medical facility 1,95,000 2,45,000

2.1.33 Leave Travel Concession [Sec. 10(5)]


If an employee goes on travel (on leave) with his family and traveling cost is reimbursed by the employer, then
such reimbursement is fully exempted.
Notes :
1. Journey may be performed during service or after retirement.
2. Employer may be present or former.
3. Journey must be performed to any place within India.
4 In case, journey was performed to various places together, then exemption is limited to the extent of cost of
journey from the place of origin to the farthest point reached, by the shortest route. E.g., if you want to go Goa
from Kolkata, you cannot go Manali first and then Goa.
5. Employee may or may not be a citizen of India.
6. Stay cost is not exempt.
Exemption : Exemption is limited to the amount actually incurred on the travel to the extent as under:

Journey performed Maximum exempted fare


By Air Air economic class fare of shortest route
By Rail Air conditioned 1st class fare of shortest route
When the place of origin and destination is connected
by rail but journey is performed by any other mode of
Same as above
transport
When the place of origin and destination is not connected by rail:
Where a recognised public transport system exists First class or deluxe class fare, as the case may be, on
such transport.
Amount equivalent to air-conditioned 1st class rail
fare, for the distance of the journey by the shortest
Where no recognised public transport system exists
route, as if journey had been performed by rail.

122 The Institute of Cost Accountants of India


Heads of Income

Notes :

a. No exemption can be claimed


without performing journey and
incurring expenses thereon.
b. Block-period: Exemption is
available in respect of 2 journeys
performed in a block of 4 calendar
years commencing from 1st January
1986.
Academically, for the A.Y. 2024-
25, the relevant block is Jan 2022 to
Dec. 2025.

c. Carry-forward facility: Where concession is not availed during the preceding block (whether on one occasion
or both), then any one journey performed in the first calendar year of the immediately succeeding block will be
additionally exempted (i.e. not counted in two journey limit)
d. Family: Family here means -
¾¾ Spouse and children of the individual; and
¾¾ Parents, brothers and sisters of the individual, who are wholly or mainly dependent on him.
e. Restriction on number of children: Exemption can be claimed for any number of children born on or before
30/9/1998. In addition, exemption is available only for 2 surviving children born on or after 1/10/1998.
However, children born out of multiple birth, after the first child, will be treated as one child only.
f. Fixed Leave travel allowance: Fixed amount paid to employees by way of leave travel allowance shall not
be exempt.
g. The exemption u/s 10(5) is for travel cost and does not include stay cost or other cost.

2.1.34 Other Perquisites


The value of any other facilities, benefits, amenities, services, rights or privileges (which is not discussed earlier)
provided by the employer shall be determined on the basis of cost to the employer under an arms length transaction,
as reduced by the employee’s contribution, if any.
Taxability of perquisites at a glance

Whether it is taxable
in the hands of
Rule / Section Perquisites Specified Non-specified
employee employee
Rent-free residential accommodation
- Unfurnished
Rule 3(1) - Furnished Yes
- Concessional
- Hotel accommodation

The Institute of Cost Accountants of India 123


Direct Taxation

Whether it is taxable
in the hands of
Rule / Section Perquisites Specified Non-specified
employee employee
Motor car
- If car is owned by employer Yes No
Rule 3(2)
- If car is owned by employee Yes Yes
Free domestic servant
- Appointed by employer Yes No
Rule 3(3)
- Appointed by employee Yes Yes
Gas, electricity or water facility
- If facility is in the name of employer Yes No
Rule 3(4)
- If facility is in the name of employee Yes Yes
Free education
- In case of reimbursement Yes Yes
Rule 3(5)
- In any other case Yes No
Free transport
- If employer is engaged in transport business Yes No
Rule 3(6)
- In any other case Yes Yes
Rule 3(7) Other fringe benefits or amenities
- (i) - Interest free loan or concessional rate of interest
- (ii) -Traveling / Touring / Holiday Home expenditure
- (iii) - Meals / Refreshments
- (iv) - Gift, voucher or token Yes
- (v) - Credit card
- (vi) - Club membership
- (vii) - Use of movable assets
- (viii) - Movable assets sold by employer to its employee
Fair market value of the specified security or sweat
Rule 3(8) & (9) Yes
equity shares allotted to the employee
Sec. 10(5) Leave travel concession No No
Income tax paid by employer on -
Sec.10(10CC) - Non-monetary perquisite No No
- In any other case Yes Yes
Medical facility
Proviso to
- In case of reimbursement Yes Yes
Sec. 17(2)
- In any other case Yes No
Any obligation of employee paid by employer (unless
Sec. 17(2)(iv) Yes Yes
otherwise specifically exempted)

124 The Institute of Cost Accountants of India


Heads of Income

Whether it is taxable
in the hands of
Rule / Section Perquisites Specified Non-specified
employee employee
Allotment/transfer of specified securities or sweat
Sec.17(2)(vi) Yes Yes
equity shares
Sec.17(2)(vii) Contribution to superannuation fund Yes Yes

2.1.35 Provident Fund


Provident fund scheme is a saving device in the hands of salaried class. It is a retirement benefit scheme. Under
this scheme, a stipulated sum is regularly deducted from the salary of the employee as his contribution towards
the fund. The employer also, generally, contributes a similar amount out of his pocket to the fund. The employer’s
and employee’s contribution are together invested in such fund. Interest earned thereon is also credited to the fund
of the employee. Thus, provident fund scheme is a great media to initiate and mobilise small savings to a large
scale. On termination of service or retirement, employee receives the whole accumulated fund, subject to certain
conditions. Hence, provident fund has four components i.e. Employer’s contribution; Employee’s contribution;
Interest on employer’s contribution; and Interest on employee’s contribution
Provident fund is of four types, viz :
a. Statutory Provident Fund (SPF): Statutory provident fund is set up under the provisions of the Provident
Funds Act, 1925. Government and Semi-Government organisations, local authorities, railways, Universities
and recognised educational institutions maintain Statutory Provident Fund.
b. Recognised Provident Fund (RPF): The provident fund scheme is framed under the Employee’s Provident
Fund and Miscellaneous Provisions Act, 1952 (hereinafter referred as PF Act). The PF Act covers any
establishment employing 20 or more persons. However, any establishment employing less than 20 persons
can also join the scheme provided employer and employee both agree to do so. Further, if an employer creates
his own scheme for provident fund then he can do so subject to recognition from the Commissioner of Income
tax.
c, Unrecognised Provident Fund (URPF): If a provident fund scheme is created by an employer, which is not
recognised by the Commissioner of Income tax, then such fund is known as Unrecognised provident fund.
d. Public Provident Fund (PPF): The Central Government has established a fund for the benefit of public to
mobilise personal savings. Any member of the public, whether salaried or self-employed, can contribute to
the fund by opening a provident fund account at any branch of the State Bank of India or its subsidiaries or
other specified bank. Even a salaried employee can simultaneously become a member of employee’s provident
fund (whether statutory, recognised or unrecognized) and public provident fund. Any amount in multiple of
₹5 (subject to minimum of ₹ 500 and maximum of ₹ 1,50,000 p.a.) may be deposited in this account. Interest
is credited every year but payable only at the time of maturity. Interest earned on this fund is exempt from tax
u/s 10(11).
Tax Treatment :

Particulars SPF RPF URPF PPF


Exempted up to 12% of Salary
Employer’s (here, salary means Basic + DA# + Not
Not taxable Not taxable
Contribution Commission as a fixed percentage Applicable
on turnover

The Institute of Cost Accountants of India 125


Direct Taxation

Particulars SPF RPF URPF PPF


Not eligible Eligible for
Employee’s Eligible for
Eligible for deduction u/s 80C for deduction deduction
Contribution deduction u/s 80C
u/s 80C u/s 80C
Exempted @ 9.5% p.a. (Interest
Interest Not Taxable rate), any excess interest will be Not Taxable Not Taxable
taxable as salary.
Exempted u/s Exempted u/s 10(12)
Lump Sum 10(11). However,
(Subject to Note 2)
withdrawal in few cases, it is
taxable* However, in few cases, it is taxable* Note 1 Not taxable
#
D.A., forming part of retirement benefit, only to be considered.
Notes :
1. Lump sum amount withdrawn from URPF

Particulars Tax treatment


Accumulated employer’s contribution Fully taxable under the head Salaries
Accumulated employee’s contribution Not taxable
Accumulated interest on employer’s contribution Fully taxable under the head Salaries
Accumulated interest on employee’s contribution Fully taxable as income from other sources
2. Lump sum amount withdrawn from RPF
a. Amount withdrawn from RPF is not taxable, if
i. Employee retires or terminates job after 5 years of continuous service; or
ii. Employee has resigned before completion of 5 years and joins another organization (who also
maintains recognized provident fund and his fund balance with current employer is transferred to the
new employer).
iii. The entire balance standing to the credit of the employee is transferred to his account under New
Pension Scheme as referred u/s 80CC
iv. Employee retires or terminates job before 5 years of continuous service -
●● by reason of ill health; or
●● by reason of contraction or discontinuance of employer’s business; or
●● any other reason beyond the control of employee.
Taxpoint : Though aforesaid conditions are satisfied, in few cases, it is taxable [Refer sec. 10(12)]

b. In any other case, amount withdrawn shall be taxable as in the case of URPF. [Refer Note 1].
Points to be remembered
1. Employer’s Contribution to the New pension System (as specified u/s 80CCD) is fully taxable under the head
‘Salaries’. However, deduction is available u/s 80CCD.

* W.e.f. A.Y. 2022-23, section 10(11)/(12) has been amended to provide that exemption shall not be available to the interest income accrued during the
previous year in the provident fund account of the employee to the extent it relates to the amount of the contribution made by such person exceeding
₹ 2,50,000 [₹ 5,00,000, if contribution by such person is in a fund in which there is no contribution by the employer of such person] in a previous
year in that fund.

126 The Institute of Cost Accountants of India


Heads of Income

2. The amount or the aggregate of amounts of any contribution made to the account of the assessee by the
employer:
(a) in a Recognised Provident Fund (RPF);
(b) in the scheme referred to in sec. 80CCD(1) [i.e., NPS]; and
(c) in an approved superannuation fund,
- in excess of ₹ 7,50,000 in a previous year shall be taxable
Taxpoint : There is combined upper limit of ₹ 7,50,000 in respect of employer’s contribution in a year to NPS,
superannuation fund and recognised provident fund and any excess contribution is taxable.
3. The annual accretion (like interest, dividend, etc.) during the previous year to the balance at the credit of the
aforesaid fund or scheme to the extent it relates to the contribution referred above shall be taxable
Taxpoint : Such accretion shall be included in the total income and shall be computed in such manner as may
be prescribed.
4. Central government contribution to Agniveer Corpus Fund is fully taxable. However, deduction is available
u/s 80CCH.

Illustration 31 :
Mr. X has the following salary structure –

Basic pay ₹ 10,000 p.m. Commission (fixed) ₹ 2,000


DA ₹ 1,000 p.m. Entertainment allowance ₹ 2,000 p.m.
X contributes ₹ 20,000 to provident fund. Employer also makes a matching contribution. Compute gross salary of
if –
a. Mr. X is a Government employee and such provident fund is a statutory provident fund.
b. Mr. X is an employee of Y Ltd. and such fund is a recognized fund.
c. Mr. X is an employee of Z Ltd. and such fund is an unrecognized fund.
Solution :
Computation of taxable salary of Mr. X for the A.Y. 2024-25 :

Case A Case B Case A


Particulars
Details Amount Details Amount Details Amount
Basic 1,20,000 1,20,000 1,20,000
Commission 2,000 2,000 2,000
Allowances
Dearness allowance 12,000 12,000 12,000
Entertainment allowance 24,000 36,000 24,000 36,000 24,000 36,000
Employer’s contribution to PF 20,000 20,000 20,000
Less : Exempted 20,0001
Nil 15,840 2
4,160 20,0001 Nil
Gross Salary 1,58,000 1,62,160 1,58,000

The Institute of Cost Accountants of India 127


Direct Taxation

Notes :
1. Contribution to statutory and unrecognised provident fund is fully exempted.
2. Contribution to recognised provident fund is exempt upto 12% of salary. Salary for such purpose –

Particulars Amount
Basic 1,20,000
Commission (as fixed) Nil
Dearness allowance 12,000
Total 1,32,000

Test Yourself
1. Miss Sanchita has the following salary structure, compute her gross salary for the A.Y.2024-25 :
a. Basic 5,000 p.m.
b. D.A. 2,500 p.m. (60% forms a part of retirement benefit)
c. Commission on turnover 20,000 during the year
d. Medical allowance 1,000 p.m.
e. She contributes 14% of her salary to RPF. Her employer contributes a similar amount.
f. Interest credited to this fund during the year is ₹ 3,000 @ 12% p.a.
Hints
1. ₹ 1,24,585;

Transferred Balance (Conversion of URPF to RPF) [Rule 11(4) of Part A of the Fourth schedule]
An organisation maintaining URPF, may later get recognition from Commissioner of Income tax. In such case, the
accumulated balance under URPF shall be converted to RPF. Tax treatment of such transferred balance will be as
under :
Calculation is made of all sums comprised in the transferred balance that would have been liable to income tax if
the recognition of the fund had been in force from the date of institution of the fund. However, in case of serious
accounting difficulty, the Commissioner may make a summary calculation of such aggregate.
Such aggregate sum is deemed to be the income received by the employee in the previous year in which the
recognition of the fund takes effect.
Note : On taxability of such conversion, assessee cannot claim relief u/s 89(1).

Illustration 32 :
Mr. Sharma has been appointed as an accountant of ABC Ltd as on 1/4/2021, since then he is working with the
same company. The salary structure and increment details are as under:
Basic ₹ 5000 - 1000 - 8000 -1500 - 14000
D.A. ₹ 3000 – 500 – 5000 – 1000 - 10000
He and his employer contribute to URPF 14% of basic and DA.
Every year 9% interest is credited to such fund. As on 1/4/2023, the fund gets recognition. Hence, the accumulated
balance in URPF was transferred to RPF. Comment on tax treatment of such transferred balance.

128 The Institute of Cost Accountants of India


Heads of Income

Solution :
Statement showing treatment of transferred balance :

Year Employer’s contribution to fund Exempted amount considering the Difference


fund as RPF
2021-2022 14% of (60,000 + 36,000) i.e. ₹ 13,440 12% of ₹ 96,000 i.e. ₹ 11,520 ₹ 1,920
2022-2023 14% of (72,000 + 42,000) i.e. ₹ 15,960 12% of ₹ 1,14,000 i.e. ₹ 13,680 ₹ 2,280
Total ₹ 4,200
Current year (i.e. 2023-24) contribution shall be treated as RPF and taxable amount will be ₹ 2,640 [being (14
-12)% of (₹ 84,000 + ₹ 48,000) i.e. 2% of ₹ 1,32,000].
Since interest rate is less than the exempted limit (i.e. 9%), hence interest portion is not taxable.
Total taxable salary on account of provident fund for the A.Y. 2024-25 is ₹ 6,840 (being ₹ 4,200 + ₹ 2,640).

Deduction from Gross Salary [Sec. 16]

2.1.36 Standard Deduction [Sec. 16(ia)


Lower of the following shall be allowed as standard deduction to all employee :
a. ₹ 50,000
b. Amount of gross salary

2.1.37 Entertainment Allowance [Sec. 16 (ii)]


Entertainment allowance is initially included in taxable allowances as fully taxable. Thereafter, a deduction
is allowed under this section from gross taxable salary. However, deduction u/s 16(ii) shall be available to the
Government employee only.
Deduction for Entertainment allowance being minimum of the following :
a. Actual Entertainment Allowance
b. ₹ 5,000/-
c. 20% of Basic Salary.
Taxpoint :
 Deduction allowed shall be irrespective of actual expenditure incurred, whether for office or personal purpose.
 No deduction is available under this section to a Non-government employee.

Illustration 33:
Compute taxable Entertainment allowance & net salary of Sri Hanuman Prasad from the following data :
Basic salary ₹ 8,000 p.m. D.A. ₹ 2,000 p.m. Taxable perquisite ₹ 35,000, Entertainment Allowance ₹ 4,000 p.m.
Out of such allowance ₹ 20,000 is expended and balance amount is saved. Assuming he is:
a. Government employee b. Non-Government employee.

The Institute of Cost Accountants of India 129


Direct Taxation

Solution :
Computation of taxable income of Sri Hanuman Prasad for the A.Y.2024-25 :

Government Employee Non-Government


Particulars Employee
Details Amount Details Amount
Basic Salary 96,000 96,000
Dearness Allowance 24,000 24,000
Entertainment Allowance 48,000 48,000
Taxable perquisite 35,000 35,000
Gross Taxable Salary 2,03,000 2,03,000
Less : Deduction u/s
16(ia) Standard Deduction 50,000 50,000
16(ii) Entertainment allowance# 5,000 55,000 Nil 50,000
Net Taxable Salary 1,48,000 1,53,000
#
Entertainment Allowance is exempted to the extent of minimum of the following :
a. Actual Entertainment Allowance ₹ 48,000
b. 20% of Basic Salary ₹ 19,200
c. Statutory amount ₹ 5,000

2.1.38 Tax on employment or professional tax [Sec. 16(iii)]


Tax on employment, profession, trade, etc. levied by a State under Article 276 of the Constitution will be allowed
as deduction on cash basis, whether paid by employee or by employer (on behalf of employee) from gross taxable
salary.
Note : If employer (on behalf of employee) pays Professional tax then :
a. Firstly, it is to be included as taxable perquisite; and
b. Further, it is allowed as deduction u/s 16(iii).

Illustration 34 :
Mr. Rohit a non-Government employee has the following salary details :

a. Basic Salary ₹ 5,000 p.m. b. D.A. ₹ 2,000 p.m.


c. Entertainment Allowance ₹ 300 p.m. d. Professional tax paid by ₹ 600
employee
e. LIC Premium paid by employer ₹ 3,600 f. Income tax paid by employee ₹ 2,000
g. Professional tax paid by employer ₹ 1,600
on behalf of employee
Find his taxable salary.

130 The Institute of Cost Accountants of India


Heads of Income

Solution :
Computation of taxable salary Mr. Rohit for the A.Y.2024-25 :

Particulars Details Amount


Basic Salary 60,000
Allowances
Dearness Allowance 24,000
Entertainment Allowance 3,600 27,600
Taxable perquisite
Professional tax paid by employer 1,600
LIC Premium paid by employer 3,600 5,200
Gross Taxable Salary 92,800
Less : Deduction u/s 92,800
16(ia) Standard Deduction 50,000
16(ii) Entertainment allowance (Assessee is a Non-government employee) Nil
16(iii) Professional Tax (₹ 1,600 + ₹ 600) 2,200 52,200
Taxable Salary 40,600
Notes :

A. Conversion of Net Salary into Gross Salary

Sometimes net basic salary is given after deduction of TDS, Loan repayment, PF deduction etc that needs to
be grossed up as under :

Net Salary = Gross Salary – Employee’s contribution to provident fund – TDS – loan repayment by employee
– other deduction from salary (if any).

Example 7 : Find basic salary of Mr. Singh having the following salary structure :

a. Net Basic Salary received ₹ 1,00,000

b. Deduction from salary 10% of basic salary as contribution to RPF

c. TDS ₹ 9,000

d. Repayment of earlier loan ₹ 35,000

In this case, Basic Salary shall be computed as under :

Net basic salary = Basic salary – TDS – Loan repayment – Contribution to RPF

Let the basic salary be X

1,00,000 = X – 9,000 – 35,000 – 0.1X

1,44,000 = 0.9X

The Institute of Cost Accountants of India 131


Direct Taxation

X = 1,60,000. Hence, basic salary for the year is ₹ 1,60,000.

B. Meaning of Salary for different purposes :

For Retirement benefit


Gratuity (covered by the Payment of Gratuity Act) (Basic + DA) last drawn
Gratuity (not covered by the Payment of Gratuity Act) (Basic +DA1 + Commission2) being average of last
10 months preceding the month of retirement.
Leave encashment (Basic +DA1 + Commission2) being average of last 10
months immediately from the retirement.
Voluntarily retirement (Basic +DA1 + Commission2) last drawn
For regular benefit
Rent Free Accommodation (Basic + DA1 + Commission2 + Bonus + Fees + Any
other taxable allowance + Any other monetary benefits
excluding perquisite)
Specified employee (Basic + DA + Commission2 + Bonus + Fees + Any
other taxable allowance + Any other monetary benefits
– Deduction u/s 16)
Entertainment Allowance Basic only
Any other case (Basic +DA1 + Commission2)
1.
DA only if it forms a part of retirement benefit. 2.
Commission as a fixed percentage on turnover.

Illustration 35 :
Mr. Bharat of Siliguri is offered an employment by Vimal & Co. Ltd., Kolkata on a basic salary of ₹ 5,500 p.m.
Other allowances are dearness allowance (not forming part of salary for retirement benefits) ₹ 4,000 p.m., medical
allowance ₹ 1,000 p.m. and bonus being 1 month’s basic salary. The company gives an option to him either to
take a rent-free accommodation in Kolkata of the fair rental value of ₹ 1000 p.m. or to accept a cash house rent
allowance of ₹ 1,000 p.m. He decides to accept house rent allowance and takes a house in Kolkata at a monthly
rent of ₹ 1,000.
Do you think he has made a wise choice from tax advantage view? State reasons.
Solution :
Computation of Gross Taxable Salary of Mr. Bharat for the A.Y.2024-25 :

When he takes HRA When he takes RFA


Particulars
Amount Amount Amount Amount
Basic 66,000 66,000
Bonus 5,500 5,500
Dearness allowance 48,000 48,000

132 The Institute of Cost Accountants of India


Heads of Income

When he takes HRA When he takes RFA


Particulars
Amount Amount Amount Amount
Medical allowance 12,000 12,000
HRA 12,000
Less : Minimum of the following
a) Actual amount received 12,000
b) 50% of salary 33,000
c) Rent paid over 10% of salary 5,400 5,400 6,600 Nil
Rent free accommodation (being 15% of salary) Nil 12,525
Gross Taxable Salary 1,38,100 1,44,025
Note : Salary for the purpose :

Particulars Accommodation HRA


Basic salary 66,000 66,000
Medical Allowance 12,000 --
Bonus 5,500 --
Total 83,500 66,000
The above computation indicates that if the assessee chooses rent-free accommodation, then his gross taxable
salary increases by ₹ 5,925 (being ₹ 1,44,025 – ₹ 1,38,100), which may increase his tax bill. Hence, assessee has
taken right decision.

Illustration 36:
Following are the particulars of income of Mrs. S. Choudhury for the Previous Year 2023-24 :
a. Basic salary @ ₹ 15,000 per month.
b. Dearness Allowance @ 60% of salary.
c. Medical Allowance @ 600 per month (Actual expenditure ₹ 5,000).
d. House Rent Allowance received @ ₹ 6,000 per month and she pays rent of ₹ 7,200 per month for her house in
Durgapur.
e. City compensatory allowance ₹ 1,500 per month.
f. She owns a car which she is using for official purposes. Her employer reimburses her @ ₹ 3,000 per month.
g. She is contributing ₹ 2,100 per month towards a recognized provident fund. The employer is also contributing
the same amount. Interest credited to R.P.F @ 11% ₹ 2,200.
h. She paid ₹ 1,800 as professional tax during the year.
Compute income from salary of Mrs. Choudhury for the assessment year 2024-25.

The Institute of Cost Accountants of India 133


Direct Taxation

Solution :
Computation of Taxable Salary of Mrs. S Choudhury for the A.Y.2024-25 :

Particulars Working Details Amount Amount


Salaries
Basic 1,80,000
Allowances
Dearness allowance 60% of basic 1,08,000
Medical Allowance 7,200
City compensatory allowance 18,000
House rent allowance 72,000
Less : Exempted u/s 10(13A)
Minimum of the following :
a. Actual HRA 72,000
b. 40% of (Basic + DA) 1,15,200
c. Rent paid – 10% (Basic + DA 57,600 57,600 14,400 1,47,600
Perquisites u/s 17(2):
Car facility - -
Employer’s contribution to RPF 25,200
Less : Exempted u/s 10(12) 12% of (Basic + DA) 25,200
Interest on RPF 2,200
Less : Exempted Upto 9% 1,800 400 400
Gross taxable salary 3,27,600
Less : Deduction u/s
16(ia) Standard Deduction 50,000
16(iii) Professional tax 1,800 51,800
Taxable Salary 2,75,800

Quick MCQs:-
1. Salary received by the Partner of a Firm is charged under the head
(a) Salaries
(b) Business Income
(c) Other Sources
(d) Its exempt from tax

134 The Institute of Cost Accountants of India


Heads of Income

2. Commission received by a Director of the Company is charged under the head.


(a) Salaries
(b) Business Income
(c) Other Sources
(d) Its exempt from tax

3. Salary is a taxable on
(a) Receipt basis
(b) due basis
(c) due to receipt basis whichever is earlier
(d) due or receipt basis whichever is later.

4. Arrears of Pension is taxable on


(a) receipt basis
(c) due basis
(d) due or receipt basis whichever is earlier
(d) due or receipt basis whichever is later

5. The Notified amount of Gratuity that is exempt


(a) ` 10,00,000
(b) ` 20,00,000
(c) ` 2,00,000
(d) ` 5,00,000

6. Uncommuted Pension received by a Government Employee is-


(a) Exempt
(b) Taxable
(c) Partially Taxable
(d) None of the above

7. Encashment of Leave Salary at the time of retirement is fully exempt in the case of-
(a) Central Government Employee
(b) State Government Employee
(c) Both Central and State Government Employees
(d) Government Employee and Employee of Local Authority.

8. Interest credited to Statutory Provident Fund shall be-


(a) Fully Exempt
(b) Exempt upto 12% p.a.
(c) Fully Taxable
(d) Exempt upto 9.5% p.a

The Institute of Cost Accountants of India 135


Direct Taxation

9. Interest credited to Recognized Provident Fund is –


(a) Fully Taxable
(b) Fully Exempt
(c) Exempt upto 12% of Salary
(d) Exempt upto 9.5% p.a.

10. For an employee in receipt of Hostel Expenditure Allowance for his 3 children, the maximum annual
allowance exempt u/s 10(14) is –
(a) D 10, 8000
(b) D 7,200
(c) D 9, 600
(d) D 3, 600

136 The Institute of Cost Accountants of India


Heads of Income

Income from House Property 2.2

A
s per sec. 22, the annual value of property consisting of any building or land appurtenant thereto of
which assessee is the owner, other than such portion of such property as he may occupy for the purposes
of any business or profession carried on by him shall be chargeable to income tax under the head
“Income from house property.”

It is an exceptional feature of this head that rather than actual income from house property, earning
capacity of house property is taxable. As stated u/s 22 that “annual value” of the property is taxable rather
than actual income of the property. (Annual value being discussed in later part of this chapter)

2.2.1 Chargeability [Sec. 22]


Annual value of the property shall be taxable under the head “Income from house property” subject to the
following :

Condition 1 : Building or land appurtenant thereto


The term ‘house property’ is not defined in Income tax Act. However, various judicial interpretation has construed
the term house property as -
 any land surrounded by wall having roof or not; and
 any land appurtenant to a building.
Notes :
a. Building includes an enclosure of bricks, stone work or even mud walls
b. Building includes residential as well as commercial houses.
c. Vacant land is not a house property. Hence, income from letting of vacant land is not taxable under this head
but taxed as business income or as income from other sources.
d. Roof is not necessary for a non-residential house property. A large stadium or a open air swimming pool is also
considered as building

The Institute of Cost Accountants of India 137


Direct Taxation

e. It should be a permanent structure meant for a useful purpose.


f. If a building consists of several flats, then each flat is considered as a separate house property.
g. An incomplete, a ruined or demolished house cannot be termed as house property.
h. Land appurtenant to a building includes car parking area, approach roads, backyards, courtyards, etc. attached
to such building.
Condition 2 : Owner
Annual value of a property is assessed to tax only in the hands of the owner even if he is not in receipt of any
income. Any person other than the owner, even though he is in receipt of rent shall not be liable to tax under this
head. That is why, income from sub-letting is not taxable under this head but under the head ‘Income from other
sources’. E.g. Mr. X being a tenant of a house property acquired it at a monthly rent of ₹ 10,000 from Mr. Y (owner
of such house property). Mr. X sublets the property to Mr. Z for a monthly rent of ₹ 12,000. Income from subletting
being ₹ 2,000 p.m. is taxable as business income or as income from other sources.
Owner includes legal owner, beneficial owner and deemed owner.
Legal owner : Legal owner means a person who has the legal title of the property as per the Transfer of Property
Act, Registration Act, etc.
Beneficial owner : For income tax purpose it is not necessary that the property must be registered in the name of
the assessee. If the assessee is enjoying the property as an owner to full extent he will be treated as a beneficial
owner of such property and will be charged under the head ‘Income from house property’.

Fictional owner or Deemed owner [Sec. 27]


U/s 27, in the following cases, a person shall be treated as deemed owner of the property and liable to tax (in such
case legal owner or beneficial owner shall not be further liable to tax)

1. Transfer to spouse or minor child [Sec.


27(i)] : When an individual transfer a house
property to -
¾¾ his or her spouse (not being a transfer
in connection with an agreement to live
apart); or
¾¾ a minor child (not being a married
daughter)
- without adequate consideration, then
transferor shall be treated as deemed
owner of such property.
E.g. : Mr. X transfers his house property worth ₹ 5,00,000 to Mrs. X out of love and affection. In such case,
though Mrs. X is the legal owner but Mr. X will be liable to tax as deemed owner of such property.
Note : In case of transfer to spouse, marriage should subsist on both the days i.e., on the day of transfer as well
as on the day when income arises.
Taxpoint :
¾¾ Transferee must be spouse or minor child other than married daughter.
¾¾ Transfer must be without adequate consideration.
¾¾ Transferred property must be a house property. E.g. Mr. X transfers cash of ₹ 5,00,000 to Mrs. X and

138 The Institute of Cost Accountants of India


Heads of Income

Mrs. X purchases a house property from such cash, then such transfer of cash and subsequent purchase of
property shall not attract provision of sec. 27(i). However, the income from such property shall be clubbed
in the hands of Mr. X as per the provision of sec. 64(1)(iv) [For detail refer chapter Clubbing of Income].
2. The holder of an impartible estate [Sec. 27(ii)] : The holder of an impartible estate (property which is not
legally divisible) is treated as deemed owner of house property. Impartible estate is an estate to which the
assessee has succeeded by grant or covenant.
3. Property held by a member of a company, society or any other association [Sec. 27(iii)] : Property held
by a member of a company, co-operative society or other association of persons to whom a building or a part
thereof is allotted or leased under House Building Scheme of the company or association, is treated as deemed
owner of that building or a part thereof.
Taxpoint :
¾¾ Assessee is the member of a company, co-operative society or other AOP.
¾¾ He has been allotted or leased a building on account of such membership.
¾¾ Though he is not the legal owner of such property, still he will be liable to tax.
4. A person who acquired a property u/s 53A of the Transfer of Property Act [Sec. 27(iiia)] : A person who
is allowed to take or retain possession of any building (or part thereof) in part performance of a contract u/s
53A of the Transfer of Property Act, 1882, is deemed as the owner of that building (or part thereof).
Taxpoint :
¾¾ Assessee has taken the possession of the property.
¾¾ He has partly performed or promised to perform the contract i.e., he has paid (or is ready to pay) a part of
the consideration.
¾¾ The contract must be in writing. Though sale-deed might not be executed in favour of the buyer, still
certain other document like ‘power of attorney’ or ‘agreement to sell’ has been executed.
5. Lessee of a building u/s 269UA(f) [Sec. 27(iiib)] : A person who acquires any right u/s 269UA(f) in or with
respect to any building or part thereof, by way of lease agreement for a period not less than 12 years is deemed
as the owner of that building (or part thereof).
Notes :
a. Lease period should not be less than 12 years [as per sec. 269UA(f)] including extension period.
b. Above provision does not include any right by way of lease from month to month or for a period not
exceeding 1 year.
E.g. : X lets out a property to Miss Y on a lease of 9 years. However, Miss Y has a right to renew the lease
for further period of 3 years. In such case, Miss Y shall be deemed as an owner of the property u/s 27.
However, if such right of renewal of lease (for 3 years) is subject to condition that at each occasion it will
be renewed for a period of 11 months, then X will be owner of the property and liable to tax u/s 22.

Condition 3: Property is not used for business or profession carried on by the assessee
When a person carries on business or profession in his own house property, annual value thereof is not taxable u/s
22 provided income of such business is chargeable to tax.
Incidences thereof
 Letting out to employees: If an assessee lets out the property to his employee, where such letting out supports
smooth flow of his business, then such letting out shall be deemed to be incidental to business and such rent

The Institute of Cost Accountants of India 139


Direct Taxation

shall be chargeable under the head “Profits & gains of business or profession”.
 Letting out to Government Agencies: Where an assessee let out his property to any Government agency
for locating branch of a nationalized bank, police station, post office, excise office, railway staff quarters,
etc. for the purpose of running the business of assessee more efficiently, such letting out shall be deemed to
be incidental to business and such rent shall be chargeable under the head “Profits & gains of business or
profession”.
 Letting out to ancillary units: Where an assessee lets out its property to ancillary units, which manufactures
components required by the assessee. Income from such letting out shall be taxable under the head “Profits &
gains of business or profession”.
Letting out property for promotion of own business –vs.- Business of letting out the property :

2.2.2 Some special cases


Foreign property
If house property is situated abroad, then annual value of such property shall be taxable as :

Assessee Condition for taxability


Ordinarily resident Always taxable
Not ordinarily resident or Non resident Income must be received in India
Note : The annual value of such property would be computed as if the property is situated in India.
Disputed ownership
Merely, due to dispute regarding the title of property, assessment cannot be postponed. In such case, person who is
in receipt of income or who enjoys the possession of the property is assessable to tax.
Composite rent
Together with rent of the building, if the owner gets charges for other services or rent of other assets provided in
the building (e.g. furniture, machinery, etc.), amount so received is termed as ‘composite rent’.
Composite Rent = Rent for building + Rent for assets / Charges for various services
Tax treatment of composite rent is as follows :

140 The Institute of Cost Accountants of India


Heads of Income

 Rent including charges for amenities or services like garden facility, food, lighting, etc. or other separable
assets (like machinery, plant, furniture): If the owner of house property gets composite rent for both property
as well as for services rendered or other separable asset, such composite rent shall be treated as under :

Particulars Taxable under the head


Sum received for the use of building. ‘Income from house property’.
Sum received for other amenities or other separable ‘Profits & gains of business or profession’; or
assets.
‘Income from other sources’
However, if segregation of composite rent is not possible, then the whole amount will be taxed either under the
head ‘Profits & gains of business or profession’ or ‘Income from other sources’.
Taxpoint : Rent from paying guest is, generally, taxable under the head ‘Income from other sources’.
 Letting of building with other inseparable assets (like machinery, plant, furniture): If letting of only building
is not possible or not acceptable to the other party, then sum received as rent from the properties is chargeable
as business income or income from other sources even if the composite rent is segregable. E.g., letting out of
hotel rooms, auditoriums, etc.

Co-ownership [Sec. 26]


If two or more persons own a house property jointly, then they are known as co-owners. If individual share of each
co-owner is definite and ascertainable then the share of each such person shall be taxable as his income from house
property.
Tax treatment :
1. Share of each co-owner in the income from the property as computed in accordance with sec. 22 to 25 shall be
included in his total income.
2. Where the house property is owned by co-owners and is occupied by each of the co-owner then all of them can
claim benefit u/s 23(2)(a) and interest on loan shall be allowed to all the co-owners to the extent of ₹ 30,000 /
₹ 2,00,000 as the case may be.
Note : Provision of Sec. 26 is mandatory and not optional.
Partner’s property used by the firm
The business carried on by the firm should be regarded as carried on by all the partners. Thus, annual letting value
of a property belonging to the assessee which is in occupation of the firm in which assessee is the partner, is not
includible in income of the assessee-partner u/s 22
Property held as stock-in-trade [Sec. 23(5)]
Where house property is held as stock-in-trade & not let out during any part of the previous year, then annual value
of such property shall be computed as under :

Period Annual Value


Up to 2 year from the end of the financial year in which Annual value of such property shall be taken to be nil.
the certificate of completion of construction of the
property is obtained from the competent authority
After the completion of aforesaid period Annual value of such property shall be computed as per
other provisions. (Discussed later)

The Institute of Cost Accountants of India 141


Direct Taxation

Doctrine of mutuality
Sec. 22 levies tax on annual value of house-property and not on actual income from house property. In case of a
club, which provides recreational facilities exclusively to its member and their guest and not to any non-members,
it is considered as a non-profit seeking person and run on no-profit no-loss basis. Such club is running on the
principle of mutuality and its members are not entitled to any share of profit. In the case of such a mutual concern,
not only the surplus of the organisation but also the annual value of the club house shall be exempted from tax.

2.2.3 Exempted properties


Income from the following house properties are exempted from tax :
1. Any one palace or part thereof of an ex-ruler, provided the same is not let out [Sec. 10(19A)].
Taxpoint : If the ex-ruler has a house property and the part of which is self-occupied and remaining let out
then only the self occupied part of the house property shall be exempted.
2. House property of a local authority [Sec. 10(20)].
3. House property of an approved scientific research association [Sec. 10(21)].
4. House property of an educational institution [Sec. 10(23C)].
5. House property of a hospital [Sec. 10(23C)].
6. House property of a person being resident of Ladakh [Sec. 10(26A)]
7. House property of a political party [Sec. 13A]
8. House property of a trade union [Sec. 10(24)]
9. A farm house [Sec. 10(1)]
10. House property held for charitable purpose [Sec. 11]
11. House property used for own business or profession [Sec. 22].

2.2.4 Computation of Income


The chapter is divided into the following categories for the purpose of computation:

¾¾ Let out property [Sec. 23(1)]


¾¾ Self-occupied property [Sec. 23(2)(a)].
¾¾ Deemed to be let out property [Sec. 23(4)].
¾¾ Property not actually occupied by the owner [Sec. 23(2)(b)]
¾¾ Partly let out and partly self occupied property [Sec. 23(3)]
¾¾ Recovery of arrears of rent and unrealized rent [Sec. 25A]

2.2.5 Let out property [Sec. 23(1)]


Computation at a glance
Computation of Income from house property of …………. for the Assessment Year ……….

142 The Institute of Cost Accountants of India


Heads of Income

Particulars Details Amount


Gross Annual Value (GAV) ****
Less : Municipal tax ****
Net Annual Value (NAV) ****
Less : Deductions u/s
24(a) Standard deduction [30% of NAV] ****
24(b) Interest on borrowed capital **** ****
Income from house property ****
Gross Annual Value (GAV) :

Normally, income tax is charged


on income, but under the head
‘Income from house property’,
tax is not charged on the rent
earned from house property but
on the inherent earning capacity
of the house property. Such
earning capacity is termed as
Annual Value. Annual value
is determined considering the
following factors :

A. Actual Rent Receivable [ARR]


Any sum receivable as rent of the house property for the previous year is an evidence for determining the
earning capacity of the building. Such actual rent receivable is to be computed on accrual basis. However,
where tenant pays rent, which is influenced by benefits provided by the owner of the property, such rent must
be disintegrated to determine actual rent i.e. De-facto rent of the property.
De facto rent = ARR – Cost of amenities.
Taxpoint : While computing actual rent receivable, outstanding rent shall be considered but advance rent
received during the financial year is not to be considered.
B. Gross Municipal Value
It means the annual value of the property decided by municipality on which they charge municipal tax. Such
valuation may also be taken as evidence of earning capacity of a property.
In metro cities (i.e. Chennai, Delhi, Kolkata, Mumbai), municipal authorities charge tax on Net Municipal
Value after giving a deduction for repairs (being 10% of Gross Municipal value) and an allowance for service
taxes (like sewerage tax, water tax etc. as a % of Net Municipal value). Hence, the relation between Gross
Municipal Value and Net Municipal Value can be concluded as under -
In metro cities NMV = GMV – 10% of GMV – Sewerage/Water Tax etc. (as a % of NMV)
In non-metro cities GMV = NMV
C. Fair or Notional rent of the property
Fair or notional rent of a property means rent fetched by a similar property in the same or similar locality.

The Institute of Cost Accountants of India 143


Direct Taxation

Though two properties might not be exactly similar still it is an indicator of rent reasonably expected from
the property. An inflated or deflated rent due to emergency, relationship and such other conditions need to be
adjusted to determine fair rent.
For instance, a property was let out to a friend for a monthly rent of ₹ 2,000 which might be let out to another
person at the rate of ₹ 2,500 p.m. In such case, fair rent of the property shall be ₹ 2,500 p.m.
D. Standard rent under the Rent Control Act
Standard rent is the maximum rent, which a person can legally recover from his tenant under the Rent Control
Act prevailing in the State in which the property is situated. A landlord cannot reasonably expect to receive
from a tenant any amount more than Standard Rent. Accordingly, it can be concluded that if the property is
covered by the Rent Control Act then Reasonable Expected Rent (RER) cannot exceed Standard Rent.
Taxpoint : Reasonable Expected Rent cannot exceed Standard Rent but can be lower than Standard Rent
Computation of Gross Annual Value
Step 1 : Calculate reasonable expected rent (RER) of the property being higher of the following:
a. Gross Municipal Value.
b. Fair Rent of the property.
Note : RER cannot exceed Standard Rent.
* Reasonable Expected Rent (RER) is also known as Annual Letting Value (ALV).
Step 2 : Calculate Actual Rent Received or Receivable (ARR) for the year less current year unrealised rent (UR)
subject to certain conditions#.
#Unrealised Rent [Rule 4]: Unrealised Rent of current year shall be deducted in full from Actual Rent
Receivable, provided the following conditions are satisfied :
i) The tenancy is bona fide;
ii) The defaulting tenant has vacated the property or steps have been taken to compel him to vacate the
property;
iii) The defaulting tenant is not in occupation of any other property of the assessee;
iv) The assessee has taken all reasonable steps to institute legal proceeding for the recovery of the unpaid rent
or has satisfied the Assessing Officer that legal proceedings would be worthless.
Step 3 : Compare the values calculated in step 1 and step 2 and take the higher one.
Step 4 : Where there is vacancy and owing to such vacancy the ‘ARR – UR’ is less than the RER, then ‘ARR -
UR’ computed in step 2 will be treated as GAV.
In nutshell, GAV shall be computed as under :
Steps Particulars Amount
Compute Reasonable Expected Rent [RER]
Gross Municipal Value (a) ****
Fair Rent (b) ****
1st
Higher of the (a) and (b) [A] ****
Standard Rent [B] ****
Reasonable Expected Rent [Lower of (A and B)] [C] ****
Actual Rent Received or Receivable (ARR) – Unrealised Rent of the current year
2nd ****
(UR) [D]

144 The Institute of Cost Accountants of India


Heads of Income

Steps Particulars Amount


3rd Gross Annual Value
Higher of C and D shall be considered as GAV ****
4th However, where ‘ARR – UR’ is lower due to vacancy, then ‘ARR - UR’ computed
****
in step 2 will be treated as GAV.
Illustration 37 : [Computation of Reasonable Expected Rent]
Calculate Reasonable Expected Rent from the following details :
Particulars House 1 House 2 House 3 House 4 House 5
Gross Municipal Value (a) 10,000 12,000 12,000 18,000 16,000
Fair Rent (b) 8,000 16,000 16,000 10,000 17,000
Higher of the [(a) and (b)] [A] 10,000 16,000 16,000 18,000 17,000
Standard Rent as per Rent Control Act [B] 10,000 14,000 N.A 8,000 20,000
Reasonable Expected Rent [Lower of [(A) & (B)] 10,000 14,000 16,000 8,000 17,000

Illustration 38 : [When there is neither unrealised rent nor vacancy period]


Calculate Gross Annual Value for the following house properties. (₹ in ‘000)
Particulars H1 H2 H3 H4 H5 H6
Gross Municipal value for the whole year 120 130 140 150 160 180
Fair rent for the whole year 105 115 135 155 175 168
Standard rent (for the whole year) NA 100 135 180 165 144
Actual rent receivable 100 110 135 175 200 100
Period of the previous year (in months) 12 12 12 12 12 10
Solution :
Computation of Gross annual value (GAV) (₹ in ‘000)

Step Particulars H1 H2 H3 H4 H5 H6
Calculation of RER
Gross Municipal Value (a) 120 130 140 150 160 1501
Fair Rent (b) 105 115 135 155 175 1401
Higher of the [(a) and (b)] [A] 120 130 140 155 175 150
Standard Rent [B] NA 100 135 180 165 1201
1st RER [Lower of (A) and (B)] 120 100 135 155 165 120
2nd ARR 100 110 135 175 200 1001
3rd Gross Annual Value 120 110 135 175 200 120
1.
In case of H6, previous year period is of 10 months, which denotes that construction or acquisition of such house
property was completed on 1st of June of the previous year, therefore, Municipal Value, Fair Rent and Standard
Rent has been proportionately reduced.

Illustration 39 : [When there is unrealised rent but no vacancy period]


Find out the gross annual value in case of the following properties let out throughout the
previous year for the assessment year 2024-25 : (₹ in ‘000)

Particulars H1 H2 H3 H4 H5
Municipal annual value 90 500 30 100 315

The Institute of Cost Accountants of India 145


Direct Taxation

Fair rent 300 300 300 300 300


Standard rent under the Rent Control Act 50 800 240 250 500
Actual rent receivable p.a. 120 600 180 360 150
Unrealised rent of the P.Y. 2023-24 (in terms of months) 2 3 1 3 2
Solution :

Computation of gross annual value : (₹ in ‘000)


Steps Particulars H1 H2 H3 H4 H5
Calculation of RER
Gross Municipal Value 90 500 30 100 315
Fair Rent 300 300 300 300 300
1st
Higher of the above [A] 300 500 300 300 315
Standard Rent [B] 50 800 240 250 500
Reasonable Expected Rent [lower of A and B] [C] 50 500 240 250 315
Calculation of (ARR – Unrealised Rent)
Actual rent receivable p.a. 120 600 180 360 150
2 nd
Unrealised rent 20 150 15 90 25
ARR – Unrealised Rent [D] 100 450 165 270 125
3rd Gross Annual Value (being higher of step 1 and step 2) 100 500 240 270 315
Assume, conditions prescribed under Rule 4 being satisfied.

Illustration 40 : [When there is vacancy period but no unrealised rent]


Find out the Gross annual value in case of the following properties : (₹ in 000)

Particulars H1 H2 H3 H4 H5 H6
Gross Municipal Value p.a. 200 300 400 500 300 300
Fair rent p.a. 300 600 750 180 200 400
Standard rent under the Rent Control Act p.a. 300 180 280 225 250 240
Actual rent p.a. 600 900 300 240 216 240
Property remains vacant (in number of month) 1 3 2 1 2 1

Solution :

Computation of Gross Annual Value : (₹ in ‘000)

Step Particulars Working H1 H2 H3 H4 H5 H6


1st Calculation of RER Higher of GMV and FR (RER 300 180 280 225 250 240
cannot exceed SR)
2nd ARR For the period actually let out 550 675 250 220 180 220
3rd Higher of above Higher of Step 1 & Step 2 550 675 280 225 250 240
4th Gross Annual Value 5501 6751 2502 2202 2503 2204

146 The Institute of Cost Accountants of India


Heads of Income

1. In H1 and H2 Actual rent receivable is already higher than RER therefore vacancy period is not making any
impact (i.e. step 4 of computation discussed earlier in theory) on GAV.
2. In H3 & H4, ARR is less than RER due to vacancy (otherwise ARR would have been ₹ 3,00,000 & ₹ 2,40,000
respectively). Therefore, GAV will be the ARR computed in step 2.
3. In H5, ARR is less than RER not only due to vacancy but also due to other factors. In such case, value of
RERshall be taken as GAV.
4. In H6, ARR is less than RER due to vacancy period otherwise ARR would have been equal to RER.

Illustration 41 :
X owns a house property in Pune, details relating to which are Municipal value ₹ 2,00,000 p.a., Fair rent ₹ 1,80,000
p.a., Standard rent ₹ 2,10,000 p.a. It is let out throughout the previous year (rent ₹ 10,000 p.m. up to 15/10/2023
and ₹ 12,000 p.m. thereafter). The property is transferred by X to Y on February 28, 2024. However, Y failed to
recover rent for March, 2024. Find gross annual value of the property in the hands of X and Y for the A.Y. 2024-25
Solution :
Since the property being sold to Y on 28/02/2024 hence previous year period for X is 11 months and for Y is 1
month, accordingly gross annual value shall be :

X Y
Steps Particulars
(11 months) (1 month)
Fair rent (FR) 1,65,000 15,000
Municipal Value 1,83,333 16,667
Higher of the above [A] 1,83,333 16,667
Standard Rent (SR) [B] 1,92,500 17,500
1st RER [Lower of A and B] [C] 1,83,333 16,667
2nd ARR – Unrealised Rent [D] 1,19,000# Nil1
3rd Gross Annual Value [Higher of C and D] 1,83,333 16,667
#
Calculation of Rent Receivable for 11 months 1.
₹ 12,000 – ₹ 12,000
[(₹ 10,000 * 6.5 months) + (₹ 12,000 * 4.5 months)] = ₹ 1,19,000

Illustration 42 : [When there is unrealised rent as well as vacancy period]


Find out the gross annual value in respect of the following properties for the A.Y. 2024-25 : (₹ in ‘000)

Particulars H1 H2 H3
Gross Municipal value 150 180 120
Fair rent 140 140 240
Standard rent 120 240 300
Actual rent if property is let out throughout the previous year 2023-24 180 300 150
Unrealised rent of the previous year 2023-24 25 40 20
Unrealised rent of the year prior to the previous year 2022-23 30 50 60
Period when the property remains vacant (in number of months) 3 1 -

The Institute of Cost Accountants of India 147


Direct Taxation

Solution :
Working: Calculation of ARR – Unrealised Rent
H1 : [{(1,80,000/12) * 9} – 25,000] = ₹ 1,10,000
H2 : [{(3,00,000/12) * 11} – 40,000] = ₹ 2,35,000
H3 : [1,50,000 – 20,000] = ₹ 1,30,000
Computation of Gross Annual Value : (₹ in ‘000)

Step Particulars Working H1 H2 H3


1st Calculation of RER Higher of GMV and FR (RER 120 180 240
cannot exceed SR)
2nd ARR less current year unrealized Working 1 110 235 130
rent (for let out period only)
3rd Higher of above Higher of Step 1 & Step 2 120 235 240
4th Gross Annual Value 1101 235 2402
1.
In H1, till step 3 ARR is less than RER due to vacancy [otherwise ARR would have been ₹ 1,55,000 (being ₹
1,80,000 – ₹ 25,000). Therefore, GAV will be the ARR computed in step 2.
2.
In H3 there is no vacancy hence step 3 gives GAV.
Illustration 43 :
Find out the gross annual value in the following cases for the A.Y. 2024-25 :
Particulars H1 (₹) H2 (₹)
Situated at Chandigarh Amritsar
Municipal value p.a. 60,000 60,000
Fair rent p.a. 70,000 70,000
Standard rent under the Rent Control Act p.a. 80,000 50,000
Actual rent p.m.:
From April 1, 2023 to July 31, 2023 5,000 6,000
From October 1, 2023 to February, 2024 9,000 8,500
For the remaining period properties were vacant.
Solution :
Working 1) Actual rent
For H1 [(5,000 * 4) + (9,000 * 5)] = ₹ 65,000 For H2 [(6,000 * 4) + (8,500 * 5)] = ₹ 66,500
Computation of gross annual value
(₹ in ‘000)
Step Particulars Working H1 H2
1st RER Higher of GMV and FR (RER cannot exceed SR) 70 50
2nd ARR Working 1 65 66.5
3rd Higher of above Higher of Step 1 & Step 2 70 66.5
4th Gross Annual Value 651 66.52

148 The Institute of Cost Accountants of India


Heads of Income

1 In H1, ARR is less than RER due to vacancy [otherwise ARR would have been ₹ 86,667 {being (₹ 65,000/9) *
12}]. Therefore, GAV will be the ARR computed in step 2.
2 In H2, Actual rent receivable is not less than RER, therefore vacancy period is not making any impact (i.e. step 4
of computation discussed earlier in theory) on GAV.
Illustration 44 :
Find out the gross annual value in respect of the following properties : (₹ in thousands)
Particulars H1 H2 H3
Value determined by the Municipality for determining Municipal tax 500 800 600
Rent of the similar property in the same locality 400 900 600
Rent determined by the Rent Control Act 700 720 700
Actual rent receivable 350 540 600
Unrealised rent of the previous year 2023-24 10 Nil 150
Period when the property remains vacant (in number of months) 5 3 2
Solution :
Computation of Gross Annual Value : (₹ in thousands)
Step Particulars Working H1 H2 H3
Higher of GMV and FR (RER
1st RER 500 720 600
cannot exceed SR)
2nd ARR less Unrealised rent 340 540 450
3rd Higher of above Higher of Step 1 & Step 2 500 720 600
(ARR less Unrealised rent) if there [{ARR/(12 – vacancy period)} *
Working 570
would have been no vacancy 12] - Unrealised rent 590 720
Is value of Step 2 less than step 1
Yes Yes No
due to vacancy
If yes than step 2 will be GAV
4th Gross Annual Value 340 540 600
otherwise step 3 shall be GAV

Test Yourself
1. Find out the Gross Annual value in respect of the following properties for the A.Y. 2024-25
(₹ in thousands)
Particulars H1 H2 H3
Gross Municipal value 150 180 120
Fair rent 140 140 240
Standard rent 120 240 300
Actual rent if property is let out throughout the previous year 2023-24 180 300 150
Unrealised rent of the previous year 2023-24 25 40 20
Unrealised rent of the previous year 2021-22 30 50 60
Period when the property remains vacant (in number of months) 3 1 -
Hints
1. H1: ₹ 1,10,000; H2: ₹ 2,35,000; H3: ₹ 2,40,000

The Institute of Cost Accountants of India 149


Direct Taxation

2.2.6 Taxes levied by local authority (Municipal Tax) [Proviso to Sec. 23(1)]
Tax levied by the municipality or local authority is deductible from Gross Annual Value (GAV). As per sec. 27(vi),
taxes levied by a local authority in respect of any property shall include service taxes levied by such local authority
in respect of such property. Municipal tax includes Service taxes like fire tax, water tax, etc. levied by a local
authority.
Such taxes shall be computed as a % of Net Municipal Value and allowed as deduction subject to the following
conditions :
1. It should be actually paid during the previous year.\
2. It must be paid by the assessee.
Taxpoint : Unpaid municipal tax or municipal tax paid by tenant shall not be allowed as deduction.
3. It must be related to the previous year or any year preceding the previous year.
Taxpoint :
Tax paid to foreign local If property is situated in a foreign country, tax paid to foreign local authority shall
authority be allowed as deduction
Tax exceeds GAV In case municipal tax paid includes tax paid for several past years and the total
(Negative NAV) amount of tax so paid by the owner exceeds GAV, then Net Annual Value (NAV)
can be negative.
Refund of tax Refund of Municipal tax paid for a property is not taxable u/s 22.
Advance Municipal Tax Municipal tax paid in advance is not allowed, as the Act provides that “the taxes
paid by the assessee levied by any local authority in respect of property shall be deducted, irrespective
of the previous year in which the liability to pay such taxes was incurred by the
owner.” [Proviso to sec. 23 of Income tax Act, 1961]
As per the above language it is construed that for claiming deduction in respect
of municipal tax, such tax must have already been levied by the local authority.
Hence payment of municipal tax in advance (liability in respect of which has not
yet incurred) shall not be allowed as deduction in the year of payment.

Illustration 45 :
Compute net annual value with the following details for the A.Y. 2024-25 :
Particulars H1 H2 H3 H4 H5 H6
Situated at Patna Anand Hyderabad Balurghat Jodhpur Etawa
Municipal Value ₹ 1,00,000 ₹ 2,00,000 ₹ 3,00,000 ₹ 4,00,000 ₹ 4,25,000 ₹ 6,00,000
Gross Annual Value ₹ 1,00,000 ₹ 2,50,000 ₹ 1,80,000 ₹ 5,00,000 ₹ 8,00,000 ₹ 5,00,000
Municipal tax for P.Y. ₹ 5,000 10% 5% 20% 12% 10%
Sewerage tax - 5% ₹ 1,000 3% ₹ 3,750 ₹ 1,000
Water Tax - 3% 5% 2% 5% -
Additional information :
a. In case of H3, municipal tax paid for the financial year 1995-96 to 2022-23 is ₹ 2,00,000.
b. In case of H4, municipal tax paid for the financial year 2024-25 is ₹ 3,000.
c. In case of H6, all taxes charged by municipality are paid to the extent of 80% (50% by owner and 30% by
tenant).

150 The Institute of Cost Accountants of India


Heads of Income

Solution :
Computation of Net Annual Value for A.Y. 2024-25 : (Amount in ₹)

Particulars H1 H2 H3 H4 H5 H6
Gross Annual Value (a) 1,00,000 2,50,000 1,80,000 5,00,000 8,00,000 5,00,000
Less : (i) Municipal Tax 5,000 20,000 2,15,0003 80,0001 51,000 30,0002
(ii) Sewerage tax - 10,000 1,000 12,000 3,750 5002
(iii) Water Tax - 6,000 15.000 8,000 21,250 -
Total (b) 5,000 36,000 2,31,000 1,00,000 76,000 30,500
Net Annual Value [(a) – (b)] 95,000 2,14,000 (-), 51,000 4,00,000 7,24,000 4,69,500
Municipal tax is calculated on municipal value.
Notes :
1. Though municipal tax is allowed on cash basis (only if paid by owner) but advance municipal tax is not
allowed.
2. Municipal tax paid by tenant is not allowed as deduction.
3. ₹ 2,00,000 (being municipal tax of past years paid during the year) + 5% of ₹ 3,00,000 (municipal tax of
current year paid during the year).

2.2.7 Deductions u/s 24

The list of deduction u/s 24 is exhaustive i.e., no deduction can be claimed in respect of expenditures which are
not specified under this section e.g., no deduction is allowed for repairs, collection charges, insurance, ground rent,
land revenue, etc.
1. Standard deduction u/s 24(a)
30% of the net annual value is allowed as standard deduction in respect of all expenditures (other than interest
on borrowed capital) irrespective of the actual expenditure incurred.
Note : Where NAV is negative or zero, standard deduction u/s 24(a) is not available.
Test Yourself
1. Calculate standard deduction available in the following cases (₹ in ‘000)
Particulars H1 H2 H3 H4 H5
Gross Annual Value 80 60 30 20 50
Municipal Tax paid 10 20 10 00 10
Repair Charges 2 20 - 5 -
Rent Collection Charges 5 10 2 20 -
Hints
1. H1: ₹ 21,000; H2: ₹ 12,000; H3: ₹ 6,000; H4: ₹ 6,000; H5: ₹ 12,000

The Institute of Cost Accountants of India 151


Direct Taxation

2. Interest on loan or borrowed capital u/s 24(b)


Interest payable on amount borrowed for the purpose of purchase, construction, renovation, repairing,
extension, renewal or reconstruction of house property can be claimed as deduction on accrual basis.
For the purpose of calculation, interest on loan is divided into two parts :

In nutshell, tax treatment is as under :

Particulars Pre-construction period Post-construction period


Starts from The day of commencement of construction or the The first day of the previous year in
day of borrowing, whichever is later which construction is completed
Ends on March 31 immediately prior to the year of completion When loan is fully paid
of construction
Tax treatment The interest incurred during aforesaid period shall The interest expenses for the year
be accumulated and allowed as deduction in 5 (on accrual basis) shall be allowed
equal installments from the year of completion of as deduction in the respective year.
construction.
Other Points :
a. Interest on borrowed capital is allowed on accrual basis even if the books of account are kept on cash basis.
b. Interest paid on fresh loan, which is taken to repay the original loan (being taken for above-mentioned purpose)
shall be allowed as deduction.
c. Interest on new loan, taken for paying outstanding interest on old loan, is not deductible

152 The Institute of Cost Accountants of India


Heads of Income

d. Amount paid as brokerage or commission, for arrangement of the loan, is not deductible.
e. Interest on loan taken for payment of municipal tax, etc. is not allowed as deduction.

Amount not deductible from Income from house property [Sec. 25]
Any interest chargeable under this Act which is payable outside India, is not allowed as deduction if :
 on such interest, tax has not been deducted at source and paid as per the provision of chapter XVIIB; and
 in respect of such interest there is no person in India who may be treated as an agent u/s 163.

Illustration 46 :
Following information are provided by an assessee for his house properties for computing interest on loan allowed
u/s 24(b) :

Particulars HP1 HP2 HP3 HP4 HP5


a) Interest on loan taken for repair of H.P. 20,000 30,000 10,000 15,000 25,000
b) Interest on loan taken for purchasing H.P. (50% 20,000 25,000 30,000 17,000 18,000
paid)
c) Interest on new loan taken for repaying old loan 10,000 12,000 13,000 14,000 16,000
which was taken for purchasing H.P.
d) Interest on loan taken for payment of interest on 10,000 10,000 10,000 10,000 10,000
earlier loan
e) Interest on loan for payment of Municipal tax 2,000 2,000 2,000 2,000 2,000
f) Interest on loan by mortgaging HP3 for business -- -- 5,000 -- --
purpose
g) Interest on loan for reconstruction of HP1 paid 20,000 -- -- -- --
outside India without deducting tax at source
h) Interest on loan for reconstruction of HP2 -- 20,000 -- -- --
payable outside India on which TDS has not
been deducted and no payment yet been made
i) Interest on loan on mortgage of HP1 for 10,000 -- -- -- --
renovation of HP2
Solution :
Calculation of ‘Interest on loan’ allowed u/s 24(b) : (₹ in ‘000)

Particulars Note HP1 HP2 HP3 HP4 HP5


a) Interest on loan taken for repair of H.P. Allowed 20 30 10 15 25
b) Interest on loan taken for purchasing 1 20 25 30 17 18
H.P.
c) Interest on new loan taken for repaying Allowed 10 12 13 14 16
old loan which was taken for purchasing
H.P.
d) Interest on loan taken for payment of Not - - - - -
interest on earlier loan
Allowed

The Institute of Cost Accountants of India 153


Direct Taxation

Particulars Note HP1 HP2 HP3 HP4 HP5


e) Interest on loan for payment of Not - - - - -
Municipal tax
Allowed
f) Interest on loan by mortgaging HP3 for 2 - - - - -
business purpose
g) Interest on loan for reconstruction 3 - - - - -
of HP1 paid outside India without
deducting tax at source
h) Interest on loan for reconstruction of 4 - - - - -
HP2 payable outside India on which
TDS has not been deducted, and no
payment yet made.
i) Interest on loan on mortgage of HP1 for 5 - 10 - - -
renovation of HP2
Interest on loan allowed u/s 24(b) 50 77 53 46 59
Notes :
1. Interest on loan is allowed on accrual basis.
2. The purpose of loan must be repair, renovation, construction or purchase of property.
3. As per sec. 25, any payment outside India without deduction of tax at source and when the payee has no agent
in India is not allowed as deduction. It is assumed that the payee has no agent in India.
4. Interest payable outside India without TDS whether paid or unpaid is not allowed as deduction. It is assumed
that the payee has no agent in India.
5. Since loan is utilised for renovation of HP2 hence deduction shall be allowed from income of HP2. Mortgage
of HP1 is irrelevant.

Illustration 47 :
Calculate interest on loan allowed for assessment year 2019-20 to 2024-25 from the following information :
Loan was taken on 1/1/2015 ₹ 5,00,000 @ 12% p.a.
Construction commenced on 1/8/2015. Construction completed on 31/3/2020. Repayment made as under :

On 1/4/2016 On 1/4/2019 On 1/4/2022 On 1/7/2023


₹ 1,00,000 ₹ 1,00,000 ₹ 1,00,000 ₹ 1,00,000
Solution :
Since construction started on 1/8/2015, hence pre-construction period starts from 1/8/2015 and since construction
completed on 31/3/2020, hence pre construction period ends on 31/3/2019 and post construction period starts from
the year 2019-20. Finally, pre construction period is from previous year 2015-16 to previous year 2018-19 and post
construction period is from previous year 2019-20 onwards. Thus, pre-construction period interest are as under :

Previous Year Amount Month Interest @ 12%


2015-16 5,00,000 8 40,000

154 The Institute of Cost Accountants of India


Heads of Income

2016-17 4,00,000 12 48,000


2017-18 4,00,000 12 48,000
2018-19 4,00,000 12 48,000
Total interest for pre-construction period 1,84,000
Total interest for pre-construction period is ₹ 1,84,000 which shall be allowed in 5 equal installments i.e., ₹ 36,800
p.a. from P.Y. 2019-20 to 2023-24.
Computation of interest :

Pre construction Post construction


Assessment Year Total Interest
period Interest period Interest
2019-20 -- -- --
2020-21 36,800 36,0001 72,800
2021-22 36,800 36,0001 72,800
2022-23 36,800 36,0001 72,800
2023-24 36,800 24,0002 60,800
2024-25 36,800 15,0003 51,800

1.
₹ 3,00,000 * 12% ₹ 2,00,000 * 12%
2.

3.
On ₹ 2,00,000 @ 12% for 3 months ₹ 6,000
On ₹ 1,00,000 @ 12% for 9 months ₹ 9,000
Total ₹ 15,000

Test Yourself
1. As on 1/1/2018, Mr. Fantoosh borrowed ₹ 5,00,000 @ 10% p.a. from State Bank of India, to be repaid in
two equal installments on 1/4/2023 and 1/4/2027. Construction completed on 17/6/2021. Due to liquidity
problem, he borrowed ₹ 3,50,000 @ 8% from his friend on 1/4/2023 and paid 1st installment of previous
loan. Calculate interest on loan allowed for assessment year 2024-25.
Hints
1. ₹ 77,500

Illustration 48 :
Mr. Rajesh owns two house properties both of which are let out. Compute his income from the following details :

Particulars H1 H2
Situated at Gaya Mumbai
Gross Municipal value 1,00,000 2,00,000
Fair rent 95,000 2,10,000
Standard rent 90,000 2,00,000
Actual rent receivable 1,00,000 1,80,000
Unrealised rent of current year 8,000 2,000

The Institute of Cost Accountants of India 155


Direct Taxation

Municipal tax 10% 1,000


Fire insurance 2,000 1,200
Repairs Nil 2,000
Interest on loan for construction (@ 12%) 10,000 Nil

Other Information :
a. Loan taken for construction is still unpaid.
b. Municipal tax of H1 is still unpaid, while, that of H2 is half paid by tenant.
Solution :
Computation of income from house property of Mr. Rajesh for the A.Y. 2024-25 :

Particulars Details Amount Amount


H1: Let out
Gross Annual Value# 92,000
Less : Municipal tax Nil
Net Annual Value 92,000
Less : Deduction u/s 24
a. Standard deduction (30% of NAV) 27,600
b. Interest on loan 10,000 37,600 54,400
H2: Let out
Gross Annual Value# 2,00,000
Less : Municipal tax 500
Net Annual Value 1,99,500
Less : Deduction u/s 24
a. Standard deduction (30% of NAV) 59,850
b. Interest on loan Nil 59,850 1,39,650
Income from House Property 1,94,050

Note : Unpaid municipal tax and municipal tax paid by tenant is not allowed.
#.
Computation of Gross Annual Value :

Particulars Details H1 H2
Reasonable Expected Rent Higher of GMV or FR subject to SR 90,000 2,00,000
Actual Rent Receivable – Unrealised Rent 92,000 1,78,000
Gross Annual Value Higher of above 92,000 2,00,000

2.2.8 Self-occupied property [Sec. 23(2)(a)]


As per sec. 23(2)(a), a house property shall be termed as self occupied property where such property or part
thereof :

156 The Institute of Cost Accountants of India


Heads of Income

 is in the occupation of the owner for the purposes of his own residence;
 is not actually let out durin g the whole or any part of the previous year; and
 no other benefit there from is derived by the owner.
Treatment : The annual value of such house or part of the house shall be taken to be nil.
If an assessee occupies more than two house properties as self-occupied, he is allowed to treat only two houses as
self-occupied at his option. The remaining self-occupied house property(ies) shall be treated as ‘Deemed to be let
out’. [Treatment of ‘deemed to be let out’ property is discussed later in this chapter.]

Taxpoint : In the light of the above provision :

Combination Treated as
Fully self occupied Self occupied property
Partly self occupied & partly vacant Self occupied property
Partly self occupied & partly let out Partly self occupied & partly let out (discussed later)
Partly self occupied & partly use for business purpose Self occupied to the extent used for self occupation

Note :

Available to Benefit u/s 23(2)(a) can be claimed by an Individual and HUF. The benefit is not
available to other assessee like company, firm, etc.
When owner want to It is not necessary that once a house property is treated as self-occupied it shall
change his option be continuously treated as self-occupied. Such option may be changed every year
without any permission.
When owner occupies When the assessee occupies his house but not in the capacity of owner then benefit
a house in some other under this section cannot be claimed. E.g. Owner let out the house to his employer &
capacity gets back the property as rent free accommodation. In such case, though the owner
himself occupies the property but as an employee of the tenant & not as an owner. In
such case, property shall be treated as let-out & not self-occupied.
When more than one If an assessee has a house property, which consist of two or more residential units
house property used in a & all such units are self-occupied used in a combined form, the annual value of the
combined form entire house shall be taken as nil as there is only one property, though it has more
than one residential unit.

Computation of taxable income of self-occupied property


Net annual value of self-occupied property shall be taken as nil. As a consequence, deduction u/s 24(a) (standard
deduction) shall also be nil. Interest on loan u/s 24(b) shall be allowed, subject to certain ceiling.
Computation at a glance :

Particulars Amount
Net Annual Value Nil
Less : Interest on borrowed capital u/s 24(b) ***
Income from house property (***)
Standard deduction u/s 24(a) is not available

The Institute of Cost Accountants of India 157


Direct Taxation

Net Annual value :


Net Annual value of two self-occupied house properties, at the choice of the assessee, is taken as nil. He can choose
those house properties as self-occupied through which tax liability can be reduced.
Normally (but not always) house property with higher gross annual value is treated as self-occupied property
but it is advised to calculate total income under the head ‘Income from house property’ by applying each option
separately and then choose the option which reduces total income.
Interest on loan u/s 24(b)
Interest on loan taken for construction, acquisition, repair, renovation or extension is allowed according to the
following table :

Maximum Interest
Conditions
allowed in aggregate
Where loan is taken on or after 1/4/1999 and following conditions are satisfied -
1. Loan is utilized for construction or acquisition of house property on or after 1-4-
1999;
2. Such construction or acquisition is completed within 5 years from the end of the
financial year in which the capital was borrowed; and
3. The lender certifies that such interest is payable in respect of the loan used for the
acquisition or construction of the house or as refinance of the earlier loan outstanding ₹ 2,00,000
(principal amount) taken for the acquisition or construction of the house.
In any other case ₹ 30,000
Taxpoint : In any case, deduction in respect of interest on loan on self-occupied properties cannot exceed
₹ 2,00,000 in a year.

Notes :
a. Calculation and deduction of interest for the period of pre and post construction, acquisition, etc. is same as
discussed in the case of let out house property.
b. Assessee shall always have nil income or loss upto ₹ 2,00,000 from properties u/s 23(2)(a).
In nutshell, treatment of interest on loan is as under :
Nature of property When loan was taken Purpose of loan Allowable
(Maximum limit)2, 3
Self-occupied On or after 01/04/1999 Construction or purchase of ₹ 2,00,000
house property1
Self-occupied On or after 01/04/1999 For Repairs of house property ₹ 30,000
Self-occupied Before 01/04/1999 Construction or purchase of ₹ 30,000
house property
Self-occupied Before 01/04/1999 For Repairs of house property ₹ 30,000
Let-out Any time Construction or purchase of No maximum limit
house property

1. Subject to other two other conditions. If other two conditions are not fulfilled, then maximum limit is restricted
to ₹ 30,000.

158 The Institute of Cost Accountants of India


Heads of Income

2. Including interest for pre-construction period.


3. Aggregate limit for all house properties treated as self-occupied.

Illustration 49 :
Mr. Pandey, owner of three houses in Chennai, furnished the following information. Compute his income from
house property for the assessment year 2024-25 :

House No. House No. House No.


Particulars 1 2 3
Self occupied Self occupied Self occupied
Standard rent under Rent Control Act 1,50,000 15,00,000 18,00,000
Municipal value 2,00,000 13,00,000 13,50,000
Fair rent 2,50,000 16,00,000 19,00,000
Municipal tax (10% of municipal value) paid
Interest on loan taken for purchases of houses 90,000 1,70,000 1,65,000
(Loan taken in P.Y. 2020-21)

Solution :
Computation of Income from House property of Mr. Pandey for the A.Y. 2024-25 :

Particulars Details Details Amount


House 1: Deemed to be Let out
Gross Annual Value (Working) 1,50,000
Less : Municipal Tax (10% of ₹ 2,00,000) 20,000
Net Annual Value (NAV) 1,30,000
Less : Deduction u/s
24(a) Standard Deduction (30% of NAV) 39,000
24(b) Interest on loan 90,000 1,29,000 1,000
House 2 & 3 : Self occupied
Net Annual Value Nil
Less : Deduction u/s
24(b) Interest on loan [(₹1,70,000 + ₹ 1,65,000), subject to max. of 2,00,000 (2,00,000)
₹ 2,00,000]
Income from House Property (1,99,000)

Working :
Computation of Gross Annual Value of House 1 :

Particulars House 1
Municipal Value (A) 2,00,000
Fair Rent (B) 2,50,000

The Institute of Cost Accountants of India 159


Direct Taxation

(C) = Higher of (A) and (B) 2,50,000


Standard Rent (D) 1,50,000
Gross Annual Value [Lower of (C) and (D)] 1,50,000

Illustration 50 :
Sri Jayram has a house property used for own residence for 9 months and for remaining 3 months of the previous
year, it was unused. Gross Municipal value of the property ₹ 6,00,000 p.a. Fair Rent ₹ 5,00,000, Standard Rent
₹ 4,00,000. He incurred repair expenditure of ₹ 10,000 & paid municipal tax ₹ 5,000 during the year. Compute
income from house property in the following cases for the A.Y. 2024-25 :
1. He borrowed ₹ 1,00,000 @ 12% (simple interest) on 17/8/1998 for purchase of the house property and such
amount as well as interest is still unpaid.
2. He borrowed ₹ 10,00,000 @ 12% (simple interest) on 17/8/1998 for purchase of the house property and such
amount as well as interest is still unpaid.
3. He borrowed ₹ 5,00,000 @ 12% (simple interest) on 17/8/1999 for construction of the house property,
construction of which was completed on 31/3/2000 and such amount is still unpaid.
4. He borrowed ₹ 20,00,000 @ 18% (simple interest) on 17/8/1999 for construction of the house property,
construction of which was completed on 31/3/2000 and such amount is still unpaid.
5. He borrowed ₹ 1,80,000 @ 15% on 1/4/1998 and further borrowed ₹ 10,00,000 @ 10% on 17/8/1999 for
construction of the house property and such amount is still unpaid. Construction completed on 1/2/2000.
6. He borrowed ₹ 5,00,000 @ 12% on 1/4/2001 for repairs of the house property.
7. He borrowed ₹ 1,80,000 @ 15% on 1/4/1998 and further borrowed ₹ 20,00,000 @ 14% on 17/8/1999 for
construction of the house property and such amount is still unpaid. Construction completed on 1/2/2000.
Solution :
Computation of income from house property in different cases :

Particulars Case 1 Case 2 Case 3 Case 4 Case 5 Case 6 Case 7


Net Annual Value Nil Nil Nil Nil Nil Nil Nil
Less : Deduction u/s
24(b) Interest on Loan 12,000 30,000 60,000 2,00,000 1,27,000 30,000 2,00,000
Income from House
(12,000) (30,000) (60,000) (2,00,000) (1,27,000) (30,000) (2,00,000)
Property
Notes - 1 2 3 4 5 6

Notes :
1. As loan was taken before 1/4/1999 hence, the maximum ceiling is ₹ 30,000.
2. As loan was taken on or after 1/4/1999 for the construction of house property, which is completed within 5
years from the end of financial year in which such loan was taken, hence the maximum ceiling is enhanced to
₹ 2,00,000. It is assumed that required certificate has been furnished.
3. Maximum ceiling is ₹ 2,00,000.
4. If loan was taken before 1/4/1999 as well as on or after 1/4/1999 then the total interest allowed in aggregate

160 The Institute of Cost Accountants of India


Heads of Income

cannot exceed ₹ 2,00,000. However, the limit for interest allowed in respect of loan taken prior to 1/4/1999
shall be ₹ 30,000. Since the first loan is taken on 1/4/1998 and construction completed on 1/2/2000. Hence the
pre-construction period is 1998-99, for which interest ₹ 27,000 (i.e. ₹ 1,80,000 * 15%) shall be allowed in 5
equal installments i.e. ₹ 5,400 every year. However, such pre-construction period interest is allowed only for 5
years i.e. from 1999-2000 to 2003-2004, therefore such interest shall not be allowed in subsequent year. Total
eligible interest on first loan is ₹ 27,000 (i.e. Pre-construction period interest Nil + Post construction period
interest ₹ 27,000). Further eligible interest on second loan is ₹ 1,00,000 (i.e. 10% of ₹ 10,00,000). Hence the
total allowable interest u/s 24(b) shall be ₹ 1,27,000.
5. The enhanced limit is only for construction or acquisition of house property, here the loan is taken for repair
purpose for which maximum ceiling is ₹ 30,000.
6. If loan was taken before 1/4/1999 as well as on or after 1/4/1999 then the total interest allowed in aggregate
cannot exceed ₹ 2,00,000. However, the limit for interest allowed in respect of loan taken prior to 1/4/1999
shall be ₹ 30,000. Since the first loan is taken on 1/4/1998 and construction completed on 1/2/2000. Hence the
pre-construction period is 1998-99, for which interest ₹ 27,000 (i.e., ₹ 1,80,000 * 15%) shall be allowed in 5
equal installments i.e. ₹ 5,400 every year. However, such pre-construction period interest is allowed only for 5
years i.e. from 1999-2000 to 2003-2004, therefore such interest shall not be allowed in subsequent years. Total
eligible interest on first loan is ₹ 27,000 (i.e., Pre-construction period interest Nil + Post construction period
interest ₹ 27,000), i.e. within the ceiling of ₹ 30,000. Further eligible interest on second loan ₹ 2,80,000 (i.e.
14% of ₹ 20,00,000). However, the total ceiling of interest in case of self occupied property cannot exceed ₹
2,00,000, hence the total allowable interest u/s 24(b) shall be ₹ 2,00,000.

Illustration 51 : [Rent free accommodation –vs.- Self occupied house property]


Miss Anjali owns a house property, which is let out, to her employer company for a monthly rent of ₹ 20,000.
Company allotted the same house to Miss Anjali as rent-free accommodation. Municipal tax paid ₹ 20,000, interest
on loan paid ₹ 90,000. Comment on tax treatment under the head income from house property.
Solution :
Though Miss Anjali owns the house and uses it for self-occupation as well, still she cannot treat the said house
property as self occupied. This is because she uses the property not as owner but as an employee.
Hence, rent received by her from employer shall be taxable as under :

Particulars Details Amount


Gross Annual Value (20,000 * 12) 2,40,000
Less : Municipal tax 20,000
Net Annual Value 2,20,000
Less : Deduction u/s
24(a) Standard deduction 66,000
24(b) Interest on loan 90,000 1,56,000
Income from house property 64,000

Note: Since the assessee receives the benefit of rent-free accommodation hence she will be further taxed under the
head ‘Salaries’ for perquisite being rent-free accommodation.

The Institute of Cost Accountants of India 161


Direct Taxation

2.2.9 Unoccupied property [Sec. 23(2)(b)]


Where an assessee has a residential house (kept for self-occupation) and it cannot actually be occupied by the
owner owing to his employment, business or profession carried on at any other place and hence he has to reside at
that place in a building not belonging to him, such house shall be termed as unoccupied property.
Taxpoint :

 Assessee has a residential house kept for self-occupation.


 The house cannot be occupied by the owner owing to his employment and no other benefit is derived from
such house. In case house remains unoccupied by the owner owing to his personal convenience, then no
benefit under this section shall be allowed
 He has to reside in a house not belonging to him, whether rent is paid for that house or not.
Treatment : Same as self occupied property.
Computation at a glance

Particulars Amount
Net Annual Value Nil
Less : Interest on borrowed capital u/s 24(b) ***
Income from house property (***)

Notes :
a. An assessee cannot claim benefit u/s 23(2)(a) as well as 23(2)(b) in the same previous year.
b. An assessee can claim benefit u/s 23(2)(b) even though he has other house properties.

2.2.10 Deemed to be let-out house property [Sec. 23(4)]


Where the assessee occupies more than two house properties as self-occupied or has more than two unoccupied
properties, then for any two of them, benefit u/s 23(2) can be claimed (at the choice of the assessee) and remaining
property or properties shall be treated as ‘deemed to be let out’.
Treatment :

1. Gross Annual value: Since assessee does not let out such property & do not receive rent, therefore GAV will
be determined from Step 1 only. Step 2, 3 & 4 of calculation GAV are irrelevant.
Taxpoint : GAV of deemed to be let out property will be the ‘Reasonable expected rent (RER)’of the property.
2. Municipal taxes and deduction u/s 24(a) and 24(b) shall be available as in the case of let out house property.

Illustration 52 :
Compute income under the head ‘Income from house property’ of Sri from the following information :

Particulars H1 H2 H3 H4
Used for Self occupied Self occupied Self occupied Own Business
Situated at Mumbai Abu Kolkata Hyderabad
Gross Municipal Value 3,00,000 2,00,000 7,00,000 3,00,000

162 The Institute of Cost Accountants of India


Heads of Income

Fair Rent 2,00,000 2,00,000 6,00,000 1,20,000


Standard Rent 3,00,000 2,40,000 7,00,000 2,00,000
Municipal Tax 15% 15% 15% 15%
Repairs 13,000 4,000 8,000 8,000
Ground Rent 20,000 Nil Nil 6,000
Land Revenue Nil 10,000 Nil Nil
Interest on Loan 40,000 10,000 2,10,000 20,000
Loan taken on 1998-99 1998-99 2019-20 1999-00

Solution :
In the given case, there are two options :
Option 1 : Take H1 & H3 as Self-Occupied (S/O) and H2 as Deemed to be Let-Out (DLO)
Option 2 : Take H1 as Deemed to be Let-Out (DLO) and H2 & H3 as Self-Occupied (S/O)
Option 3 : Take H3 as Deemed to be Let-Out (DLO) and H1 & H2 as Self-Occupied (S/O)
Total income under the head house property shall be computed applying each option separately and then the option,
which yields least income under this head, shall be opted.

Option1 Option2 Option3


Particulars H1 & H3 H2 DLO H1 DLO H2 & H3 H3 DLO H1 & H2
S/O S/O S/O
Gross Annual Value Nil 2,00,000 3,00,000 Nil 7,00,000 Nil
Less : Municipal Tax (15% of Nil 30,000 45,000 Nil 1,05,000 Nil
Municipal value)
Net Annual Value (A) Nil 1,70,000 2,55,000 Nil 5,95,000 Nil
Less : Deduction u/s
24(a) Standard deduction (30% Nil 51,000 76,500 Nil 1,78,500 Nil
of NAV)
24(b) Interest on loan 2,00,0002 1,00,000 40,000 2,00,0002 2,10,000 30,0001
Total deduction (B) 2,00,000 1,51,000 1,16,500 2,00,000 3,88,500 30,000
Income from house property (-) 2,00,000 19,000 1,38,500 (-) 2,00,000 2,06,500 (-) 30,000
[(A) – (B)]
Income from house property (-) 1,81,000 (-) 61,500 1,76,500

Notes :
1. In case of H1 & H2 loan was taken prior to 1/4/1999.
2. Since loan was taken for construction on or after 1/4/1999.
3. Since H4 is used for own business purpose so it is not taxable under this head.
Total income under the head Income from house property as per option 1 is (-) ₹ 1,81,000
Computation of Income from house property of Sri for the A.Y. 2024-25 :

The Institute of Cost Accountants of India 163


Direct Taxation

Particulars Details Details Amount


H1 & H3 : Self-occupied u/s 23(2)(a)
Net Annual Value Nil
Less : Deduction u/s
24(b) Interest on loan
- For H1 (Max Limit) 30,000
- For H2 (Max Limit) 2,00,000
Subject to maximum of ₹ 2,00,000 2,30,000 2,00,000 (2,00,000)
H2 : Deemed to be let out u/s 23(4)
Gross Annual Value 2,00,000
Less : Municipal Tax 30,000
Net Annual Value 1,70,000
Less : Deduction u/s
24(a) Standard Deduction (30% of NAV) 51,000
24(b) Interest on loan 1,00,000 1,51,000 19,000
Income from house property (1,81,000)

2.2.11 Partly self-occupied and partly let-out [Sec. 23(3)]


Where a house or part of the house, which is self-occupied, is let out during any part of the previous year, such
property is termed as ‘Partly self-occupied and partly let out’. Further, such division may be made in the following
ways :

1) Area wise division 2) Time wise division 3) Area as well as Time wise division

Case 1) Area wise Division


In this case, a house property consists of two or more independent units and one or more of which are self-occupied
and remaining units are let out.
Treatment :
 Self-occupied portion & let out portion shall be treated as two separate house (i.e. Unit A & Unit B);
 Common value like municipal value, fair rent, standard rent, municipal tax and interest shall be
proportionately divided;
 Income of both units shall be computed separately.
Illustration 53 :
Miss Paro has a house property having two separate residential units (unit A covering 40% of total area and unit B
covering 60% of total area). Unit A is self-occupied by the assessee and unit B is let out to Sri Devdas for a monthly
rent of ₹ 3,000. With the following further information, compute her taxable income from house property :
Municipal Value ₹ 1,00,000 Municipal Tax 10%
Fair Rent ₹ 1,20,000 Interest on Loan ₹ 30,000
Standard Rent ₹ 2,00,000 Annual charge ₹ 5,000

164 The Institute of Cost Accountants of India


Heads of Income

Solution :
Computation of Income from house property of Miss Paro for the A.Y. 2024-25 :

Unit A Unit A
Particulars Working
Details Amount Details Amount
Gross Annual Value 1 Nil 72,000
Less : Municipal Tax Nil 6,000
Net Annual Value Nil 66,000
Less : Deduction u/s
24(a) Standard Deduction Nil 19,800
24(b) Interest on loan 40:60 12,000 12,000 18,000 37,800
Income from house property (-) 12,000 28,200
Income under the head ‘Income from house property’ 16,200

Working 1 : Computation of Gross Annual Value (GAV) :

Steps Particulars Working Unit A Unit B


Municipal Value 40:60 40,000 60,000
Fair Rent 40:60 48,000 72,000
Standard Rent 40:60 80,000 1,20,000
1. RER Higher of MV & FR (RER cannot exceed SR) NIL (As S/O) 72,000
2. ARR ₹ 3,000 * 12 - 36,000
3. Gross Annual Value Higher of Step 1 & 2 - 72,000

(Case 2) Time wise division - In such case, the house property is self occupied by the assessee for a part of the year
and let out for remaining part of the year.
Treatment :
In such case, assessee will not get deduction for the self-occupied period and income will be computed as if the
property is let out throughout the year. In this regard, it is to be noted that the reasonable expected rent (RER) shall
be taken for the full year but the actual rent receivable (ARR) shall be taken only for the let-out period.
Illustration 54 :
Mr. Rana used his house property for self-occupation till 1/8/2023 and let out the same for remaining period for
rent of ₹ 6,000 p.m. Compute his income from house property from the following details:
Municipal value ₹ 1,00,000, Fair Rent ₹ 80,000, Standard Rent ₹ 96,000, Municipal tax 16%, Interest on loan
₹ 10,000

The Institute of Cost Accountants of India 165


Direct Taxation

Solution :
Computation of income from house property of Mr. Rana for the A.Y. 2024-25 :

Particulars Working Details Amount


Municipal Value 1,00,000
Fair Rent 80,000
Standard Rent 96,000
Reasonable Expected Rent Higher of MV & FR (RER cannot exceed SR) 96,000
Actual Rent Receivable ₹ 6,000 * 8 48,000 96,000
Gross Annual Value Higher of RER and ARR 16,000
Less : Municipal Tax 16% of Municipal Value 80,000
Net Annual Value
Less : Deduction u/s
24(a) Standard deduction 30% of NAV 24,000
24(b) Interest on loan 10,000 34,000
Income from house property 46,000

Illustration 55 :
How shall your answer differ if in the above illustration, property is let out to tenant from 1/4/2023 to 1/12/2023
and from 1/12/2023 to 1/3/2024, it was self-occupied. Standard rent of such property is ₹ 50,000.
Solution :
Computation of Income from house property of Mr. Rana for the A.Y. 2024-25 :

Particulars Working Details Amount


Municipal Value 1,00,000
Fair Rent 80,000
Standard Rent 50,000
Reasonable Expected Rent Higher of MV & FR (RER cannot exceed SR) 50,000
Actual Rent Receivable ₹ 6,000 * 8 48,000
Higher of RER and ARR 50,000
However, ARR is less than RER due to vacancy period [otherwise ARR would have
been ₹ 54,000 (being ₹ 6,000 * 9)] therefore ARR shall be treated as GAV.
Gross Annual Value 48,000
Less : Municipal Tax 16% of MV 16,000
Net Annual Value 32,000
Less : Deduction u/s
24(a) Standard Deduction 30% of NAV 9,600
24(b) Interest on Loan 10,000 19,600
Income from house property 12,400

166 The Institute of Cost Accountants of India


Heads of Income

Illustration 56 :
Miss Rani used her house property for self-occupation till 1/9/2023 and let out the same for remaining period for
rent of ₹ 6,000 p.m. Municipal tax paid ₹ 5,000, interest on loan accrued ₹ 10,000. Compute her taxable income
from house property.
Solution :
Computation of income from house property of Miss Rani for the A.Y. 2024-25 :
Steps Particulars Working Details Amount
Fair Rent (Note) ₹ 6,000 * 12 72,000
1. Reasonable Expected Rent Fair Rent 72,000
2. Actual Rent Receivable ₹ 6,000 * 7 42,000
3. Gross Annual Value Higher of RER and ARR 72,000
Less : Municipal Tax 5,000
Net Annual Value 67,000
Less : Deduction u/s
24(a) Standard Deduction 30% of NAV 20,100
24(b) Interest on Loan 10,000 30,100
Income from house property 36,900
Note : Since the property has been let out for 7 months @ ₹ 6,000 p.m., therefore fair rent of the property is ₹
72,000 (being ₹ 6,000 * 12).

Case 3) Area as well as Time wise Division


Merger of Case 1 and Case 2

Illustration 57 :
Mr. Ajnabi has a house property in Cochin. The house property has two equal dimension residential units. Unit 1 is
self occupied throughout the year and unit 2 is let out for 9 months for ₹ 10,000 p.m. and for remaining 3 months
it was self-occupied. Compute his taxable income from the following details :
Municipal value ₹ 2,00,000, Fair Rent ₹ 1,60,000, Standard rent ₹ 3,00,000, Municipal tax 10% (60% paid by
assessee), Interest on loan ₹ 40,000, Expenditure on repairs ₹ 20,000.
Solution :
Working
1. Computation of Gross Annual Value (GAV)

Particulars Working Unit 1 Unit 2


Municipal Value 1:1 1,00,000 1,00,000
Fair Rent 1:1 80,000 80,000
Standard Rent 1:1 1,50,000 1,50,000
Reasonable Expected Rent Higher of MV & FR (RER cannot exceed SR) Nil 1,00,000
Actual Rent Receivable ₹ 10,000 * 9 - 90,000
Gross Annual Value Higher of Step 1 & 2 Nil 1,00,000

The Institute of Cost Accountants of India 167


Direct Taxation

2. Municipal tax = 10% of ₹ 2,00,000 = ₹ 20,000 being divided in the ratio 1:1 between Unit 1 and Unit 2. Out of
such Municipal tax only 60% is paid, therefore, Municipal tax allowed as deduction in case of Unit 2 is only
₹ 6,000 [i.e. ₹ 20,000 * ½ * 60%].
3. Interest on loan is divided in unit A and unit B in 1:1 as both units are of equal dimension.

Computation of income from house property of Mr. Ajnabi for the A.Y. 2024-25

Unit 1 Unit 2
Particulars Working
Details Amount Details Amount
Gross Annual Value 1 Nil 1,00,000
Less : Municipal Tax 2 Nil 6,000
Net Annual Value Nil 94,000
Less : Deduction u/s
24(a) Standard Deduction Nil 28200
24(b) Interest on loan 3 20,000 20,000 20000 48,200
Income from house property (-) 20,000 45,800
Conclusion : Income under the head Income from house property is ₹ 25,800 (being ₹45,800 – ₹20,000).

2.2.12 Recovery of Unrealized Rent and Arrears Rent [Sec. 25A]


Applicability
The assessee has received arrears of rent received from a tenant or the unrealised rent realised subsequently from
a tenant
Tax Treatment :
The amount so received shall be taxable under the head ‘Income from house property’ in the year of receipt after
deducting standard deduction @ 30% of such amount.
Arithmetically, taxable amount shall be -

70% × [Recovery of Arrear Rent or Unrealised Rent]


Taxpoint :
 No other deduction shall be allowed from such income except standard deduction i.e. 30% of such receipt.
(even legal expenditure shall not be allowed as deduction)
 The income is taxable on cash basis.
Note : Such receipt shall be chargeable as income from house property although the assessee is not the owner of
such property in the year of receipt.

Illustration 58 :
Mr. Lucky Ali owns a house property let out since 1/4/2019 to a school for monthly rent of ₹ 10,000. There was
no change in rent till 31/3/2023. On 1/4/2023, as per court decision rent was increased to ₹ 12,000 p.m. with
retrospective effect from 1/4/2021 and duly paid by school in the same year. Legal expenditure for such suit has
been incurred by Mr. Ali ₹ 30,000. Discuss tax treatment u/s 25A.

168 The Institute of Cost Accountants of India


Heads of Income

Solution :
Arrears rent belongs to the period 1/4/2021 to 31/3/2023 i.e., for 24 months.
Arrears rent received = ₹ 2,000 * 24 months = ₹ 48,000
Such rent is taxable in the year of receipt as under :

Particulars Amount
Arrears of rent received 48,000
Less : Standard deduction u/s 24(a) equal to 30% of such rent 14,400
Income from house property u/s 25A 33,600
Note: Legal expenditure is not deductible.

Illustration 59 :
X Ltd. has two house properties both of which are vacant. Municipal value of 1st house property is ₹ 1,00,000 and
that of 2nd is ₹ 80,000. It has computed income from house property as under :

Particulars Details Amount


HP1: Self occupied [Sec. 23(2)(a)]
Net Annual Value (NAV) Nil
Less : Interest on loan u/s 24(b) Nil
Income from HP1 Nil
HP2: Deemed to be let out [Sec. 23(4)]
Gross Annual Value (GAV) 80,000
Less : Municipal tax Nil
Net Annual Value (NAV) 80,000
Less : Standard deduction u/s 24(a) @ 30% of NAV 24,000 56,000
Income from house property 56,000
Do you agree with the computation of income from house property of the assessee.
Solution :
In the above computation, X Ltd. Has claimed benefit of self-occupation, whereas, such benefit can be claimed
only by an individual or HUF. A company form of assessee cannot claim such benefit. Hence, income under the
head Income from house property will be as under :
Computation of income from house property of X Ltd. For the A.Y. 2024-25 :

Particulars Details Amount


HP1: Deemed to be let out [Sec. 23(4)]
Gross Annual Value (GAV) 1,00,000
Less : Municipal tax Nil

The Institute of Cost Accountants of India 169


Direct Taxation

Particulars Details Amount


Net Annual Value (NAV) 1,00,000
Less : Standard Deduction u/s 24(a) @ 30% of NAV 30,000 70,000
HP2: Deemed to be let out [Sec. 23(4)]
Gross Annual Value (GAV) 80,000
Less : Municipal tax Nil
Net Annual Value (NAV) 80,000
Less : Standard Deduction u/s 24(a) @ 30% of NAV 24,000 56,000
Income from house property 1,26,000

Illustration 60 :
Mr. Abul Hasan owns three houses at Ranchi. He furnishes the following particulars for the previous year 2022-23 :
House No. I : The house was constructed in 2022 and let out to a friend at a monthly rent of ₹ 10,000 upto
31.1.2024 and thereafter, it was let out at its fair rent of ₹ 15,000 per month. He has paid
₹ 15,000 as municipal taxes @ 10% of Municipal Value. He has also paid fire insurance
premium of ₹ 2,000.
House No. II : Ground floor is let out @ ₹ 20,000 p.m. first floor, identical to ground floor, is occupied by
him for his residence. Municipal taxes paid @ 20% amounted to ₹ 80,000.
House No. III : The house was constructed in 2012 and is used for his business. The annual value of this
house is ₹ 1,00,000 and he spent ₹ 5,000 as municipal taxes and ₹ 2,000 for repairs.
Other information :
A loan of ₹ 40,00,000 has been taken on 01-6-2021 for construction of House No. II. Construction of the house was
completed on 01-6-2022. He repaid the entire loan on 31-12-2023. Interest on loan is payable @ 12% p.a. Compute
his income from house property for the A.Y. 2024-25.
Solution :
Computation of Income from House Property of Mr. Abul Hasan for the A.Y. 2024-25 :

Particulars Details Details Amount


House 1 : Let out
Gross Annual Value 1,80,000
Less : Municipal Tax 15,000
Net Annual Value 1,65,000
Less : Deduction u/s
24(a) Standard Deduction 49,500
24(b) Interest on loan Nil 49,500 1,15,500
House 2 : Ground Floor (Let out)
Gross Annual Value 2,40,000

170 The Institute of Cost Accountants of India


Heads of Income

Particulars Details Details Amount


Less : Municipal Tax [50%] 40,000
Net Annual Value 2,00,000
Less : Deduction u/s
24(a) Standard Deduction 60,000
24(b) Interest on loan 2,20,000 2,80,000 (80,000)
House 2 : First Floor (Self occupied)
Net Annual Value Nil
Less : Deduction u/s
24(b) Interest on loan 2,00,000 (2,00,000)
House 3 : Used in own business Nil
Income from House Property (1,64,500)

Workings :
1 Fair Rent : Since 1st house is let out by assessee to his friend @ ₹ 10,000 p.m. and the same property is let out to
other tenant @ ₹ 15,000 p.m., this signifies that 2nd house has fair rent ₹ 15,000 * 12 = ₹ 1,80,000.
2 Calculation of Interest to be deducted in A.Y. 2024-25

Previous Year Month Interest


Pre-construction Interest
2020-21 10 4,00,000
1/5 of pre-construction (a)
th
80,000
Post-construction interest (b) [₹ 40,00,000 × 12% x 9/12] 9 3,60,000
Total interest charged (a) + (b) 4,40,000
50% for Ground Floor 2,20,000

Illustration 61 :
Sarju Middey is the owner of 2 houses in Kolkata. From the following particulars of the houses, compute his
income from house property for the assessment year 2024-25 :

House A : Let-out to an employee of the business of Sarju @ ₹ 5,000 p.m. which is necessary for the purpose
of business. Municipal tax paid ₹ 3,000 and interest on loan taken for purchasing the house
amounted to ₹ 9,000.
House B : The house consists of 3 identical flats. First flat is used by him for his own business. Second flat is
used by him for his own residence. The third flat is let out at a monthly rent of ₹ 15,000. Municipal
taxes paid @ 5% amounted to ₹ 20,250.
Other information :
a. Unrealised rent for the P.Y. 2023-24 relating to third flat of House B amounted to ₹ 10,000.

The Institute of Cost Accountants of India 171


Direct Taxation

b. A loan of ₹ 20,00,000 was taken on 01.07.2020 for construction of the House B. Construction of House B was
completed on 01.06.2022. Interest on loan is 12% p.a. No repayment was made.

Particulars Details Details Amount


Flat III : Let out
Gross Annual Value (Working 1) 1,80,000
Less : Municipal Tax 20,250/3 6,750
Net Annual Value 1,73,250
Less : Deduction u/s
24(a) Standard Deduction 51,975
24(b) Interest on loan [₹ 3,24,000 / 3] 1,08,000 1,59,975 13,275
Flat II : Self occupied
Net Annual Value Nil
Less : Deduction u/s
24(b) Interest on loan [₹ 3,24,000 / 3] 1,08,000 (1,08,000)
Income from House Property 94,725

Working 1 : Computation of Gross Annual Value

Steps Particulars Working Unit 3


Municipal Value (₹ 20,250 / 5%) / 3 1,35,000
Fair Rent ₹ 15,000 × 12 1,80,000
Standard Rent -
1 Reasonable Expected Rent Higher of MV & FR (RER cannot exceed SR 1,80,000
2 Actual Rent Receivable ₹ 15,000 × 12 – ₹ 10,000 1,70,000
3 Gross Annual Value Higher of Step 1 & 2 1,80,000
Working 2 : Calculation of Interest to be deducted in respect of House 2

Previous Year Month Interest


Pre-construction Interest
2020-21 9 1,80,000
2021-22 12 2,40,000
Total 4,20,000
1/5 of pre-construction (a)
th
84,000
Post-construction interest (b) 12 2,40,000
Total interest (a) + (b) 3,24,000

172 The Institute of Cost Accountants of India


Heads of Income

Quick MCQs:-
1. The basis of chargeability of income under the head income from house property is
(a) Rental value
(b) Annual value
(c) Value fixed by the government
(d) None of the above

2. Income from vacant plot is taxable under the head-


(a)Income from House Property
(b) Income from Other Sources
(c) Profits & Gains of Business or Profession
(d) Capital Gains

3. House Property Income is exempt for-


(a) Local Authority
(b) Charitable Trust
(c) Political Property
(c) All of the above.

4. Advance Municipal Tax paid-


(a) Shall be allowed as deduction in the year of payment
(b) can be claimed in the year in which it falls due
(c) cannot be claimed
(d) None of above

5. A has two house properties. Both are self-occupied. The Annual Value-
(a) of both house shall be Nil
(b) one house shall be Nil
(c) of no house shall be Nil
(d) Both (a) & (b)

6. “A” borrowed D 5,00,000 at 12% p.a on 01-04-2015 for construction of house property which was
completed on 15-03-2024 and let out. The amount is still unpaid. The deduction on account of interest for
the previous year 2023-24 shall be-
(a) D 60,000
(b) D 96,000
(c) D 1,80,000
(d) D 2,40,000

The Institute of Cost Accountants of India 173


Direct Taxation

7. Unrealized Rent is a deduction from-


(a) Actual Rent
(b) Net Annual Value
(c) Income from the head House Property
(d) None of above

8. Megha received D 30, 000 as arrears of rent during the previous year 2023-24. The amount taxable under
section 25A would be_______
(a) Nil
(b) D 21,000
(c) D 30,000
(d) D 25,000

9. Ms. Shilpa let out a property for D 20,000 per month during the year 2022-23. The municipal tax on the
let-out property was enhanced retrospectively. Hence, she paid D 60,000 as municipal tax which included
arrears of municipal tax of D 45,000.Her income from house property is-
(a) D 1,80,000
(b) D 1,26,000
(c) D 1,57,500
(d) D 1,36,500

10. Jaya and Vijaya are co-owners of a self-occupied property. They own 50% share each. The interest paid
by each co-owner during the previous year 2023-24 on loan (taken for acquisition of property during the
year 2004) is D 2,05,000. The amount of allowable deduction in respect of each co-owner is –
(a) D 2,05,000
(b) D 1,02,500
(c) D 2,00,000
(d) D 1,00,000

174 The Institute of Cost Accountants of India


Heads of Income

Profits and Gains of Business or


2.3
Profession
2.3.1 Meaning of Business & Profession
Business [Sec. 2(13)]
Business includes –
 any trade, commerce or manufacture; or
 any adventure or concern in the nature of trade, commerce or manufacture.
Generally, business means recurring economic activity, but for income tax purpose an isolated activity may be
termed as business depending upon facts and circumstances. Following elements shall be considered to judge a
transaction as business transaction:

¾¾ Nature of commodity ¾¾ Intention of the party ¾¾ Efforts applied in transaction

¾¾ Periodicity of transaction ¾¾ Nature of transaction (whether incidental to a business or not)

Profession [Sec. 2(36)]


Profession includes vocation. Profession requires purely intellectual skill or manual skill on the basis of some
special learning and qualification gathered through past training or experience e.g. chartered accountant, doctor,
lawyer etc. Professional skill can be acquired only after patient study (in a particular system either a college,
university or institute) and application (i.e. experience)
Vocation implies natural ability of a person to do some particular work e.g. singing, dancing, etc. The term “vocation”
is different from the term “hobby”. Vocation must have the earning feature. It can be treated as an earning means by
which a man passes his life. Unlike profession, vocation does not require a degree or special learning.

Notes
1. Profit Motive: If the motive of an activity is pleasure only, it shall not be treated as business activity.
2. Business vs Profession: An income arising out of trade, commerce, manufacture, profession or vocation
shall have the same treatment in Income tax Act. However, a little segregation is required to be made between
business and profession while applying sec. 44AA, sec. 44AB, sec. 40AD, sec. 44ADA, etc. (discussed later
in this chapter).

2.3.2 Income chargeable under the head Profits & gains of business or profession [Sec. 28]
Sec. 28 enlists the incomes, which are taxable under the head ‘Profits & gains of business or profession’:
1. Profits & gains of any business or profession [Sec. 28(i)]: Any income from business or profession including
income from speculative transaction shall be taxable under this head.

The Institute of Cost Accountants of India 175


Direct Taxation

2. Compensation to Management agency [Sec. 28(ii)]: Any compensation/other payment due to or received -

By In connection with
Any person managing the affairs of an Indian
company Termination or modification of terms and conditions of
Any person managing the affairs of any company his appointment
in India
Any person holding an agency in India for any part
Termination of agency or the modification of terms and
of the activities relating to the business of any other
conditions in relation thereto
person
The vesting in the Government or in any corporation
Any person owned/controlled by the Government, of the
management of any property or business.
The termination or the modification of the terms and
Any person
conditions, of any contract relating to his business
3. Income of trade or professional association’s [Sec. 28(iii)]: Income derived by a trade, professional or
similar association from rendering specific services to its members shall be taxable under this head.
Note: This is an exception to the general principle that a surplus of mutual association cannot be taxed.
4. Export incentive [Sec. 28(iiia) (iiib) & (iiic)]: An export incentive in form of -
¾¾ Profit on sale of import license or duty entitlement pass book. [Sec. 28(iiia)/(iiid)/(iiie)]
¾¾ Cash assistance received/receivable by an exporter under a scheme of the Government of India [Sec.
28(iiib)]
¾¾ Duty draw back (received/receivable) for export e.g. duty drawback, etc. [Sec. 28(iiic)]
5. Perquisite from business or profession [Sec. 28(iv)]: The value of any benefit or perquisite, whether
convertible into money or not, arising from business or profession shall be taxable under this head.
Example 1: If an authorized dealer of a company receives a car (over and above his commission) from the
company on achieving sale-target then market value of such car shall be taxable under the head ‘Profits &
gains of business or profession’.
6. Remuneration to partner [Sec. 28(v)]: Any interest salary, bonus, commission or remuneration received by
a partner from the firm (or Limited Liability Partnership) shall be taxable as business income in the hands of
the partner to the extent allowed in hands of firm (or Limited Liability Partnership) u/s 40(b).
7. Amount received or receivable for certain agreement [Sec. 28(va)]: Any sum, whether received or
receivable in cash or in kind, under an agreement for -
¾¾ not carrying out any activity in relation to any business or profession; or
¾¾ not sharing any know-how, patent, copyright, trade mark, licence, franchise or any other business or
commercial right of similar nature or information or technique likely to assist in the manufacture or
processing of goods or provisions for services.
Exceptions: The aforesaid provision is not applicable in respect of the following:
a. any sum received or receivable in cash or in kind on account of transfer of the right to manufacture,
produce or process any article or thing; or right to carry on any business or profession, which is chargeable
under the head Capital gains;

176 The Institute of Cost Accountants of India


Heads of Income

b. any sum received as compensation from the multilateral fund of the Montreal Protocol on Substances that
Deplete the Ozone Layer under the United Nation Environment Programme, in accordance with the terms
of agreement (whether or not in writing, whether or not intended to be enforceable by legal proceedings)
entered into with the Government of India
8. Keyman Insurance Policy [Sec. 28(vi)]: Any sum received under a Keyman Insurance Policy including
bonus on such policy. As per sec. 10(10D) Keyman insurance policy is a life insurance policy taken by a person
on the life of another person who is or was -
¾¾ an employee of the first mentioned person; or
¾¾ in any manner whatsoever connected with the business of the first mentioned person.
and includes such policy which has been assigned to a person, at any time during the term of the policy, with
or without any consideration
9. Conversion of stock into capital asset [Sec. 28(via)]: The fair market value of inventory as on the date on
which it is converted into, or treated as, a capital asset. (Discussed later in the chapter)
10. Recovery against certain capital assets covered u/s 35AD [Sec. 28(vii)]: Any sum received or receivable
(in cash or kind) on account of any capital asset (other than land or good­will or financial instrument) being
demolished, destroyed, dis­carded or transferred, if the whole of the expenditure on such capital asset has been
allowed as a deduction u/s 35AD (Discussed later in this chapter).

General Points
1. Chargeability: As per sec. 145(1), income chargeable under the head “Profits & gains of business or
profession” or “Income from other sources”, shall subject to the provision of sec. 145(2), is to be computed in
accordance with the method of accounting (i.e. either on cash or on accrual basis) regularly followed by the
assessee. However, there are certain expenditures specified u/s 43B, which shall be deductible only on cash
basis.
As per sec. 145(3), where the Assessing Officer is not satisfied about the correctness or completeness of
the accounts of the assessee, or has not been regularly followed by the assessee, or income has not been
computed in accordance with the notified standards, the Assessing Officer may make an assessment in the
manner provided u/s 144 i.e. Best Judgment Assessment.
2. Negative income: Income includes negative income i.e. loss.
3. Notional profit: A person cannot do business with himself, hence notional profit is not taxable. E.g. If proprietor
withdraws goods costing ₹ 10,000 for personal use at an agreed value of ₹ 12,000 then profit of ₹ 2,000 shall
not be taxable.
4. Anticipated profit or loss: Anticipated or potential profit or loss, which may or may not arise in future are not
considered for deriving taxable income.
5. Legality of business: There is no difference between legal or illegal business from income tax point of view.
Even income of illegal business shall be taxable.
6. Compilation of income of all business or profession: If an assessee carries on several business or profession,
then income from all business or profession shall be merged together.
7. Business or profession must be carried on during the previous year. Income is chargeable under the head
“Profits & gains of business or profession” only if the business is carried on by the assessee during the previous
year. It is not necessary that the business should continue throughout the year or till the end of previous year.

The Institute of Cost Accountants of India 177


Direct Taxation

Exceptions
However, in the following cases, income may be charged under the head Profits & gains of business or
profession even though the business is not carried on during the previous year:
Sections Details
176(3A)/(4) Applicability: Where any business or profession is discontinued in any year and any sum
received after the discontinuance.
Treatment: The sum so received shall be deemed to be the income of the recipient & charged
to tax accordingly in the year of receipt, if such sum would have been included in the total
income of the person who carried on the business had such sum been received before such
discontinuance
*41(1) Recovery of any amount earlier allowed as deduction
*41(2) Balancing charge in case of power sector unit
*41(3) Sale of an asset used for scientific research
*41(4) Bad debt recovery which was earlier allowed as deduction
*41(4A) Amount withdrawn from a reserve created u/s 36(1)(viii)
* Discussed later in this chapter

2.3.3 Incomes not taxable under the head Profits and gains of business or profession
Following incomes are though in the nature of profits and gains of business or profession, shall not be taxable under
this head:
1. Rent from house property is taxable u/s 22 under ‘Income from house property’ even though -
¾¾ the assessee is engaged in the business of letting out properties on rent; or
¾¾ such property is held as stock in trade
Note: However, where letting of house supporting the smooth running of the business (i.e. incidental to
business), then such income shall be taxable under this head.
2. Dividend on shares is taxable u/s 56(2)(i) under the head ‘Income from other sources’ even though the
assessee deals in shares and such shares are held as stock in trade. The provision is not applicable in case of
interest on securities held as stock in trade.
3. Winning from lotteries, races etc. are taxable under the head ‘Income from other sources’ even if such
income is derived through regular business activity.
Treatment of lottery ticket held as stock in trade: However, where an assessee deals in lottery tickets and
some of the lottery tickets remained unsold, any winning from such unsold lottery ticket shall be treated as
incidental to business and taxed under the head ‘Profits and gains of business or profession
4. Exempted income by virtue of sec. 10, 11 or 13A.
5. Sum taxable under the head ‘Capital gains’ for the purpose of sec. 28 (va) shall not be taxable under this
head. E.g. profit on sale of route permit shall not be taxable under the head ‘Profits & gains of business or
profession’.

178 The Institute of Cost Accountants of India


Heads of Income

2.3.4 Expenditures allowed as deduction


General Notes
1. Capital -vs.- Revenue expenditure: Capital expenditures are not allowed as deduction, unless & until
expressly allowed whereas revenue expenditures are allowed as deduction until & unless expressly disallowed
under the Income tax Act.
2. Expenditure of non-assessable business: Any expenditure of a non-assessable business is not allowed as
deduction. For instance, expenditure for earning agricultural income is not allowed as deduction from income
of an assessable business.
3. Expenditure must relate to the business of the assessee: Expenditure must have been incurred by the
assessee for its business.
Note: In case where the assessee incurs expenditure for its own business, the mere fact that the benefit of such
expenditure is enjoyed by some other person, cannot deny the admissibility of the expenditure.
4. Anticipated loss or expenditure: Subject to certain exceptions, no deductions are allowed for anticipated
losses. E.g. provision for bad & doubtful debts.
5. Notional expenditure: No one can earn income from himself/herself. For instance, rent paid to a sole proprietor,
salary to proprietor, interest on capital to proprietor, etc. are not income in the hands of the proprietor. Hence,
it is not deductible from the income of business as expenditure.
6. Onus to proof: Onus to proof lies with the assessee. It is the responsibility of the assessee to prove that a
particular expenditure is to be allowed as deduction in his case.

2.3.5 Specific Deductions


As per sec. 29, income under this head will be computed considering the provisions of sec. 30 to 43DB, which
decides the admissibility of expenditures for computing income under this head.

2.3.6 Rent, rates, taxes, repairs & insurance for building [Sec. 30]
Rent, rates, taxes, repairs & insurance for premises used for the purpose of business or profession shall be allowed
under this section. Points to be noted in this regard:
1. Use of building: The building is to be used for the purpose of business or profession. However, if the building
is not exclusively used for the purpose of business or profession then deduction shall be restricted to a fair
proportion of above expenditure which the Assessing Officer may determine [Sec. 38(2)].
2. Notional Rent: Rent paid to proprietor is disallowed but rent paid by firm to its partner for using his premises
is an allowed expenditure.
3. Current repair vs Capital repair: Only current repairs are allowed as deduction. Capital repairs are not
allowed as deduction whether the assessee occupies the building as a tenant or as a landlord.
Current repair (irrespective of the amount involved) means -
 a repair incurred to preserve and maintain an existing asset; and
 a repair which does not result in a new or fresh advantage.
4. Municipal taxes: Rates & taxes (for e.g. land revenue, municipal tax, etc) are deductible on cash basis [Sec.

30 read with sec. 43B]

The Institute of Cost Accountants of India 179


Direct Taxation

2.3.7 Repairs & insurance of machinery, plant & furniture [Sec. 31]
Repairs & insurance of plant, machinery & furniture are allowed as deduction. Points to be noted in this regard:
1. Use of asset: The asset must be used for the purpose of business or profession. However, if the asset is not
exclusively used for the purpose of business or profession then deduction shall be restricted to a fair proportion
of above expenditure, which the Assessing Officer may determine [Sec. 38(2)].
2. Current repair vs Capital repair: Only current repairs are allowed as deduction.
Example 2:
¾¾ Heavy expenditure incurred for replacement of part of a ship without creating any asset is deductible
¾¾ Any expenditure on the replacement of petrol engine by a diesel engine on his vehicle is allowed u/s 31.
3. Rent for furniture, plant or machinery: Only repairs & insurance of machinery, plant & furniture is covered
under this section. Rent paid for use of such assets is deductible u/s 37(1).

2.3.8 Depreciation [Sec. 32]


Sec. 32 provides for depreciation on -
Tangible assets Building, Machinery, Plant and Furniture.
Intangible assets Know how, Copyright, Trade Mark, Patent, Licence, Franchise, or any other business or
commercial right of the similar nature acquired on or after 1/4/1998
However, it does not include goodwill

Conditions for claiming depreciation


Depreciation is allowed provided the following conditions are satisfied:

Condition 1: Asset must be owned by the Condition 2: Asset must be used for the purpose of business or
assessee. profession during the previous year.

Notes
¾¾ Beneficial owner: Assessee need not be a
registered owner, even a beneficial owner ¾¾ Passive use -vs.- Active use: Use includes active use
can claim depreciation. as well as passive use. Active use means actual use of the
property for the purpose of business or profession. Whereas
passive use includes “ready to use”. It means, if a property
¾¾ Co-owner: In case of joint ownership,
was not actually used for business or profession but was
depreciation is allowed on proportionate
ready to use in the previous year, in such case, assessee can
basis.
claim depreciation on such assets

¾¾ Property acquired on hire purchase: In ¾¾ Partly used for business or profession: As per sec.
case of hire purchase, the buyer can claim 38, if an asset is partly used for business or profession
depreciation even though he does not get and partly used for personal purpose, then proportionate
legal title of the asset till he pays the last depreciation (as determined by the Assessing Officer) shall
instalment. be allowed.

180 The Institute of Cost Accountants of India


Heads of Income

¾¾ Capital expenditure on a property by ¾¾ House property let out to tenant for smooth running
the lessee: Where an assessee being a of the business: If an assessee lets out a property to his
lessee of a property incurs any capital employee and where such letting-out supports smooth
expenditure by way of improvement, flow of his business, then rent received from employee
extension, super construction, etc. on a shall be chargeable under the head “Profits & gains of
building being used for his business or business or profession” and such property shall be eligible
profession, he is entitled to depreciation for depreciation u/s 32. Similarly, where an assessee makes
in respect of such capital expenditure. available his property to any Government agency for
locating branch of a nationalized bank, police station, post
¾¾ Sec. 53A of Transfer of Property Act:
office, tax office, railway staff quarters, etc. for the purpose
Possessor of an immovable property u/s
of running the business of assessee more efficiently, then
53A of Transfer of Property Act can claim
such letting out shall be deemed to be incidental to business
depreciation even though he is not the
and depreciation on such building shall be allowed u/s 32.
registered owner of the property.

Method of computing depreciation (other than power units)


The method of computing depreciation as per Income tax Act is entirely different from accountancy method. For
Income tax purpose, assets are categorised into Block of Assets.

Block of Assets [Sec. 2(11)]


Block of assets means a group of assets of same nature, in respect of which same rate of depreciation is charged.
In other words, to fall in the same block, the following two conditions are to be satisfied:
 Assets must be of same nature;
Tangible assets being building, machinery, plant or furniture, and
Intangible assets, being know-how, patents, copy-rights, trade marks, licenses, franchises or any other business
or commercial rights of similar nature acquired on or after 1-4-1998 (it does not include goodwill);
 Rate of depreciation on such asset must be same.

Method of Depreciation
Depreciation shall be allowed on written down value method at the rates prescribed.
Calculation of depreciation (at a glance)
Particulars Amount
W.D.V of the block at the beginning of the previous year ***
Add: Assets (falling within the block) acquired during the previous year ***
ABC
Less: Sale Proceeds of assets (falling within the block) sold during the previous year [$subject to (DE)$
max. of ABC]
Written Down Value [# XYZ cannot be negative] XYZ#
Less: Depreciation (as a % on XYZ) (***)
Opening WDV for 1st day of next year ****

The Institute of Cost Accountants of India 181


Direct Taxation

When depreciation is not charged


Depreciation is not charged in the following two cases:
1. When ‘DE’ (Sale proceeds) exceeds ABC, the excess shall be treated as short term capital gain.
2. When ‘XYZ’ (Value of block before depreciation) is positive but the block does not have any asset. In such
case, such positive value shall be treated as short term capital loss.

Significance of date of purchase (Effect of time on depreciation)


Where -
a. an asset is acquired by the assessee during the previous
year; and
b. is put to use in the same previous year for less than 180
days,
- the depreciation in respect of such asset is restricted
to 50% of the normal depreciation.
Except above, date of purchase has no relevance.
Taxpoint: There is no significance of date of sale for
computation of depreciation.

Extract of depreciation-rate
Block Nature of Asset
Buildings1 5% Residential building other than hotels and boarding
Buildings 10% Non residential building, godown, office, factory, etc. including hotels and boarding
Buildings 40% Temporary construction
Furniture 10% Any furniture including electrical fittings
Plant/Machinery 20% Ocean going ships, vessels, speed boats
Plant/Machinery 30% Motor car (including lorries and buses) used for hiring purposes
Motor car, other than used in a business of running them on hire, acquired and put
Plant/Machinery 30%
to use between 23-08-2019 and 31-03-2020
Motor buses, motor lorries and motor taxis used in a business of running them on
Plant/Machinery 45%
hire, acquired and put to use between 23-08-2019 and 31-03-2020
Computer including computer software
Plant/Machinery 40%
Books owned by a professional
Plant/Machinery 40% Air or water pollution control equipment
Plant/Machinery 15% Oil Wells
In general (if nothing is mentioned regarding nature of plant & machinery and
Plant/Machinery 15%
including motor car not used for hiring purpose)
Intangible assets3 25% Acquired after 31/3/98
1.
Buildings include roads, bridges, culverts, wells (excluding oil wells) and tube wells.
2.
Plant does not include tea bushes or live stocks or buildings or furniture & fittings.
3
Patent, Know-how, Copy-rights, Trade-mark, Licences, Franchises and other business or commercial right of
similar nature (it does not include goodwill)

182 The Institute of Cost Accountants of India


Heads of Income

Illustration 62:
M/s Anita Enterprises has written down value in furniture block (depreciation rate 10%) as on 1/4/2023 ₹ 80,000.
The block consists of two furniture X and Y.
Compute depreciation u/s 32 for the A.Y. 2024-25 in the following cases:
Case A Furniture X sold for ₹ 20,000 on 1/5/2023
Case B Furniture X sold for ₹ 1,00,000 on 1/1/2024
Case C Furniture X sold for ₹ 1,00,000 and Furniture S purchased for ₹ 35,000 as on 1/7/2023
Case D Furniture X sold for ₹ 10,000 and Furniture S purchased for ₹ 40,000 as on 1/7/2023
Case E Furniture X sold for ₹ 10,000 and Furniture S purchased for ₹ 40,000 as on 11/11/2023
Case F Furniture X sold for ₹ 2,00,000 and Furniture S purchased for ₹ 40,000 as on 11/11/2023
Case G Furniture X and Furniture Y both sold for ₹ 10,000 and ₹ 35,000 respectively.
Case H Furniture X and Furniture Y both sold for ₹ 10,000 and ₹ 35,000 respectively as on 11/11/2023. New
Furniture T purchased for ₹ 5,000 as on 1/7/2023.
Case I Furniture Z purchased for ₹ 40,000 on 1/7/2023 and the same being put to use on 11/11/2023.
Case J Furniture Q purchased for ₹ 50,000 on 1/7/2023 but put to use on 1/11/2024.
Case K Furniture R purchased for ₹ 30,000 on 1/7/2022 but put to use on 11/11/2023.
Case L Furniture S purchased for ₹ 10,000 on 1/7/2023 but put to use on 11/11/2023 and Furniture X and Y
sold for ₹ 10,000 and ₹ 6,000 respectively.
Case M Furniture R purchased for ₹ 30,000 on 1/7/2023 and sold the same for ₹ 40,000 on 11/11/2023.
Case N Sold Furniture X and Y for ₹ 95,000 on 1/7/2023 & purchased Furniture R for ₹ 30,000 on 11/11/2023
Case O Sold Furniture X for ₹ 90,000 on 11/7/2023 and following Furniture put to use -
¾¾ Furniture A on 18/12/2023, purchased on 17/12/2023 for ₹ 30,000;
¾¾ Furniture B on 18/2/2024, purchased on 15/8/2023 for ₹ 50,000;
¾¾ Furniture Z on 18/4/2023, purchased on 17/7/2022 for ₹ 60,000;
¾¾ Furniture P on 8/12/2023, purchased on 17/5/2022 for ₹ 10,000;
¾¾ Furniture Q on 1/4/2024, purchased on 31/3/2024 for ₹ 20,000.
Assume in all cases new furniture is charged to deprecation @ 10%
Solution :
Computation of depreciation for the A.Y. 2024-25

Particulars Case A Case B Case C Case D Case E Case F Case G Case H


Block: Furniture (10%)
W.D.V. as on 1/4/2023 80,000 80,000 80,000 80,000 80,000 80,000 80,000 80,000
Add: Purchase Nil Nil 35,000 40,000 40,000 40,000 Nil 5,000
80,000 80,000 1,15,000 1,20,000 1,20,000 1,20,000 80,000 85,000
Less: Sale Proceeds 20,000 80,000# 1,00,000 10,000 10,000 1,20,000# 45,000 45,000
60,000 Nil 15,000 1,10,000 1,10,000 Nil 35,000 40,000
Depreciation 6,000 Nil 1,500 11,000 9,0001 Nil Nil 4,000
Short term capital gain 20,000 80,000
Short term capital Loss (35,000)

The Institute of Cost Accountants of India 183


Direct Taxation

#
Sale Proceeds cannot exceed Opening WDV as increased by actual cost of asset acquired during the previous
year. Excess, if any, shall be considered as short term capital gain.

Computation of depreciation for the A.Y. 2024-25

Particulars Case I Case J Case K Case L Case M Case N


Block: Furniture (10%)
W.D.V. as on 1/4/2023 80,000 80,000 80,000 80,000 80,000 80,000
Add: Purchase 40,000 Nil 30,000 10,000 30,000 30,000
1,20,000 80,000 1,10,000 90,000 1,10,000 1,10,000
Less: Sale Proceeds Nil Nil Nil 16,000 40,000 95,000
1,20,000 80,000 1,10,000 74,000 70,000 15,000
Depreciation 10,000 2
8,000 11,000 3
6,900 4
7,000 7505
Short term capital gain
Short term capital Loss
1
(₹ 70,000 * 10%) + (₹ 40,000 * 10% * ½) 2
(₹ 80,000 * 10%) + (₹ 40,000 * 10% * ½)
3
Though the asset was put to use for less than 180 days but since it was not acquired in the current year hence
provision for ½ year depreciation shall not be applicable
4
(₹ 10,000 * 10% * ½) + (₹ 64,000 * 10%) 5
(₹ 15,000 * 10% * ½)

Case O: Computation of depreciation for the A.Y. 2024-25

Particulars Details Amount


Block: Furniture (10%)
W.D.V. as on 1/4/2023 80,000
Addition during the year:
–– Furniture A (eligible for ½ year depreciation) 30,000
–– Furniture B (eligible for ½ year depreciation) 50,000
–– Furniture Z 60,000
–– Furniture P 10,000 1,50,000
2,30,000
Less: Sale Proceeds 90,000
1,40,000
Depreciation [{₹ 80,000 * 10% * ½} + {₹ 60,000 i.e. (₹ 1,40,000 – ₹ 80,000) * 10%)] 10,000
Since, furniture Q was put to use on 1/4/2024, therefore depreciation shall not be charged in the P.Y.2023-24.

184 The Institute of Cost Accountants of India


Heads of Income

Illustration 63:
Mr. X, a grower and manufacturer of tea, purchased machinery (15%) on 10-04-2022 for ₹ 10 lakh. He computed
depreciation for A.Y. 2024-25 as given below; needs your comment on his working:

Particulars Amount

Opening W.D.V. as on 1/4/2022 Nil

Add: Assets purchased during the year 10,00,000

10,00,000

Less: Depreciation for the P.Y. 2022-23 [₹ 10,00,000 * 15% * 40%] 60,000

(As he is engaged in the business of growing and manufacturing tea; hence 60% is considered as
part of agricultural income)

Opening W.D.V. as on 1/4/2023 9,40,000

Less: Depreciation for the P.Y. 2023-24 [₹ 9,40,000 * 15% * 40%] 56,400

Opening W.D.V. as on 1/4/2023 8,83,600

Further, compute his business income for A.Y. 2024-25 assuming that his income before depreciation and without
reducing element of agricultural income is ₹ 8,00,000/-
Solution :
The method of computation of depreciation followed by Mr. X is not correct as Expl. 7 to sec.43(6) provides that:
“Where the income of an assessee is derived, in part from agriculture and in part from business chargeable to
income-tax under the head “Profits and gains of business or profession”, for computing the written down value of
assets acquired before the previous year, the total amount of depreciation shall be computed as if the entire income
is derived from the business of the assessee under the head “Profits and gains of business or profession” and the
depreciation so computed shall be deemed to be the depreciation actually allowed under this Act.”
The correct computation of depreciation is as follow:

Particulars Amount
Opening W.D.V. as on 1/4/2022 Nil

Add: Assets purchased during the year 10,00,000

10,00,000

Less: Depreciation for the P.Y. 2022-23 [₹ 10,00,000 * 15%]

(Considering the entire income as taxable income) 1,50,000

Opening W.D.V. as on 1/4/2023 8,50,000

Less: Depreciation for the P.Y. 2023-24 [₹ 8,50,000 * 15%] 1,27,500

Opening W.D.V. as on 1/4/2024 7,22,500

The Institute of Cost Accountants of India 185


Direct Taxation

Computation of business income of Mr. X for A.Y. 2024-25

Particulars Amount

Income before depreciation and without reducing element of agricultural income 8,00,000

Less: Depreciation 1,27,500


6,72,500
Less: Agricultural Income being 60% of above 4,03,500
Profits and Gains of Business or Profession 2,69,000

2.3.9 Additional depreciation [Sec. 32(1)(iia)]


Applicability
Additional depreciation is applicable on all assessee engaged in the business of manufacture or production of any
article or thing or in the business of generation, transmission or distribution of power
Conditions to be satisfied
1. Assessee must be an industrial undertaking, which manufactures or produces any article or thing or in the
business of generation, transmission or distribution of power.
2. Assessee acquired and installed after 31st March, 2005, a new plant or machinery, other than the following:
¾¾ Ships and air crafts; or
¾¾ Any plant or machinery which was used either within or outside India by any other person before such
installation; or
¾¾ Any plant or machinery installed in office premises or any residential accommodation or guest house; or
¾¾ Any office appliances or road transport vehicle; or
¾¾ Any plant or machinery, which is allowed for 100% deduction (whether by way of depreciation or
otherwise) in the previous year.
Taxpoint: Additional depreciation shall be available only on plant and machinery and not on other asset like
furniture, building, etc.
Rate of additional depreciation
Rate of additional depreciation is 20% of actual cost of such plant or machinery.
 Where, if the asset is acquired and put to use for less than 180 days then additional depreciation @ 10% (i.e.,
50% of 20%) of actual cost shall be allowed in that previous year and the deduction for the balance 10% shall
be allowed in the immediately succeeding previous year.
Taxpoint
1. Additional depreciation shall be reduced while computing the closing WDV of the respective block.
2. Additional depreciation is not available if the new plant or machinery is sold in the year of acquisition.
3. Additional depreciation is not available if the power unit is claiming depreciation under straight line method
i.e. u/s 32(1)(i)

186 The Institute of Cost Accountants of India


Heads of Income

Provision Illustrated
B Ltd., a newly formed manufacturing concern, has furnished you the following details to compute Depreciation
allowed for the A.Y. 2023-24 and 2024-25:
Assets Date of Acquisition Cost of Acquisition Rate of depreciation
Plant A 02/04/2022 5,00,000 15%
Plant B 07/05/2022 3,00,000 15%
Plant C 15/12/2022 2,00,000 15%
Plant D 05/05/2023 1,00,000 15%
Solution:
Computation of Additional Depreciation

Additional depreciation
Assets Rate Cost
A.Y. 2023-24 A.Y. 2024-25
Plant A 20% 5,00,000 1,00,000 Nil#
Plant B 20% 3,00,000 60,000 Nil#
Plant C 10% 2,00,000 20,000 20,000
Plant D 20% 1,00,000 Nil 20,000
Total 1,80,000 40,000
Calculation of Depreciation u/s 32 of Plant (15%) for the A.Y.2023-24 and 2024-25
Particulars Details Amount
W.D.V. as on 1/4/2022 -
Add: Purchase during the year 10,00,000
10,00,000
Less: Sale during the year Nil
10,00,000
Depreciation (normal) [(₹ 8,00,000 * 15%) + (₹ 2,00,000 * 15% * ½)] 1,35,000
Additional depreciation (as computed above) 1,80,000 3,15,000
W.D.V. as on 1/4/2023 6,85,000
Add: Purchase during the year 1,00,000
7,85,000
Less: Sale during the year Nil
7,85,000
Depreciation (normal) [₹ 7,85,000 *15%] 1,17,750
Additional depreciation (as computed above) 40,000 1,57,750
W.D.V. as on 1/4/2024 6,27,250

Illustration 64:
An industrial undertaking, which commenced the manufacturing activity with effect from 1st September, 2023 has
acquired the following assets during the previous year 2023-24:

The Institute of Cost Accountants of India 187


Direct Taxation

Assets Date of acquisition Date when put to use Cost of acquisition


Factory building 4-4-2023 1-9-2023 50,00,000
Plant & Machinery
Machinery A 5-5-2023 1-9-2023 2,00,000
Machinery B 7-6-2023 1-9-2023 5,00,000
Machinery C 30-8-2023 1-9-2023 10,00,000
Machinery D 1-9-2023 31-10-2023 4,00,000
Machinery E 1-1-2024 28-2-2024 3,00,000
Machinery F (second hand) 11-1-2024 13-1-2024 2,00,000
Motor car 1-2-2024 1-2-2024 5,00,000
Air-conditioner (installed in the office) 1-2-2024 2-2-2024 1,00,000
Compute the depreciation allowable for the assessment year 2024-25 and the written down value as on 1st April
2024.
Solution :
Computation of depreciation allowable for the A.Y. 2024-25
Particulars Building Plant & Machinery1
Rate 10% 15%
W.D.V. as on 1-4-2023 Nil Nil
Add: Purchase during the year 50,00,000 32,00,000
50,00,000 32,00,000
Less: Sale proceeds Nil Nil
50,00,000 32,00,000
Depreciation on above 5,00,000 7,77,500
W.D.V. as on 1-4-2024 45,00,000 24,22,500 (Note)
1.
Block consists of Machinery A to Machinery F, Motor Car & Air-conditioner.
2.
Depreciation on plant and machinery

- Normal [(₹ 17,00,000 * 15%) + (₹ 15,00,000 * 15% * ½)] 3,67,500


- Additional (on Machinery A to E) [(₹ 17,00,000 * 20%) + (₹ 7,00,000 * 20% * ½)] 4,10,000
Total Depreciation 7,77,500
Note
1. Asset which was put to use for less than 180 days is eligible for ½ year depreciation. However, additional
depreciation of ₹ 70,000/- (i.e. ₹ 7,00,000 * 20% * ½) shall be available in the A.Y.2025-26.
2. Additional depreciation is not available on following assets:

Asset Reason
Factory building As it is not plant and machinery
Machinery-F As it is a second hand machinery

188 The Institute of Cost Accountants of India


Heads of Income

Asset Reason
Motor car As it is a road transport vehicle
Air conditioner As it is installed in office

2.3.10 Treatment of Slump sale


Slump sale [Sec. 2(42C)]: It means transfer of undertaking(s) for a lump sum consideration without assigning
values to the individual assets of such undertaking(s).
Computation of written down value of block of assets in case of slump sale

Particulars Amount
W.D.V of the block at the beginning of the previous year ***
Add: Purchase during the previous year ***
Mno
Less: Sale consideration for assets sold (to the maximum of mno) (****)
Pqr
Less: WDV (Note) of the asset sold under slump sale (abc)
[Value of deduction at this stage i.e. abc cannot exceed pqr]
XYZ
Less: Depreciation (as a % on XYZ) (***)
WDV of the block at the end of year ****
Note: Written down value of the asset sold under slump sale
Particulars Amount
Original cost of asset sold under slump sale ***
Less: Depreciation (actual) allowed on such asset in respect of any previous year (***)
commencing before 1987-88
Less: Depreciation (notional) that would have been allowable from the previous year (***)
1987-88 onwards as if the asset is only asset in the relevant block.
Written down value of the asset sold under slump sale ***

2.3.11 Depreciation in case of Power Units


An undertaking engaged in the business of generation or generation and distribution of power may charge
depreciation (in respect of asset acquired after 31/3/1997) at its choice under -
 Written-down value method as followed by all other assessee (usual); or
 Straight-line method at the prescribed rate in ‘Appendix IA’ of the Income Tax Rules on actual cost of asset
(not the block value of asset)
However, such option shall be exercised before the due date of furnishing return of income. Further, it may be noted
that once the option is exercised, it shall be applicable for all subsequent assessment years.

The Institute of Cost Accountants of India 189


Direct Taxation

Note: Additional depreciation is not available to the assessee who claims depreciation as per Straight-Line Method.

Terminal Depreciation and Balancing Charge


Applicable
Assessee engaged in generation or generation and distribution of power.
to
1. Assessee must follow the straight-line method of depreciation at specified rates.
Conditions 2. The asset is sold, discarded, demolished or destroyed in the previous year (other than the
previous year in which it is first brought into use).
Terminal Depreciation Balancing Charge
Loss on transfer of such asset is Profit on transfer of such asset to the maximum of accumulated
treated as terminal depreciation. depreciation shall be treated as balancing charge.
Taxpoint: The difference between sale price and actual cost shall be
treated as capital gain.
¾¾ Terminal depreciation = + ve
Meaning
value of [WDV of assets – Taxpoint:
(Sale value or Scrap value)]
Balancing Charge = - ve value of [WDV of assets – (Sale
¾¾ Terminal depreciation is value + Scrap value)]
written off in the books of
However, balancing charge cannot exceed accumulated
accounts.
depreciation claimed on such asset.
As per sec. 41(2), balancing charge is fully taxable as business
Terminal depreciation is fully
income in the previous year in which such income falls due.
Treatment allowed as deduction as a business
The provision holds good even if the business does not exist
loss.
in that year.

Illustration 65:
Important Ltd. is a power-generating unit. On 1-4-2021, it purchased a plant of ₹ 50,00,000 eligible for depreciation
@ 15% on SLM. Compute balancing charge or terminal depreciation assuming the plant is sold on 21/4/2023 for:
A) ₹ 7,50,000 B) ₹ 30,00,000 C) ₹ 45,00,000 D) ₹ 55,00,000

Solution :
Computation of capital gain or balancing charge or terminal depreciation for the A.Y.2024-25

Amount
Particulars Note
A B C D
Written down value as on 1/4/2023 1 35,00,000 35,00,000 35,00,000 35,00,000
Less: Sale Proceeds 7,50,000 30,00,000 45,00,000 55,00,000
Balance 27,50,000 5,00,000 (-) 10,00,000 (-) 20,00,000
Terminal depreciation 27,50,000 5,00,000 Nil Nil
Balancing Charge 2 Nil Nil 10,00,000 15,00,000
Short term capital gain 2 Nil Nil Nil 5,00,000

190 The Institute of Cost Accountants of India


Heads of Income

Notes
1. Computation of Written down value as on 1/4/2023

Particulars Amount
Original cost 50,00,000
Less: Depreciation for the year 2021-22 7,50,000
WDV as on 1/4/2022 42,50,000
Less: Depreciation for the year 2022-23 7,50,000
WDV as on 1/4/2023 35,00,000
2. Balancing charge cannot exceed accumulated depreciation claimed on such asset. The total negative balance
in case D is ₹ 20,00,000 but the accumulated depreciation is ₹ 15,00,000 only. Hence, balancing charge is
restricted to ₹ 15,00,000 & the balance i.e. ₹ 5,00,000 shall be treated as short-term capital gain.

Test Yourself
1. Taj Electric Supply Company Ltd. which was charging depreciation on straight line method and whose actual
cost of the asset was ₹ 20,00,000 and written down value ₹ 18,72,300 sold the said asset during 2023-24 after
2 years. What will be the tax treatment for assessment year 2024-25 if the asset is sold for:
i. ₹ 30,000;
ii. ₹ 18,72,300;
iii. ₹ 19,80,000;
iv. ₹ 21,00,000
Hints
(i) ₹ 18,42,300 Terminal Depreciation (ii) Nil (iii) ₹ 1,07,700 Balancing Charge (iv) ₹ 1,27,700 Balancing charge
and ₹ 1,00,000 Short Term Capital Gain
In all the cases, no further depreciation is allowable to the assessee in respect of such asset.

2.3.12 Actual cost of assets [Sec. 43(1)]


In calculation of actual cost, apart from cost price of the asset, following expenditure incurred relating to such asset
shall be included:
 Expenses directly related to acquisition of the asset including travelling expenditure incurred for acquiring
asset.
 Expenses necessary to bring the asset to site, installation, and to make it ready to use, e.g. carriage inward,
loading and unloading charges, installation cost, trial run cost, etc.
 Expenses incurred to increase the capacity of the asset or to make it fit prior to its use.
Actual cost means the actual cost of the assets to the assessee, as reduced by that portion of the cost thereof, if any,
as has been met directly or indirectly by any other person or authority. Eg.: If an asset is purchased for ₹ 5,00,000
and Government grant received for the same ₹ 1,00,000, then the actual cost of the asset for tax purpose shall be
₹ 4,00,000.

The Institute of Cost Accountants of India 191


Direct Taxation

Following points shall be considered -

Particulars Actual cost of acquisition


In respect of acquisition of any asset or part thereof:
a) payment or aggregate of payments made to a person in a day is made
otherwise than by an account payee cheque drawn on a bank or an
account payee bank draft or use of electronic clearing system through
Assets acquired against cash a bank account or through other prescribed electronic mode; and
b) such payment exceeds ₹ 10,000
c) Such payment shall be ignored for the purposes of determination of
actual cost.
Actual cost to the previous owner as reduced by -
(a) Amount of depreciation actually allowed under this Act till the A.Y.
Asset acquired by way of gift or 1987-88; and
inheritance (b) Amount of depreciation that would have been allowable to the
assessee from A.Y. 1988-89, as if the asset was the only asset in the
relevant block.
Interest treatment in case of asset Before asset is put to use Interest to be added to actual cost
acquired out of borrowed fund After asset is put to use Interest is allowed u/s 36(1)(iii)
Any subsidies received from the Grant or subsidies will be subtracted from cost of acquisition of such asset.
Government or any other authority
for purchase of an asset
Actual cost of asset shall be reduced by the amount of input tax credit
GST included in the invoice
taken against GST
Asset acquired from any person
Actual cost to be determined by the AO with the prior permission of
using the asset for his business or
Deputy Commissioner
profession with a view to avoid tax

WDV at the time of first transfer or the price paid for reacquisition,
Reacquisition of transferred asset
whichever is lower

Asset acquired by an assessee from


another person and given on lease
WDV of the asset to the transferor
to the same person who had earlier
claimed depreciation on such asset

Building used for personal purpose Cost of purchase or construction of the building as reduced by the notional
subsequently brought into business depreciation by applying the rate applicable on the date of such conversion
Asset, which was acquired outside Actual cost to the assessee, as reduced by an amount equal to the amount of
India, is brought by a non-resident depreciation calculated at the rate in force that would have been allowable
assessee to India and used for had the asset been used in India for the said purposes since the date of its
the purposes of his business or acquisition by the assessee
profession

192 The Institute of Cost Accountants of India


Heads of Income

Particulars Actual cost of acquisition


Any capital asset transferred by Actual cost to the transferee company shall be taken to be the same as
a holding company to its 100% it would have been if the transferor company had continued to hold the
subsidiary company or vice versa capital asset for the purpose of its business
where transferee company is an
Indian company.
Any capital asset transferred by Actual cost to the amalgamated company shall be taken to be the same as it
the amalgamating company to the would have been if the amalgamating company had continued to hold the
amalgamated company where the capital asset for the purpose of its own business
amalgamated company is an Indian
company
Any capital asset transferred by the
Actual cost to the resulting company shall be taken to be the same as it
demerged company to the resulting
would have been if the demerged company had continued to hold the
company where the resulting
capital asset for the purpose of its own business
company is an Indian company
Capital asset is acquired by the Actual cost of the asset shall be deemed to be the amount which would
assessee under a scheme for have been regarded as actual cost had there been no such corporatisation
corporatisation of a recognised
stock exchange in India
Where an assessee was not required Actual cost shall be reduced by the total amount of depreciation on such
to compute his total income under asset, provided in the books of account (as adjusted by amount attributable
Income tax Act for any previous to the revaluation of assets) of the assessee in respect of such previous
year(s) preceding the relevant year(s) preceding the relevant previous year
previous year
The actual cost of any capital asset Actual cost of the asset shall be taken as nil. (Discussed later in this chapter)
on which deduction is allowable u/s
Note: If such asset is transferred to other person as gift, etc., actual cost of
35AD
asset shall be taken as Nil in hands of transferee.
Conversion of inventory into capital Where inventory is converted into capital asset and such converted asset
asset is used for the purposes of business or profession, the actual cost of such
asset to the assessee shall be the fair market value which has been taken
into account for the purposes of sec. 28(via)

Illustration 66:
Dr. R purchased a house property on 1-12-2021 for ₹ 10,00,000. Till 1-12-2023, the same was self-occupied as a
residence. On this date, the building was brought into use for the purpose of his medical profession. What would
be the depreciation allowable for the assessment year 2024-25?
Solution :
In case a building is used for personal purpose subsequently brought into business, the cost of acquisition shall
be the purchase or construction cost of the building as reduced by the notional depreciation by applying the rate
applicable on the date of such conversion. In the given case cost of asset for the business shall be computed as
under:

The Institute of Cost Accountants of India 193


Direct Taxation

Particulars Building
Rate of depreciation 10%
Cost of building on 1.12.2021 10,00,000
Less: Depreciation (Being used for less than 180 days hence, depreciation charged 50% of normal 50,000
depreciation i.e. ₹ 10,00,000 * 50% * 10%)
WDV on 31.3.2022 9,50,000
Less: Depreciation 95,000
WDV on 31.3.2023 8,55,000
Computation of depreciation u/s 32
Cost of building on 1/4/2023 8,55,000
Depreciation for the year 2023-24 85,500
WDV on 01.04.2023 7,69,500

Illustration 67:
Roshan started a business of designing on 01-04-2022. He acquired a laptop on 01-04-2022 for ₹ 50,000 for his
business use. Since his gross total income for the previous year 2022-23 is only ₹ 55,000/-, he did not file his return
of income. During the previous year 2023-24, his business income before depreciation u/s 32 is ₹ 5,60,000. Since
he is required to file his return of income for the assessment year 2024-25, he seeks your advice for computing
depreciation. Please compute depreciation on his behalf assuming that:
a. He is maintaining books of account from 01-04-2022 but did not provide any depreciation on laptop.
b. He is maintaining books of account from 01-04-2022 and provided depreciation ₹ 8,000 on laptop.
c. He is maintaining books of account from 01-04-2023.
Solution :
Computation of depreciation in various cases:

Particulars Case (a) & (c) Case (b)


Cost of Laptop as on 01-04-2022 50,000 50,000
Less: Depreciation for the P.Y. 2022-23 provided in the books of account Nil 8,000
WDV on 31.3.2023 50,000 42,000
Less: Depreciation for the P.Y. 2023-24 @ 40% 20,000 16,800
WDV on 31.3.2024 30,000 25,200

Illustration 68:
A car was purchased by S on 10.8.2019 for ₹ 3,25,000 for personal use is brought into the business of the assessee on
01.12.2023, when its market value is ₹ 1,50,000. Compute the actual cost of the car and the amount of depreciation
for the Assessment year 2024-25 assuming the rate of depreciation to be 15%.

194 The Institute of Cost Accountants of India


Heads of Income

Solution :
Computation of depreciation on car for the A.Y. 2024-25

Particulars Amount
Cost of the car (Note 1) 3,25,000
Less: Depreciation @ 15% (Note 2) 48,750
Closing W.D.V. 2,76,250
1. As per explanation 5 to Sec. 43(1), where any building used for personal purpose subsequently brought into
business, then the cost of purchase or construction of the building as reduced by the notional depreciation
by applying the rate applicable on the date of such conversion shall be taken as actual cost of such building.
However, such provision is applicable only in case of building.
2. Where an asset is acquired by the assessee during the previous year and is put to use in the same previous year
for less than 180 days, the depreciation in respect of such asset is restricted to 50% of the normal depreciation.
However, in the case, car was not acquired in the P.Y. 2023-24, hence such provision is not applicable.

Illustration 69:
Compute depreciation u/s 32 for the A.Y. 2024-25 from the following information:
a. W.D.V. of plant and machinery (15%) as on 01-04-2023 ₹ 10,00,000
b. Plant D acquired on 10-07-2023 for ₹ 5,00,000/-. ₹ 1,00,000 has been paid in cash to the vendor and balance
amount has been paid through an account payee cheque. Such plant was put to use on the same day.
c. The assessee is engaged in the business of manufacturing of industrial paints.
Solution :
Computation of depreciation for A.Y. 2024-25
Particulars Amount Amount
WDV as on 01-04-2023 10,00,000
Add: Actual cost of Plant D acquired during the year [₹ 5,00,000 – ₹ 1,00,000] 4,00,000
14,00,000
Less: Depreciation for the P.Y. 2023-24 [₹ 14,00,000 x 15%] 2,10,000
Less: Additional Depreciation for the P.Y. 2023-24 [₹ 4,00,000 x 20%] 80,000 2,90,000
WDV on 01-04-2023 11,10,000

2.3.13 Consequence of changes in rate of exchange of currency [Sec. 43A]


Conditions
1. Assessee has acquired any asset in any previous year from a country outside India;
2. In consequence of a change in the rate of exchange during any previous year after the acquisition of such asset,
there is an increase or reduction in the liability of the assessee (as compared to the liability existing at the time
of acquisition of the asset) at the time of making payment -
a. towards the whole or a part of the cost of the asset; or
b. towards repayment of the whole or a part of the moneys borrowed by him from any person, directly or
indirectly, in any foreign currency specifically for the purpose of acquiring the asset along with interest.
The Institute of Cost Accountants of India 195
Direct Taxation

Treatment
The amount by which such liability is increased or reduced at the time of making the payment (irrespective of the
method of accounting adopted by the assessee) shall be added to or deducted from the actual cost (as reduced by
depreciation already claimed) of the asset
Taxpoint
 If such increase or decrease arises after the depreciable asset is transferred (but block exists), then such increase
or decrease shall be adjusted in the WDV. If, however, block is cease to exist, then such amount shall be treated
as capital receipt or expenditure.
 Where the whole or any part of the liability aforesaid is met, not by the assessee, but, directly or indirectly, by
any other person or authority, the liability so met shall not be taken into account for the purposes of this section.

Illustration 70:
Narang Textiles Ltd. purchased a machinery from Germany for Euro 1,00,000 on 03-09-2022 through a term
loan from Fortune Bank Ltd. The exchange rate on the date of acquisition was ₹ 65. The assessee took a forward
exchange rate on 05-10-2023 when the rate specified in the contract was ₹ 67 per USD. Compute depreciation for
the assessment years 2023-24 and 2024-25. Ignore additional depreciation.
Solution :
Computation of Depreciation
Particulars Amount
Opening W.D.V. as on 1/4/2022 Nil
Add: Assets purchased during the year [Euro 1,00,000 * 65] 65,00,000
65,00,000
Less: Depreciation for the P.Y. 2022-23 [₹ 65,00,000 * 15%] 9,75,000
Opening W.D.V. as on 1/4/2023 55,25,000
Add: Difference in Conversion rate [Euro 1,00,000 * 2] 2,00,000
57,25,000
Less: Depreciation for the P.Y. 2023-24 [₹ 57,25,000 * 15%] 8,58,750
Opening W.D.V. as on 1/4/2024 48,66,250

2.3.14 Taxation of foreign exchange fluctuation [Sec. 43AA]


Any gain (or loss), being computed in accordance with the ICDS, arising on account of any change in foreign
exchange rates shall be treated as income (or loss).
Taxpoint:
 Such gain or loss shall arise in respect of all foreign currency transactions, including those relating to:
i. monetary items and non-monetary items;
ii. translation of financial statements of foreign operations;
iii. forward exchange contracts;
iv. foreign currency translation reserves*
 The provision of sec. 43AA is not applicable in respect of cases covered u/s 43A (like computation of actual
cost of the asset, etc)
* ICDS is not dealing with foreign currency translation reserves

196 The Institute of Cost Accountants of India


Heads of Income

2.3.15 Unabsorbed depreciation [Sec. 32(2)]


Depreciation which could not be fully deducted from profits and gains of current year of business or profession
(due to insufficient profit), is termed as unabsorbed depreciation.
Treatment: The unabsorbed depreciation can be deducted from income under any other head (except with Casual
income and Salaries) of the same assessment year.
If depreciation still remains unabsorbed, it can be carried forward for indefinite period and can be set off against
any income (except with Casual income and Salaries) of the assessee.
Notes
1. It is not necessary that the same business should be continued.
2. For set-off purpose following order is to be followed:
¾¾ Current year depreciation;
¾¾ Brought forward business loss;
¾¾ Unabsorbed depreciation
3. Unabsorbed depreciation shall be (subject to sec. 72 and sec. 73) added to the amount of the depreciation for
the following previous year and deemed to be the depreciation-allowance for that previous year, and so on for
the succeeding previous years.
4. Unabsorbed depreciation shall be allowed to be carried forward for any number of years and such carried
forward unabsorbed depreciation may be set off against any income, other than salary income and winning
from lotteries, cross word puzzles, etc.
5. Unabsorbed depreciation can be carried forward even return of income has not been filed.

2.3.16 Mandatory provision of Depreciation


From the A.Y. 2002-03, if all conditions of sec. 32 are satisfied, depreciation shall be available whether the assessee
has claimed the same or not.

2.3.17 Depreciation in case of amalgamation, demerger or succession


In the year of -
 Amalgamation;
 Demerger;
 Succession [referred in sec. 47(xiii), (xiiib) & (xiv) or sec. 170]
depreciation u/s 32 shall be apportioned between –
 the amalgamating company and the amalgamated company
 the demerged company and the resulting company
 the predecessor and the successor
- in the ratio of number of days for which the asset was used by them

The Institute of Cost Accountants of India 197


Direct Taxation

2.3.18 Special deduction for assessee engaged in Tea, Coffee or Rubber growing &
manufacturing business [Sec. 33AB and Rule 5AC]
Applicable to All assessee carrying on business of growing and manufacturing of the followings in India:
a. Tea; b. Coffee; or c. Rubber
Conditions to be 1. Deposit of amount: Assessee must deposit (hereinafter referred to as special account) an
satisfied amount in:
¾¾ National Bank for Agriculture & Rural Development (NABARD) in an account
maintained by him in accordance with, and for the purpose specified in the scheme
approved by Tea Board, Coffee Board or Rubber Board, as the case may be; or
¾¾ An account in accordance with, and for the purpose specified in a scheme approved by
Tea Board or Coffee Board or Rubber Board, as the case may be, with prior approval
of the Central Government.
2. Time of deposit: The amount must be deposited within 6 months from the end of the
previous year or before the due date of furnishing the return of income, whichever is
earlier.
3. Audit of accounts: Accounts of assessee should be audited by a chartered accountant &
the report of an auditor in Form 3AC is required to be uploaded one month prior to the
due date of filing of return
Note: In case, where the assessee is required under any other law to get his accounts
audited, it shall be sufficient compliance if such assessee gets the accounts audited under
such law and furnishes the report in Form 3AC.
Quantum of Minimum of the following -
Deduction
a. Amount so deposited (as discussed above); or
b. 40% of the profit of such business computed under the head “Profits & gains of business
or profession” before allowing any deduction u/s 33AB and before adjusting brought
forward business loss.

Other points

1. Excess Deposit: Any excess deposit made during a previous year is not treated as deposit made for the next
year(s).

2. Restriction on utilisation of amount for certain purposes: No deduction shall be allowed in respect of any
amount, being credited in special account, utilised for the purpose of:
¾¾ Purchase of plant or machinery to be installed in any office premises / residential accommodation /
accommodation in the nature of guest-house.
¾¾ Purchase of any office appliances (other than computer)
¾¾ Purchase of any plant or machinery, the entire cost of which is allowed as deduction in form of depreciation
or otherwise in any one previous year.
¾¾ Purchase of any plant or machinery to be installed in an industrial undertaking for constructing,
manufacturing or producing any items specified in Schedule XI of the Act.
Note: If any amount is so utilised, then the whole of such amount so utilised shall be deemed to be the profits
and gains of business of the previous year in which such misutilisation takes place.

198 The Institute of Cost Accountants of India


Heads of Income

3. Withdrawal from account:

During continuation of business: The amount credited to such special account shall be withdrawn only for
the purpose(s) specified in respective schemes.

If the amount so withdrawn is not utilised for the specified purpose in the same previous year then the amount
not so utilised shall be treated as income of the year.

On closure of business: Apart from the specified purpose(s) of scheme, the amount deposited may be
withdrawn in the following circumstances: -

Case Tax Treatment


Closure of business# Fully taxable
Dissolution of firm# Fully taxable
Death of the tax payer $ Not taxable
Partition of Hindu Undivided Family $ Not taxable
Liquidation of company $ Not taxable
#
The amount withdrawn shall be taxable under the head “Profits & gains of business or profession” as if the
business is continued or the firm had not been dissolved.
$
It is not taxable even though the amount has not been utilised for any of the purposes specified in the scheme.

4. Double deduction is not permissible

¾¾ Where an amount standing to the credit of the assessee in the special account is utilised by the assessee
for the purposes of any expenditure in connection with such business in accordance with the scheme, then
such expenditure shall not be allowed in computing the income chargeable under the head ‘Profit and gains
of business or profession’.
¾¾ Where the assessee is a firm, AOP or BOI, then deduction under this section shall not be allowed in
computation of income of any partner/member.
¾¾ Where any deduction in respect of an amount deposited in any special account has been allowed in any
previous year, no deduction shall be allowed in respect of such amount in any other previous year.
5. Restriction on sale of new asset: If any asset is acquired as per the scheme, then such asset cannot be sold or
transferred within 8 years from the end of the previous year in which it was acquired. If such asset is sold or
otherwise transferred, then such part of the cost of such asset as is relatable to the deduction allowed earlier
under this section will be treated as profit.

However, in the following cases, above provision shall not be applicable -

¾¾ Sale or transfer to the Government, local authority, statutory corporation or Government company.
¾¾ Sale or otherwise transfer, in connection with the succession of a firm by a company, provided the following
conditions are satisfied –
a. All assets & liabilities of firm (immediately before succession) become the assets & liabilities of the
company.
b. All shareholders of the company were partners of the firm immediately before the succession.
c. The scheme continues to apply to the company in the manner applicable to the firm.

The Institute of Cost Accountants of India 199


Direct Taxation

6. Calculation of taxable income

Nature of Business Calculation of Income


Tea growing & manufacturing business (Rule 8) 40% of [Income from business – Deduction u/s
33AB]
Coffee growing & manufacturing business (Rule 25% / 40% of [Income from business – Deduction
7B) u/s 33AB]
Rubber growing & manufacturing business (Rule 35% of [Income from business – Deduction u/s
7A) 33AB]

2.3.19 Site Restoration Fund [Sec. 33ABA & Rule 5AD]


Applicable to All assessee engaged in the business of -
¾¾ Prospecting for petroleum or natural gas; or
¾¾ Extraction or production of petroleum or natural gas; or
¾¾ Both
- in India.
Conditions to be 1. Agreement with the Central Government: The Central Government has entered into
satisfied an agreement with the assessee for such business.
2. Deposit of amount: The assessee must deposit an amount -
¾¾ With the State Bank of India in an account (herein after referred to as special
account) maintained
¾¾ in an account (hereinafter referred as Site Restoration Account) opened by the
assessee
- in accordance with and for the purposes specified in a scheme approved by the
Government of India in the Ministry of Petroleum & Natural Gas.
Treatment of interest: Any amount credited in the special account or site restoration
account by way of interest shall be deemed to be a deposit.
3. Time of deposit: Such amount must be deposited before the end of the previous year.
4. Audit of books of account: Accounts must be audited & auditor’s report should be
filed in Form 3AD is required to be uploaded one month prior to the due date of filing
of return
Note: In case, where the assessee is required under any other law to get his accounts
audited, it shall be sufficient compliance if such assessee gets the accounts audited
under such law and furnishes the report of audit required under such other law and a
report in Form 3AD.
Quantum of Minimum of the following:
Deduction a. Amount so deposited (as discussed above); or
b. 20% of the profit of such business computed under the head “Profits & gains of
business or profession” before allowing deduction under this section and before
adjusting brought forward business loss.

200 The Institute of Cost Accountants of India


Heads of Income

Other points
1. Excess Deposit: Any excess deposit made during a previous year is not treated as deposit made for the next
year(s).

2. Restriction on utilisation of amount for certain purposes: No deduction shall be allowed in respect of any
amount, being credited in special account or site restoration account, utilised for the purpose of -

¾¾ Purchase of plant and machinery to be installed in any office premises / residential accommodation /
accommodation in the nature of guest-house.
¾¾ Purchase of office appliances (other than computer)
¾¾ Purchase of a plant or machinery, the entire cost of which is allowed as deduction in the form of depreciation
or otherwise in computation of business income of any one previous year.
¾¾ Purchase of a plant or machinery to be installed in an industrial undertaking for constructing, manufacturing
or producing any items specified in Schedule XI of the Act.
Note: If any amount is so utilised, then the whole of such amount shall be deemed to be the profit and gains of
business of the previous year in which such mis-utilisation takes place

3. Withdrawal from account

During continuation of business: The amount credited to such special account or the site restoration account
shall be withdrawn only for the purpose(s) specified in respective scheme.

If the amount withdrawn in a year is not utilised for the specified purpose in the same previous year then the
amount not so utilised shall be treated as income of the year.
On closure of account: Where any amount standing to the credit of the assessee in the special account or in
the site restoration account is withdrawn on closure of the account during any previous year, the following
amount shall be deemed to be the profits & gains of business or profession (whether business is continued or
not) -
Particulars Amount
Amount so withdrawn from the account ****
Less: Amount, if any, payable to the Central Government by way of profit or
(****)
production share as provided in the agreement u/s 42
Taxable amount ****
Note: In case of closure of business, the amount stated above shall be taxable as if the business is in existence.
4. Double deduction is not permissible
¾¾ Where any amount standing to the credit of the assessee in the special account or site restoration account is
utilised by the assessee for the purpose of any expenditure in connection with such business in accordance
with the scheme, then such expenditure shall not be allowed in computing the income chargeable under the
head ‘Profit and gains of business or profession’.
¾¾ Where the assessee is a firm, AOP or BOI, the deduction under this section shall not be allowed in the
computation of the income of any partner/member.
¾¾ Where any deduction in respect of any amount deposited in any special account or site restoration account
has been allowed in any previous year, then no deduction shall be allowed in respect of such amount in
any other year.
5. Restriction on sale of such asset: If any asset is acquired as per the scheme, then such asset cannot be sold

The Institute of Cost Accountants of India 201


Direct Taxation

or transferred within 8 years from the end of the previous year in which it was acquired. If such asset is sold
or otherwise transferred, then such part of the cost of the asset as is relatable to the deduction allowed shall be
treated as taxable profit under the head “Profits & gains of business or profession”, in the year in which the
asset is transferred. However, in the following cases, the provision shall not be applicable -
¾¾ Sale or otherwise transfer to the Government, local authority, statutory corporation or Government
Company.
¾¾ Sale or otherwise transfer, in connection with the succession of a firm by a company, subject to following
conditions:
a. All assets and liabilities of the firm, immediately before the succession became assets and liabilities
of the company.
b. All the shareholders of the company were the partners of the firm immediately before succession.
c. The scheme continues to apply to the company in the manner applicable to the firm
2.3.20 Scientific Research [Sec. 35]
Scientific research means any activity for the extension of knowledge in the fields of natural or applied science
including agriculture, animal husbandry or fisheries [Sec. 43(4)]
Such research can be categorised either as -
a. In-House research : Research done by the assessee himself (in connection with his business)
b. Research through : Any sum paid to outside agencies, engaged in scientific research, to be used for
outside institutions scientific research
In-House research
Revenue After Where the assessee himself carries on scientific research related to his
expenditure commencement of business and incurs revenue expenditure, such expenses are allowed
business as deduction in the year in which such expenditure is incurred by the
sec. 35(1)(i)
assessee.
Before Following revenue expenditures (certified by the prescribed authority)
commencement of incurred during 3 years immediately before commencement of business,
business shall be allowed as deduction in the year of commencement of business –
¾¾ Payment of salary to an employee engaged in scientific research
(excluding perquisite).
¾¾ Purchase of materials used for scientific research.
Capital After Any capital expenditure incurred (other than land) for scientific research,
Expenditure commencement of related to the business of the assessee, will be allowed as deduction in
business full. 100% deduction shall be allowed for such capital expenditure, in the
sec.35(1)(iv)
year in which the expenditure is so incurred.
/sec.35(2)
Before Any capital expenditure incurred (other than land) during 3 years
commencement of immediately preceding the year of commencement of business shall
business be deemed to have been incurred in the year in which the business
commenced and is allowed as deduction in that year.

Note: Where a deduction is allowed in any previous year in respect of any capital expenditure
for scientific research, no deduction u/s 32 shall be allowed on such assets. [Sec. 35(2)(iv)]

202 The Institute of Cost Accountants of India


Heads of Income

Notes: In-house research for a purpose not related to the business of the assessee shall not be allowed as deduction.

In-house research & development expenses incurred by certain companies [Sec. 35(2AB)]

Applicable to Company engaged in the business of bio-technology or any business of manufacture or production
of any article or thing (other than those specified in the 11th Schedule)

Conditions to 1. The expenditure shall be incurred on in-house scientific research and development facility
be satisfied including capital expenditure (other than expenditure in the nature of cost of any land or
building).
2. In-house research and development facility shall be approved by the Secretary, Department
of Scientific and Industrial Research.
3. The assessee must enter into an agreement with the prescribed authority -
¾¾ for co-operation in such research and development facility; and
¾¾ fulfils such conditions with regard to maintenance of accounts and audit thereof and
furnishing of reports in such manner as may be prescribed.

100% of the expenditure incurred on in-house research and development facility shall be allowed.
Notes
Deduction  If the above conditions are not satisfied, then deduction may be claimed as per the provision
of sec. 35(1)(i) or sec. 35(2).
 Where deduction is allowed in any previous year in respect of any capital expenditure under
this section, then no deduction u/s 32 shall be allowed on such asset.

The Institute of Cost Accountants of India 203


Direct Taxation

Research through outside institutions


Deduction @ 100% shall be allowed in respect of expenditure on Research through following outside institution:

Institution Purpose
Any payment to National Laboratory or a University Scientific research undertaken under programme
or Indian Institute of Technology or a specified person. approved by the prescribed authority (whether related
[Sec. 35(2AA)] to business or not)
Any payment made to a notified (by the Central
Government) research association or to an approved Scientific research (whether related to business or not)
university, college or other institutions3 [Sec. 35(1)(ii)]
Any payment made to a notified (by the Central
Research in Social science or Statistical Research
Government) research association, university, college or
(whether related to business or not)
other institution3 [Sec. 35(1)(iii)].
Any payment to an approved Indian company (main
object of whom is scientific research & development)3 Scientific research (whether related to business or not)
[Sec. 35(1)(iia)]
1.
National laboratory means a scientific laboratory functioning at the national level under the aegis of the Indian
Council of Agricultural Research, the Indian Council of Medical Research, the Council of Scientific and Industrial
Research, the Defence Research and Development Organisation, the Department of Electronics, the Department
of Bio-Technology or the Department of Atomic Energy and which is approved as National Laboratory by the
prescribed authority.
2.
Such association, University, college or institution must be approved in accordance with prescribed guidelines
and must be notified by the Central Government.
3.
The deduction in respect of any sum paid to the research association, university, college or other institution or
company shall be allowed on the basis of a certificate issued by the donee.
Other points
1. Such association, University, college or institution must be approved in accordance with prescribed guidelines
and must be notified by the Central Government.
2. Withdrawal of approval: Deduction shall not be denied merely on the ground that subsequent to the payment
made by the assessee, the approval granted to the association, university, IIT, etc. has been withdrawn.
3. Carry forward of unabsorbed scientific research expenditure: Unabsorbed capital expenditure can
be carried forward for unlimited years and set off in any subsequent assessment year(s) like unabsorbed
depreciation.
4. Effect of amalgamation [Sec. 35(5)]: Provisions of sec. 35 shall apply to the amalgamated company, as it
would have been applied to the amalgamating company, if the latter had not transferred such asset.

Illustration 71:
Dynamic India & Co. commences production on 16/8/2023. It incurred the following expenses related to scientific
research, find deduction u/s 35 for the P.Y. 2023-24.

Date Particulars Amount Purpose


18/8/2023 Paid to an Approved University for research in Social science 50,000 Non- business
15/10/2023 Paid to a scientist (not the employee of the company) 30,000 Business

204 The Institute of Cost Accountants of India


Heads of Income

Date Particulars Amount Purpose


18/11/2023 Paid to approved National laboratory 60,000 Non- business
Purchase of land & building for in house research (cost of land
15/12/2023 5,00,000 Business
₹ 1,50,000)
18/12/2023 Purchase of car to carry research-workers 2,00,000 Business
16/8/2020
to Capital expenditure (including cost of land ₹ 1,00,000) 5,00,000 Business
15/8/2023
“ Purchase of material 3,00,000 Business
“ Payment of salary (other than Perquisites) 2,00,000 Business
“ Perquisites provided to research-personnel 1,00,000 Business
“ Other revenue expenditure 80,000 Business
1/4/2020 to
Capital expenditure (other than land) 9,00,000 Business
15/8/2020
“ Payment of salary (other than Perquisites) 50,000 Business
1/4/2018 to
Capital expenditure 80,000 Business
31/3/2020
“ Revenue expenditure 30,000 Business

Solution :

Computation of deduction u/s 35 to Dynamic India & Co. for the A.Y. 2024-25

Date Particulars Section Amount Deduction


1/4/2018 to Capital expenditure NA1 80,000 Nil
31/3/2020 Revenue expenditure NA1 30,000 Nil
1/4/2020 to Capital expenditure (other than land) NA1 9,00,000 Nil
15/8/2021 Payment of salary (other than Perquisites) NA1 50,000 Nil
Capital expenditure (excluding cost of land ₹ 1,00,000) 35(2) 5,00,000 4,00,000
Payment of salary (other than Perquisites) 35(1)(i) 2,00,000 2,00,000
16/8/2020
to Perquisites provided to research-personnel NA2 1,00,000 Nil
15/8/2023
Purchase of material 35(1)(i) 3,00,000 3,00,000
Other Revenue expenditure NA2 80,000 Nil
18/8/2023 Paid to an Approved University for research in Social
science
35(1)(iii) 50,000 50,000

15/10/2023 Paid to a scientist (not the employee of the company) 35(1)(i) 30,000 30,000
18/11/2023 Paid to approved National laboratory 35(2AA) 60,000 60,000
15/12/2023 Purchase
1,50,000)
of land & building (excluding cost of land ₹ 35(2) 5,00,000 3,50,000

18/12/2023 Purchase of car to carry research-workers 35(2) 2,00,000 2,00,000


Total amount of deduction 15,90,000

The Institute of Cost Accountants of India 205


Direct Taxation

Note
1.
Expenditure (whether revenue or capital) incurred before 3 years immediately preceding the year of commencement
of business shall not be allowed as deduction.
2
Expenditure incurred within 3 years prior to commencement of business: Only material and salary (other than
perquisite) of research personnel shall be allowed as deduction

Sale of asset used for scientific research [Sec. 41(3)]

Sale consideration to the extent of cost of such asset shall be taxable as business income
Without having been in the year of sale. The excess of sale consideration over original cost (or indexed cost of
used for other purpose acquisition) is taxable as capital gain u/s 45. This is applicable even if the business is not
in existence in that year)
Sale consideration shall be subtracted from relevant block of assets. It is to be noted that
After being used for
at the time of conversion of scientific research asset into normal business asset, the cost
other purposes
of acquisition shall be taken as nil in the relevant block.

Illustration 72:
Awishkar Enterprises purchased machinery for ₹ 5,00,000 as on 18/8/2022 for scientific research.
On 17/7/2023, the research work being completed. On 31/3/2024, the machinery being sold for -
Case 1) ₹ 1,00,000
a. After using the same for business purpose other than scientific research. The WDV of the respective block is
₹ 4,80,000. Depreciation rate 15%.
b. Without using the same for any other purpose
Case 2) ₹ 7,00,000
a. After using the same for business purpose other than scientific research. The WDV of the respective block is
₹ 4,80,000. Depreciation rate 15%.
b. Without using the same for any other purpose
State tax implications

Solution :
Tax impact in case 1(a) and 2(a)
Particulars Case 1(a) Case 2(a)
Opening WDV of the block 4,80,000 4,80,000
Add: Addition during the year being machinery earlier used for scientific research Nil Nil
Less: Sale value of the machinery [ Max. to the extent of (Opening WDV +
#
(1,00,000) (4,80,000)#
Addition made)]
Written down value before charging depreciation 3,80,000 Nil
Less: Depreciation @ 15% (57,000) Nil
Written down value after charging depreciation 3,23,000 Nil
Short term capital gain [@ Excess sale proceeds] Nil 2,20,000@

206 The Institute of Cost Accountants of India


Heads of Income

Tax impact in case 1(b) and 2(b)


Case 1(b) Case 2(b)
Particulars
Details Amount Details Amount
Profits & gains of business or profession
Being minimum of the following
• Earlier deduction claimed 5,00,000 5,00,000
• Surplus i.e. {Sale consideration – (Cost of assets –
Earlier deduction allowed in respect of such asset)} 1,00,000 1,00,000 7,00,000 5,00,000
Capital gains
Sale consideration NA1 7,00,000
Less: Cost of acquisition NA1 (5,00,000)
Short-term capital gain (loss) NA1 2,00,000
1Capital gain shall arise only if the sale price exceeds the cost of the asset held for scientific research [CIT –vs.-
Artex Mfg. Co. (SC)]

2.3.21 Deduction in respect of expenditure on specified business [Sec. 35AD]


Applicable to Specified assessee engaged in the business of:
a. setting up and operating a cold chain facility##;
b. setting up and operating a warehousing facility for storage of agricultural produce; or
c. laying and operating a cross-country natural gas or crude or petroleum oil pipeline network
for distribution, including storage facilities being an integral part of such network
Note: The project has been approved by the Petroleum and Natural Gas Regulatory Board
and being notified by the Central Government.
d. building and operating, anywhere in India, a hotel of two-star or above category as classified
by the Central Government;
e. building and operating, anywhere in India, a hospital with at least 100 beds for patients;
f. developing and building a notified housing project under a scheme for slum redevelopment
or rehabilitation framed by the Central Government (or a State Government)
g. developing and building a notified housing project under a scheme for affordable housing
framed by the Central Government (or a State Government)
h. production of fertilizer in India;
i. setting up and operating an inland container depot or a container freight station notified or
approved under the Customs Act, 1962;
j. bee-keeping and production of honey and beeswax;
k. setting up and operating a warehousing facility for storage of sugar
l. laying and operating a slurry pipeline for the transportation of iron ore
m. setting up and operating a semi-conductor wafer fabrication manufacturing unit, and which
is notified by the Board in accordance with such guidelines as may be prescribed
n. developing or maintaining and operating or developing, maintaining and operating a new
infrastructure facility by an Indian company (or consortium thereof) / authority / board
/ corporation having agreement with Central Government or State Government or local
authority or any statutory body

The Institute of Cost Accountants of India 207


Direct Taxation

Conditions to Cross-country oil pipeline Other Business


be satisfied Owned by An Indian company or by a consortium of such Any assessee
companies or by an authority or a board or a corporation
established or constituted under any Central/State Act
Date of On or after 01-04-2007 On or after date
commencement given in Note*
of business below
Restriction on It has made not less than such proportion of its total No Restriction
usage pipeline capacity as specified by the Petroleum and
Natural Gas Regulatory Boardβ available for use on
common carrier basis by any person other than the
assessee or an associated person#.
New Business Such business should not be set up by splitting up, or the reconstruc­tion, of
a business already in existence.
New Plant Such business should not be set up by the transfer to the specified business
Machinery of machinery or plant previously used for any purpose
Exceptions
i. A plant or machinery is deemed as new asset if:
a. Such plant or machinery is imported into India;
b. Depreciation on such asset has not been allowed under this Act to
any person; and
c. The assessee was the first user of such asset in India.
ii. Where the total value of old plant and machinery transferred to the new
business does not exceed 20% of total value of plant and machinery
used in such business, then this condition is deemed to be satisfied.
Audit Books of account should be audited by a Chartered Accountant
Quantum of 100% of capital expenditure incurred during the previous year, wholly and exclusively, for the
deductions purposes of any specified busi­ness.
Capital Expenditure shall not include the following:
a. Any expenditure incurred on the acquisition of any land or goodwill or financial instrument
b. Any expenditure in respect of which the payment (or aggregate of payments made to a
person in a day), otherwise than by an account payee cheque drawn on a bank or an account
payee bank draft or use of electronic clearing system through a bank account or through
other prescribed electronic mode, exceeds ₹ 10,000
Pre-commencement expenditure: Where the expenditure is incurred prior to the commencement
of its operations which has been capitalised in the books of account of the assessee on the date
of commencement of its operations, shall be allowed as deduction in the previous year in which
the assessee commences such business.

*
Date of commencement of business (on or after)
Warehousing facility Hotel Slum redevelopment Fertilizer Inland container depot Warehousing
for storage of 01-04-2010 01-04-2010 01-04-2011 01-04-2012 facility for

agricultural produce Hospital Affordable housing Cold chain facility Bee-keeping for storage of
01-04-2009 01-04-2010 01-04-2011 01-04-2009 01-04-2012 Sugar

Slurry pipeline for transportation of iron-ore semi-conductor wafer fabrication manufacturing unit Infrastructure facility 01-04-2012
01-04-2014 01-04-2014 01-04-2017

208 The Institute of Cost Accountants of India


Heads of Income

Other Points ¾¾ Option: The deduction under this section is optional in nature. For claiming deduction
under this section, assessee is required to claim the same.
¾¾ No Double Deduction: No deduction for such expenditure shall be allowed to the assessee
under any other section in any previous year or under this section in any other previous year.
Further, the assessee shall not be allowed any deduction u/s 10AA or 80HH to 80RRB in
respect of the specified business for the same or any other assessment year.
¾¾ Restriction of use of the asset: Any asset in respect of which a deduction is claimed and
allowed under this section shall be used only for the specified business, for a period of 8
years beginning with the previous year in which such asset is acquired or constructed.
Consequences of usage of asset otherwise than for specified purpose: Where such asset
is used for a purpose other than the specified business during that period, then following
amount shall be deemed to be the income of the assessee chargeable under the head
“Profits and gains of business or profession” of the previous year in which the asset is so
used:
The total amount of deduction so claimed and allowed in one or more previous ***
years
Less: Depreciation allowable in accordance with the provisions of section 32, as ***
if no deduction under this section was allowed
Deemed Income ***
However, the provision of reversal of deduction shall not be applied to a company which
has become a sick industrial company u/s 17(1) of the Sick Industrial Companies (Special
Provisions) Act, 1985, during that period.
¾¾ Actual cost of asset for depreciation: The actual cost of any capital asset for the purpose
of computing depreciation, on which deduction has been allowed to the assessee u/s 35AD,
shall be treated as ‘nil’.
¾¾ Treatment of Realisation: If the whole of the expenditure on capital asset has been allowed
as a deduction u/s 35AD, any sum received or receivable (in cash or kind) on account of
such capital asset being demolished, destroyed, dis¬carded or transferred shall be taxable
as business income.
¾¾ Carry forward and set off of losses [Sec. 73A]: Any loss, computed in respect of such
specified business shall be set off only against profits and gains, if any, of any other specified
busi¬ness. Further, if there is no such profit or such loss is not fully adjusted with such
profit, the unabsorbed loss shall be carried forward for set off against the profits and gains, if
any, of any specified business in the next assessment year and so on. [Further Refer Chapter
‘Set-off and Carry Forward]
¾¾ Transfer of operation: Where the assessee builds a hotel of two-star or above category
as classified by the Central Government and subsequently, while continuing to own the
hotel, transfers the operation thereof to another person, the assessee shall be deemed to be
carrying on the specified business.
¾¾ Inter-unit transfer: Where -

The Institute of Cost Accountants of India 209


Direct Taxation

1. Assessee carries on at least two units


2. Out of such units at least one is eligible u/s 35AD and at least one is not eligible for
exemption
3. Goods or services are transferred from eligible unit to any non eligible unit or vice
versa
4. The consideration for such transfer does not correspond to the market value of such
goods as on the date of transfer
then, deduction shall be computed as if the transfer, in either case, had been made at the
market value$ of such goods or services as on that date.
#
An “associated person”, in relation to the assessee, means a person:
a. who participates, directly or indirectly, or through one or more intermediaries in the management or control or
capi­tal of the assessee;
b. who holds, directly or indirectly, 26% of equity share capital of the assessee;
c. who appoints more than half of the Board of direc­tors or members of the governing board, or one or more
executive directors or executive members of the governing board of the assessee (it is to be noted that
appointing power does not suffice the purpose); or
d. who guarantees not less than 10% of the total borrowings of the assessee;
##
“Cold chain facility” means a chain of facilities for storage or transportation of agricultural and forest produce,
meat and meat products, poultry, marine and dairy products, products of horticulture, floriculture and apiculture
and proc­essed food items under scientifically controlled conditions including refrigeration and other facilities
necessary for the preservation of such produce.
$
Market value in relation to any goods or services

Case Market value means


Sold or The price that such goods or services would fetch if these were sold by the unit in the open market,
supplied subject to statutory or regulatory restrictions, if any
Acquired The price that such goods or services would cost if these were acquired by the unit from the open
market, subject to statutory or regulatory restrictions, if any.
β
The Petroleum and Natural Gas Regulatory Board has specified following conditions for common carrier:

Following proportion of total pipeline capacity should available for


Particulars use on common carrier basis by any person other than the assessee
or an associated person
Natural gas pipeline network 1/3rd of total pipeline capacity
Petroleum product pipeline network 1/4th of total pipeline capacity
“Infrastructure facility” means:
i. road including toll road, a bridge or a rail system;
ii. highway project including housing or other activities being an integral part of the highway project;
iii. water supply project, water treatment system, irrigation project, sanitation and sewerage system or solid waste
management system;
iv. port, airport, inland waterway, inland port or navigational channel in the sea

210 The Institute of Cost Accountants of India


Heads of Income

2.3.22 Amortisation of telecom-licence fee [Sec. 35ABB]


Applicable to All assessee
a. Assessee has incurred capital expenditure for acquiring any right to operate
telecommunication services.
b. Payment for such expenditure has actually been made.
Conditions to be Notes
satisfied
1. Such expenditure may be incurred before or after commencement of business.
2.Revenue expenditure incurred relating to telecom licence fee shall not be eligible
for deduction u/s 35ABB. However, assessee can claim deduction u/s 37(1) for such
expenditure.
Actual expenditure incurred and paid shall be allowed as deduction in equal installments
over the period for which the license remains in force starting from the year as under -
Case Period starts from
Deduction u/s Where the license-fee is paid before the The previous year in which such business
35ABB(1) commencement of business. commenced.
When license is acquired after
The previous year in which license fee has been
commencement of business
actually paid.
In any other case
Note: No depreciation is allowed on such capital expenditure.

Where such license is sold in full u/s 35ABB(2) & (3)


 Loss on sale shall be deductible as business loss in the year of sale.
 Profit on sale, to the extent of aggregate of deduction allowed in preceding year(s) shall be treated as business
income.
Capital gain treatment: The excess of sale consideration over original cost (or indexed cost of acquisition) is
taxable as capital gain u/s 45.
Where such licence is transferred in a scheme of amalgamation or demerger
The amalgamated company or resulting company (being Indian company) as the case may be shall be entitled to
claim deduction u/s 35ABB for the residual period as if the amalgamating or demerged company had not transferred
the licence.

Similar, deduction is also available u/s 35ABA for capital expenditure incurred for acquiring any right to use
spectrum for telecommunication services

Illustration 73:
Telefast Ltd., a company providing telecommunication services, obtain a telecom licence on 20-4-2023 for a period
of 10 years which ends on 31-3-2033 (licence fee being ₹ 18 lakh). Find out the amount of deduction u/s 35ABB
of the Income Tax Act, 1961, if:
(a) the entire amount is paid on 6-5-2023; (b) the entire amount is paid on 1-4-2024;
(c) the entire amount is paid in equal installments on 30-4-2023; 30-4-2024 and 30-4-2025
Solution :
Tax consequence u/s 35ABB in several previous years

The Institute of Cost Accountants of India 211


Direct Taxation

Particulars Case (a) Case (b) Case (c)


Deduction u/s 35ABB(1) in previous year:
2023-24 1,80,000 NA 60,0002
2024-25 1,80,000 2,00,0001 1,26,6672
2025-24 to 2032-33 1,80,000 2,00,000 2,01,6672
1. In case (b), though licence was acquired in the previous year 2023-24 but payment was made in the previous
year 2024-25, hence deduction shall be available u/s 35AAB(1) in 9 years (10 year – 1 year) starting from the
previous year 2024-25. Amount of deduction shall be ₹ 18,00,000 / 9 = ₹ 2,00,000.
2. In case (c), payment has been made in three installments. Hence deduction u/s 35ABB(1) shall be available as
under:

Particulars 1st Installment 2nd Installment 3rd Installment Total


Amount of installment 6,00,000 6,00,000 6,00,000
Remaining life (in years) 10 9 8
Deduction available in
- P.Y.: 2023-24 60,000 - - 60,000
- P.Y.: 2024-25 60,000 66,667 - 1,26,667
- P.Y.: 2025-26 to 2032-33 60,000 66,667 75,000 2,01,667

Illustration 74:
Twinkle Enterprises has acquired a telecom licence. Details in respect of such licence are as under:

Particulars Particulars
Acquisition cost ₹ 1,00,000 Life of licence 10 years
Date of purchase 16/8/2022 Licence sold 100%
Payment terms Lump sum Date of sale of licence 15/3/2025
Date of first payment 16/8/2022 Sale value ₹ 1,20,000

State the tax consequence in the several previous years up to 2024-25 related to such transactions.
Solution :
Tax consequence u/s 35ABB in several previous years up to 2024-25
Particulars Licence A
Deduction u/s 35ABB(1) in previous year:
2022-23 10,000
2023-24 10,000
2024-25 Nil1
Business income on sale of licence u/s 35ABB(3) in the P.Y.2024-25 20,0002
Capital gain on sale of licence in the P.Y.2024-25 20,0002
1. No deduction is available in the previous year in which licence is 100% sold or otherwise transferred.
2. Sale of licence

212 The Institute of Cost Accountants of India


Heads of Income

Particulars Details Amount


Profits & Gains of Business or Profession
Being minimum of the following
 Earlier deduction claimed for the P.Y. 2022-23 and 2023-24 20,000
 Surplus i.e. {Sale proceeds – (Cost of assets - Earlier deduction allowed in respect
of such asset)} [1,20,000 – (1,00,000 – 20,000) = 40,000] 40,000 20,000
Capital gains
Sale consideration 1,20,000
Less: Cost of acquisition 1,00,000
Short-term capital gain (as asset is not held for more than 3 years) 20,000

Illustration 75:
Tweety Enterprises has acquired telecom licence. Details in respect of this licence are as under:

Particulars Particulars
Acquisition cost ₹ 3,00,000 Life of licence 7 years
Date of purchase 14/7/2020 Licence sold 40%
Payment terms Lump sum Date of sale of licence 12/12/2022
Date of first payment 14/7/2021 Sale value ₹ 1,20,000
State the tax consequence in the several previous years up to 2023-24 related to such transactions.
Solution :
Tax consequence u/s 35ABB in several previous years up to 2023-24
Particulars Amount
Deduction u/s 35ABB(1) in previous year:
2020-21 Nil1
2021-22 50,0001
2022-23 50,0001
2023-24 20,0002
Business income on sale of licence u/s 35ABB(3) in the P.Y.2023-24 Nil
Capital gain on sale of licence in the P.Y.2023-24 Nil
1. Though licence was acquired in the previous year 2020-21 but payment was made in the previous year 2021-
22, hence deduction shall be available u/s 35AAB(1) in 6 years (7 year – 1 year) starting from the year 2021-
22. Amount of deduction will be ₹ 3,00,000/6 = ₹ 50,000.
2. Sale of part of licence B

Particulars Details Amount


Balance of licence as on 1/4/2022 ₹ 3,00,000 – ₹ 1,00,000 2,00,000
Less: Sale of licence 1,20,000
Balance value 80,000
Balance life 4 years
Deduction in the previous year 2023-24 ₹ 80,000 / 4 20,000

The Institute of Cost Accountants of India 213


Direct Taxation

2.3.23 Payment to associations and institutions for carrying out rural development
programmes [Sec. 35CCA]
Where an assessee incurs any expenditure by way of payment of any sum—
a. to an association or institution, which has as its object the undertaking of any programme of rural development,
to be used for carrying out any programme of rural development approved by the prescribed authority and the
assessee furnishes a certificate from such association or institution ; or
b. to an association or institution, which has as its object the training of persons for implementing programmes
of rural development and the assessee furnishes a certificate from such association or institution; or
c. the National fund for rural development; or
d. to the National Urban Poverty Eradication Fund set up and notified by the Central Government in this behalf,
the assessee shall, be allowed a deduction of the amount of such expenditure incurred during the previous year.
Double deduction is not permissible: Where a deduction under this section is claimed and allowed for any
assessment year in respect of any expenditure, deduction shall not be allowed in respect of such expenditure u/s
80G or any other provision of this Act.
Withdrawal of approval: Deduction shall not be denied merely on the ground that subsequent to the contribution
made by the assessee, the approval granted to such programme, etc. has been withdrawn.

2.3.24 Expenditure on agricultural extension project [Sec. 35CCC]


Where an assessee incurs any expenditure on notified agricultural extension project, such expenditure is fully
allowed.
Double deduction is not permissible: Where a deduction under this section is claimed and allowed for any
assessment year in respect of any expenditure, deduction shall not be allowed in respect of such expenditure under
any other provisions of this Act for the same or any other assessment year.

2.3.25 Expenditure on skill development project [Sec. 35CCD]


Where a company incurs any expenditure (not being expenditure in the nature of cost of any land or building) on
any notified skill development project, such expenditure is fully allowed.
Double deduction is not permissible: Where a deduction under this section is claimed and allowed for any
assessment year in respect of any expenditure, deduction shall not be allowed in respect of such expenditure under
any other provisions of this Act for the same or any other assessment year.

2.3.26 Amortisation of preliminary expenses [Sec. 35D & Rule 6AB]


Meaning: Following expenses are known as preliminary expenses -
1. Expenses in connection with –
¾¾ Preparation of project report;
¾¾ Preparation of feasibility report;
¾¾ Conducting market survey or any other survey necessary for the business;
¾¾ Engineering services related to the business.
Note: Above work must be carried on by the assessee himself or by any concern.

214 The Institute of Cost Accountants of India


Heads of Income

2. Legal charges for drafting any agreement between the assessee and any other person for any purpose related to
the setting up or conduct of business of the assessee.
3. Legal charges for drafting & printing of Memorandum of Association & Articles of Association (in case of
company-assessee only).
4. Registration fees under provisions of the Companies Act, 1956 (in case of company-assessee only).
5 Expenses in connection with public issue of shares in or debentures of the company being underwriting
commission, brokerage & charges for drafting, typing, printing & advertisement of the prospectus (in case of
company-assessee only).
6. Any other prescribed expenditure.
Tax Treatment
An Indian company or a resident non-corporate assessee.
Applicable to Taxpoint: A foreign company, which is resident in India, is not covered under this
section.
1. Assessee has incurred certain amount as preliminary expense.
2. Purpose of expense
¾¾ Where such expense is incurred before commencement of business then expense
must be incurred for setting up a new undertaking or business.
¾¾ Where such expense is incurred after commencement of business then expense
Conditions must be incurred in connection with extension of any undertaking or in connection
with setting up a new unit.
3. Report of a chartered accountant: In the case of a non-corporate assessee, an
audit report from a chartered accountant should be submitted one month prior to the
due date of filing of return relating to the year in which such expenditure was first
claimed.
In case of non-corporate resident assessee 5% of the ‘cost of project1’.
Total preliminary
expense (maximum
amount) eligible for 5% of the ‘cost of project1’ or ‘capital
In case of Indian company
deduction employed2’ whichever is higher

1/5th of the total eligible preliminary expense is allowed in 5 equal annual installments
Amount of deduction starting from the year in which the business commences or unit expanded or the new unit
commences production or operation.
In case of transfer of undertaking under the scheme of amalgamation or demerger, the
amalgamated company or resulting company (being Indian company) shall be entitled to
Effect of claim deduction u/s 35D for the residual period as if the amalgamation or demerger had
amalgamation or not taken place [Sec. 35D(5) & (5A)].
demerger
Note: In the year of amalgamation or demerger, deduction shall be available to
amalgamated company or resulting company as the case may be.

1
Cost of Project

The Institute of Cost Accountants of India 215


Direct Taxation

In case of new business: Actual cost of fixed assets (being land, buildings, leaseholds, plant machinery, furniture,
fittings and railway sidings) which are shown in the books of the assessee as on the last day of the previous year in
which the business commences.
In case of an existing business: Actual cost of fixed assets (being land, building, leaseholds, plant, machinery,
furniture, fittings and railway-sidings) which are shown in the books of the assessee as on the last day of the previous
year in which the extension of industrial undertaking is completed or new industrial undertaking commences
production or operation, in so far as such fixed assets have been acquired or developed in connection with the
extension of the industrial undertaking or setting up of the new industrial unit.
2
Capital employed
In case of new business: The aggregate of issued share capital, debentures & long-term borrowings as on the last
day of the previous year in which the business commences.
In case of existing business: The aggregate of the issued share capital, debentures and long term borrowings as on
the last day of the previous year in which the extension is completed so far as such capital etc. have been issued or
obtained in connection with the extension of the business.

Illustration 76:
Jardine Ltd. is an existing Indian company, which sets up a new industrial unit. It incurs the following expenditure
in connection with the new unit:

Particulars Amount
Preparation of project report 4,00,000
Market survey 5,00,000
Legal and other charges for issue of additional capital required for the new unit 2,00,000
Total 11,00,000
The following further data is given:

Particulars Amount
Cost of project 30,00,000
Capital employed in the new unit 40,00,000
What is the deduction admissible to the company u/s 35D?
Solution :
Calculation of admissible preliminary expenditure

Particulars Amount
Cost of project (A) 30,00,000
Capital employed (B) 40,00,000

Eligible preliminary expenditure being minimum of the following:


a) Actual expenditure 11,00,000
b) 5% of (A) or (B) whichever is higher (i.e. ₹ 40,00,000 * 5%) 2,00,000
Amount eligible for amortization 2,00,000
Amount of deduction u/s 35D for current year (till 5 years) being 1/5th of ₹ 2,00,000 40,000

216 The Institute of Cost Accountants of India


Heads of Income

2.3.27 Deduction of expenses incurred in case of amalgamation or demerger [Sec. 35DD]


Applicable to: An Indian company
Conditions
1. Assessee has incurred certain expenditure wholly & exclusively for the purpose of amalgamation or demerger.
2. No deduction has been claimed for such expenses under any other section.
Quantum of deduction: 1/5th of expenses so incurred for a period of 5 years commencing from the year in which
amalgamation or demerger takes places.

2.3.28 Amortisation of expenditure incurred under VRS [Sec. 35DDA]


Applicable to: All assessee

Condition: Assessee has incurred any expenditure, by way of compensation to employees in connection with their
voluntary retirement.

Quantum of deduction: 1/5th of expenditure so paid for a period of 5 years commencing from the year in which
such expenditure was paid.

Effect of amalgamation or demerger: In case of transfer of undertaking under the scheme of amalgamation or
demerger, the amalgamated company or resulting company (being Indian company) as the case may be, shall be
entitled to claim deduction u/s 35DDA for the residual period as if the amalgamation or demerger had not taken
place.

Effect of succession of business: Where there has been eductibledn of business, whereby a firm or proprietary
concern is succeeded by a company fulfilling the conditions laid down in sec. 47 (xiii) & (xiv) or a private company
or unlisted public company is succeeded by a limited liability partnership fulfilling the conditions laid down in sec.
47 (xiiib), the provisions of this section shall apply to the successor concern, as they would have applied to the
predecessor, if eductibledn of business had not taken place. Further, it is to be noted that:

a. No deduction shall be allowed to amalgamating company, demerged company, a firm, proprietary or other
concern in the previous year in which amalgamation, demerger or succession, as the case may be, takes place.

b. No deduction shall be allowed in respect of such expenditure under any other provisions of the Act.

A comparative study of Sec. 10(10C) {under the head “Salaries”} and Sec. 35DDA {under the head “Profits
& gains of business or profession”}

 Exemption u/s 10(10C) for “Compensation for voluntary retirement” is not available to employee of the
partnership firm, HUF, proprietorship firm, etc. Deduction u/s 35DDA can be claimed by all assessee.

 Exemption u/s 10(10C) for “Compensation for voluntary retirement” is available only if the scheme is
approved by the Board. Deduction can be claimed u/s 35DDA even if the scheme is not approved by the Board.

The Institute of Cost Accountants of India 217


Direct Taxation

2.3.29 Amortisation of expenditure on prospecting etc. for development of minerals [Sec.


35E]
Applicable to Any Indian company and any other resident assessee.
Conditions to 1. Assessee is engaged in operations relating to prospecting for or extraction or production of
be satisfied mineral specified in Seventh Schedule.
2. Expenditure has been incurred by the assessee on –
a. Prospecting for any mineral specified in Seventh Schedule; or
b. Development of a mine or other natural deposit of any such mineral.
Period during which expenditure is incurred: Expenditure incurred during following pe-
riod shall qualify for deduction –
¾¾ In the previous year in which commercial production commences; and
¾¾ At any time during the period of 4 years preceding the year in which commercial
production commences.
Expenditures which are not qualified for deduction: Following expenditures do not qual-
ify for deduction:
¾¾ Expenditure on acquisition of site or any right in or over such site; or
¾¾ Expenditure on acquisition of deposits of mineral or any rights in or over such deposits;
¾¾ Expenditure of capital nature (being building, plant, machinery or furniture) in respect
of which depreciation allowance is admissible u/s 32.
¾¾ Any expenditure which is met directly or indirectly by any other person or authority and
any sale, salvage, compensation or insurance moneys realized by the assessee in respect
of any property or rights brought into existence as a result of the expenditure shall be
excluded.
3. In case of a non-corporate assessee, accounts of the assessee, for the year(s) in which the
expenditure is incurred, have been audited by a chartered accountant and the audit report in
Form 3AE (electronically) must be uploaded one month prior to the due date of filing of the
return of income of the first year in which deduction is claimed.
Quantum of Total eligible expenditure shall be allowed in 10 equal installments from the year of commer-
Deduction cial production. However, deduction in a previous year cannot exceed income (before making
deduction under this section) of the previous year arising from the commercial exploitation of
any mine(s)
Treatment of unabsorbed amount: The unabsorbed amount of installment relating to any pre-
vious year can be carried forward and added to the installment of the succeeding year.
In this manner, unabsorbed amount can be carried forward maximum up to the 10th previous year
commencing from the year when commercial production starts.

Taxpoint
 In case of transfer of undertaking (of an Indian company) in a scheme of amalgamation or demerger, the
amalgamated company or resulting company (being Indian company) shall be entitled to claim deduction u/s
35E for the residual period as if no amalgamation or demerger had taken place.

218 The Institute of Cost Accountants of India


Heads of Income

No deduction shall be allowed to amalgamating company or demerged company in the previous year in which
amalgamation or demerger takes place.
 No deduction shall be allowed in respect of such expenditure under any other provisions of this Act.

2.3.30 Insurance premium for stocks & stores [Sec. 36(1)(i)]


Any amount paid as insurance premium against risk of damage or destruction of stocks & stores, used for the
purpose of business or profession is allowable as deduction in full.
Paid means actually paid or incurred according to the method of accounting upon the basis of which the profits and
gains of business are computed. [Sec. 43(2)]

2.3.31 Insurance premium for life of cattle [Sec. 36(1)(ia)]


Any amount paid as insurance premium by a federal milk co-operative society on the lives of cattle owned by the
members of a primary milk co-operative society affiliated to it, is allowed as deduction in full.

2.3.32 Insurance premium for health of employees [Sec. 36(1)(ib)]


Any premium paid (other than by cash) by the assessee as an employer to effect or to keep in force an insurance on
the health of his employees under a scheme framed in this behalf by—
 the General Insurance Corporation of India & approved by the Central Government
 any other insurer and approved by the Insurance Regulatory and Development Authority
- shall be allowed as deduction.

2.3.33 Bonus or commission to employees [Sec. 36(1)(ii)]


Any bonus or commission (other than in lieu of profit or dividend) paid1 to employees shall be allowed as deduction.
Such amount must have been actually paid before the due date of furnishing return [Sec. 43B]

2.3.34 Interest on borrowed capital [Sec. 36(1)(iii)]


Amount of interest paid in respect of capital borrowed for the purposes of business or profession shall be allowed
as deduction.
Conditions
To claim deduction under this section following conditions must be satisfied -
a. The assessee must have borrowed money.
b. The money so borrowed must have been used for the purpose of business or profession during the previous
year.
c. The assessee must have incurred interest on the borrowed amount.
Other points
1. Interest paid to another person: Interest should be paid to another person. Hence, interest on capital to
proprietor is disallowed expenditure. However, interest on capital to partners is allowed u/s 40(b) [to be
discussed in the chapter ‘Firm Assessment’]
2. Interest paid by one unit of the assessee to another unit is not deductible.
3. Interest paid to relative is allowed as deduction subject to sec. 40A(2) i.e. if the interest paid is in excess of
market rate then excess portion shall be disallowed.

The Institute of Cost Accountants of India 219


Direct Taxation

4. Need of borrowed capital: Whether borrowed money is needed or not, is at the discretion of the assessee and
income tax authority cannot examine the same. It is sufficient that money has been borrowed and applied in the
business and the need of such borrowings cannot be challenged by the Assessing Officer
5. Interest on share capital is not allowed
6. Interest on borrowings made for acquiring & installing assets:

Interest for the period Treatment of interest


Prior to commencement of business
Interest is to be added to actual cost of the
After commencement of business but before asset is put to
asset
use
After asset is put to use Interest is allowed u/s 36(1)(iii)
7. Borrowed money used partly for business purpose: If borrowed money is utilised in earning non assessable
income, interest on such borrowing shall not be allowed as deduction.
8. Interest on money borrowed to pay income tax is not allowed
Note: Interest on money borrowed for payment of sales tax is allowed as deduction.
9. Interest paid outside India without deducting tax at source is not allowed.
10. Revenue vs Capital expenditure: Amount borrowed may be applied for the purpose of revenue expenditure
or capital expenditure.
11. Interest on money borrowed for investing as capital in partnership firm is allowed as deduction. Brokerage
& Commission for arranging loan paid to an agent is not allowed under this section but allowed u/s 37(1).
12. Other interest: Interest other than interest on borrowed capital e.g. interest on deferred payment for purchased
of asset, interest on delayed payment of electricity charges, interest on purchase price of raw-material, etc.
shall not be allowed under this section but can be claimed u/s 37(1)

2.3.35 Discount on issue of Zero Coupon Bonds (ZCB) [Sec. 36(1)(iiia)]


Meaning: As per Sec.2(48) “Zero Coupon Bond” means a bond—
–– issued by any infrastructure capital company or infrastructure capital fund or public sector company or
scheduled bank on or after 1/6/2005;
–– in respect of which no payment and benefit is received or receivable before maturity or redemption from the
issuer; and
–– which the Central Government may, by notification in the Official Gazette, specify in this behalf.
Treatment: Discount on issue of Zero Coupon Bonds shall be allowed on pro rata basis having regard to the period
of life of such bond.
Infrastructure capital company [Sec.2(26A)] means
Such company which makes investment by way of acquiring shares or providing long term finance to any enterprise
or undertaking wholly engaged in the following business:
 Business referred u/s 80IA or 80IAB;
 Any undertaking developing and building a housing project referred u/s 80IB(10);
 A project for constructing a hotel for not less than 3 star category;

220 The Institute of Cost Accountants of India


Heads of Income

 Project for constructing a hospital with at least 100 beds for patients.
Infrastructure capital fund [Sec.2(26B)] means
Such fund operating under a trust deed established to raise money by the trustees for investment by way of acquiring
shares or providing long term finance to any enterprise or undertaking wholly engaged in the following business:
 Business referred u/s 80IA, 80IAB
 Any undertaking developing and building a housing project referred u/s 80IB(10)
 A project for constructing a hotel for not less than 3 star category.
 Project for constructing a hospital with at least 100 beds for patient

2.3.36 Contribution towards RPF & Superannuation Fund [Sec. 36(1)(iv)]


Any sum paid1, by the employer towards recognised provident fund or an approved superannuation fund as per
rules specified in the fourth schedule of the Act is allowed as deduction in full.
1.
Such amount must have been actually paid before the due date of furnishing return [Sec. 43B]
Taxpoint:
 Contribution towards unrecognised provident fund is not allowed as deduction.
 Contribution towards statutory provident fund is allowed as deduction u/s 37(1).

2.3.37 Contribution towards Notified Pension Scheme U/s 80CCD [Sec. 36(1)(iva)]
Any sum paid by the assessee, as an employer, by way of contribution towards a pension scheme, as referred to in
section 80CCD, on account of an employee is allowed as deduction.
Maximum Limit: Such contribution should not exceed 10% of the salary of the employee in the previous year.
“Salary” includes dearness allowance, if the terms of employment so provide, but excludes all other allowances
and perquisites

2.3.38 Contribution towards approved gratuity fund [Sec. 36(1)(v)]


Any sum paid1 as employer’s contribution towards an approved gratuity fund created by him exclusively for the
benefit of his employees under an irrevocable trust is allowed as deduction.
1.
Such amount must have been actually paid before the due date of furnishing return [Sec. 43B]

2.3.39 Employee’s contribution towards staff welfare scheme [Sec. 36(1)(va)]


Any sum received by an employer from his employees as contribution towards -
 Provident Fund; or
 Superannuation Fund; or
 Any other fund set up under the provision of the Employee’s State Insurance Act, 1948; or
 Any other fund for the welfare of such employees

- is treated as an income of the employer. Subsequently, when such sum is credited by the employer to the employee’s
account in the relevant fund on or before the due date of crediting such contribution prescribed under the relevant

The Institute of Cost Accountants of India 221


Direct Taxation

Act#, then deduction is allowed.


#
Example 3: As per the provisions Employee State Insurance Act, 1948 (ESI), all the contributions under this
Act are to be deposited within 21 days of the following month. Similarly, all contributions under the Employees’
Provident Fund and Miscellaneous Provisions Act, 1952 must be deposited within 15 days of the following month.

Taxpoint

If employees contribution is deposited by the employer on or before the No treatment.


due date1
If employees contribution is not deposited by the employer on or before Taxable as business income.
the due date1
1.
Due date means the date by which the assessee is required as an employer to credit an employee’s contribution
to the employee’s account in the relevant fund under any Act, rule, order or notification issued there under or
under any standing order, award, contract of service or otherwise

2.3.40 Allowance in respect of dead or useless animals [Sec. 36(1)(vi)]


Sec. 36(1)(vi) provides for deduction in respect of animals used for the purpose of business or profession.
Conditions
a. Animals are used for the purpose of business or profession.
b. Such animals are not held as stock-in-trade.
c. Such animals have died or become permanently useless for such purpose.
Quantum of deduction
Difference between actual cost of the animals to the assessee and the amounts realised, if any, in respect of carcasses
or sale of animals is allowed as deduction.

2.3.41 Bad Debts [Sec. 36(1)(vii)]


Any debt or part thereof, which becomes bad shall be allowed as deduction.
Taxpoint: It is the assessee, who decides whether a debt has become bad or not and the Assessing Officer can never
insist the assessee for production of proof that the debt had became bad.
Conditions
1. Debt must be incidental to the business or profession of the assessee. There must be a close nexus between the
debt and the business of the assessee.
Example 11: Bad debt arising out of advances made by a lawyer to his client to assist him in purchasing
properties is not admissible as bad debt. As it is not the business of lawyer to provide loans. Such loss is not
allowed in any provision of the Act
2. The debt has been considered as income of the assessee of that previous year or of earlier previous years.
Example 12: Advance given to supplier for purchase of raw-material later forfeited, is not allowed as deduction
under this section, this is because the same has never been a part of income. However deduction can be claimed
u/s 37(1).
Exception: Bad debt arising due to insolvency of borrower is allowed as deduction provided money has been

222 The Institute of Cost Accountants of India


Heads of Income

lent in ordinary course of money lending business (even though such money lent had never been a part of
income)
3. It must have been written off in the accounts of the assessee.
Taxpoint: Provision for bad debt is not allowed as deduction.
¾¾ Exception: Where the amount of such debt has been taken into account in computing the income of the
assessee of the previous year in which the amount of such debt becomes irrecoverable or of an earlier
previous year on the basis of notified Income Computation and Disclosure Standards (ICDS) without
recording the same in the accounts, then, such debt shall be allowed in the previous year in which such
debt becomes irrecoverable and it shall be deemed that such debt has been written off as irrecoverable in
the accounts.
4. Business must be carried on during the previous year or any part of the previous year.
Taxpoint: Bad debt of a discontinued business is not allowed as deduction even though the assessee has any
other business continued.
5. It must be of a revenue nature
Taxpoint: Bad debt arising due to insolvency of a debtor for sale of an asset (not goods) is not allowed as
deduction.
Notes
a. Bad debt is not allowed as deduction to the assessee who maintains accounts on cash basis.
b. Bad debts are also allowed in the hands of successor of the business.

Recovery of bad debts [Sec. 41(4)]


As per sec. 41(4), where a deduction has been allowed in respect of a bad debt or part of debt u/s 36(1)(vii), then,
if the amount subsequently recovered on any such debt or part thereof is greater than the difference between the
debt or part of debt and the amount so allowed, the excess shall be deemed to be profits and gains of business or
profession.
Mathematically

Particulars Amount
Amount recovered ******
Less: Bad debt claimed – Bad debt allowed as deduction ******
Taxable bad debt recovery ******
Note: Such recovery shall be taxable irrespective of the fact whether the business is continued or not.

2.3.42 Provision for Bad Debts [Sec. 36(1)(viia)]


In the case of following person, provision for bad debts shall be allowed to the extent of specified amount:

Person To that extent PBDD is allowed


A. Scheduled bank [not being a foreign bank] or a non- 8.5% of the total income (computed before
scheduled bank or a co-operative bank other than a primary making any deduction under this section and
agricultural credit society or a primary co-operative Chapter VIA)
agricultural and rural development bank

The Institute of Cost Accountants of India 223


Direct Taxation

Person To that extent PBDD is allowed


Additional Deduction for the Person covered under (A)
a. 10% of the aggregate average advances made by the rural branches of such bank.
b. Further, at its option, further deduction shall also be allowed for an amount not exceeding the income derived
from redemption of securities in accordance with a scheme framed by the Central Government provided such
income has been disclosed in the return of income under the head “Profits and gains of business or profession”
B. foreign bank or Public financial institution or State 5% of the total income (computed before making
financial corporation or State industrial investment any deduction under this section and Chapter
corporation or Non-banking financial company VIA)
Notes
 In case of person covered under this section, the deduction for bad debts u/s 36(1)(vii) shall be limited to an
amount by which such debts exceed the credit balance in the provision for bad and doubtful debts. If the actual
bad debt during the previous year is less or equal to the provision for doubtful debts, no deduction for bad debt
shall be allowed.
 Rural branch means a branch of a scheduled bank or a non-scheduled bank situated in a place which has a
population of not more than 10000 according to the last preceding census of which the relevant figures have
been published before the first day of the previous year.

2.3.43 Deduction is respect of Special Reserve [Sec. 36(1)(viii)]


Deduction is allowed in respect of any special reserve created and maintained by a specified entity in respect of
reserve created from the specified business. The amount of deduction shall be least of the following:
a. Amount transferred to the reserve account during the previous year; or
b. 20% of the profits derived from the specified business of providing long-term finance; or
c. twice the amount of the paid-up share capital and the general reserve.
Notes
Meaning of specified entity and specified business
Specified Business means the business of
Specified Entity
providing long-term finance for
A financial corporation u/s 2(72) of the Companies Act, a. industrial or agricultural development;
2013 or a financial corporation which is a public sector
b. development of infrastructure facility in India;
company or a banking company or a co-operative bank
or
other than a primary agricultural credit society or a primary
co-operative agricultural and rural development bank c. development of housing in India
A housing finance company Construction or purchase of houses in India for
residential purposes.
Any other financial corporation including a public company Development of infrastructure facility in India
Long-term finance means any loan or advance where the terms under which moneys are loaned or advanced
provide for repayment along with interest thereof during a period of not less than 5 years.
Treatment on withdrawal from Special Reserve Account
As per sec. 41(4A), if a deduction has been allowed in respect of any special reserve created under this section,
any amount subsequently withdrawn from such special reserve shall be deemed to be profits or gains of business
or profession and shall be taxable in the year of such withdrawal. The above rule holds good even if the business
is no longer in existence in the year of such withdrawal.

224 The Institute of Cost Accountants of India


Heads of Income

2.3.44 Expenditure on promotion of family planning among employees [Sec. 36(1)(ix)]


Applicable to: Company only
Purpose of such expenditure: Such expenditure must have been incurred for promotion of family planning among
its employees.
Quantum of Deduction
 Revenue expenditure is fully allowed as deduction.
 Capital expenditure shall be allowed in 5 equal installments commencing from the previous year in which it
is incurred.
Note: Where deduction is allowed in respect of any expenditure under this section then no deduction shall be
allowed u/s 32 or any other provisions of this Act.
Treatment of unabsorbed capital expenditure: As in case of unabsorbed depreciation.
Sale of assets acquired for family planning: Treated in the same manner as in case of sale of assets used for
scientific research.

2.3.45 Expenditure incurred by a corporation or a body corporate [Sec. 36(1)(xii)]


Any expenditure (not being in the nature of capital expenditure) incurred by a corporation or a body corporate, by
whatever name called, if,—
a. it is constituted or established by a Central, State or Provincial Act;
b. such corporation or body corporate, having regard to the objects and purposes of the respective Act is notified
by the Central Government in the Official Gazette for the purposes of this clause; and
c. the expenditure is incurred for the objects and purposes authorised by the Act under which it is constituted or
established;
- shall be allowed as deduction.

2.3.46 Credit Guarantee Fund Trust [Sec. 36(1)(xiv)]


Any sum paid by a public financial institution by way of contribution to specified credit guarantee fund trust for
small industries shall be allowed as deduction.

2.3.47 Securities Transaction Tax [Sec. 36(1)(xv)]


Any amount of Securities Transaction Tax (STT) paid by the assessee during the previous year shall be allowed
as deduction provided income arising from such transactions is included in the income computed under the head
“Profits and gains of business or profession.’

2.3.48 Commodities Transaction Tax [Sec. 36(1)(xvi)]


Any amount of Commodities Transaction Tax (CTT) paid by the assessee in respect of the taxable commodities
transactions entered into in the course of his business during the previous year shall be allowed as deduction
provided income arising from such transactions is included in the income computed under the head “Profits and
gains of business or profession.’

The Institute of Cost Accountants of India 225


Direct Taxation

2.3.49 Purchase of Sugarcane [Sec. 36(1)(xvii)]


The amount of expenditure incurred by a co-operative society engaged in the business of manufacture of sugar for
purchase of sugarcane at a price which is equal to or less than the price fixed or approved by the Government shall
be allowed.

2.3.50 Loss as per ICDS [Sec. 36(1)(xviii)]


Marked to market loss or other expected loss as computed in accordance with the ICDS shall be allowed.

2.3.51 General deductions [Sec. 37(1)]Amended


Any expenditure which is not specifically provided in any provisions (discussed earlier) of the Act and fulfills
following conditions, shall be allowed as deduction under this section -
1. It must be real and not notional, fictitious or in lieu of distribution of profit.
2. It must be expended wholly & exclusively for the purpose of business or profession carried on by the assessee.
3. It must have been incurred in the previous year.
4. It must not be a personal expenditure.
5. It must not be a capital expenditure.
6. It must be lawful and not have been incurred for any purpose, which is an offence or prohibited, under any law.
¾¾ It also includes the expenditure incurred by an assessee:
i. for any purpose which is an offence or prohibited by any law for the time being in force, in India or
outside India; or
ii. to provide any benefit or perquisite to a person, whether or not carrying on a business or exercising a
profession, and acceptance of such benefit or perquisite by such person is in violation of any law or
rule or regulation or guideline, as the case may be, for the time being in force, governing the conduct
of such person [see Taxpoint 2 given below]; or
iii. to compound an offence under any law for the time being in force, in India or outside India.’
Taxpoint:
1. Corporate Social Responsibility: Any expenditure incurred by an assessee on the activities relating to corporate
social responsibility referred to in section 135 of the Companies Act, 2013 shall not be deemed to be an
expenditure incurred by the assessee for the purposes of the business or profession.
2. The claim of any expense incurred in providing any Gift, Travel facility, Hospitality, Cash or monetary grant or
similar freebees in violation of the provisions of Indian Medical Council (Professional Conduct, Etiquette and
Ethics) Regulations, 2002 shall be inadmissible u/s 37(1) of the Income Tax Act being an expense prohibited by
the law. This disallowance shall be made in the hands of such pharmaceutical or allied health sector Industries
or other assessee which has provided said freebees and claimed it as a deductable expense in its accounts
against income. Further, the sum equivalent to value of freebees enjoyed by the aforesaid medical practitioner
or professional associations is also taxable as business income or income from other sources as the case may
be. [Circular 05/2012 dated 01-08-2012]

2.3.52 Advertisement in souvenir etc. of a political party [Sec. 37(2B)]


Expenditure incurred by an assessee on advertisement in any souvenir, brochure, tract, pamphlet or like, published
by a political party is disallowed.

226 The Institute of Cost Accountants of India


Heads of Income

2.3.53 Disallowed Expenditure [Sec. 40]


Whereas sec. 30 to 37(1) deals with the expenditure, which are allowed, the Act also lists certain expenditures
which are specifically disallowed. Sec. 40 states that, notwithstanding anything to the contrary in sec. 30 to 38,
certain amounts shall not be deducted in computing the income chargeable under the head “Profits & gains of
business or profession”. Such provisions are discussed below:
Following expenditures are expressly disallowed u/s 40(a):
a. Interest, royalty, fees for technical services payable to a non-resident or outside India [Sec.40(a)(i)]
Any interest, royalty, fees for technical services or other sum chargeable under this Act, which is payable ,—
¾¾ outside India; or
¾¾ in India to a non resident (not being a company) or to a foreign company,
- on which tax is deductible at source under Chapter XVIIB; and
such tax -
¾¾ has not been deducted; or
¾¾ after deduction, has not been paid within the due date of submission of return of income u/s 139(1).
Taxpoint:
¾¾ Where in respect of any such sum, tax has been deducted in any subsequent year, or has been deducted
during the previous year but paid after the due date of submission of return of income u/s 139(1), such
sum shall be allowed as a deduction in computing the income of the previous year in which such tax has
been paid.
¾¾ Where an assessee fails to deduct the whole or any part of the tax on any sum but is not deemed to be
an assessee in default under the first proviso to sec. 201(1), then, it shall be deemed that the assessee has
deducted and paid the tax on such sum on the date of furnishing of return of income by the payee.
Notes
1. Royalty shall have the same meaning as in sec. 9(1)(vi).
2. Fees for technical services shall have the same meaning as in sec. 9(1)(vii).
b. Any sum payable to a resident on which TDS provision is applicable [Sec. 40(a)(ia)]
30% of any sum payable to a resident on which tax is deductible at source under Chapter XVII-B if:
Such tax:
¾¾ has not been deducted; or
¾¾ after deduction, tax has not been paid on or before the due date of furnishing return of income
Notes
1. Where such tax has been deducted in any subsequent year, or tax has been paid after the due date of
furnishing return of relevant assessment year, then the amount disallowed earlier (i.e., 30% portion)
shall be allowed as deduction in the following assessment year
Where such tax has been deducted in any subsequent year Disallowed amount shall be allowed as a
Where such tax has been deducted in the relevant financial deduction in computing the income of the
year but tax has been paid after the due date of furnishing previous year in which such tax has been
return of relevant assessment year paid

The Institute of Cost Accountants of India 227


Direct Taxation

2. Where an assessee fails to deduct the whole or any part of the tax but is not deemed to be an assessee in
default under the first proviso to section 201(1), then, it shall be deemed that the assessee has deducted
and paid the tax on such sum on the date of furnishing of return of income by the resident payee.
 This relaxation is not available where the payer has deducted tax but fails to deposit such tax to the
credit of the Central Government.
 As per first proviso to sec.201(1), the payer is not deemed as an assessee in default:
i. Such resident recipient has furnished his return of income u/s 139
ii. Such resident recipient has taken into account such sum for computing income in such return
of income; and
iii. Such resident recipient has paid the tax due on the income declared by him in such return of
income,
iv. The payer furnishes a prescribed certificate to this effect from a chartered accountant
3. Commission or brokerage includes any payment received or receivable by a person acting on behalf of
another person for services rendered (not being professional services) or for any services in the course
of buying or selling of goods or in relation to any transaction relating to any asset, valuable article or
thing, not being securities.
c. Consideration on which Equalisation Levy is applicable [Sec. 40(a)(ib)]
Any consideration paid or payable to a non-resident for a specified service on which equalisation levy is
deductible and such levy:
i. has not been deducted or
ii. after deduction, has not been paid on or before the due date of furnishing return of income.
Taxpoint: Where in respect of any such consideration, the equalisation levy has been deducted in any
subsequent year or has been deducted during the previous year but paid after the due date of furnishing return
of income, such sum shall be allowed as a deduction in computing the income of the previous year in which
such levy has been paid.
d. Income tax (Indian or foreign) [Sec. 40(a)(ii)]
Even, income tax paid by the assessee on income of predecessor is not deductible. The term “tax” includes any
surcharge or cess, by whatever name called, on such tax.
Taxpoint: Professional tax is an allowed expenditure [Circular No. 16 & 18 dated 18/9/69]
e. Wealth-tax [Sec. 40(a)(iia)]
f. Royalty, licence fees, etc. payable by State Government Undertaking [Sec. 40(a)(iib)]

Any amount
a. paid by way of royalty, licence fee, service fee, privilege fee, service charge or any other fee or charge,
by whatever name called, which is levied exclusively on; or
b. which is appropriated, directly or indirectly, from,
a State Government undertaking by the State Government.
Note: State Government undertaking includes—
i. a corporation established by or under any Act of the State Government;

228 The Institute of Cost Accountants of India


Heads of Income

ii. a company in which more than 50% of the paid-up equity share capital is held by the State Government;

iii. a company in which more than 50% of the paid-up equity share capital is held by the entity referred to in
above clauses (whether singly or taken together);

iv. a company or corporation in which the State Government has the right to appoint the majority of the
directors or to control the management or policy decisions, directly or indirectly, including by virtue of
its shareholding or management rights or shareholders agreements or voting agreements or in any other
manner;

v. an authority, a board or an institution or a body established or constituted by or under any Act of the State
Government or owned or controlled by the State Government;

g. Any payment which is chargeable under the head “Salaries”, if it is payable—

(i) outside India; or (ii) to a non-resident,

and if the tax has not been paid in India thereon nor deducted therefrom [Sec. 40(a)(iii)]

h. Any payment to a provident fund or any other fund established for the benefit of employees of the assessee
in respect of whom the assessee has not made effective arrangement to secure that tax shall be deducted at
source from any payment made from the fund, which are taxable under the head ‘Salaries’ [Sec. 40(a)(iv)]; and

i. Any tax on non-monetary perquisite [which is exempt in the hands of employee u/s 10(10CC)] actually paid
by employer on behalf of employee [Sec. 40(a)(v)].

2.3.54 Payment made to relatives in excess of requirement [Sec. 40A(2)]


Any payment made by an assessee to a related person1 shall be disallowed to the extent it is excess or unreasonable2
as per the Assessing Officer.
1.
Related person: Sec. 40A(2)(b) deals with the related person. Before explaining the provision, it is imperative
to understand the meaning of the terms “Relative” and “Person having substantial interest”

Relative u/s 2(41) means the spouse, brother, sister or any lineal ascendant or descendant of that individual.

Person having substantial interest: A person is deemed to have substantial interest in the business or
profession, if

i. in a case, where the business or profession is carried on by a company, such person is at any time during
the previous year, the beneficial owner of equity share carrying not less than 20% of voting power;

ii. in any other case, such person is at any time during the previous year, the beneficially entitled to not less
than 20% of the profits of such business or profession.

List of related persons in case of different assessee

For the assessee Related Person means


Relative
An Individual
A person in whose business or profession the individual has substantial interest.

The Institute of Cost Accountants of India 229


Direct Taxation

For the assessee Related Person means


Director of the company or any relative of the director
A person in whose business or profession the company or any of its director or relative of
such director has substantial interest.
Any other company carrying on business or profession in which the aforesaid company
A Company
has substantial interest.
E.g. X Ltd. holds 20% equity shares in Y Ltd., the assessee. Further, X Ltd. also holds
20% equity shares in Z Ltd. Z Ltd. shall also be considered as relative for Y Ltd. provided
Z Ltd. is carrying on business or profession.
Partner of the firm or relative of partner
A Firm A person in whose business or profession the firm or any of its partner or relative of such
partner has substantial interest.
A member of the Association or a relative of the member.
An AOP A person in whose business or profession the AOP or any of its member or relative of such
member has substantial interest.
A member of the family or relative of such person
An HUF A person in whose business or profession the HUF or any of its member or relative of such
member has substantial interest.
¾¾ An individual who has a substantial interest in the business or profession of the
assessee or the relative of such individual.
¾¾ A company, which has a substantial interest in the business or profession of the
assessee or the director of such company or relatives of such a director.
¾¾ A Firm/HUF/AOP etc., which has a substantial interest in the business or profession
of the assessee or the partner/member of such firm/HUF/AOP or relatives of such
Any assessee partner/member.
¾¾ A company, one of whose director has a substantial interest in the business or
profession of the assessee or directors of such company or any relative of such
directors.
¾¾ Firm, AOP, HUF, one of whose partner/member has a substantial interest in the
business or profession of the assessee or any partner/member of such Firm/AOP/
HUF or any relative of such person.
2.
Excessive or unreasonable: Whether any expenditure is in excess or unreasonable is to be decided after
considering the fair market value of the goods, services or facilities for which payment is made or the legitimate
need of the business or profession of the assessee or the benefit arising to the assessee therefrom.
3.
Where an assessee sells his goods at a lower rate, there is no expenditure incurred by him, hence sec. 40A(2)
shall not be invoked.

2.3.55 Consequences of payment exceeding ₹ 10,000/- otherwise than by account payee


cheque or demand draft [Sec. 40A(3)/(3A)]
Applicability: Any expenditure in respect of which payment has been made in excess of ₹ 10,000 in a day otherwise

230 The Institute of Cost Accountants of India


Heads of Income

than by an account payee cheque or account payee bank draft or use of electronic clearing system through a bank
account or through other prescribed electronic modes[Sec. 40A(3)]
Treatment: 100% of such payment shall be disallowed.
Exception: In the case of payment made for plying, hiring or leasing goods carriages (hereinafter referred to as
Road Transport), the limit of ₹ 10,000 has been increased upto ₹ 35,000.
Taxpoint: The monetary limit for attracting sec.40A(3) are as follows:

Case Monetary Limit


Payment made for plying, hiring or leasing goods carriages ₹ 35,000
Payment made for other expenses ₹ 10,000
Other points
¾¾ Where:
a. an allowance has been made in the assessment for any year in respect of any liability incurred by the
assessee for any expenditure; and
b. subsequently during any previous year the assessee makes any payment in violation of this provision,
then, the payment so made shall be deemed to be the profits and gains of business or profession of such
subsequent year [Sec. 40A(3A)]
¾¾ If an assessee makes payment of two different bills (none of them exceeds ₹ 10,000 / ₹ 35,000) at the same
time in cash to the same person, provision of sec. 40A(3) is not attracted.
¾¾ If an assessee makes payment of a single bill (exceeding ₹ 10,000 / ₹ 35,000) on different days to the same
person in cash, provision of sec. 40A(3) is not attracted, provided any of the payment does not exceed ₹ 10,000
/ ₹ 35,000.
¾¾ Where payment is made over ₹ 10,000 (or ₹ 35,000) at a time, partly by account payee cheque & partly in cash
but the payment in cash alone at one time does not exceed ₹ 10,000 (or ₹ 35,000), assessee is not attracted by
sec. 40A(3).
¾¾ The provision of sec. 40A(3) is attracted only when such expenditure is claimed as deduction u/s 30 to 37.

Exceptions [Rule 6DD]


Under the following circumstances as prescribed under Rule 6DD, provision of Sec.40A(3) is not attracted even
the payment in excess of ₹ 10,000 (or ₹ 35,000) has been made otherwise than by a account payee cheque or
account payee bank draft or other prescribed modes –
a. Where the payment is made to—
¾¾ The Reserve Bank of India or any banking company
¾¾ The State Bank of India or any subsidiary bank
¾¾ Any co-operative bank or land mortgage bank;
¾¾ Any primary agricultural credit society or any primary credit society
¾¾ The Life Insurance Corporation of India
b. Where the payment is made to the Government and, under the rules framed by it, such payment is required to
be made in legal tender;

The Institute of Cost Accountants of India 231


Direct Taxation

c. Where the payment is made by—


¾¾ Any letter of credit arrangements through a bank;
¾¾ A mail or telegraphic transfer through a bank;
¾¾ A book adjustment from any account in a bank to any other account in that or any other bank;
¾¾ A bill of exchange made payable only to a bank;
¾¾ The use of electronic clearing system through a bank account;
¾¾ A credit card;
¾¾ A debit card
d. Where the payment is made by way of adjustment against the amount of any liability incurred by the payee for
any goods supplied or services rendered by the assessee to such payee i.e., Book Adjustment;
e. Where the payment is made for the purchase of
¾¾ Agricultural or forest produce; or
¾¾ The produce of animal husbandry (including livestock, meat, hides and skins) or dairy or poultry farming;
or
¾¾ Fish or fish products; or
¾¾ The products of horticulture or apiculture,
- to the cultivator, grower or producer of such articles, produce or products;
f. Where the payment is made for the purchase of the products manufactured or processed without the aid of
power in a cottage industry, to the producer of such products;
g. Where the payment is made in a village or town, which on the date of such payment is not served by any bank,
to any person who ordinarily resides, or is carrying on any business, profession or vocation, in any such village
or town;
h. Where any payment is made to an employee of the assessee or the heir of any such employee, on or in
connection with the retirement, retrenchment, resignation, discharge or death of such employee, on account of
gratuity, retrenchment compensation or similar terminal benefit and the aggregate of such sums payable to the
employee or his heir does not exceed ₹ 50,000;
i. Where the payment is made by an assessee by way of salary to his employee after deducting the income-tax
from salary in accordance with the provisions of section 192 of the Act, and when such employee -
i. is temporarily posted for a continuous period of 15 days or more in a place other than his normal place
of duty or on a ship; and
ii. does not maintain any account in any bank at such place or ship;
j. Where the payment was required to be made on a day on which the banks were closed either on account of
holiday or strike;
k. Where the payment is made by any person to his agent who is required to make payment in cash for goods or
services on behalf of such person;
l. Where the payment is made by an authorised dealer or a money changer against purchase of foreign currency
or travellers cheques in the normal course of his business.

232 The Institute of Cost Accountants of India


Heads of Income

Example 13:
Points to be kept in mind Examples
If an assessee makes payment to Mr. X against
a bill of ₹ 50,000 on a same day but at different
Entire ₹ 50,000 shall be disallowed u/s 40A(3)
time but each time the amount of payment does
not exceed ₹ 10,000.
If an assessee makes payment of two different X paid to Y ₹ 12,000 in cash against his Bill No.482 of ₹ 7,000
bills (none of them exceeds ₹ 10,000) at the same and Bill No.572 of ₹ 5,000.
time to the same person in cash, provision of sec.
40A(3) is not attracted. Nothing shall be disallowed under this section.
X paid to Y in cash (against bill 421) of ₹ 23,000 as follows -
If an assessee makes payment of a single bill
(exceeding ₹ 10,000) on different days to the On 7/12/2023: ₹ 8,000
same person in cash, provision of sec. 40A(3) is On 8/12/2023: ₹ 9,000
not attracted, provided any of the payment does On 9/12/2023: ₹ 6,000
not exceed ₹ 10,000.
Nothing shall be disallowed.
Where payment is made over ₹ 10,000 at a time,
partly by account payee cheque & partly in cash X paid to Y (against bill 712) of ₹ 50,000, in form of account
but the payment in cash alone at one time does payee cheque ₹ 42,000, bearer cheque ₹ 3,000 and in cash ₹
not exceed ₹ 10,000, assessee is not attracted by 5,000. Nothing shall be disallowed.
sec. 40A(3).
If part of the expenditure is already disallowed X purchased goods from his brother of ₹ 14,000 (market
under any provision of this Act, then disallowance value of which is ₹ 8,000) and paid in cash. ₹ 6,000 shall be
shall be calculated on the allowed portion of the disallowed u/s 40A(2) and nothing shall be disallowed u/s
expenditure. 40A(3) as allowed expenditure does not exceed ₹ 10,000.
Mr. X made following payment in cash to a road transport
operator for their respective bills:
The monetary limit for payment to Road Carrier - ₹ 23,000 to Mr. A on 10-05-2023 against his Bill No. 540
is ₹ 35,000 - ₹ 32,000 to Mr. B on 10-12-2023 against his Bill No. 770
- ₹ 37,000 to Mr. C on 10-01-2024 against his Bill No. 992
Payment made to Mr. C shall be disallowed fully u/s 40A(3)
Where an allowance has been made in the
assessment year in respect of any liability Mr. X purchased goods on credit on 7/7/2023 for ₹ 60,000 and
incurred by the assessee for any expenditure claimed the expenditure as deduction in the A.Y. 2024-25. On
and subsequently during any previous year the 7/7/2025 he paid to creditor ₹ 60,000 in cash.
assessee makes any payment in respect thereof
in a sum exceeding ₹ 10,000 otherwise than by Since amount has been paid in cash in excess of ₹ 10,000,
a account payee cheque or account payee bank therefore the allowance originally made shall be considered as
draft, the allowance originally made shall be income of the previous year in which such payment has been
considered as income of the previous year in made i.e. P.Y.2025-26.
which such payment has been made.

2.3.56 Provision for Gratuity [Sec. 40A(7)]


No deduction shall be allowed in respect of any provision (by whatever name called) made by the assessee for the
payment of gratuity to his employees.

The Institute of Cost Accountants of India 233


Direct Taxation

Exceptions: Any provision made by the assessee for the purpose of payment of a sum -
 by way of any contribution towards an approved gratuity fund; or
 for the purpose of payment of any gratuity, that has become payable during the previous year;
- shall be allowed as deduction.

2.3.57 Certain contributions not deductible [Sec. 40A(9)]


No deduction shall be allowed in respect of any sum paid by the assessee as an employer towards setting up or
formation of, or as contribution to, any fund, trust, company, AOP, BOI, society or other institution for any purpose.
Exception: Where such sum is paid by way of contribution towards approved superannuation fund, recognised
provident fund, approved gratuity fund, NPS as per the requirements under any law, then such sum is allowed.
Taxpoint: Contribution to unrecognized provident fund is disallowed.

2.3.58 Expenditures allowed on cash basis [Sec. 43B]


Deduction in respect of following expenses are allowed only if payment is made on or before the due date for
furnishing return of income u/s 139(1)1 of the previous year in which such liability is incurred:
1. Any sum payable2 by way of tax, duty, cess, fee, by whatever name called, under any law for the time being
in force.
2. Any sum payable as bonus or commission to employees for services rendered.
3. Any sum payable as interest on loan or borrowing from any
¾¾ public financial institutions (i.e., IFCI, LIC, etc.);
¾¾ a State financial corporation; or
¾¾ State industrial investment corporation.
4. Any sum payable by the assessee as interest on any loan or borrowing from a deposit taking non-banking
financial company or systemically important non-deposit taking non-banking financial company, in accordance
with the terms and conditions of the agreement governing such loan or borrowing
¾¾ Systemically important non-deposit taking non-banking financial company means a non-banking financial
company which is not accepting or holding public deposits and having total assets of not less than ₹ 500
crore as per the last audited balance sheet and is registered with the RBI.
¾¾ Deposit taking non-banking financial company means a non-banking financial company which is accepting
or holding public deposits and is registered with the RBI.
5. Any sum payable as interest on any loans and advances from a scheduled bank or a co-operative bank other
than a primary agricultural credit society or a primary co-operative agricultural and rural development bank in
accordance with the terms and conditions of the agreement governing such loan or advances.
6. Any sum payable by an employer in lieu of any leave at the credit of employee (i.e. leave encashment).
7. Any sum payable by an employer by way of contribution to any provident fund, superannuation fund, gratuity
fund or any other fund for the welfare of employees.
8. Any sum payable by the assessee to the Indian Railways for the use of railway assets.
9. Any sum payable to Micro or Small Enterprises [Sec 15 of MSMED ACT 2006].

234 The Institute of Cost Accountants of India


Heads of Income

Taxpoint:
 If payment is not made before the date mentioned above, then no allowance shall be allowed in respect of the
outstanding liability. Deduction can, however, be claimed in the year of payment.
 The method of accounting followed by the assessee is irrelevant.
 Sec.43B is applicable only on those expenditure which are allowed under the provisions of this Act, if an
expenditure is already disallowed under the provision of this Act, payment of such expenditure within time
shall not be sufficient to make it an allowed expenditure. E.g. Income tax paid ₹ 5,000 before due date of filing
of return, is not allowed as deduction.
1.
In general, due date for furnishing return of income u/s 139(1)

- Where audit of books of account is compulsory under any law : 31st October* of the A.Y.
- In any other case : 31st July of the A.Y.
Any sum payable means a sum for which the assessee incurred liability in the previous year even though such
2.

sum might not have been payable within that year under the relevant law i.e. liability must have been accrued
whether falls due or not.
Notes
1. Sec.43B is applicable only if the assessee is following mercantile system of accounting. However, if an assessee
follows cash basis of accounting, deduction shall be allowed only in the year in which payment is made, even
though the payment has been made on or before due date of filing of return.
2. The provision that “for claiming deduction, payment must be made on or before the due date of filing of
return” shall be applied only for the relevant previous year in which such liability is incurred. If payment is
made afterwards, deduction shall be allowed in the previous year in which payment is actually made, without
considering the due date of filing of return.
Example 14: Mr. X paid professional tax ₹ 10,000 related to previous year 2019-20 on 7/5/2023. Deduction
for such expenditure shall be allowed in the P.Y 2023-24 and not in P.Y.2022-23.
3. Where outstanding interest on loan (taken from Banks, NBFC, PFIs, etc.) is converted into loan debenture or
any other instrument by which the liability to pay is deferred to a future date, then such interest is not deemed
as interest paid.
4. Where a deduction in respect of the aforesaid expenditure is allowed in an earlier year on accrual basis the
same will not again be allowed as deduction under this section on payment basis.
5. As per sec. 36(1)(va), any sum received by an employer from his employees as contribution towards provident
fund, superannuation fund, any other fund set up under the provision of the ESI Act, 1948 or any other fund
for the welfare of such employees, is treated as an income of the employer. Subsequently, when such sum is
credited by the employer to the employee’s account in the relevant fund on or before the due date of crediting
such contribution prescribed under the relevant Act, then deduction is allowed. If such contribution is not
deposited within time allowed as per the provisions of the relevant Act, the deduction shall never be allowed,
i.e. not now then never.
Illustration 77:
Debit side of the profit and loss account of Mayank Ltd. shows the following expenses, which have been due but
are outstanding as on 31-3-2024

* 30th November, in case of assessee required to furnish Audit Report u/s 92E

The Institute of Cost Accountants of India 235


Direct Taxation

Payment outstanding on 31-3-2023 First payment Second payment


Particulars Amount Date Amount paid Date Amount paid
Leave encashment expenses 65,000 01-06-2024 15,000 25-12-2024 50,000
Interest payable to Bank 14,000 10-06-2024 3,000 13-12-2024 11,000
Bonus payable to employees 87,000 02-05-2024 30,000 30-09-2024 57,000
Interest payable to LIC loan 75,000 13-05-2024 50,000 10-01-2025 25,000
Due date for filing return of income is 31-10-2024
Find out the previous years in which the aforesaid payments are deductible. The company maintains books of
accounts on the basis of mercantile system of accounting. (CS June 2003)
Solution :
As per provision of sec. 43B following payment shall be allowed.

Deduction allowed in the assessment year


Particulars Amount paid
2024-25 2025-26
Leave encashment expenses 65,000 15,000 50,000
Interest payable to Bank 14,000 3,000 11,000
Bonus payable to employees 87,000 87,000 -
Interest payable to LIC loan 75,000 50,000 25,000

2.3.59 Deemed profit chargeable to tax as business Income


The following receipts are chargeable to tax as business income even if the business to which such receipt is related
is not in existence during the previous year.

Section Contents
41(1) Recovery against any deduction (given below)
41(2) Balancing charge (discussed earlier)
41(3) Sale of asset used for scientific research (discussed earlier)
41(4) Recovery of Bad debts (discussed earlier)
41(4A) Withdrawal from special reserve created by financial corporation u/s 36(1)(viii)
176(3A) & (4) Recovery after discontinuance of business or profession (given below)
Recovery against any deduction [Sec. 41(1)]
Conditions
1. Where an allowance or deduction is allowed in any assessment year in respect of loss, expenditure, or trading
liability incurred by the assessee; and
2. Subsequently during any previous year such assessee has obtained, whether in cash or in any other manner
any amount in respect of such loss, expenditure, or any benefit in respect of such trading liability by way of
remission or cessation thereof.
Treatment: The amount obtained or benefit accrued shall be deemed to be the profits & gains of business or
profession and chargeable to income tax as income of the previous year in which such benefit is obtained.

236 The Institute of Cost Accountants of India


Heads of Income

Notes
1. The provision holds good, whether the business or profession in respect of which such allowance or deduction
has been made is in existence in that year or not.
2. The first and the most necessary ingredient for the application of sec. 41(1) is that an allowance or deduction
must have been made previously while computing the taxable income
3. Where benefit has been obtained by the successor in business, such benefit shall be taxable in hands of
successor.
Recovery after discontinuance of business or profession [Sec. 176(3A) and (4)]
Condition: Business or profession is discontinued in any year due to death or retirement of the person carrying on
such business or profession.
Treatment: Any sum received after such discontinuance shall be deemed to be the income of the recipient and
chargeable to tax in the year of receipt.

2.3.60 Maintenance of books of account [Sec. 44AA & Rule 6F]


Person
Turnover or income criteria
falling
Case
under this Maintenance of Accounts
category Existing business New business
Gross receipts in the profession Gross total receipts Maintain such books of
exceeds ₹ 1,50,000 in all of in the profession for account and other documents as
A Persons the three years immediately that year is likely to prescribed by Rule 6F2
carrying on preceding the previous year. exceed ₹ 1,50,000.
specified Gross receipts in the profession Gross total receipts Maintain such books of account
professions1 does not exceed ₹ 1,50,000 in the profession and other documents as may
B in any one of the three years for that year is not enable the Assessing Officer to
immediately preceding the likely to exceed ₹ compute their taxable income
previous year. 1,50,000. under the Income-tax Act.
Profit from such profession or Income/ total sales, Maintain such books of account
business exceeds ₹ 1,20,000 etc. is likely to and other documents as may
(in case of individual & HUF ₹ exceed the said enable the Assessing Officer to
2,50,000); or amount. compute their taxable income
under the Income-tax Act.
The total sales or turnover or
C Persons
gross receipts thereof is in
carrying
excess of ₹ 10,00,000 (in case of
on a “non-
individual & HUF ₹ 25,00,000),
specified
profession - in any of the 3 years
or any immediately preceding the P.Y.
business”: Aforesaid limit does not exceed Income is not likely Not required to maintain any
in all of the 3 years immediately to exceed said limit books of account.
preceding the P.Y. and total sales,
D
turnover or gross
receipt is not likely
to exceed said limit.

The Institute of Cost Accountants of India 237


Direct Taxation

Person
Turnover or income criteria
falling
Case
under this Maintenance of Accounts
category Existing business New business
An assessee (covered u/s 44AE, 44BB or 44BBB) who claims Maintain such books of account
E income from such business to be lower than the deemed income & other documents as may
computed in accordance with the respective sections. enable the AO to compute
Where the provision of sec. 44AD(4) is applicable and income of the his taxable income under the
F assessee exceeds the maximum amount which is not chargeable to Income-tax Act.
income-tax (i.e. basic exemption limit)
Specified Profession: Legal, medical, engineering, architectural profession or profession of accountancy, technical
1.

consultancy, interior decoration, information technology, company secretary, authorised representative, film artist
or any other profession as is notified by the Board in the Official Gazette.
2.
Following books of account are required to be maintained as per Rule 6F
a. Cash book;
b. Journal, if mercantile system of accounting is followed;
c. Ledger;
d. Carbon copies of machine numbered bills, exceeding ₹ 25, issued by the person; and
e. Original bills wherever issued to the person and receipts in respect of expenditure incurred by the person or,
where such bills and receipts are not issued and expenditure incurred does not exceed ₹ 50, payment vouchers
prepared and signed by the person.
f. Assessee engaged in medical profession are required to maintain two more books -
¾¾ Daily Case Register in Form 3C.
¾¾ Inventory records of drugs, medicines and other consumable accessories used in the profession.
Notes
1. Period for which books of account is to be maintained [Rule 6F(5)]: The books of account and other documents
shall be kept and maintained for a period of 6 years* from the end of the relevant assessment year.
2. Penalty: Where an assessee fails to comply with the provision of sec 44AA, he shall be liable to pay penalty
u/s 271A of ₹ 25,000.
3. As per sec. 2(12A), books or books of account includes ledgers, day-books, cash books, account-books and
other books, whether kept in the written form or in electronic form or in digital form or as print-outs of data
stored in such electronic form or in digital form or in a floppy, disc, tape or any other form of electro-magnetic
data storage device.

2.3.61 Tax Audit [Sec. 44AB]


Following assessee are required to get their accounts audited by a chartered accountant and to furnish (electronically)
the audit report in a specified form one month prior to the due date of filing of return of income:
1. An assessee carrying on business
Condition: Total sales, turnover or gross-receipts of business for the previous year exceeds ₹ 1 crore.
* Notice u/s 147 for income escaping assessment may be issued upto 10 years from the end of the relevant assessment year.

238 The Institute of Cost Accountants of India


Heads of Income

Exception 1: Where a person:


¾¾ Declares profits and gains for the previous year u/s 44AD; and
¾¾ His total sales / turnover / gross receipts in business does not exceed ₹ 2 crore in the previous year,
- then, the provision of tax audit is not applicable.

Exception 2: If the following conditions are satisfied, then the higher threshold limit of ₹ 10 crore shall be
applicable for a person carrying on business:
a. aggregate of all amounts received including amount received for sales, turnover or gross receipts during
the previous year, in cash, does not exceed 5% of the said amount; and
b. aggregate of all payments made including amount incurred for expenditure, in cash, during the previous
year does not exceed 5% of the said payment.
In nutshell, applicability of tax audit in case of business assessee are as under:

Case Applicability
Turnover exceeds ₹ 10 crore Applicable
Turnover does not exceed ₹ 2 crore and assessee is covered u/s 44AD Not applicable
Turnover does not exceed ₹ 10 crore and aforesaid conditions are satisfied Not applicable
Turnover does not exceed ₹ 10 crore but exceed ₹ 1 crore and aforesaid
Applicable
conditions are not satisfied (assessee is not covered u/s 44AD)

2. An assessee carrying on profession


Condition: Gross receipts of profession for the previous year exceeds ₹ 50 lacs.

3. An assessee covered u/s 44AE, 44BB or 44BBB


Condition: Assessee has claimed that his income from such business is lower than the deemed income
computed in accordance with the respective section.

4. An assessee covered u/s 44ADA


Condition: Assessee has claimed that:
a. his income is lower than the presumptive income (computed u/s 44ADA); and
b. his income exceeds the maximum amount which is not chargeable to income-tax (i.e. basic exemption
limit)
5. An assessee covered u/s 44AD(4) and his income exceeds the maximum amount which is not chargeable to
income-tax in any previous year
Penalty: If any assessee does not upload such audit report one month prior to the due date of filing of return of
income, then he is liable to pay penalty being lower of the following:
¾¾ ½ percent of turnover or gross receipt; or
¾¾ ₹ 1,50,000.

The Institute of Cost Accountants of India 239


Direct Taxation

2.3.62 Special provision in case of income of public financial institutions, etc. [Sec. 43D]
Notwithstanding anything to the contrary to any other provision of the Act following shall be applicable:

In case of Nature of income Tax treatment


A public financial institution, a scheduled Income by way of interest in
bank, a co-operative bank other than a primary relation to such categories of
agricultural credit society or a primary co- prescribed (as per guidelines
operative agricultural and rural development issued by the Reserve Bank Chargeable to tax in the
bank, a State financial corporation or a State of India) categories of bad or previous year in which -
industrial investment corporation, a deposit doubtful debts
taking non-banking financial company, a ¾¾ it is credited to the
systemically important non-deposit taking profit and loss account;
non-banking financial company or
Public company being registered with Income by way of interest in ¾¾ it is actually received.
National Housing Bank relation to such categories of
prescribed (as per guidelines - whichever is earlier
issued by the National Housing
Bank) categories of bad or
doubtful debts

2.3.63 Method of accounting in certain cases [Sec. 145A]


Valuation of stock
 The valuation of inventory shall be made at lower of actual cost or net realisable value computed in accordance
with the ICDS.
 The valuation of purchase and sale of goods or services and of inventory shall be adjusted to include the
amount of any tax, duty, cess or fee (by whatever name called) actually paid or incurred by the assessee to
bring the goods or services to the place of its location and condition as on the date of valuation. Such tax, duty,
etc. shall include all such payment notwithstanding any right arising as a consequence to such payment.
 The inventory being securities not listed on a recognised stock exchange, or listed but not quoted on a
recognised stock exchange with regularity from time to time, shall be valued at actual cost initially recognised
in accordance with the ICDS.
 The inventory being securities other than above, shall be valued at lower of actual cost or net realisable value
in accordance with the ICDS
 The inventory being securities held by a scheduled bank or public financial institution shall be valued in
accordance with the ICDS after taking into account the extant guidelines issued by the Reserve Bank of India
in this regard.
 The comparison of actual cost and net realisable value of securities shall be made category-wise.

2.3.64 Taxability of certain income [Sec. 145B]


 Interest received on compulsory acquisition: The interest received by an assessee on any compensation or
on enhanced compensation, as the case may be, shall be deemed to be the income of the previous year in which
it is received.
 Export incentives: Any claim for escalation of price in a contract or export incentives shall be deemed to be
the income of the previous year in which reasonable certainty of its realisation is achieved.

240 The Institute of Cost Accountants of India


Heads of Income

 Subsidy: Assistance in the form of a subsidy or grant or cash incentive or duty drawback or waiver or
concession or reimbursement (by whatever name called) shall be deemed to be the income of the previous year
in which it is received, if not charged to income-tax in any earlier previous year.

Computation of income on basis of estimation


To give relief to small taxpayers from maintenance of books of account and from getting the accounts audited, the
Income-tax Act has framed the presumptive taxation scheme u/s 44AD, 44ADA and 44AE.
Computation of Business Profit on Presumptive Basis [Sec. 44AD]
A resident individual, resident Hindu undivided family or a resident partnership firm
Note: The provision is not applicable in case of the following:
¾¾ Limited liability partnership firm
Applicable ¾¾ A person carrying on profession as referred to u/s 44AA
to ¾¾ A person earning income in the nature of commission or brokerage; or
¾¾ A person carrying on any agency business
¾¾ A person carrying on the business of plying, hiring or leasing goods carriages referred to in
sec. 44AE
a. Eligible Business: Assessee must be engaged in any business other than the business
referred above
b. Maximum Turnover: Total turnover or gross receipts in the previous year of eligible
Conditions business should not exceed ₹ 2 crore. (₹ 3 crore where 95% of the amount received are in
non-cash mode).
c. Restriction on claiming deductions: The assessee has not claimed any deduction u/s 10AA
or 80HH to 80RRB in the relevant assessment year.
¾¾ Where amount of turnover or gross receipts is received by an account 6% of such turnover
payee cheque or account payee bank draft or use of ECS or through or receipts
other prescribed electronic modes during the previous year or before
Estimated the due date of filing return of income
income
¾¾ In any other case 8% of such turnover
or receipts
However, a taxpayer can voluntarily declare a higher income in his return.
Notes
1. No Deduction in respect of expenses: The estimated income is comprehensive and no further deductions
relating to expenses shall be allowed.
2. Depreciation: Depreciation is deemed to have been already allowed. The written down value of asset will be
calculated, as if depreciation has been allowed.
3. Deductions: The above estimated income is aggregated with other income of the assessee, from any other
business or under any other heads of income. Further deduction under chapter VIA (other than those mentioned
above) shall be available to the assessee as usual.
4. Brought forward loss: Brought forward loss (if any) shall be subtracted from such estimated income as per
provisions of this Act.

The Institute of Cost Accountants of India 241


Direct Taxation

5. Provision is not applicable [Sec. 44AD(4)]: Where an eligible assessee:


a. declares profit for any previous year in accordance with the provisions of this section (i.e., specified
percentage of the turnover); &
b. declares lower profit (i.e., less than specified percentage of the turnover) for any of the 5 assessment
years relevant to the previous year succeeding aforesaid previous,
then, he shall not be eligible to claim the benefit of the provisions of this section for 5 assessment years
subsequent to the assessment year relevant to the previous year in which he has declared lower profit.
E.g. an assessee claims to be taxed on presumptive basis u/s 44AD for A.Y. 2022-23. For A.Y. 2023-24 and
2024-25, he offers income on the basis of presumptive taxation scheme. However, for A.Y. 2025-26, he did
not opt for presumptive taxation Scheme. In this case, he will not be eligible to claim benefit of presumptive
taxation scheme for next 5 A.Y.s, i.e. from A.Y. 2026-27 to 2030-31.
6. Effect on the assessee if sec. 44AD(4) is applicable: An assessee to whom provision of sec. 44AD(4) is
applicable and whose total income exceeds the maximum amount which is not chargeable to tax (i.e., basic
exemption limit), he shall be required:
¾¾ To maintain books of account and other documents as required u/s 44AA; and
¾¾ To get his accounts audited and furnish a report of such audit as prescribed u/s 44AB

Illustration 78:
X Co., a firm, is engaged in the business of trading of cloth (turnover of 2023-24 being ₹ 57,80,000, out of which ₹
25,00,000 has been received in account payee cheque). It wants to claim the following deductions:

Particulars Amount
Salary and interest to partners [as permitted by sec. 40(b)] 60,000
Salary to employees 4,90,000
Depreciation 2,70,000
Cost of materials used 35,90,000
Other expenses 13,45,000
Total 57,55,000
Net profit (₹ 57,80,000 – ₹ 57,55,000) 25,000
Determine the net income of X & Co. for the assessment year 2024-25 assuming that (i) taxable interest income is
₹ 90,000; (ii) Long term capital gain is ₹ 1,40,000; and (iii) the firm is eligible for a deduction of ₹ 15,000 under
sec. 80G.
Solution :
Since turnover from business does not exceed ₹ 2 crore, hence sec. 44AD is applicable. However, income computed
as per provision other than provision of sec. 44AD is less than estimated income, hence, the firm may be assessed
for such lesser income provided following conditions are satisfied –
a. Maintain books of account as prescribed u/s 44AA; and
b. Get accounts audited u/s 44AB.
Where it maintains accounts and gets it audited
Computation of total income of X & Co. for the A.Y. 2024-25

242 The Institute of Cost Accountants of India


Heads of Income

Particulars Amount
Profits and gains of business or profession: Income from cloth business 25,000
Capital gains: Long term capital gain 1,40,000
Income from Other Sources: Interest Income 90,000
Gross Total Income 2,55,000
Less: Deduction u/s 80G 15,000
Total Income 2,40,000
It is assumed that all the expenditures are allowed.
Where it does not maintain account or fails to get accounts audited
Computation of total income of X & Co. for the A.Y.2024-25

Particulars Details Amount


Profits and gains of business or profession
Income from cloth business (being 6% of ₹ 25,00,000) 1,50,000
Income from cloth business (being 8% of ₹ 32,80,000) 2,62,400 4,12,400
Capital gains: Long term capital gain 1,40,000
Income from Other Sources: Interest Income 90,000
Gross Total Income 6,42,400
Less: Deduction u/s 80G 15,000
Total Income 6,27,400

2.3.65 Computation of Professional Income on Presumptive Basis [Sec. 44ADA]


Applicable to Any resident individual and resident firm (other than LLP)
a. Engaged in Profession: Assessee must be engaged in any profession referred to in sec. 44AA
(i.e., Legal, medical, engineering, architectural profession or profession of accountancy,
Conditions technical consultancy, interior decoration, etc.)
b. Maximum Receipts: Gross receipts of the assessee in the previous year should not exceed
₹ 50 lakh.(₹ 75 lakh where 95% of the amount received are in non-cash mode).
Estimated 50% of the gross receipts.
income However, a taxpayer can voluntarily declare a higher income in his return.
Notes
1. Deduction u/s 30 to 38: The estimated income is comprehensive and no further deductions u/s 30 to 38 shall
be allowed.
2. Depreciation: Depreciation is deemed to have been already allowed. The written down value of asset will be
calculated, as if depreciation has been allowed.
3. Deductions: The above estimated income is aggregated with other income of the assessee, from any other
business or under any other heads of income. Further deduction under chapter VIA shall be available to the
assessee as usual.
4. Brought forward loss: Brought forward loss (if any) shall be subtracted from such estimated income as per
provisions of this Act.

The Institute of Cost Accountants of India 243


Direct Taxation

5. Effect if assessee declares lower income: An assessee can declare his income lower than the estimated income
as per provision of this section. In such case he will have to:
¾¾ Maintain books of account and other documents as required u/s 44AA if his total income exceeds the
maximum exemption limit; and
¾¾ Get his accounts audited and furnish a report of such audit as prescribed u/s 44AB (irrespective of amount
of turnover or gross receipts) if his total income exceeds the maximum exemption limit.
Note: Assessee can change his option from year to year

2.3.66 Business of plying, leasing or hiring goods carriage [Sec. 44AE]


Applicable to All assessee engaged in the business of plying, hiring or leasing goods carriage.
Number of carriages: Assessee must not own more than 10 goods carriages at any time during
the previous year.
Owner of carriages includes a buyer under hire purchase or installment system even if the
Condition whole amount is unpaid.
Goods carriage means any motor vehicle constructed or adapted for use solely for the carriage
of goods, or any motor vehicle not so constructed or adapted when used for the carriage of
goods;
Income from each goods carriage shall be
Type of Goods Carriage Presumptive Income (Per month or part of a month)
Heavy ₹ 1,000 per ton of gross vehicle weight or unladen weight
Estimated
income Other ₹ 7,500
1. Income shall be calculated from the month when assessee acquired the property whether
it has been put to use or not.
2. An assessee can declare higher income.
Notes:
1. Heavy goods vehicle means any goods carriage, the gross vehicle weight of which exceeds 12000 kilograms
2. Deduction u/s 30 to 38: The estimated income is comprehensive and no further deductions u/s 30 to 38 shall
be allowed.
3. Deduction u/s 40(b): In the case of a firm, deduction in respect of remuneration and interest to partner u/s
40(b) shall be further deductible from income so computed.
4. Depreciation: Depreciation is deemed to have been already allowed. The written down value of asset will be
calculated, as if depreciation has been allowed.
5. Deductions: The above estimated income is aggregated with other income of the assessee, from any other
business or under any other heads of income. Further deduction under chapter VIA shall be available to the
assessee as usual.
6. Brought forward loss: Brought forward loss (if any) shall be adjusted from such estimated income.
7. Maintenance of books of account and audit: An assessee, who estimates income from such business as per
section 44AE, or a higher income, is not required to -
¾¾ Maintain books of account u/s 44AA; and
¾¾ Get his accounts audited u/s 44AB
- in respect of his income from such business.

244 The Institute of Cost Accountants of India


Heads of Income

However, he has to comply with the requirements of both sec. 44AA and 44AB in respect of his other
businesses. Further to note that in computing the monetary limits u/s 44AA and 44AB, the gross receipts or
income from the said business shall be excluded.
8. Effect if assessee declares lower income: An assessee can declare his income lower than the estimated
income as per provision of this section. In such case he will have to
¾¾ Maintain books of account and other documents as required u/s 44AA; and
¾¾ Get his accounts audited and furnish a report of such audit as prescribed u/s 44AB irrespective of amount
of turnover or gross receipts.
Note: Assessee can change his option from year to year.

Illustration 79:
Mr. Sukhvinder is engaged in the business of plying goods carriages. On 1st April, 2023, he owns 10 trucks (out
of which 6 are heavy good vehicles of (unladen weight of each is 20 ton)). On 2/5/2023, he sold one of the heavy
goods vehicles & purchased a light goods vehicle on 6th May, 2023. This new vehicle could however be put to use
only on 15-6-2023.
Compute the total income of Mr. Sukhvinder for the A.Y. 2024-25, taking note of the following data:

Particulars Amount Amount


Freight Charges collected 8,70,000
Less: Operational expenses 6,25,000
Depreciation as per Sec. 32 1,85,000
Other Office expenses 15,000 8,25,000
Net Profit 45,000
Other business and non-business income 70,000
Solution :
Alternative 1) Direct estimation of income u/s 44AE

Vehicle No. of vehicle Details Amount


Light 4 ₹ 7,500 * 4 vehicles * 12 months 3,60,000
Heavy 5 ₹ 1,000 * 5 vehicles * 12 months * 20 ton 12,00,000
Heavy 1 ₹ 1,000 * 1 vehicle * 2 months * 20 ton
#
40,000
Light 1 ₹ 7,500 * 1 vehicles * 11# months 82,500
Income from business of plying goods carriage 16,82,500
Add: Other business and non-business income 70,000
Total Income 17,52,500
#
Income shall be calculated from the month when assessee acquired the property whether it has been put to use or
not. For this purpose, any fraction of the month shall be considered as month.

The Institute of Cost Accountants of India 245


Direct Taxation

Alternative 2) Computation of income as per the provision of sec. 28 to 38

Particulars Amount Amount


Freight charges collected 8,70,000
Less: Expenditure related to business
Operational expenses 6,25,000
Depreciation u/s 32 1,85,000
Other office expenses 15,000 8,25,000
Income from business of plying goods carriage 45,000
Add: Other business and non-business income 70,000
Total Income 1,15,000
Since Mr. Sukhvinder has lower taxable income in alternative 2 hence his total income is ₹ 1,15,000. But to claim
such lower income than the estimated income (computed in alternative 1) as per provision of section 44AE, he will
have to —
- Maintain books of account as required u/s 44AA; and - Get his accounts audited.

2.3.67 Computation of income from construction and service contracts [Sec. 43CB]
The profits and gains arising from a construction contract or a contract for providing services shall be determined
on the basis of percentage of completion method in accordance with the ICDS.
Taxpoint:
 Profits and gains arising from a contract for providing services:

Case Method
Contract for providing services with duration of not more than 90 days Project completion method
A contract for providing services involving indeterminate number of acts over Straight line method
a specific period of time

 For the purpose of percentage of completion method:


¾¾ the contract revenue shall include retention money;
¾¾ the contract costs shall not be reduced by any incidental income in the nature of interest, dividends or
capital gains.

2.3.68 Consequences of undisclosed income or investment Amended


Following undisclosed income shall be charged to tax –

Section Conditions Tax Treatment


a. Any sum credited in the books of account of the
assessee; and
Sec. 68 The sum so credited may be
b. Assessee offers no explanation about the nature and charged to tax as the income of
Cash Credit source thereof (or the explanation offered by him the assessee of that previous year.
is not satisfactory in the opinion of the Assessing
Officer)

246 The Institute of Cost Accountants of India


Heads of Income

Section Conditions Tax Treatment

Notes
¾¾ Where the assessee is a company, (not being a company in which the public are
substantially interested) and the sum so credited consists of share application money,
share capital, share premium or any such amount by whatever name called, any
explanation offered by such assessee-company shall be deemed to be not satisfactory,
unless:
–– the person, being a resident in whose name such credit is recorded in the books of
such company also offers an explanation about the nature and source of such sum
so credited; and
–– such explanation in the opinion of the Assessing Officer aforesaid has been found
to be satisfactory:
¾¾ Where the sum so credited consists of loan or borrowing or any such amount, by
whatever name called, any explanation offered by such assessee shall be deemed to be
not satisfactory, unless,—
–– the person in whose name such credit is recorded in the books of such assessee also
offers an explanation about the nature and source of such sum so credited; and
–– such explanation in the opinion of the Assessing Officer aforesaid has been found
to be satisfactory
However, if the person, in whose name the sum (share application or loan or borrowing)
referred to therein is recorded, is a venture capital fund or a venture capital company as
referred to in sec.10(23FB), then the aforesaid notes are not applicable.

a. In a previous year the assessee has made investments


which were not recorded in the books of account;
Sec. 69 and Value of the investment may be
Unexplained Assessee offers no explanation about the nature and deemed to be the income of the
investments source of the investments (or the explanation offered by assessee for such previous year.
him is not satisfactory in the opinion of the assessing
officer)

a. In a previous year the assessee is found to be the


owner of any money, bullion, jewellery, or other
valuable article;
Sec. 69A The money and the value of such
b. Such money, bullion, jewellery, or other valuable
assets may be deemed to be the
Unexplained article is not recorded in the books of account; and
income of the assessee for such
money, etc. c. The assessee offers no explanation about the nature
previous year.
and source of acquisition of such assets (or the
explanation offered by him is not satisfactory in the
opinion of the Assessing Officer)

The Institute of Cost Accountants of India 247


Direct Taxation

Section Conditions Tax Treatment


a. In a previous year the assessee has made investments
or is found to be the owner of any bullion, jewellery
Sec. 69B or other valuable article;
b. Assessing Officer finds that the amount expended
Amount of on making such investments or in acquiring such The excess amount may be
Investments, etc., bullion, jewellery or other valuable article exceeds deemed as income of the assessee
not fully disclosed the amount recorded in this behalf in the books of for such previous year.
in books of account maintained by the assessee; and
account c. The assessee offers no explanation about such
excess amount (or the offered explanation is not
satisfactory in the opinion of the AO)
a. In a previous year an assessee had incurred any
expenditure; and Amount incurred on such
Sec. 69C b. Assessee offers no explanation about the source of expenditure or part thereof, may
such expenditure or part thereof (or the explanation, be deemed to be the income of the
Unexplained
if any, offered by him is not satisfactory in the assessee for such previous year.
expenditure etc.
opinion of the Assessing Officer)
Note: The proviso to sec. 69C provides that notwithstanding anything contained in any
other provisions of the Act, such unexplained expenditure which is deemed to be the income
of the assessee shall not be allowed as a deduction in any year under any head of income.
Amount so borrowed or repaid
Any amount is borrowed on a hundi from, or any shall be deemed to be the
amount due thereon is repaid to, any person otherwise income of the person borrowing
than through an account payee cheque. or repaying the amount for
the previous year in which the
Sec. 69D amount was borrowed or repaid
Amount borrowed as case may be.
or repaid on hundi Notes
¾¾ For avoiding double taxation, if in any case any amount borrowed on a hundi has been
taxed under this section, such person shall not be liable to tax again in respect of such
amount on repayment of such amount.
¾¾ Amount repaid shall include the amount of interest paid on the amount borrowed.
Income referred to in sec. 68 to sec. 69D shall be taxable @ 60% (+ SC @ 25% + cess)
Tax Rate without giving effect to any deduction in respect of any expenditure or allowance or set off
of any loss. [Sec. 115BBE]

General Illustrations
Illustration 80:
Mr. Sunil is a practicing Chartered Accountant. He also runs a private coaching institute. His bank accounts for the
year ended 31/3/2024 is given below:
Receipts ₹ Payments ₹
To Balance b/f 20,000 By Office expenses 18,000

248 The Institute of Cost Accountants of India


Heads of Income

Receipts ₹ Payments ₹
To Audit fees 2,00,000 By Municipal tax on property 800
To Income from other professional work 1,00,000 By Coaching expenses 800
To Coaching fees 1,200 By Personal expenses 5,000
To Interest on Investment 2,000 By Membership fees 500
To Examiner’s fees 1,000 By Life insurance premium 13,000
To Rent from property 5,000 By Income tax 5,000
By Motor Car purchased 1,80,000
By Motor Car expenses 10,200
By Insurance of property 1,600
By Balance c/d 94,300
3,29,200 3,29,200

Additional Information
a. 20% of motor car expenses is in respect of profession.
b. Depreciation allowance for motorcar is ₹ 27,000, if wholly used for profession.
c. Outstanding fees on 31-3-2024 ₹ 2,000. Whereas ₹ 500 receivable from Mita is considered as bad.
d. Outstanding fees of P.Y. 2020-21 ₹ 10,000 received during the year, which is included in the audit fees.
e. Office expenses include payment of ₹ 2,000 incurred during the previous year 2022-23.
Compute his gross total income for the A.Y. 2024-25 assuming he maintains accounts on cash basis.
Solution :
Computation of total income of Mr. Sunil for the A.Y. 2024-25
Particulars Workings Details Details Amount
Income from house property
Gross Annual Value Rent received 5,000
Less: Municipal tax 800
Net Annual Value 4,200
Less: Deduction u/s
24(a) Standard Deduction 30% of NAV 1,260
24(b) Interest on loan Nil 1,260 2,940
Profits & gains of business or profession
Audit fees 2,00,000
Income from other professional work 1,00,000 3,00,000
Less: Expenses allowed
Office expenses 18,000

The Institute of Cost Accountants of India 249


Direct Taxation

Particulars Workings Details Details Amount


Membership fees 500
Motor car expenses 20% of expenses 2
2,040
Depreciation on motor car 20% of depreciation 2
5,400 25,940 2,74,060
Income from other sources
Interest on investment 2,000
Coaching fees 1,200
Less: Coaching expenses 800 400
Examiners’ fees 1,000 3,400
Gross Total Income 2,80,400
Notes
1. Insurance premium on property is not deductible from income from house property.
2. As 20% use of motor car is related to professional purpose, hence as per sec. 38 expenditure and depreciation
is apportioned.
3. Payment of LIC premium is a personal expense. However, deduction u/s 80C is available.
4. Income tax is specifically disallowed u/s 40(a).
5. As per sec. 145, income chargeable under the head “Profits & gains of business or profession” shall be
computed only in accordance with the method of accounting regularly followed by the assessee. In this case,
assessee follows cash system of accounting.

Illustration 81:
From the following particulars of Shri Khote for the year ending 31st March, 2024, find out his taxable income from
business for the assessment year 2024-25:

Particulars ₹ Particulars ₹
To Opening Stock 1,20,000 By Sales 2,14,20,000
To Purchases 2,10,00,000 By Profit on sale of import licence 5,000
To Salaries 25,000 By Gift received 24,000
To Legal Expenses 10,000 By Closing Stock 2,00,000
To Bad Debts 5,000
To Rent 50,000
To Interest on loan 2,500
To Depreciation 15,000
To Income tax paid 2,000
To Outstanding Customs Duty 25,000
To Advertisement 2,000

250 The Institute of Cost Accountants of India


Heads of Income

Particulars ₹ Particulars ₹
To Legal expenses 12,000
To Contribution towards URPF 5,000
To General expenses 17,500
To Traveling expenses 1,00,000
To Net Profit 2,58,000
2,16,49,000 2,16,49,000
In computing the income, the following facts are to be taken into consideration:
1. Interest on loan is paid to brother of Shri Khote for loan taken for payment of advance income tax.
2. During the previous year 2018-19, assessee had claimed ₹ 45,000 as bad debt out of which only ₹ 35,000 was
allowed. During the previous year, he recovers ₹ 25,000.
3. Contribution towards unrecognised provident fund was paid within time.
4. Legal expenses include ₹ 2,000 paid for preparation of income tax return.
5. Stock is undervalued by 10%.
6. Gift received was given by a supplier for achieving target sale.
7. Outstanding customs duty has been paid on 31-12-2024.
8. During the previous year, he comes to know that his former employee had embezzled cash of ₹ 5,000 on 31-
3-2023, which was not accounted for.
9. Traveling expenses include ₹ 50,000 being cost of trip to Singapore by an employee for 10 days. However,
only 8 days of trip is useful to business and 2 days has been allowed as holiday to employee.
10. Rent includes expenditure on extension of shed on rented building ₹ 26,000. However, such extension was
completed on 1-5-2024 with total cost of ₹ 50,000.
11. General expenses includes –
¾¾ Salary of ₹ 1,200 paid to domestic servant.
¾¾ Compensation of ₹ 2,000 paid for retrenchment of an employee.
Compute his business income for the A.Y. 2024-25
Solution :
Computation of Profits and gains of business or profession of Shri Khote for the A.Y. 2024-25

Particulars Notes Details Amount


Net profit as per Profit and Loss A/c 2,58,000
Add: Expenditure disallowed but debited in P/L A/c
Income tax paid 1 2,000
Outstanding Customs Duty 2 25,000
Contribution towards unrecognised provident fund 3 5,000

The Institute of Cost Accountants of India 251


Direct Taxation

Particulars Notes Details Amount


Interest on loan 4 2,500
Expenditure on extension of building shed 5 26,000
Salary paid to domestic servant 6 1,200
Add: Income taxable but not credited to P/L A/c
Recovery of bad debts [₹ 25,000 – (₹ 45,000 – ₹ 35,000)] 7 15,000 76,700
3,34,700
Less: Expenditure allowed but not debited to P/L A/c
Embezzlement by employee 8 5,000
3,29,700
Adjustment for valuation of stock
Add: Under valuation of closing stock 12 22,222
Less: Under valuation of opening stock 13 13,333 8,889
Profits and gains of business or profession 3,38,589
Notes
1. Income tax is specifically disallowed u/s 40(a).
2. Customs Duty paid after due date of filing of return shall not be allowed as deduction [Sec. 43B]
3. Contribution to unrecognised provident fund is disallowed.
4. Interest on loan taken for payment of advance tax is disallowed.
5. Extension of building shed is an expenditure of capital nature, hence disallowed u/s 30.
6. Any expenditure of personal nature is disallowed.
7. As per sec. 41(4), where a deduction has been allowed in respect of bad debt or part of debt u/s 36(1)(vii),
then if the amount subsequently recovered on any such debt or part is greater than the difference between the
debt or part of debt and the amount so allowed, the excess shall be deemed to be profits and gains of business
or profession.
8. Loss by embezzlement of cash by employee is allowed as deduction in the year in which such fact was known
to the assessee.
9. Legal expenditure for preparation of income tax return is allowed expenditure u/s 37(1).
10. Gift received for achieving target-sale is perquisite related to business and shall be taxable u/s 28.
11. Traveling expenditure shall be fully allowed as deduction, as trip was for 10 days out of which 8 days spent
for business purpose and remaining 2 days trip shall be treated as staff welfare expenditure being allowed u/s
37(1).
12. Under valuation of closing stock
Actual value of closing stock (₹ 2,00,000/90%) = ₹ 2,22,222
Under valuation of closing stock is 10% of ₹ 2,22,222 = ₹ 22,222

252 The Institute of Cost Accountants of India


Heads of Income

13. Under valuation of opening stock


Actual value of opening stock (₹ 1,20,000/90%) = ₹ 1,33,333
Under valuation of opening stock is 10% of ₹ 1,33,333 = ₹ 13,333

Illustration 82:
During the previous year 2023-24, profit and loss account of Shri Raj, proprietor of Raj Enterprises engaged in the
business of readymade garments, shows profits of ₹ 1,50000. With the following information, compute his taxable
income from business –
a. Interest on capital ₹ 5,000
b. Purchases include goods of ₹ 12,000 from his younger brother in cash. However, market value of such goods
is ₹ 9,000.
c. Interest paid outside India ₹ 1,00,000 without deducting tax at source.
d. Penalty paid to Government for non-filing of GST return ₹ 5,000
e. Penalty paid to customer for non-fulfilling of order within time ₹ 10,000
f. Bad debts ₹ 1,00,000. Money has been advanced for purchase of Building.
g. Revenue expenditure on promoting family planning among employees ₹ 10,000.
h. Premium paid on health of employees ₹ 6,000 in cash
i. Premium paid on health of his relatives ₹ 6,000 in cheque
j. Employer’s contribution to RPF ₹ 12,000. One-half of the amount is paid after due date as per relevant Act
but before 31-3-2024.
k. Employees contribution to RPF ₹ 10,000. ½ of the amount is paid after due date as per relevant Act.
l. Interest on late payment of professional tax ₹ 1,000 (yet to be paid)
m. Interest on loan from State Bank of India ₹ 10,000 (₹ 5,000 is not paid till due date of filing of return)
n. Interest on late refund from income tax department ₹ 500
o. Sale includes sale to Raj ₹ 10,000. (Cost of such goods ₹ 8,000; Market value of such goods ₹ 12,000)
p. He received ₹ 80,000 from a debtor at a time in cash.
q. Recovery of bad debt ₹ 10,000 (out of which ₹ 8,000 was allowed as deduction during A.Y.2018-19)
r. Depreciation (being not debited in accounts) ₹ 20,000 allowed as deduction u/s 32
Solution :
Computation of Profits and gains of business or profession of Shri Raj for the A.Y. 2024-25

Particulars Note Details Amount


Net profit as per Profit and Loss account 1,50,000
Add: Expenditure disallowed but debited in P/L A/c
Interest on capital 1 5,000

The Institute of Cost Accountants of India 253


Direct Taxation

Particulars Note Details Amount


Payment to relative in excess of market value of goods 2 3,000
Interest paid outside India without deducting tax at source 3 1,00,000
Penalty paid to government for non-filing of GST return 4 5,000
Bad debt 6 1,00,000
Premium paid on health of employees in cash 8 6,000
Premium paid on health of his relatives in cheque 9 6,000
Employees contribution to RPF 11 5,000
Interest on loan from State Bank of India 13 5,000
Cost of goods sold to himself 14 8,000 2,43,000
3,93,000
Less: Expenditure allowed but not debited in P/L A/c
Depreciation u/s 32 20,000
Less: Income not taxable but credited to P/L A/c
Sales to himself (goods withdrawn for personal purpose) 14 10,000
Recovery of bad debts 15 2,000
Less: Income taxable under other head but credited to P/L A/c
Interest on late refund from income tax department 16 500 32,500
Profits and gains of business or profession 3,60,500
Notes
1. Interest on capital to proprietor is not allowed as no one can earn from a transaction with himself.
2. Any unreasonable payment to relative is disallowed u/s 40A(2). Hence, ₹ 3,000 is disallowed. Since cash
payment towards allowed expenditure (i.e. ₹ 9,000) does not exceed ₹ 10,000, hence provision of sec. 40A(3)
is not applicable.
3. Interest paid outside India without deducting tax at source is disallowed u/s 40(a).
4. Any payment made for infringement of law is disallowed.
5. Payment made for non-fulfilling of contract is not a payment for infringement of law. Hence, allowed u/s
37(1).
6. Bad debt is allowed only when such debt has been taken into account as income of previous year or any
earlier previous year(s) [Sec. 36(1)(vii)]. Since, the debt is in respect of purchase of a building, which was not
considered as income of any previous year, hence it is disallowed.
7. Any expenditure for promoting family planning is allowed to company assessee [Sec. 36(1)(ix)]. However,
such expenditure (revenue in nature) incurred by assessee other than company shall be allowed u/s 37(1).
8. Payment of insurance premium on health of employees in cheque is allowed u/s 36(1)(ib).
9. Payment of insurance premium on health of relative is not related to business, hence disallowed.

254 The Institute of Cost Accountants of India


Heads of Income

10. Employer’s contribution towards RPF is allowed if payment is made before due date of filing of return
irrespective of fact that such payment was made after due date prescribed in the relevant Act.
11. Any sum received from employees as their contribution towards RPF is allowed only when such sum has been
credited to such fund within the due date prescribed in the relevant Act [Sec. 36(1)(va)].
12. Interest on late payment of professional tax is not a penalty but compensatory in nature. Hence, it is allowed
u/s 37(1). Further such interest is not governed by the provisions of sec. 43B.
13. Any interest payable to any scheduled bank is allowed on cash basis [Sec. 43B]. Hence, unpaid amount is
disallowed.
14. Any expenditure of personal nature is not allowed. Further, no one can earn from a transaction with himself.
Hence, sale made to himself is not treated as income.
15. Bad debt recovery is treated as income in the year of recovery to the extent of bad debt allowed in the earlier
year [Sec. 41(4)]
16. Interest on late refund of income tax is taxable under the head ‘Income from other sources’.
17. Receipt from debtor ₹ 80,000 in cash is not attracted by provision of sec. 40A(3).

Quick MCQs:-

1. Under the head Business & Profession, the method of accounting which an assessee can follow shall be –
(a) Mercantile System only
(b) Cash System only
(c) Mercantile or Cash System at the assessee’s wish
(d) Hybrid System

2. Sums received by an employer from Keyman Insurance Policy taken on the life of the employee shall be-
(a) Exempt
(b) Taxable under the head Business and Profession
(c) Taxable under the head Other Sources
(d) Taxable in the hand of the employee

3. Depreciation is allowed in case of –


(a) Tangible Assets only
(b) Intangible Assets only
(c) Tangible and Intangible Assets
(d) None of above

4. A car is imported after 01-04-2023 by N Ltd. from London to be used by its employee. N Ltd. shall be
allowed depreciation on such car at-
(a) 15%
(b) 40%
(c) Nil
(d) None of the above

The Institute of Cost Accountants of India 255


Direct Taxation

5. Unabsorbed Depreciation which could not be set of in the same assessment year, can be carried forward
for –
(a) 8 Years
(b) Indefinitely
(c) 4 years
(d) None of above

6. Unabsorbed Depreciation brought forward from an earlier year of a particular business can be set off
from-
(a) The same business
(b) any head of income
(c) any Business Income
(d) any head of income except salary but first form business income.

7. The maximum deduction for Site Restoration Fund under Section 33ABA shall be-
(a) the amount deposited in scheme
(b) 20%of the profits from such business
(c) 20% of the amount deposited in the scheme
(d) Lower of (a) and (b)

8. If an Assessee carries on any Scientific Research related to his business, he shall be allowed deduction u/s
35 on account of –
(a) Revenue Expenditure
(b) Capital Expenditure
(c) Both Revenue and Capital Expenditure
(d) Both Revenue and Capital Expenditure

9. Donation for Social of Statistical Research shall be allowed as deduction to the extent of –
(a) 50% of the donation so made
(b) 100% of the donation so made
(c) 125% of the donation so made
(d) 150% of the donation so made.

10. In case of Non-Company assessee, the total Preliminary Expenses incurred are allowed as deduction of
the extent of –
(a) 2% of the cost of the project
(b) 5% of the cost of the project
(c) 10% of the cost of the project
(d) None of the above

256 The Institute of Cost Accountants of India


Heads of Income

11. Capital Expenditure incurred on family planning amongst employees of the Company assessee is allowed
as deduction-
(a) in full
(b) in 5 equal installments
(c) in 10 equal installments
(d) None of above.

12. Deduction under Section 37(1) shall be given for expenditure of-
(a) Revenue Nature
(b) Capital Nature
(c) Both Revenue and Capital Nature
(d) None of above

13. Tax audit is compulsory in case a person is carrying on business whose gross Turnover/Sales /Receipts,
as the case may be, exceeds-
(a) D 10 Lakhs
(b) D 60 Lakhs
(c) D 1 Crore
(d) None of the above

14. If the Assessee opts for presumptive income scheme under section 44AD or 44AE, then the assessee
shall:
(a) not be entitled to any deduction u/s 30 to 37
(b) be entitle to deduction u/s 30 to 37
(c) Some are entitled
(d) Some are not entitled to deduction u/s 30 to 37

15. For an eligible new industrial undertaking fulfilling the conditions, additional depreciation in respect of a
machinery costing D 10 lakhs acquire and installed on 04-10-2023 is-
(a) D 1,00,000
(b) D 1,50,000
(c) D 2,00,000
(d) None of the above.

The Institute of Cost Accountants of India 257


Direct Taxation

Capital Gains 2.4


As per sec. 45(1), profits or gains arising on transfer of a capital asset shall be chargeable under the head “Capital
Gains”.
Taxpoint: Following are the essential conditions to be satisfied to charge any income under the head “Capital
Gains”:
a. There must be a capital asset.
b. The assessee transfers such capital asset.
c. There must be profit or gain (including negative profit or gain) on such transfer.

The transferred asset should be capital asset at the time of transfer.

2.4.1 Basis of Charge


Capital gain shall be taxable in the previous year in which the asset is transferred.
However, in some cases, capital gain is taxable in the previous year in which consideration is received rather than
in the previous year in which transfer took place e.g. compulsory acquisition by the Government (discussed later
in this chapter).

2.4.2 Capital Asset [Sec. 2(14)]


Capital asset means –
 any kind of property held by an assessee, whether or not in connection with his business or profession;
 any securities held by a Foreign Institutional Investor which has invested in such securities in accordance with
the regulations made under the Securities and Exchange Board of India Act, 1992;
 any unit linked insurance policy to which exemption u/s 10(10D) does not apply due to applicability of the
fourth and fifth provisos thereof *
Note: Capital asset may be movable or immovable or tangible/corporeal (furniture, jewellery, etc.) or
intangible/incorporeal (goodwill, tenancy right, copy right, etc.)

* Fourth and fifth proviso provides that

Where any unit linked insurance policy (ULIP), is issued on or after 01-02-2021 and the premium payable for any of the previous year during the term
of such policy exceeds ₹ 2,50,000

Where the premium is payable, by a person, for more than one ULIP, issued on or after 01-02-2021, the exemption shall apply only with respect to those
ULIP, where the aggregate amount of premium does not exceed the aforesaid limit in any of the previous year during the term of any of those policies.

258 The Institute of Cost Accountants of India


Heads of Income

“Property” includes any rights in or in relation to an Indian company, including rights of management or
control or any other rights whatsoever
- but does not include the following:
1. Stock in trade
Stock in trade, consumable stores or raw materials held for business or profession.

However, any securities held by a Foreign Institutional Investor which has invested in such securities in
accordance with the regulations made under the Securities and Exchange Board of India Act, 1992 shall not
be treated as stock-in-trade
Treatment of profit on sale of stock
Such profit shall be taxable under the head “Profits & gains of business or profession”
2. Personal effect
Personal effect means any movable property held for personal use of the assessee or for any dependent member
of his family but excludes the followings:

A. jewellery# B. archaeological collections C. drawings


D. paintings E. sculptures; or F. any work of art

Taxpoint
¾¾ An immovable property and aforesaid assets held for personal use are not personal effect and hence are
capital assets. E.g. a house property even though used for personal purpose cannot be treated as personal
effect and shall fall within the definition of capital assets.
¾¾ Securities are not personal effect.
¾¾ Personal effect includes wearing apparel, furniture, car, cycle, scooter used by the assessee for personal
purpose.
¾¾ Intangible asset does not have personal effect.
#
Jewellery includes –
¾¾ ornaments made of gold, silver, platinum, any other precious metal or any alloy containing one or more
of such precious metals. It is immaterial whether or not such ornaments contain any precious or semi-
precious stones and whether or not such ornaments are worked or sewn into any wearing apparel;
¾¾ precious or semi precious stones whether or not set in any furniture utensil or other article or worked or
sewn in any wearing apparel. E.g. loose diamond shall be treated as jewellery.
 Treatment of profit on sale of personal effect
Any income on transfer of personal effect shall not be treated as capital gain. Such income is in the
nature of capital receipt and hence shall not be taxed under any head.
3. Agricultural land in rural area
Agricultural land in India is not a capital asset except the following –
a. land which is situated within the jurisdiction of any Municipality (whether known as a municipality,
municipal corporation, notified area committee, town area committee, town committee, or by any other
name) or Cantonment Board having population of 10,000 or more; or
b. in any area within the distance, measured aerially,—
The Institute of Cost Accountants of India 259
Direct Taxation

Area within the aerial distance from the local


Population of the municipality or cantonment
limits of such municipality or cantonment
board
board is non-rural area
More than 10,000 but not exceeding 1,00,000 Upto 2 kilometres
More than 1,00,000 but not exceeding 10,00,000 Upto 6 kilometres
More than 10,00,000 Upto 8 kilometres
Notes
i. Population, according to the last preceding census of which the relevant figures have been published
before the first day of the previous year, shall be considered.
ii. If such land is not agricultural land, it will be treated as capital asset irrespective of its location.
iii. If agricultural land is located outside India, it will be treated as capital asset.
Treatment of profit on sale of agricultural land in rural area of India
Profit on sale of agricultural land in rural area shall not be treated as capital gain. Such income is in the nature
of capital receipt and hence shall not be taxed under any head.

Municipality or Cantonment board


Population ⇓
Upto 10,000 Agro Land not treated as Capital Asset

Within 10,001 to 1,00,000 Agro Land treated as Capital Asset

Within 1,00,001 to 10,00,000 Agro Land treated as Capital Asset

More than 10,00,000 Agro Land treated as Capital Asset

Area ⇒ Local limit 2 Km 6 Km 8 Km Beyond 8


Km
4. Gold Bonds
Following gold bonds issued by the Central Government are not capital asset:

 6.5% Gold Bond, 1977  7% Gold Bonds, 1980; and  National Defence Gold Bond, 1980
5. Special Bearer Bond
Special Bearer Bond, 1991 issued by the Central Government are not capital asset.
Note: It is not necessary that the assessee should be the initial subscriber.

6. Gold Deposit Bonds


Gold Deposit Bonds issued under the Gold Deposit Scheme, 1999 or deposit certificates issued under the Gold
Monetisation Scheme, 2015 notified by the Central Government are not capital asset.
Note: Interest on aforesaid bonds or deposits are exempt [Sec. 10(15)]

260 The Institute of Cost Accountants of India


Heads of Income

2.4.3 Types of Capital Asset

Exceptions

In the following cases, an asset shall be termed as a short-term capital asset (STCA) if it is held for not more than
following period before the date of transfer:
12 months 24 months
� Equity or preference share in a company (listed in India) ¾ ¾ Equity or preference share in an unlisted
company
¾¾ Any security e.g. debenture, Government securities, etc. ¾¾ Immovable property being land or building
(listed in India) or both
¾¾ A unit of an equity oriented fund* (whether quoted or not)
¾¾ Zero-Coupon Bonds (whether quoted or not)
¾¾ Units of UTI (whether quoted or not)

Short-term capital gain (STCG) arises on transfer of short-term capital assets (STCA) and long-term capital gain
(LTCG) arises on transfer of long-term capital assets (LTCA). However, any gain on transfer of an asset on which
depreciation is allowed as per WDV method u/s 32(1)(ii) shall be taxable as short-term capital gain (irrespective
of their period of holding) [Sec. 50].

2.4.4 Period of holding


While computing the period of holding of any capital asset, the following points are to be considered to decide the
nature of asset, whether short term or long term –
* "Equity Oriented Fund" means a fund set up under a scheme of a mutual fund specified u/s 10(23D) or under a scheme of an insurance company
comprising unit linked insurance policies to which exemption u/s 10(10D) does not apply on account of the applicability of the fourth and fifth
provisos thereof and:
i. in a case where the fund invests in the units of another fund which is traded on a recognised stock exchange,:
a. a minimum of 90% of the total proceeds of such fund is invested in the units of such other fund; and
b. such other fund also invests a minimum of 90% of its total proceeds in the equity shares of domestic companies listed on a recognised stock
exchange; and
ii. in any other case, a minimum of 65% of the total proceeds of such fund is invested in the equity shares of domestic companies listed on a recognised
stock exchange.
Taxpoint:
� The percentage of equity shareholding or unit held in respect of the fund, as the case may be, shall be computed with reference to the annual average
of the monthly averages of the opening and closing figures.
� In case of a scheme of an insurance company comprising ULIP to which exemption u/s 10(10D) does not apply on account of the applicability of
the fourth and fifth provisos thereof, the minimum requirement of 90% or 65%, as the case may be, is required to be satisfied throughout the term
of such insurance policy.

The Institute of Cost Accountants of India 261


Direct Taxation

a. Period after liquidation: In case of shares, if company goes into liquidation, the period after the date of
commencement of liquidation is to be excluded.
b. Date of transfer: For calculating the period of holding of a capital asset, the date on which the asset is
transferred is to be excluded.
c. Conversion of preference shares into equity shares: In such case, period of holding includes the period for
which the preference shares were held by the assessee. Similarly, period of holdings of units in the consolidated
plan of the scheme of the mutual fund includes the period for which units in a consolidating plan of a mutual
fund scheme were held by the assessee.

2.4.5 Transfer [Sec. 2(47)]


Transfer in relation to a capital asset includes:
a. Sale, Exchange & Relinquishment of the asset;
b. Extinguishment of any right in an asset;
c. Compulsory acquisition of an asset under any law;
d. Conversion of asset into stock-in-trade by the owner;
e. Any transaction of immovable property u/s 53A of the Transfer of Property Act, 1882;
f. Any transaction which has the effect of transferring or enabling the enjoyment of any immovable property.
g. Maturity or redemption of a zero coupon bond
Taxpoint:
 Above definition is indicative and not exhaustive
 Above definition is applicable only in relation to capital assets and not otherwise.
 It also includes
1*

¾¾ disposing of or parting with an asset or any interest therein, or


¾¾ creating any interest in any asset in any manner whatsoever,
directly or indirectly, absolutely or conditionally, voluntarily or involuntarily, by way of an agreement
(whether entered into in India or outside India) or otherwise, notwithstanding that such transfer of rights
has been characterised as being effected or dependent upon or flowing from the transfer of a share or
shares of a company registered or incorporated outside India.
Exchange
Exchange includes barter which means mutual transfer of ownership of one thing for the ownership of another
Notes: In case of exchange though there is only one transaction the tax liability arises on both the parties. The sale
consideration shall be taken as the fair market value of assets received.
Relinquishment of the asset
Relinquish literally means ‘to withdraw from’ or ‘to abandon’ or ‘to give up any thing or any right’ or ‘to cease

* The Apex Court in the case of Vodafone International Holdings B.V. –vs.- UOI [2012] 204 Taxman 408 has held that the transfer of shares of the
foreign holding company do not results in an extinguishment of the foreign company control of the Indian company and it also does not constitute an
extinguishment and transfer of an asset situated in India. Transfer of the foreign holding company’s share off-shore, cannot result in an extinguishment
of the holding company right of control of the Indian company nor can it be stated that the same constitutes extinguishment and transfer of an asset/
management and control of property situated in India.
To nullify the decision of the Apex Court, Finance Act, 2012 has inserted the said explanations w.r.e.f. 01-04-1962.

262 The Institute of Cost Accountants of India


Heads of Income

to hold’ or ‘to surrender’. Hence, relinquishment means act of surrendering. In other words, it means interest of a
person in a property is either given up, abandoned, or surrendered but the property in which right is relinquished
continues to exist.
Taxpoint:
 Right of the assessee in the asset is given up or abandoned;
 Asset itself continues to exist after such relinquishment and becomes the property of someone else.
 There must be mutual consent of the parties. A unilateral action of writing off the claim in the books of account
cannot be treated as relinquishment.
Extinguishment of any right in an asset
Extinguishment literally means ‘to put a total end to’ or ‘total destruction’ or ‘blot out of existence’ or ‘annihilation’.
Extinguishment does not mean extinguishment of asset itself but to extinguishment of holder’s right to the asset
and such right cannot be held by someone else
Incidences of extinguishments –
 Cancellation of licences: Abandonment of a project and termination of industrial licence, shall be treated as
transfer.
 Reduction of share capital: When a part of the share capital is paid to the shareholder by a company, such
reduction of share capital shall be treated as extinguishment of proportionate right in the shares and such
shareholder shall be liable to capital gain.
When there is a reduction of capital by a company and amounts are distributed to shareholder, such amount
has two components –
¾¾ Distribution attributable to accumulated profits i.e. chargeable as deemed dividend u/s 2(22)(d);
¾¾ Distribution attributable to capital i.e. subject to tax u/s 45
 Forfeiture of share: Forfeiture or surrender of shares indicates extinguishment or relinquishment of right of
shareholder in such shares, which have been forfeited by the company. As in CIT vs Vania Silk Mills Pvt. Ltd
it was held that the term extinguishment includes all possible transactions which results in the destruction,
annihilation, termination, cessation or cancellation of any right in an asset whether corporeal or incorporeal.
Though there is no consideration in case of share forfeiture or surrender, still such transaction shall be treated
as transfer and liable to capital gain.
Compulsory acquisition of an asset under any law
Normally, sale means a mutual will full agreement between two or more parties. But for the purpose of sec. 2(47),
transfer includes compulsory acquisition of any property under any law in force.
Conversion of asset into stock in trade by the owner
Generally, a transfer requires two or more parties, but in Income tax Act even one party’s involvement may
constitute transfer. As per sec. 2(47)(iv) where an asset is converted by the owner into or treated by him as ‘stock
in trade’ of the business carried on by him, such conversion or treatment shall constitute transfer.
Any transaction of immovable property u/s 53A of the Transfer of Property Act
Any transaction of immovable property in which possession is allowed against part performance of the contract
shall be treated as transfer [Sec. 53A of the Transfer of Property Act, 1882].
Any transaction which has enabled the enjoyment of any immovable property
Any transaction which has the effect of transferring or enabling the enjoyment of any immovable property whether

The Institute of Cost Accountants of India 263


Direct Taxation

by way of becoming a member of, or acquiring shares in a co-operative society, company or other association of
persons or by way of any agreement or any arrangement or in any other manner, is treated as transfer.
Property held by a member of a company, co-operative society or other association of persons to whom a building
or a part thereof is allotted or leased under House Building Scheme of the company or association, is treated as
deemed owner of that building or a part thereof.
Under the scheme of owning flats in co-operative housing societies, the legal ownership in the flats can be said
to vest in the individual members themselves and not in the co-operative societies. Therefore for all purposes
including attachment and recovery of tax, etc. the individual member should be regarded as the legal owner.
Taxpoint
 Assessee is a member of a company, co-operative society or other AOP.
 He has been allotted or leased a building on account of such membership.

2.4.6 Transactions not regarded as transfer (Sec. 46 & 47)


By virtue of sec. 46(1) and sec. 47 the following transactions do not constitute transfer for the purpose of capital
gain -

Section Transaction
Any distribution of capital assets in the event of liquidation by a company to its shareholders shall
46(1)
not be treated as transfer in the hands of company.
47(i) Any distribution of capital assets on the total or partial partition of an HUF.
Any transfer of a capital asset under a gift or will or an irrevocable trust.
47(iii) Exception: Gift of shares acquired through Employees Stock Option Plan (ESOP) shall be treated
as Transfer
47(iv) Any transfer of a capital asset by a 100% holding company to its Indian subsidiary company.
47(v) Any transfer of a capital asset by a 100% subsidiary company to its Indian holding company
Any transfer, in a scheme of amalgamation, of a capital asset by the amalgamating company to the
amalgamated company if the amalgamated company is an Indian company.
47(vi)
Taxpoint
¾¾ Transfer must be in a scheme of amalgamation.
¾¾ Transferee company must be an Indian company.
Any transfer, in a scheme of amalgamation, of a capital asset being a share or shares held in an
Indian company, by the amalgamating foreign company to the amalgamated foreign company, if –
a. At least 25% of the shareholders of the amalgamating foreign company continue to remain
shareholders of the amalgamated foreign company; and
b. Such transfer does not attract tax on capital gains in the country, in which the amalgamating
47(via) company is incorporated
Taxpoint
¾¾ Such transfer is in a scheme of amalgamation by the amalgamating foreign company to the
amalgamated foreign company.

264 The Institute of Cost Accountants of India


Heads of Income

Section Transaction
¾¾ Transferred asset must be a capital asset being a share or shares held in an Indian company.
¾¾ At least 25% of the shareholders of the amalgamating foreign company continue to remain
shareholders of the amalgamated foreign company.
¾¾ Such transfer does not attract tax on capital gain in the country, in which the amalgamating
company is incorporated.
Any transfer of a capital asset by a banking company to a banking institution in a scheme of
47(viaa) amalgama­tion of such banking company with such banking institution sanc­tioned and brought into
force by the Central Government u/s 45(7) of the Banking Regulation Act, 1949.
Any transfer, in a scheme of amalgamation, of a capital asset, being a share of a foreign company,
(referred to in the Explanation 5 of sec.9(1)(i)), which derives, directly or indirectly, its value
substantially from the share or shares of an Indian company, held by the amalgamating foreign
company to the amalgamated foreign company, if:
47(viab)
a. at least 25% of the shareholders of the amalgamating foreign company continue to remain
shareholders of the amalgamated foreign company; and
b. such transfer does not attract tax on capital gains in the country in which the amalgamating
company is incorporated.

¾¾ Any transfer, in a relocation, of a capital asset by the original fund to the resulting fund;
¾¾ Any transfer by a shareholder or unit holder or interest holder, in a relocation, of a capital asset
being a share or unit or interest held by him in the original fund in consideration for the share
or unit or interest in the resultant fund.
Taxpoint
¾¾ Original fund means a fund established or incorporated or registered outside India, which
collects funds from its members for investing it for their benefit and fulfils the following
conditions:
a. the fund is not a person resident in India;
b. the fund is a resident of a country or a specified territory with which an agreement
referred to in sec. 90or 90A has been entered into; or is established or incorporated or
registered in a notified country or a specified territory;
c. the fund and its activities are subject to applicable investor protection regulations in the
47(viiac) &
country or specified territory where it is established or incorporated or is a resident; and
(viiad)
d. fulfils such other conditions as may be prescribed;
¾¾ Relocation means transfer of assets of the original fund, or of its wholly owned special purpose
vehicle, to a resultant fund on or before 31-03-2023, where consideration for such transfer is
discharged in the form of share or unit or interest in the resulting fund to,—
a. shareholder or unit holder or interest holder of the original fund, in the same proportion
in which the share or unit or interest was held by such shareholder or unit holder or
interest holder in such original fund, in lieu of their shares or units or interests in the
original fund; or
b. the original fund, in the same proportion as referred above, in respect of which the share
or unit or interest is not issued by resultant fund to its shareholder or unit holder or
interest holder;
¾¾ Resultant fund means a fund established or incorporated in India in the form of a trust or a
company or a limited liability partnership, which—

The Institute of Cost Accountants of India 265


Direct Taxation

Section Transaction
a. has been granted a certificate of registration as a Category I or Category II or Category III
Alternative Investment Fund, and is regulated under the Securities and Exchange Board
of India (Alternative Investment Fund) Regulations, 2012 made under the Securities and
Exchange Board of India Act, 1992 or International Financial Services Centre Authority
Act, 2019; and
b. is located in any International Financial Services Centre as referred to in sec. 80LA(1A)
Any transfer of capital asset by India Infrastructure Finance Company Ltd to an institution
47(viiae) established for financing the infrastructure and development, set up under an Act of Parliament and
notified by the Central Government.
Any transfer of capital asset, under a plan approved by the Central Government, by a public sector
47(viiaf) company to another notified public sector company or to the Central Government or to a State
Government
Any transfer, in a scheme of demerger, of capital asset by the demerged company to the resulting
47(vib)
company, if the resulting company is an Indian company.
Any transfer, in a scheme of demerger, of a capital asset, being a share or shares held in an Indian
company, by the demerged foreign company to the resulting foreign company, if –
a. The shareholders holding not less than three-fourths in value of the shares of the demerged
foreign company continue to remain shareholders of the resulting foreign company; and
b. Such transfer does not attract tax on capital gain in the country, in which the demerged foreign
company is incorporated:
47(vic) Taxpoint:
¾¾ Such transfer is in a scheme of demerger by the demerged foreign company to the resulting
foreign company.
¾¾ Transferred asset must be a capital asset being a share or shares held in an Indian company.
¾¾ Shareholders holding not less than 75% in value of the shares of the demerged foreign company
continue to remain shareholders of the resulting foreign company
¾¾ Such transfer does not attract tax on capital gains in the country, in which the demerged foreign
company is incorporated
any transfer in a business reorganisation, of a capital asset by the predecessor co-operative bank to
47(vica)
the successor co-operative bank or to the converted banking company
Any transfer by a shareholder, in a business reorganisation, of a capital asset being a share or
shares held by him in the predecessor co-operative bank if the transfer is made in consideration of
47(vicb)
the allotment to him of any share or shares in the successor co-operative bank or to the converted
banking company
Any transfer in a demerger, of a capital asset, being a share of a foreign company (referred to in
the Explanation 5 of sec. 9(1)(i)), which derives, directly or indirectly, its value substantially from
the share or shares of an Indian company, held by the demerged foreign company to the resulting
foreign company, if:
a. the shareholders, holding not less than 3/4th in value of the shares of the demerged foreign
47(vicc)
company, continue to remain shareholders of the resulting foreign company; and
b. such transfer does not attract tax on capital gains in the country in which the demerged foreign
company is incorporated.
Provided that the provisions of sections 391 to 394 of the Companies Act, 1956 shall not apply in
case of demergers referred above.

266 The Institute of Cost Accountants of India


Heads of Income

Section Transaction
Any transfer or issue of shares by the resulting company, in a scheme of demerger to the shareholders
47(vid) of the demerged company if the transfer or issue is made in consideration of demerger of the
undertaking.
Any transfer by a shareholder, in a scheme of amalgamation, of share(s) held by him in the
amalgamating company, if –
47(vii) a. The transfer is made in consideration of the allotment to him of any share or shares in the
amalgamated company except where the shareholder itself is the amalgamated company, and
b. The amalgamated company is an Indian company.
Any transfer of a capital asset, being foreign currency convertible bonds or Global Depository
Receipts referred to in sec. 115AC(1), made outside India by a non-resident to another non-resident.
Taxpoint:
47(viia) ¾¾ Transferred asset must be either ‘foreign currency convertible bonds’ or ‘Global Depository
Receipts’.
¾¾ Transfer has been made by a non-resident to another non-resident.
¾¾ Transfer has been made outside India.
Any transfer, made outside India, of a capital asset being rupee denominated bond of an Indian
company issued outside India, by a non-resident to another non-resident
47(viiaa) Taxpoint: In case of non-resident, any gains arising on account of appreciation of rupee against a
foreign currency at the time of redemption of rupee denominated bond of an Indian company held
by him, shall be ignored for the purposes of computation of full value of consideration under this
section
Any transfer of a capital asset, being—
bond or Global Depository Receipt referred to in sec. 115AC(1); or
rupee denominated bond of an Indian company; or
47(viiab) derivative, or
other notified securities
made by a non-resident on a recognised stock exchange located in any International Financial
Services Centre provided the consideration for such transaction is paid or payable in foreign
currency.
Any transfer of a capital asset, being a Government Security carrying a periodic payment of
47(viib) interest, made outside India through an intermediary dealing in settlement of securities, by a non-
resident to another non-resident
Any transfer of Sovereign Gold Bond issued by the RBI under the Sovereign Gold Bond Scheme,
47(viic)
2015, by way of redemption, by an assessee being an individual.
47(viid) Transformation of physical gold into e-gold receipts and vice-versa made as per SEBI Act 1992
Any transfer of a capital asset being a work of art, archaeological, scientific or art collection,
book, manuscript, drawing, painting, photograph or print, to the Government or a University or the
47(ix) National Museum, National Art Gallery, National Archives or any such other public museum or
institution as may be notified by the Central Government in the Official Gazette to be of national
importance or to be of renown throughout any State or States.
47(x) Any transfer by way of conversion of bonds or debentures, debenture-stock or deposit certificates
in any form of a company into shares or debentures of that company.
Any transfer by way of conversion of bonds referred to in sec. 115AC(1)(a) into shares or
47(xa)
debentures of any company

The Institute of Cost Accountants of India 267


Direct Taxation

Section Transaction
Any transfer by way of conversion of preference shares of a company into equity shares of that
47(xb)
company
Any transfer of a land of a sick industrial company, made under a scheme prepared and sanctioned
u/s 18 of the Sick Industrial Companies (Special Provisions) Act, 1985 where such sick industrial
company is being managed by its workers’ co-operative. Such transfer must have been made
47(xii)
during the period commencing from the previous year in which the said company has become a
sick industrial company and ending with the previous year during which the entire net worth of
such company becomes equal to or exceeds the accumulated losses.
Any transfer of a capital asset by a firm to a company as a result of succession of the firm by a
company in the business carried on by the firm subject to following conditions:
a. All assets and liabilities of the firm relating to the business immediately before the succession
become the assets and liabilities of the company.
b. All the partners of the firm immediately before the succession become the shareholders of the
company in the same proportion in which their capital accounts stood in the books of the firm
on the date of succession.
c. The partners of the firm do not receive any consideration or benefit, directly or indirectly, in
any form or manner, other than by way of allotment of shares in the company; and
d. The aggregate of the shareholding in the company of the partners of the firm is not less than
50% of the total voting power in the company and their shareholding continues to be as such
47(xiii) for a period of 5 years from the date of succession.
Taxpoint
¾¾ Transfer must have taken place as a result of succession of the firm to a company.
¾¾ All assets and liabilities related to the business must have been transferred.
¾¾ All the partners become the shareholders of the company in their capital ratio (as on the date
of the succession)
¾¾ The whole consideration shall be paid by allotment of shares in the company
¾¾ Partners (altogether) must hold atleast 50% of the total voting power of the company
¾¾ Lock in period for above share is 5 years from the date of succession.
Any transfer of a capital asset to a company in the course of demutualisation or corporatisation
of a recognized stock exchange in India as a result of which an association of persons or body of
individuals is succeeded by such company, subject to the following conditions –
a. All assets and liabilities of the AOP or BOI relating to the business immediately before the
succession become the assets and liabilities of the company;
b. The demutualisation or corporatisation of a recognized stock exchange in India is carried out
in accordance with a scheme for demutualisation or corporatisation which is approved by the
SEBI
Any transfer of a membership right of a recognized stock exchange in India for acquisition of shares
and trading or clearing rights in that recognized stock exchange in accordance with a scheme for
demutualisation or corporatisation which is approved by SEBI
47(xiiia)
Any transfer of -
a. a capital asset or intangible asset by a private company or unlisted public company (hereafter
referred to as the company) to a limited liability partnership (LLP); or

268 The Institute of Cost Accountants of India


Heads of Income

Section Transaction
b. a share(s) held in the company by a shareholder as a result of conversion of the company into
a limited liability partnership (LLP)
shall not regarded as a transfer, if following conditions are satisfied:
i. All the assets and liabilities of the company immediately before the conversion become the
assets and liabilities of the LLP;
ii. All the shareholders of the company immediately before the conversion become the partners
of the LLP and their capital contribution and profit sharing ratio in the LLP are in the same
proportion as their shareholding in the company on the date of conversion;
iii. The shareholders of the company do not receive any consideration or benefit other than by way
of share in profit and capital contribution in the LLP;
47(xiiib)
iv. The aggregate of the profit sharing ratio of the shareholders of the company in the LLP shall
not be less than 50% at any time during the period of 5 years from the date of conversion;
v. The total sales, turnover or gross receipts in business of the company in any of the 3 previous
years preceding the previous year in which the conversion takes place does not exceed ₹ 60 lakh;
vi. The total value of the assets as appearing in the books of account of the company in any of the 3
previous years preceding the previous year in which the conversion takes place does not exceed
₹ 5 crore; and
vii. No amount is paid (directly or indirectly) to any partner out of balance of accumulated profit
standing in the accounts of the company on the date of conversion for a period of 3 years from
the date of conversion.
Where a sole proprietary concern is succeeded by a company in the business carried on by it as
a result of which the sole proprietary concern sells or otherwise transfers any capital asset to the
company, subject to following conditions –
a. All assets and liabilities of the sole proprietary concern relating to the business immediately
before the succession become the assets and liabilities of the company;
b. Proprietor holds not less than 50% of the total voting power in the company and his shareholding
continues to remain as such for a period of 5 years from the date of succession; and
c. The sole proprietor does not receive any consideration or benefit, directly or indirectly, in any
form or manner, other than by way of allotment of shares in the company.
47(xiv)
Taxpoint
¾¾ Transfer must have taken place as a result of succession of the proprietorship concern to a
company.
¾¾ All assets and liabilities related to the business must have been transferred.
¾¾ The whole consideration shall be paid by allotment of shares in the company.
¾¾ Proprietor must hold at least 50% of the total voting power of the company.
¾¾ Lock in period for above share is 5 years from the date of succession.
Note: Sec. 47(xiii) & (xiv) exempts the capital gain on transfer of capital asset and not stock in
trade. Therefore, if stock is transferred at profits, it will be taxable as business income.

The Institute of Cost Accountants of India 269


Direct Taxation

Section Transaction
Any transfer in a scheme for lending of any securities under an agreement or arrangement, which
47(xv) the assessee has entered into with the borrower of such securities and which is subject to the
guidelines issued by the SEBI or the RBI.
Any transfer of a capital asset in a transaction of reverse mortgage under a scheme made and
47(xvi)
notified by the Central Government
Any transfer of a capital asset, being share of a special purpose vehicle (referred to in sec. 10(23FC))
47(xvii)
to a business trust in exchange of units allotted by that trust to the transferor.
Any transfer by a unit holder of a capital asset, being a unit or units, held by him in the consolidating
scheme of a mutual fund, made in consideration of the allotment to him of a capital asset, being a
unit or units, in the consolidated scheme of the mutual fund.
The exemption is available only the consolidation of two or more schemes of equity oriented fund
or of two or more schemes of a fund other than equity oriented fund.
“Consolidated scheme” means the scheme with which the consolidating scheme merges or which
is formed as a result of such merger.
47(xviii)
“Consolidating scheme” means the scheme of a mutual fund which merges under the process of
consolidation of the schemes of mutual fund in accordance with the Securities and Exchange Board
of India (Mutual Funds) Regulations, 1996 made under the Securities and Exchange Board of India
Act, 1992.
In the case of a capital asset, being a unit or units, which becomes the property of the assessee in
consideration of a transfer referred to in this clause, period of holding shall includes the period for
which the unit or units in the consolidating scheme of the mutual fund were held by the assessee.
Any transfer by a unit holder of a capital asset, being a unit or units, held by him in the consolidating
47(xix) plan of a mutual fund scheme, made in consideration of the allotment to him of a capital asset,
being a unit or units, in the consolidated plan of that scheme of the mutual fund.
2.4.7 Computation of Capital Gains [Sec. 48]
Short-term Capital Gain means the gain arising on transfer of short-term capital asset [Sec. 2(42B)].
Long-term Capital Gain means the gain arising on transfer of long-term capital asset [Sec. 2(29B)].

Computation of Short Term Capital Gain (STCG)


At a glance, computation of capital gain of _____ for the Assessment Year ……..

Particulars Details Amount


Sale consideration (Full value of consideration) ****
Less: Expenses on transfer ****
Net sale consideration ****
Less: i) Cost of acquisition ****
ii) Cost of improvement **** ****
Short Term Capital Gain ****
Less: Exemption u/s 54B, 54D, 54G, etc. (****)
Taxable Short Term Capital Gain *****

270 The Institute of Cost Accountants of India


Heads of Income

Note: No deduction shall be allowed in computing the income chargeable under the head “Capital gains” in respect
of any sum paid on account of securities transaction tax.
The meaning of terms used in the computation:
i. Sale consideration (full value of consideration)
It refers to sale value of the asset (in form of money or money’s worth).
Consideration in installments: In case, consideration is receivable in installment in different years, the entire
value of the consideration shall be taxable in the year of transfer.
Fair market value deemed to be full value of consideration in certain cases [Sec. 50D]: Where the
consideration received or accruing as a result of the transfer of a capital asset by an assessee is not ascertainable
or cannot be determined, then, for the purpose of computing income chargeable to tax as capital gains, the fair
market value* of the said asset on the date of transfer shall be deemed to be the full value of the consideration
received or accruing as a result of such transfer.
ii. Expenses on transfer
It means any expenditure incurred wholly and exclusively in connection with such transfer such as, brokerage
or commission incurred for securing buyer, cost of stamp and registration fee by the vendor, traveling expenses,
etc. It is reduced from sale consideration to get net sale consideration.
Notes
a. Expenditure must be necessary to affect the transaction and should not be vague
b. Expenditure on transfer may be incurred prior to or after completion of transfer
c. If expenditure has been allowed as deduction under any other heads of income then the same cannot be
claimed as deduction u/s 48.
iii. Cost of Acquisition [Sec. 55(2)]
Cost of acquisition includes expenditure incurred for acquiring the asset or completing the title of the asset.
For instance –
¾¾ Sum paid for discharge of mortgage debt to clear charge over the property (created by previous owner) is
a part of cost of acquisition.
¾¾ Litigation expenditure incurred by a shareholder to get the shares registered in his name will form part of
cost of acquisition of shares.
iv. Cost of Improvement [Sec. 55(1)(b)]
Cost of improvement means an expenditure incurred to increase the productive quality of the asset. It includes
all expenditures of a capital nature incurred in making any additions or alterations to the capital asset.
Notes
a. Any expenditure which is deductible in computing the income chargeable under any other head of
income shall not be treated as cost of improvement.
b. An expenditure incurred by a shareholder to file a suit to amend articles of association, which results in
appreciation of value of share shall be treated as cost of improvement.
Example 17:
Mr. Divesh had purchased a golden ring as on 17/8/2022 for ₹ 20,000. On 1/05/2023, he has sewn a diamond on

* Fair market value means the price that the capital asset would ordinarily fetch on sale in the open market on the relevant date and
where such price is not ascertainable, the price as may be determined in accordance with the rules made under this Act.

The Institute of Cost Accountants of India 271


Direct Taxation

it costing ₹ 25,000. On 1/08/2023, he sold such ring for ₹ 80,000 and incurred brokerage for arranging customer
₹ 5,000. Compute capital gain.
Solution :
Computation of capital gain of Mr. Divesh for the A.Y. 2024-25
Particulars Details Amount
Sale consideration 80,000
Less: Expenses on transfer 5,000
Net sale consideration 75,000
Less: i) Cost of acquisition 20,000
ii) Cost of improvement 25,000 45,000
Short Term Capital Gain 30,000

2.4.8 Computation of Long Term Capital Gain (LTCG)


At a glance, computation of capital gain of _____ for the Assessment Year ………….
Particulars Details Amount
Sale consideration (Full value of consideration) ****
Less: Expenses on transfer ****
Net sale consideration ****
Less: i) Indexed cost of acquisition ****
ii) Indexed cost of improvement **** ****
Long Term Capital Gain ****
Less: Exemption u/s 54, 54B, 54D, 54EC, 54F, etc. ****
Taxable Long Term Capital Gain ****
Note: No deduction shall be allowed in computing the income chargeable under the head “Capital gains” in respect
of any sum paid on account of securities transaction tax.
The meaning of terms used in the computation:
i. Indexed cost of acquisition
“Indexed cost of acquisition” means the ‘cost of acquisition’ (as discussed in case of short term capital
gain) adjusted according to the price level of the year of sale. As per explanation to sec.48, “Indexed cost of
acquisition” is an amount which bears to the ‘cost of acquisition’ the same proportion as Cost Inflation Index
for the year in which the asset is transferred bears to the Cost Inflation Index for the first year in which the asset
was held by the assessee or for the year beginning on 1/4/2001, whichever is later.

Index of the year of transfer


Taxpoint: Indexed cost of acquisition = Cost of acquisition ×
Index of the year of acquisition
ii. Indexed cost of improvement
“Indexed cost of improvement” means the ‘cost of improvement’ (as discussed in case of short term capital
gain) adjusted according to the price level of year of sale. As per explanation to sec. 48, “indexed cost of any
improvement” is an amount, which bears to the cost of improvement the same proportion as Cost Inflation

272 The Institute of Cost Accountants of India


Heads of Income

Index for the year in which the asset is transferred bears to the Cost Inflation Index for the year in which the
improvement to the asset took place

Index of the year of transfer


Indexed cost of improvement = Cost of improvement ×
Index of the year of improvement
iii. Cost Inflation Index
Cost inflation index, in relation to a previous year, means such Index as the Central Government may, having
regard to 75% of average rise in the Consumer Price Index (urban) for the immediately preceding previous
year to such previous year, by notification in the Official Gazette, specify, in this behalf. Cost Inflation Index
for different financial years is as follows:

Financial Year Index Financial Year Index Financial Year Index


100 2011-12 184 2021-22 317
2002-03 105 2012-13 200 2022-23 331
2003-04 109 2013-14 220 2023-24 348
2004-05 113 2014-15 240
2005-06 117 2015-16 254
2006-07 122 2016-17 264
2007-08 129 2017-18 272
2008-09 137 2018-19 280
2009-10 148 2019-20 289
2010-11 167 2020-21 301
Note: Indexed cost of acquisition has to be ascertained with reference to the date of acquisition and not with
reference to the date when such asset became a capital asset.

Treatment of assets acquired before 1/4/2001


Cost of acquisition If an asset is acquired before 1/4/2001 then its cost of acquisition shall be higher of the
following:
a. Actual cost of acquisition (ignoring cost of improvement incurred before 1/4/2001);
or
b. Fair market value* of the asset as on 1/4/2001 [Sec. 55]
Note: In case of a capital asset, being land or building or both, the fair market value of
such asset on 01-04-2001 shall not exceed the stamp duty value, wherever available, of
such asset as on 01-04-2001.
Exception: The option is not available in case of –
¾¾ Asset on which depreciation is allowed u/s 32(1)(ii);
¾¾ Self generated assets (other than bonus share)
Cost of improvement Any cost of improvement incurred by the assessee or the previous owner before 1/4/2001
shall not be considered.
Indexation Where an asset is acquired before 1/4/2001, then indexation benefit shall be available
from the year 2001-02.

The Institute of Cost Accountants of India 273


Direct Taxation

Illustration 83
On 23rd December, 2023, Rajat sold 500 grams of gold, the sale consideration of which was ₹ 14,00,000. He had
acquired this gold on 20th August, 2000 for ₹ 4,00,000. Fair market value of 500 grams of gold on 1st April, 2001
was ₹ 3,60,000. Find out the amount of capital gain chargeable to tax for the assessment year 2024-25.
Solution :
Computation of capital gains of Rajat for the A.Y. 2024-25

Particulars Working Amount


Sale consideration 14,00,000
Less: Expenses on transfer Nil
Net Sale consideration 14,00,000
Less: Indexed cost of acquisition ₹ 4,00,0001 x 348/100 13,92,000
Long term capital gain 8,000
1.
If an asset is acquired before 1/4/2001 then its cost of acquisition will be higher of the following:
a) Actual cost of acquisition; or b) Fair market value of the asset as on 1/4/2001

Illustration 84:
Mr. Anand has purchased a house property as on 17/08/2002 for ₹ 5,00,000. On 1/05/2004, he constructed a new
floor on the same house at a cost of ₹ 2,50,000. On 1/10/2023, he sold such house for ₹ 18,00,000 and incurred
brokerage @ 2% for arranging customer. Compute capital gain.
Solution :
Computation of capital gain of Mr. Anand for the A.Y. 2024-25

Particulars Working Details Amount


Sale consideration 18,00,000
Less: Expenses on transfer 2% of ₹ 18,00,000 36,000
Net sale consideration 17,64,000
Less: i) Indexed cost of acquisition ₹ 5,00,000 x (348/105) 16,57,143
ii) Indexed cost of improvement ₹ 2,50,000 x (348/113) 7,69,912 24,27,055
Long Term Capital Gain (6,63,055)

2.4.9 Deemed or Notional Cost of Acquisition [Sec. 49(1)]


A. In certain cases (stated below) cost of acquisition of the previous owner1 of the property shall be deemed to be
the cost of acquisition for the assessee:
a. Assets received on total or partial partition of HUF [Sec. 49(1)(i)].
b. Assets received under a gift or will [Sec. 49(1)(ii)].
c. Assets received by succession, inheritance or devolution [Sec. 49(1)(iii)(a)].
d. Assets received on dissolution of a firm, BOI or AOP [Sec. 49(1)(iii)(b)].
e. Assets received on liquidation of a company [Sec. 49(1)(iii)(c)].
f. Assets received under a trust (whether revocable or irrevocable) [Sec. 49(1)(iii)(d)].
g. Assets received under business reorganization subject to certain conditions [Sec. 49(1)(iii)(e)] e.g.,

274 The Institute of Cost Accountants of India


Heads of Income

assets received by a 100% Indian subsidiary company from its holding company or received by an
Indian holding company from its 100% subsidiary company or received by an amalgamated Indian
company from the amalgamating company, asset received on conversion of a company into LLP, etc.
h. Asset (being a self-acquired property of a member) received by an HUF from its member [Sec. 49(1)
(iv)].
B. In certain cases (stated below) cost of acquisition of the capital asset shall be:
a. Where capital asset, being equity share of a company, became the property of the assessee on conversion
of preference shares into equity shares of that company, the cost of acquisition of corresponding
preference shares shall be considered as cost of acquisition of such converted equity shares [Sec.
49(2AE)]. Similarly, cost of units in the consolidating plan of the scheme of the mutual fund shall be
deemed to be the cost of units in a consolidated plan of a mutual fund scheme.
b. Assets covered u/s 56(2)(x) [Sec. 49(4)] [Refer chapter ‘Income from Other Sources’]
c. Conversion of inventory into capital assets [Discussed later on]
d. Where the capital asset, being share or shares of a company, is acqired by a non-resident assessee on
redemption of Global Depository Receipts referred to in sec. 115AC(1)(b) held by such assessee, the
cost of acqisition of the share or shares shall be the price of such share or shares prevailing on any
recognised stock exchange on the date on which a request for such redemption was made.
 In the case of a capital asset, being share or shares of a company, which is acquired by the non-
resident assessee on redemption of Global Depository Receipts referred to in sec. 115AC(1)(b) held
by such assessee, the holding period shall be reckoned from the date on which a request for such
redemption was made
e. In case of capital asset being declared under the Income Declaration Scheme, 2016, and the tax, surcharge
and penalty have been paid as per the Scheme on the fair market value of the asset as on the date of
commencement of the Scheme (01-06-2016), the cost of acquisition of the asset shall be deemed to be
the fair market value of the asset which has been taken into account for the purposes of the said Scheme*.
f. Where the capital asset, being a unit of a business trust, became the property of the assessee in
consideration of a transfer as referred to in sec. 47(xvii), the cost of acquisition of the asset shall be
deemed to be the cost of acquisition to him of the share referred to in sec. 47(xvii)
 “Business trust” means a trust registered as an Infrastructure Investment Trust or a Real Estate
Investment Trust, the units of which are required to be listed on a recognised stock exchange, in
accordance with the regulations made under the Securities Exchange Board of India Act, 1992 and
notified by the Central Government in this behalf [Sec. 2(13A)]
g. The cost of acquisition of a unit in the segregated portfolio shall be the amount which bears, to the cost
of acquisition of a unit held by the assessee in the total portfolio, the same proportion as the net asset
value of the asset transferred to the segregated portfolio bears to the net asset value of the total portfolio
immediately before the segregation of portfolios. The cost of the acquisition of the original units held by
the unit holder in the main portfolio shall be deemed to have been reduced by the amount as so arrived
Taxpoint
¾¾ While computing period of holding in case of capital asset, being a unit in a segregated portfolio, the
period, for which the original unit or units in the main portfolio were held by the assessee, shall be included.
¾¾ SEBI has, vide circular SEBI/HO/IMD/DF2/CIR/P/2018/160 dated December 28, 2018, permitted creation
of segregated portfolio of debt and money market instruments by Mutual Fund schemes. As per the SEBI
* In case of an immovable property declared under the scheme, the period for which such property is held shall be reckoned from the date on
which such property is acquired if the date of acquisition is evidenced by a deed registered with any authority of a State Government. If it
is not evidenced then from 01-06-2016.

The Institute of Cost Accountants of India 275


Direct Taxation

circular, all the existing unit holders in the affected scheme as on the day of the credit event shall be allotted
equal number of units in the segregated portfolio as held in the main portfolio. On segregation, the unit
holders come to hold same number of units in two schemes –the main scheme and segregated scheme.
Various aspects of Sec.49(1) discussed below:
Where cost of acquisition of
Cost of acquisition of previous owner.
Cost of previous owner is ascertainable.
acquisition of an Where cost of acquisition The fair market value of the asset on the date on which the
asset of previous owner is previous owner had acquired the same shall be deemed to
unascertainable be the cost of acquisition.
Where a capital asset became the property of the assessee by any of the modes specified in
Improvement
sec. 49(1), cost of improvement includes improvement expenditure incurred by the previous
expenditure
owner.
Period of holding of the previous owner shall be considered for classifying the asset into short
Period of holding
term or long-term capital asset.
Indexation
benefit on cost of Indexation benefit shall be available from the year acquisition.
acquisition
Index of the year of transfer
Taxpoint: Indexed cost of acquisition = Cost of acquisition ×
Index of the year of acquisition
Indexation
benefit on cost of Indexation benefit shall be available from the year of acquisition*.
improvement
Index of the year of transfer
Taxpoint: Indexed cost of improvement = Cost of improvement ×
Index of the year of improvement
Previous owner means the last owner who acquired such asset through a mode other than the modes mentioned
above.
Illustration 85:
Mrs. Parminder has jewellery, being gifted on 1/04/2005 by her brother Jitendar. Jitendar acquired such asset
for ₹ 60,000 as on 1/07/1995. On 1/07/2002, Jitendar has sewn a diamond worth ₹ 25,000 in such jewellery. On
1/04/2009, Mrs. Parminder incurred polish expenditure on such jewellery costing ₹ 5,000. As on 1/04/2023, Mrs.
Parminder sold such jewellery for ₹ 12,00,000. Brokerage @ 1% of sale value was paid by her. The fair market
value of the jewellery as on –
1/04/2001 is ₹ 2,00,000; 1/04/2005 is ₹ 5,00,000; and 1/04/2023 is ₹ 7,50,000.
Compute capital gain in hands of Mrs. Parminder for the A.Y. 2024-25.
Solution :
Computation of capital gain in the hands of Mrs. Parminder for the A.Y. 2024-25
Particulars Working Details Amount
Sale consideration 12,00,000
Less: Expenses on transfer 1% of ₹ 12,00,000 12,000
Net sale consideration 11,88,000

* In case of CIT –vs.- Manjula J Saha 16 Taxmann 42 (Bombay), the Hon’ble Bombay High Court has held that index benefit is available from the year
in which asset was acquired by the previous owner or for the year beginning on the 1st day of April, 2001, whichever is later.

276 The Institute of Cost Accountants of India


Heads of Income

Less: i) Indexed cost of acquisition ₹ 2,00,000 * 348/117 6,96,000


ii) Indexed cost of improvement ₹ 25,000 * 348/105 82,857 7,78,854
Long Term Capital Gain 40,914
Notes:
1. Fair market value as on 1/04/2005 and 1/04/2022 are irrelevant.
2. Cost of acquisition shall be taken as cost of acquisition in the hands of previous owner or fair market value as
on 1/04/2001, whichever is higher. However, indexation benefit shall be available from the year in which the
assessee acquired the property.
3. Benefit of indexation in case of cost of improvement shall be available when the actual improvement
expenditure was incurred.
4. Polish expenditure on jewellery does not amount to improvement as such expenditure is not a capital
expenditure and does not increase value of the asset.

In nutshell
1. If an asset is acquired before 1/04/2001 then its cost of acquisition will be higher of a) Actual cost of
acquisition or b) Fair market value of the asset as on 1/04/2001.
2. Any cost of improvement before 1/04/2001 shall not be considered.
3. Index benefit shall be available from the year when the improvement expenditure incurred whether by
current owner or by the previous owner.
4. In order to find out whether an asset is short term or long term capital asset, the period of holding of previous
owner shall be considered

Cases where indexation benefit is not available even on transfer of long term capital asset
 Debenture or Bonds: In case of transfer of bonds and debentures other than capital indexed bonds issued by
the Government or Sovereign Gold Bond issued by the RBI under the Sovereign Gold Bond Scheme, 2015
[Sec. 48].
 Slump Sale: Transfer of an undertaking or division in a slump sale [Sec. 50B]. (Discussed later in this chapter)
 Virtual Digital Asset: Transfer of crypto currency [Sec. 115BBH] (Discussed later in this chapter)
 Equity shares and equity oriented fund referred to in sec. 112A (Discussed later in this chapter)
 Certain transactions by a non-resident: In case of a non-resident, capital gain arising on the transfer of
shares in or debentures of an Indian company acquired in foreign currency. It will be computed as per First
Proviso to sec. 48 (discussed later in this chapter). Further, in case of transaction u/s 115AB, 115AC, etc. index
benefit is not available.
 Transfer of Global Depository Receipt: Transfer of Global Depository Receipt purchased in foreign
currency by a resident individual being employee of an Indian Company [Sec. 115ACA]

Illustration 86:
Miss Isha has 1,000 10% Debentures of X Ltd. acquired on 17/04/2009 for ₹ 120 each. As on 1/02/2024, she sold
such asset for ₹ 1,45,000. Brokerage @ ½ % of sale value was paid by her. Compute capital gain.
Solution :
Computation of capital gain in the hands of Miss Isha for the A.Y. 2024-25

The Institute of Cost Accountants of India 277


Direct Taxation

Particulars Working Details Amount


Sale consideration 1,45,000
Less: Expenses on transfer ½ % of ₹ 1,45,000 725
Net sale consideration 1,44,275
Less: i) Cost of acquisition 1,000 Debentures @ ₹ 120 each 1,20,000
ii) Cost of improvement Nil 1,20,000
Long Term Capital Gain 24,275
Note: Though the asset sold is long term capital asset still indexation shall not be applicable as indexation benefit
is not available on transfer of debentures.

2.4.10 Notional sale consideration


In some cases, notional sale value is considered rather than actual sale value e.g.

Section Case Deemed value of consideration


Compensation received in cash or fair market
Insurance claim received on damage or
45(1A) value (as on date of receipt) of assets received as
destruction of capital asset
compensation.
Conversion of capital assets into stock in Fair market value of the asset as on date of such
45(2)
trade conversion.
Transfer of capital assets by a partner/
The amount recorded in books of account of the
45(3) member to firm/AOP/BOI as capital
firm/AOP/BOI, as value of such assets.
contribution
Money so received or the market value of other
Assets received by shareholders on
46(2) assets on the date of distribution, as reduced by the
liquidation
amount assessed as dividend u/s 2(22)(c)
Shares debentures or warrants transferred
Fifth proviso
by the employee by way of gift or under Market value of the asset as on date of such transfer
to sec. 48
an irrevocable trust.
Value determined for payment of stamp duty (if
50C Transfer in case of land or building or both
consideration declared by the assessee is less)
Detail study shall be made in respective sections, later in this chapter. Further, aforesaid list is not exhaustive.

2.4.11 Treatment of Advance money received and forfeited [Sec. 51]


W.e.f. A.Y. 2015-16*, any sum of money received as an advance or otherwise in the course of negotiations for
transfer of a capital asset shall be taxable as ‘Income from Other Sources’, if,—
(a) such sum is forfeited; and
(b) the negotiations do not result in transfer of such capital asset [Sec. 56(2)(ix)]

* Where any capital asset has previously undergone negotiation for its transfer, any advance or other money received and retained by the
assessee (not by the previous owner) on or before 31-03-2014 in respect of such negotiation shall be deducted from the cost for which the
asset was acquired or the WDV or the fair market value, as the case may be.

278 The Institute of Cost Accountants of India


Heads of Income

Taxpoint: Where any sum of money, received as an advance or otherwise in the course of negotiations for transfer
of a capital asset, has been included in the total income of the assessee for any previous year as per sec. 56(2)(ix),
then, such sum shall not be deducted from the cost for which the asset was acquired or the written down value or
the fair market value, as the case may be, in computing the cost of acquisition.

2.4.12 Treatment of compensation paid by the transferor


If transferor commits default and transferee receives any compensation (apart from refund of advance money), then
compensation received by the transferee shall be treated as sale consideration against transfer of (relinquishment
of) purchasing right and charged to capital gain.
Taxpoint: Treatment of compensation paid by the transferor shall be as under –

Nature of asset transferred Right to purchase


Nature of transfer Relinquishment
Cost of acquisition Nil
Sale consideration Amount of compensation

Computation of capital gain in certain cases

2.4.13 Capital gain in case of insurance claim [Sec. 45(1A)]


As per the Supreme Court in CIT vs Vania Silk Mills (P) Ltd., any compensation received as insurance claim for
loss of capital assets shall not be taxable. However, w.e.f. A.Y. 2000-2001, sec. 45(1A) has been inserted, which
nullify the impact of the above case law (to some extent).
As per provision of this section, any compensation received from an insurance company for the specified damages#
is treated as transfer. Such transfers are liable to capital gain in the year of receipt of compensation.
# Specified damages
Damages caused due to –
 Flood, typhoon, hurricane, cyclone, earthquake or other natural calamities; or
 Riot or civil disturbance; or
 Accidental fire or explosion; or
 Action by an enemy and an action taken in combating an enemy (whether with or without a declaration of war)

The Institute of Cost Accountants of India 279


Direct Taxation

For computation of capital gain, various terms to be interpreted as under:

If compensation
Compensation so received
received is in cash
Sale consideration
If compensation Fair Market value (as on date of receipt) of
received is in kind assets received as compensation.
Cost of acquisition / Cost of improvement
As usual
/ Expense on transfer
Indexation benefit available (if any) Till year of destruction
Taxable In the year of receipt of compensation
Tax rate As applicable in the year of receipt of compensation
Note: Compensation received for damage of machinery shall be treated u/s 41(1) to the extent expenditure incurred
on repair and renovation of damaged machinery and such repair and renovation expenditure shall be allowed u/s 31.
However, compensation in excess of repair & renovation expenditure shall be treated as capital receipt and shall not
be liable to tax. Sec. 45(1A) shall not be applicable as there is no transfer of capital asset.
Compensation received for any other damages to capital asset: Compensation received for any other damages
of capital asset (e.g. loss of capital assets due to theft, road accident or sunk of ship due to technical error, etc.) shall
be treated as a capital receipt and shall not be taxable.
Compensation received for any damages to non-capital asset: A revenue receipt may be chargeable u/s 28 or 56.
E.g. compensation received on theft of stock in trade shall be taxable as business income.
Destruction of asset without insurance: Where an asset is destroyed and there is no insurance or insurance
compensation is not received, neither sec. 45(1A) nor sec. 45 shall be attracted and destruction of asset shall not be
treated as transfer. Cost of such asset destroyed is not an allowable loss under the Income Tax Act.
Example 2: Mr. X acquired a residential building for self occupation on 5/5/2020 for ₹ 90 lacs. On 7/4/2023, the
property caught fire. No insurance was made. Such loss of ₹ 90 lacs is not an allowable loss under Income tax Act.

Illustration 87:
Lucky has a house property acquired on 18/08/2009 for ₹ 6,00,000. He used the house for his own residential
purpose. On 18/08/2012 he incurred capital expenditure on re-construction of house ₹ 3,00,000. On 15/05/2023, he
brought office goods (inflammable) worth ₹ 1,00,000 at home to be delivered to a party staying near to his home. At
the night of that day accidental fire took place and damaged the whole house property, furniture worth ₹ 5,00,000
and business stock.
Insurance claim received on 18/08/2023 –
1. for the house ₹ 1,00,000 in cash & a new house allotted to him (fair market value of which is ₹ 44,00,000 on
18/08/2023);
2. for house-hold furniture ₹ 2,00,000; and
3. for stock ₹ 80,000.
State –
–– Tax-treatment under the head Capital gains.
–– How shall your answer differ if such compensation is received by the assessee on 15/04/2024.

280 The Institute of Cost Accountants of India


Heads of Income

Solution :
As the damage occurred due to accidental fire, such case is governed by the provision of sec. 45(1A)
Computation of capital gain in the hands of Lucky for the A.Y. 2024-25

Particulars Workings Details Amount


Sale consideration of house ₹ 1,00,000 + ₹ 44,00,000 45,00,000
Less: Expenses on transfer Nil
Net sale consideration 45,00,000
Less: i) Indexed cost of acquisition ₹ 6,00,000 * 348/148 14,10,810
ii) Indexed cost of improvement ₹ 3,00,000 * 348/200 5,22,000 19,32,810
Long Term Capital Gain 25,67,190
For Furniture: No capital gain liability arises as furniture is a personal asset of the assessee and hence not a
capital asset. Compensation received on loss of furniture shall be treated as capital receipt and hence not liable
to tax.
For Stock: Compensation received on loss of stock shall be liable to tax u/s 28. In the given case, loss of ₹ 20,000
(₹ 1,00,000 – ₹ 80,000) shall be allowed under the head “Profits & gains of business or profession”
In case such compensation is received on 15/04/2024 then the capital gain of ₹ 25,67,190 as computed above shall
be taxable in the Assessment year 2025-26.

2.4.14 Capital gain on ULIP [Sec. 45(1B)]


Situation
Any person receives at any time during any previous year any amount under a unit linked insurance policy, to
which exemption u/s 10(10D) does not apply on account of the applicability of the fourth and fifth provisos thereof
(i.e. excess premium paid then prescribed limit), including the amount allocated by way of bonus on such policy
Treatment
Any profits or gains arising from receipt of such amount by such person shall be chargeable to income-tax under
the head “Capital gains” and shall be deemed to be the income of such person of the previous year in which such
amount was received and the income taxable shall be calculated in such manner as may be prescribed.

2.4.15 Capital gain on conversion of capital assets into stock in trade [Sec. 45(2)]
From A.Y. 1985-86 onwards, conversion of capital assets into stock in trade shall be treated as transfer u/s 2(47).
For computation of capital gain, various terms to be interpreted are as under:

Sale consideration Fair Market value as on date of such conversion.


Cost of acquisition/Cost of improvement/
As usual
Expenditure on transfer
Indexation benefit available (if any) Till year of conversion
Taxable In the year in which such stock is actually sold
Tax rate As applicable in the year of actual sale of such stock
[Actual sale value – FMV as on date of conversion –
Treatment of difference of actual sale value and Fair
Expenditure on transfer] shall be treated as business
market value as on date of conversion
Income.

The Institute of Cost Accountants of India 281


Direct Taxation

Illustration 88:
Ali has 10,000 shares of X (P) Ltd. acquired on 15/05/1981 for ₹ 12 each. On 15/07/1983 he converted 6,000 of
such shares into stock in trade. On that date, market value of such share was ₹ 15 each. On 1/05/2007, he further
converted 2,000 of such shares into stock in trade. On such date, market value of the share was ₹ 30 each. On
17/02/2024, he sold all shares for ₹ 225 each. Brokerage incurred 2%. State tax treatment. Fair market value of such
shares as on 01-04-2001 was ₹ 16/-
Solution :
In the given case, shares held by Ali can be divided into three categories –
Category A: 6,000 shares being converted into stock in trade as on 15/07/1983: Sec. 45(2) is applicable on the
conversion effected on or after 1/04/1984. Hence, on conversion of 6,000 shares, sec. 45(2) shall not be applicable.
Hence, on transfer of such asset no capital gain arises.
Category B: 2,000 shares being converted into stock in trade as on 1/05/2007: As per sec. 45(2), such conversion
shall be deemed to be transfer u/s 2(47) and shall be liable to tax in the year of actual sale. For this purpose, fair
market value as on date of conversion (₹ 30 each) shall be treated as sale consideration and fair market value on 01-
04-2001 shall be considered as cost of acquisition. The difference between actual sale value (₹ 225) and fair market
value as on date of conversion (₹ 30 each) shall be treated as business income.
Category C: 2,000 shares (remaining) held as investments: Transfer of such share shall be liable to capital gain.
Computation of capital gain in the hands of Ali for the A.Y. 2024-25

Category B Category C
Particulars
Details Amount Details Amount
Sale consideration 2,000 * ₹ 30 60,000 2,000 * ₹ 225 4,50,000
Less: Expenses on transfer Nil 2% of ₹ 4,50,000 9,000
Net Sale Consideration 60,000 4,41,000
Less: i) Indexed cost of acquisition 2,000*16*129/100 41,280 2000*16*348/100 1,11,360
ii) Indexed cost of improvement Nil Nil
Long Term Capital Gain 18,720 3,29,640
Computation of Profits & gains of business or profession in the hands of Ali for the A.Y. 2024-25

Category A Category B
Particulars
Details Amount Details Amount
Sale consideration 6,000 * ₹ 225 13,50,000 2,000 * ₹ 225 4,50,000
Less: Expenses on transfer 2% of ₹ 13,50,000 27,000 2% of ₹ 4,50,000 9,000
Net Sale Consideration 13,23,000 4,41,000
Less: Cost of goods sold ₹ 151 * 6,000 90,000 ₹ 302 * 2,000 60,000
Profits & gains of business or profession 12,33,000 3,81,000
1. Provision of sec. 45(2) shall be applicable from A.Y. 1985-86, since such shares are converted before A.Y.
1985-86, hence the difference between cost price (₹ 12) and market price (₹ 15) on the date of such conversion
shall neither be liable to capital gain nor business income. Further cost of goods sold shall be taken as fair
market value as on the date of such conversion i.e. ₹ 15.
2. Fair market value of share on conversion of such asset into stock in trade.

282 The Institute of Cost Accountants of India


Heads of Income

2.4.16 Computation of Capital gain in case of depreciable assets [Sec. 50]


The capital asset which forms a part of a block of assets* in respect of which depreciation
has been allowed u/s 32(1)(ii) as per WDV method.
E.g. a residential house property being not used for business purpose, on which no
Meaning
depreciation is allowed under Income tax Act shall be treated as a non-depreciable asset.
Whereas if the same house is used for the residence of employee of the business and
depreciation has been claimed under I.T. Act then it shall be treated as depreciable asset.
Nature of Capital Capital gain arising on transfer of depreciable asset shall always be a short-term capital
gain gain.
Benefit of indexation Indexation benefit cannot be claimed on such asset.
For computation of capital gain on transfer of such asset, refer Depreciation (u/s 32) of the
Computation
chapter “Profits & gains of business or profession”
Note: Depreciable asset itself may be a long term capital asset or short term capital asset depending upon
the period of holding (whether held for more than 36 months or not), however gain on transfer of aforesaid
depreciable asset shall always be short term capital gain.

2.4.17 Transfer of security by depository [Sec. 45(2A)]


An assessee may have security in form of a physical script or in dematerialized form. The securities held in
dematerialized form shall be dealt as per sec. 45(2A) and for computation of capital gain, various terms to be
interpreted are as under:
Cost of acquisition Cost of acquisition and the period of holding of any securities shall be determined
Period of holding on the basis of the first-in-first-out method (FIFO)#.
Sale consideration
Benefit of indexation As usual
Expenditure on transfer
#
In this connection, CBDT vide Circular No. 768, dated 24/6/1998 has clarified that:
 In case of assessee who has securities partly in physical form and partly in dematerialized form, FIFO method
will be applied only in respect of the dematerialized holding. This is because in case of sale of dematerialized
securities, the securities held in physical form are not considered as they continue to remain in the possession
of the investor and are identifiable separately.
 An assessee may have more than one account in case of depository system and in such case FIFO technique
shall be applied for each account separately.
 For deciding FIFO technique the date of entry in ‘De-Mat Account’ is significant and the date of purchase of
security is irrelevant.

2.4.18 Capital gain on transfer of capital assets by a partner/member to Firm/AOP/BOI as


capital contribution [Sec. 45(3)]
Where a partner or member (whether existing or new) transfers any of its capital assets to firm, AOP or BOI of
which he is a member by way of capital contribution, the same shall be treated as transfer. Tax treatment of such
transfer shall be as under:

* Block of asset does not include goodwill of business or profession

The Institute of Cost Accountants of India 283


Direct Taxation

The amount recorded in books of account of the firm/AOP/


Sale consideration
BOI, as value of such assets.
Cost of acquisition/Cost of improvement/
As usual
Expenditure on transfer
Taxable In the year of such transfer
Indexation benefit As usual
Note: Fair market value of such asset is irrelevant to decide sale consideration.

Illustration 89:
ABC & Co. has three partners A, B and C sharing profit or loss in the ratio 5:3:2. They admitted D as a new
partner on 31/03/2023 for 1/5th share and D is to bring ₹ 2,00,000 as his capital which he brought in form of
furniture (earlier used in his home) ₹ 50,000 immediately & further brought jewellery of which fair market value
is ₹ 2,00,000 on 2/04/2023 (however such assets was recorded in the books at ₹ 1,60,000). D had acquired such
jewellery for ₹ 45,000 on 7/07/2001. Compute capital gain in the hands of Mr. D.
Solution :
In the given case, Mr. D brought two assets:
a. Furniture, being a personal effect, is not a capital asset and hence, not liable to capital gain.
b. Jewellery being a capital asset, hence liable to capital gain as under –
Computation of capital gain in the hands of Mr. D for the A.Y. 2024-25

Particulars Working Details Amount


Sale consideration 1,60,000
Less: Expenses on transfer Nil
Net sale consideration 1,60,000
Less: i) Indexed cost of acquisition ₹ 45,000 * 348/100 1,56,600
ii) Indexed cost of improvement Nil 1,56,600
Long Term Capital Gain 3,400
Sale consideration shall be the amount recorded in books of account of the firm and market value of such asset
is irrelevant.

2.4.19 Capital gain on transfer of capital assets by a firm/AOP/BOI to partner/member by


way of distribution on its dissolution [Sec. 45(4) r.w.s 9B]
Situation
Where a specified person receives during the previous year any money or capital asset or both from a specified
entity in connection with the reconstitution of such specified entity,
 “Specified entity” means a firm or other association of persons or body of individuals (not being a company
or a co-operative society);
 “Specified person” means a person, who is a partner of a firm or member of other association of persons or
body of individuals (not being a company or a co-operative society) in any previous year.
 “Reconstitution of the specified entity” means, where—

284 The Institute of Cost Accountants of India


Heads of Income

c. one or more of its partners or members, as the case may be, of such specified entity ceases to be partners
or members; or
d. one or more new partners or members, as the case may be, are admitted in such specified entity in such
circumstances that one or more of the persons who were partners or members, as the case may be, of
the specified entity, before the change, continue as partner or partners or member or members after the
change; or
e. all the partners or members, as the case may be, of such specified entity continue with a change in their
respective share or in the shares of some of them;
Treatment
In the hands of specified entity u/s 9B
Any profits and gains arising from such deemed transfer of capital asset or stock in trade or both, as the case may
be, by the specified entity shall be:
 deemed to be the income of such specified entity of the previous year in which such capital asset or stock in
trade or both were received by the specified person; and
 chargeable to income-tax as income of such specified entity under the head “Profits and gains of business or
profession” or under the head “Capital gains”
 for aforesaid purposes, fair market value of the capital asset or stock in trade or both on the date of its receipt
by the specified person shall be deemed to be the full value of the consideration received or accruing as a result
of such deemed transfer of the capital asset or stock in trade or both by the specified entity.
In the hands of specified entity u/s 45(4)
Any profits or gains arising from such receipt by the specified person shall be chargeable as income of such
specified entity under the head “Capital gains”.
 It shall be deemed to be the income of such specified entity of the previous year in which such money or capital
asset or both were received by the specified person.
 Transfer of stock in trade shall be treated as business profit
Method of computation of profits or gains
Such profits or gains shall be determined in accordance with the following formula:
A=B+C-D
where,
A = Income chargeable as capital gains of the specified entity
 If the value of “A” is negative, its value shall be deemed to be zero.
B = Value of any money received by the specified person from the specified entity on the date of such receipt;
C = Fair market value of the capital asset received by the specified person from the specified entity on the
date of such receipt.
D = Balance in the capital account (represented in any manner) of the specified person in the books of account
of the specified entity at the time of its reconstitution.
 The balance is to be calculated without taking into account the increase in the capital account of the
specified person due to revaluation of any asset or due to self-generated goodwill or any other self-
generated asset.
 “Self-generated goodwill” and “Self-generated asset” mean goodwill or asset, as the case may be,
which has been acquired without incurring any cost for purchase or which has been generated during
the course of the business or profession.

The Institute of Cost Accountants of India 285


Direct Taxation

Taxpoint
 When a capital asset is received by a specified person from a specified entity in connection with the reconstitution
of such specified entity, this provision shall operate in addition to the provisions of sec. 9B and the taxation
under the said provisions thereof shall be worked out independently.

2.4.20 Capital gain on transfer by way of compulsory acquisition [Sec. 45(5)]


Sec. 45(5) is applicable on the following cases –
 When the transfer of a capital asset (other than urban agro-land) is by way of compulsory acquisition under
any law; or
 When a capital asset is transferred and the consideration of transfer is to be determined or approved by the
Central Government (not by a State Government) or the Reserve Bank of India.
Tax treatment of initial compensation received [Sec. 45(5)(a)]
Sale consideration Total compensation or consideration received or receivable
Cost of acquisition / Cost of
As usual
improvement
Expenditure on transfer Legal expenditure incurred to recover claim shall be allowed as deduction.
In the year when such compensation / consideration (or part thereof) is first
Taxable
received.
Indexation benefit available Till the year of compulsory acquisition.
Tax treatment of enhanced compensation [Sec. 45(5)(b)]
Where the assessee has received enhanced compensation, then such enhanced compensation shall be taxable under
this section as under:

Total enhanced compensation or consideration received


[See Note].
Sale consideration Taxpoint: Such compensation shall be taxable in the year
of receipt only. If a part installment received then only part
amount shall be taxable and not the entire amount.
As usual
Expenditure on transfer Litigation expenses for getting the enhanced compensation
are deductible as expenses on transfer sec. 45(5)(b).
Cost of acquisition/Cost of improvement Nil
In the year when such enhanced compensation or
Taxable
consideration is received.
Nature of gain on enhanced compensation (whether
As in case of initial compensation or consideration.
STCG or LTCG)
Interest on enhanced compensation (if any) Taxable as ‘Income from other sources’
Note:
 Any amount of compensation received in pursuance of an interim order of a court, Tribunal or other authority
shall be deemed to be income chargeable under the head “Capital gains” of the previous year in which the final
order of such court, Tribunal or other authority is made.
 Where by reason of the death of the person who made the transfer, or for any other reason, the enhanced

286 The Institute of Cost Accountants of India


Heads of Income

compensation or consideration is received by any other person, the gain so computed in case of enhanced
compensation or consideration shall be deemed to be the capital gain, of such other person.
Tax treatment of reduced compensation [Sec. 45(5)(c)]
Where in any assessment year, such compensation or consideration (whether original or enhanced) referred above
is reduced by any court, tribunal or other authority, then capital gain of that year shall be recomputed considering
the compensation or consideration so reduced by such court, Tribunal or other authority.

Illustration 90:
Sunil has a house property acquired on 7/07/1995 for ₹ 3,00,000. He incurred improvement expenditure on such
property ₹ 70,000 on 16/08/2000 and ₹ 50,000 on 17/07/2010. Market value of such property as on 1/04/2001 is
₹ 4,50,000. On 16/08/2013, such property is compulsorily acquired by the Government and compensation decided
at ₹ 11,50,000. 20% of the compensation received on 31/03/2024 and balance on 2/06/2024.
On further appeal, on 16/08/2024 enhanced compensation is declared by the Government ₹ 2,00,000. Expenditure
incurred to get enhanced compensation is ₹ 11,000. Such compensation received on 18/08/2025. Compute income
under the head Capital Gains of Sunil for the assessment year 2024-25, 2025-26 and 2026-27.
Solution :
Computation of capital gains of Sunil for the A.Y. 2024-25
Particulars Working Details Amount
Sale consideration 11,50,0001
Less: Expenses on transfer Nil
Net sale consideration 11,50,000
Less: i) Indexed cost of acquisition ₹ 4,50,0002 * 2204/100 9,90,000
ii) Indexed cost of improvement3 ₹ 50,000 * 2204/167 65,868 10,55,868
Long Term Capital Gain 94,132
1. The initial compensation (i.e. ₹ 11,50,000) decided by the Government shall be treated as sale consideration.
2. Cost of acquisition is the original cost of acquisition (i.e. ₹ 3,00,000) or Fair market value as on 1/04/2001 (i.e.
₹ 4,50,000) whichever is higher.
3. Cost of improvement incurred before 1/04/2001 is to be completely ignored.
4. Though the property was compulsorily acquired by the Government in the P.Y 2013-14 but the compensation
was received in the P.Y.2023-24, therefore the amount shall be taxable in the P.Y. 2023-24, however indexation
benefit shall be available till the previous year 2013-14.
Computation of capital gains of Mr. Sunil for the A.Y. 2025-26: As the assessee has not received enhanced
compensation during the P.Y.2024-25, hence nothing is taxable in the A.Y. 2025-26.
Computation of capital gains of Mr. Sunil for the A.Y. 2026-27

Particulars Working Details Amount


Sale Consideration Enhanced compensation 2,00,000
Less: Expenses on transfer 11,000
Net Sale Consideration 1,89,000
Less: i) Indexed cost of acquisition Nil
ii) Indexed cost of improvement Nil Nil
Long Term Capital Gain 1,89,000

The Institute of Cost Accountants of India 287


Direct Taxation

In case of enhanced compensation, the cost of acquisition shall be taken as nil and the nature of capital gain shall
be same as that of initial compensation.

Exemption in case of certain compulsory acquisition [Sec. 10(37)]


Applicable to: An individual or an HUF
Conditions

1. Assessee has transferred urban agricultural land (being a capital asset).

2. Such land was used for agricultural purposes by such HUF or individual or his parents during the period of 2
years immediately preceding the date of transfer.

3. Such land is transferred –

¾¾ by way of compulsory acquisition under any law, or


¾¾ for a consideration to be determined or approved by the Central Government or the RBI.
4. The compensation or consideration for such transfer is received by such assessee on or after 1/04/04.

Treatment: Income on such transfer shall be exempted.

Exemption in case of transfer under Land Pooling Scheme [Sec. 10(37A)]

Applicable to: An individual or an HUF

Conditions

1. Assessee is the owner of asset being land or building or both on 02-06-2014

2. Such asset is transferred under the Land Pooling Scheme covered under the Andhra Pradesh Capital City Land
Pooling Scheme (Formulation and Implementation) Rules, 2015 made under the provisions of the Andhra
Pradesh Capital Region Development Authority Act, 2014

Treatment: In that case, capital gain on following transfer shall be exempted:

a. Transfer of land or building or both under the scheme;

b. Transfer of land pooling ownership certificate issued under the scheme to the assessee in respect of transfer
referred in (a);

c. Transfer of reconstituted plot or land received by the assessee against transfer mentioned in (a) within 2 years
from the end of the financial year in which the possession of such plot or land was handed over to him.

Taxpoint: Where such reconstituted plot or land is not transferred within said 2 years, the cost of acquisition of
such capital asset shall be deemed to be its stamp duty value as on the last day of the 2nd financial year after the end
of the financial year in which the possession of the said capital asset was handed over to the assessee.

2.4.21 Capital gain on Joint Development Agreement [Sec. 45(5A)]


Situation
 The assessee is an individual or a HUF

288 The Institute of Cost Accountants of India


Heads of Income

 Such assessee transfer capital asset being land or building or both under a specified agreement to another
person
¾¾ Specified agreement means a registered agreement in which a person owning land or building or both,
agrees to allow another person to develop a real estate project on such land or building or both, in
consideration of a share, being land or building or both in such project, whether with or without payment
of part of the consideration in cash
Treatment
Sale consideration Stamp duty value on the date of issue of completion certificate of his share being land
or building or both in the project + The consideration received in cash, if any.
Cost of acquisition Proportionate cost of the asset transferred
Benefit of indexation Available upto the year in which completion certificate is issued
Taxable In the year in which completion certificate for the whole or part of the project is issued
by the competent authority

Taxpoint
 The cost of acquisition of capital asset, being share in the project, in the form of land or building or both,
(covered under aforesaid provisions), shall be the amount which is deemed as full value of consideration while
computing capital gain u/s 45(5A).
 Competent authority means the authority empowered to approve the building plan by or under any law for the
time being in force.
 Where the assessee transfers his share in the project on or before the date of completion certificate, in that case,
aforesaid provision is not applicable and the capital gains shall be taxable in the previous year in which such
transfer takes place. Further, capital gain shall be computed as per other provisions.

2.4.22 Capital gain on distribution of assets by companies in its liquidation [Sec. 46]
On liquidation of a company, after payment of all its outside liabilities, remaining assets are disposed off in any of
the following two manners:
1. Remaining assets are sold in the market and sale proceeds distributed among shareholders.
2. Remaining assets itself are distributed among shareholders.
Above transaction shall be treated as under for computation of capital gain.
Tax Treatment in the hands of Company [Sec. 46(1)]
At the time of liquidation of company, any transfer by way of distribution of assets among its shareholder is not
treated as transfer for the purpose of capital gain. Hence, in case 2 when assets are directly distributed among
shareholders, nothing shall be charged to tax under the head ‘Capital gains’.
On the other hand, in case 1, when assets are sold in market (thereafter sale proceeds is distributed among
shareholders), then such sale of assets by the company in market shall be treated as transfer and chargeable under
the head ‘Capital gains’ as under:

Sale consideration As usual

Cost of acquisition/improvement/Expense on transfer As usual

The Institute of Cost Accountants of India 289


Direct Taxation

Taxable In the year when such assets are sold.


When the company does not distribute the assets directly to shareholders but sells it in market (referred as 1st
transfer) & distributes the proceeds among shareholders then, though such 1st transfer shall be liable to capital
gain, but distribution of proceeds among shareholder shall not be liable to capital gain.

Tax treatment in the hands of the shareholders [Sec. 46(2)]


When a shareholder receives money or other assets at the time of liquidation of the company then such receipts
[excluding the amount of dividend u/s 2(22)(c)] shall be liable to capital gain as under –

Total amount (or market value of assets) received on


Liquidation by the shareholder ****
Less: Amount of deemed dividend u/s 2(22)(c) (****)
Sale consideration Sale consideration ****
Deemed dividend [Sec. 2(22)(c)]: Any distribution of assets by a company, at the
time of liquidation, is treated as deemed dividend to the extent of accumulated
profit at the time of liquidation of company.
Cost of acquisition/Cost of
As usual
improvement

Notes
a. When distribution is made by liquidator, and the amount of distribution is less than the total amount of
Shareholders’ Fund, the distribution is deemed to take place in proportion of share capital and accumulated
profit as appears immediately before the distribution in the books of account of the company. [Refer Dividend
in chapter “Income from other sources”]

290 The Institute of Cost Accountants of India


Heads of Income

b. Payment in installments by the liquidator to shareholders: In case payment is received by the shareholder
in installments then the cost of acquisition shall be deducted from the earlier installment(s). Once the cost of
acquisition is absorbed by the earlier installment then subsequent installment (less expenditure on transfer)
shall be fully taxable.
c. The distributed asset may not be a capital asset, still provision of sec. 46(2) shall be attracted. E.g. if on
liquidation, company distributed its rural agricultural land (not being a capital asset) to its shareholder. Such
shareholder shall be liable to capital gain, as the transferred asset (on the part of shareholder) is share in the
company

Illustration 91 :
Balance sheet of Purva India (P) Ltd. as on 31/12/2023
Liabilities Amount Assets Amount
Equity Share capital of ₹ 10 each 8,00,000 Land 6,00,000
Preference Share capital 1,00,000 Building (WDV as per IT Act) 3,00,000
Reserves 2,00,000 Machinery (WDV as per IT Act) 4,00,000
Loan 6,00,000 Current Asset 10,00,000
Creditors 6,00,000
23,00,000 23,00,000
Additional information
Company went into liquidation on the balance sheet date and all current assets and building realized at book value.
The realized money was applied in payment of outside liabilities and preference shareholder. Utkarsh is a holder
of 10% equity share and 20% preference share of the company. Equity shares were originally acquired by him on
16/08/2002 at face value. However, he subscribed to preference share on 1-04-2022, which was issued at par. He
received a part of land (MV ₹ 5,00,000) and cash (for preference share) ₹ 20,000. Compute capital gain in hands
of company & Utkarsh.
Solution :
Computation of capital gain for the A.Y. 2023-24 in the hands of Purva India (P) Ltd
As per sec. 46(1), at the time of liquidation of company, any transfer by way of distribution of assets to its
shareholders is not treated as transfer for the purpose of capital gain. However, if any asset is sold in the market
the same shall be liable to capital gain.
In the given case, only one capital asset is sold i.e. building and the same being sold at book value, hence no capital
gain liability arises in the hands of company.
Computation of capital gain for the A.Y.2024-25 in the hands of Utkarsh
Particulars Working Details Amount
In case of equity share
Sale consideration 4,80,0001
Less: Expenses on transfer Nil
Net Sale Consideration 4,80,000
Less: i) Indexed cost of acquisition (10% of ₹ 8,00,000) * 348/105 2,65,143
ii) Indexed cost of improvement Nil 2,65,143
Long Term Capital Gain 2,14,857
In case of Preference share
Sale consideration 20,000

The Institute of Cost Accountants of India 291


Direct Taxation

Particulars Working Details Amount


Less: Expenses on transfer Nil
Net Sale Consideration 20,000
Less: i) Cost of acquisition 20% of ₹ 1,00,000 20,000
ii) Cost of improvement Nil 20,000
Short Term Capital Gain Nil
1.
Market value of land received ₹ 5,00,000
Less: Dividend u/s 2(22)(c) being 10% of accumulated profit (10% of ₹ 2,00,000) ₹ 20,000
Sale consideration ₹ 4,80,000

2.4.23 Capital Gain on buy back of own securities [Sec. 46A]


Buy back of specified securities (other than shares) is a transfer for its holder and shall be treated as under:

Sale consideration Amount received by a security-holder from the company.


Cost of acquisition/Cost of improvement As usual
Taxable In the year when such securities are purchased by the company.
Taxpoint: In case of buy back of shares, any income (or capital gain) in hands of shareholder is exempt u/s 10(34A).
However, company (listed or unlisted) itself is liable to pay additional tax @ 20% (+SC + Cess) u/s 115QA.

2.4.24 Transfer in case of total or partial partition of HUF [Sec 47(i) & 49(1)]
As per sec. 47(i), any distribution of capital assets in kind by HUF on its total or partial partition is not treated as
transfer u/s 47(i). However, when such assets is further transferred by its member then tax treatment shall be as
under:

Sale consideration As usual


Cost of acquisition Cost of assets in the hands of HUF
Index benefit on cost of acquisition shall be applicable from When the member holds such assets
Index benefit on cost of improvement shall be applicable When the actual improvement expenditure was
from incurred.
For determination of LTCA or STCA, period of
Determination of nature of asset
holding of the HUF shall be considered.

2.4.25 Capital gain on conversion of debentures into shares [Sec. 49(2A)]


Conversion of debentures (including debenture stock and deposit certificate) into shares or debentures are not
treated as a transfer.
Tax impact on sale of such converted shares shall be as under:

Cost of acquisition of new Cost of old asset (convertible debentures) shall be taken as cost of acquisition of new
asset asset (converted share).
Holding Period Starts from the date of acquisition of such debenture [Rule 8AA]
Benefit of indexation shall be available from the date of allotment of new asset
Indexation benefit
(converted share)
Note: Indexation benefit is not applicable in case of debenture

292 The Institute of Cost Accountants of India


Heads of Income

2.4.26 Zero Coupon Bond


As per Sec. 2(48) “Zero Coupon Bond” means a bond—
–– issued by any infrastructure capital company or infrastructure capital fund or infrastructure debt fund or public
sector company or scheduled bank;
–– in respect of which no payment and benefit is received or receivable before maturity or redemption from the
issuer; and
–– which the Central Government may, by notification in the Official Gazette, specify in this behalf.
Determination of nature of asset: A Zero Coupon Bond shall be treated as Long term capital asset if it is held by
the transferor for more than 12 months.
Treatment: Redemption or maturity or sale of such bond shall be treated as transfer.
Taxpoint:
Tax impact on sale or redemption or maturity of Zero coupon Bond shall be as under:

Sale consideration Redemption price, Maturity value or sale proceeds as the case may be.
Cost of acquisition of new asset As usual
Determination of nature of asset Long term capital asset if held for more than 12 months.
Indexation benefit Benefit of indexation shall not be available.
Notes:
1. Tax rate: Long term capital gain on such bond shall be taxed at 10% without indexation.
2. Tax treatment in hands of issuing company: Discount (i.e. the difference between the amount receivable
by the issuing company and the amount payable by such company on maturity or redemption of such bond)
shall be allowed as deduction u/s 36(1)(iiia) on pro rata basis during the life of the bond.

2.4.27 Valuation of consideration in case of land or building or both [Sec. 50C]


Conditions
a. Capital asset being land or building or both is transferred.
b. Value adopted or assessed or assessable by the stamp valuation authority exceeds 110% of actual consideration.
Tax treatment
Full value of consideration shall be the value adopted or assessed or assessable# by any authority of a State
Government (i.e. Stamp Valuation authority) for the purpose of payment of stamp duty.
#
Assessable means the price which the stamp valuation authority would have adopted or assessed, if it were
referred to such authority for the purposes of the payment of stamp duty.
Taxpoint: Where value adopted or assessed or assessable by the stamp valuation authority does not exceed 110%
of actual consideration or where such value is less than actual consideration, then actual consideration shall be
considered as full value of consideration.

The Institute of Cost Accountants of India 293


Direct Taxation

Provision Illustrated
Mr. Raj has a self-occupied house property acquired 10 months ago for ₹ 5,00,000. He sold such property for ₹
6,00,000 to Rajshree.
Case (a): Stamp duty authority for the purpose of levying stamp duty adopted value of ₹ 6,25,000.
Case (b): Stamp duty authority for the purpose of levying stamp duty adopted value of ₹ 6,75,000.
Compute capital gain on such transfer.
Solution
Computation of capital gain

Particulars Case (a) Case (b)


Sale consideration
- Actual Consideration 6,00,000 6,00,000
- Value adopted for stamp duty 6,25,000 6,75,000
Value adopted for stamp duty (i.e. ₹ 6,25,000) does not exceed 110% of actual
consideration (i.e., ₹ 6,60,000 being 110% of ₹ 6,00,000) 6,00,000
Value adopted for stamp duty (i.e. ₹ 6,75,000) exceeds 110% of actual 6,75,000
consideration (i.e., ₹ 6,60,000 being 110% of ₹ 6,00,000)
Less: Cost of acquisition 5,00,000 5,00,000
Short Term Capital Gain 1,00,000 75,000

Reference to Valuation Officer


The Assessing Officer can refer the case to the Valuation Officer (as appointed under Wealth Tax Act) for the
purpose of valuation of asset transferred if following conditions are satisfied:
 Assessee claims before any Assessing Officer that the value adopted or assessed or assessable by such authority
(i.e. Stamp Valuation authority) exceeds the fair market value of the property as on the date of transfer; &
 The value so adopted or assessed or assessable by the stamp valuation authority has not been disputed in any
appeal or revision or no reference has been made before any other authority, court or the High Court.
Consequences where the value is determined by the Valuation Officer

Case Result
If the value determined by the Valuation Officer
Value adopted or assessed or assessable for the purpose of
exceeds the value adopted or assessed or assessable
stamp duty shall be taken as full value of consideration.
for the purpose of stamp duty
If the value determined by the Valuation Officer does
Value determined by the Valuation Officer shall be taken
not exceed the value adopted or assessed or assessable
as full value of consideration.
for the purpose of stamp duty
Taxpoint: Where the valuation is referred to the Valuation Officer, sale consideration of the asset shall be taken as
minimum of the following –
 Value adopted or assessed or assessable for the purpose of stamp duty;
 Value determined by the Valuation Officer.

294 The Institute of Cost Accountants of India


Heads of Income

Difference in date of agreement and date of registration


Situation
Where:
a. the date of the agreement fixing the amount of consideration; and
b. the date of registration for the transfer of the capital asset
are not the same.
Condition
The amount of consideration, or a part thereof, has been received by way of an account payee cheque or account
payee bank draft or by use of electronic clearing system through a bank account or through specified electronic
modes, on or before the date of the agreement for transfer.
Treatment
the value adopted or assessed or assessable by the stamp valuation authority on the date of agreement may be taken
for the purposes of computing full value of consideration for such transfer.
Revision of value of such asset: If the value adopted for stamp duty purposes is revised in any appeal, revision
or reference, the assessment earlier made shall be amended to re-compute the capital gains by taking the revised
value as sale consideration [Sec. 155]

Provision Illustrated
Mr. Rajesh has a self-occupied house property acquired on 15/11/2022 for ₹ 6,00,000. He sold such property for
₹ 5,00,000 to Jayshree as on 18/08/2023. Stamp duty authority for the purpose of levying stamp duty adopted
value of ₹ 10,00,000.
Assessing Officer for computing capital gain on such transfer is taking sale consideration ₹ 10,00,000 u/s 50C. On
request of Mr. Raj, the matter being referred to Valuation Officer and value determined by the Valuation Officer is
₹ 9,00,000. Compute capital gain on such transfer.
Solution
Computation of capital gains in the hands of Mr. Raj for the A.Y. 2024-25

Particulars Details Amount


Sale consideration 9,00,000
Less: Expenses on transfer Nil
Net Sale Consideration 9,00,000
Less: Cost of acquisition 6,00,000
Less: Cost of improvement Nil 6,00,000
Short Term Capital Gain 3,00,000

Illustration 92:
Mr. A, who transfers land and building on 2-01-2024, furnishes the following information:
a. Net consideration received ₹ 10 lakhs;
b. Value adopted by stamp valuation authority, which was not contested by Mr. A ₹ 12 lakhs;
c. Value ascertained by Valuation Officer on reference by the Assessing Officer ₹ 13 lakhs;

The Institute of Cost Accountants of India 295


Direct Taxation

d. This land was distributed to Mr. A on the partial partition of his HUF on 1-04-2001. Fair market value of the
land as on 1-04-2001 was ₹ 1,00,000.
e. A residential building was constructed on the above land by Mr. A at a cost of ₹ 2,00,000 (construction
completed on 1-12-2004) during the Financial Year 2004-05.
Compute capital gain.
Solution :
Computation of capital gain of Mr. A for A.Y. 2024-25

Particulars Amount Amount


Sale Consideration (Higher of following)
- Actual consideration 10,00,000
- Value as per Valuation Officer (max. of value adopted for stamp valuation) 12,00,000 12,00,000
Less: Indexed Cost of acquisition
- Land [₹ 1,00,000 * 348/100] 3,48,000
- Building [₹ 2,00,000 * 348/113] 6,15,929 9,63,929
Long term capital gain 2,36,071

2.4.28 Valuation of consideration in case of unquoted shares [Sec. 50CA]


Where capital asset, being share of a company other than a quoted share, is transferred for a consideration which
is less than the fair market value of such share, the fair market value shall be deemed to be the full value of
consideration.
“Quoted share” means the share quoted on any recognised stock exchange with regularity from time to time, where
the quotation of such share is based on current transaction made in the ordinary course of business.
The provision is not applicable to any consideration received or accruing as a result of transfer by prescribed class
of persons.

2.4.29 Capital gain in the case of self-generated assets [Sec. 55(2)(a)]


As per sec. 55(2)(a), following assets are self generated asset if they are not purchased –

1. Goodwill of a business or profession; or 2. Tenancy rights; or


3. Trademark or brand name associated with a business or profession; or 4. Stage carriage permits;
5. Right to manufacture, produce or process any article or thing; or 6. Loom hours
7. Right to carry on any business or profession; or
Cost of acquisition of above assets
As per sec. 55(2)(a), cost of acquisition of such assets shall be determined as under –

Case Cost of acquisition


Acquisition of such asset by purchase Purchase price
In the case falling u/s 49(1)(i) to (iv) and where such asset was acquired by Purchase price of such previous
the previous owner by purchase owner
In any other case Nil

296 The Institute of Cost Accountants of India


Heads of Income

Treatment of self-generated asset generated before 1/4/2001: If aforesaid assets are developed or purchased
before 1/4/2001, the option of adopting fair market value shall not be applicable and even in such case the cost of
acquisition shall be computed as per the above table.
Depreciable Asset: Where the capital asset, being goodwill of a business or profession, in respect of which a
deduction on account of depreciation u/s 32(1) has been obtained by the assessee upto assessment year 2020-21,
the cost of acquisition (being purchase price) shall be reduced by the depreciation obtained by the assessee.
Cost of improvement [Sec. 55(1)(b)]
In case of goodwill, right to manufacture or produce any article or right to carry on any business or profession, cost
of improvement shall always be nil.
Treatment of other self-generated assets
Sec. 55(2)(a) gives an exhaustive list of self-generated capital assets which are liable to tax. Any other self-
generated asset (excluding bonus shares and right entitlements), not mentioned above, shall be out of purview of
tax i.e. fully exempted.
Instances of other self-generated assets transfer of which does not amount to capital gains:
 Transfer of trees grown spontaneously (self-generated)
 A self-generated formula if cost of its acquisition or improvement cannot be reasonably ascertained.
Taxpoint: Tax treatment on transfer of such assets shall be as under –
Goodwill of a business or profession, right to carry on a business or profession, right to manufacture or process
any article
Sale consideration Actual
Cost of acquisition Nil
Cost of improvement Nil
Expenditure on transfer Actual
Capital gain Sale consideration less expenditure on transfer
Tenancy right, stage carriage permits, loom hours, trade-mark & brand name associated with the business
Sale consideration Actual
Cost of acquisition Nil
Cost of improvement Actual
Expenditure on transfer Actual
Sale consideration less cost (or indexed cost) of improvement less expenditure on
Capital gain
transfer.
The above rule is applicable only in the case of self-generated asset.

Illustration 93:
Liza transferred the following assets on 2-05-2023, determine capital gain for the A.Y. 2024-25

Particulars Cost FMV 1/04/2001 Sale value


Land acquired in 1976 25,000 1,00,000 30,00,000
Goodwill of business [Business commenced on 1-05-1995] Nil 40,000 2,00,000
Tenancy right Nil 30,000 3,00,000
Brokerage paid on transfer @ 2%

The Institute of Cost Accountants of India 297


Direct Taxation

Solution :
Computation of capital gains in the hands of Liza for the A.Y. 2024-25

Particulars Workings Land Goodwill of business Tenancy right


Sale consideration 30,00,000 2,00,000 3,00,000
Less: Expenses on transfer 2% of above 60,000 4,000 6,000
Net Sale Consideration 29,40,000 1,96,000 2,94,000
Less: Indexed cost of acquisition ₹ 1,00,000 * 348/100 3,48,000 - -
As per sec. 55(2)(a) - Nil Nil
Less: Indexed cost of improvement Nil Nil Nil
Long Term Capital Gain 25,92,000 1,96,000 2,94,000

2.4.30 Capital gain in case of bonus share [Sec. 55(2)(aa)(iiia)]


As per sec. 55(2)(aa)(iiia), the cost of acquisition of financial asset, being allotted on the basis of holding of any
other financial asset to the assessee without any payment, shall be taken to be nil. Hence, cost of acquisition of
bonus share shall be taken as nil. However, if such asset is acquired before 1/4/2001 then its cost of acquisition
shall be taken as fair market value as on 1/4/2001.
Taxpoint: Tax treatment on transfer of bonus shares shall be as under:

Sale consideration As usual


Expenditure on transfer As usual
Period of holding Starts from the date of allotment of such share
Fair market value of such share as on
If bonus shares are allotted before 1/4/2001
Cost of acquisition 1/4/2001
If bonus shares are allotted on or after 1/4/2001 Nil

Illustration 94 :
Mr. Rocky purchased 1,000 shares of Azad (P) Ltd @ ₹ 12 per share as on 1/08/1998. Company declared one bonus
share for every two shares held on 31/03/2000. As on 7/07/2006, Rocky got 500 shares of the same company as gift
from his friend Rakesh (Rakesh acquired such share on 1/04/2001 @ ₹ 14 per share). As on 1/03/2023, company
further declared one bonus share for every five shares held. On 1/01/2024, Rocky sold all the shares @ ₹ 50 each.
Find capital gain of Mr. Rocky.
Solution :
Number of shares held by Mr. Rocky

Total number of
Shares No. of shares
shares to the date
Original share 1,000 1,000
Bonus share declared on 31/03/2000 [1,000 shares/2] 500 1,500
Share received as gift from Rakesh as on 7/07/2006 500 2,000
Bonus share declared on 1/03/2023 [2,000 shares/5] 400 2,400

298 The Institute of Cost Accountants of India


Heads of Income

Computation of capital gains in the hands of Mr. Rocky for the A.Y. 2024-25

Bonus share
Bonus share
Original acquired Gifted
Particulars Workings acquired
share before shares
after 1/4/2001
1/4/2001
No of shares held 1,000 500 500 400
Cost of acquisition per share Note ₹ 14 ₹ 14 ₹ 14 Nil
Sale consideration Shares held * ₹ 50 50,000 25,000 25,000 20,000
Less: Expenses on transfer Nil Nil Nil Nil
Net Sale Consideration 50,000 25,000 25,000 20,000
Less: Indexed cost of 1,000*₹ 14*348/100 48,720 - - -
acquisition 500 * ₹ 14 * 348/100 - 24,360 - -
500 * ₹ 14 * 348/122 - - 19,967 -
Less: Cost of acquisition - - - Nil
Less: Indexed cost of
Nil Nil Nil -
improvement
Less: Cost of improvement - - - Nil
Long Term Capital Gain 1,280 640 5,033 -
Short Term Capital Gain - - - 20,000

Note: Cost of acquisition in case of -


Higher of Original Cost (₹ 12) or fair market value as on 1/04/2001 (₹
Original share
14)
Bonus share acquired before 1/04/2001 Fair market value as on 1/04/2001 (₹ 14)
Gifted share Actual cost of the previous owner (₹ 14)
Bonus share acquired after 1/04/2001 Nil

2.4.31 Bonus Stripping [Sec. 94(8)] Amended


Conditions for applicability
Where—
a. any person buys or acquires any securities or units$ within a period of 3 months prior to the record date#;
b. such person is allotted additional securities or units without any payment on the basis of holding of such
securities or units on such date;
c. such person sells or transfers all or any of the original securities or original units within a period of 9 months
after such date, while continuing to hold all or any of the additional securities or units referred to in clause (b).
Tax treatment
a. Loss, if any, arising to him on account of such purchase and sale of all or any of such securities or units shall
be ignored for the purposes of computing his income chargeable to tax; and
b. Notwithstanding anything contained in any other provision of this Act, the amount of loss so ignored shall be
deemed to be the cost of purchase or acquisition of such additional securities or units referred to in clause (b)
as are held by him on the date of such sale or transfer.
The Institute of Cost Accountants of India 299
Direct Taxation

$
Unit shall mean:
a. a unit of a business trust defined u/s 2(13A);
b. a unit defined in the Explanation to sec. 115AB i.e., unit of mutual fund specified u/s 10(23D) or of Unit Trust
of India; or
c. beneficial interest of an investor in an Alternative Investment Fund, defined in regulation 2(1)(b) of the
Securities and Exchange Board of India (Alternative Investment Funds) Regulations, 2012, made under the
Securities and Exchange Board of India Act, 1992, and shall include shares or partnership interests
#
Record date means such date as may be fixed by –
a. a company;
b. a Mutual Fund or the Administrator of the specified undertaking or the specified company referred to in the
Explanation to sec. 10(35); or
c. a business trust defined in sec. 2(13A); or
d. an Alternative Investment Fund defined in regulation 2(1)(b) of the Securities and Exchange Board of India
(Alternative Investment Funds) Regulations, 2012, made under the Securities and Exchange Board of India
Act, 1992,
for the purposes of entitlement of the holder of the securities or units, as the case may be, to receive dividend,
income, or additional securities or units without any consideration, as the case may be

Illustration 95 :
Mr. A purchased 5,000 units of a mutual fund as on 10/10/2023 @ ₹ 80. On 1/11/2023, the fund declares 4 bonus
units for every 5 units held. The record date for declaring bonus is 01/11/2022.
On 01/12/2023, Mr. A sold 2,500 original units @ ₹ 20.
On 10/01/2024, Mr. A sold 1,000 bonus units @ ₹ 25.
On 12/02/2024, Mr. A sold remaining 2,500 original units @ ₹ 30.
On 14/03/2024, Mr. A sold remaining 3,000 bonus units @ ₹ 45.
All units sold to his friend. Compute capital gain.
Solution :
Computation of capital gain of Mr. A for the A.Y.2024-25

Particulars Workings Amount


Capital gain on transfer of 2,500 original units on 1/12/2023
Sale consideration 2,500 units @ ₹ 20 50,000
Less: Cost of acquisition 2,500 units @ ₹ 80 2,00,000
Cost of bonus units as per provision of Sec. 94(8) (1,50,000)
Taxable short term capital gain Nil#
#
As per Sec. 94(8), above loss shall be ignored for the purposes of computing his income, and the amount of loss
so ignored shall be deemed to be the cost of acquisition of such bonus unit. Hence, the cost of 4,000 bonus units
(5,000 units / 5 * 4) is ₹ 1,50,000 i.e. cost of each bonus units is ₹ 37.5 (₹ 1,50,000 / 4,000 units).

300 The Institute of Cost Accountants of India


Heads of Income

Particulars Workings Amount


Capital gain on transfer of 1,000 bonus units on 10/01/2024
Sale consideration 1,000 units @ ₹ 25 25,000
Less: Cost of acquisition 1,000 units @ ₹ 37.5 as per Sec. 94(8) 37,500
Short term capital gain (12,500)
Capital gain on transfer of 2,500 original units on 12/02/2024
Sale consideration 2,500 units @ ₹ 30 75,000
Less: Cost of acquisition 2,500 units @ ₹ 80 2,00,000
Cost of remaining bonus units as per provision of sec. 94(8) (1,25,000)
Taxable short term capital gain Nil#
#
As per Sec. 94(8), above loss shall be ignored for the purposes of computing his income, and the amount of loss
so ignored shall be deemed to be the cost of acquisition of remaining bonus unit. Hence, the cost of 3,000 bonus
units (4,000 units – 1,000 units) shall be as under:
Loss on transfer of original units on 1/12/2024 ₹ 37.50 per unit
Loss on transfer of original units on 12/02/2024 ₹ 1,25,000 / 3,000 units ₹ 41.67 per unit
₹ 79.17 per unit
Capital gain on transfer of 3,000 bonus units on 14/03/2024
Sale consideration 3,000 units @ ₹ 45 1,35,000
Less: Cost of acquisition 3,000 units @ ₹ 79.17 as per sec. 94(8) 2,37,510
Short term capital gain (1,02,510)

2.4.32 Capital gain in case of transfer of right share and right entitlement [Sec. 55(2)(aa)]
Right Share: Where, by virtue of holding a share or any other security, (hereinafter this clause referred to as the
financial asset), the assessee becomes entitled to subscribe to any additional financial asset, then such additional
financial asset can be termed as right share [Sec. 55(2)(aa)]. Cost of acquisition of such right share shall be the
amount actually paid by him for acquiring such right share.
Right Entitlement: An assessee can endorse his right to acquire additional financial asset (as stated above) in
favour of other person. Such endorsement of right is termed as right renouncement. Cost of acquisition of such
right entitlement shall be taken as nil.
Tax treatment of right issue and right entitlements shall be as under:

Shares acquired by Right


Case Right shares Right Entitlement
Renouncee
Amount paid for acquisition of
Cost of Acquisition Right issue price Nil right entitlements + Amount
paid to company for right share
Period of holding The date of allotment of The date of declaration of The date of allotment of such
starts from such shares such right by the company shares
Amount charged from Amount charged from
Sale consideration Amount charged from transferee
transferee transferee

The Institute of Cost Accountants of India 301


Direct Taxation

Illustration 96:
Mr. Raunak purchased 1,000 shares of Zey (P) Ltd @ ₹ 12 per share as on 1/08/2022. As on 1/05/2023, company
declared one right share for each share held @ ₹ 15 each. Mr. Raunak renounced 40% of such right in favour of
Miss Rani @ ₹ 2 per share and for balance, he subscribed to the company. On 1/07/2023, Mr. Raunak and Miss
Rani sold all the shares to one of their friend @ ₹ 50 each. Find capital gain of both the assessee.
Solution :
Working: Details of shares and right entitlements transferred

Date Particulars No. of shares Rate Cost of acquisition


1/08/2021 Original shares 1,000 12 12,000
1/05/2022 Right shares 600 15 9,000
1/05/2022 Right entitlement 400 Nil Nil
Computation of capital gain in the hands of Mr. Raunak for the A.Y. 2024-25

Particulars Workings Original share Right share Right entitlement


No. of shares held 1,000 600 400
Shares held × ₹ 50 50,000 30,000 -
Sale consideration
Shares renounced × ₹ 2 - - 800
Less: Expenses on transfer Nil Nil Nil
Net Sale Consideration 50,000 30,000 800
Less: Cost of acquisition As calculated above 12,000 9,000 Nil
Less: Cost of improvement Nil Nil Nil
Short Term Capital Gain 38,000 21,000 800
Computation of capital gain in the hands of Miss Rani for the A.Y. 2024-25
Particulars Details Amount
Sale Consideration 400 * ₹ 50 20,000
Less: Expenses on transfer Nil
Net Sale Consideration 20,000
Less: Cost of acquisition 400 * ₹ 17 #
6,800
Less: Cost of improvement Nil
Short Term Capital Gain 13,200
No. of shares acquired 400 shares
#
Cost of acquisition of each such shares
Amount paid for acquisition of such share entitlements ₹2
Amount paid to company for such right shares ₹ 15 ₹ 17

302 The Institute of Cost Accountants of India


Heads of Income

2.4.33 Conversion of inventory into capital assets


Where inventory is converted into, or treated as, a capital asset, the fair market value of such inventory as on the
date of such conversion is considered as business income u/s 28(via). If such capital asset is transferred then it will
be treated as under:

Sale consideration As usual


Cost of acquisition Fair Market Value taken as income u/s 28(via)
Period of holding Starts from the date of such conversion or treatment
Benefit of indexation Available from the year of such conversion or treatment

2.4.34 Capital Gain on Virtual Digital Assets [Sec. 115BBH r.w.s 2(47A)] New
As per sec. 2(47A), virtual digital asset means:
a. any information or code or number or token (not being Indian currency or foreign currency), generated
through cryptographic means or otherwise, by whatever name called, providing a digital representation of
value exchanged with or without consideration, with the promise or representation of having inherent value,
or functions as a store of value or a unit of account including its use in any financial transaction or investment,
but not limited to investment scheme; and can be transferred, stored or traded electronically;
b. specified non-fungible token or any other token of similar nature, by whatever name called;
c. any other notified digital asset
Taxpoint: The Central Government may exclude any digital asset from the definition of virtual digital asset.
Tax Treatment
Rate of tax
Any income from the transfer of any virtual digital asset (VDA) shall be taxable @ 30%.
Computation of income on transfer of such assets
While computing such income:
d. Deduction in respect of any expenditure (other than cost of acquisition, if any) or allowance or set off of any
loss shall not be allowed to the assessee.
e. Set off of loss from transfer of the virtual digital asset shall not be allowed against other income
f. Loss from transfer of the virtual digital asset shall not be allowed to be carried forward to succeeding assessment
years.
g. Index benefit is not allowed.
In nutshell,
Cost of acquisition ü Allowed (without index)
Other expenditure (like improvement expenses, Not allowed
û
expenses on transfer, etc.)
Loss on transfer of VDA û No adjustment with other income
Loss from other activities û No adjustment with income on transfer of VDA
Carry forward of loss on transfer of VDA û Not allowed
Taxpoint: Where VDA has been held as investment, the income on transfer shall be taxable under the head
Capital Gains, however, if VDA has been held for trading purpose, the income may be taxable as business income.
However, in either of the case, computation shall be made as per aforesaid provision and rate of tax will be 30%.

The Institute of Cost Accountants of India 303


Direct Taxation

2.4.35 Tax on Capital Gain


Tax on short term capital gain (STCG)
Tax on STCG on transfer of certain assets on which Security transaction tax has been charged [Sec. 111A]
STCG, in certain cases, shall be taxed @ 15% + surcharge* (if applicable) + Health and Education Cess.
Applicable to
All assessee
Conditions to be satisfied
1. Nature of asset: A short-term capital asset, being an equity share in a company or a unit of an equity oriented
fund or a unit of a business trust.
2. Securities transaction tax: Such transaction is chargeable to securities transaction tax.
Tax rate
Such short-term capital gains shall be taxed @ 15% + surcharge + Health and Education Cess.
Notes
1. Concessional rate shall be applicable on short term capital gain arises from a transaction undertaken in a
foreign currency on a recognised stock exchange located in any International Financial Services Centre even
though STT is not applicable on such transaction
2. Securities transaction tax is applicable only on transfer through recognized stock exchange.
3. In the case of an individual or an HUF being a resident, where the total income as reduced by such short-term
capital gains is below exempted ceiling i.e. ₹ 2,50,000 or ₹ 3,00,000 or ₹ 5,00,000 (as the case may be), then
such short-term capital gains shall be reduced by the amount by which the total income so reduced falls short
of such exempted ceiling and the tax on the balance of such short-term capital gains shall be computed @ 15%.
Taxpoint: Where the total income as reduced by such short-term capital gains is below exempted ceiling, then
tax on such short-term capital gains shall be –
15% of [Such STCG – (Exempted ceiling – Total income excluding such STCG)].
4. No deduction under chapter VIA can be claimed from such short term capital gain.
Illustration 97:
Short term capital gain on transfer of shares on which STT is paid ₹ 1,30,000
Other income ₹ 1,66,000
Calculate tax of Mr. X aged 45 years?

* In case of individual, HUF, AOP, BOI and other artificial person, surcharge on dividend income and income covered u/s 111A , 112 and 112A cannot
exceed 15%. In nutshell, applicable surcharge are as under:

Situation % of Surcharge
Total income including income covered u/s 111A, 112 and 112A does not exceed ₹ 50 lakhs Nil

Total income including income u/s 111A, 112 and 112A exceeds ₹ 50 lakhs but does not exceed ₹ 1 crore 10%

Total income including income u/s 111A, 112 and 112A exceeds ₹ 1 crore but does not exceed ₹ 2 crore 15%

Total income excluding income u/s 111A, 112 and 112A exceeds ₹ 2 crore but does not exceed ₹ 5 crore 25%

Total income excluding income u/s 111A, 112 and 112A exceeds ₹ 5 crore 37%

Total income including dividend, income u/s 111A, 112 and 112A exceeds ₹ 2 crore, if case is not falling under any of the 15%
aforesaid situations

304 The Institute of Cost Accountants of India


Heads of Income

Solution :
Computation of tax for the A.Y.2024-25
Particulars Other income STCG
Income ₹ 1,66,000 ₹ 1,30,000
Exemption limit on which no tax is charged i.e. ₹ 2,50,000 ₹ 1,66,000 ₹ 84,000
Nil ₹ 46,000
Tax rate Nil 15%
Tax ₹ 6,900
Less: Rebate u/s 87A ₹ 6,900
Tax after Rebate u/s 87A Nil
Add: Health & Education Cess Nil
Tax Liability (Rounded off) Nil Nil
Short-term capital gain in any other case: In any other case, short-term capital gain is to be taxed as per usual
rates, as applicable to any other income.
Tax on long term capital gain in certain cases [Sec. 112A]
Applicable to: All assessee
Conditions:
a. The total income of the assessee includes income chargeable under the head “Capital gains”;
b. The capital gains arise from the transfer of a long-term capital asset being an equity share in a company or a unit
of an equity-oriented fund* or a unit of a business trust;
c. Securities Transaction Tax (STT) has been levied:
Capital Asset STT has been paid
Equity share in a company STT has been paid on acquisition and transfer of such capital asset.
Exception:
In the following cases, condition of levying STT on acquisition is
not applicable:
a. Equity share acquired before 01-10-2004
b. The nature of acquisition which is notified (like IPO, ESOP,
etc.) [Notification No. 60/2018 dated 01-10-2018]

* “Equity Oriented Fund” means a fund set up under a scheme of a mutual fund specified u/s 10(23D) or under a scheme of an insurance company
comprising unit linked insurance policies to which exemption under clause (10D) of the said section does not apply on account of the applicability of
the fourth and fifth provisos thereof, and:
i. in a case where the fund invests in the units of another fund which is traded on a recognised stock exchange,:
a. a minimum of 90% of the total proceeds of such fund is invested in the units of such other fund; and
b. such other fund also invests a minimum of 90% of its total proceeds in the equity shares of domestic companies listed on a recognised stock
exchange; and
ii. in any other case, a minimum of 65% of the total proceeds of such fund is invested in the equity shares of domestic companies listed on a
recognised stock exchange.
Taxpoint:
¾¾ The percentage of equity shareholding or unit held in respect of the fund, as the case may be, shall be computed with reference to the annual
average of the monthly averages of the opening and closing figures.
¾¾ In case of a scheme of an insurance company comprising ULIP to which exemption u/s 10(10D) does not apply on account of the applicability
of the fourth and fifth provisos thereof, the minimum requirement of 90% or 65%, as the case may be, is required to be satisfied throughout the
term of such insurance policy.

The Institute of Cost Accountants of India 305


Direct Taxation

Capital Asset STT has been paid


Unit of an equity-oriented fund or a unit of a STT has been paid on transfer of such capital asset
business trust
Taxpoint: The condition of payment of STT in either case is not applicable in case where transfer has been
undertaken on a recognised stock exchange located in any International Financial Services Centre provided the
consideration for such transfer is received or receivable in foreign currency.
Treatment
Such long-term capital gain shall be taxable as under:

Case Rate of Taxation


Where such long term capital gain does not exceed ₹ 1,00,000 Nil
Where such long term capital gain exceeds ₹ 1,00,000 10%+ on income exceeding ₹ 1,00,000
Taxpoint:
 Rabate u/s 87A is not available from tax on aforesaid long term capital gain.
 In the case of resident individual or resident Hindu undivided family, where the total income as reduced by
such long-term capital gains is below the maximum amount which is not chargeable to income-tax, then, the
long-term capital gains, shall be reduced by the amount by which the total income as so reduced falls short of
the maximum amount which is not chargeable to income-tax [See Example 1 & 2 below]
Tax Computation

a) In the case of an individual or a Hindu undivided family [a resident]—


Where [Total Income - Long-term capital gains] > Exempted ceiling (i.e. ₹ 2,50,000 or 3,00,000 or
5,00,000, as the case may be)
Tax on LTCG @ 20%
Tax on [Total income - such long-term capital gains] As per the slab
Where [Total Income - Long-term capital gains] < Exempted ceiling (i.e. ₹ 2,50,000 or 3,00,000 or
5,00,000, as the case may be)
Tax on LTCG – [Exempted ceiling – (Total Income - Long-term capital gains)] @ 20%
Tax on [Total income - Long-term capital gains] Nil
Taxpoint: Above method of computation is not applicable in case of non-resident.
Example 19: Mr. Janardan, a resident aged 52 years, has other income of ₹ 1,70,000 and LTCG ₹ 90,000. He shall
be liable to tax as follows –
Computation of tax liability

Particulars LTCG Other income Total


Taxable income 90,000 1,70,000 2,60,000
Exempted up to ₹ 2,50,000 #(₹ 2,50,000 – ₹ 1,70,000) 80,000# 1,70,000 2,50,000
Balance 10,000 Nil 10,000

306 The Institute of Cost Accountants of India


Heads of Income

Particulars LTCG Other income Total


Tax rate 20% Slab
Tax liability before rebate 2,000 Nil 2,000
Less: Rebate u/s 87A 2,000
Tax payable Nil

Example 20: Mr. Nishith, a resident aged 40 years, has other income of ₹ 70,000 and LTCG u/s 112A ₹ 3,65,000.
He shall be liable to tax as follows –
Computation of tax liability
Particulars LTCG u/s 112A Other income Total
Taxable income 3,65,000 70,000 4,35,000
Exempted up to ₹ 2,50,000 #(₹ 2,50,000 – ₹ 70,000) 1,80,000# 70,000 2,50,000
Balance 1,85,000 Nil 1,85,000
Tax rate @applicable on income in excess of ₹ 1,00,000 10%@ Slab
Tax liability before rebate 8,500 Nil 8,500
Less: Rebate u/s 87A NA
Tax after rebate 8,500
Add: Health & Education Cess 340
Tax and Cess Payable 8,840

Example 21: Joseph, a non-resident has LTCG ₹ 1,00,000. He shall be liable to tax @ 20% on LTCG, i.e. ₹ 20,000
(₹ 1,00,000 * 20%) (plus cess), even total income does not exceed maximum exempted limit.

Deduction under chapter VIA [Sec. 112(2)]: No deduction under chapter VIA (i.e. u/s 80C to 80U) is
available against LTCG.

Option available on capital gains in respect of shares, securities and units [Proviso to Sec. 112]
Applicability
Long-term capital asset being security listed in any recognized stock exchange in India are transferred –
 As per sec. 2(h) of the Securities Contracts (Regulation) Act, 1956, securities include:
i. Shares, scripts, stocks, bonds, debentures, debenture stock or other marketable securities of a like nature
in or of any incorporated company or other body corporate.
ii. Government securities.
iii. Such other instruments as may be declared by the Central Government to be securities.
iv. Rights or interest in securities.

The Institute of Cost Accountants of India 307


Direct Taxation

Treatment
Capital gain may be calculated and assessed in any of the following two options:

Step Option 1 Option 2


1 st
Find sale consideration Find sale consideration
Deduct expenditure on transfer, indexed cost of Deduct expenditure on transfer, cost of acquisition and
2nd acquisition and indexed cost of improvement cost of improvement.
as usual.
Taxpoint: No indexation benefit shall be available.
3rd The balancing amount [i.e., (1) – (2)] is long The balancing amount [i.e., (1) – (2)] is long term
term capital gain capital gain.
4th Such capital gain shall be taxed @ 20% + Such capital gain shall be taxed @ 10% + surcharge (if
surcharge (if any) + education cess. any) + education cess.
Note: Though the tax rate in option 2 is lower, but since no indexation benefit is available, it is difficult to state
which option is better for assessee. However, in the case of transfer of listed debentures and listed bonds, Option 2
will be better as compared to Option 1, as because in such asset indexation benefit is not available.

Illustration 98:
Mr. Jadu has sold following assets on 31/03/2024:

Assets transferred Cost Acquired on Sold for Expenses on transfer


Land ₹ 4,00,000 19/08/2009 ₹ 15,00,000 ₹ 40,000
Government securities ₹ 10,000 17/07/2006 ₹ 1,00,000 ₹ 5,000
Debentures (listed) ₹ 20,000 17/04/2010 ₹ 1,00,000 ₹ 2,000
Compute his tax liability.

Solution :
Computation of capital gain of Mr. Jadu for the A.Y. 2024-25
Alternative 1) Taxing LTCG at higher rate (20%) and allowing benefit of indexation

Particulars Land Government securities Debenture


Sale consideration 15,00,000 1,00,000 Not to be
applied1
Less: Expenditure on transfer 40,000 5,000

Net sale consideration 14,60,000 95,000

Less: Indexed cost of acquisition (Note) 9,40,540 28,525

Less: Indexed cost of improvement - -

Long Term Capital Gain 5,19,460 66,475

308 The Institute of Cost Accountants of India


Heads of Income

Particulars Land Government securities Debenture


Tax on Long Term Capital Gain @ 20%
1,03,897 13,295
[Subject to alternative 2 (given below)]

Alternative 2) Taxing LTCG at lower rate (10%) and not allowing indexation benefit
Particulars Land Government securities Debenture
Sale consideration No such 1,00,000 1,00,000
option
Less: Expenditure on transfer 5,000 2,000
available
Net Sale consideration 95,000 98,000
Less: Cost of acquisition (Note) 10,000 20,000
Less: Cost of improvement - -
Long Term Capital Gain 85,000 78,000
Tax on Long Term Capital Gain @ 10% 8,500 7,800
[Subject to alternative 1 (given above)]
Tax liability for LTCG Lower of
1,03,892 8,500 7,800
(alternative 1) & (alternative 2)
1.
As benefit of indexation is not available.

Note: Computation of cost of acquisition or indexed cost of acquisition

Assets Cost Year Indexed cost of acquisition (ICOA)


Land ₹ 9,40,540 2009-10 ₹ 9,40,540 i.e. (₹ 4,00,000 × 348 / 148)
Government securities ₹ 28,525 2006-07 ₹ 28,525 i.e. (₹ 10,000 × 348 / 148)
100 Debentures ₹ 20,000 2010-11 No benefit of indexation

Deduction from Capital gain


From capital gain so computed, several deductions are allowed as per provisions of sections 54, 54B, 54D, 54EC,
54EE, 54F, 54G, 54GA and 54GB. Provisions of these sections are discussed as under:

The Institute of Cost Accountants of India 309


Direct Taxation

2.4.36 Deduction from capital gain on sale of residential house property [Sec. 54]
Applicable to Individual or HUF
1. Assessee has transferred a long-term residential house, income of which is taxable under
the head “Income from house property”.
2. Assessee must acquire one new residential house within prescribed time limit.
Alternate Option
When Available: Where the amount of the capital gain does not exceed ₹ 2 crore.
Option: The assessee may, at his option, purchase or construct two residential houses in
India
Restriction: Where during any assessment year, the assessee has exercised this option, he
shall not be subsequently entitled to exercise the option for the same or any other assessment
Conditions year. That means, the option is available once in lifetime of the assessee.
The new residential house should be in India.
Taxpoint:
¾¾ Capital asset must be a long term capital asset.
¾¾ Property must be a residential house whether let-out or self occupied.
¾¾ Income of such property must be taxable u/s 22.
¾¾ Land transferred appurtenant to a house property (assessable u/s 22) together with such
house property, also qualifies for deduction u/s 54.
¾¾ The new residential house(s) may not be taxable u/s 22. e.g. a new house acquired for the
residence of employees shall be eligible for deduction.
Within a period of ‘1 year before, or 2 years after, the date of transfer’
For Purchase Example 6: If a property is transferred on 17/8/2022, then the new house
property may be purchased at any time between 17/08/2021 to 17/8/2024.
Within a period of 3 years after the date of transfer.
Time limit for
acquisition of For Construction Construction may start at any time but must be completed within stipulated
new assets time
Scheme of
Applicable [Refer ‘Capital Gains Account Scheme’ given below].
deposit
In case of compulsory acquisition of such capital asset by the Government, the time limit shall
start from receipt of compensation or part thereof.
Minimum of the following:
Deduction ¾¾ Investment in the new asset(s) [excess of ₹ 10 crore is not considered]; or
¾¾ Capital gain
If the newly acquired residential house is transferred within 3 years from the date of
acquisition of new assets, then the benefit availed earlier shall be revoked. Such revoked
Revocation of income shall be reduced from cost of acquisition of new asset.
benefit If the amount held in Capital Gains Deposit Account Scheme (1988) is unutilized, then
such amount shall be taxable as long-term capital gain in the previous year in which the period
of 3 years from the date of transfer expires.

310 The Institute of Cost Accountants of India


Heads of Income

Notes
1. Legal title of the house: Holding of legal title is not necessary. It is sufficient that assessee has made the full
(or substantial) payment within the time limit even though the transfer deed has not been registered and the
possession is given after stipulated time.
2. Transfer of part of house: Exemption u/s 54 is available on sale of part of the house if the same is an
independent unit.
3. Treatment of Land: The cost of land is integral part of the residential house.
4. Treatment in hands of legal heir: The benefit of sec. 54 is also available to the legal heir of deceased assessee
provided he fulfills conditions of sec. 54.
Capital Gains Accounts Scheme, 1988

Introduction If the new asset is not acquired till the due date of submission of return of income#, then the
taxpayer will have to deposit the money in ‘Capital Gains Deposit Account’ with a nationalized
bank. The proof of deposit should be submitted along with the return of income. On the basis
of actual investment and the amount deposited in the deposit account, exemption will be given
to the taxpayer.

Due date of filing of return being 31st October (where audit is required) & 31st July (in any
#. *

other case)
Utilisation of The taxpayer is to acquire a new asset by withdrawing from the deposit account.
amount New asset must be acquired within specified time, provided in the relevant section.
If deposit The unutilized amount will become chargeable to tax in the previous year in which the specified
amount remains time limit expires. Chargeable amount shall be –
unutilized
For sec. 54, 54B, 54D, 54G, 54GA: Unutilised amount
For sec. 54F & 54GB: Unutilised amount × Capital gain [Capital gain in excess of ₹ 10 crore
is not considered]
Net sale consideration [excess of ₹ 10 crore is not considered]
Nature of gain It will be taxable as short term or long-term capital gain depending upon the status of the
original capital gain.
Notes
a. The unutilized amount can be withdrawn by the taxpayer after the expiry of the aforesaid time limit.
b. The unutilized amount in Capital Gain Account Scheme (1988), in hands of legal heir of deceased individual,
cannot be taxed. [Circular No.743 dated 6/5/1996]

Illustration 99:
Mr. Sidhartha has a residential house property taxable u/s 22. Such property is acquired on 12/08/2005 for
₹ 2,00,000. The property is sold on 1/03/2024 for ₹ 25,00,000. He acquired another residential house on 31/03/2024
for ₹ 17,00,000 for self-occupation. On 1/03/2025, he sold such new residential house for ₹ 30,00,000. Compute
his capital gain for the A.Y. 2024-25 and 2025-26.

* In some cases, 30th November

The Institute of Cost Accountants of India 311


Direct Taxation

Solution :
Computation of capital gain of Mr. Sidhartha for the A.Y. 2024-25

Particulars Details Amount


Sale consideration 25,00,000
Less: Expenditure on transfer Nil
Net sale consideration 25,00,000
Less: Indexed cost of acquisition ₹ 2,00,000 × 348 / 117 5,94,872
Less: Indexed cost of improvement Nil
Long term capital gain 19,05,128
Less: Exemption u/s 54 17,00,000
Taxable Long term Capital gain 2,05,128
Computation of capital gain of Mr. Sidhartha for the A.Y. 2025-26

Particulars Amount Amount


Sale consideration 30,00,000
Less: Expenditure on transfer Nil
Net sale consideration 30,00,000
Less: Cost of acquisition 17,00,000
Less: Earlier exemption claimed u/s 54 (17,00,000) Nil
Short term capital gain 30,00,000

2.4.37 Deduction from capital gain on transfer of agro land [Sec. 54B]
Applicable to Individual or HUF
1. Assessee must have transferred a capital asset being an agricultural land (whether long
term or short term).
Taxpoint: A rural agricultural land is not a capital asset hence on transfer of rural land
no capital gain arises.
2. Such agricultural land must have been used by the individual or his parents or by such
HUF for agricultural purposes for at least 2 years, prior to its transfer.
Notes
a) An agricultural land, acquired for a period of less than 2 years before the date of
Conditions transfer and used for agricultural purpose from the date of acquisition, shall not be
eligible for deduction.
b) Where the assessee obtains land on partition of HUF, the above condition is satisfied
if the assessee and HUF taken together have used the land for agricultural purpose
for at least 2 years immediately before the transfer.
c) Where an assessee was using the property as a tenant and later on purchased the said
property, then while calculating the period of 2 years (as stated above), the period
during which assessee was using the property as tenant shall also be included.
3. Assessee must purchase a new land for agricultural purpose. The new land may be in
urban area or rural area.

312 The Institute of Cost Accountants of India


Heads of Income

Time limit for Within 2 years after the date of transfer.


acquisition of new
Scheme of deposit Applicable [Refer ‘Capital Gains Account Scheme’].
assets
Minimum of the following:
Deduction ¾¾ Investment in the new asset; or
¾¾ Capital gain
If the newly acquired agricultural land is transferred within 3 years from the date of
acquisition of new assets, then the benefit availed earlier shall be revoked. Such revoked
income shall be reduced from cost of acquisition of new asset.
Revocation of
If the amount held in Capital Gains Deposit Account Scheme (1988) is unutilized,
benefit and its
then such amount shall be taxable as short term or long-term capital gain (depending on
treatment
the nature of gain on original transfer) in the previous year in which the period of 2 years
from the date of transfer expires.
Case: Assessee sold an urban agricultural land and invested the capital gain in rural
agricultural land (to claim deduction under this section) and immediately sold such newly
acquired rural agricultural land.
Case Effect: Since new agricultural land sold before 3 years, therefore exemption shall be
revoked and benefit claimed earlier shall be subtracted from cost of acquisition of new
asset. However, the new agricultural land is not liable to capital gain as it is not a capital
asset (being a rural agricultural land).
Exemption u/s Subject to certain conditions, in case of compulsory acquisition of urban agro-land by the
10(37) Government, income shall be exempted u/s 10(37).

2.4.38 Deduction from capital gain on compulsory acquisition of land and building forming
part of industrial undertaking [Sec. 54D]
Applicable to All assessee
1. Assessee must have transferred a capital asset being a land or building or any right
therein, forming part of an industrial undertaking.
Taxpoint: Asset may be a short term or long-term capital asset.
2. Such capital asset has been compulsorily acquired under any law for the time being
in force.
3. Such capital asset was used for industrial purpose by the assessee for at least 2
years prior to its transfer.
Conditions Notes
a) A land or building acquired for a period of less than 2 years and used for
industrial purpose from the date of acquisition, shall not be eligible for
deduction.
b) Where the assessee obtains such capital asset on partition of HUF, the above
condition is satisfied if the assessee and HUF taken together have used the
asset for industrial purpose for at least 2 years immediately before the transfer

The Institute of Cost Accountants of India 313


Direct Taxation

c)
Where an assessee was using the property as a tenant and later on purchased
the said property then while calculating the period of 2 years (as stated above),
the period during which assessee was using the property as tenant shall also
be included.
4. Assessee must purchase any other land or building or construct a building, for the
purpose of shifting or reestablishing the said undertaking or setting up another
industrial undertaking
Time-limit for Within 3 years after the date of receipt of compensation or any part thereof.
acquiring assets Scheme of deposit Applicable [Refer ‘Capital Gains Account Scheme’].
Minimum of the following:
Deduction ¾¾ Investment in the new asset; or
¾¾ Capital gain
If the newly acquired land or building is transferred within 3 years from the date
of acquisition of new assets, then the benefit availed earlier shall be revoked. Such
revoked income shall be reduced from cost of acquisition of new asset.
Revocation of benefit
and its treatment If the amount held in Capital Gains Deposit Account Scheme (1988) is unutilized,
then such amount shall be taxable as short term or long-term capital gain (depending
on the nature of gain on original transfer) in the previous year in which the period of 3
years from the date of receipt of compensation expires.
Illustration 100:
X Ltd. has a building acquired on 17/08/2022 for ₹ 5,00,000. The assessee, as a tenant, earlier used the building,
for industrial purpose since last 7 years and even after purchase it is continuously used for industrial purpose.
Such building is compulsorily acquired by Government at an agreed value of ₹ 12,00,000 as on 15/07/2023. The
compensation was received on 1/03/2024. The written down value of the block (consist of 3 buildings) as on
1/04/2023 is ₹ 7,50,000.
The company acquired a new building for industrial purpose for ₹ 2,00,000 as on –
Case A) 31/03/2024; Case B) 2/04/2024. Determine his taxable capital gain.
Solution :
X Ltd. purchased the building on 17/8/2022 and transfer took place on 15/07/2023, i.e. after acquisition of ownership,
asset is used for industrial purpose for less than 2 year. However, the assessee was using the same building for
industrial purpose as a tenant before acquisition, for a period of 7 years hence it can claim the benefit of sec. 54D.
Computation of capital gain in the hands of X Ltd. for the A.Y. 2024-25
Particulars Case A Case B
Written Down Value as on 1/04/2023 7,50,000 7,50,000
Add: Acquisition of new asset during the year 2,00,000 -
9,50,000 7,50,000
Less: Sale value of asset (compensation received from Government) 12,00,000 12,00,000
Short Term Capital gain 2,50,000 4,50,000
Less: Exemption u/s 54D 2,00,000 2,00,000
Taxable Short Term Capital Gain 50,000 2,50,000

314 The Institute of Cost Accountants of India


Heads of Income

2.4.39 Deduction from capital gain on acquisition of certain bonds [Sec. 54EC]
Applicable to All assessee
1. Assessee must have transferred any long-term capital asset being land or building
or both.
2. Assessee acquires ‘long term specified assets’.
Long-term specified asset means any bond redeemable after 5 years, issued by
Conditions a. the National Highways Authority of India (NHAI);
b. the Rural Electrification Corporation Ltd.;
c. Power Finance Corporation Limited;
d. Indian Railway Finance Corporation Limited;
e. any other bond being notified by the Central Government

Within 6 months after the date of transfer


In case of compulsory acquisition, time limit starts from the date of receipt of
compensation.
Time limit for acquisition
of new assets In case of conversion of capital asset into stock in trade, the time limit of 6 months
for making investme nts in specified assets should be taken from the date of actual
sale of stock in trade and not from the date when capital asset is converted into stock.
[Circular No.791 dated 2/6/2000]

Minimum of the following:


¾¾ Investment in the new asset; or
¾¾ Capital gain
Amount of Deduction Taxpoint:
¾¾ The investment made by an assessee in the long-term specified asset, from capital
gains arising from transfer of one or more original assets, during the financial
year in which the original asset or assets are transferred and in the subsequent
financial year does not exceed ₹ 50 lakhs
Earlier benefit shall be revoked if such bond is transferred or converted into money
within 5 years of its acquisition or a loan is taken on security of the new asset within
the said period.
Revocation of benefit
Taxpoint: In case where an assessee takes any loan or advance on the security of such
specified asset, he shall be deemed to have converted such specified asset into money
on the date on which such loan or advance is taken.
Treatment of revoked Such revoked income shall be treated as long-term capital gain in the year of transfer
income of new asset.
Scheme of deposit Not applicable
Notes
Benefit u/s 54EC on depreciable asset: In the case “CIT vs Assam Petroleum Industries (P) Ltd. (2003) (Gau)
“it was decided that benefit u/s 54EC shall be available on depreciable asset held by the assessee for more than
specified months. It was held by the Court that the depreciable asset is no where defined as short term capital asset.
Under sec. 50 it has been merely mentioned that gain on transfer of a depreciable asset shall be taxed as short term
capital gain, but that is not sufficient to treat depreciable asset (held for more than specified months) as short term
capital asset.
The investment made for availing benefit u/s 80C shall not be further eligible for deduction u/s 54EC.

The Institute of Cost Accountants of India 315


Direct Taxation

2.4.40 Deduction from capital gain on investment in units of a specified fund [Sec. 54EE]
Applicable to All assessee
1. Assessee must have transferred any long-term capital asset.
2. Assessee acquires ‘long term specified assets’.
Conditions
Long-term specified asset means a unit or units, issued before 01-04-2019, of such
fund as may be notified by the Central Government
Time limit for acquisition
Within 6 months after the date of transfer
of new assets
Minimum of the following:
¾¾ Investment in the new asset; or
¾¾ Capital gain
Amount of Deduction Taxpoint:
¾¾ The investment made by an assessee in the long-term specified asset, from capital
gains arising from transfer of one or more original assets, during the financial
year in which the original asset or assets are transferred and in the subsequent
financial year does not exceed ₹ 50 lakhs
Earlier benefit shall be revoked if such bond is transferred or converted into money
within 3 years of its acquisition or a loan is taken on security of the new asset within
the said period.
Revocation of benefit
Taxpoint: In case where an assessee takes any loan or advance on the security of such
specified asset, he shall be deemed to have converted such specified asset into money
on the date on which such loan or advance is taken.
Treatment of revoked Such revoked income shall be treated as long-term capital gain in the year of transfer
income of new asset.
Scheme of deposit Not applicable

2.4.41 Deduction from capital gain on transfer of capital assets other than residential
house property [Sec. 54F]
Applicable to Individual or HUF
Conditions 1. Assessee must have transferred a long-term capital asset other than a residential house
property.
2. Assessee must acquire one residential house within prescribed time limit, income of
which is taxable u/s 22.
3. Such new house should be situated in India.
4. Assessee does not own more than one residential house property, income of which is
taxable u/s 22 (other than new house), on the date of transfer.
5. Assessee does not purchase, within 2 years, or construct, within 3 years of transfer of
the original asset, any other residential house, income of which is taxable u/s 22.

316 The Institute of Cost Accountants of India


Heads of Income

Time limit for For Purchase Within a period of ‘1 year before, or 2 years after, the date of transfer’.
acquisition of new For Construction Within a period of 3 years after the transfer
assets
Construction may start at any time but must be completed within
stipulated time
Scheme of
Applicable [Refer ‘Capital Gains Account Scheme’].
deposit
Note: In case of compulsory acquisition the time limit starts from the date of initial receipt
of compensation.
Amount of Minimum of the following:
Deduction
Investment in the new asset x Capital gain
¾¾
Net Sale consideration#
¾¾ Capital gain

Note - Investment in new asset in excess of ₹ 10 crore is not considered


#
Net sale consideration = Sale consideration – Expenditure on transfer
Revocation of 1. If the newly acquired residential house is transferred within 3 years after the date of
benefit and its its acquisition, benefit availed earlier shall be revoked.
treatment
2. If another residential house is purchased (apart from newly acquired residential
house property) by the assessee within 2 years or constructed within 3 years after the
date of transfer of original asset, benefit availed earlier shall be revoked.
Taxpoint: The time limit shall be determined from the date of transfer of original asset
even in the case when asset is compulsory acquired by the Government.
If the amount, held in Capital Gains Deposit Account Scheme (1988), is unutilized,
3.
benefit availed earlier shall be revoked.
Treatment of ¾¾ Revocation due to case 1 & 2 above
revoked income Such revoked income (exemption) shall be taxable as long-term capital gain in the year
of revocation of condition.
¾¾ Revocation due to case 3 above
Chargeable amount shall be –
Unutilised amount for which benefit u/s 54F is availed x Original capital gain
Net sale consideration (excess of ₹ 10 crore is not considered)
- taxable as long term capital gain of the previous year in which 3 years from the date
of transfer of asset expires.
Notes: Refer notes of sec. 54.

Illustration 101:
Sonu has jewellery acquired on 17/07/2010 for ₹ 5,00,000. On 18/08/2013 Sonu incurred improvement expenditure
on such jewellery by adding diamond to it worth ₹ 3,00,000. On 18/08/2021, he transferred such jewellery to his
friend Monu for ₹ 40,00,000.
Sonu already has a self-occupied house property in Lucknow, however on 17/03/2022 he purchased another
residential house property for ₹ 30,00,000 for the purpose of letting out.

The Institute of Cost Accountants of India 317


Direct Taxation

As on 5/04/2023, his friend offered him house worth ₹ 25,00,000 (Value for Stamp duty purpose is only ₹ 14,00,000/-
) for ₹ 15,00,000 only & Sonu purchased the same.
On 7/04/2024, Sonu sold the new house acquired from his friend for ₹ 19,00,000. Value determined for the purpose
of stamp duty purposes ₹ 22,00,000 and market value as on the date of transfer is ₹ 26,00,000. Compute capital
gain in hands of Sonu for several years.
Solution :
Computation of capital gain of Sonu for the A.Y. 2023-24
Particulars Workings Details Amount
Sale consideration for Jewellery 40,00,000
Less: Expenditure on transfer Nil
Net sale consideration 40,00,000
Less: Indexed cost of acquisition ₹ 5,00,000 × 317 / 167 9,49,102
Less: Indexed cost of improvement ₹ 3,00,000 × 317 / 220 4,32,273 13,81,375
Long term capital gain 26,18,625
Less: Exemption u/s 54F [₹ 30,00,000 / ₹ 40,00,000 x ₹ 26,18,625] 19,63,969
Taxable Long term capital gain 6,54,656
Computation of capital gain of Sonu for the A.Y. 2024-25: Since the assessee acquired another house property
therefore the earlier exemption availed u/s 54F shall be revoked and shall be liable to long term capital gain. Hence
taxable long-term capital gain for the A.Y.2024-25 is ₹ 19,63,969.
Computation of capital gain of Sonu for the A.Y. 2025-26

Particulars Details Amount


Sale consideration Value determined for Stamp duty [Sec. 50C] 22,00,000
Less: Expenditure on transfer Nil
Net sale consideration 22,00,000
Less: Cost of acquisition 15,00,000
Less: Cost of improvement Nil 15,00,000
Short term capital gain 7,00,000

Illustration 102:
Mr. X has sold following assets during the year 2023-24
Items Cost of acquisition Sale consideration Year of acquisition
Land ₹ 10 lacs ₹ 150 lacs 1998-99
Jewellery ₹ 30 lacs ₹ 120 lacs 2008-09
On 31/03/2024, he has purchased a residential house of ₹ 30,00,000 for self occupation as he had no other house
till date. Compute capital gain.
Solution :
Computation of capital gains in the hands of Mr. X for the A.Y. 2024-25
Particulars Details Land Jewellery
Sale Consideration 1,50,00,000 1,20,00,000
Less: Expenses on transfer Nil Nil

318 The Institute of Cost Accountants of India


Heads of Income

Particulars Details Land Jewellery


Net Sale Consideration 1,50,00,000 1,20,00,000
Less: Indexed cost of Acquisition 10,00,000 * 348 / 100 34,80,000
Less: Indexed cost of Acquisition 30,00,000 * 348 / 137 76,20,438
Less: Indexed cost of improvement Nil Nil
Long Term Capital Gain 1,15,20,000 43,79,562
Less: Exemption u/s 54F Working 1 23,04,000 Nil
Long Term Capital Gain 92,16,000 43,79,562
Working 1
In the given case assessee can claim benefit u/s 54F, for any of the LTCG (land or jewellery).
A Comparative study is made under to decide from which LTCG such deduction should be claimed –

Particulars Working Land Jewellery


Long Term Capital Gain A 1,15,20,000 43,79,562
Net sale consideration B 1,50,00,000 1,20,00,000
Benefit u/s 54F A/B * ₹ 30,00,000 23,04,000 10,94,891
Since deduction is higher in case of Land hence the deduction u/s 54F is ₹ 23,04,000

2.4.42 Deduction from capital gain on transfer of capital assets in case of shifting of
industrial undertaking from urban areas [Sec. 54G]
Applicable to All assessee
Conditions 1. Assessee must have transferred a capital asset, being –
¾¾ A machinery or plant or building or land; or
¾¾ Any rights in building or land,
¾¾ used for the purposes of the business of an industrial undertaking situated in an
urban area.
2. Such transfer is effected in the course of shifting of such industrial undertaking to any
area other than an urban area.
3. Assessee has within a period of 1 year before, or 3 years after, the date of transfer —
a.
purchased new machinery or plant for the purpose of business of the industrial
undertaking in the area to which the said undertaking is shifted;
b. acquired building or land or constructed building for the purposes of his business
in the said area;
c. shifted the original asset and transferred the establishment of such undertaking
to such area; and
d. incurred expenses on such other purpose as may be specified in a scheme framed
by the Central Government for the purposes of this section.
Time limit for Within 1 year before, or 3 years after, the date of transfer.
acquiring new asset. Scheme of deposit Applicable [Refer ‘Capital Gains Account Scheme’].

The Institute of Cost Accountants of India 319


Direct Taxation

Amount of Minimum of the following:


Deduction  Amount expended for the above purposes [as stated in (a) to (d) above];
 Capital gain
Revocation of If the newly acquired assets are transferred within 3 years from its date of acquisition, then
benefit and its the benefit availed earlier shall be revoked. Such revoked income shall be subtracted from
Treatment cost of acquisition of newly acquired assets.
If the amount held in Capital Gains Deposit Account Scheme (1988) is unutilized, then
such amount shall be taxable as short term or long-term capital gain (depending on the
nature of gain on original transfer) in the previous year in which the period of 3 years from
the date of transfer expires.
Note: Urban area means any such area within the limits of a municipal corporation or municipality as the Central
Government may, having regard to the population, concentration of industries, need for proper planning of the
area and other relevant factors, by general or special order, declare to be an urban area for the purposes of this
section.

Illustration 103 :
X Ltd. is shifting its undertaking from Jaipur to Napasar (other than urban area). In this regard it sold its 4
machineries and 2 sets of furniture during the previous year 2023-24 as under –

Depreciation Book Depreciation Book


Machinery Sold for Furniture Sold for
Rate Value Rate value
A 15% 2,00,000 3,00,000 X 10% 1,00,000 2,00,000
B 15% 3,00,000 8,00,000 Z 10% 60,000 90,000
C 15% 5,00,000 6,00,000
D 30% 6,00,000 5,00,000

WDV of the block of asset as under –


Name of the Block Block consist of WDV as on 1/04/2023
Machinery 15% A, B & C 11,00,000
Machinery 30% D&E 9,00,000
Furniture 10% X&Z 1,50,000
X Ltd. is seeking whether the transaction shall be taxable as slump sale or not and compute capital gain.
On 7/04/2023, assessee further purchased machineries worth ₹ 3,70,000 and land of ₹ 1,00,000 for the purpose of
new industrial undertaking. Compute capital gain.
Solution :
In the given case though assessee has transferred a group of assets still the transaction cannot be taxed as slump sale
because value has been assigned to individual asset. The tax treatment shall be as under:
Particulars Details Amount
On sale of Machinery A, B & C

320 The Institute of Cost Accountants of India


Heads of Income

Particulars Details Amount


Block: Machinery (Rate 15%)
W.D.V. as on 1/04/2023 11,00,000
Add: Purchase during the year Nil
11,00,000
Less: Sale during the year 17,00,000
Short Term Capital Gain 6,00,000
On sale of Machinery D
Block: Machinery (Rate 30%)
W.D.V. as on 1/04/2023 9,00,000
Add: Purchase during the year Nil
9,00,000
Less: Sale during the year 5,00,000
WDV before depreciation 4,00,000
Less: Depreciation @ 30% 1,20,000
WDV after depreciation 2,80,000
Short Term Capital Gain Nil
On sale of Furniture X & Z
Block: Furniture (Rate 10%)
W.D.V. as on 1/04/2023 1,50,000
Add: Purchase during the year Nil
1,50,000
Less: Sale during the year 2,90,000
Short Term Capital Gain 1,40,000
Total Short Term Capital Gain 7,40,000
Less: Exemption u/s 54G 4,70,000
Taxable Short Term Capital Gain 2,70,000

Illustration 104:
Raj Ltd. has an industrial undertaking operating in an urban area. During the year 2023-24, it sold its two machineries
for ₹ 2,00,000 and ₹ 3,80,000 in course of shifting its industrial undertaking from urban area to rural area. The
written down value of the block (consists of 4 machineries) as on 1/04/2023 was ₹ 3,50,000.
The company incurred ₹ 20,000 expenses on other purposes as specified in a scheme framed by the Central
Government for the purposes of this section. Determine its taxable capital gain, if the company acquired new
machinery for industrial purpose for ₹ 50,000 as on –
Case A) 28/03/2024; Case B) 12/04/2024
Solution :
Computation of capital gain in the hands of Raj Ltd. for the A.Y. 2024-25
Particulars Case A Case B
Written Down Value as on 1/04/2023 3,50,000 3,50,000
Add: Acquisition of new asset during the year 50,000 Nil
4,00,000 3,50,000
Less: Sale value of assets 5,80,000 5,80,000
Short Term Capital gain 1,80,000 2,30,000
Less: Exemption u/s 54G – for new machinery purchased 50,000 50,000

The Institute of Cost Accountants of India 321


Direct Taxation

Particulars Case A Case B


- for expenditure incurred 20,000 20,000
Taxable Short Term Capital Gain 1,10,000 1,60,000

2.4.43 Deduction from capital gain on transfer of capital assets in case of shifting of
industrial undertaking from urban areas to Special Economic Zone [Sec. 54GA]
Applicable to All assessee
Conditions 1. Assessee must have transferred a capital asset, being –
¾¾ A machinery or plant or building or land; or
¾¾ Any rights in building or land,
–– used for the purposes of the business of an industrial undertaking situated in an
urban area.
2. Such transfer is effected in the course of shifting of such industrial undertaking to
Special Economic Zone (SEZ).
Note: The SEZ may be developed in any urban area or any other area.
3. Assessee has within a period of 1 year before, or 3 years after, the date of transfer -
i. purchased new machinery or plant for the purpose of business of the industrial
undertaking in the SEZ to which the said undertaking is shifted;
ii. acquired building or land or constructed building for the purposes of his
business in the SEZ;
iii. incurred expenditure on shifting the original asset and transferred the
establishment of such undertaking to such SEZ; and
iv. incurred expenses on such other purpose as may be specified in a scheme
framed by the Central Government for the purposes of this section.
Time limit for acquiring Within 1 year before, or 3 years after, the date of transfer.
new asset.
Scheme of deposit Applicable [Refer ‘Capital Gains Account Scheme’].
Amount of Deduction Minimum of the following:
¾¾ Amount expended for the above purposes [as stated in (a) to (d) above];
¾¾ Capital gain
Revocation of benefit If the newly acquired assets are transferred within 3 years from its date of acquisition,
and its then the benefit availed earlier shall be revoked. Such revoked income shall be subtracted
Treatment from cost of acquisition of newly acquired assets.
If the amount held in Capital Gains Deposit Account Scheme (1988) is unutilized, then
such amount shall be taxable as short term or long-term capital gain (depending on the
nature of gain on original transfer) in the previous year in which the period of 3 years
from the date of transfer expires.

322 The Institute of Cost Accountants of India


Heads of Income

Applicable to All assessee


Note: Urban area means any such area within the limits of a municipal corporation or municipality as the Central
Government may, having regard to the population, concentration of industries, need for proper planning of the
area and other relevant factors, by general or special order, declare to be an urban area for the purposes of this
section.

2.4.44 Exemption under more than one provision


An assessee can claim exemption under more than one section (from sec. 54 to 54GA) if conditions of the respective
sections are fulfilled. E.g. An assessee deriving long term capital gain on sale of a residential house can claim
benefit u/s 54 by investing a part of the capital gain in acquisition of a new residential house property and as well as
claim benefit u/s 54EC by investing remaining part of the capital gain in acquisition of specified securities.

2.4.45 Extension of time for acquiring new asset or depositing or investing amount of
capital gain [Sec. 54H]
Applicability ¾¾ Where the transfer of the original asset is by way of compulsory acquisition under any
law; and
¾¾ Amount of compensation awarded for such acquisition is not received by the assessee on
the date of such transfer.
Treatment The period for acquiring the new asset or the period available to the assessee for depositing the
amount of capital gain in relation to such compensation as is not received on the date of the
transfer, shall be reckoned from the date of receipt of such compensation.
Notes
a. It is irrespective of anything contained in sec. 54, 54B, 54D, 54EC and 54F.
b. Enhanced Compensation: In case of enhanced compensation, the period for acquiring the new asset shall
commence from the date of receipt of such enhanced compensation.

2.4.46 Reference to Valuation Officer [Sec. 55A]


With a view to ascertaining the fair market value of a capital asset for the purposes of this chapter [e.g. sec. 45(1A),
45(2), 45(4), 46(2), 55 and 2(47)] the Assessing Officer may refer the valuation of capital asset to a Valuation
Officer.
Cases where reference to Valuation Officer can be made

Case Condition
Where the value of the asset as claimed If the Assessing Officer is of opinion that the value so claimed is at
by the assessee is in accordance with the variance with its fair market value
estimate made by a registered valuer.

The Institute of Cost Accountants of India 323


Direct Taxation

In any other case If the Assessing Officer is of the opinion—


1. That the fair market value of the asset exceeds the value of the
asset as claimed by the assessee by more than
¾¾ 15% of the value of the asset as so claimed; or
¾¾ by more than ₹ 25,000
whichever is less.
2. That having regard to the nature of the asset and other relevant
circumstances, it is necessary to do so.
Valuation Officer has the same meaning, as in sec. 2 (r) of the Wealth-tax Act, 1957.

Quick MCQs:-

1. Capital Gain arises from the transfer of –


(a) any Asset
(b) any Capital
(c) Tangible assets only.
(d) Intangible assets only.

2. Distribution of assets by a Company at the time of liquidation shall be regarded as a transfer and subject
to Capital Gain-
(a) in the hands of the Company
(b) in the hands of the Shareholders
(c) in the hands of both Company as well as Shareholders
(d) neither in the hands of a Company nor in the hands of Shareholders.

3. If share are acquired on conversion of debentures, the cost of acquisition of such share shall be-
(a) Market Value of Shares on the date of conversion
(b) Market Value of the debentures on the date of conversion.
(c) Cost of acquisition of the debentures.
(d) None of above.

4. The cost of acquisition of shares under Employees Stock Option Scheme shall be-
(a) Fair Market Value of Shares on the date of offer
(b) Fair Market Value of the shares on the date of exercise of option
(c) Fair Market Value of share on the date of vesting of option.
(d) Price at which it was offered to the employee.

5. Conversion of Capital Asset into Stock in Trade will result into Capital Gain of the previous year-
(a) in which such conversion took place
(b) in which such converted asset is old or otherwise transferred

324 The Institute of Cost Accountants of India


Heads of Income

(c) any of the above


(d) none of the above

6. Where a Partner transfers any Capital Asset into the business of Firm, the sale consideration of such asset
to the Partner shall be-
(a) Market Value of such asset on the date of such transfer
(b) Price at which it was recorded in the books of the firm
(c) Cost of such asset to the Partner
(d) Leas of the above

7. Where the entire block of the depreciable assets is transferred after 36 months, there will be-
(a) Short-Term Capital Gain
(b) Long-Term Capital Gain
(c) Short- Term Capital Gain or Loss
(d) Long- Term Capital Gain or Loss

8. For claiming exemption u/s 54, the assessee should purchase Residential Property with in –
(a) 2 years after the date of transfer
(b) 3 years after the date of transfer
(c) 1 year before or 2 years after the date of transfer
(d) 1 year before & 3 years after the date of transfer.

9. Exemption u/s 54EC shall be available to –


(a) Any Assessee
(b) Individual Only
(c) Individual or HUF
(d) Company Assessee only

10. Exemption under Section 54F shall not be allowed if the assessee, on the date of transfer, owns-
(a) any Residential House
(b) a Residential House which is let out
(c) a Hose which is self occupied
(d) more than one Residential House.

11. For claiming exemption u/s 54G, the assessee shall acquired the new asset within-
(a) 2 years from the date of transfer
(b) 3 years from the date of transfer
(c) 1 year before or 2 years after the date of transfer
(d) 1 year before or 3 years after the date of transfer

The Institute of Cost Accountants of India 325


Direct Taxation

Income from Other Sources 2.5


As per sec. 56(1), any income, which is not specifically exempted and not chargeable under any other heads of
income, shall be chargeable under the head “Income from other sources”. This is the last and residuary head of
income.
Taxpoint:
A receipt shall be taxable under this head if the following conditions are satisfied:
 Such receipt shall be a taxable income; and
 Such income does not specifically fall under any one of the other four heads of income (i.e. ‘Salaries’, ‘Income
from house property’, ‘Profits and gains of business or profession’ or ‘Capital gains’).
Sec. 56(2) lays down a list of incomes, which are taxable under this head. Such list is not exhaustive. Apart from
the income stated in sec. 56(2) any other income, which is fulfilling all the above conditions, shall be taxable under
this head.
Sec. 56(2) lays down the list of incomes, which are specifically taxable under this head:
Income absolutely chargeable under this head
1. Dividends [Sec. 56(2)(i)] [discussed later]
2. Casual income e.g. Winning from lotteries, etc. [Sec. 56(2)(ib)] [discussed later]
3. Gift [Sec. 56(2)(x) [discussed later]
4. Share premium in excess of fair market value of shares [Sec. 56(2)(viib) [discussed later]
5. Income by way of interest received on compensation or on enhanced compensation [Sec.56(2)(viii)] [discussed
later]
6. Sum of money received as an advance or otherwise in the course of negotiations for transfer of a capital asset,
if:
a. such sum is forfeited; and
b. the negotiations do not result in transfer of such capital asset [Sec. 56(2)(ix)] [Discussed in Capital Gain]
Income chargeable under this head if not charged under the head ‘Profits and gains of business or profession’
7. Any sum received by the assessee from his employees as contribution to provident fund, etc. [Sec. 56(2)(ic)]
8. Interest on securities [Sec. 56(2)(id)] [discussed later]
9. Income from letting of machinery, plant or furniture [Sec. 56(2)(ii)] [discussed later]
10. Composite Rent [Sec. 56(2)(iii)] [discussed later]

326 The Institute of Cost Accountants of India


Heads of Income

Income chargeable under this head if not charged under the head ‘Profits and gains of business or profession’
or under the head ‘Salaries’-
11. Any sum (including bonus) received under a Keyman Insurance Policy [Sec. 56(2)(iv)]
Keyman Insurance Policy means a life insurance policy taken by a person on the life of another person, who is
either the employee or is connected in any manner with the business of the former person [Explanation to Sec.
10(10D)]
12. Any compensation or other payment, due to or received by any person, in connection with the termination of
his employment or the modification of the terms and conditions relating thereto. [Sec. 56(2)(xi)]
13. Receipts from life insurance policy issued on or after 01.04.2023, if premium payable for any of the previous
years during the term policy exceeds ₹ 5,00,000 [except in the event of death of policy holder].
Apart from above, the following incomes are also chargeable under this head by virtue of sec. 56(1). In this
regard it is to be noted that the following list is merely indicative and not exhaustive.
1. Income from sub-letting of a house property.
2. Interest on bank deposits.
3. Interest on company deposits, interest on loans, etc.
4. Remuneration received from a person other than his employer for evaluation of answer scripts. However, if
such remuneration is received from employer, then the same will be taxable under the head “Salaries”.
5. Rent from a vacant land.
6. Insurance commission.
7. Income from undisclosed sources
8. Income from private tuition.
9. Interest on income tax refund.
Taxpoint: Income tax refund itself is not an income.
10. Family pension received by the family members of a deceased employee [discussed later]
11. Dividend received from a co-operative society.
12. Directors’ sitting fee for attending Board Meetings.
13. Income from activity of owning and maintaining race-horses.
14. Stipend to trainee.
15. Interest on employee’s contribution towards unrecognized provident funds at the time of payment of lump
sum amount

2.5.1 Basis of chargeability [Sec.145]


Income under this head shall be chargeable on ‘accrual’ or ‘cash’ basis depending on the method of accounting
regularly followed by the assessee (i.e. either mercantile or cash system of accounting).
Exception: Dividend is charged as per the method specified in sec. 8 (discussed later in this chapter)

2.5.2 Casual Income: Winning from lotteries, crossword puzzles, etc. [Sec. 56(2)(ib)]
Winnings from -
1. Lotteries;
2. Crossword puzzles;

The Institute of Cost Accountants of India 327


Direct Taxation

3. Races including horse races;


4. Gambling and betting of any nature or form; or
5. Card games, game show or entertainment program on television or electronic mode and any other game of
any sort,
6. On-line gaming.
- are taxable under this head.
Lottery includes winnings from prizes awarded to any person by draw of lots or by chance or in any other manner
whatsoever, under any scheme or arrangement by whatever name called.
Card game and other game of any sort includes any game show, an entertainment programme on television or
electronic mode, in which people compete to win prizes or any other similar game.
Exemption/deduction [Sec. 58(4)]: Such income shall be fully taxable & no deduction shall be allowed.
Tax rate [Sec. 115BB]: Tax is charged at a flat rate of 30%.
Notes
a. Income of jockey: Income of jockey from such profession is not treated as winning from horse races.
b. Winning from a motor car rally: Winning from a motor car rally is a return for skill and effort and cannot be
treated as casual income but taxable as normal income
c. Lottery held as stock in trade: Winning from lottery to an agent or trader out of its unsold stock (tickets)
shall be treated as incidental to business and taxed under the head “Profits & gains of business or profession”
d. Expenditure to be deducted: No deduction can be claimed from such income even if such expenditure is
incurred exclusively and wholly for earning such income.
e. Deduction: Deduction u/s 80C to 80U is not available from such income.

Method of grossing up of income / Conversion of income received into gross income in case of casual income
Sometime in the problem, lottery income received is given rather than lottery income. In such case, students
are required to gross up the lottery income received. Relation between lottery income earned and lottery income
received is as under -
Lottery income received = Lottery income earned – Tax deducted at source on such income#
#
Tax deducted at source: By virtue of sec. 194B & 194BB, tax is deductible at sources @ 30% on payment in
respect of winning from lotteries, etc.
Procedure of grossing up, in case of resident individual or HUF, are as follow-

Lottery Income Received = Gross Lottery Income – TDS @ 30% on Gross Lottery Income.
Lottery Income Received = 70% of Gross Lottery Income
Lottery Income Received
Gross Lottery Income =
70%

Note: Tax is not deducted in following cases, hence, there is no need of grossing up.-
a. If the amount of winning from lottery etc. or horse race is not more than ₹ 10,000.

328 The Institute of Cost Accountants of India


Heads of Income

b. In case of winning from gambling, betting, on-line gaming and racing other than horse race e.g. camel races,
etc.

2.5.3 Income from machinery, plant or furniture let on hire [Sec. 56(2)(ii)]
Income from letting of machinery, plant or furniture on hire is charged to tax under this head, if such income is not
chargeable under the head “Profits and gains of business or profession”.
Notes
a. Any income by way of letting out an asset as a part of business activity or as commercial asset shall be taxable
under the head “Profits & gains of business or profession”.
b. In case of temporary discontinuance of business due to any reason without any intention of the assessee to part
with or close the business, if the business assets are leased out for a certain period, then such lease rent shall
be taxed under the head “Profits & gains of business or profession”.

2.5.4 Income from machinery, plant or furniture let on hire along with building (Composite
Rent) [Sec. 56(2)(iii)]
Generally, income from letting of building is taxable under the head Income from house property; but if such
letting is inseparable from letting of machinery, plant or furniture, then income from such letting is charged to tax
under the head “Income from other sources” if not taxed under the head “Profits & gains of business or profession”.
[Further refer the chapter “Income from house property”].
Notes
1. If letting of such building alone is acceptable, then income from letting of building is taxable under the head
‘Income from house property’.
2. Mere knowledge of rent charged against each asset does not make it separable, unless and until the property
is separately letable.

2.5.5 Deductions allowed against income u/s 56(2)(ii) & 56(2)(iii) [Sec. 57(ii) & (iii)]
By virtue of sec. 57(ii) and (iii), following expenditures are deductible from such income:
1. Current repairs shall be allowed as deduction [as per sec. 30(a)(ii) or 31(i)].
2. Insurance premium paid for machinery, plant, furniture or building [as per sec. 30(c) or 31(ii)]
3. Depreciation and unabsorbed depreciation (as per sec. 32).
4. Any other revenue expenditure expended, during the previous year, wholly and exclusively for earning such
income.

2.5.6 Family pension


Meaning: Family pension means a regular monthly amount payable by the employer to a person belonging to the
family of a deceased employee (e.g. widow or legal heirs of a deceased employee)
Tax Treatment: It is taxable under the head “Income from other sources” after allowing standard deduction.
Standard Deduction [Sec. 57(iia)]
Minimum of: 1/3rd of such pension; or ₹ 15,000.
Relief u/s 89 on arrears of family pension
Relief is available on arrears of family pension received by the family member of a deceased employee, as in case
of arrears/advance salary.

The Institute of Cost Accountants of India 329


Direct Taxation

Notes
Lump-sum payment made gratuitously or by way of compensation or otherwise to the widow or other legal heirs
of an employee, who dies while still in service, is non-taxable income.
Ex-gratia payment made to the widow or other legal heir of an employee, who dies while still in active service
would not be taxable as income provided it is paid by the Central Government or State Government or local
authority or Government or public sector undertaking.

Illustration 105:
Sunder died on 31st July 2023 while being in Central Government service. In terms of rules governing his service,
his widow Mrs. Sunder is paid a family pension of ₹ 10,000 p.m. and dearness allowance of 40% thereof. State
whether the amount of family pension is assessable in her hands, and if so, under what head of income. Can she
claim any relief/deduction on such receipt? Compute taxable income for the assessment year 2024-25 and tax
thereon.
Solution :
Computation of gross total income of Mrs. Sunder for the A.Y.2024-25

Particulars Details Amount


Income from other sources
Family pension [(₹ 10,000 + ₹ 4,000) x 8] [From 01-08-2023 to 31-03-2024] 1,12,000
Less: Standard deduction being minimum of the following:
a) 1/3rd of the pension 37,333
b) Statutory limit 15,000 15,000
Total Income 97,000
Tax on above Nil
It is assumed that other income of Mrs. Sunder is nil.

2.5.7 Gift [Sec. 56(2)(x)] Amended

Immovable Property Movable asset at


Cash Immovable Property Movable asset
at concessional value concessional value

Following receipts by any person shall be considered as his income:

Category Nature of Conditions to be satisfied for Extent of Income Remarks


Receipt considering income
A Any sum a. During the previous year, such person The whole of the Aggregate amount
of money has received any sum of money aggregate value of of cash gift received
(cash, cheque, draft, etc.) from one such sum shall be during the period
or more persons considered as income shall be considered.
of that previous year.
b. Such sum is received without
consideration
c. The aggregate value of such receipt
during the previous year exceeds ₹
50,000

330 The Institute of Cost Accountants of India


Heads of Income

Category Nature of Conditions to be satisfied for Extent of Income Remarks


Receipt considering income
B Any a. During the previous year, such person The stamp duty value The limit of
immovable has received immovable property of such property ₹ 50,000/- is
property b. Such immovable property is received shall be considered applicable per
without consideration. as income of that incidence
c. The stamp duty value of such previous year.
property exceeds ₹ 50,000
d. Such asset is a capital asset in hands
of recipient.
C Any a. During the previous year, such person The stamp duty value The limit (₹
immovable has received immovable property of such property Less 50,000/- or 10%)
property b. Such immovable property is received consideration paid, is applicable per
for a consideration shall be considered incidence.
c. Stamp duty value of such property as income of the
exceeds such consideration previous year.
d. Such excess is more than the higher
of the following:
–– ₹ 50,000; or
–– 10% of the consideration
Note: Where the date of the agreement
fixing the amount of consideration for
the transfer of immovable property and
the date of registration are not the same,
the stamp duty value on the date of the
agreement may be taken. This benefit is
available only in a case where the amount
of consideration or a part thereof, has
been paid by of an account payee cheque
or an account payee bank draft or by use
of electronic clearing system through a
bank account or through other prescribed
electronic modes, on or before the date
of the agreement for the transfer of such
immovable property.
D Any a. During the previous year, such person The whole of the Aggregate amount
movable has received movable property from aggre­gate fair of gift received
property one or more persons market value of such during the period
property shall be shall be considered.
b. Such movable property is received
considered as income
without consideration
of the previous year.
c. The aggregate fair market value of
such receipts during the previous
year exceeds ₹ 50,000
d. Such asset is a capital asset in hands
of recipient.

The Institute of Cost Accountants of India 331


Direct Taxation

Category Nature of Conditions to be satisfied for Extent of Income Remarks


Receipt considering income
E Any During the previous year, such person has The aggregate fair Aggregate amount
movable received movable property from one or market value of of gift received
property more persons such property Less during the period
Such movable property is received for a consideration paid, shall be considered.
consideration. shall be considered
as income of the
Such consideration is less than the
previous year.
aggregate fair market value of the property
by an amount exceeding ₹ 50,000
Such asset is a capital asset in hands of
recipient.
Taxpoint:
a. The limit of ₹ 50,000/- is also for per category. In other words, one may receive cash gift of ₹ 35,000 and gift
in kind of ₹ 36,000 without attracting any tax.
b. In case of dispute in stamp duty valuation: Refer section 50C discussed in chapter ‘Capital Gains’

Provisions Illustrated
Cash Gift
a. Mr. Anand received cash gift of ₹ 1,00,000 from his friend Mr. Lathi as on 10-10-2023
₹ 1,00,000 shall be considered as income of Mr. Anand for A.Y. 2024-25
b. As on 12-11-2023, Mr. Ajay received cash gift of ₹ 25,000 each from his 3 friends (one of them is a non-
resident)

332 The Institute of Cost Accountants of India


Heads of Income

Since aggregate amount of gift exceeds ₹ 50,000, hence, entire amount of gift i.e., ₹ 75,000 shall be considered as
income of Mr. Ajay for A.Y. 2024-25
Gift of immovable property
a. As on 10-01-2024, Mr. Pati received a piece of land (stamp duty value is ₹ 75,000) from his friend Mr. Raja
without any consideration. Such land was acquired by Raja in 2001-02 for ₹ 10,000.
Consequence of such gift in hands of Mr. Pati
i. Since stamp duty value of the property exceeds ₹ 50,000, hence ₹ 75,000 shall be considered as income
of Mr. Pati u/s 56(2)(x) for A.Y. 2024-25.
ii. By virtue of amended provisions of sec.49(4), cost of acquisition of such land in hands of Mr. Pati shall
be ₹ 75,000. Suppose as on 03-06-2025, Mr. Pati sold such land for ₹ 2,00,000 (stamp duty value as on
that date is ₹ 2,00,000), computation of capital gain in hands of Mr. Pati for A.Y. 2026-27 are as under:
Particulars Amount
Sale Consideration 2,00,000
Less: Expenses on transfer Nil
2,00,000
Less: Cost of acquisition$ 75,000
Short term capital gain# 1,25,000
$
Since the transferred capital assets has been subject to income tax u/s 56(2)(x) in the A.Y.2024-25, the
cost of acquisition of such asset shall be deemed to be the value which has been taken into account for the
purpose of sec.56(2)(x) i.e., ₹ 75,000 – Sec.49(4). Further, it is to be noted that sec. 49(1) (i.e., in case of
gifted assets, cost of previous owner shall be considered as cost of acquisition in hands of the assessee) is
not applicable.
#
As per explanation 1 to sec. 2(42A), while determining holding period of capital asset, being asset
acquired through gift, period of holding of previous owner shall also be included. However, this rule is
not applicable in case where such gift is subject to income tax u/s 56(2)(x).
Reason for non-applicability of said explanation
The explanation 1 to sec. 2(42A), inter-alia, provides the following:
“(b) in the case of a capital asset which becomes the property of the assessee in the circumstances
mentioned in sub-section (1) of section 49, there shall be included the period for which the asset was held
by the previous owner referred to in the said section.”
Since the said explanation is applicable only in circumstances covered u/s 49(1). However, assets taxable
u/s 56(2)(x) is covered u/s 49(4).
Consequence of such gift in hands of Mr. Raja
Mr. Raja has transferred the capital asset by way of gift, which is not considered as transfer (sec. 47),
hence no tax is required to be paid by him. The segregation of gift u/s 49(1) or u/s 49(4) is applicable only
for determining cost of acquisition in the hands of transferee. However, sec. 47 (which provides the list
of transaction do not constitute transfer) does not provide any segregation.
b. On 10-10-2023, Dipak received a piece of land at Napasar from his friend Vikash (stamp duty value ₹ 46,000)
as a gift. Such land was acquired by Mr. Vikash in 2001 for ₹ 5,000/-. Further, as on 01-11-2023, Dipak also
received another piece of land at Bikaner (stamp duty value ₹ 30,000) without any consideration from Anil
(acquired in 2021-22 for ₹ 10,000/-)

The Institute of Cost Accountants of India 333


Direct Taxation

On 31-03-2024, Mr. Dipak sold land at Napasar to Kedar for ₹ 30,000/- and land at Bikaner to Nath for ₹
90,000/-. Stamp duty value of such land as on the date of sale is ₹ 50,000/- (Napasar) and ₹ 85,000/- (Bikaner).
Consequence of such gift in hands of Mr. Dipak
1. Applicability of sec.56(2)(x): Nothing shall be taxable as stamp duty value (at the time of gift) of
property at each occasion does not exceed ₹ 50,000/-.
2. Cost of acquisition: Since the value of the asset is not subject to income-tax u/s 56(2)(x), hence cost of
acquisition of previous owner shall be considered as cost of acquisition of such assets in hands of Dipak.
Consequently, cost of acquisition of land at Napasar is ₹ 5,000/- and that of land situated in Bikaner is
₹ 10,000/-
3. Period of Holding: Since the value of the asset is not subject to income-tax u/s 56(2)(x), hence period
of holding of previous owner shall also be included in the holding period of Dipak.
4. Computation of capital gain on transfer: Computation of capital gain in hands of Dipak are as under:
Particulars Land at
Napasar Bikaner
Sale Consideration (Being higher of the following – Sec.50C)
- Actual Consideration 30,000 90,000
- Stamp duty value 50,000 85,000
50,000 90,000
Less: Expenses on transfer Nil Nil
50,000 90,000
Less: Indexed Cost of acquisition [₹ 5,000 * 348 / 348]* 5,000 -
Less: Cost of acquisition - 10,000
Long term capital gain 45,000 -
Short term capital gain - 80,000
Consequence of such gift in hands of Mr. Vikash and Mr. Anil
Mr. Vikash and Mr. Anil has transferred the capital asset by way of gift, which is not considered as transfer
(sec. 47), hence no tax is required to be paid by him.

Acquisition of immovable property at concessional amount


a. As on 10-01-2024, Tipu purchased a piece of land (stamp duty value is ₹ 75,000) from his friend Mr. Uday
against consideration of ₹ 30,000. Mr. Uday has acquired such land for ₹ 25,000 in F.Y. 2006-07. On 10-04-
2025, Tipu sold such land for ₹ 1,75,000 (stamp duty value ₹ 1,60,000/-).
Consequences in hands of Mr. Tipu
1. Applicability of sec.56(2)(x): Since the difference between stamp duty value of the land (as on 10-01-
2024) and actual consideration does not exceed ₹ 50,000, hence, the provision of sec. 56(2)(x) is not
applicable.
2. Cost of acquisition: ₹ 30,000 shall be considered as cost of acquisition of such land in hands of Tipu.

* Alternatively, index benefit may be taken from the date of acquisition of such land by Vikash

334 The Institute of Cost Accountants of India


Heads of Income

3. Capital Gain: Computation of Capital gain for A.Y. 2026-27 are as under.

Particulars Amount
Sale Consideration (Higher of ₹ 1,75,000 and ₹ 1,60,000 – Sec.50C) 1,75,000
Less: Expenses on transfer Nil
1,75,000
Less: Cost of acquisition 30,000
Short term capital gain #
1,45,000
#
It is to be noted that this is not the case of gift, hence period of holding commenced from 10-01-2024.
Consequences in hands of Mr. Uday
Computation of capital gain in hands of Mr. Uday for A.Y. 2024-25

Particulars Amount
Sale Consideration (Higher of ₹ 75,000 and ₹ 30,000 – Sec.50C) 75,000
Less: Expenses on transfer Nil
75,000
Less: Indexed cost of acquisition [₹ 25,000 × 348 / 122] 71,311
Long term capital gain 3,689

b. As on 10-02-2024, Mundhra Services Pvt Ltd purchased a piece of land (stamp duty value is ₹ 7,50,000) from
one of the friend, Mita, of the Director, against consideration of ₹ 3,90,000. Mita has acquired such land for
₹25,000 in 2000 (Fair Market Value as on 1-4-2001 was ₹ 32,000). On 10-04-2025, the company has sold such
land for ₹ 10,75,000 (stamp duty value ₹ 12,50,000/-).
Consequences in hands of Mundhra Services Pvt Ltd
1. Applicability of sec.56(2)(x): Since the difference between stamp duty value of the land (as on 10-02-
2023) and actual consideration exceeds ₹ 50,000, hence, the provision of sec. 56(2)(x) is applicable.
Consequently, ₹ 3,60,000/- shall be considered as ‘Income from Other Sources’ in hands of the company
in the A.Y.2024-25.
2. Cost of acquisition: ₹ 7,50,000 shall be considered as cost of acquisition of such land in hands of the
company.
3. Capital Gain: Computation of Capital gain for A.Y. 2026-27 are as under
Particulars Amount
Sale Consideration (Higher of ₹ 10,75,000 and ₹ 12,50,000 – Sec.50C) 12,50,000
Less: Expenses on transfer Nil
12,50,000
Less: Cost of acquisition 7,50,000
Short term capital gain #
5,00,000
#
It is to be noted that this is not the case of gift, hence period of holding commenced from 10-02-2024.

The Institute of Cost Accountants of India 335


Direct Taxation

Consequences in hands of Mita


Computation of capital gain in hands of Mita for A.Y. 2024-25

Particulars Amount
Sale Consideration (Higher of ₹ 7,50,000 and ₹ 3,90,000 – Sec.50C) 7,50,000
Less: Expenses on transfer Nil
7,50,000
Less: Indexed cost of acquisition [₹ 32,000 x 348 / 100] 1,11,360
Long term capital gain 6,38,640

c. On 10-10-2023, Suresh purchased a piece of land from his friend Anuj (stamp duty value ₹ 46,000) for
₹ 30,000. Further, as on 01-11-2023, he also purchased house property (stamp duty value ₹ 1,50,000) from
Ajay for ₹ 1,00,000. Further, as on 01-01-2024, his another friend Jitu give him a house property without any
consideration (stamp duty value ₹ 35,000). Jitu purchased such property on 10-04-2001 for ₹ 5,000/-
Consequences in hands of Suresh
1. Applicability of sec.56(2)(x): Nothing shall be taxable as the difference between stamp duty value of
property and actual consideration at each occasion does not exceed ₹ 50,000/-.
2. Cost of acquisition and period of holding: Since the value of the asset is not subject to income-tax u/s
56(2)(x), hence cost of acquisition and period of holding thereof are as under:
Property Cost of acquisition Period of holding commenced from
Land purchased from Anuj ₹ 30,000 10-10-2023
House purchased from Ajay ₹ 1,00,000 01-11-2023
House received from Jitu Cost of previous owner 10-04-2001 (However, index benefit is
i.e. ₹ 5,000 available from F.Y. 2023-24)
Consequences in hands of Anuj, Ajay and Jitu
 Anuj and Ajay: Capital gain shall be computed. Provision of sec.50C is applicable.
 Jitu: No treatment
d. On 10-10-2023, Sonam purchased a piece of land from her friend Shweta (stamp duty value ₹ 12,54,000) for
₹ 12,00,000.
Consequences in hands of Sonam
i. Applicability of sec.56(2)(x): Since the difference between actual consideration and stamp duty value
does not exceed 10% of actual consideration (being ₹ 1,20,000 i.e., 10% of ₹ 12,00,000), hence nothing
shall be taxable u/s 56(2)(x) even though the difference between actual consideration and stamp duty
value exceeds ₹ 50,000.
ii. Cost of acquisition: ₹ 12,00,000
Consequences in hands of Shweta
Similarly, ₹ 12,00,000 being actual consideration shall be considered as full value of consideration.
e. On 10-10-2022, Priya purchased a piece of land from her friend Monica (stamp duty value ₹ 4,45,000) for
₹ 4,00,000.

336 The Institute of Cost Accountants of India


Heads of Income

Consequences in hands of Priya


1. Applicability of sec.56(2)(x): Nothing shall be taxable as the difference between stamp duty value of
property and actual consideration does not exceed ₹ 50,000/-.
2. Cost of acquisition: ₹ 4,00,000
Consequences in hands of Monica
₹ 4,45,000 being stamp duty value shall be considered as full value of consideration as stamp duty value
exceeds 110% of actual consideration.
–– Actual consideration ₹ 4,00,000
–– 110% of above ₹ 4,40,000
–– Stamp duty value ₹ 4,45,000
Provision of sec. 50C do not have criteria like minimum difference should be ₹ 50,000.

Gift of movable property


a. As on 10-01-2024, Narendra received gold necklace (fair value is ₹ 1,85,000) from Vivek without any
consideration.
Consequences in hands of Narendra
i. Applicability of sec. 56(2)(x): Since fair market value of gift exceeds ₹ 50,000, hence the provision is
applicable and consequently ₹ 1,85,000 shall be taxable as income from other sources in A.Y. 2024-25.
ii. Cost of acquisition of such necklace: ₹ 1,85,000/- being the value taken for computing income u/s
56(2)(x)
iii. Period of holding: Period of holding will be commenced from 10-01-2024.
Consequences in hands of Vivek
Gift is not considered as transfer; hence nothing shall be taxable in his hands.
b. On 10-10-2023, Mohan received shares from his friend Uttam (fair value ₹ 36,000 and not acquired through
ESOP) as a gift. Further, as on 01-11-2023, he also received gold chain (fair value ₹ 30,000) without any
consideration from another friend.
Consequences in hands of Mohan
i. Applicability of sec. 56(2)(x): Since aggregate fair market value of both gifts exceeds ₹ 50,000, hence
the provision is applicable and consequently ₹ 66,000 shall be taxable as income from other sources in
A.Y. 2024-25.
It is to be noted that in case of immovable property, limit of ₹ 50,000/- is applicable on per incidence.
However, in case of movable property (and cash), the limit of ₹ 50,000/- is applicable for all gift received
during the previous year.
ii. Cost of acquisition of such necklace: ₹ 36,000/- (for shares) and ₹ 30,000 (for gold chain) being the
value taken for computing income u/s 56(2)(x)
iii. Period of holding: Period of holding will commence from 10-10-2023 (for shares) and 01-11-2023 (for
gold chain)
Consequences in hands of Friends
Gift is not considered as transfer, hence nothing shall be taxable in their hands.

The Institute of Cost Accountants of India 337


Direct Taxation

Acquisition of movable property at concessional amount


a. On 10-10-2023, Kamal purchased jewellery from Kishore (fair value ₹ 46,000) for ₹ 30,000. Kishore has
acquired such jewellery in 10-04-2023 for ₹ 25,000
Consequences in hands of Kamal
i. Applicability of sec. 56(2)(x): Since fair market value of jewellery does not exceed ₹ 50,000, hence the
provision is not applicable.
ii. Cost of acquisition of such necklace: ₹ 30,000/- (for jewellery) being the actual consideration.
iii. Period of holding: Period of holding will commence from 10-10-2023
Consequences in hands of Kishore: Computation of capital gain in hands of Kishore are as under:

Particulars Kishore
Sale Consideration 30,000#
#
Sec. 50C is not applicable in case of movable property
Less: Expenses on transfer Nil
30,000
Less: Cost of acquisition 25,000
Short term capital gain 5,000
b. On 10-10-2023, Janak purchased jewellery from his friend Giri (fair value ₹ 66,000) for ₹ 30,000. Further, as
on 01-11-2023, he also purchased silver utensil (fair value ₹ 1,00,000) from another friend for ₹ 75,000.
Consequences in hands of Janak
Applicability of sec. 56(2)(x): The difference between fair market value and actual consideration are as
follow:

Particulars Amount Amount


Fair Market Value of
- Jewellery 66,000
- Silver utensil 1,00,000
1,66,000
Less: Actual consideration
- Jewellery 30,000
- Silver utensil 75,000 1,05,000
Difference between fair market value and actual consideration 61,000
The aforesaid value exceeds ₹ 50,000 hence, sec. 56(2)(x) is applicable, consequently ₹ 61,000 shall be
taxable
Cost of acquisition of such assets shall be fair market value considered for aforesaid computation.
Consequences in hands of Friends

Capital gain shall be computed. However, it is to be noted that sec. 50C is not applicable on movable asset

338 The Institute of Cost Accountants of India


Heads of Income

Exceptions
This section shall not apply to any sum of money or any property received
1. from any relative#.
#
Relative here means—
In case of an individual

i. spouse of the individual;


ii. brother or sister of the individual;
iii. brother or sister of the spouse of the
individual;
iv. brother or sister of either of the
parents of the individual;
v. any lineal ascendant or descendant
of the individual;
vi. any lineal ascendant or descendant of the spouse of the individual;
vii. spouse of the person referred to in clauses (ii) to (vi).
In case of HUF: Any member thereof
Example 1: Relatives means: Suppose Mr. Vikash is an assessee
1. Wife of Mr. Vikash.
2. Brother or sister of Mr. Vikash.
3. Brother or sister of wife or Mr. Vikash.
4. Brother or sister of father of Mr. Vikash. Brother or sister of mother of Mr. Vikash.
5. Great grandfather, grandfather and father of Mr. Vikash.
6. Son, grandson and great grandson of Mr. Vikash.
7. Great grandfather, grandfather and father of wife of Mr. Vikash.
8. a Brother’s wife or sister’s husband of Mr. Vikash.
b. Brother’s wife or sister’s husband of wife of Mr. Vikash.
c. Brother’s wife or sister’s husband of father of Mr. Vikash.
d. Brother’s wife or sister’s husband of mother of Mr. Vikash.
e. Great grandmother, grandmother and mother of Mr. Vikash.
f. Son’s wife, great grandson’s wife, great grandson’s wife of Mr. Vikash.
g. Great grandmother, grandmother, and mother of wife of Mr. Vikash.
2. on the occasion of the marriage of the individual (whether gift is received from relative or outsiders).
3. under a will or by way of inheritance.
4. in contemplation of death of the payer or donor.
5. from local authority

The Institute of Cost Accountants of India 339


Direct Taxation

6. by an individual, from any person, in respect of any expenditure actually incurred by him on his medical
treatment or treatment of any member of his family, for any illness related to COVID-19 subject to prescribed
conditions
7. by a member of the family of a deceased person,—
A. from the employer of the deceased person (without limit); or
B. from any other person or persons (aggregate upto ₹ 10 lakh),
where
i. the cause of death of such person is illness related to COVID-19
ii. the payment is received within 12 months from the date of death of such person; and
iii. any other prescribed conditions are satisfied
Taxpoint: Family here means -
 Spouse and children of the individual; and
 Parents, brothers and sisters of the individual, who are wholly or mainly dependent on him.
8. from or by any fund or foundation or university or other educational institutions or hospital or other medical
institutions or any trust or institution referred u/s 10(23C), provided it is not received by any person referred
to in sec. 13(3).
9. from or by any trust or institution registered u/s 12A or 12AA or 12AB, provided it is not received by any
person referred to in sec. 13(3).
10. from an individual by a trust created or established solely for the benefit of relative of the individual.
11. by way of distribution at the time of total or partial partition covered u/s 47(i)
12. by way of transactions in the nature of amalgamation or demerger covered u/s 47(iv) or (v) or (vi) or (via) or
(viaa) or (vib) or (vic) or (vica) or (vicb) or (vid) or (vii) or (viiac) or (viiad) or (viiae) or (viiaf).
13. from such class of persons and subject to such conditions, as may be prescribed.
14. Exemption of Compensation on account of Disaster [Sec. 10(10BC)]: Any amount received or receivable
from the Central Government or a State Government or a local authority by an individual or his legal heir by
way of compensation on account of any disaster, except the amount received or receivable to the extent such
individual or his legal heir has been allowed a deduction under this Act on account of any loss or damage
caused by such disaster.
Other Points
1. Fair market value of a property, other than an immov­able property, means the value determined in accordance
with the method as may be prescribed;
2. Property (including virtual digital assets) means the following capital asset of the assessee, namely

a. Immovable property being land or building or both b. Shares & securities


c. Archaeological collections d. Paintings e. Jewellery f. Drawings
g. Sculptures h. Any work of art i. Bullion
Taxpoint:
 Property includes virtual digital asset like crypto currency.

340 The Institute of Cost Accountants of India


Heads of Income

 As per sec. 2(47A), virtual digital asset means:


a. any information or code or number or token (not being Indian currency or foreign currency), generated
through cryptographic means or otherwise, by whatever name called, providing a digital representation
of value exchanged with or without consideration, with the promise or representation of having inherent
value, or functions as a store of value or a unit of account including its use in any financial transaction or
investment, but not limited to investment scheme; and can be transferred, stored or traded electronically;
b. a non-fungible token or any other token of similar nature, by whatever name called;
c. any other digital asset, as the Central Government may, specify.
However, Central Government may exclude any digital asset from the definition of virtual digital asset
subject to such conditions as may be specified therein.
 Property does not include furniture, clothes, etc. (provided it does not fall in the definition of Jewellery).
 If an assessee receives aforesaid assets without consideration (or against inadequate consideration in some
cases) as stock in trade, the provision of this section shall not apply.
3. Stamp duty value means the value adopted or assessed or assessable by any authority of the Central Government
or a State Government for the purpose of payment of stamp duty in respect of an immovable property.

2.5.8 Share premium in excess of fair market value [Sec. 56(2)(viib)] Amended
Where a company, not being a company in which the public are substantially interested, receives, in any previous
year, from any person being a resident, any consideration for issue of shares that exceeds the face value of such
shares, the aggregate consideration received for such shares as exceeds the fair market value of the shares shall be
treated as income of the company.

2.5.9 Income by way of interest received on compensation or on enhanced compensation


[Sec.56(2)(viii)]
Interest received by an assessee on compensation or on enhanced compensation, as the case may be, shall be
deemed to be the income of the year in which it is received.
Tax Treatment: It is taxable under the head “Income from other sources” after allowing standard deduction of
50% of such income.
E.g., During the previous year 2023-24, Mr. X received ₹ 65,000 (₹ 45,000 pertaining to the previous year 2022-
23) as interest on delayed compensation. Such interest after allowing standard deduction shall be considered as an
income of the previous year 2023-24 (irrespective of previous year to which such interest pertains). Thus, ₹ 32,500
(i.e., ₹ 65,000 – ₹ 32,500 being standard deduction @ 50%) shall be considered as income of the previous year
2023-24.

2.5.10 Employee’s contribution towards staff welfare fund or scheme [Sec. 56(2)(ic)]
Any amount received or deducted by an employer from employee towards any -
 Provident Fund;
 Superannuation Fund;
 Fund set up under the provisions of Employee’s State Insurance Act, 1948; or
 Other fund set up for the welfare of such employees,

The Institute of Cost Accountants of India 341


Direct Taxation

shall be treated as income of the employer under this head if not taxable under the head “Profits & gains of business
or profession”. Subsequently, when such sum is credited by the employer to the employee’s account in the relevant
fund on or before the due date prescribed under the relevant Act, then deduction of equal amount is available.
Taxpoint

¾¾ If employee’s contribution taken and deposited No treatment


within time
¾¾ If employee’s contribution taken and not deposited Taxable as income from other source if not taxable as
within time business income.

2.5.11 Interest on Securities [Sec. 56(2)(id)]


As per sec. 2(28B), “interest on securities” means -
a. Interest on any security of the Central Government or a State Government;
b. Interest on debentures or other securities issued by or on behalf of -
¾¾ a local authority; or
¾¾ a company; or
¾¾ a corporation established by a Central, State or Provincial Act.
Tax treatment

Case Treatment

When the securities are held as stock-in-trade Interest on securities is charged to tax under the head
‘Profits & gains of business or profession’
When the securities are held otherwise than as stock- Interest on such securities is charged to tax under the
in-trade. head ‘Income from other sources’
Chargeability: It is taxable as per cash basis or due basis, depending on the method of accounting regularly
followed by the assessee. However, where no method of accountancy is followed, then it shall always be taxable
on due basis.
Due date of interest: Interest shall be taxable when such interest falls due. Interest on securities shall not accrue
on day-to-day basis. It accrues on the due date of interest as prescribed by the issuing authority. Entire interest shall
be charged in the hands of assessee who holds security on such date (irrespective of the date of acquisition of such
security). E.g. Mr. X acquired debenture of A Ltd. as on 7/9/2022. Due date of interest on debenture is 31st March
every year. Interest on such debenture shall be taxable in hands of Mr. X for the whole year.
Taxpoint: In case of death of assessee before the due date, the income of the whole year shall be taxable in the
hands of recipient of income.
Grossing-up of interest if it is given net of TDS: Sometime in the problem, Interest income received is given
rather than Interest earned. In such case, students are required to gross up the interest income received. Relation
between interest income earned and interest income received is as under:
Interest income received = Interest income earned – Tax deducted at source on such income#
Accordingly, Gross income from interest = [Interest received * 100/ (100 - TDS rate i.e.10%)]
#
Tax is required to be deducted @ 10%. However, in certain cases, tax is not required to be deducted (For further
details refer Chapter Tax Deducted at Source)

342 The Institute of Cost Accountants of India


Heads of Income

Expenditure allowed as deductions


By virtue of sec. 57(i) and (iii), the following expenditure are deductible from interest income:
a. Collection expenditure
b. Interest on loan
c. Any other expenditure
Taxpoint: However, any expenses covered u/s 58 shall not be allowed
Avoidance of tax by certain transaction in securities [Sec. 94]
1. Bond Washing Transactions [Sec. 94(1)]: Interest on securities shall not accrue on day-to-day basis. It accrues
on the due date of interest as prescribed by issuing authority. Entire interest shall be charged in the hands of
assessee who holds security on such date (irrespective of the date of acquisition of such security). Tax liability
may be evaded by transferring securities just before the due date of interest (interest includes dividend) to any
person (like friend or relative who has low income) and reacquiring the same, after the interest is received by
the transferee. With this practice, income, which should have been charged at higher rate, shall be charged at
lower rate or nil rates. To avoid these practices, sec. 94(1) provides that -
where an assessee transfers the securities before the due date of interest and reacquires the same, then the
interest received by the transferee will be deemed to be the income of the transferor.
E.g: Mr. X transferred 1,000 10% debentures (due date of interest of such debenture is 31st March every year),
to his brother Mr. Y on 27/03/2023 to evade tax. Such security is repurchased by him on 5/04/2023. Interest
for the previous year 2022-23, though received by Mr. Y shall be taxable in hands of Mr. X due to sec. 94(1).
2. Transaction relating to securities resulting into less or no income: Sec. 94(2) provides that -
¾¾ Where a person has had at any time during the previous year any beneficial interest in securities; &
¾¾ The result of any transaction relating to such securities is that, either no income is received by him or the
income received by him is less than the income from such securities on day to day basis,
- then the income from such securities for such year shall be deemed to be the income of such person.
Exceptions to sec. 94(1) & 94(2)[Sec. 94(3)]
However, nothing provided in sec. 94(1) and (2) shall be applicable, if the owner of securities satisfies the AO that -
i. There has been no avoidance of income tax or the avoidance of income tax is exceptional and not systematic;
and
ii. There was not any avoidance of income tax u/s 94 during 3 years preceding the previous year.
Note: The Assessing Officer may (by notice in writing) require any person to furnish within such time (not less than
28 days), such particulars as he considers necessary in respect of all securities -
 of which such person was the owner; or
 in which he had a beneficial interest at any time during the period specified in the notice [Sec. 94(6)]

2.5.12 Dividend [Sec. 2(22)]


Dividend, in general, means the amount received by a shareholder (whether in cash or in kind) in proportion to his
shareholding in a company whether out of past or present income; or taxable or exempted income; or revenue or
capital income. However, the Income-tax Act gives an inclusive definition of dividend.
As per sec. 2(22), the following payments or distributions by a company to its shareholders are deemed as dividends
to the extent of accumulated profits of the company:

The Institute of Cost Accountants of India 343


Direct Taxation

a. Any distribution of accumulated profits (whether capitalized or not), which results in the release of assets of
the company [Sec. 2(22)(a)]
Notes
1. Treatment of bonus share: Bonus share declared by the company to its equity share-holders shall not
be treated as dividend as there is no release of asset.
2. Valuation: In case of release of asset other than cash, the market value of the asset and not the book
value shall be considered as deemed dividend in the hands of shareholder.
3. Exception: Above provision is not applicable in case of amalgamation
b. Any distribution of -
¾¾ Debenture, debenture-stock, deposit certificates in any form whether with or without interest to its
shareholders (equity as well as preference); and
¾¾ Shares to preference shareholders by way of bonus,
- to the extent to which company possess accumulated profit (whether capitalized or not) [Sec. 2(22)(b)]
Taxpoint: Such distribution shall be treated as dividend in the hands of recipient even there is no release of
assets of the company.
c. Distribution made on liquidation to the extent to which company possess accumulated profit immediately
before liquidation (whether capitalized or not) [Sec. 2(22)(c)]
Notes
1. Payment to preference shareholder: Any distribution of asset made in respect of preference share
(issued for full cash consideration) shall not be treated as dividend.
2. Distribution out of profits earned after liquidation: Any distribution made out of profits earned after
the date of liquidation shall not be treated as dividend u/s 2(22)(c).
3. Exception: Above provision is not applicable in case of amalgamation
d. Distribution made on reduction of capital of the company to the extent it possesses accumulated profit (whether
capitalized or not) [Sec. 2(22)(d)]
Exception: Any distribution in respect of preference share shall not be treated as dividend.
e. Any payment (whether in cash or in kind) by a company in which public are not substantially interested to the
extent of accumulated profit (excluding capitalized profit) -
i. by way of loan or advance to its equity shareholder, who is beneficial owner of the shares, holding not
less than 10% of voting power in the company (hereinafter referred as specified shareholder);
Taxpoint
The shareholder should have atleast 10% voting power as on the date of when loan is given.
ii. by way of loan or advance to a concern (whether HUF, Firm, AOP, BOI or a Company) of which such
specified shareholder is a member or partner at the time of such payment and has substantial interest# in
such concern; or

Substantial interest: A person shall be deemed to have substantial interest in a concern, if he is beneficially
#

344 The Institute of Cost Accountants of India


Heads of Income

entitled to not less than 20% of income of such concern (20% of voting power in case of company) at any
time during the previous year.
Taxpoint:
 The clause is applicable where –
a. Such shareholder holds at least 10% voting right in the payer company as on the date when
loan is given;
b. Such specified shareholder is a member or partner in the payee concern as on the date of loan;
and
c. Such shareholder should have atleast 20% share in profit / voting power in the payee concern
at any time during the previous year.
iii. by way of loan or advance to any person on behalf of or for the benefit of such specified shareholder
[Sec. 2(22)(e)]
Notes
a. No exemption on repayment of such loan: Loan to a specified-shareholder is treated as deemed
dividend even if such loan or any part of such loan was refunded before the end of previous year.
b. Advance means an advance given in the nature of loan (with or without interest), which shall be refunded
to the company. Trade advances which are in the nature of commercial transactions would not fall within
the ambit of the word ‘advance’ in sec. 2(22)(e).
Exceptions
a. Set-off of loan with forthcoming declaration of dividend: In case the loan granted to member is
adjusted with the forthcoming dividend, then such dividend (newly declared) shall not be taxable in the
hands of shareholder who adjusted such loan.
b. Advance or loan in the ordinary course of business: Sec. 2(22)(e) is not applicable if an advance
or loan is made by a company in the ordinary course of its business, where the lending of money is
substantial part of the business of the company.

Meaning of Accumulated profit


For the purpose of dividend u/s 2(22), accumulated profit shall be:

Case Accumulated profit


Company which is not in liquidation Up to the date of distribution or payment
Company which is in liquidation Up to the date of liquidation
Dividend u/s 2(22)(e) Up to the date of grant of such loan or advance.

Taxpoint: In the case of an amalgamated company, the accumulated profits, whether capitalised or not, or loss,
as the case may be, shall be increased by the accumulated profits, whether capitalised or not, of the amalgamating
company on the date of amalgamation.
Exceptions
Following payments shall not be treated as ‘Dividend’ -
1. Any payment made by a company on buy-back of shares

The Institute of Cost Accountants of India 345


Direct Taxation

2. Any distribution of shares made in accordance with the scheme of demerger


Tax treatment
Dividend (from foreign company, domestic company or co-operative society) is taxable in the hands of the recipient.
Basis of charge [Sec. 8]
Dividend is not taxable on the basis of accounting method, but charged as per the following schedule -

Case Year of taxability


Normal Dividend Year in which it is declared by the company
Interim Dividend Year in which amount of dividend is unconditionally made available
Deemed Dividend Year in which it is distributed or paid by the company

Place of accrual [Sec. 9(1)(iv)]


Dividend shall be deemed to accrue or arise in India, if it is declared by an Indian company.

Deduction available from dividend income [Sec. 57]


No deduction shall be allowed from the dividend income, or income in respect of units of a Mutual Fund other than
deduction on account of interest expense. Further, in any previous year such deduction shall not exceed 20% of the
dividend income, or income in respect of such units, included in the total income for that year.
Taxpoint: Apart from interest expenses (max. upto 20% of dividend income), no deduction shall be allowed from
dividend income.

2.5.13 Specific disallowance [Sec. 58]


Following expenditures shall not be deducted from any income under this head:
1. Any personal expenses of the assessee. [Sec. 58(1)(a)(i)]
2. Any interest which is payable outside India on which tax has not been deducted at source. [Sec. 58(1)(a)(ii)]
3. Any salary payable outside India on which tax has not been deducted at source. [Sec. 58(1)(a)(iii)]
4. 30% of any payment made to a resident on which TDS provision is applicable without deducting TDS as
referred u/s 40(a)(ia)
5. Any amount paid as Wealth tax or Income tax. [Sec. 58(1A)]
Taxpoint: Interest paid on amounts borrowed for meeting tax liability is not deductible.
6. Any amount specified u/s 40A like -
¾¾ payment to relative in excess of requirement; or
¾¾ payment in excess of ₹ 10,000 otherwise than an account payee cheque/draft/specified electronic modes
[Sec.58(2)]
7. No deduction in respect of any expenditure shall be allowed in computing the income by way of any winnings
from lotteries, crossword puzzles, races including horse races, card games and other games of any sort or
form, gambling or betting of any form or nature, etc. taxable under the head “Income from other sources”.
[Sec. 58(4)]

346 The Institute of Cost Accountants of India


Heads of Income

Note: Above provision shall not apply in computing the income of an assessee, being the owner of horses
maintained by him for running in horse races, from the activity of owning and maintaining such horses.

2.5.14 Deemed Profits [Sec. 59]


Sec. 59 provides that where -
a. An allowance or deduction has been allowed for any year in respect of loss, expenditure or trading liability
incurred by the assessee; and
b. Subsequently, any amount is obtained, as revocation of such loss, expenditure or remission of liability, whether
in cash or in any other manner, during any previous year,
- then such amount received or amount remitted shall be charged to tax.
Note: Above provision holds good even in case of succession or inheritance.

General Illustrations
Illustration 106:
Compute taxable income under the head Income from other sources of Mrs. X from the following data:

Particulars Amount
Private tuition fee received 10,000
Winning from lottery 2,000
Award from KBC (a TV show) [Gross] 3,20,000
Pension from employer of deceased husband 25,000
Interest on bank deposit 25,000
Directors fee (Gross) 5,000
Letting out of vacant land 25,000
Remuneration for checking the examination copy of employer’s school 10,000
Remuneration for checking the examination copy of C.A 10,000
Income tax refund 5,000
Interest on income tax refund 100
Composite rent (related expenditures are ₹ 5,000) 10,000
Rent on sub-letting of house property (rent paid to original owner ₹ 12,000) 20,000
Income tax paid 2,000
Payment made for personal expenses 18,000
Payment made to LIC as premium 2,000

The Institute of Cost Accountants of India 347


Direct Taxation

Solution :
Computation of income of Mrs. X under the head Income from other source for the A.Y. 2024-25
Particulars Details Amount
Private tuition fee received 10,000
Casual income
Winning from lottery 2,000
Award from KBC (a TV show) [Gross] 3,20,000
Pension 25,000
Less: Standard deduction
1)1/3rd of amount received (i.e. ₹ 8,333)
2)₹ 15,000 8,333 16,667
Interest on bank deposit 25,000
Director’s fee 5,000
Letting out of vacant land 25,000
Remuneration for checking the examination copy of employer school Taxable as Salary ---
Remuneration for checking examination copy of CA 10,000
Income tax refund Not an income ---
Interest on income tax refund 100
Composite rent 10,000
Less: Expenditure 5,000 5,000
Rent on sub-letting of house property 20,000
Less: Rent paid to original owner 12,000 8,000
Income from Other Source 4,26,767
Note: Payment of income tax and personal expenses is not deductible in any case.

Illustration 107:
Shri Anil follows cash basis of accounting and has furnished the Receipts & Payment A/c of previous year 2023-24
for computing his income:

Particulars Receipts Payments


Interest on listed debenture of A Ltd. 16,200
Letting of building & machinery @ ₹ 15,000 p.m. under a composite lease 1,50,000
Collection charges 1,000
Repairs 5,000
Capital repairs 16,000
Interest paid outside India without deducting tax on loan taken for 8,000
construction of building
Gift from father 6,000
Ground rent received (related to financial year 2021-22) 600
The following additional information are also provided -
Allowable depreciation on Building and Machinery - ₹ 4,000
Fire Insurance on Building and Machinery (not paid) - ₹ 1,000

348 The Institute of Cost Accountants of India


Heads of Income

Solution :
Computation of income from other sources of Anil for A.Y.2024-25
Particulars Details Details Amount Amount
Interest on debenture of A Ltd. ₹ 16,200 / 90% 18,000
Interest received on letting of assets 1,50,000
Less: Expenses paid
Collection charges 1,000
Repair 5,000
Capital repairs3 Nil
Depreciation 4,000 10,000 1,40,000
Gift from father [As received from relative] Nil
Ground rent received 600
Income from other source 1,58,600
Notes
Since assessee follows cash basis of accounting, hence, income shall be chargeable and expenditure shall be
allowed on cash basis.
Debenture income required to be grossed up.
Capital repairs are not allowed.
Interest paid outside India without deducting tax at source shall not be deductible expenditure.

Quick MCQs:-

1. Income under the head “Income from Other Sources” is taxable on –


(a) Due Basic
(b) Receipt Basis
(c) On the basis of method of accounting regularly employed by the Assessee
(d) None of the above.

2. Income from Lottery, Crossword Puzzle, Race, Card Games etc., are taxable at:
(a) Normal Slab Rate of Income Tax like any other income
(b) Flat Rate of 20%
(c) Flat Rate of 30% plus Surcharge, if applicable,
(d) Flat Rate of 30% plus Surcharge of 10%

3. Winning from Lotteries, Crossword Puzzles, Horse Races & Other Races, Card Game, etc. are casual
income & hence-
(a) fully exempt
(b) exempt upto D 5,000
(c) fully taxable
(d) None of above

The Institute of Cost Accountants of India 349


Direct Taxation

4. Gift of D 5,00,000 received on 10th July 2023 through Account Payee Cheque from a non-relative
regularly assessed to income-tax, is-
(a) A Capital Receipt not chargeable to tax
(b) Chargeable to tax as Income from Other Sources
(c) Chargeable to tax a Business Income
(d) Exempt upto D 50,000 & balance chargeable to tax under Other Sources.

5. Adi received D 70,000 from his friend on the occasion of his birthday.
(a) Entire amount of D 70, 000 is taxable
(b) D 25,000 is taxable
(c) The entire amount is exempt
(d) None of the above

6. Family Pension received by a widow of a member of the armed forces where the death of the member
has occurred in the coursed of the operational duties in the circumstances and subject to prescribed
conditions, is -
(a) Exempt upto D 3,00,000
(b) Exempt upto D 3,50,000
(c) Totally exempt u/s 10(19)
(d) Totally chargeable to tax.

7. Mr.X, aged 61 years, earned dividend of D 12,00,000 from ABC Ltd. In P.Y. 2023-24. Interest on loan
taken for purpose of investment in ABC Ltd., is D 3,00,000. Income included in the hands of Mr. X for
P.Y. 2023-24 would be-
(a) D 12,00,000
(b) D 9,60,000
(c) D 9,00,000
(d) D 2,00,000

8. If the converted property is subsequently partitioned among the members of the family, the income
derived from such converted property as is received.
(a) as the income of the karta of the HUF
(b) as the income of the spouse of the transferor
(c) as the income of the HUF.
(d) as the income of the transferor- member

350 The Institute of Cost Accountants of India


Heads of Income

Exercise
A. Theoretical Questions:
¾¾ Multiple Choice Questions:
1. Which of the following is not taxable under head ‘Salaries’?
a. Remuneration paid to the lecturer of a college for setting a question paper by a university.
b. Salary received by a member of the Parliament.
c. Commission received by an employee director of a company.
d. Both (a) and (b) above
2. If a domestic servant is engaged by the employer and salary is paid by him, the perquisite is
a. Taxable in the hands of all employees
b. Not taxable in the hands of both specified and non-specified employers
c. Taxable in the hands of specified employees only
d. Taxable to the extent of ₹ 120 per person in the hands of all employees.
3. Which of the following is taxable under the head ‘salaries’?
1. Salary received by a Member of State Legislature.
2. Commission received by an employee director of a company.
3. Family pension received
4. Both (a) and (b) above
4. Who among the following is a specified employee?
a. A director of a company
b. An employee drawing a salary of ₹ 15,000 p.m.
c. A person who is an owner of equity shares carrying 10% voting power in the employer
company.
d. Both (a) and (b) above
5. Net Annual Value of a self-occupied property treated as such is:
1. Fair Rent
2. Nil
3. Reasonable Expected Rent as reduced by municipal tax paid during the previous year.
4. None of the Above
6. A house property located outside India is:
a. Taxable in hands of all assessee
b. Taxable in hands of non resident assessee
c. Taxable in hands of resident and ordinarily resident assessee
d. Exempted from tax in India.
e. None of the Above

The Institute of Cost Accountants of India 351


Direct Taxation

7. Following assessee(s) can considered a house property as self occupied:


a. Individual & HUF
b. All assessee
c. All assessee other than company
d. All assessee other than firm
8. Which of the following deductions is /are not allowed in case of a deemed to be let-out house?
a. New construction allowance
b. Repairs
c. Vacancy allowance
d. All of the above
9. Which of the following is not allowed as a deduction for computation of business Income?
a. Loss incurred due to theft in factory after working hours
b. Anticipated future losses
c. Loss caused by white ants
d. Loss due to accidental fire in stock-in-trade
10. The preliminary expenses that can be amortized under the Income Tax Act, 1961 has to be restricted
to _________ of the cost of project.
a. 3%
b. 5%
c. 8%
d. 20%
11. Expenditure on promotion of family planning is an allowance as deduction u/s. 36(1)(ix) of the
Income Tax Act, 1961 in case of
a. Individual
b. Firm
c. HUF
d. Company
12. In case of loss, a partnership firm may claim deduction in respect of remuneration to partner to the
extent of:
a. ₹ 1,50,000/-
b. ₹ 1,50,000/- or actual remuneration, whichever is lower
c. ₹ 1,50,000/- or 90% of book profit, whichever is lower
d. Nil
13. U/s 54, capital gain will be allowed as exemption if the house property under transfer is held for
a. Less than 12 months preceding the date of transfer
b. More than 12 months preceding the date of transfer

352 The Institute of Cost Accountants of India


Heads of Income

c. Less than 36 months preceding the date of transfer


d. More than 24 months preceding the date of transfer
14. Personal effect do not cover the followings
a. Immovable property
b. Drawings
c. Jewellery
d. All of the above
15. Profit on sale of rural agricultural land is
a. Not taxable as it is agricultural income
b. Not taxable under the head ‘Capital gains’ but under the head ‘Income from Other Sources’
c. Not taxable as rural agricultural land is not considered as a capital asset
d. Taxable if it is compulsorily acquired.
16. Cost of acquisition of self-generated asset is nil, the exception is:
a. Goodwill
b. Route permit
c. Bonus shares acquired before 01-04-2001
d. Loom hours
17. While computing taxable interest on delayed compensation, a standard deduction is allowed @
a. 50%
b. 30%
c. 15%
d. Nil
18. The provision of sec.56(2)(x) is applicable on
a. All assessee
b. Only on corporate assessee
c. On an individual only
d. On an individual and HUF only
19. While computing income from other sources, deduction is not allowed to the assessee for:
a. Personal expenditure
b. Direct tax
c. Interest payable outside India without TDS
d. All of the above
20. Gift received by an individual in certain circumstances is not taxable, one of them is:
a. Any gift received from family friend
b. Any gift received on the occasion of any marriage in the family

The Institute of Cost Accountants of India 353


Direct Taxation

c. Any gift received on the occasion of the marriage of the individual-assessee


d. All of the above
[Answer : 1 - d ; 2 - c ; 3 - b ; 4 - d ; 5 - b ; 6 - c ; 7 - a ; 8 - d ; 9 - b ; 10 - b ; 11 - d ;
12 - b ; 13 - d ; 14 - d ; 15 - c ; 16 - c ; 17 - a ; 18 - a; 19 - d ; 20 - c]

B. Numerical Questions
¾¾ Multiple Choice Questions
1. An individual purchased a painting on 01-11-2023 for ₹ 5,00,000 though fair market value of the
asset is ₹ 5,25,000. Income taxable u/s 56(2)(x) is:
a. ₹ 25,000 i.e., difference between market value and actual consideration
b. Nil as this is not gift
c. Nil as difference between market value and actual consideration does not exceed ₹ 50,000
d. The provision of sec. 56(2)(x) is not applicable for any transaction entered during P.Y. 2022-
23.
[Answer : 1- c]

¾¾ Comprehensive Numerical Problems


1. Sri Ghanshyam, an employee of G. Manufacturing (P) Ltd. in Kolkata, gives the following particulars
of his income during the year ended 31st March, 2024:
i. He was appointed in this company on 1st January, 2018 in the scale of pay of ₹ 8,000-500-
14,000 per month.
ii. Dearness Allowance (as per terms of employment) ₹ 4,000 per month.
iii. City compensatory allowance ₹ 1,300 per month.
iv. Tiffin Allowance ₹ 200 per month.
v. Children’s education allowance ₹ 700 per month (for his two children).
vi. He is provided with a rent free unfurnished accommodation at Kolkata for which the employer
pays a monthly rent of ₹ 3,000.
vii. A gardener and a sweeper also provided by the company at a monthly remuneration of ₹ 800
and ₹ 200 respectively.
viii. He owns a 1.5 cc self-driven car which is used by him both for official and private purposes,
the running and maintenance expenses of which is borne by the company.
ix. He paid ₹ 4,000 as income tax and ₹ 1,320 as professional tax during the year.
Compute taxable income from salary of Sri Ghanshyam for the A.Y. 2024-25.
[Answer : Hints: ₹ 2,11,705]

2. Compute the total depreciation allowance for the year ending 31.3.2024:
a. Original cost of motor car as on 31.3.2023 ₹ 12,00,000.

354 The Institute of Cost Accountants of India


Heads of Income

b. Depreciation allowed up to 31.3.2023 ₹ 3,00,000.


c. Addition made during the year on 5.12.2023 ₹ 3,00,000.
d. Sale during the year: Original cost ₹ 15,00,000; Depreciation charged upto 31.8.2023
₹ 37,500; Selling price ₹ 2,00,000.
Rate of depreciation is 15%.
[Answer : Hints: ₹ 1,27,500]

3. Shri Ansh furnishes the following particulars of his incomes for the previous year 2023-24. Compute
his income under the head “Income from other sources”.


Dividend on equity shares 600
Dividend on preference shares 3,200
Income from letting on hire of building & machinery under composite lease 17,000
Interest on Bank Deposits 2,500
Directors sitting fees received 1,200
Ground rent received 600
Income from undisclosed sources 10,000
Income from lotteries received 70,000
The following deductions are claimed by him:
Collection charges of dividend 20
Allowable depreciation on building and machinery 4,000
Fire insurance on building and machinery 100
[Answer : Hints. ₹ 1,31,000]

4. Mr. Bose in an Indian resident and employed in a company. During 2023–24 financial year his
income was as under:
i. Basis salary ₹ 20,000 per month.
ii. Dearness allowance 30% of basic salary (50% of which forms part of salary for retirement
purpose)
iii. Medical allowance ₹ 500 per month (Actual expenses for the year was ₹ 4,000)
iv. He and his employer both contributes @ 15% of salary towards a Recognised Provident Fund
and interest credited to RPF @ 12.5% p.a. amounted to ₹ 5,000 during the previous year.
v. He is provided with a furnished accommodation in Kolkata. The company owns the house
and cost of furniture provided is ₹ 50,000.
vi. He is also given a 1.6 liters cubic capacity self driven car for both personal and office use.
Entire expenses are borne by the company

The Institute of Cost Accountants of India 355


Direct Taxation

vii. He is provided by his employer with fee gas and electricity costing ₹ 12,000 for private use.
viii. He paid ₹ 1,600 towards professional tax and his employer paid ₹12,000 as insurance
premium on his life.
ix. He contributed ₹ 12,000 towards medical insurance premium, ₹25,000 towards PPF, ₹10,000
towards Prime Minister’s Drought Fund and ₹ 12,000 to an approved association for
promoting family planning.
On the basis of aforesaid information, you are requested to answer the following:
¾¾ What would be his taxable allowances?
¾¾ What would be value taxable perquisite in his hands?
¾¾ What would be his total income?
[Answer : Hints: (a) ₹ 78,000; (b) ₹ 92,900; (c) ₹ 2,61,380]

¾¾ References:
https://www.incometaxindia.gov.in/
https://www.incometax.gov.in/
https://www.indiabudget.gov.in/

356 The Institute of Cost Accountants of India


Total Income and Tax Liability of
3
Individuals & HUF
This Module includes
3.1 Income of Other Person included in Assesses Total Income (Clubbing of Income)
3.2 Set off and Carry Forward of Losses
3.3 Deductions, Rebate and Relief
3.4 Taxation of Individual (including AMT but excluding Non-resident) & HUF
3.5 Advance Tax
3.6 Tax Deducted at Source & Tax Collected at Source (excluding Non-resident)
3.7 Filing of Return of Income
3.8 PAN
3.9 Self-Assessment & Intimation

The Institute of Cost Accountants of India 357


Total Income and Tax Liability of
Individuals & HUF
SLOB Mapped against the Module:

1. To acquire application-oriented knowledge and skill for appreciating various provisions of the Income
Tax Act, 1961 and attain abilities to solve problems while computing tax liabilities.

2. To attain abilities to apply various provisions of direct taxation laws, including assessment to identify
the impacts thereof on business decisions.

3. To gather knowledge of various compliance related provisions of direct taxation laws and attain
abilities to ensure due compliance to avoid any eventual risks.

Module Learning Objectives:

After studying this module, the students will be able to –

 Identify the applicability of the provisions relating to clubbing of income

 Understanding the relationship between various sources of income and heads of income

 Appreciate the provision relating to carry forward of losses

 Appreciate the provisions relating to various compliances.

358 The Institute of Cost Accountants of India


Total Income and Tax Liability of Individuals & HUF

Income of Other Person included in Assesses


3.1
Total Income (Clubbing of Income)
3.1.1 Introduction
Generally, an assessee is taxed on income
accruing to him only and he is not liable to
tax for income of another person. However,
there are certain exceptions to the above rule
(mentioned u/s 60 to 64). Sec. 60 to 64 deals
with the provisions of clubbing of income,
under which an assessee may be taxed in
respect of income accrued to other person,
e.g. certain income of minor child shall be
clubbed in the hands of his parents, income
from asset transferred to spouse for inadequate
consideration shall be clubbed in the hands
of the transferor, etc. These provisions have
been enacted to counteract the tendency on
the part of the taxpayers to dispose off their
income or income-generating assets to escape
tax liability

3.1.2 General rules

 Computation of income to be clubbed


The income, which is to be clubbed, shall be first computed in the hands of recipient and all expenditure related
to such income shall be allowed as per the respective provisions of the Act and thereafter the net income shall
be clubbed.
E.g.: Standard deduction u/s 24(a) from Income from house property shall be allowed in the hands of the
recipient and thereafter the net income shall be clubbed.
 Clubbing Head
Income shall be, first, computed in the hands of recipient and then clubbing shall be made head wise.
E.g.: Bank interest of minor child shall be clubbed under the head “Income from other sources” of parent.
 Deduction under chapter VIA
If the clubbed income is eligible for deduction u/s 80C to 80U, then such deduction shall be allowed to the

The Institute of Cost Accountants of India 359


Direct Taxation

assessee in whose hands such income is clubbed. E.g. If interest on saving bank account of the minor is
clubbed in the hands of parent u/s 64(1A) then parent can claim deduction u/s 80TTA.
 Clubbing of negative income
As per explanation 2 to sec. 64, clubbing of income includes clubbing of negative income i.e. where an income
is liable to be clubbed, loss from the same source shall also be clubbed. Clubbing provisions is mandatory
and shall be applied even in those cases where the application of such provision causes loss of revenue to the
Income tax department.

3.1.3 Transfer of income without transferring assets [Sec. 60]


Where an income is transferred without
transferring the asset yielding such income,
then income so transferred shall be clubbed
in the hands of the transferor.
The above provision holds good -
 whether the transfer is revocable or not;
or
 whether the transaction is effected before
or after the commencement of this Act.

3.1.4 Revocable Transfer [Sec. 61]


If an assessee transfers an asset under a revocable transfer, then income generated from such asset, shall be clubbed
in the hands of the transferor.

Revocable transfer
As per sec. 63(a), a transfer shall be deemed
to be revocable if -
 It contains any provision for the
retransfer (directly or indirectly) of any
part or whole of the income/assets to the
transferor; or
 It, in any way, gives the transferor a
right to re-assume power (directly or
indirectly) over any part or whole of the
income/assets.
Exceptions [Sec. 62]
As per sec. 62(1), the provision of sec. 61 shall not apply to an income arising to a person by virtue of -
i. A transfer by way of creation of a trust which is irrevocable during the lifetime of the beneficiary;
ii. Any transfer which is irrevocable during the lifetime of the transferee; or
iii. Any transfer made before 1.4.61, which is not revocable for a period exceeding 6 years.
In any case, the transferor must not derive any benefit (directly or indirectly) from such income.
Note: As per sec. 62(2), income, in any of the above exceptional case, shall be taxable as under -

360 The Institute of Cost Accountants of India


Total Income and Tax Liability of Individuals & HUF

Situation Taxable in hands of


When the power to revoke the transfer arises (whether such power is exercised or not) Transferor
When the power to revoke the transfer does not arise Transferee
Illustration 1
Discuss the tax treatment in the following cases:

Case Answer
Harish has transferred certain securities owned Since the asset has been transferred under a revocable clause,
by him to a trust for his married sister, Harsha, hence, by virtue of sec. 61, income accrued on securities during
as on 1/7/2022. He has the power to revoke the the previous year 2022-23 ₹ 1,20,000 and 2023-24 ₹ 1,50,000,
trust at his desire. On 31/3/2024, he revoked though received by Harsha, shall be taxable in the hands of
such trust. Income accrued for the previous Harish.
year 2022-23 and 2023-24 are ₹ 1,20,000 and
₹ 1,50,000 respectively and such income is
received and enjoyed by Harsha.
Raja transferred his property on 1/4/2022 to Since asset has been transferred under a revocable clause, hence,
Rani with a clause that, he will take property by virtue of sec. 61, income accrued on such property shall be
back from Rani whenever he requires. Raja chargeable in hands of Raja for previous year 2022-23 as well as
was in need of money on 1/4/2023 and he took for 2023-24. This is because all income arising to the transferor
back property from Rani. The property yields by virtue of any revocable transfer shall be chargeable in hands
annual income of ₹ 2,00,000. of the transferor as and when the power to revoke the transfer
arises whether or not such power has been exercised. Since Raja
had the power of revocation from beginning therefore income
shall be clubbed in his hand from the date of transfer.
Seema transferred on 1/4/2022 her property Since asset has been transferred which is irrevocable during the
to Neema for the life time of Neema with a lifetime of the transferee hence, by virtue of sec. 62, income
clause that after death of Neema, property shall accrued on such property shall be chargeable in hands of Neema
be back to Seema. Neema died on 1/4/2023. for the P.Y. 2022-23. However, income for the P.Y. 2023-24
Seema has not taken back the property till shall be chargeable in hands of the Seema as because after death
31/3/2024. Property yields annual income of ₹ of Neema, Seema got the power to revoke the transfer and when
1,00,000. the power to revoke the transfer arises, income is clubbed in
hands of transferor even the power has not been exercised.

3.1.5 Remuneration to Spouse [Sec. 64(1)(ii)]


The total income of an individual shall include income arising (directly or indirectly) to the spouse by way of
salary, commission, fees or any other remuneration (whether in cash or in kind) from a concern in which such
individual has substantial interest1.
Taxpoint: Any other income, which is not specified above, even if it accrues to spouse from the concern in which
the assessee has substantial interest, shall not be clubbed.
1.
Substantial interest: An individual shall be deemed to have substantial interest in a concern if —

He beneficially holds not less than 20% of its equity shares at any time during the previous
In case of
year. Such share may be held by the assessee or partly by assessee and partly by one or more
company
of his relatives.

The Institute of Cost Accountants of India 361


Direct Taxation

He is entitled to not less than 20% of the profits of such concern at any time during the previous
Other concern
year. Such share of profit may be held by the assessee himself or together with his relatives.
Relative here includes spouse, brother or sister or any lineal ascendant or descendant of that individual [Sec.
2(41)].
Exception
Income generated through technical or professional qualification# of the spouse is not to be clubbed in the total
income of the individual.
#
Technical or professional qualification: The term technical or professional qualification must be construed
in a liberal manner as the term has not been defined in the Act. It does not necessarily relate to technical
or professional qualification acquired by obtaining a certificate, diploma or degree or in any other form,
from a recognised body like University or Institute. It can be treated as fitness to do a job or to undertake
an occupation requiring intellectual skill and also includes technicality generated through experience, skill
etc. Technical qualification includes specialization in a particular subject (e.g. accountancy, management,
commerce, science, technology etc.).

Notes
Where both, husband and wife, have substantial Remuneration from such concern will be included in
interest in a concern the total income of husband or wife, whose total income
When both, husband and wife, have substantial interest excluding such remuneration, is higher.
in a concern and both are drawing remuneration from that Where such income is once included in the total income
concern without possessing any specific qualification. of either of the spouse, then such income arising in
any subsequent years cannot be included in the total
income of the other spouse unless the Assessing Officer
is satisfied that it is necessary to do so. However,
Assessing Officer will do so only after giving to the
other spouse an opportunity of being heard.
When both, husband and wife, are not having any Remuneration from such concern will not be clubbed.
other income
When both, husband and wife, have substantial interest
in a concern and both are drawing remuneration from that
concern without possessing any specific qualification
and both are not having any other income apart from the
said remuneration.

362 The Institute of Cost Accountants of India


Total Income and Tax Liability of Individuals & HUF

Computation of salary, fee, commission, Income prescribed in sec. 64(1)(ii) shall be first
remuneration etc computed (allowing all deductions from the respective
income) in the hands of recipient and thereafter net
income shall be clubbed in the hands of the other spouse.
E.g. salary, remuneration, etc shall be first calculated as
per provisions of sec. 15 to 17, in the hands of recipient
and thereafter, net taxable salary shall be clubbed in the
hands of the other spouse.
Illustration 2:
Ram and Mrs. Ram hold 20% and 30% equity shares in Anand Ltd. respectively. They are employed in Anand
Ltd. (taxable salary being ₹ 2,40,000 p.a. and ₹ 3,60,000 p.a. respectively) without any technical or professional
qualification. Other incomes of Ram and Mrs. Ram are ₹ 70,000 and ₹ 1,00,000 respectively. Find out the net
income of Ram and Mrs. Ram for the assessment year 2024-25.
Solution :
When both husband and wife have substantial interest in a concern & both are drawing remuneration from that
concern without possessing any specific qualification, then remuneration from such concern will be included in
the total income of husband or wife, whose total income excluding such remuneration, is higher. In the given case,
Since Mrs. Ram has higher income therefore salary of Mr. Ram will be clubbed in hands of Mrs. Ram.
Computation of gross total income of Ram and Mrs. Ram for the A.Y. 2024-25

Particulars Mr. Ram Mrs. Ram


Salaries 2,40,000 3,60,000
Clubbing of income as per sec. 64(1)(ii) (2,40,000) 2,40,000
Income from other sources 70,000 1,00,000
Gross Total Income 70,000 7,00,000

Illustration 3:
Mr. & Mrs. Om both are working in A Ltd. without possessing any technical or professional qualification. From the
following details compute their income for the A.Y. 2024-25:

Particulars Mr. Om Mrs. Om


Taxable Salary from A Ltd. ₹ 2,20,000 ₹ 70,000
Other income ₹ 50,000 ₹ 80,000
Share of holdings: Case 1 15% 6%
Case 2 3% 17%
Case 3 18% 1%
Solution :
Computation of gross total income of Mr. Om and Mrs. Om for the A.Y. 2024-25

Case 1 Case 2 Case 3


Particulars
Mr. Om Mrs. Om Mr. Om Mrs. Om Mr. Om Mrs. Om
Taxable Salary of Mr. Om 2,20,000 2,20,000 2,20,000
Taxable Salary of Mrs. Om 70,000 70,000 70,000

The Institute of Cost Accountants of India 363


Direct Taxation

Case 1 Case 2 Case 3


Particulars
Mr. Om Mrs. Om Mr. Om Mrs. Om Mr. Om Mrs. Om
Other Income 50,000 80,000 50,000 80,000 50,000 80,000
Gross Total income 50,000 3,70,000 50,000 3,70,000 2,70,000 1,50,000
Notes
1. Substantial interest means assessee beneficially holds (whether individually or partly by assessee and partly
by one or more of his relatives) not less than 20% of equity shares of company.
2. When both, husband and wife, have substantial interest in a concern and both are drawing remuneration from
that concern without possessing any specific qualification, remuneration from such concern will be included
in the total income of husband or wife, whose total income excluding such remuneration, is higher.
3. In case 3, since none of the spouse has substantial interest in the concern hence provision of sec. 64(1)(ii)
shall not be attracted.

Illustration 4:
Mr. & Mrs. X working in A Ltd. without possessing any qualification. From the following details compute their
income for the A.Y. 2024-25:

Particulars Mr. X Mrs. X


Share of holdings 15% 6%
Taxable salary from A Ltd. ₹ 1,20,000 ₹ 60,000
Case 1) Other income ₹ 50,000 ₹ 80,000
Case 2) Other income ₹ 90,000 ₹ 65,000
Case 3) Other income Nil Nil
Solution :
Computation of gross total income of Mr. X and Mrs. X for the A.Y. 2024-25

Case 1 Case 2 Case 3


Particulars
Mr. X Mrs. X Mr. X Mrs. X Mr. X Mrs. X
Salary of Mr. X - 1,20,000 1,20,000 - 1,20,000 -
Salary of Mrs. X - 60,000 60,000 - - 60,000
Other Income 50,000 80,000 90,000 65,000 Nil Nil
Gross Total income 50,000 2,60,000 2,70,000 65,000 1,20,000 60,000
Notes
1. When both, husband and wife, have substantial interest in a concern and both are drawing remuneration from
that concern without possessing any specific qualification, remuneration from such concern will be included
in the total income of husband or wife, whose total income excluding such remuneration, is higher.
2. In case 3, when both husband and wife have substantial interest in a concern and both are drawing
remuneration from that concern without possessing any specific qualification and both are not having any
other income apart from said remuneration, then clubbing shall not be made.

364 The Institute of Cost Accountants of India


Total Income and Tax Liability of Individuals & HUF

3.1.6 Income from asset transferred to spouse [Sec. 64(1)(iv) & (vii)]
Asset transferred to Spouse [Sec. 64(1)(iv)]
In computing the total income of an individual [subject to the provisions of sec. 27(i)], income arising from assets
transferred to spouse without adequate consideration, shall be included in the income of that individual.

Taxpoint: In the following cases clubbing provision shall not be attracted on transfer of property to spouse -
 When such transfer is for adequate consideration; or
 The transfer is under an agreement to live apart; or
 Where the asset transferred is house property (as such transfer will be governed by Sec. 27)
Notes

Marital relationship The relationship of husband and wife must subsist on the date of transfer of assets as
well as on the date of accrual of income i.e. no clubbing provision shall be attracted if -
¾¾ transfer is made before marriage; or
¾¾ on the date of accrual of income, transferee is not the spouse of transferor.
Pin money Income from assets acquired by spouse out of pin money or household savings is not
subject to clubbing
Sec. 27(i) -vs.- Sec. Where such transferred asset is a house property then the same will not be covered by
64(1)(iv) in case of sec. 64(1)(iv) but governed by sec. 27(i) [Deemed owner in case of Income from house
transfer of house property]. In this regard following instances are notable -
property ¾¾ Mr. X transferred his house property (worth ₹ 15,00,000) in favour of Mrs. X
out of love and affection without any consideration. Income generated from such
property is ₹ 1,00,000 shall be taxable in the hands of Mr. X by virtue of sec. 27(i)
without attracting provision of sec. 64(1)(iv).
¾¾ Capital gain generated on transfer of such property shall be first computed in the
hands of Mrs. X and then shall be clubbed in hands of Mr. X by virtue of sec. 64(1)
(iv).
¾¾ Mr. X gifted Mrs. X ₹ 15,00,000 out of love and affection. Mrs. X purchased
a house property from such gifted money. Income generated from such house
property is ₹ 1,00,000. Such income shall be first computed in hands of Mrs. X and
then shall be clubbed in the hands of Mr. X by virtue of sec. 64(1)(iv).

The Institute of Cost Accountants of India 365


Direct Taxation

Notes applicable to Sec. 64(1)(iv), (vi), (vii) & (viii)

Nature of Consideration must be measurable in terms of money. Therefore, natural love and affection,
consideration spiritual or religious benefit cannot be treated as adequate consideration.
Inadequate If property has been transferred Mr. X transferred 1000 10% debentures of ₹ 100 each
consideration to spouse or son’s wife directly or (Market value ₹ 100 each) to Mrs. X for ₹ 25,000.
indirectly for a consideration which Income earned at the end of the year on such debenture
is inadequate, then only the part of ₹ 10,000.
income which is referable to transfer
Treatment: ₹ 2,500 (i.e. ₹ 10,000 * ₹ 25,000 / ₹
for inadequate consideration, shall be
1,00,000) shall be taxable in hands of Mrs. X and
clubbed.
remaining ₹ 7,500 (i.e. ₹ 10,000 – ₹ 2,500) shall be
clubbed in the hands of Mr. X.
Form of Asset The asset so transferred need not Mr. X transferred to Mrs. X securities worth ₹ 5,00,000.
remain in its original form. Mrs. X exchanged such securities for a house property.
Income from such property shall also be liable to
clubbing u/s 64(1)(iv).
Accretion of Income arising to the transferee from Mr. X transferred to Mrs. X shares of a foreign company
asset the accretion of such property shall worth ₹ 5,00,000. Mrs. X received bonus share on such
not be clubbed in the total income of shares. Dividend on original shares shall be clubbed in
the transferor. the hands of Mr. X but dividend on bonus shares shall
not be clubbed and taxable in the hands of Mrs. X.
Capital gain Profit on sale of property, which is gifted to spouse or son’s wife, shall be clubbed in hands of
the transferor.
Income on Income arising to the transferee from Mr. X transferred to Mrs. X shares worth ₹ 5,00,000.
income the accumulated income of such Mrs. X sold such shares for ₹ 6,00,000 (i.e. earned a
property is not to be clubbed. profit of ₹ 1,00,000) and acquired 12% Debentures of
another company.
Capital gain of ₹ 1,00,000 and interest on debenture
of ₹ 60,000 (i.e. 12% of ₹ 5,00,000) shall be clubbed
in hands of Mr. X u/s 64(1)(iv). However, interest on
debenture of ₹ 12000 (i.e. 12% of ₹ 1,00,000) shall not
be clubbed and taxable in the hands of Mrs. X.
Asset Where an asset is transferred under a trust for the benefit of spouse or son’s wife the provision
transferred of sec. 64(1) shall be applicable. It does not make any difference even if the assessee appoints
under a trust himself as a trustee.
Treatment After the death of either husband or wife clubbing provision of Sec. 64(1)(iv) will not be
after death attracted, as a widow or widower is not a spouse.
Cross Gift If the Assessing Officer is convinced Mr. A gifted his property to Mrs. X and Mr. X gifted
that some gifts are inter-related equivalent another property to Mrs. A.
with each other in a circuitous way,
Treatment In such case it is apparent that such
and there is an intention to evade
transactions are interrelated and are parts of the same
provisions of sec. 64(1), he may
transaction. In such case income from such property of
apply the provisions of sec. 64(1).
Mrs. A and Mrs. X shall be clubbed in hands of Mr. A
and Mr. X respectively.

366 The Institute of Cost Accountants of India


Total Income and Tax Liability of Individuals & HUF

Asset invested If assets so transferred, is invested in business then tax treatment shall be as under:
in the business
Type of business Income to be clubbed
Income of such business * Value of such assets as on the 1st day of the P.Y.
Proprietary
Total investment in the business by the transferee as on the same day
Interest on capital * Value of such assets as on the 1st day of the P.Y.
Partnership
Total investment in the firm by the transferee as on the same day
Taxpoint: Profit from the firm shall not be clubbed as it is exempted u/s 10(2A).

Illustration 5:
Mr. X gifted 1,000 shares of a non-domestic company worth ₹ 6,00,000 (acquired on 15/3/2022) to Mrs. X out of
natural love and affection as on 15/4/2023.
On 31/1/2024, Mrs. X received dividend ₹ 60,000 on such shares in India.
On 1/2/2024, Mrs. X sold such shares for ₹ 10,00,000 and received consideration in India.
Show tax treatment, if on 1/2/2024, Mrs. X invested ₹ 10,60,000 in -
Case A A house property from which rent accrued in the previous year 2023-24 is ₹ 53,000.
Case B A newly formed partnership firm and contributed initial capital. Interest received (taxable portion) on
such contribution ₹ 13,250 and share of profit ₹ 20,000.
Case C A newly started proprietorship business & contributed capital, profit accrued for the year is ₹ 42,400.
Solution :
Statement showing treatment of provision of sec. 64(1)(iv) in hands of Mr. X for the A.Y. 2024-25

Case A Case B Case C


Particulars
Mr. X Mrs. X Mr. X Mrs. X Mr. X Mrs. X
Dividend from non-domestic company 60,000 - 60,000 - 60,000 -
Short term Capital gain (Note 1) 4,00,000 - 4,00,000 - 4,00,000 -
Income from house property (Note 2) 21,000 16,100 - - - -
Interest on capital in partnership firm (Note 3) - - 7,500 5,750 - -
Share of profit in firm [Exempted u/s 10(2A)] - - - - - -
Profit from proprietorship business (Note 3) - - - - 24,000 18,400
Total 4,81,000 16,100 4,67,500 5,750 4,84,000 18,400
Note 1: Computation of capital gain and its treatment

Particulars Amount
Sale consideration 10,00,000
Less: Cost of acquisition 6,00,000
Short term capital gain 4,00,000
Treatment: Since the shares have been gifted by Mr. X to Mrs. X the provision of sec. 64(1)(iv) shall be
applicable and such gain shall be taxable in hands of Mr. X.

The Institute of Cost Accountants of India 367


Direct Taxation

Note 2: Computation of Income from house property of Mrs. X for the A.Y. 2024-25

Particulars Amount
Gross Annual Value 53,000
Less: Municipal tax Nil
Net Annual value 53,000
Less: Deduction u/s 24(a) [being 30% ₹ 53,000] 15,900
Income from house property 37,100
Note 3: Statement showing distribution of income between Mr. X & Mrs. X

Since the new investment has been acquired out of -


Gifted money (equivalent) ₹ 6,00,000
Capital gain ₹ 4,00,000
Dividend income ₹ 60,000
Hence, income shall be clubbed in hands of Mr. X to the extent of 600/1060 and in hands of Mrs. X 460/1060
Taxpoint: Income arising to the transferee from the accretion of such property or from accumulated income of
such property is not to be clubbed in the total income of the transferor.
Case In hands of Mr. X In hands of Mrs. X
A (₹ 37,100 * 600) / 1,060 = ₹ 21,000 (₹ 37,100 * 460) / 1,060 = ₹ 16,100
B (₹ 13,250 * 600) / 1,060 = ₹ 7,500 (₹ 13,250 * 460) / 1,060 = ₹ 5,750
C (₹ 42,400 * 600) / 1,060 = ₹ 24,000 (₹ 42,400 * 460) / 1,060 = ₹ 18,400
Note: In case of new source of income, previous year starts from the date when such new source of income comes
into existence.

Assets transferred to AOP or other person for the benefit of spouse [Sec. 64(1)(vii)]
In case an asset is transferred to other person or an association of persons, otherwise than for adequate consideration,
for immediate or deferred benefit of spouse, then income on asset so transferred shall be clubbed in the hands of
the transferor (to the extent income from such asset is for the immediate or deferred benefit of his or her spouse).

Income from assets transferred to son’s wife [Sec. 64(1)(vi) & (viii)]
The provisions are enumerated here-in-below:

In computing the total income of an individual, income arising (directly or


indirectly) from assets transferred to son’s wife (after 31.5.1973), without adequate
Transfer to son’s wife
consideration, shall be included in income of that individual. Aforesaid relationship
[Sec. 64(1)(vi)]
must subsist on the date of transfer of assets as well as on the date of accrual of
income.
Transfer to AOP or other In case an asset is transferred to other person or an association of persons (after
person for the benefit of 31.5.1973), otherwise than for adequate consideration, for immediate or deferred
son’s wife benefit of son’s wife, then income on asset so transferred shall be clubbed in the
hands of the transferor (to the extent income from such asset is for the immediate or
[Sec. 64(1)(viii)] deferred benefit of son’s wife). See notes given below sec. 64(1)(iv)

368 The Institute of Cost Accountants of India


Total Income and Tax Liability of Individuals & HUF

3.1.7 Income of minor child [Sec. 64(1A)]


Income of a minor child shall be clubbed with income of the parent whose total income (excluding this income) is
higher.

Exceptions
The above clubbing provision shall not apply in
the following cases -
1. The income arises or accrues to the minor
child due to any manual work done by him;
or
2. The income arises or accrues to the minor
child due to his skill, talent, specialised
knowledge or experience; or
3. The minor child is suffering from any
disability of nature specified u/s 80U.

Be careful, there is no exception for minor married daughter like exception to sec. 27 (Deemed Owner)

Exemption [Sec. 10(32)]


In case income of a minor child is clubbed in hands of parent as per provision of sec. 64(1A), the assessee (parent)
can claim exemption of an amount being minimum of the following -
a. ₹ 1,500; or
b. Income so clubbed
Taxpoint: Such exemption shall be available for each child (irrespective of the number of children) whose income
is so clubbed.
When marriage does not subsist between parents
In case marital relationship does not subsist at the time of accrual of income to the minor child, income of minor
child shall be clubbed with income of that parent who maintains the minor child during the previous year.
Taxpoint: Income of the minor child shall be clubbed in hands of parent in the following manner -

Relation between parents Tax treatment


When marriage subsists With the income of that parent whose total income excluding this income is higher
When marriage does not
With the income of that parent who maintains the minor child in the previous year
subsist
Notes

Clubbing in Where any such income is once clubbed with the total income of either parent, then any
subsequent year(s) such income arising in any subsequent years shall not be clubbed with the total income of
the other parent, unless the Assessing Officer is satisfied. However, the Assessing Officer
will do so only after giving an opportunity of being heard to the other spouse.

The Institute of Cost Accountants of India 369


Direct Taxation

Child Child in relation to an individual includes a stepchild & adopted child but does not include
a grandchild [Sec. 2(15B)]
Income of married Though sec. 27(i) [Deemed owner of house property] specifically excludes married
daughter daughter but sec. 64(1A) does not have this exception, hence income arising to minor
married daughter shall be clubbed in the hands of parent.
When neither of Income of minor child cannot be added with the income of the guardian if the guardian is
the parent is alive not the parent of the minor.
Capital gain Profit on sale of the property, which is gifted to minor child, shall be clubbed in hands of
parent as per the provision of sec. 64(1A)
Income of the Income of that year shall be treated as under:
year when minor
attains majority Income arose to the period Clubbing provision
When child does not attain age of majority Such income shall be clubbed in hands of
parent
When child attains the age of majority and Such income shall not be clubbed and
afterwards taxable in hands of assessee himself (i.e.
child)

Illustration 6:
Mr. & Mrs. Mantri have income under the head “Profits & gains of business or profession” of ₹ 3,00,000 and
₹ 4,00,000 respectively. They have 7 children. From the following details compute taxable income of Mr. and Mrs.
Mantri for the A.Y. 2024-25:
 1st child (aged 26 years) is a chartered accountant. His annual income from profession is ₹ 4,00,000. His
income from house property is ₹ 30,000. He has a son (4 years old) who has earned interest on fixed deposit
of ₹ 5,000.
 2nd child (aged 17 years being a married daughter) who is a stage singer, earned income of ₹ 1,00,000. She
earned interest on fixed deposit ₹ 8,000. Such fixed deposit has been made out of such singing income.
 3rd child (aged 16 years) is suffering from disability specified u/s 80U (to the extent 55%) blind. He has
received interest income of ₹ 40,000 for loan given to a private firm. He is dependent on Mrs. Mantri.
 4th child (aged 14 years) has earned income of ₹ 45,000 out of his physical and mental effort. Expenditure
incurred to earn such income is ₹ 15,000. His loss from house property is ₹ 30,000.
 5th child (aged 12 years) is a partner in a partnership firm from which he earned interest income (taxable) of
₹ 40,000 and share of profit of ₹ 35,000. Other two partner of the firm are Mr. & Mrs. Mantri.
 6th child (aged 9 years) has 1,000 debentures of ₹ 100 each of a public sector company acquired through will of
his Grand father. Interest income on such debenture is ₹ 10,000. Expenditure incurred to collect such interest
is ₹ 200. Such debenture was sold and long-term capital gain earned ₹ 25,000.
 7th child (aged 7 years) has earned interest on fixed deposit ₹ 500.
Solution :
Computation of total income of Mr. and Mrs. Mantri for the A.Y. 2024-25

370 The Institute of Cost Accountants of India


Total Income and Tax Liability of Individuals & HUF

Mr. Mantri Mrs. Mantri


Particulars
Details Amount Details Amount
Income from house property
Income of 4th Child: Loss from house property (30,000)
Profits & gains of business or profession
Business income 3,00,000 4,00,000
Add: Income of 5th Child
Interest from partnership firm 40,000
Share of profit [Exempt u/s 10(2A)] Nil
Less: Exemption u/s 10(32) (1,500) 4,38,500
Capital gains
Income of 6th Child
Long term capital gain [#Deduction can be claimed once] 25,000#
Income from other sources
Income of 2nd Child
Interest income 8,000
Less: Exemption u/s 10(32) (1,500) 6,500
Income of 6 Child
th

Interest on debenture [₹ 10000 – ₹ 200] 9,800


Less: Exemption u/s 10(32) (1,500) 8,300
Income of 7 Child
th

Interest on Fixed Deposit 500


Less: Exemption u/s 10(32) (500) Nil
Gross Total Income 3,00,000 4,48,300
Less: Deduction u/s 80DD Nil 75,000
Total Income 3,00,000 3,73,300
Notes
a. Income of the first child shall not be clubbed with the income of Mrs. Mantri as he is a major. Income of
grand child (son of first child) will be clubbed with the income of his parent.
b. The clubbing provision of sec. 64(1A) shall not apply where -
¾¾ the income arises or accrues to the minor child due to any manual work done by him; or
¾¾ the income arises or accrues to the minor child due to his skill, talent, specialised knowledge or
experience; or
¾¾ the minor child is suffering from any disability of nature as specified u/s 80U.
c. Income shall be first computed head wise in the hands of recipient and then clubbing shall be carried out
head wise.

The Institute of Cost Accountants of India 371


Direct Taxation

3.1.8 Conversion of self acquired property into HUF Property [Sec. 64(2)]
Applicability

An individual, being a member of an HUF,


has converted1 a property2 after 31/12/1969
(being self acquired asset of the individual)
into property of HUF of which he is a member,
otherwise than for adequate consideration.

Tax-Treatment
For the purpose of computation of total income of such individual for any assessment year commencing on or after
1/4/1971, the income derived from such converted property (property so converted or transferred by individual to
HUF) or any part thereof shall be deemed to arise to the individual and not to the family.
Where the converted property has been the subject matter of partition (whether partial or total) amongst the
members of the family, the income derived from such converted property as is received by the spouse shall be
clubbed in the hands of transferor.
Taxpoint:

 Asset was originally self acquired property of the individual


 Such asset is transferred directly or indirectly to HUF hotch-pot for inadequate consideration.
 Treatment is as under -

Case Income to be clubbed in hands of transferor


Before partition The entire income from such property
After partition Income from the assets attributable to the spouse of transferor.
1
Such conversion can be made by: a) the act of impressing such separate property with the character of property
belonging to the family; or b) throwing it into the common stock of the family.
Property here includes any interest in property whether movable or immovable; the proceeds of sale thereof and
2.

any money or investment for the time being representing the proceeds of sale thereof; and where the property is
converted into any other property by any method, such other property.
Note: Asset need not be in its original form.

Illustration 7:
Mr. Todi is a member of HUF. It consists of Mr. Todi, Mrs. Todi, Mr. Todi’s major son (Mr. A) & Mr. Todi’s minor
son (B)
On 1/4/2022, Mr. Todi transferred his house property acquired through his personal income to the HUF without
any consideration.
On 1/7/2023, HUF is partitioned and such property being divided equally.
Net annual value of the property for the P.Y. 2022-23 is ₹ 80,000 & that for the P.Y. 2023-24 is ₹ 1,00,000. State
tax treatment for both the years.

372 The Institute of Cost Accountants of India


Total Income and Tax Liability of Individuals & HUF

Solution :
Computation of income from house property in the hands of Mr. Todi for the A.Y. 2023-24
Particulars DetailsAmount
Net Annual Value (NAV) 80,000
Less: Standard deduction u/s 24(a) 30% of NAV 24,000
Income from house property 56,000
Tax treatment for the A.Y.2023-24: Since Mr. Todi transferred his house property acquired out of personal
income to his HUF without adequate consideration, therefore income generated from such house property i.e.
₹ 56,000 shall be clubbed in hands of Mr. Todi as per provision of sec. 64(2).
Tax treatment for the A.Y.2024-25: In the previous year 2023-24, partition took place on 1/7/2023; hence the
treatment shall be as under:
Particulars Details Amount
Net Annual Value (NAV) 1,00,000
Less: Standard deduction u/s 24(a) 30% of NAV 30,000
Income from house property 70,000
Income earned till partition from April’ 2023 to June’ 2023 i.e. ₹ 17,500 [(₹ 70,000/12) * 3] shall be clubbed in
hands of Mr. Todi and income earned after partition i.e. ₹ 52,500 [(₹ 70,000/12) * 9] shall be divided among the
family members. However, as per provision of sec. 64(2) income of Mrs. Todi shall be clubbed in hands of Mr.
Todi.
Particulars Mr.Todi Mrs.Todi Mr. A B
Income from house property before partition clubbed in hands of
Mr. Todi as per sec. 64(2) 17,500 - - -
Share of Income from house property ₹ 52,500/4 13,125 13,125 13,125 13,125
Income clubbed as per provision of sec. 64(2) + 13,125 (13,125) - -
Income clubbed as per provision of sec. 64(1A)1 + 13,125 - - (13,125)
Less: Exemption u/s 10(32) (1,500) - - -
Total income from house property 55,375 Nil 13,125 Nil
1.
It is assumed that Mrs. Todi has no other income.

3.1.9 Liability of the transferee [Sec. 65]


Applicability
Where, by reason of the -
 provisions contained in this Chapter; or
 provisions contained in sec. 27(i)
the income from any asset (or from membership in a firm) of a person other than the assessee, is included in the
total income of the assessee.
Impact

On the service of a notice of


demand by the Assessing
Officer in this behalf, the
person in whose name
name such asset stands (or who is a member of the firm) shall be liable to pay that portion of the tax levied on
the assessee which is attributable to the income so included.

The Institute of Cost Accountants of India 373


Direct Taxation

Notes
Such liability cannot exceed the value of assets so transferred.
Where any such asset is held jointly by more than one person, they shall be jointly and severally liable to pay the
tax which is attributable to the income from the assets so included.
Taxpoint: After application of provisions of clubbing (on transfer of property without adequate consideration as
discussed above in several sections), income is taxable and tax liability arises in the hands of the transferor. But
sec. 65 empowers the income tax authorities to serve demand notice (in respect of tax on clubbed income) upon
transferee.

3.1.10 Important general notes


Where an income is includible u/s 64 and tax has been deducted at source from such
TDS income, the credit of tax deducted at source shall be given to the person in whose hands
the income is taxable [Sec. 199].
The advance tax paid by the income earner (say spouse or minor child) with reference
to such income is not eligible for adjustment towards the tax liability of the individual
Advance Tax
in whose hands such income has been clubbed. In such case, it is open to the payer of
advance tax (i.e. spouse or the minor child) to apply for refund of advance tax so paid.
Mode of Clubbing Unless there are compelling circumstances the modes of clubbing cannot be changed by
cannot be changed the ITO. Merely, for the benefit of the revenue such mode cannot be changed.
for the benefit of the
revenue

Illustration 8:
A proprietary business was started by Smt. S. Mundhra in the year 2020. As on 1.4.2022 her capital in business
was ₹ 3,00,000.
Her husband, Shri V. Mundhra, gifted ₹ 2,00,000, on 10.04.2022, which amount Smt. S invested in her business
on the same date. Smt. S earned profits from her proprietary business for the financial years 2022-23, ₹ 1,50,000
and financial year 2023-24 ₹ 3,90,000. Compute the income, to be clubbed in the hands of S’s husband for the A.Y.
2024-25.
Solution :
Where asset transferred to spouse is invested in the proprietary business then proportionate share being calculated
in following manner shall be clubbed in the hands of transferor:

Income of such business * Value of such assets as on the 1st day of the P.Y.
Total investment in the business by the transferee as on the 1st day of the P.Y.
= ₹ 3,90,000 * ₹ 2,00,000 / ₹ 6,50,0001 = ₹ 1,20,000
Hence, ₹ 1,20,000 shall be clubbed in the hands of husband of Smt. S
1.
Computation of total investment in the business on 1-4-2023

Particulars Amount
Investment as on 1-4-2022 ₹ 3,00,000
Add: Investment of gift from husband on 10-4-2022 ₹ 2,00,000

374 The Institute of Cost Accountants of India


Total Income and Tax Liability of Individuals & HUF

Add: Net profit earned during the year 2022-23 (assumed reinvested in the business) ₹ 1,50,000
Total investment in the business as on 1-4-2022 ₹ 6,50,000
Income arising from accretion to transferred asset shall not be liable to clubbing.
Assume, Net profit earned during the year 2022-23 is retained in the business.
Alternatively, one can assume that net profit earned during the year 2022-23 is withdrawn. In such case income to
be clubbed shall be computed as under:

Income of such business * Value of such assets as on the 1st day of the P.Y
Total investment in the business by the transferee as on the 1st day of the P.Y.
= ₹ 3,90,000 * ₹ 2,00,000 / ₹ 5,00,000 = ₹ 1,56,000
Hence, ₹ 1,56,000 shall be clubbed in the hands of husband of Smt. S
1.
Computation of total investment in the business on 1-4-2023

Particulars Amount
Investment as on 1-4-2022 ₹ 3,00,000
Add: Investment of gift from husband on 10-4-2022 ₹ 2,00,000
Total investment in the business as on 1-4-2023 ₹ 5,00,000

Illustration 9:
Mr. Singh is a trader. Particulars of his income and those of the members of his family are given below. These
incomes relate to the previous year ended 31st March, 2024:
Particulars Amount
Income from business (Mr. Singh’s) 90,000
Salary derived from an educational institution by Mrs. Singh. She is the principal of the 50,000
institution
Interest on company deposits derived by Master Deep Singh (minor son). These deposits were 12,000
made in the name of Deep Singh by his father’s father about 6 years ago.
Receipts from sale of paintings and drawings made by minor Dipali Singh (minor daughter of 60,000
Mr. & Mrs. Singh and noted child artist)
Income by way of lottery earnings by Master Dipindar Singh (minor son of Mr. Singh) 6,000
Discuss whether the above will form part of the assessable income of any individual and also compute the assessable
income of Mr. Singh.
Solution :
Computation of Total Income for the A.Y. 2024-25
Amount
Particulars Details
Mr. Singh Mrs. Singh Minor Daughter
Salaries
Salary of Mrs. Singh 50,000
Profits and gains of business or profession
Business income 90,000
Sale of paintings & drawings (made by daughter) 60,000

The Institute of Cost Accountants of India 375


Direct Taxation

Amount
Particulars Details
Mr. Singh Mrs. Singh Minor Daughter
Income from other sources
Interest income of 1st minor son 12,000
Less: Exemption u/s 10(32) 1,500 10,500
Lottery Income of 2nd minor son 6,000
Less: Exemption u/s 10(32) 1,500 4,500
Gross Total Income 1,05,000 50,000 60,000
Less: Deduction under Chapter VI A Nil Nil Nil
Total Income 1,05,000 50,000 60,000

Illustration 10:
Balu is the Karta of a HUF, whose members derive income as given below:

Particulars Amount
Income from Balu’s own business 50,000
Mrs. Balu a dermatologist (taxable salary) 80,000
Minor son Deepak (earning interest on fixed deposits with ABC Ltd., which were gifted to him
15,000
by his grandfather)
Minor daughter Priya gave a dance performance and received remuneration 1,00,000
Deepak got winnings from lottery (gross) 2,00,000
Explain how the above will be taxed.
Solution :
Computation of Total Income for the A.Y. 2024-25

Particulars Details Balu Mrs. Balu Priya


Salaries
Salary of Mrs. Balu - 80,000 -
Profits & gains of business & profession 50,000 - -
Income from other sources
Interest on fixed deposit 15,000 - - -
Less: Exemption u/s 10(32) 1,500 - 13,500 -
Winning from lotteries - 2,00,000 -
Remuneration from dance performance - - 1,00,000
Gross Total Income 50,000 2,93,500 1,00,000
Less: Deduction under Chapter VI A Nil Nil Nil
Total Income 50,000 2,93,500 1,00,000
Note: Interest income of minor son Deepak will be taxable in the hands of his parent and not in the hands of his
grand father.

Illustration 11:
Ram & Mrs. Ram submit the following particulars of their income relevant for the assessment year 2024-25:
1. Mrs. Ram receives a salary of ₹ 30,000 (computed) and taxable allowance of ₹ 22,500 from G Ltd. in which
her father-in-law holds 25% of equity share capital and she herself holds 10% of equity capital.

376 The Institute of Cost Accountants of India


Total Income and Tax Liability of Individuals & HUF

2. Ram receives a salary of ₹ 11,340 (computed) from S Ltd., in which Mrs. Ram, her sisters and brother hold
21% of equity share capital.
3. Ram is a (10% share) partner in EFG, a partnership firm, which is engaged in the business identical to G
Ltd. and receives ₹ 75,000 as salary and interest from the firm during the relevant previous year. None of the
relatives of Ram has any interest in the firm.
4. Mrs. Ram is employed by the firm EFG on a salary of ₹ 9,000 p.a. (computed)
5. Interest of minor daughter of Ram from a partnership firm is ₹ 43,500
6. Mrs. Ram transferred ₹ 1,00,000 on 20-5-1973 and ₹ 70,000 on 15-6-1973 to her daughter-in-law, who invested
the aforesaid fund in a partnership firm at 12% p.a. Out of interest income, her daughter-in-law has purchased
debentures in an Indian companies and during the previous year she receives ₹ 2,000 as interest.
Find out taxable income of Mr. and Mrs. Ram on the following assumptions:
a. That Ram is not a beneficial shareholder in G Ltd. throughout the previous year.
b. That Ram holds 10 shares in G Ltd. only during 15 days of the previous year.

Solution :
Computation of total income for A.Y. 2024-25
Case a Case b
Particulars
Mr. Ram Mrs. Ram Mr. Ram Mrs. Ram
Salary from G Ltd. (computed) 52,500 52,500
Salary from S Ltd. (computed) 11,340 11,340
Business income (salary and interest from EFG) 75,000 75,000
Salary from EFG (computed) 9,000 9,000
Interest income of son’s wife (int. on deb.)# 8,400 8,400
Income of minor daughter 42,000 42,000
(₹ 43,500 – ₹ 1,500) [sec. 10(32)]
Total Income 75,000 1,23,240 1,69,500 28,740
#
for transfer before 31-5-1973 clubbing is not applicable

Quick MCQs:-

1. Clubbing provisions under Sec. 64(1)(vi) are applicable where the asset is transferred by an individual for
inadequate consideration to –
(a) Daughter’s Husband
(b) Son’s Wife
(c) Major Son
(d) Major Daughter

2. As per Sec. 64(1A), income accruing to a minor shall be clubbed in the income of –
(a) Father
(b) Mother

The Institute of Cost Accountants of India 377


Direct Taxation

(c) Father or Mother at their option


(d) Parent whose income before this clubbing is greater

3. If any income has to be clubbed under Sec. 64, it will be clubbed under the head
(a) Income from Other Sources
(b) Relevant Head to which it belongs
(c) Choice of assessee
(d) None of the above

4. Where a member of a HUF has converted or transferred his self-acquired property for inadequate
consideration into joint family property, income arising there from is taxable.
(a) As the income of the Transferor Member
(b) In the hand of the HUF
(c) In the hands of Karta of HUF
(d) Completely exempt from tax

5. Mr. Grits cash D 1,00,000 to his brother’s wife Mrs. B, Mr. B gifts cash of D 1,00,000 to Mrs . A From
cash gifted to her, Mrs. A invests in a fixed deposit, income there from is D 10,000. Aforesaid D 10,000
will be included in the total income of –
(a) Mrs. A
(b) Mr. B
(c) Mrs. A
(d) Mrs. B

378 The Institute of Cost Accountants of India


Total Income and Tax Liability of Individuals & HUF

Set off and Carry Forward of Losses 3.2


3.2.1 Introduction
For computation of Gross Total
Income (GTI), income from
various sources is computed
under the five heads of income.
If all the sources and heads are having positive income (i.e. profit) then the same can simply be added to compute
GTI. However, if certain source(s) or certain head(s) have negative income (i.e. loss) then such loss needs to be
adjusted with income of another source(s) or head(s). Set off means adjustment of loss from one source or one
head against income from another source or another head.
If a negative income is not fully set off in the current year, then the unabsorbed loss shall be carried forward to
subsequent years subject to certain restrictions and conditions [e.g. Income from other sources (other than losses
from activity of owning and maintaining horse races) cannot be carried forward.]

Loss which could


not be adjusted shall
be carried forward
to next year subject
to certain conditions
and exceptions

3.2.2 Inter Source adjustment (Intra-Head adjustment) [Sec. 70]


When the net result of any source of income is loss, it can be set off
against income from any other source under the same head. Sec. 70
deals with the set off of loss from one source against income from
another source under the same head of income, subject to the following
exceptions –
a. Long term capital loss can be set off only against long term capital
gain [Sec. 70(3)]. However, short-term capital loss can be set off
against short term as well as long term capital gains [Sec. 70(2)].

The Institute of Cost Accountants of India 379


Direct Taxation

Example 1: Computation of Income under the head “Capital gains” of Mr. X for the A.Y. 2024-25
Particulars Case A Case B
Short term capital gain 80,000 (90,000)
Long term capital gain (40,000) 1,20,000
Income under the head “Capital gains” 80,000 30,000
b. Loss of a speculation business can be set off only against the profits of a speculation business, under the head
‘Profits and gains of business or profession’. However, loss from a non-speculative business can also be set off
against income from speculation business [Sec. 73(1)]
Example 2: Computation of “Profits & gains of business or profession” of Mr. X for the A.Y. 2024-25
Particulars Case A Case B
Speculation business 80,000 (90,000)
Non speculation business (40,000) 1,00,000
Profits & gains of business or profession 40,000 1,00,000
c. Loss of a specified business covered u/s 35AD can be set off only against the profits of other specified
business. However, loss from a non-specified business can be set off against income from specified business.
d. Loss incurred in activity of owning and maintaining race horses can be set off against income from such
activity only [Sec. 74A]
Example 3: Computation of “Income from other sources” of Mr. X for the A.Y. 2024-25

Particulars Case A Case B


Activity of owning and maintaining race-horses 80,000 (90,000)
Rent from land (20,000) 1,30,000
Income from other sources 60,000 1,30,000

Taxpoint: Above provisions provides restriction on losses, however income from aforesaid source (i.e. a to d)
is available for setting off any loss from any other source under the same head.
e. Loss from a source, income of which is exempt u/s 10, cannot be set off against any income.
f. No loss can be set off against winning from lotteries, crossword puzzles, races, card games, gambling or
betting, etc. [Sec. 58(4) & 115BB]. Similarly, set off of losses is not permissible against unexplained income,
investment, money, etc. chargeable u/s 68 / 69A / 69B / 69C / 69D [Sec. 115BBE]
Example 4: Computation of “Income from other sources” of Mr. X for the A.Y. 2024-25

Particulars Case A Case B


Activity of owning and maintaining race-horses 10,000 (90,000)
Lottery income 20,000 1,30,000
Losses on letting out furniture not related to business (40,000) (50,000)
Income from other sources 20,000 1,30,000

380 The Institute of Cost Accountants of India


Total Income and Tax Liability of Individuals & HUF

Illustration 12:
Compute gross total income of Mr. X in following cases -

Source of income Case I Case II


Income from house property (A) 30,000 40,000
Income from house property (B) (10,000) (25,000)
Speculation income 80,000 (70,000)
Business income (30,000) 50,000
Income from activity of owning and maintaining race-horses business (A) (50,000) 10,000
Income from activity of owning and maintaining race-horses business (B) 20,000 (6,000)
Income from agricultural business (25,000) 10,000
Short term capital gain (transaction A) 30,000 (20,000)
Short term capital gain (transaction B) (10,000) 5,000
Long term capital gain (transaction A) (30,000) 45,000
Long term capital gain (transaction B) 10,000 (2,000)
Income from lottery 40,000 -
Income from horse races 10,000 25,000
Income on card games (5,000) (3,000)
Interest on securities 20,000 10,000
Solution :
Computation of gross total income of Mr. X for the A.Y. 2024-25

Case I Case II
Particulars
Details Amount Details Amount
Income from House Property
House Property A 30,000 40000
House Property B (10,000) 20,000 (25000) 15,000
Profits & Gains of Business or Profession
Speculation income 80,000 (70000) Nil3
Other business (30,000) 50,000 50,000
Income from agricultural business (Exempted) Nil Nil
Capital Gains
Short term capital gain
Transaction A 30,000 (20000)
Transaction B (10,000) 20,000 5000
Long term capital gain
Transaction A (30,000) 45000
Transaction B 10,000 Nil2 (2000) 28,000
Income from other sources
Casual income
Income from lottery 40,000 -
Income from horse races 10,000 25000

The Institute of Cost Accountants of India 381


Direct Taxation

Case I Case II
Particulars
Details Amount Details Amount
Income on card games (losses not to be considered) Nil 50,000 Nil 25,000
Income from activity of owning & maintaining race-horses
Business (A) (50,000) 10000
Business (B) 20,000 Nil 1
(6000) 4,000
Other income
Interest on securities 20,000 10,000
Gross Total Income 1,60,000 1,32,000
Losses to be carried forward
1.
Loss from activity of owning & maintaining race horse (30,000) -
2.
Long term capital loss (20,000) -
3.
Speculation loss - (70,000)

3.2.3 Inter head adjustment [Sec. 71]


Where in respect of any assessment year, the net result of
any head of income is a loss, the same can be set off against
the income under any other heads for the same assessment
year, subject to the following exceptions:
a. Capital gains: Loss under the head ‘Capital gains’
cannot be set off against income under any other head.
However, loss under any other head, e.g. business loss,
shall be allowed to be set off against income under the
head ‘Capital gains’
Example 5: Computation of “Gross total income” of
Mr. X for the A.Y. 2024-25

Particulars Case A Case B


Long term capital gain 80,000 (90,000)
Short term capital gain (60,000) (30,000)
Income from house property (20,000) 1,30,000
Gross total income Nil 1,30,000
b. Loss of a speculation business: Loss under the head ‘Profits and gains of business or profession’ due to
speculation business cannot be set off against any other income except profits of speculation business.
However, loss under any other head, e.g. loss from house property, shall be allowed to be set off against
income of a speculation business.
c. Loss of a specified business covered u/s 35AD cannot be set off against income taxable under other head.
However, loss from other head can be set off against income from specified business.
d. Loss from activity of owning and maintaining race-horses: Loss under the head ‘Income from other
sources’ due to activity of owning and maintaining race-horses cannot be set off against any other income
except profit from activity of owning and maintaining race-horses [Sec. 74A]. However, loss under any other

382 The Institute of Cost Accountants of India


Total Income and Tax Liability of Individuals & HUF

head, e.g. business loss, shall be allowed to be set off against income from activity of owning and maintaining
race-horses.
Taxpoint: Above provisions provides restriction on losses, however income from aforesaid source (i.e. a to d)
is available for setting off any loss from any other head of income.
e. Loss under the head ‘Income from house property’: Loss in excess of ₹ 2,00,000 under the head ‘Income
from house property’ cannot be set off with income under other heads of income.
Example 6: Computation of “Gross total income” of Ms. M for the A.Y. 2024-25
Particulars Case A Case B
Profit and gains of business or profession 2,80,000 2,80,000
Income from house property (20,000) (2,30,000)
Gross total income 2,60,000 80,000
f. Loss under the head Profits and Gains of Business or Profession: Loss under the head “Profits and gains of
business or profession” cannot be setoff from income under the head “Salaries”.
g. Loss from a source, income of which is exempt u/s 10: Such loss cannot be set off against any taxable
income.
h. Income from winning from lotteries, etc.: Any loss cannot be set off against winning from lotteries, crossword
puzzles, races, card games, gambling, betting, etc. [Sec. 58(4) & 115BB]. Similarly, set off of losses is not
permissible against unexplained income, investment, money, etc. chargeable u/s 68 / 69A / 69B / 69C / 69D
[Sec. 115BBE]
Example 7: Computation of “Gross total income” of Mr. X for the A.Y. 2024-25
Particulars Case A Case B
Salaries 1,20,000 1,20,000
Profits and gains of business or profession (50,000) (30,000)
Income from house property 20,000 (20,000)
Gross total income 1,20,000 1,00,000

Illustration 13 :
Compute gross total income of Mr. Jacky from following data -
Source of income Amount
Income under the head ‘Salaries’ 2,60,000
Income from house property (A) 60,000
Income from house property (B) (2,80,000)
Speculation income 20,000
Business income (1,30,000)
Income from activity of owning and maintaining race-horses (1,50,000)
Income from agricultural business (1,25,000)
Short term capital gain 30,000
Long term capital gain (1,00,000)
Income from lottery 10,000
Income from horse races 1,70,000
Interest on securities after adjusting interest expenses and other expenses (70,000)

The Institute of Cost Accountants of India 383


Direct Taxation

Solution :
Computation of gross total income of Mr. Jacky for the A.Y.2024-25

Particulars Details Amount Amount


Salaries 2,60,000
Income from house property
House property A 60,000
House Property B [#Max. limit, balance shall be carried forward] (2,80,000) (2,00,000)#
Profits & gains of Business or Profession
Speculation income 20,000
Other business income (1,30,000)
Income from agricultural business [exempted u/s 10(1)] Nil (1,10,000)
Capital Gains
Short term capital gain 30,000
Long term capital gain (1,00,000) Nil
Long term capital loss cannot be set off against any income & shall be
carried forward
Income from Other Sources
Casual income
Income from lottery 10,000
Income from horse races 1,70,000 1,80,000
As no loss can be set off against such income.
Income from activity of owning and maintaining race-horses (1,50,000) Nil
Loss from activity of owning and maintaining race-horses cannot be set
off against any income & shall be carried forward
Other income
Interest on securities (70,000)
Gross Total Income (90,000)1 1,80,000
Conclusion
From salary income, losses under the head income from other sources ₹ 70,000 is adjusted first and then loss
under the head ‘Income from house property’ considering following:
¾¾ Loss under the head ‘Profits and gains of business or profession’ cannot be set-off against salary income; and
¾¾ Loss under the head income from other sources, if any, cannot be carried forward.
Thereafter, loss under the head ‘Income from house property’ of ₹ 10,000 and losses under the head ‘Profits
& gains of business or profession’ up to ₹ 20,000 is adjusted against Short term capital gain of ₹ 30,000 and
remaining losses under the head Profits and gains of business or profession ₹ 90,000 and loss under the head
‘Income from house property’ ₹ 20,000 (including loss over and above ₹ 2,00,000) shall be carried forward.
Alternatively, loss under the head Income from house property of ₹ 30,000 and loss under the head Profits &
gains of business or profession of ₹ 80,000 can be carried forward

384 The Institute of Cost Accountants of India


Total Income and Tax Liability of Individuals & HUF

Test Yourself
1. Compute Gross Total Income for the assessment year 2024-25 and losses to be carried forward.

Particulars Amount
Net Profit from cotton business 1,60,000
Net loss from automobile business (90,000)
Loss from house property in Kolkata (75,000)
Income from house property in Mumbai 70,000
Loss from Speculative business (1,20,000)
Income from Salary 72,000
Short term capital loss on jewellery (1,20,000)
Long term capital gain on Shares 90,000
2. From the following information, compute income of Ms. Tewari for the P.Y. 2023-24:

Particulars Amount (₹ )
Income from house 1 (let out) 2,00,000
Loss from house 2 (self-occupied) 50,000
Salary Income 5,00,000
Loss from non-speculative Business 3,00,000
Income from speculative Business 1,00,000
Short Term Capital Loss 60,000
Long term Capital gain 50,000

Hints
1. Gross Total Income: ₹ 1,37,000; 2. ₹ 5,00,000

3.2.4 General notes


1. Sec. 70 vs Sec. 71: First, intra head set-off shall be made, thereafter inter head set-off shall be made. In other
words, sec. 71 will be applicable after application of sec. 70.
Illustration 14:
Smart has computed his tax liability as under –
Particulars Details Amount
Income from business A 4,50,000
Long term capital gain 20,000
Less: Income from business B u/s 71 (10,000) 10,000
Income from other sources 50,000
Gross Total Income 5,10,000
Less: Deduction u/s 80C to 80U -
Total Income 5,10,000
Tax liability 15,080
Comment on the above computation.

The Institute of Cost Accountants of India 385


Direct Taxation

Solution :
Computation made by Smart is incorrect, as because first intra head set-off shall be made, thereafter inter head
set-off can be made. The correct computation is shown below -
Computation of total income and tax liability of Smart for the A.Y. 2024-25
Particulars Details Amount
Profits & Gains of Business or Profession
Income from business A 4,50,000
Income from business B u/s 70 (10,000) 4,40,000
Capital Gains
Long term capital gain 20,000
Income from other sources 50,000
Gross Total Income 5,10,000
Less: Deduction u/s 80C to 80U -
Taxable Income 5,10,000
Tax liability 16,640
2. Priority of set off u/s 71: The Act does not lay any priority relating to set off of losses, hence, losses
which cannot be carried forward should be adjusted first.
Illustration 15 :
Mr. Bhola has furnished you the following data –
Income from house property (₹ 1,30,000)
Salaries (Net) ₹ 80,000
Income from other sources (₹ 90,000)
Income from lotteries ₹ 3,50,000
Mr. Bhola is seeking your advice relating to set off and carry-forward.
Solution :
Statement showing application of sec. 71
Particulars Amount
Salaries 80,000
Income from house property (1,30,000)
Income from other sources
Winning from lotteries 3,50,000
Other income (90,000)
Gross total income 3,50,000
Conclusion
Casual income shall be fully taxable as no loss can be set off against such income. 3,50,000
Losses to be carried forward
a) Loss under the head “Income from house property” (1,30,000)
b) Loss under the head “Income from other sources”, as such loss cannot be carried forward. Nil
Income under the head ‘Salaries’ is first adjusted with the loss under the head ‘Income from other sources’ as the
same cannot be carried forward. Though loss under the head ‘Income from other sources’ is ₹ 90,000 and such
loss could be adjusted with income under the head ‘Salaries’ only to the extent of ₹ 80,000 still the remaining loss
of ₹ 10,000 cannot be carried forward.

386 The Institute of Cost Accountants of India


Total Income and Tax Liability of Individuals & HUF

3. Mandatory feature of sec. 70 and 71: Assessee is bound to follow sec. 70 and 71, there is no choice
or option whether to set off the loss or not. Even partial set off is not permissible when entire loss can
otherwise be set off.

Illustration 16:
Mr. Tej Singh has the following income details – ₹

Income from house property (30,000)


Salaries 4,80,000
Income from other sources 30,000
He has computed his Gross total income for the A.Y.2024-25 as under

Particulars Amount
Income from house property (10,000)
Salaries 4,80,000
Income from other sources 30,000
Gross total income 5,00,000
Carry forward of loss under the head income form house property 20,000
– Comment
Solution :
Assessee is bound to follow sec.70 and 71 and there is no choice or option whether to set off the loss or not. Even
partial set off is not permissible when full loss can otherwise be set off. Hence in the given case income of Mr. Tej
Singh shall be ₹ 4,80,000 (carry forward of loss shall not be permissible) as computed under:

Particulars Amount
Income from house property (30,000)
Salaries 4,80,000
Income from other sources 30,000
Gross total income 4,80,000
4. Set-off of clubbed income: From the clubbed income, one can set off the losses.

3.2.5 Carry Forward of Loss


In case where the income of an assessment year is insufficient to set off the losses of the year then such losses
(which could not be set off) can be carried forward to subsequent assessment year(s) for set off against income of
such subsequent year(s). However, all losses cannot be carried forward, e.g. losses under the head ‘Income from
other sources’ (other than loss from ‘Activity of owning and maintaining race-horses’) cannot be carried forward.
Following losses can be carried forward:
1. Loss under the head ‘Income from house property’ [Sec. 71B]
2. Loss under head “Profits and gains of business or profession” other than speculation loss [Sec. 72]
3. Loss from speculation business [Sec. 73]
4. Loss from specified business covered u/s 35AD [Sec. 73A]
5. Loss under the head ‘Capital gains’. [Sec. 74]
6. Loss from ‘Activity of owning and maintaining race horses’. [Sec. 74A]

The Institute of Cost Accountants of India 387


Direct Taxation

3.2.6 Loss under the head ‘Income from House Property’ [Sec. 71B]
Loss under the head ‘Income from house property’ can be carried forward and can be set off against income under
the same head only.
Period for which carry-forward shall be allowed: 8 assessment years immediately succeeding the assessment
year in which such loss is first computed.
Filing of return: Loss under the head ‘Income from house property’ can be carried forward even when a belated
return is filed [as sec. 80 and 139(3) are not applicable to sec. 71B].

Illustration 17:
Compute Gross total income of Mrs. Shikha from following details for the A.Y.2024-25 ₹
Income from house property A 60,000
Income from house property B (1,50,000)
Income from house property C 1,00,000
Income from other sources 1,00,000
Losses u/s 22 for the A.Y. 2023-24 (30,000)
Losses u/s 22 for the A.Y. 1998-99 (15,000)
Losses u/s 56 for the A.Y. 2023-24 (45,000)
Solution :
Computation of gross total income of Mrs. Shikha for the A.Y. 2024-25
Particulars Details Details Amount
Income from house property
House property A 60,000
House property B (1,50,000)
House property C 1,00,000 10,000
Loss u/s 22 for the A.Y. 2023-24 2
(10,000) Nil
Income from other sources 1,00,000
Gross Total Income 1,00,000
Notes
1. Remaining unabsorbed loss relevant to the A.Y. 2023-24 ₹ 20,000 (i.e. ₹ 30,000 – ₹ 10,000) shall be carried
forward to subsequent assessment years (maximum up to A.Y. 2031-32).
2. Loss under the head ‘Income from house property’ relevant to the A.Y. 1998-99, cannot be carried forward.
3. Loss under the head ‘Income from other sources’ cannot be carried forward.
4. Sec. 71 shall apply only after application of sec. 70. Hence, current year loss of house property B cannot be
set off against income from other sources.

3.2.7 Carry forward & set off of business loss other than speculation loss [Sec. 72]
Loss under the head “Profits and gains of business or profession” (other than speculation loss) can be carried
forward and set off against income under the same head.
Note: For this purpose business profit includes profits derived from a business activity but assessable under the
heads other than ‘Profit and gains of business or profession’, e.g. Dividend income when shares are held as stock,
though taxable under the head ‘Income from other sources’ but business loss can be set off against such income.

388 The Institute of Cost Accountants of India


Total Income and Tax Liability of Individuals & HUF

Period for which carry-forward shall be allowed: 8 assessment years immediately succeeding the assessment
year in which such loss is first computed.
Academic note: In the assessment year 2023-24, losses prior to the assessment year 2015-2016 cannot be set off.
Exceptions to the above period
1. Closure of business due to specified reasons [Sec. 33B]
Situation: Where any loss remains unabsorbed of a business undertaking, which is discontinued due to
damages caused by -
i. flood, typhoon, hurricane, cyclone, earthquake or other convulsion of nature; or
ii. riot or civil disturbance; or
iii. accidental fire or explosion; or
iv. action by an enemy or action taken in combating an enemy (whether with / without a declaration of war)
Condition: Such business is re-established, reconstructed or revived by the assessee, within a period of 3 years
from the end of the previous year in which such mishap took place.
Treatment: Losses of such business [including the past eligible losses (to be carried forward)] shall be
adjusted with profit of the year in which business is so revived and if the loss cannot be wholly setoff, then it
shall further be allowed to be carried forward for 7 years.
2. Set off of losses relating to the year of closure of business against deemed profit [Sec. 41(5)]
Situation: Where any part of loss (not being a speculation loss), which arose during the previous year remains
unabsorbed in the previous year in which business ceased to exist.
Treatment: Such loss shall be allowed to be carried forward for any number of years (without restriction of 8
years) against income chargeable to tax u/s 41(1); (3); (4); (4A).
Taxpoint: Such loss can be set off even if the return of loss is not submitted in time.
Notes
1. Business need not be continued: The business losses can be carried forward, even the business in respect of
which the loss was originally computed, is not carried on during the previous year.
2. Filing of return in time: As per sec. 80, the business loss cannot be carried forward unless it is determined in
pursuance of a return filed by the assessee u/s 139(3), within time as prescribed u/s 139(1).
3. Treatment of unabsorbed depreciation, etc.: Unabsorbed depreciation, unabsorbed scientific research
expenditure and unabsorbed family planning expenditure are not covered by sec. 72. Such losses can be
carried forward for any number of years.
4. Order of set off: In case of insufficient profit, losses shall be set off in the following order:
a. Current year’s depreciation [Sec. 32(1)], capital expenditure on scientific research [Sec. 35(1)] and
capital expenditure on family planning [Sec. 36(1)(ix)];
b. Brought forward business or profession losses [Sec. 72(1)],
c. Unabsorbed depreciation [Sec. 32(2)], unabsorbed expenditure on family planning [Sec. 36(1)(ix)],
unabsorbed capital expenditure on scientific research [Sec. 35(4)]

The Institute of Cost Accountants of India 389


Direct Taxation

5. Loss from specified business covered u/s 35AD (i.e, cold chain facility, cross-country natural gas pipeline,
etc.) shall be adjusted only from profit from specified business [Further refer sec.73A].
6. Assessee must be same who incurred the loss: Business losses can be carried forward and set off against the
profits of the assessee who incurred the loss i.e. assessee must be same to carry forward the loss. However,
this rule has the following exceptions –
a. Amalgamation: Business losses and unabsorbed depreciation of an amalgamating company can be set-
off against the income of the amalgamated company if the amalgamation is within the meaning of sec.
72A of the Income tax Act. (Discussed later in this chapter).
b. Succession: Business losses and unabsorbed depreciation of a proprietary concern or a partnership firm
or a specified company succeeded by a company or limited liability partnership as per sec. 47(xiii),
(xiiib) and (xiv), can be carried forward by succeeded company or limited liability partnership.
c. Inheritance: As per sec. 78(2), where the assessee acquires the business through inheritance, losses of
such business may be carried forward for balance number of years.
d. Demerger: In case of demerger, loss of demerged company shall be carried forward and set-off by
resulting company. (Discussed later in this chapter).
Note: In following case losses cannot be carried forward
i. Business, of an HUF where the business of the HUF is taken over by the Karta of HUF;
ii. Proprietorship business taken over by a firm in which proprietor is one of the partner;
iii. A firm being succeeded by another firm;
iv. A firm where the business of the firm is taken over by one of the partner of the firm,

3.2.8 Set off and Carry forward of unabsorbed depreciation


Set off and carry forward of unabsorbed depreciation shall be governed by sec. 32(2) and not by sec. 72. Depreciation,
which could not be fully absorbed in any previous year, owing to -
 there being no profits or gains chargeable for that previous year; or
 the profits or gains chargeable being less than the amount of depreciation.
Tax treatment: Allowance or the part of the allowance of depreciation which remains unabsorbed shall be (subject
to sec. 72 and sec. 73) added to the amount of the depreciation for the following previous year and deemed to be
the depreciation-allowance for that previous year, and so on for the succeeding previous years.
Taxpoint: Unabsorbed depreciation shall be allowed to be carried forward for any number of years and such
carried forward unabsorbed depreciation may be set off against any income, other than -
 Income under the head “Salaries”
 Winning from lotteries, cross word puzzles, etc.
Notes
1. Continuation of business: Unabsorbed depreciation can be carried forward even if the business in respect of
which the loss was originally computed, is not carried on during the previous year.
2. Filing of return: Unabsorbed depreciation can be carried forward even if the return of income has not been
filed within time.

390 The Institute of Cost Accountants of India


Total Income and Tax Liability of Individuals & HUF

Illustration 18:
Mr. Arun is running several businesses since last so many years. From the following details, compute his taxable
business income:

Year to Whether Year of disc-


Current year Brought forward
Nature of business which b/f business ontinuation
income (b/f) loss
loss relates continued of business
Readymade garments 1,00,000 2,30,000 2017-18 Yes NA
1,70,000 2003-04
Retail of cosmetics 80,000 50,000 Unabsorbed Yes NA
2018-19
depreciation
Whole sale of soft-toys Nil 80,000 2020-21 No 2021-22
30,000 being bad
Wholesale of fruits 70,000 2006-07 No 2006-07
debt recovery
30,000
Optical 70,000 2005-06 No 2006-07
Bad debt recovery
90,000 - - Yes NA
Investment business 15,000
Dividend from - - Yes NA
foreign company
40,000
Medicines - - No 1997-98
Bad debt recovery
Manufacturing of school 2017-18 no
- 40,000 No 2018-19
uniform return filed
Solution :
Computation of gross total income of Mr. Arun for the A.Y. 2024-25

Particulars Details Amount


Profits & gains of business or profession
Readymade garments Current year income 1,00,000
Less: Brought forward loss (2,30,000) (1,30,000)
Retail of cosmetics Current year income 80,000
Whole sale of soft-toys Current year income Nil
Less: Brought forward loss (80,000) (80,000)
Wholesale of fruits Current year income being bad debt recovery 30,000
Less: Brought forward loss (30,000)1 Nil
Optical-business Current year income being bad debt recovery 30,0001
Investment-business 90,000
Medicine-business Current year income (being bad debt recovery) 40,000
Manufacturing of school uniform Current year income Nil2
30,000
Income from other sources: Dividend income 15,000
45,000

The Institute of Cost Accountants of India 391


Direct Taxation

Particulars Details Amount


Less: Unabsorbed depreciation of retail of cosmetics business (45,000)#
Gross total income Nil
#
Though unabsorbed depreciation is ₹ 50,000 still only ₹ 45,000 is claimed to be set off and remaining to be
carried forward for unlimited years, as there is insufficient profit.
Notes
1. Losses from wholesale of fruits business which arose during the P.Y. 2006-07, being the year in which
business ceased to exist and which could not be set off against any other income of that previous year or any
subsequent years can be set off against income chargeable to tax u/s 41(4) i.e. bad debt recovery. Such loss
is allowed to be carried forward for any number of years (without restriction of 8 years). However, in case of
Optical business, brought forward loss relates to P.Y. 2005-06 whereas business ceases to exist in P.Y. 2006-
07, therefore such loss cannot be set off against income chargeable u/s 41(4).
2. As per sec. 80, business loss cannot be carried forward unless it is determined in pursuance of a return filed
by the assessee u/s 139(3), within time as prescribed u/s 139(1).

3.2.9 Carry forward and Set off of Speculation loss [Sec. 73]
1. Speculative transaction means a transaction in which contract for purchase and sale of any commodity
including stock and shares, is periodically or ultimately settled otherwise than by the actual delivery or transfer
of the commodity or scripts. [Sec. 43(5)]
2. As per explanation to sec. 73, where any part of the business of a company consists of purchase and sale of
shares of other companies, such company shall be deemed to be carrying on speculation business to the extent
of purchase and sale of shares. However, this rule is not applicable in case of companies -
a. of which gross total income mainly consists of income which is chargeable under the head “Income from
house property”, “Capital gains”, and “Income from other sources”; or
b. of which principal business is the business of trading in shares or banking or granting of loans and
advances.
Notes: Above explanation covers only transactions of purchase and sale of shares. Debentures, units of
UTI or of Mutual Funds are not covered by this explanation.
Treatment
Losses from speculative transactions or business can be carried forward and set off against income from speculative
business only.
Period for which carry forward shall be allowed: 4 assessment years immediately succeeding the assessment
year in which such loss is first computed.
Academic note: In the assessment year 2024-25, losses prior to the assessment year 2020-21 cannot be set off.
As per sec. 73(3) in respect of allowance on account of depreciation or capital expenditure on scientific research,
the provisions of sec. 73 shall not apply it can be carried forward to any number of years.
Notes
1. Continuity of business: It is not necessary that the same speculation business must be continued in the year
of carry forward and set off of the losses.
2. Filing of return: As per sec. 80, the loss cannot be carried forward unless it is determined in pursuance of a
return filed by the assessee u/s 139(3), within time as prescribed u/s 139(1)
3. Derivative Trading: An eligible transaction in respect of trading in derivative referred to in sec.2(ac) of the
Securities Contracts (Regulation) Act, 1956 carried out in a recognised stock exchange shall not be treated

392 The Institute of Cost Accountants of India


Total Income and Tax Liability of Individuals & HUF

as speculative transaction. Similarly, an eligible transaction in respect of trading in commodity derivatives


carried out in a recognised stock exchange (and liable for Commodities Transaction Tax in case of trading
in commodity derivatives other than agricultural commodity derivatives) shall not be treated as speculative
transaction.
Taxpoint: In respect of trading in agricultural commodity derivatives, the requirement of chargeability of
commodity transaction tax is not applicable.
4. Losses of illegal speculative business: Loss arising from illegal speculative business cannot be carried
forward to the subsequent years for set off against the profits of another speculative business.

Illustration 19:
Compute total income of X Ltd. under following cases –
Business A: Business of Ice cream
Business B: Business consists of purchase and sale of shares of other companies (being treated as speculative
business as per explanation of sec. 73)
Case 1 Case 2 Case 3 Case 4
Particulars
A B A B A B A B
Income of P.Y. 2023-24 80,000 20,000 75,000 85,000 (56,000) 1,50,000 3,00,000 (50,000)
B/f loss of P.Y. 2022-23 70,000 65,000 85,000 50,000 90,000 65,000 30,000 90,000
Unabsorbed depreciation - - - - - - - 20,000
Solution :
Computation of total income of X Ltd. for the A.Y. 2024-25
Case 1 Case 2 Case 3 Case 4
Particulars
A B A B A B A B
Income of P.Y. 2023-24 80,000 20,000 75,000 85,000 (56,000) 1,50,000 3,00,000 (50,000)
Income after sec. 70 80,000 20,000 75,000 85,000 - 94,000 3,00,000 (50,000)
B/f loss of P.Y. 2022-23 70,000 65,000 85,000 50,000 90,000 65,000 30,000 90,000
Unabsorbed depreciation - - - - - - - 20,000
10,000 (45,000) (10,000) 35,000 (90,000) 29,000 2,50,0001 (50,000)
Taxable income after
application of sec. 72 & 10,000 25,000 Nil 2,50,000
73 for the A.Y. 2024-25
P.Y.23-
24
P.Y. P.Y.
Loss to be carried forward (50,000)
- 2022-23 2022-23 - -
to next assessment year P.Y.22-
(45,000) (61,000) 23
(90,000)
Unabsorbed depreciation
- - - - - - - -
to be carried forward
1.
After adjusting unabsorbed depreciation of business B.

The Institute of Cost Accountants of India 393


Direct Taxation

3.2.10 Carry forward & set off of loss from specified business covered u/s 35AD [Sec.73A]
Losses from specified business covered u/s 35AD can be carried forward and set off against income from other
specified business (whether eligible for deduction u/s 35AD or not).
Period for which carry-forward shall be allowed: No time limit is prescribed.
Filing of return: As per sec. 80, the loss cannot be carried forward unless it is determined in pursuance of a return
filed by the assessee u/s 139(3), within time as prescribed u/s 139(1).

Illustration 20:
MNP Ltd. commenced operations of the business of a new four-star hotel in Chennai on 1-4-2023. The company
incurred capital expenditure of ₹ 40 lakh during the period January, 2023 to March, 2023 exclusively for the above
business, and capitalized the same in its books of account as on 1st April, 2023. Further, during the Previous Year
2023-24, it incurred capital expenditure of ₹ 2.5 crore (out of which ₹ 1 crore was for acquisition of land) exclusively
for the above business. Compute the income under the heading “profits and gains of business or profession” for the
assessment year 2024-25, assuming that MNP Ltd. has fulfilled all the conditions specified for claim of deduction
u/s 35AD and has not claimed any deduction under Chapter VI-A under the heading “C.-Deductions in respect of
certain incomes”. The profits from the business of running this hotel (before claiming deduction u/s 35AD) for
the A.Y. 2024-25 is ₹ 80 lakhs. Assume that the company also has another existing business of running a four-star
hotel in Kanpur, which commenced operations 5 years back, the profits from which was ₹ 130 lakhs for assessment
year 2024-25.
Solution :
Computation of business income of MNP Ltd. for A.Y. 2024-25
Particulars ₹ in lakhs
Income from specified business covered u/s 35AD [Working 1] (110)
Income from another hotel business 130
Profit and gains of business or profession 20
Working 1: Computation of income from specified business u/s 35AD
Particulars ₹ in lakhs ₹ in lakhs
Net profit from specified business before deduction u/s 35AD 80
Less: Deduction u/s 35AD being capital expenses incurred
- Capital expenditure incurred in earlier year 40
- Capital expenditure incurred in the previous year less cost of acquisition of land 150 190
Income from specified business covered u/s 35AD 110

3.2.11 Carry forward and set off of capital loss [Sec. 74]
Losses under the head ‘Capital gains’ can be carried forward and set off against income under the same head,
subject to the restriction that the loss on transfer of long-term capital assets can be set off only against long term
capital gain.
Taxpoint: Loss on transfer of short-term capital assets can be set off against any income under the head capital gain
(whether short-term or long-term).
Period for which carry-forward shall be allowed: 8 assessment years immediately succeeding the assessment
year in which such loss is first computed.
Academic note: In the assessment year 2024-25, losses prior to the assessment year 2016-2017 cannot be set off.
Filing of return: As per sec. 80, the loss cannot be carried forward unless it is determined in pursuance of a return

394 The Institute of Cost Accountants of India


Total Income and Tax Liability of Individuals & HUF

filed by the assessee u/s 139(3), within time as prescribed u/s 139(1).
Illustration 21:
Compute taxable income under following cases for the A.Y. 2024-25:
Case 1 Case 2 Case 3 Case 4
Particulars
STCG LTCG STCG LTCG STCG LTCG STCG LTCG
Income of the P.Y. 2023-24 1,00,000 (30,000) (30,000) 1,00,000 1,00,000 (20,000) (30,000) 1,00,000
B/f loss of P.Y. 2022-23 50,000 - - 50,000 60,000 50,000 10,000 20,000
Solution :
Computation of taxable income for the A.Y.2024-25

Case 1 Case 2 Case 3 Case 4


Particulars
STCG LTCG STCG LTCG STCG LTCG STCG LTCG
Income of the P.Y.2023-24 1,00,000 (30,000) (30,000) 1,00,000 1,00,000 (20,000) (30,000) 1,00,000
Taxable income after
1,00,000 (30,000) - 70,000 1,00,000 (20,000) - 70,000
application of sec. 70
Brought forward loss of
50,000 - - 50,000 60,000 50,000 10,000 20,000
P.Y.2022-23
50,000 (30,000) - 20,000 40,000 (70,000) - 40,000
Taxable income after
application of sec. 74 for 50,000 20,000 40,000 40,000
the A.Y. 2024-25
PY 23-
24
P.Y.
Loss to be carried forward (20,000)
- 23-24 - - - - -
to next assessment year PY 22-
(30,000) 23
(50,000)

3.2.12 Carry forward and set off of losses from activity of owning and maintaining race
horses [Sec. 74A]
In case of an assessee, being the owner of horses maintained by him for running in horse races (race horses), the
amount of loss incurred by the assessee in the activity of owning and maintaining race horses in any assessment
year shall be set off only against income from the activity of owning and maintaining race horses.
Period for which carry forward shall be allowed: 4 assessment years immediately succeeding the assessment
year in which such loss is first computed.
Notes
1. Continuity of activity: Activity of owning and maintaining race horses must be carried on by the assessee in
the previous year in which set off is claimed.
2. Treatment of other race animals: Sec. 74A states only about losses from activity of owning and maintaining
race horses, other race animals are governed by sec. 72.
3. Filing of return: To claim benefit of carry forward, return of loss u/s 139(3) must be filed within time as per
sec. 139(1).

The Institute of Cost Accountants of India 395


Direct Taxation

4. Horse Race: Horse race means a race upon which wagering or betting on horses may be lawfully made.
5. Amount of loss incurred by the assessee in the activity of owning and maintaining race horses means—

Particulars ₹
Amount of expenditure (not being in the nature of capital expenditure) laid out or expended by him
wholly and exclusively for the purposes of maintaining race horses ***
Less: Income by way of stake money ***
Loss incurred in the activity of owning and maintaining race horses ***
‘Income by way of stake money’ means the gross amount of prize money received on a race horse by the
owner thereof.

Test Yourself
1. From the following information, compute Mr. Rahaman’s total income and the amount of loss to be carried
forward for the assessment year 2024-25:

a. Income from house – 1 40,000
b. Loss from house – 2 30,000
c. Income from garment business 50,000
d. Loss from Jute business 25,000
e. Loss from speculative business 15,000
f. Long term capital gain 30,000
From the assessment year 2023-24, the balance of following losses were brought forward:
a. Loss from the garment business ₹ 10,000
b. Long-term capital loss ₹ 10,000
c. Loss from house property ₹ 10,000
Hints
1. ₹ 1,35,000

Quick MCQs:-

1. Mr. B incurred Short Term Capital Loss of D 10,000 on Sale of Shares through the National Stock
Exchange. Such Loss can be set –off-
(a) Only against Short Term Capital Gains
(b) Against both STCG and LTCG
(c) Against any head of Income
(d) None of the above.

2. Loss under the head Income from House Property can be carried forward-
(a) Only if the return is furnished before the due date mentioned u/s 13(1)
(b) Even if the return is not furnished

396 The Institute of Cost Accountants of India


Total Income and Tax Liability of Individuals & HUF

(c) even if the return is furnished after the due date


(d) None of the above

3. Loss on account of owing & maintaining the race horses can be carried forward
(a) for 8 years
(b) for 4 years
(c) indefinitely
(d) None of the above

4. The maximum period for which Speculation Loss can be carried forward is
(a) 4 Years
(b) 8 Years
(c) Nil
(d) Any number of years

5. Brought forward loss from house property of D 3,10,000 of A.Y 2023-24 is allowed to be set-off against
income from house property of A.Y. 2024-25 of D 5,00,000 to the extent of –
(a) D 2,00,000
(b) D 3,10,000
(c) D 2,50,000
(d) D 1,00,000

The Institute of Cost Accountants of India 397


Direct Taxation

Deductions, Rebate and Relief 3.3

DEDUCTIONS
From Gross Total Income (being aggregate of income under five heads), assessee can claim several deductions
as specified in chapter VIA on fulfillment of prescribed conditions as laid down in the respective section. After
allowing these deductions, total income of the assessee is arrived & tax is charged on it at the prescribed rates.

3.3.1 Basic Rules


1. Deductions not available from: Deductions under chapter VIA are not available from -
¾¾ long-term capital gain;
¾¾ short term capital gain covered u/s 111A (i.e., STCG on which STT is charged); and
¾¾ casual income like winning from lotteries, races, etc.
2. Limit of deduction: The aggregate amount of deduction under chapter VIA cannot exceed Gross Total Income
of the assessee excluding -
¾¾ long term capital gain;
¾¾ short term capital gain covered u/s 111A;
¾¾ casual income like winning from lotteries, card-games, horse races, etc.; and
¾¾ income referred in Sec.115A, 115AB, 115AC, 115ACA, etc.
3. Deduction must be claimed: Deduction under chapter VIA shall be available only if the assessee claims for it.
4. Double deduction not permissible: Where deduction under any section of chapter VIA has been claimed then
the same shall not qualify for deduction in any other section.
Following table shows the position of deduction in computation of total income and gross tax liability
Particulars Details Details Details Details Amount
Salaries ***
Income from house property ***
Profits & gains of business or profession ***
Capital gains ***
Income from other sources ***
Gross Total Income (GTI) ***
Casual Remaining
STCG covered u/s
Sub division of Gross Total Income LTCG income e.g. income of Total
111A
lottery etc. GTI

398 The Institute of Cost Accountants of India


Total Income and Tax Liability of Individuals & HUF

Particulars Details Details Details Details Amount


Less: Deductions u/s 80C to 80U - - - (***) (***)
Total Income (TI) *** *** *** *** ****
Apply tax rate 15% 20%/10% 30% slab rate
Gross tax liability **** **** **** **** ****

3.3.2 Deduction u/s 80C in respect of LIC premium, contributions to PF, etc.
Applicable to
An Individual or a Hindu Undivided Family (whether resident or non-resident)
Condition to be satisfied
Assessee has made a deposit or an investment in any one or more of the listed items (as given below) during the
previous year.
Particulars Notes
Category A
Applicable to Individual & HUF both
1. Life insurance premium paid by a 1. Insurance policy can be taken on life of the following:
person to effect or to keep in force a. In case of an individual: Himself, spouse and child
an insurance policy (life policy or (whether major or minor) of such individual;
endowment policy) [Sec. 80C(2)(i)]
b. In case of HUF: Any member of the HUF.
2. Maximum limit: Premium on insurance policy in excess of
following % of the actual sum assured shall be ignored.
Insured is disable2 or Insured is
Policy issued suffering from disease any other
specified u/s 80DDB person
Upto 31-03-2012 20% 20%
During P.Y. 2012-13 10% 10%
On or after 01-04-2013 15% 10%
3. Actual capital sum assured in relation to a life insurance policy
shall mean the minimum amount assured under the policy on
happening of the insured event at any time during the term of the
policy, not taking into account:
i. the value of any premium agreed to be returned; or
ii. any benefit by way of bonus or otherwise over and above
the sum actually assured, which is to be or may be received
under the policy by any person.
2. Contribution made towards Public 1. Subscription should be in the name of following persons:
provident fund (PPF). [Sec. 80C(2) a. In case of individual: Such individual, his spouse and
(v)] child (whether major or minor);
b. In case of HUF: Any member of HUF.
2. Contribution must not be in form of repayment of loan

2
Disable or severe disable as referred to in sec. 80U

The Institute of Cost Accountants of India 399


Direct Taxation

Particulars Notes
3. Any subscription to National Savings 3. If contribution is made in joint names, the person who has
Certificates, VIII Issue and IX Issue contributed the money is eligible to claim deduction.
[Sec. 80C(2)(ix)] 4. An individual can claim deduction in respect of certificates
purchased in the name of his spouse or minor child.
5. Treatment of accrued interest: Deduction is also available on
accrued interest which is reinvested. i.e., interest upto penultimate
year of lock-in-period is eligible for deduction.
4. Contribution for participating in the Contribution can be made in the names of following persons:
Unit-linked Insurance Plan (ULIP) In case of individual: Such individual, spouse and child (major or
of Unit Trust of India (UTI) or ULIP minor) of such individual;
of LIC Mutual fund u/s 10(23D) In the case of HUF: Any member of HUF.
formerly known as Dhanraksha
1989. [Sec.80C(2)(x)]
5. Sum paid to effect or keep in force a
contract for notified annuity plan of
the LIC or any other insurer. [Sec.
80C(2)(xii)]
6. Subscription to notified units of a Eligible scheme: Equity Linked Saving Scheme, 2005.
specified Mutual fund u/s 10(23D)/
administrator or the specified
company as referred in sec. 2 of UTI.
[Sec. 80C(2)(xiii)]
7. Any sum paid as subscription to National Housing Bank (Tax Saving) Term Deposit Scheme, 2008
Home Loan Account Scheme or
notified pension fund of the National
Housing Bank. [Sec 80C(2)(xv)]
8. Any sum paid as subscription to Such deposit scheme shall be of -
a notified deposit scheme. [Sec.  Public sector companies engaged in providing long-term finance
80C(2)(xvi)] for construction or purchase of houses in India for residential
purpose; or
 Any authority constituted in India for the purpose of satisfying
the need for housing accommodation or for the purpose of
planning, development or improvement of cities, towns, villages
or for both.
9. Any payment for purchase or # Specified person includes -
construction of a residential house i. the Central or State Government; or
property (the income from which
ii. any bank, including a co-operative bank; or
is chargeable to tax under the head
“Income from house property), by iii. the National Housing Bank; or
way of - iv. Life Insurance Corporation; or
v. any public company formed and registered in India with the main
object of carrying on the business of providing long-term finance
for construction or purchase of houses in India for residential
purposes; or

400 The Institute of Cost Accountants of India


Total Income and Tax Liability of Individuals & HUF

Particulars Notes
a. any instalment due under vi. any company in which the public are substantially interested or
any self-financing or other any co-operative society, where such company or co-operative
scheme of any development society is engaged in the business of financing the construction
authority, housing board or of house; or
other authority engaged in the
vii. the assessee’s employer where such employer is an authority
construction and sale of house
or a board or a corporation or any other body established or
property on ownership basis;
constituted under a Central or State Act;
or
viii. the assessee’s employer where such employer is a public
b. any instalment due to any
company or public sector company, or a university established
company or co-operative
by law or a college affiliated to such university or local authority
society of which the assessee
or co-operative society.
is a shareholder or member
towards the cost of the house Notes:
property allotted to him; or  Any expenditure in respect of which deduction is allowable u/s
c. repayment of the amount 24 is not eligible for deduction.
borrowed by the assessee from  Repayment of loan borrowed for acquiring commercial property
specified person# are not entitled for deduction.
d. stamp duty, registration fee and  Repayment of loan taken for repair, alteration, renovation,
other expenses for the purpose addition are not eligible for deduction
of transfer of such house
property to the assessee [Sec.
80C(2)(xviii)]
10. Any amount invested in - Eligible issue of capital means an issue made by a public company
a. Debentures of or equity shares formed and registered in India or a public financial institution and the
in an eligible issue of capital; or entire proceeds of the issue are utilized wholly and exclusively for the
purpose of any business referred to in sec. 80IA(4).
b. Eligible issue of capital of any
public financial institution. [Sec.
80C(2)(xix)]
11. Subscription to units of any mutual
fund u/s 10(23D) provided amount
of subscription to such units is
subscribed only in the eligible issue
of capital [Sec. 80C(2)(xx)]
12. Investment as term deposit for a
period of 5 years or more with a Such term deposit scheme shall be framed and notified by the Central
scheduled bank. [Sec. 80C(2)(xxi)] Government.

13. Notified Bonds issued by the


National Bank for Agriculture and
Rural Development (NABARD)
[Sec. 80C(2)(xxii)]

The Institute of Cost Accountants of India 401


Direct Taxation

Particulars Notes
14. Senior Citizens Savings Scheme
Rules, 2004 [Sec. 80C(2)(xxiii)]

15. 5 year time deposit in an account


under the Post Office Time Deposit
Rules, 1981 [Sec. 80C(2)(xxiv)]
Applicable to Individual only
1. Payment by an individual in respect 1. Annuity may be taken in the name of the individual, spouse and
of non-commutable deferred annuity. any child of such individual.
[Sec. 80C(2)(ii)] 2. Such contract does not contain a provision for the exercise by
the insured of an option to receive a cash payment in lieu of the
payment of the annuity
2. Any sum deducted from salary of Maximum limit: 20% of salary of the employee
a Government employee for the
purpose of securing to him a deferred
annuity or making provision for his
wife or children. [Sec. 80C(2)(iii)]
3. Contribution made towards statutory Contribution must not be in form of repayment of loan
provident fund and recognised
provident fund. [Sec. 80C(2)(iv)&
(vi)]
4. Contribution made towards an
approved superannuation fund. [Sec.
80C(2)(vii)]
5. Subscription to any notified Subscription should be in the name of following persons:
Government security or any notified In the case of an individual: Such Individual or any girl child of
deposit scheme (i.e., Sukanya that individual, or any girl child for whom such person is the legal
Samriddhi Account Scheme) [Sec. guardian, if the scheme so specifies
80C(2)(viii)]
6. Contribution to any notified pension
fund set up by a Mutual Fund u/s
10(23D) or by the administrator or
the specified company referred u/s 2
of the UTI [Sec. 80C(2)(xiv)]
7. Any payment by way of tuition fees to Admission fee: Tuition fees may be at the time of admission or
any university, college, school or other thereafter
educational institution situated within
Donation to school, etc: Such payment does not include any payment
India for the purpose of full-time
towards any development fees or donation or payment of similar
education. [Sec. 80C(2)(xvii)]
nature.
Restriction on number of child:
Private tuition fee is not covered.
Deduction shall be allowed in respect of
maximum 2 children.

402 The Institute of Cost Accountants of India


Total Income and Tax Liability of Individuals & HUF

Particulars Notes
8. Contribution to a specified account
(i.e., NPS Tier-II account) of the
notified pension scheme referred to
in sec. 80CCD for a fixed period of
not less than 3 years by an employee
of the Central Government
Notes
1. Deduction not available in certain cases: Deduction is not available from short-term capital gain covered
u/s 111A and long-term capital gain.
2. Cash basis: For the purpose of Deduction u/s 80C, amount paid, invested or deposited shall be considered
on payment basis. Payments, which have become due during the previous year but not paid till the end of the
previous year, shall not be eligible for deduction. Above rule holds good, even though the assessee follows
mercantile system of accounting.
Quantum of deduction
Deduction under this section shall be minimum of the following:
 Aggregate of the eligible contributions, expenditure or investments (discussed above)
 ₹ 1,50,000

Illustration 22:
Calculate the amount of deduction u/s 80C: ₹ in ‘000

Particulars A B C D
Gross total income 520 150 90 - 30
Amount invested/contributed /expended in specified items 175 70 100 50
Deduction u/s 80C 150 70 90 Nil

Illustration 23:
Calculate the amount of deduction u/s 80C from following data ₹ in ‘000

Particulars A B C D E F G
Payment of LIC premium 5 8 6 8 4 - 10
LIC policy amount 80 70 50 40 50 - 40
NSC purchased 20 15 18 17 35 85 20
Repayment of housing loan 20 25 10 - 12 - 80
Contribution to Unit linked insurance plan 10 5 - - 3 2 3
School fee paid for one child 10 15 6 18 3 5 -
Amount deposited in PPF 5 40 10 6 7 9 -
Notified units of Mutual fund 50 80 60 10 40 100 Nil
Gross Total Income 370 180 320 190 160 540 75
Total income consists of income under the head “Profit and gains of business or profession” only.

The Institute of Cost Accountants of India 403


Direct Taxation

Solution :
Calculation of deduction u/s 80C (₹ in ‘000)

Particulars A B C D E F G
Aggregate of investment u/s 80C(2)
Payment of LIC premium 5 7* 5* 4* 4 - 4*
*(to the maximum of 10% of policy amount)
NSC purchased 20 15 18 17 35 85 20
Repayment of housing loan 20 25 10 - 12 - 80
Contribution to ULIP 10 5 - - 3 2 3
School fee paid for one child 10 15 6 18 3 5 -
Notified units of Mutual fund 50 80 60 10 40 100 Nil
Deposit in PPF 5 40 10 6 7 9 -
Total 120 187 109 55 104 201 107
Gross Total Income 370 180 320 190 160 540 75
Deduction u/s 80C 120 150 109 55 104 150 75
Lock in period
Lock in period in following cases -
Life Insurance Policy: The Life insurance policy (point 1) cannot be surrendered unless premium for 2 years on
such policy has been paid.
Housing Loan: The house acquired (point 10) cannot be transferred before 5 years from the end of financial year
in which the possession of such property is obtained by assessee.
Unit Linked Insurance Plan: The participation in plan (point 5) cannot be ceased before contribution in respect
of such participation has been paid for 5 years.
Shares or debentures of infrastructure company or power company or mutual fund: Equity shares, debentures,
etc. (point 11) cannot be sold or otherwise transferred within 3 years from the date when name of the assessee for
those shares or debentures has been entered in the register of member or debenture holder by the company or the
Mutual fund.
Investment in Senior Citizen Saving Scheme / Post office Time Deposit: Such amount, including interest accrued
thereon, shall not be withdrawn by the assessee from his account, before the expiry of 5 years from the date of its
deposit. However, any amount received by the nominee or legal heir of the assessee, on the death of such assessee,
shall not be included.
Consequence in case of violation of lock in period
If the above lock in period is violated, then entire amount of deduction allowed earlier in any previous year, shall
be treated as taxable income in the year in which default is made.

3.3.3 Deduction u/s 80CCC in respect of contribution to Pension Fund


Applicable to
An individual (irrespective of residential status or citizenship of the individual)
Condition to be satisfied

404 The Institute of Cost Accountants of India


Total Income and Tax Liability of Individuals & HUF

1. Amount paid under an annuity plan: During the previous year, assessee has paid or deposited a sum under
an annuity plan of the Life Insurance Corporation of India (LIC) or any other insurer for receiving pension
from the fund referred to in Sec. 10(23AAB).
2. Payment out of taxable income: The amount must be paid out of income which is chargeable to tax. However,
it is not necessary that such income relates to current year.
Quantum of deduction
Minimum of the following -
a. Amount deposited; or
b. ₹ 1,50,000
Other Points
a. Treatment of Interest or Bonus accrued: Interest or bonus accrued or credited as per the scheme to the
assessee’s account shall not be eligible for deduction.
b. Withdrawal from such fund [Sec. 80CCC(2)]: Any amount received by the assessee or his nominee
as pension; or on surrender of such annuity is taxable in the hands of recipient in the year of receipt.
Note: Interest or bonus received from such fund shall also be taxable.
c. Deduction u/s 80C [Sec.80CCC(3)]: Deduction u/s 80C will not be available for the amount paid or
deposited and for which deduction has been claimed u/s 80CCC.

3.3.4 Deduction u/s 80CCD in respect of contribution to pension scheme Amended


Applicable to
An individual
Condition to be satisfied
During the previous years, the assessee has paid or deposited any amount in his account under a pension scheme
notified by the Central Government (New Pension System and Atal Pension Yojna).
Quantum of Deduction
Deduction u/s 80CCD(1)
A. In case of salaried individual
Lower of the following ₹
 Amount so paid or deposited ***
 10% of his salary1 in the previous year ***
***
Add: Whole of the contribution made by the employer to such account to the maximum of 10% (14% ***
where such contribution is made by the Govt.3) of his salary1 in the previous year.
Amount of Deduction ***
B. In case of other individual
Lower of the following
 Amount so paid or deposited
 20% of his gross total income in the previous year
Additional Deduction u/s 80CCD(1B)

3
Central or State

The Institute of Cost Accountants of India 405


Direct Taxation

Lower of the following shall also be eligible for deduction ₹


 Contribution to the scheme by any individual [Other than amount claimed and allowed as ***
deduction u/s 80CCD(1)]
 ₹ 50,000 ***
Other Points
1. “Salary” here means Basic + Dearness allowance, if the terms of employment so provide.
2. Where any amount standing to the credit of the assessee in his account, (in respect of which a deduction has
been allowed), together with the amount accrued thereon is received by the assessee or his nominee, whether –
a. on account of closure or his opting out of the pension scheme; or
b. as pension received from the annuity plan purchased or taken on such closure or opting out,
- shall be deemed to be the income of the assessee or his nominee, as the case may be, in the previous year
in which such amount is received, and shall accordingly be charged to tax.
Exemption
i. As per sec. 10(12A), in case of withdrawal from the NPS by an assessee on closure of account or on his
opting out of the pension scheme, 60% of the total amount payable to him at the time of such closure or
his opting out of the scheme shall be exempt. i.e., 40% shall be taxable.
ii. As per sec. 10(12B), partial withdrawal upto 25% of the amount contributed by the employee under the
pension scheme, shall be exempted fully.
iii. Where amount withdrawn is used for purchasing an annuity plan in the same previous year (i.e. year of
withdrawal), such receipt shall not be taxable.
iv. Amount received on account of closure or opting out of the pension scheme by the nominee, on the death
of the assessee, shall be exempted in hands of the nominee.
3. Where any amount paid or deposited by the assessee has been allowed as a deduction, no deduction with
reference to such amount shall be allowed u/s 80C.

3.3.5 Deduction u/s 80CCE: Limit on deductions u/s 80C, 80CCC and 80CCD
The aggregate amount of deductions under section 80C, section 80CCC and section 80CCD [other than deduction
in respect of employer’s contribution and additional deduction u/s 80CCD(1B)] shall not exceed ₹ 1,50,000.
Taxpoint: An assessee can claim deduction u/s 80C, 80CCC & 80CCD:
Particulars Amount
Deduction u/s 80C ****
Deduction u/s 80C ****
Deduction u/s 80CCD [Other than deduction in respect of Employer’s Contribution and additional ****
deduction u/s 80CCD(1B)]
Total [Restricted to maximum of ₹ 1,50,000 u/s 80CCE] *****
Add: Contribution to the pension scheme by any individual allowable u/s 80CCD(1B) [Sub. to a ****
maximum of ₹ 50,000/-]
Add: Employer’s contribution to New Pension System referred to in Sec. 80CCD [Subject to max. ****
of 10% or 14% of salary]
Deduction available u/s 80C, 80CCC & 80CCD *****

406 The Institute of Cost Accountants of India


Total Income and Tax Liability of Individuals & HUF

Illustration 24:
X completed his studies on 1-04-2023 and was immediately employed by X Ltd. on the following terms:
Basic Salary ₹ 20,000 p.m.
DA ₹ 5,000 p.m. (forming a part of retirement benefit)
Bonus ₹ 50,000
During the year, his employer contributed ₹ 33,000 to the pension scheme being notified u/s 80CCD of the Income
Tax Act, 1961. X also contributed similar amount. His income from house property is ₹ 50,000. During the year he
contributed ₹ 15,000 to pension plan of LIC, to PPF ₹ 1,00,000 and paid LIC premium of ₹ 16,000 (Policy value ₹
1,20,000). Compute his total income.
Solution :
Computation of total income of X for the A.Y.2024-25
Particulars Amount Amount Amount
Salaries
Basic (₹ 20,000 * 12) 2,40,000
Bonus 50,000
Dearness allowances (₹ 5,000 * 12) 60,000
Contribution to pension scheme 33,000
Gross Salary 3,83,000
Less: Standard Deduction u/s 16(ia) 50,000 3,33,000
Income from house property 50,000
Gross Total Income 3,83,000
Less: Deduction u/s
80C - LIC premium to maximum of 10% of Policy value (i.e. 10% 12,000
of ₹ 1,20,000)
- Contribution in PPF 1,00,000 1,12,000
80CCC (Pension plan of LIC) 15,000
80CCD (Notified pension scheme)
- Own contribution being lower of:
a) Amount contributed i.e. ₹ 33,000
b) 10% of (Basic and DA forming part of retirement 30,000
benefit), i.e., 10% of (₹ 2,40,000 + ₹ 60,000)
1,57,000
As per sec.80CCE, total deduction u/s 80C + 80CCC + 80CCD 1,50,000
(other than employer’s contribution) cannot exceed ₹ 1,50,000
80CCD(1B): Contribution to NPS (Amount not claimed above 3,000
subject to max. of ₹ 50,000/-)
80CCD(2): Employer contribution
a) Amount contributed i.e. ₹ 33,000
b) 10% of (Basic and DA forming part of retirement benefit), 30,000 1,83,000
i.e., 10% of (₹ 2,40,000 + ₹ 60,000)
Total Income 2,00,000
Deduction u/s 80CCH in respect of contribution to ACF contribution to Agniveer Corpus Fund is fully allowed as
deduction.

The Institute of Cost Accountants of India 407


Direct Taxation

3.3.6 Deduction u/s 80D in respect of Medical Insurance Premium


Applicable to
An individual or an HUF (irrespective of residential status or citizenship)
Conditions to be satisfied
1. Payment for health insurance or medical check-up: The assessee has made payment for health insurance
of the following person:
Expenditure on
Category Assessee Nature of Payment Quantum of Deduction
behalf of
1 Individual a. Payment of Himself/herself, Lower of the following:
Mediclaim insurance spouse or dependent a. Aggregate of
premium#; or children –– Premium paid; or
–– Contribution made; or
b. Contribution to the
–– Preventive health check up (upto
Central Government
₹ 5,000)
Health Scheme or
b. ₹ 25,000 p.a.
any other notified
Senior Citizen
Health Scheme4
Where the person, for whom such
c. Preventive health premium (not for payment made
check up expenditure for preventive health check up)
is paid, is a senior citizen, then
maximum limit of deduction
shall be increased to ₹ 50,000
instead of ₹ 25,000.
2 Individual a. Payment of Parents (whether Lower of the following:
Mediclaim insurance dependent or not) a. Aggregate of
premium#
–– Premium paid; or
b. Preventive health –– Preventive health check up
check up expenditure (upto ₹ 5,000)
–– ₹ 25,000 p.a.
b. Senior Citizen
Where the person, for whom such
premium (not for payment made
for preventive health check up)
is paid, is a senior citizen, then
maximum limit of deduction
shall be increased to ₹ 50,000
instead of ₹ 25,000.
Note:
The deduction for payment made for preventive health check up (for self, spouse, dependent children
and parents) for category 1 & 2 does not exceed in the aggregate ₹ 5,000 subject to overall limit of
₹ 25,000/- or ₹ 50,000/-

4
Contributory Health Service Scheme of the Department of Atomic Energy

408 The Institute of Cost Accountants of India


Total Income and Tax Liability of Individuals & HUF

Expenditure on
Category Assessee Nature of Payment Quantum of Deduction
behalf of
3 HUF Payment of Mediclaim Any member of the Lower of the following:
insurance premium# family.
a. Premium Paid; or
b. ₹ 25,000
Senior Citizen
Where the person, for whom such
premium is paid, is a senior citizen,
then maximum limit of deduction
shall be increased to ₹ 50,000 instead
of ₹ 25,000.
4 Individual Amount paid on account Expenditure Lower of the following:
/ HUF of medical expenditure incurred for any of  Medical Expenditure incurred; or
provided mediclaim the following person
insurance is not paid on who is a senior  ₹ 50,000
the health of such person citizen:
In case of Individual
a. Himself/herself,
spouse; or
b. dependent
children; or
c. Either or both
of the parents
In case of HUF:
a. Any member of
the family
Note:
The deduction for category 4 is available only from the overall limit of category 1 or category 2
or category 3 respectively. In other words, maximum deduction under category 1, 2, 3 and 4 are as
under:
Category Maximum Deduction
Category 1 & 4 ₹ 50,000/-
Category 2 & 4 ₹ 50,000/-
Category 3 & 4 ₹ 50,000/-

Dependent children: Children are said to be dependant if their own resources are not sufficient enough to support
them.
#
Such mediclaim insurance policy should be in accordance with the scheme framed in this behalf by -
¾¾ General Insurance Corporation of India (GIC) & approved by the Central Government (i.e. Mediclaim
insurance policy); or
¾¾ Any other insurer who is approved by the Insurance Regulatory and Development Authority.

The Institute of Cost Accountants of India 409


Direct Taxation

2. Mode of payment: The premium or medical expenditure must be paid by any mode other than cash. However,
payment shall be made by any mode, including cash, in respect of any sum paid on account of preventive
health check-up.
3. Payment out of taxable income: The amount must be paid out of income, which is chargeable to tax. However,
it is not necessary that such income relates to current year.
Taxpoint:
 Where lumpsum health insurance premium is paid (single premium) covering insurance for more than a year,
then, deduction is available on proportionate basis.
 For claiming higher deduction of ₹ 50,000, payer need not be a senior citizen but person insured must be a
senior citizen.
 Senior citizen means an individual resident in India who is of the age of 60 years or more at any time during
the relevant previous year

Illustration 25:
X, 40 years old, paid following sums by cheque -

Person Insured Amount


Mediclaim Insurance premium for X 18,000
Mediclaim Insurance premium for Mrs. X (not dependant on X) 5,000
Contribution to the Central Government Health Scheme for Dependant Daughter 3,000
Mediclaim Insurance premium for Son (not dependant on X) 3,000
Contribution to the Central Government Health Scheme for Brother (dependant on X) 1,000
Mediclaim Insurance premium for Mother-in law (dependant on X age 70 years) 900
Mediclaim Insurance premium for Mother (dependant on X age 59 years) 26,500
Mediclaim Insurance premium for Grand parents (dependant on X) 1,600
Mediclaim Insurance premium for Father (not dependant on X age 61 years) 32,000
Contribution to the Central Government Health Scheme for Father 2,000
Total 68,000
Compute deduction available to Mr. X u/s 80D.
Solution :
Computation of deduction u/s 80D available to Mr. X

Person Insured Notes Amount Amount


X Himself 18,000
Mrs. X Spouse 5,000
Daughter Dependent 3,000
Son Not dependent Nil
Brother Not eligible Nil
Mother in law Not eligible Nil
Grand parents Not eligible Nil
Qualifying amount 26,000

410 The Institute of Cost Accountants of India


Total Income and Tax Liability of Individuals & HUF

Person Insured Notes Amount Amount


Restricted to the maximum limit 25,000
Add: Additional deduction for parents
Mother 26,500
Restricted to the maximum limit 25,000
Add: Additional deduction for insurance of Father (a Senior Citizen) 32,000
Add: Contribution to the Central Government Health Scheme for Father (Not Nil
eligible)
Qualifying amount 57,000
Restricted to the maximum limit 50,000 50,000
Deduction u/s 80D 75,000

Illustration 26:
Mr. Ram (38 years) has incurred following expenses:

Particulars ₹
Mediclaim Insurance premium paid for himself 9,000
Mediclaim Insurance premium paid for spouse 8,000
Mediclaim Insurance premium paid for dependent children 6,000
Mediclaim Insurance premium paid for father (62 years) 18,000
Preventive health-check up expenditure for father 6,000
Preventive health-check up expenditure for himself (paid in cash) 4,000
Compute deduction available to Mr. Ram u/s 80D.
Solution :
Computation of deduction u/s 80D available to Mr. Ram

Particulars Amount Amount


Mediclaim Insurance premium paid for himself 9,000
Mediclaim Insurance premium paid for spouse 8,000
Mediclaim Insurance premium paid for dependent children 6,000
Qualifying amount (A) 23,000
Add: Additional deduction for parents
Mediclaim Insurance premium paid for father (B) 18,000
Add: Expenditure incurred for preventive health check up Incurred Max. Limit
Preventive health-check up expenditure for father ! Max. limit
6,000 5,000!
Preventive health-check up expenditure for himself # [₹25,000 – (A)] 4,000 2,000#
7,000
Restricted to overall maximum limit for preventive health check ups (C) 5,000
Deduction u/s 80D (A + B + C) 46,000

The Institute of Cost Accountants of India 411


Direct Taxation

Illustration 27:
Mr. Shyam (40 years) has incurred following expenses:

Particulars ₹
Mediclaim Insurance premium paid for himself 12,000
Mediclaim Insurance premium paid for spouse 11,000
Mediclaim Insurance premium paid for dependent children 6,000
Mediclaim Insurance premium paid for mother (76 years) 18,000
Preventive health-check up expenditure for mother 8,000
Medical expenditure incurred for father (78 years) 39,000
Compute deduction available to Mr. Shyam u/s 80D.
Solution :
Computation of deduction u/s 80D available to Mr. Shyam

Particulars Amount Amount


Mediclaim Insurance premium paid for himself 12,000
Mediclaim Insurance premium paid for spouse 11,000
Mediclaim Insurance premium paid for dependent children 6,000
Deductible amount (A) [Maximum] 25,000
Add: Additional deduction for parents
Mediclaim Insurance premium paid for mother (B) 18,000
Add: Expenditure incurred for preventive health check up Incurred Max.
Preventive health-check up expenditure for mother ! Max. limit 8,000 5,000!
Restricted to overall maximum limit for preventive health check 5,000
ups (C)
Add: Medical expenditure incurred for father being senior citizen 39,000 27,000@ 27,000
(D) @[₹50,000 – (B) – (C)]
Deduction u/s 80D (A + B + C + D) 75,000

Illustration 28:
Mr. Rahim (55 years) has incurred following expenses:

Particulars ₹
Mediclaim Insurance premium paid for himself 10,000
Mediclaim Insurance premium paid for spouse 10,000
Mediclaim Insurance premium paid for dependent children 5,000
Mediclaim Insurance premium paid for mother (76 years) 9,000
Mediclaim Insurance premium paid for father (82 years) 39,000
Preventive health-check up expenditure for father 6,000

412 The Institute of Cost Accountants of India


Total Income and Tax Liability of Individuals & HUF

Particulars ₹
Medical expenditure incurred for father 14,000
Compute deduction u/s 80D.
Solution :
Computation of deduction u/s 80D available to Mr. Rahim

Particulars Amount Amount


Mediclaim Insurance premium paid for himself 10,000
Mediclaim Insurance premium paid for spouse 10,000
Mediclaim Insurance premium paid for dependent children 5,000
Deductible amount (A) 25,000
Add: Additional deduction for parents
Mediclaim Insurance premium paid for mother 9,000
Mediclaim Insurance premium paid for father 39,000
Deductible amount (B) 48,000
Add: Expenditure incurred for preventive health check up Incurred Max. Limit
Preventive health-check up expenditure for father ![₹50,000 – (B)] (C) 6,000 2,000! 2,000
Add: Medical expenditure incurred for father being super-senior citizen (D) Nil
(As mediclaim insurance premium is paid on the health of father)
Deduction u/s 80D (A + B + C + D) 75,000

3.3.7 Deduction u/s 80DD in respect of maintenance of dependant disable relative Amended
Applicable to
A resident individual (irrespective of citizenship) or a resident HUF
Conditions to be satisfied
1. Assessee has a dependant disable relative:
a. Dependant Relative

In the case of Relative includes


Individual Spouse, children, parents, brothers and sisters of the individual
HUF Any member of the Hindu Undivided Family
Dependant Relative: A relative is said to be dependant if he wholly or mainly depends on such
individual or HUF for his support and maintenance.
b. Disability shall have the meaning assigned to it in Sec. 2(i) of the Persons with Disabilities (Equal
Opportunities, Protection of Rights and Full Participation) Act, 1995 and includes “autism”, “cerebral
palsy” and “multiple disability” referred to in sec. 2(a), 2(c) and 2(h) of the National Trust for Welfare of
Persons with Autism, Cerebral Palsy, Mental Retardation and Multiple Disabilities Act, 1999.
Person with disability means a person as referred to in Sec. 2(f) of the Persons with Disabilities (Equal
Opportunities, Protection of Rights and Full Participation) Act, 1995 or sec. 2(j) of the National Trust for

The Institute of Cost Accountants of India 413


Direct Taxation

Welfare of Persons with Autism, Cerebral Palsy, Mental Retardation and Multiple Disabilities Act, 1999
(i.e., a person suffering with 40% or more of one or more ‘disabilities’ as certified by a medical authority).
Disability includes blindness, low vision, leprosy-cured, hearing impairment, locomotor disability, mental
retardation, mental illness.
3. No benefit u/s 80U to disable relative: The disable individual has not claimed benefit u/s 80U.
4. Expenditure on disable relative: Assessee has -
¾¾ incurred any expenditure for the medical treatment (including nursing), training and rehabilitation of a
dependant, being a person with disability; or
¾¾ paid or deposited any amount in an approved scheme for the maintenance of a disable dependant being
framed by the Life Insurance Corporation or any other insurer or the Administrator# or Unit Trust of India.
Taxpoint
The scheme shall provide for payment of annuity or lump sum amount for the benefit of a dependant,
being a person with disability,—
i.in the event of the death of the individual or the member of the HUF in whose name subscription to
the scheme has been made; or
ii. on attaining the age of 60 years or more by such individual or the member of the HUF, and the
payment or deposit to such scheme has been discontinued.
Note: Though assessee needs to fulfill the above condition, the amount of deduction shall not be affected by
the actual expenditure incurred on the above two purposes.
4. Medical certificate: Assessee shall furnish a copy of the certificate issued by the medical authority$ along
with the return of income in respect of the assessment year for which the deduction is claimed.
$
Medical authority means medical authority as per Sec. 2(p) of the Persons with Disabilities (Equal
Opportunities, Protection of Rights and Full Participation) Act, 1995 or such other medical authority as may,
by notification, be specified by the Central Government for certifying “autism”, “cerebral palsy”, “multiple
disabilities”, “person with disability” and “severe disability” referred to in sec. 2(a), 2(c), 2(h), 2(j) and 2(o)
of the National Trust for Welfare of Persons with Autism, Cerebral Palsy, Mental Retardation and Multiple
Disabilities Act, 1999 (i.e., specified hospital or institution.)
Quantum of deduction

Relative is suffering from severe disability ₹ 1,25,000


Relative is suffering from disability but not severe disability ₹ 75,000
Taxpoint: Deduction shall be irrespective of actual expenditure incurred i.e. deduction is statutory in nature.
Person with severe disability means
¾¾ a person with 80% or more of one or more disabilities, as referred to in sec. 56(4) of the Persons with
Disabilities (Equal Opportunities, Protection of Rights and Full Participation) Act, 1995; or
¾¾ a person with severe disability referred to in sec. 2(o) of the National Trust for Welfare of Persons with
Autism, Cerebral Palsy, Mental Retardation and Multiple Disabilities Act, 1999.
Other points
a. Revision of medical certificate
Condition: Where the extent of disability requires reassessment after a period stipulated in the medical
certificate.

414 The Institute of Cost Accountants of India


Total Income and Tax Liability of Individuals & HUF

Treatment: After the expiry of previous certificate, deduction under this section shall be allowed only if a
new certificate is obtained from the medical authority and a copy thereof is furnished along with the return of
income.
b. Treatment when handicapped person predeceases [Sec. 80DD(3)]
Condition: Where the assessee has deposited any amount in annuity plan of LIC or UTI, etc. for the benefit of
disabled person and such person predeceases.
Treatment: Any amount received from such annuity plan shall be deemed to be the income of the assessee of
the previous year in which such amount is received by the assessee.
However, nothing shall be taxable where amount is received by the dependant, being a person with disability,
before his death, by way of annuity or lump sum on attaining the age of 60 years or more by such individual
or the member of the HUF, and the payment or deposit to such scheme has been discontinued.

Illustration 29:
Compute total income of Sri Bhandari from following information:
Taxable salary (Net) ₹ 75,000
Income from other sources ₹ 20,000
Agricultural income ₹ 4,000
He deposited in LIC annuity plan ₹ 18,000
He paid medical insurance premium by cheque for his dependant blind mother (certified as severe disable person),
aged 68 years, ₹ 20,000.
Solution :
Computation of Total income of Sri Bhandari for the A.Y. 2024-25

Particulars Details Amount


Salaries 75,000
Income from Other Sources 20,000
Gross Total Income 95,000
Less: Deduction u/s
80CCC (Paid in LIC annuity plan) 18,000
80D (Medical insurance premium for mother being senior citizen) 20,000
80DD (Dependant severe disable relative) 1,25,000
(Subject to maximum of gross total income) 95,000
Total Income Nil

3.3.8 Deduction u/s 80DDB in respect of medical treatment


Applicable to

A resident individual (irrespective of citizenship) or a resident HUF

The Institute of Cost Accountants of India 415


Direct Taxation

Conditions to be satisfied

1. Expenditure incurred on the medical treatment of relative: The assessee has, during the previous year,
actually paid any amount for the medical treatment of a specified disease or ailment as prescribed in rule
11DD. Expenditure is incurred for treatment of the assessee himself or for a dependant relative#.
#
Dependant Relative

In the case of Relative


Individual Spouse, children, parents, brothers and sisters of the individual.
HUF Any member of the HUF
Dependant Relative: A relative is said to be dependant if he wholly or mainly depends on such individual
or HUF for his support and maintenance.
2. Medical Prescription: Assesse should obtained the prescription for such medical treatment from a neurologist,
an oncologist, a urologist, a haematologist, an immunologist or such other specialist, as may be prescribed.
Quantum of deduction
Minimum of the following -
a. Actual expenditure incurred by the assessee; or
b. ₹ 40,000
Deduction for senior citizen: If such expenditure is incurred for a senior citizen#, then the maximum amount of
deduction shall be enhanced to ₹ 1,00,000.
Taxpoint: For claiming higher deduction of ₹ 1,00,000/-, payer need not be a senior citizen but person for whom
such expenditure has been incurred must be a senior citizen.
Treatment of Mediclaim or amount reimbursed by the employer: Deduction under this section shall be reduced
by the amount received, if any -
 under an insurance from an insurer; or
 reimbursed by an employer,
- for the medical treatment of the person.
Taxpoint:

Quantum of Deduction
Patient Deduction shall be minimum of the following
Senior citizen ¾¾ [(Actual expenditure incurred by assessee) – (Amount reimbursed by employer or
medi-claim received)]
¾¾ [₹ 1,00,000 – (Amount reimbursed by the employer or mediclaim received)]
Other ¾¾ [₹ 40,000 – (Amount reimbursed by the employer or mediclaim received)]
¾¾ [(Actual expenditure incurred by assessee) – (Amount reimbursed by employer or
mediclaim received)]
Note: In any case, the total deduction cannot exceed ₹ 1,00,000

416 The Institute of Cost Accountants of India


Total Income and Tax Liability of Individuals & HUF

Specified diseases as per rule 11DD are -


1) Neurological disease: (a) Dementia (b) Dystonia Musculorum Deformans (c) Motor Neuron Disease (d)
Ataxia (e) Chorea (f) Hemiballismus (g) Aphasia (h) Parkinson’s Disease (all neurological disease must have
disability of 40%and above);
2) Cancer; 3) Full Blown Acquired Immuno Deficiency Syndrome (AIDS); 4) Chronic Renal Failure; 5)
Hemophilia; and 6) Thalassaemia.

Illustration 30:
Find the amount of deduction u/s 80DDB for the following cases:

Name of the Assessee P Q R S T


Residential status of the assessee Ordinarily Not Non Resident Resident but Resident
resident ordinarily not Indian
resident Citizen
Expenditure incurred for medical ₹ 6,000 ₹ 80,000 ₹ 1,00,000 ₹ 72,000 ₹ 80,000
treatment (specified disease) of
dependant brother
Age of Brother 28 63 68 88 52
Residential status of dependant Resident Non resident Resident Resident Resident
Medical Insurance claim received. - - - - ₹ 8,000
Solution :
Amount of deduction available u/s 80DDB shall be as under:
Particulars P Q R S T
Deduction u/s 80DDB 6,000 40,000 1 -2 72,0003 32,000 4
Notes
1. As the brother is non-resident, therefore, senior citizen benefit is not available.
2. As the assessee himself is a non-resident, hence no deduction u/s 80DDB is available.
3. As the brother is resident having age of 88 years, hence he is a senior citizen. However, actual expenditure
incurred by the assessee is less than ₹ 1,00,000, therefore deduction shall be restricted to actual expenditure
incurred.
4. As the medical insurance claim is received, hence the amount of deduction (i.e. ₹ 40,000) shall be reduced by
the amount of claim received.

3.3.9 Deduction u/s 80E in respect of repayment of loan for higher education
Applicable to
An Individual (irrespective of residential status and citizenship of the individual).
Conditions to be satisfied
1. Loan from specified institution: The assessee had taken a loan from -
¾¾ a financial institution; or

The Institute of Cost Accountants of India 417


Direct Taxation

Financial Institution means a banking company to which the Banking Regulation Act, 1949 applies
(including any banking institution referred to in sec. 51 of that Act) or any other specified financial
institution.
¾¾ an approved charitable institution
Approved Charitable Institution means an institution established for charitable purposes and approved by
the prescribed authority u/s 10(23C) or an institution referred to in Sec. 80G(2)(a)
2. Purpose of loan: The loan must have been taken for the purpose of pursuing higher education of himself/
herself or for any other following persons:

a. Spouse b. Children (dependent or not); or c. the student for whom the individual is the legal guardian
“Higher education” means any course of study pursued after passing the Senior Secondary Examination or its
equivalent from any school, board or university recognised by the Central Government or State Government or
local authority or by any other authority authorised by the Central Government or State Government or local
authority to do so
3. Payment of interest: The assessee pays interest on such loan.
4. Payment out of taxable income: The amount must be paid out of income chargeable to tax. However, it is not
necessary that such income relates to the current year.
Quantum of deduction
Amount paid during the year by way of payment of interest.
Maximum permissible period for which deduction is available [Sec.80E(2)]
Deduction under this section shall be allowed for the initial assessment year and 7 assessment years immediately
succeeding the initial assessment year$ or until interest is paid by the assessee in full, whichever is earlier.
$
Initial Assessment Year means the assessment year relevant to the previous year, in which the assessee starts
repaying the loan or interest thereon.
Taxpoint
¾¾ The deduction is available for a maximum period of 8 consecutive years.
¾¾ The period starts from the year in which the assessee starts paying the interest on such loan.

3.3.10 Deduction u/s 80EE in respect of interest on loan taken for residential house property
Applicable to
Individual (resident or non-resident)
Conditions to be satisfied
1. Loan: The assessee has taken loan for acquisition of the residential house property
2. Sanction of Loan: The loan has been sanctioned by the financial institution5 during the Previous Year 2016-17.
3. Amount of Loan: The amount of loan sanctioned for acquisition of the residential house property does not
exceed ₹ 35 lakhs.
5
"Financial institution" means a banking company to which the Banking Regulation Act, 1949 applies including any bank or banking institution
referred to in sec. 51 of that Act or a housing finance company.
"Housing finance company" means a public company formed or registered in India with the main object of carrying on the business of providing long-
term finance for construction or purchase of houses in India for residential purposes.

418 The Institute of Cost Accountants of India


Total Income and Tax Liability of Individuals & HUF

4. Value of Residential Property: The value of the residential house property does not exceed ₹ 50 lakhs.
5. No other residential property: The assessee does not own any residential house property on the date of sanction
of the loan.
Quantum of Deduction
Minimum of the following:
a. Interest on loan payable for the previous year
b. ₹ 50,000
Other Points
Double deduction is not available: Where a deduction under this section is allowed for any interest, deduction shall
not be allowed in respect of such interest under any other provisions of the Act for the same or any other assessment
year.
Taxpoint: It is irrelevant whether such interest pertains to pre-construction period or post construction period.
Provision Illustrated
Compute deduction u/s 80EE in the following cases:
(₹ in lakh)
Case 1 Case 2 Case 3 Case 4 Case 5 Case 6 Case 7
Assessee P Q R S T U X (HUF)
Date of Sanction of Loan 01-10-16 01-10-16 01-10-16 01-10-16 01-10-16 01-10-15 01-10-16
Amount of loan ₹ 30 ₹ 20 ₹ 40 ₹ 20 ₹ 10 ₹ 20 ₹ 20
Value of the property ₹ 40 ₹ 55 ₹ 50 ₹ 35 ₹ 35 ₹ 35 ₹ 35
Other residential property No No No Yes No No No
owned by the Assessee on the
date of sanction
Interest for the P.Y. ₹ 1.20 ₹ 1.20 ₹ 1.80 ₹ 1.20 ₹ 0.40 ₹ 1.00 ₹ 1.20
Deduction u/s 80EE ₹ 0.50 Nil Nil Nil ₹ 0.40 Nil Nil
Notes 1 2 3 4 5 6 7
1. Assessee can claim ₹ 70,000 being excess interest, as deduction u/s 24(b)
2. As value of the property exceeds ₹ 50 lac hence, deduction u/s 80EE is not available. However, assessee can
claim deduction u/s 24(b)
3. As amount of loan exceeds ₹ 35 lac hence, deduction u/s 80EE is not available. However, assessee can claim
deduction u/s 24(b).
4. As assessee owns other residential house property on the date of sanction of loan, hence, deduction u/s 80EE
is not available. However, assessee can claim deduction u/s 24(b)
5. Assessee can claim ₹ 40,000 as deduction u/s 80EE
6. Loan was not sanctioned during the previous year 2016-17
7. The deduction u/s 80EE is not available to HUF, however, assessee can claim deduction u/s 24(b).

The Institute of Cost Accountants of India 419


Direct Taxation

3.3.11 Deduction u/s 80EEA in respect of interest on loan taken for certain house property
Applicable to
Individual (resident or non-resident) not eligible for deduction u/s 80EE
Conditions to be satisfied
1. Loan: The assessee has taken loan for acquisition of the residential house property from any financial institution.
2. Sanction of Loan: The loan has been sanctioned by the financial institution (as defined u/s 80EE) during 01-
04-2019 to 31-03-2022.
3. Value of Residential Property: The stamp duty value of the residential house property does not exceed ₹ 45
lakhs.
4. No other residential property: The assessee does not own any residential house property on the date of sanction
of the loan.
5. No deduction u/s 80EE: The assessee is not eligible for deduction u/s 80EE.
Quantum of Deduction
Minimum of the following:
a. Interest on loan payable for the previous year
b. ₹ 1,50,000
Other Points
Double deduction is not available: Where a deduction under this section is allowed for any interest, deduction shall
not be allowed in respect of such interest under any other provisions of the Act for the same or any other assessment
year.
Provision Illustrated
Compute deduction u/s 80EEA in the following cases:
(₹ in lakh)
Case 1 Case 2 Case 3 Case 4 Case 5 Case 6
Assessee A B C D E F (HUF)
Date of Sanction of Loan 01-10-21 01-10-21 01-10-21 01-10-21 01-10-18 01-10-20
Amount of loan ₹ 30 ₹ 30 ₹ 40 ₹ 30 ₹ 20 ₹ 20
Stamp Duty Value of the property ₹ 40 ₹ 55 ₹ 45 ₹ 35 ₹ 35 ₹ 35
Other residential property owned by the
No No No Yes No No
Assessee on the date of sanction
Interest for the P.Y.2021-22 ₹ 2.20 ₹ 1.20 ₹ 1.20 ₹ 1.20 ₹ 1.00 ₹ 1.20
Deduction u/s 80EE ₹ 1.50 Nil ₹ 1.20 Nil Nil Nil
Notes 1 2 3 4 5 6
1. Assessee can claim ₹ 70,000 being excess interest, as deduction u/s 24(b)
2. As value of the property exceeds ₹ 45 lac hence, deduction u/s 80EEA is not available. However, assessee can
claim deduction u/s 24(b)
3. Deduction cannot exceed interest on loan.
4. As assessee owns other residential house property on the date of sanction of loan, hence, deduction u/s 80EEA

420 The Institute of Cost Accountants of India


Total Income and Tax Liability of Individuals & HUF

is not available. However, assessee can claim deduction u/s 24(b)


5. Loan was not sanctioned during the specified time
6. The deduction u/s 80EEA is not available to HUF, however, assessee can claim deduction u/s 24(b).

3.3.12 Deduction u/s 80EEB in respect of purchase of electric vehicle


Applicable to
Individual (resident or non-resident)
Conditions to be satisfied
1. Loan: The assessee has taken loan for purchase of an electric vehicle from any financial institution.
¾¾ Electric vehicle means a vehicle which is powered exclusively by an electric motor whose traction energy
is supplied exclusively by traction battery installed in the vehicle and has such electric regenerative
braking system, which during braking provides for the conversion of vehicle kinetic energy into electrical
energy.
¾¾ Financial institution means a banking company to which the Banking Regulation Act, 1949 applies, or any
bank or banking institution referred to in sec. 51 of that Act and includes any deposit taking non-banking
financial company or a systemically important non-deposit taking non-banking financial company as
defined u/s 43B.
2. Sanction of Loan: The loan has been sanctioned by the financial institution during 01-04-2019 and 31-03-2023.
Quantum of Deduction
Minimum of the following:
a. Interest on loan payable for the previous year
b. ₹ 1,50,000
Other Points
Double deduction is not available: Where a deduction under this section is allowed for any interest, deduction shall
not be allowed in respect of such interest under any other provisions of the Act for the same or any other assessment
year.

3.3.13 Deduction u/s 80G in respect of donations to certain funds, etc.


Applicable to
All assessee (irrespective of residential
status and citizenship of the assessee).
Conditions to be satisfied
1. Donation: Assessee must donate
(not in kind) to specified Funds or
Organisations (as listed below).

Taxpoint: Donation in kind shall not qualify for deduction


2. Mode of donation: Donation in excess of ₹ 2,000 shall be made by any mode (but not in kind) other than cash.
Taxpoint: Cash donation upto ₹ 2,000 shall qualify for deduction.

The Institute of Cost Accountants of India 421


Direct Taxation

3. Proof of donation: Proof of donation, in original, should be attached with the return of income.
Other Points
a. Specified funds or organizations: Specified Funds or Organisations are divided into two categories:
ii. On which limit is not applicable (Item No.1 to 26 given in the list) (hereinafter referred as Category A
Organisation).
iii. On which limit is applicable (Item No.27 to 33 given in the list) (hereinafter referred as Category B
Organisation).
b. Double deduction is not permissible: Where deduction under this section has been allowed, the same shall
not qualify for deduction under any other section for the same or any other Assessment Year
c. Risk Verification: Claim of the assessee for a deduction in respect of any donation made to an institution or
fund to which the provisions of sec. 80G(5) apply, shall be allowed on the basis of information relating to said
donation furnished by the institution or fund to the prescribed income-tax authority or the person authorised
by such authority, subject to verification in accordance with the risk management strategy formulated by the
Board from time to time.
Taxpoint:
¾¾ The limit is applicable only on category B organizations or funds.
¾¾ The limit is applicable on qualifying amount of donation and not on deduction.
¾¾ The limit is not on individual donation but on aggregate donation.
Computation of quantum of deduction

Particulars Amount
100% or 50% of donation to category A organizations or funds ***
Add: 100% or 50% of donation to category B organizations or funds (subject to the Limit#) ***
Deduction u/s 80G ***
#
Limit: 10% of Adjusted Gross total income (hereinafter referred as Adj. GTI)
Adjusted GTI = Gross total income – Long term capital gain – Short term capital gain covered u/s 111A - All
deductions u/s 80C to 80U other than deduction u/s 80G - Income referred u/s 115A, 115AB,
115AC, etc.
Taxpoint: While calculating Adjusted GTI, casual income like winning from lotteries, etc. shall be included.
List of specified organizations or funds

Maximum Deduction (as a % of


Donee
limit net qualifying amount)
1. National defence fund set up by the Central Government NA 100%
2. Prime Minister’s Drought Relief Fund NA 50%
3. Prime Minister’s National Relief Fund NA 100%
4. Prime Minister Citizen Assistance and Relief in Emergency NA 100%
Situations Fund (PM CARES FUND)
5. Prime Minister’s Armenia Earthquake Relief Fund NA 100%
6. Africa (Public Contributions- India) Fund NA 100%
7. National Children’s Fund NA 100%

422 The Institute of Cost Accountants of India


Total Income and Tax Liability of Individuals & HUF

Maximum Deduction (as a % of


Donee
limit net qualifying amount)
8. National Foundation for Communal Harmony NA 100%
9. An approved university or educational institution of national NA 100%
eminence
10. The Chief Minister’s Earthquake Relief Fund, Maharashtra. NA 100%
11. Any fund set up by the Government of Gujarat for providing NA 100%
relief to victims of earthquake in Gujarat
12. Zila Saksharta Samiti NA 100%
13. National or State Blood Transfusion Council NA 100%

14. Fund set up by a State Government for medical relief to the NA 100%
poor
15. Army Central Welfare Fund or Air Force Central Welfare NA 100%
Fund or Indian Naval Benevolent fund
16. Andhra Pradesh Chief Minister’s Cyclone Relief Fund NA 100%
17. National Illness Assistance Fund NA 100%

18. Chief Minister’s Relief Fund or Lieutenant Governor’s Relief NA 100%


Fund
19. National Sports Fund or National Cultural Fund or Fund for NA 100%
Technology Development and Application set up by the Central
Government
20. National Trust for Welfare of Persons with Autism, Cerebral NA 100%
Palsy, Mental Retardation and Multiple Disabilities
21. Swachh Bharat Kosh6 NA 100%
22. Clean Ganga Fund (Only for resident donor)7 NA 100%
23. National Fund for Control of Drug Abuse NA 100%
24. Any other fund or any institution which satisfies conditions Limit 50%
mentioned in Sec 80G(5)
25. Government or any local authority to be utilised for any Limit 50%
charitable purpose other than the purpose of promoting family
planning
26. An authority constituted in India by or under any law enacted Limit 50%
either for the purpose of dealing with and satisfying the need
for housing accommodation or for the purpose of planning,
development or improvement of cities, towns & village, or for
both
27. Any corporation specified in section 10(26BB) for promoting Limit 50%
the interest of minority community

6
The sum is eligible u/s 80G only if the amount is not spent as Corporate Social Responsibility u/s 135(5) of the Companies Act, 2013.
7
The sum is eligible u/s 80G only if the amount is not spent as Corporate Social Responsibility u/s 135(5) of the Companies Act, 2013.

The Institute of Cost Accountants of India 423


Direct Taxation

Maximum Deduction (as a % of


Donee
limit net qualifying amount)
28. Government or any approved local authority, institution or Limit 100%
association to be utilised for the purpose of promoting family
planning
29. Any temple, mosque, gurdwara, church or other place notified Limit 50%
as historic, archaeological or artistic importance or to be a
place of worship by the Central Government (for renovation or
repair)
30. The Indian Olympic Association or to any notified institute Limit 100%
for development of infrastructure for sports and games or
sponsorship of sports and games in India (only donation by a
company)

Illustration 31:
Compute total income for the A.Y.2024-25 of Miss Dipika, a resident individual, from the following details:

Particulars Amount
Profits and gains of business or profession 80,000
Income from Other Sources 10,000
Long-term Capital Gains 5,00,000
Payment of medical insurance premium on own life 5,000
Donation to National Foundation for communal harmony 4,000
Donation to the fund set up by the Gujarat Govt. for providing Relief to victims of earthquake 5,000
in Gujarat
Donation to Prime Minister’s Drought Relief Fund 6,000
Donation to Approved Charitable Institution 12,000
Donation to Central Government for promotion of family planning 3,000
Donation to a poor boy for higher education 10,000
Donation of cloth to an approved institution worth 12,000
Donation to charitable institution for construction of home for a particular community 8,000
Solution :
Computation of total income of Miss Dipika for the A.Y. 2024-25

Particulars Details Amount


Profits & gains of business or profession 80,000
Capital gains: Long term capital gains 5,00,000
Income from Other Sources 10,000
Gross Total Income 5,90,000
Less: Deduction under chapter VIA
Sec. 80D (Assumed premium is paid by cheque) 5,000
Sec. 80G (Donation) 17,750$ 22,750
Total Income 5,67,250

424 The Institute of Cost Accountants of India


Total Income and Tax Liability of Individuals & HUF

$
Statement showing amount of deduction u/s 80G:

Donation made to Amount Rate Deduction


Donation to National Foundation for Communal Harmony 4,000 100% 4,000
Donation to the fund set up by the Gujarat Government for providing 5,000 100% 5,000
relief to victims of earthquake in Gujarat
Donation to Prime Minister’s Drought Relief Fund 6,000 50% 3,000
Donation to the Central Government for promotion of family planning 3,000* 100% 3,000
Donation to approved charitable institution 5,500* 50% 2,750
Total amount of donation u/s 80G 17,750
*
Calculation of Deduction for donation on which limit is applicable:

Computation of limit
Adjusted GTI GTI – LTCG – Deductions other than deduction u/s ₹ 5,90,000 – ₹ 5,00,000 – ₹ 5,000 = ₹
80G 85,000
Limit 10% of Adj. GTI 10% of ₹ 85,000 = ₹ 8,500
Application of limit: There is restriction on amount of donation qualifying under this section. If assessee opts to
adjust qualifying amount with donation made to approved charitable institution, then he is eligible for deduction
to the extent of 50% only. So, it is beneficial for him to adjust qualifying amount, first, with donation made to
the ‘Central Government for promotion of family planning’ as it is eligible for 100% deduction. Hence, donation
shall be restricted -
Donation to Central Government for promotion of family planning 3,000
Donation to approved charitable institution (₹ 8,500 – ₹ 3,000) 5,500
Total 8,500
Donations which are not eligible for deduction u/s 80G are stated below:
Donation Reason
Donation to a poor boy for higher education Donation has not been given to a specified organization
Donation of cloth to an approved institution Donation has been made in kind
Donation to charitable institution for construction of Amount donated is for the benefit of a particular
home for particular community community

3.3.14 Deduction u/s 80GG in respect of house rent paid


Applicable to
An Individual (irrespective of the residential status and citizenship of the individual)
Conditions to be satisfied
1. No House rent allowance: Assessee is not receiving House Rent Allowance (HRA).
2. No house at the place of employment: He or
his spouse or minor child or HUF of which he is
a member, should not own any residential house
at a place where the assessee resides, perform the
duties of his office, or employment or carries on his
business or profession.

The Institute of Cost Accountants of India 425


Direct Taxation

3. No claim for the benefit of self-occupied house property: Assessee should not treat any residential house
situated at other places as self-occupied property u/s 23(2)(a) or 23(4)(a).
4. Proof for payment of rent: A declaration in Form 10BA should be filed for expenditure incurred by him
towards payment of rent.
Taxpoint: Rent must be paid for a residential house property whether furnished or unfurnished.
Quantum of deduction
Minimum of the following:
i. ₹ 5,000 per month;
ii. 25% of Adjusted Gross total income for the year (referred as Adj. GTI); or
iii. The excess of actual rent paid for accommodation over 10% of Adjusted Gross total income.
Arithmetically, [Rent paid - 10% of Adj. GTI]
Note
Adjusted GTI = Gross total income – Long term capital gain – STCG taxable u/s 111A - All deduction u/s 80’s
other than sec. 80GG – Income u/s 115A, 115AB, 115AC, etc.
Taxpoint: While calculating Adjusted GTI, casual income like winning from lotteries etc. shall be included.
Treatment in case of accommodation at concessional rent provided by employer
Where an assessee has been provided an accommodation by his employer at concessional rent, then rent paid by
him shall be:
 deducted while valuing such perquisite; and
 eligible for deduction u/s 80GG.

Illustration 32:
Compute total income of Sri Bajaj of Delhi from the following data:

Particulars Amount
Profits & gains of business or profession 80,000
Income from house property (let-out and situated at Kolkata) 40,000
Income from other sources 10,000
Rent paid for office 8,000
Rent paid for residential house 40,000
Solution :
Calculation of total income of Sri Bajaj for the A.Y. 2024-25

Particulars Amount
Profits & gains of business or profession 80,000
Income from house property 40,000
Income from other sources 10,000
Gross Total Income 1,30,000
Less: Deduction u/s 80GG #
27,000

426 The Institute of Cost Accountants of India


Total Income and Tax Liability of Individuals & HUF

Particulars Amount
Total Income 1,03,000
#
Computation of deduction u/s 80GG
Particulars Working Details Amount
Least of the following shall be deductible:
1. ₹ 5,000 per month ₹ 5,000 * 12 60,000
2. 25% of Adjusted Gross total income 25% of ₹ 1,30,000# 32,500
3. Excess of rent paid over 10% of Adj. GTI ₹ 40,000 - (10% of ₹ 1,30,000 )
#
27,000 27,000
#
Adjusted GTI = Gross total income – Long term capital gain – Short term capital gain covered u/s 111A - All
deduction under 80’s other than section 80GG – Income u/s 115A, etc. = ₹ 1,30,000

Note: Rent paid for office is irrelevant for the purpose of Sec. 80GG.

3.3.15 Deduction u/s 80GGA in respect of donations for scientific research, etc.
Applicable to
An assessee, who is not having any income under the head “Profits & gains of business or profession”.
Conditions to be satisfied
1. During the previous year, assessee paid any sum -

Purposes Amount paid to


For Rural ¾¾ Approved association or institution, being engaged in any approved programme
development of rural development, to be used for carrying any rural development programme
provided the assessee furnishes a certificate as referred in sec. 35CCA(2);
¾¾ Approved association or institution engaged in training the persons for
implementing programme of rural development provided that the assessee
furnishes a certificate referred in sec. 35CCA(2A);
¾¾ Notified Rural Development Fund for the purpose of sec. 35CCA(1)(c)
For poverty Notified National Urban Poverty Eradication Fund setup and notified by the Central
eradication Government for the purpose of sec. 35CCA(1)(d).
For Scientific Research association, uni­versity, college or to other institution as approved u/s 35(1)
research (ii) to be used for scientific research.
For social science Research association which has as its object the undertaking of research in social
or statistical science or statistical research or to a university, college or other institution approved
research u/s 35(1)(iii) to be used for research in social science or statisti­cal research.
For other purpose A public sector company or a local authority or an association or institution approved
by the National Committee for carrying out any eligible project or scheme, provided a
certificate is obtained from such company etc. as mentioned u/s 35AC(2)(a).
2. Mode of Payment: Payment in excess of ₹ 2,000 to aforesaid purpose shall be made by any mode other than
cash.
Quantum of deduction
Amount actually paid is fully deductible.
Other Points

The Institute of Cost Accountants of India 427


Direct Taxation

Withdrawal of approval: Deduction shall not be denied merely on the ground that subsequent to the contribution
made by the assessee, the approval granted (or notification notifying eligible projects) has been withdrawn.
Risk Verification: Claim of the assessee for a deduction in respect of any sum referred to in sec. 80GGA(2), shall
be allowed on the basis of information relating to said sum furnished by the payee to the prescribed income-tax
authority or the person authorised by such authority, subject to verification in accordance with the risk management
strategy formulated by the Board from time to time.

3.3.16 Deduction in respect of contributions to political parties [Sec. 80GGB]


Applicable to
An Indian company
Condition
Assessee has contributed any sum (by any mode other than cash), in the previous year, to any political party or
an electoral trust.
Quantum of deduction
100% of such contribution made in the previous year.

3.3.17 Deduction in respect of contributions given by any person to political parties [Sec.
80GGC]
Applicable to
All assessee except local authority and every artificial juridical person wholly or partly funded by the Government
Condition
Assessee has contributed any sum (by any mode other than cash), in the previous year, to any political party or
an electoral trust.
Quantum of deduction
100% of such contribution made in the previous year.

3.3.18 Deductions in respect of profits and gains from industrial undertakings or enterprises
engaged in infrastructure development, etc. [Sec. 80-IA]
Deduction under this section is available for industrial undertaking engaged in the following business–
 Infrastructure Facility
 Industrial parks or special economic zone;
 Power generation, transmission and distribution
Infrastructure facility
Applicable to
Indian Company or any consortium (combination) of such company or by an authority or a board or a corporation
or any other body established or constituted under any Central or State Act.
Conditions to be satisfied
1. Nature of business: The enterprise should carry on the business of -

428 The Institute of Cost Accountants of India


Total Income and Tax Liability of Individuals & HUF

a. Developing; or
b. Operating and maintaining; or
c. Developing, operating and maintaining,
- any infrastructure facility.
Infra-structure facility means —
a. A road including toll road, a bridge or a rail system;
b. A highway project including housing or other activities being an integral part of the highway project;
c. A water supply project, water treatment system, irrigation project, sanitation and sewerage system or
solid waste management system;
d. A port, airport, inland waterway or inland port or navigational channel in the sea.
2. Agreement: Assessee has entered into an agreement with the Central Government or a State Government or a
local authority or any other statutory body for -
i. developing; or
ii. operating and maintaining; or
iii. developing, operating and maintaining
- a new infrastructure facility.
3. Commencement of business: It has started or starts operating & maintaining the facility on or after the 1-4-
1995 but before 01-04-2017.
Quantum of Deduction

Case Quantum of Deduction Period of deduction


For infrastructure facility being -
A port, airport, inland waterway For any 10 consecutive years out of 15 years
100% of the profit derived
or inland port or navigational commencing from the year in which the
from such business
channel in the sea undertaking begins to operate.
Any housing and other For any 10 consecutive years out of 20
development activities which are 100% of the profit derived years commencing from the year in which
an integral part of the highway from such business the undertaking begins to operate subject to
project certain other conditions#
For any 10 consecutive years out of 20 years
100% of the profit derived
Any other facility commencing from the year in which the
from such business
undertaking begins to operate.
The conditions for claiming deduction in respect of any profit derived from housing and other development
activities, which are an integral part of the highway project are as follows -
i. Profit of such undertaking shall be computed in prescribed manner.
ii. Such profit has been transferred to special reserve account;
iii. The reserve shall be utilised for highway project excluding housing and other activities before the expiry
of 3 years following the previous year in which such amount was transferred; &

The Institute of Cost Accountants of India 429


Direct Taxation

iv. Every assessee shall maintain separate accounts for such activities & shall submit a certificate in Form
10CCC from an accountant, specifying the amount credited to the reserve account and the amount
utilised during the relevant previous year for the highway project.
Notes
a. Method of computation of profit of housing or other activities, which are an integral part of the highway
project

In a case where the annual profits can be


arrived at in accordance with the regular The profits computed as per provisions of the Act
method of accounting followed
The profits based on the percentage of completion of such
In any other case
activities during the previous year.
b. Consequence when reserve is not so utilized: Where any reserve is not so utilised or unutilized, then such
amount shall be taxable as income of the previous year in which transfer to reserve took place.

Industrial Park or Special Economic Zone


Applicable to
All assessee
Condition to be satisfied
1. Nature of business: Such undertaking should be engaged in the business of -
 Developing; or
 Developing and operating or maintaining; or
 Operating,
- industrial park or special economic zone notified by Central Government.
2. Commencement of business: Such undertaking develops, develops and operates or maintains and operates
such park or zone for the following period -

In case of Period
Industrial park 1-4-1997 to 31-3-2011
Special Economic Zone 1-4-2001 to 31-3-2006

Quantum of Deduction
100% of profit derived from such business for any 10 consecutive years out of 15 years commencing from the year
in which the undertaking begins to operate.
From the date of application of sec. 80-IAB, no deduction shall be available u/s 80-IA for SEZ notified on or after
1-4-2005.

Power generation, transmission and distribution


Applicable to
All assessee

430 The Institute of Cost Accountants of India


Total Income and Tax Liability of Individuals & HUF

Conditions to be satisfied
1. Nature of business: Such undertaking is set up in India for the generation or generation and distribution of power.
2. Commencement of business: Such undertaking commences business during the following time schedule:

In case of Period
Generation of power 1-4-1993 to 31-3-2017
Transmission or distribution by laying a network of new transmission or distribution
1-4-1999 to 31-3-2017
lines
Substantial renovation and modernization1 of the existing transmission or distribution
1-4-2004 to 31-3-2017
lines
“Substantial renovation and modernisation” means an increase in the plant and machinery in the network
1.

of transmission or distribution lines by at least 50% of the book value of such plant and machinery as on the
1-4-2004.
3. New Business: Business should not be formed by splitting up or reconstruction of an existing business.
Exception
However, this condition is not applicable in the following cases:
¾¾ In case of reconstruction, splitting up or reorganization of State Electricity Board;
¾¾ In case when conditions given u/s 33B are satisfied, which are as follows -
a. The business of an industrial undertaking carried on in India is discontinued in any previous year by
reason of extensive damage to, or destruction of, any building, machinery, plant or furniture owned
by the assessee being used for business purpose.
b. Such damage was caused due to -
i. flood, typhoon, hurricane, cyclone, earthquake or other convulsion of nature; or
ii. riot or civil disturbance; or
iii. accidental fire or explosion; or
iv. action by an enemy or action taken in combating an enemy (whether with or without a declaration
of war),
c. Such business is re-established, reconstructed or revived by the assessee at any time before the
expiry of 3 years from the end of previous year in which damage was caused.
4. New Plant & Machinery: Such undertaking should not be formed by transfer of machinery or plant previously
used for any purpose.
Exception
i. In case of reconstruction, splitting up or reorganization of State Electricity Board;
ii. A plant or machinery is deemed as new asset if the following conditions are satisfied -
a. Such plant or machinery is imported into India;
b. Depreciation on such asset has not been allowed under this Act to any person; and
c. The assessee was the first user of such asset in India.
iii. Where the total value of old plant and machinery transferred to the new business does not exceed 20%
of total value of plant and machinery used in such business, then this condition is deemed to be satisfied.

The Institute of Cost Accountants of India 431


Direct Taxation

Quantum of Deduction
100% of profit derived from such business for any 10 consecutive years out of 15 years commencing from the year
in which the undertaking begins to operate.
Note: In case of undertaking engaged in transmission or distribution by laying a network of new transmission or
distribution lines, deduction shall be limited to the extent of profit derived from laying of such network of new
lines.

3.3.19 Deduction in respect of profits and gains by an undertaking or enterprise engaged


in development of Special Economic Zone [Sec. 80-IAB]
Applicable to
All assessee
Conditions to be satisfied
1. It must be engaged in the business of developing a Special Economic Zone (SEZ).
2. Such SEZ shall be notified on or after 1-4-2005 under the Special Economic Zones Act, 2005
3. Such development has begun before 01-04-2017.
Quantum of deduction
100% of profit derived from such business for 10 consecutive assessment years out of 15 years commencing from
the year in which such SEZ has been notified.
Other Points
a. Where a developer transfers the operation and maintenance of such SEZ to another developer, then such other
developer shall claim deduction for remaining period.
b. From the date of application of this section, no deduction shall be available u/s 80-IA for SEZ notified on or
after 1-4-2005.
c. Where any deduction is allowed under this section, then deduction shall not be allowed under any other
provisions.
d. Return of income in time: Refer sec. 80-IA
e. Audit report: Refer sec. 80-IA
f. Power of Assessing Officer to recompute profit: Refer sec. 80-IA
g. No deduction shall be allowed if the deduction has not been claimed in the return of income.
h. Profit from such business is only source of income: Refer sec. 80-IA

3.3.20 Deduction in respect of eligible start-up [Sec. 80-IAC] Amended


Applicable to
An eligible start-up
 Eligible start-up means a company or a limited liability partnership engaged in eligible business which fulfils
the following conditions:
a. it is incorporated on or after 01-04-2016 but before 01-04-2024;

432 The Institute of Cost Accountants of India


Total Income and Tax Liability of Individuals & HUF

b. the total turnover of its business does not exceed ₹ 100 crore in the previous year relevant to the
assessment year for which deduction is claimed; and
c. it holds a certificate of eligible business from the notified Inter-Ministerial Board of Certification.
 Eligible business means a business carried out by an eligible start up engaged in innovation, development
or improvement of products or processes or services or a scalable business model with a high potential of
employment generation or wealth creation.
Conditions to be satisfied
1. Eligible Business: The assessee should be engaged in the eligible business
2. New Business: Business should not be formed by splitting up or reconstruction of an existing business.
Exception
However, this condition is not applicable when conditions given u/s 33B are satisfied, which are as follows -
¾¾ The business of an industrial undertaking carried on in India is discontinued in any previous year by
reason of extensive damage to, or destruction of, any building, machinery, plant or furniture owned by
the assessee being used for business purpose.
¾¾ Such damage was caused due to -
i. flood, typhoon, hurricane, cyclone, earthquake or other convulsion of nature; or
ii. riot or civil disturbance; or
iii. accidental fire or explosion; or
iv. action by an enemy or action taken in combating an enemy (whether with or without a declaration of
war),
¾¾ Such business is re-established, reconstructed or revived by the assessee at any time before the expiry of
3 years from the end of previous year in which damage was caused.
3. New Plant and Machinery: Such undertaking should not be formed by transfer of machinery or plant
previously used for any purpose.
Exception
i. A plant or machinery is deemed as new asset if the following conditions are satisfied -
a. Such plant or machinery is imported into India;
b. Depreciation on such asset has not been allowed under this Act to any person; and
c. The assessee was the first user of such asset in India.
ii. Where the total value of old plant and machinery transferred to the new business does not exceed 20%
of total value of plant and machinery used in such business, then this condition is deemed to be satisfied.
Quantum of deduction
100% of profit derived from such business for 3 consecutive assessment years out of 10 years commencing from
the year in which such eligible start-up is incorporated.
Other Points
a. Where any deduction is allowed under this section, then deduction shall not be allowed under any other
provisions.

The Institute of Cost Accountants of India 433


Direct Taxation

b. Audit report: Refer sec. 80-IA


c. Return of income in time: Refer sec. 80-IA
d. Power of Assessing Officer to recompute profit: Refer sec. 80-IA
e. No deduction shall be allowed if the deduction has not been claimed in the return of income.
f. Profit from such business is only source of income: Refer sec. 80-IA

3.3.21 Deduction in respect of profits and gains from certain industrial undertakings other
than infrastructure development undertakings [Sec. 80-IB]
Deduction under this section is available to undertaking engaged in –
1. Industrial undertaking
2. Mineral oils
3. Processing, preservation and packaging of fruits or vegetables, meat or integrated business of handling, storage
& transportation of foodgrains

Industrial Undertaking [Sec. 80-IB(2)]


Applicable to
All assessee having industrial undertaking located in the backward State being Jammu & Kashmir
Conditions to be satisfied
 Nature of business: It must be engaged in the business of manufacturing or producing any article or thing
other than article or thing specified in the list in the 11th Schedule and in the manufacture or production of any
article or thing specified in Part C of the 13th Schedule; or
¾¾ Manufacture, with its grammatical variations, means a change in a non-living physical object or article
or thing:
a. resulting in transformation of the object or article or thing into a new and distinct object or article
or thing having a different name, character and use; or
b. bringing into existence of a new and distinct object or article or thing with a different chemical
composition or inte­gral structure.
¾¾ Manufacture or production of article or thing does not include construction of dam, bridge, road or
building
¾¾ Undertaking located in the State of Jammu and Kashmir shall not manufacture or produce any article
mentioned in the 11th Schedule as well as following items (being mentioned in Part C of 13th Schedule):
a. Cigarettes/cigars of tobacco, manufactured tobacco and substitutes
b. Distilled/brewed alcoholic drinks
c. Aerated branded beverages and their concentrates:.
Exception
However, following undertakings may produce any article specified in the 11th schedule -
 A small-scale industrial undertaking; or
 An industrial undertaking in an industrially backward State (or Union territories) specified in the 8th
Schedule.

434 The Institute of Cost Accountants of India


Total Income and Tax Liability of Individuals & HUF

2. New Business: Business should not be formed by splitting up or reconstruction of an existing business.
Exception: Refer sec. 80-IAC.
3. New Plant and Machinery: Such undertaking should not be formed by transfer of machinery or plant
previously used for any purpose.
Exception: Refer sec. 80-IAC.
4. No. of worker: Such industrial undertaking employs specified number of workers -

Undertaking manufacturing articles or thing Minimum number of worker


With the aid of power 10 workers
Without the aid of power 20 workers
5. Commencement of business: Such undertaking starts its operation within 1-4-1993 to 31-3-2012
Quantum of Deduction

Deduction where undertaking is owned by


Company Co-operative society Any other person
100% for 5 years and 30% for next 100% for 5 years and 25% for next 100% for 5 years and 25% for next
5 years 7 years 5 years
The deduction shall be commenced from assessment year relevant to the previous year in which it begins to
manufacture or produce articles or thing

Minerals Oil [Sec. 80-IB(9)]


Applicable to
All assessee
Conditions to be satisfied
1. Nature of business: The undertaking shall be engaged in the production or refining of mineral oil in any part
of India.
2. Time: The undertaking has begun commercial production within the following time schedule –

Nature of business Time


¾¾ Production of mineral oil After 31-3-1997 but before 01-04-2017
¾¾ Refining of mineral oil After 30-9-1998 but before 31-03-2012
¾¾ Commercial production of natural gas* On or after 01-09-2009 but before 01-04-2017

* in blocks licensed under the VIII Round of bidding for award of exploration contracts (hereafter referred to
as “NELP-VIII”) under the New Exploration Licencing Policy announced by the Government of India vide
Resolution No. O-19018/22/95-ONG.DO.VL, dated 10th February, 1999 or in blocks licensed under the IV
Round of bidding for award of exploration contracts for Coal Bed Methane blocks. The deduction shall not
apply to blocks licensed under a contract awarded after 31-03-2011 under the New Exploration Licencing
Policy announced by the Government of India vide Resolution No. O-19018/22/95-ONG.DO.VL, dated 10-
02-1999 or in pursuance of any law for the time being in force or by the Central or a State Government in any
other manner.

The Institute of Cost Accountants of India 435


Direct Taxation

Quantum of Deduction
100% of the profits derived from such business for a period of 7 consecutive years starting from assessment
year relevant to the previous year in which the undertaking commences the commercial production or refining of
mineral oil.

Processing, preservation and packaging of fruits or vegetables, meat or integrated business of handling,
storage & transportation of foodgrains [Sec.80-IB (11A)]
Applicable to
All assessee
Conditions to be satisfied
1. The undertaking shall be engaged in -
¾¾ the business of processing, preservation and packaging of fruits or vegetables; or
¾¾ the business of processing, preservation and packaging of meat and meat products or poultry or marine
or dairy products; or
¾¾ the integrated business of handling, storage and transportation of foodgrains.
2. Commencement of operations: The aforesaid business begins to operate on or after following period:

Nature of business Business begins to operate


Processing, preservation and packaging of meat and meat products or On or after 01-04-2009
poultry or marine or dairy products
Other specified business On or after 01-04-2001
Quantum of deduction

Period of deduction Percentage of deduction


First 5 years 100% of profit from such business
For next 5 years 25% (30% in case of company) of profit from such business

Other Points applicable on any undertaking covered by Sec. 80IB


For followings, refer sec. 80-IA
1. Return of income in time
2. Audit Report
3. Power of Assessing Officer to recompute profit
4. Double deduction is not allowed
5. Computation of profit of eligible business
6. Deduction must be claimed in the return of income

3.3.22 Deduction in respect of profits and gains from Housing Projects [Sec. 80-IBA]
Applicable to
All assessee

436 The Institute of Cost Accountants of India


Total Income and Tax Liability of Individuals & HUF

Conditions to be satisfied
1. Housing Project or Rental Housing Project: The assessee is engaged in the business of
a. developing and building housing projects; or
b. developing and building rental housing project.
Housing project shall be a project which fulfils the following conditions:
a. The project is approved by the competent authority after 01-06-2016, but on or before 31-03-2022.
b. The project is completed within a period of 5 years from the date of approval by the competent authority.
 Where the approval in respect of a housing project is obtained more than once, the project shall be
deemed to have been approved on the date on which the building plan of such housing project was
first approved by the competent authority;
 The project shall be deemed to have been completed when a certificate of completion of project as a
whole is obtained in writing from the competent authority.
c. The carpet area of the shops and other commercial establishments included in the housing project does
not exceed 3% of the aggregate carpet area.
d. The project should satisfy the following measuring criteria:

Maximum Size Minimum


Minimum
of carpet area of utilisation of
Location of the plot Size of plot
the residential Floor Area
of land
unit Ratio
Within the metropolitan cities of Bengaluru, 1,000 sq. 60 sq. meters 90%
Chennai, Delhi National Capital Region (limited meters
to Delhi, Noida, Greater Noida, Ghaziabad,
Gurugram, Faridabad), Hyderabad, Kolkata and
Mumbai (whole of Mumbai Metropolitan Region)
Any other place 2,000 sq. 90 sq. meters 80%
meters
e. The project is the only housing project on such plot of land;
f. The stamp duty value of a residential unit in the housing project does not exceed ₹ 45 lakh.
g. Where a residential unit in the housing project is allotted to an individual, no other residential unit in the
housing project shall be allotted to the individual or the spouse or the minor children of such individual;
h. The assessee maintains separate books of account in respect of the housing project.
Quantum of Deduction
100% of the profits derived from such business.
Other Points
 Deduction is not available to any assessee who executes the housing project as a works-contract awarded by
any person (including the Central Government or the State Government).
 Where the housing project is not completed within the specified period, then deduction has been claimed and
allowed earlier, shall be deemed to be the income of the assessee chargeable under the head “Profits and gains
of business or profession” of the previous year in which the period for completion so expires.

The Institute of Cost Accountants of India 437


Direct Taxation

 Double deduction is not available.


 Return of income in time: Refer sec. 80-IA
 Competent authority means the authority empowered to approve the building plan by or under any law for the
time being in force.
 Floor Area Ratio means the quotient obtained by dividing the total covered area of plinth area on all the floors
by the area of the plot of land.
 Housing project means a project consisting predominantly of residential units with such other facilities and
amenities as the competent authority may approve subject to the provisions of this section.
 Rental housing project means a project which is notified by the Central Government on or before 31-03-2022
and fulfils such conditions as may be specified in the said notification.
 Residential unit means an independent housing unit with separate facilities for living, cooking and sanitary
requirements, distinctly separated from other residential units within the building, which is directly accessible
from an outer door or through an interior door in a shared hallway and not by walking through the living space
of another household.
 Carpet area means the net usable floor area of an apartment, excluding the area covered by the external walls,
areas under services shafts, exclusive balcony or verandah area and exclusive open terrace area, but includes
the area covered by the internal partition walls of the apartment.

3.3.23 Special provisions in respect of certain undertakings in North- Eastern States [Sec.
80-IE]
Applicable to
All assessee
Conditions to be satisfied
1. Time and Nature of business: The assessee begins following activity during 01-04-2007 and 31-03-2017:
a. manufacture or production of any eligible article or thing; or
Eligible article or thing means the article or thing other than the following:
i. Tobacco and manufactured tobacco substitutes;
ii. Pan masala;
iii. Plastic carry bags of less than 20 microns as specified by the Ministry of Environment and Forests;
and
iv. Goods produced by petroleum oil or gas refineries.
b. undertakes substantial expansion; or
Substantial expansion means increase in the investment in the plant and machinery by at least 25% of
the book value of plant and machinery (before taking depreciation in any year), as on the first day of the
previous year in which the substantial expansion is undertaken.
c. carry on any eligible business.
i. hotel (not below two star category);
ii. adventure and leisure sports including ropeways;
iii. providing medical and health services in the nature of nursing home with a minimum capacity of
25 beds;

438 The Institute of Cost Accountants of India


Total Income and Tax Liability of Individuals & HUF

iv. running an old-age home;


v. operating vocational training institute for hotel management, catering and food craft, entrepreneurship
development, nursing and para-medical, civil aviation related training, fashion designing and
industrial training;
vi. running information technology related training centre;
vii. manufacturing of information technology hardware; and
viii. Bio-technology
2. Location: Such activity takes place in any of the North-Eastern States.
North-Eastern States means the States of Arunachal Pradesh, Assam, Manipur, Meghalaya, Mizoram,
Nagaland, Sikkim and Tripura.
3. New Business: It is not formed by splitting up, or the reconstruction, of a business already in existence
Exception: Refer sec. 80-IAC
4. New Plant & Machinery: It is not formed by the transfer to a new business of machinery or plant previously
used for any purpose.
Exception: Refer sec. 80-IAC
5. Return of income is required to be furnished within due date.
6. A report of audit from chartered accountant (certifying that the deduction has been correctly claimed) shall be
furnished one month prior to the due date of filing of the return of income.

Quantum of deduction
100% of the profits and gains of such business for a period of 10 consecutive assessment years, beginning from the
assessment year relevant to the previous year in which such undertaking begins to manufacture or produce articles
or things, or completes substantial expansion.

Other Points
a. Double deduction is not available.
b. Power of Assessing Officer to recomputed profit: Refer sec. 80-IA
c. Profit from such business is only source of income: Refer sec. 80-IA
d. No deduction shall be allowed if the deduction has not been claimed in the return of income.

3.3.24 Deduction u/s 80JJA in respect of profits and gains of business of collecting and
processing of bio-degradable waste
Applicable to
All assessee
Conditions to be satisfied
Assessee is engaged in business of collecting and processing or treating of bio-degradable waste for –
a) generating power; or b) producing bio-fertilizers, bio-pesticides or other biological agents; or c) producing bio-
gas; or d) making pellets or briquettes for fuel; or e) organic manure.

The Institute of Cost Accountants of India 439


Direct Taxation

Quantum of deduction
100% of the profit derived from such business for a period of 5 consecutive years from the year of commencement
of such business.
Other Points
a. Double deduction is not allowed.
b. Inter-unit transfer: Refer sec. 80-IA

c. Return of income in time: Refer sec. 80-IA


d. No deduction shall be allowed if the deduction has not been claimed in the return of income.

3.3.25 Deduction u/s 80JJAA in respect of employment of new workmen


Applicable to
Any assessee subject to tax audit under sec. 44AB
Conditions to be satisfied
1. New Business: Business is not formed by splitting up, or the reconstruction, of an existing business. Exception:
Refer sec. 80-IAC.
2. No business reorganization: Business is not acquired by the assessee by way of transfer from any other
person or as a result of any business reorganization.
3. Audit: Books of account should be audited and report thereof should be submitted one month prior to the due
date of the filing of the return of income.
4. Claimed in the return: No deduction shall be allowed if the deduction has not been claimed in the return of
income.
Quantum of deduction
30% of additional employee cost incurred in the course of such business in the previous year, for 3 assessment
years including the assessment year relevant to the previous year in which such employment is provided
Other Points
 Return of income in time: Refer sec. 80-IA
 Additional employee cost means the total emoluments paid or payable to additional employees employed
during the previous year.
¾¾ In the case of an existing business, the additional employee cost shall be nil, if:
a. there is no increase in the number of employees from the total number of employees employed as
on the last day of the preceding year;
b. emoluments are paid otherwise than by an account payee cheque or account payee bank draft or
by use of electronic clearing system through a bank account or through other specified electronic
modes.
¾¾ In the first year of a new business, emoluments paid or payable to employees employed during that
previous year shall be deemed to be the additional employee cost.

440 The Institute of Cost Accountants of India


Total Income and Tax Liability of Individuals & HUF

 Additional employee means an employee who has been employed during the previous year and whose
employment has the effect of increasing the total number of employees employed by the employer as on the
last day of the preceding year, but does not include:
a. an employee whose total emoluments are more than ₹ 25,000 per month; or
b. an employee for whom the entire contribution is paid by the Government under the Employees’ Pension
Scheme notified in accordance with the provisions of the Employees’ Provident Funds and Miscellaneous
Provisions Act, 1952; or
c. an employee employed for a period of less than 240 days (150 days in case of an assessee who is engaged
in the business of manufacturing of apparel or footwear or leather products) during the previous year; or
Taxpoint: Where an employee is employed during the previous year for a period of less than 240 days (or
150 days), but is employed for a period of 240 days (or 150 days) in the immediately succeeding year, he
shall be deemed to have been employed in the succeeding year.
d. an employee who does not participate in the recognised provident fund.
 Emoluments means any sum paid or payable to an employee in lieu of his employment by whatever name
called, but does not include:
a. any contribution paid or payable by the employer to any pension fund or provident fund or any other fund
for the benefit of the employee under any law for the time being in force; and
b. any lump-sum payment paid or payable to an employee at the time of termination of his service or
superannuation or voluntary retirement, such as gratuity, severance pay, leave encashment, voluntary
retrenchment benefits, commutation of pension and the like.
Provision Illustrated
ABC Ltd., an existing company appoints following employees during the F.Y. 2023-24.

Case No. of employees Date of appointment Salary (in ₹ per person per month)
1 10 02-May-2023 25,000
2 15 01-June-2023 28,000
3 20 01-March-2024 24,500
Compute deduction u/s 80JJAA
Solution
Computation of deduction u/s 80JJAA

Case Total salary (in ₹) Amount of deduction


1 27,50,000 ₹ 8,25,000 i.e., 30% of ₹ 27,50,000
2 42,00,000 Nil as salary exceeds ₹ 25,000 p.m.
3 4,90,000 Nil as employees are not employed for the required number of days during the
previous year

The Institute of Cost Accountants of India 441


Direct Taxation

3.3.26 Deduction u/s 80LA in respect of certain incomes of Offshore Banking Units Amended
Applicable to
 A scheduled bank owning an offshore banking unit in a Special Economic Zone (SEZ); or
 Any other bank incorporated by or under the laws of a country outside India and having an Offshore banking
unit in SEZ.
 Unit of an International Financial Services Center.
Offshore Banking Unit means a branch of a bank in India located in the SEZ and has obtained the permission
u/s 23(1)(a) of the Banking Regulation Act, 1949.
International Financial Services Center means an International Financial Services Center which has been
approved by the Central Government u/s 18(1) of the Special Economic Zones Act, 2005.
Taxpoint:
 Assessee must be a scheduled bank;
 It has a branch in India located in SEZ
Conditions to be satisfied
1. Certificate of a chartered accountant: Assessee must furnish along with the return of income, a report of a
chartered accountant in Form 10CCF, certifying that the deduction has been correctly claimed.
2. Submission of permission: A copy of the permission obtained u/s 23(1)(a) of the Banking Regulation Act,
1949 or copy of permission or registration obtained under the International Financial Services Centre Authority
Act, 2019 is required to be furnished along with the return of income.
Quantum of deduction

In case of Unit of an International Financial Services Center


–– For any 10 consecutive assessment years, at the option of the assessee, out of 15 years,
beginning with the assessment year relevant to the previous year in which the permission,
100% of the
u/s 23(1)(a) of the Banking Regulation Act, 1949 or permission or registration under the
income
Securities and Exchange Board of India Act, 1992 or permission or registration under the
International Financial Services Centre Authority Act, 2019 was obtained.
In any other case
–– For first 10 consecutive years beginning with the year in which the permission u/s 23(1)(a)
100% of the
of the Banking Regulation Act, 1949 was obtained or permission or registration under the
income
SEBI Act, 1992 or any other relevant law was obtained.
Income here means –
a. Income from an offshore banking unit in a SEZ;
b. Income from the business referred in Sec. 6(1) of the Banking Regulation Act, 1949 with an undertaking
located in a SEZ or any other undertaking which develops or develops and operates or operates and maintains
a SEZ;
c. Income from any Unit of the International Financial Services Centre from its business for which it has been
approved for setting up in such a Centre in a Special Economic Zone;
d. Income from the transfer of an asset, being an aircraft or a ship, which was leased by such unit to a person,
subject to the condition that the unit has commenced operation on or before 31-03-2024.
Other Points
a. Double deduction is not allowed

442 The Institute of Cost Accountants of India


Total Income and Tax Liability of Individuals & HUF

b. Inter-unit transfer: Refer sec. 80-IA


c. Return of income in time: Refer sec. 80-IA
d. No deduction shall be allowed if the deduction has not been claimed in the return of income.

3.3.27 Deduction in respect of inter-corporate dividend [Sec. 80M]


Applicable to
Domestic Company
Conditions to be satisfied
a. Dividend Income: Gross total income of the assessee includes any income by way of dividends from any
other domestic company or a foreign company or a business trust.
b. Dividend Distribution: Assessee distributes dividend among its shareholder within due date
¾¾ Due date means the date one month prior to the due date for furnishing the return of income.
Quantum of Deduction
Minimum of the following:
 Dividend so received by the assessee; or
 Dividend distributed by the assessee within due date
Other Points
No Double Deduction: Where any deduction, in respect of the amount of dividend distributed by the domestic
company, has been allowed in any previous year, no deduction shall be allowed in respect of such amount in any
other previous year.

3.3.28 Deduction in respect of income of Producer Companies [Sec. 80PA]


Applicable to
Producer Company as defined u/s 581A of the Companies Act, 2013
Conditions to be satisfied
a. Turnover: Total turnover of the assessee is less than ₹ 100 crore in any previous year.
b. Profit from eligible business: Gross total income of the assessee includes any income from eligible business.
Eligible Business means—
a. the marketing of agricultural produce grown by the members; or
b. the purchase of agricultural implements, seeds, livestock or other articles intended for agriculture for the
purpose of supplying them to the members; or
c. the processing of the agricultural produce of the members;
Quantum of deduction
100% of the profits and gains attributable to such eligible business
Other Points
a. No deduction shall be available u/s 80PA from A.Y. 2025-26
The Institute of Cost Accountants of India 443
Direct Taxation

b. No double deduction
c. Return of income in time: Refer sec. 80-IA

3.3.29 Deduction u/s 80QQB in respect of royalty income of authors of books


Applicable to
A resident individual (irrespective of his citizenship)
Conditions to be satisfied
1. Profession of assessee: Assessee is an author or joint author of a book.
2. Nature of book: The book should be a work of literary, artistic or scientific nature. It shall not include
brochures, commentaries, diaries, guides, magazines, journals, newspapers, pamphlets, text-books for school,
tracts and other publications of similar nature, by whatever name called.
3. Nature of income: Gross total income of the assessee includes any income by way of –
a. any lump sum consideration for the assignment or grant of any of his interest in the copyright of book; or
b. royalty or copyright fees (whether receivable in lump sum or otherwise) in respect of the book. earned
in India or earned from any source outside India.
Taxpoint: Consideration includes advance receipt of such nature, which is not returnable.
However, when such income is earned from a source outside India then –
¾¾ Remittance: Such receipts should be brought into India by the assessee in convertible foreign exchange#
within 6 months from the end of the previous year or within such further period as the competent authority*
may allow in this behalf; and
¾¾ Certificate in Form 10H: The assessee must furnish a certificate in the Form 10H (obtained from the
prescribed authority) along with his return of income.
#
Convertible foreign exchange means foreign exchange which is for the time being treated by the Reserve
Bank of India as convertible foreign exchange for the purpose of the Foreign Exchange Regulation Act,
1973 and any rules made thereunder.
Competent authority means the Reserve Bank of India or such other authority as is authorized under any
law for the time being in force for regulating payments and dealings in foreign exchange.
4. Report: Assessee must furnish a certificate along with the return of income in Form 10CCD duly verified by
the person responsible for making such payment to the assessee.
Quantum of deduction
1. Deduction in case of lump sum royalty in lieu of all rights of the assessee in the book :-
Minimum of the following –
a. 100% of such income; or
b. ₹ 3,00,000

444 The Institute of Cost Accountants of India


Total Income and Tax Liability of Individuals & HUF

2. Deduction in other case –

Case 1 Case 2

Royalty or fee is not more than 15% of value of Royalty or fee is more than 15% of value of the book
the book

Minimum of the following – Minimum of the following –


 100% of such income; or  100% of such income considering the royalty as
15% of value of book sold; or
 ₹ 3,00,000
 ₹ 3,00,000

Taxpoint: In case of royalty or copyright fees i.e. not a lump sum consideration, deduction shall be restricted
to 15% of the value of books (before allowing expenses attributable to such income) sold during the previous
year.
Computation of deduction

Income Deduction
Income received in Lump sum
Minimum of the following –
Income earned in
a. 100% of such income (Receipts – Expenditure relating to such income)
India
b. ₹ 3,00,000
Minimum of the following –
Income earned a. Money brought into India in convertible foreign exchange within time limit –
outside India Expenditure incurred relating to such income
b. ₹ 3,00,000
Income received in a mode other than lump sum
Minimum of the following -
Income earned in a. 100% of income (Receipts – Expenditure relating to such income)
India b. 15% of sale value of the book – Expenditure relating to such income
c. ₹ 3,00,000
Minimum of the following -
a. Money brought in India in convertible foreign exchange within time limit–
Income earned Expenditure incurred relating to such income
outside India
b. 15% of sale value of the book – Expenditure incurred relating to such income
c. ₹ 3,00,000
Other point
a. Double deduction is not allowed
b. Return of income in time: Refer sec. 80-IA
c. No deduction shall be allowed if the deduction has not been claimed in the return of income.

The Institute of Cost Accountants of India 445


Direct Taxation

3.3.30 Deduction u/s 80RRB in respect of royalty on patents


Applicable to

A resident individual (irrespective of citizenship of the individual), being a patentee (i.e. owner or co-owner of a
patent)

Patentee means a person(s) who is true and first inventor of the invention and whose name is entered on the Patent
register as the patentee (as owner or co-owner of a patent) in accordance with the Patent Act, 1970.

Conditions to be satisfied
1. Nature of income: Assessee has earned income (either in India or outside India) by way of royalty in respect
of patent registered on or after 1-4-2003 under the Patents Act, 1970.
Taxpoint:
¾¾ Royalty, in respect of a patent, means consideration (including any lump sum consideration) for—
i. the transfer of all or any rights (including the granting of a licence) in respect of a patent; or
ii. the imparting of any information concerning the working of, or the use of, a patent; or
iii. the use of any patent; or
iv. the rendering of any services in connection with the activities referred above.

¾¾ It includes advance royalty, which is not refundable.

¾¾ It does not include -


a. any capital sum received for sale of patent, which is chargeable under the head Capital gains
b. consideration for sale of product manufactured with the use of patented process or of the patented
article for commercial use .

However, when such income is earned from a source outside India, then –
¾¾ Remittance: Such receipts should be brought into India by the assessee in convertible foreign exchange#
within 6 months from the end of the previous year or within such further period as the competent authority*
may allow in this behalf; and
¾¾ Certificate in Form 10H: The assessee must furnish a certificate in the Form 10H from the prescribed
authority along with his return of income
#
Convertible foreign exchange means foreign exchange which is for the time being treated by the Reserve
Bank of India as convertible foreign exchange for the purpose of the Foreign Exchange Regulation Act,
1973 and any rules made there under.
* Competent authority means the Reserve Bank of India or such other authority as is authorized under any
law for the time being in force for regulating payments and dealings in foreign exchange.

2. Certificate to be attached: Assessee must furnish a certificate in Form 10CCE duly signed by the prescribed
authority along with the return of income.
Prescribed authority here means Controller u/s 1(b) of the Patent Act, 1970

446 The Institute of Cost Accountants of India


Total Income and Tax Liability of Individuals & HUF

Quantum of deduction
Case Deduction
Minimum of the following -
Income earned in India ¾¾ 100% of such income; or
¾¾ ₹ 3,00,000
Minimum of the following -
Income earned outside India ¾¾ Income in respect of money brought into India in convertible foreign
exchange within prescribed time limit; or
¾¾ ₹ 3,00,000
Deduction should not exceed the royalty amount as per licence: Where a compulsory licence is granted in
respect of any patent under the Patents Act, 1970, the income by way of royalty for the purpose of allowing
deduction under this section shall not exceed the amount of royalty under the terms and conditions of a licence
settled by the Controller under that Act.

Notes
a. Revocation of patent
Situation: Where the patent is revoked by the Controller or the High Court and the assessee is subsequently
disqualified as patentee.
Treatment: Earlier deduction allowed under this section shall be deemed to be wrongly allowed and Assessing
Officer has the power to rectify the assessment-order within 4 years from the end of the previous year in which
such order of the High Court or the Controller was passed
b. Double deduction is not permissible
c. Return of income in time: Refer sec. 80-IA
d. No deduction shall be allowed if the deduction has not been claimed in the return of income.

3.3.31 Deduction in respect of interest on deposits in savings account [Sec. 80TTA]


Applicable to
An individual (other than senior citizen covered u/s 80TTB) or a Hindu Undivided Family
Conditions to be satisfied
Gross total income of an assessee includes any income by way of interest on deposits (not being time deposits8) in
a savings account with:
–– a banking company;
–– a co-operative society engaged in carrying on the business of banking (including a co-operative land mortgage
bank or a co-operative land development bank); or
–– a Post Office

8
“Time deposits” means the deposits repayable on expiry of fixed periods.

The Institute of Cost Accountants of India 447


Direct Taxation

Quantum of deduction
Minimum of the following
a. Interest on such deposits in saving account
b. ₹ 10,000
Other Points
 As per Notification No. 32/2011 dated 03-06-2011, interest on Post Office Saving Bank is exempt u/s 10(15(i)
to the extent of the interest of ₹ 3,500 (in case of single account) and ₹ 7,000 (in case of joint account)
 Where such income is derived from any deposit in a savings account held by, or on behalf of, a firm, an association
of persons or a body of individuals, no deduction shall be allowed under this section in respect of such income
in computing the total income of any partner of the firm or any member of the association or any individual of
the body.

3.3.32 Deduction in respect of interest on deposits in case of senior citizens [Sec. 80TTB]
Applicable to
Senior Citizen
Conditions to be satisfied
Gross total income includes any income by way of interest on deposits with:
–– A banking company applies;
–– A co-operative society engaged in carrying on the business of banking (including a co-operative land mortgage
bank or a co-operative land development bank); or
–– A Post Office
Quantum of deduction
Minimum of the following
a. Interest on such deposits
b. ₹ 50,000
Other Point
 Where such income is derived from any deposit held by, or on behalf of, a firm, an association of persons or a
body of individuals, no deduction shall be allowed.

3.3.33 Deduction u/s 80U in respect of person with disability


Applicable to
A resident individual (irrespective of citizenship of the individual)
Conditions to be satisfied
1. Assessee is a disable-individual: Assessee, at any time during the previous year, is certified by the medical
authority to be a person with disability.
2. Report: Assessee must furnish a copy of the certificate issued by the medical authority in the prescribed form
along with the return of income, in respect of the assessment year for which the deduction has been claimed.

448 The Institute of Cost Accountants of India


Total Income and Tax Liability of Individuals & HUF

Quantum of Deduction

Assessee is suffering from severe disability (i.e. disability to the extent of 80% or more) ₹ 1,25,000
Assessee is suffering from disability but not severe disability (i.e. disability to the extent of
₹ 75,000
40% or more but less than 80%)
Deduction under this section is irrespective of actual expenditure incurred i.e. deduction is statutory.
Disability, Person with disability and Person with severe disability: Refer Sec.80DD
Other points
Revision of medical certificate
Condition: Where the extent of disability requires reassessment after a period stipulated in the aforesaid certificate.
Treatment: After the expiry of previous certificate, deduction under this section shall be allowed only if a new
certificate is obtained from the medical authority and a copy thereof is furnished along with the return of income.
Note: Deduction u/s 80U is for disable-assessee whereas deduction u/s 80DD is for dependant disable-relative

Illustration 33:
Mr. X, is suffering from low-vision (certified as severe disability). He has following incomes details -

Net Salary ₹ 45,000


Short term capital gain ₹ 45,000
Long term capital gain ₹ 1,50,000
Mrs. X, suffering from leprosy (certified as 50% disable), is fully dependant on Mr. X. Compute his total income.
Solution :
Since Mr. X is suffering from severe disability, he can claim deduction u/s 80U of ₹ 1,25,000. Further, Mrs. X
is also a person with disability & dependant on Mr. X, therefore, Mr. X can also claim deduction u/s 80DD of ₹
75,000.
Computation of total income of Mr. X for the A.Y. 2024-25

Particulars Details Amount


Salaries 45,000
Capital gains
Short term capital gains 45,000
Long term capital gains 1,50,000 1,95,000
Gross Total Income 2,40,000
Less: Deduction u/s
80DD (Relative with disability) 75,000
80U (Assessee is a person with severe disability) 1,25,000 90,0001
Total Income 1,50,000
1.
Total deduction u/s 80C to 80U cannot exceed GTI excluding LTCG, STCG covered u/s 111A, and casual
income like winning from lotteries, etc.

The Institute of Cost Accountants of India 449


Direct Taxation

Illustration 34:
Preeti furnished following details -

Particulars Amount
Income from Business
Business A (Readymade garments) 50,000
Business C (Processing bio-degradable wastes) (started during previous year) 25,000
Long term capital gain 25,000
Short term capital gain 65,000
Saving Bank interest 2,000
Interest on deposits with IDBI 1,000
Interest paid on loan taken for deposits in IDBI 1,200
Interest on NSC (including last year interest ₹ 1,500) 4,200
Investment in NSC 500
LIC premium paid on the life of dependent blind mother (Sum assured ₹ 1,00,000) 12,000
Investment in LIC annuity plan 8,000
Donation to National Defence Fund 12,500
Compute her total income and tax liability.

Solution :
Computation of total income of Preeti for A.Y.2024-25

Particulars Amount Amount Amount


Profits and gains of business or profession
Business A 50,000
Business C 25,000 75,000
Capital gains
Long term capital gain 25,000
Short term capital gain 65,000 90,000
Income from other sources
Saving Bank interest 2,000
Interest on deposit with IDBI 1,000
Less: Interest paid on loan 1,200 (200)
Interest on NSC 4,200 6,000
Gross Total Income 1,71,000
Less: Deduction u/s
80C$ 3,200
80CCC (Deposit in LIC annuity plan) 8,000
80DD (Dependent blind mother) 75,000
80G (Donation to National Defence Fund) 12,500

450 The Institute of Cost Accountants of India


Total Income and Tax Liability of Individuals & HUF

80JJA (Business of processing bio-degradable wastes) 25,000


80TTA (Saving Bank Interest) 2,000 (1,25,700)
Total Income 45,300
Note: Tax liability is nil as total income is within exempted limit.
$
Calculation of deduction u/s 80C
Interest on NSC (excluding last year interest i.e. ₹ 4,200 – ₹ 1,500) ₹ 2,700
Investment in NSC ₹ 500
LIC premium paid on the life of mother (Premium paid for policy taken on life of parents) Nil
Total ₹ 3,200
Illustration 35:
Vikash has the following salary structure – ₹
Basic salary 8,000 p.m.
Dearness allowance 1,000 p.m.
Entertainment allowance 500 p.m.
Children education allowance (for 3 children) 1,000 p.m.
Contribution to RPF by his employer 10,000 p.a.
Own contribution to RPF 10,000 p.a.
His employer also provides rent-free furnished accommodation at Kolkata for which his employer paid ₹ 12,000 p.m.
(₹ 2,000 for furniture and ₹ 10,000 for accommodation). However, he is charged ₹ 1,000 p.m. for accommodation
and ₹ 1,000 p.m. for furniture.
He also supplied following details for computing total income –
a. Fixed Deposit interest ₹ 52,000
b. Rent received ₹ 1,50,000 from a house at Mumbai
c. Income from royalty on a book (artistic nature) @ 20% ₹ 3,20,000
d. He contributed ₹ 5,000 to approved scientific research association
e. He contributed ₹ 10,000 to a political party
f. Interest on Government securities ₹ 14,000
g. Investment in PPF ₹ 25,000
h. His date of birth is 7-9-1979.
Compute total income.
Solution :
Computation of total income of Vikash for the A.Y.2024-25
Particulars Amount Amount Amount Amount
Salaries
Basic 96,000
Allowances
Dearness Allowance 12,000
Entertainment Allowance 6,000
Children Education Allowance 12,000

The Institute of Cost Accountants of India 451


Direct Taxation

Particulars Amount Amount Amount Amount


Less: Exempted (₹ 100 * 12 * 2) 2,400 9,600 27,600
Perquisites
Rent free furnished accommodation
- For accommodation, lower of 15% of salary1 or rent paid 18,540
- For furniture 24,000
42,540
Less: Rent paid for accommodation and furniture 24,000 18,540 18,540
Employer’s contribution to RPF 10,000
Less: Exempted (12% of salary1) 12,960 Nil
1,42,140
Less: Standard Deduction u/s 16(ia) 50,000 92,140
Income from house property
Gross Annual Value (being rent received) 1,50,000
Less: Municipal tax paid Nil
Net Annual Value (NAV) 1,50,000
Less: Deduction u/s
24(a) Standard Deduction @ 30% of NAV 45,000
24(b) Interest on loan Nil 45,000 1,05,000
Income from other sources
Fixed Deposit interest 52,000
Royalty income 3,20,000
Interest on Government securities 14,000 3,86,000
Gross Total Income 5,83,140
Less: Deduction u/s
80C: Own contribution to RPF 10,000
Contribution to PPF 25,000 35,000
80GG (House rent paid) being minimum of -
a) ₹ 5,000z p.m. * 12 60,000
b) 25% of Adjusted GTI2 73,285
c) Rent paid over 10% of Adjusted GTI2
(₹ 12,000 – ₹ 29,314) Nil Nil
80GGA: Donation to scientific research association 5,000
80GGC: Contribution to political party 10,000
80QQB: Royalty income being minimum of -
a) 100% of income 3,20,000
b) 15% of sale value of the book [(3,20,000/20) * 15] 2,40,000
c) Statutory amount 3,00,000 2,40,000 2,90,000
Total Income 2,93,140

452 The Institute of Cost Accountants of India


Total Income and Tax Liability of Individuals & HUF

Note
Salary for the purpose of –

Particulars Accommodation RPF


Basic salary 96,000 96,000
Dearness allowance 12,000 12,000
Entertainment allowance 6,000 -
Children Education allowance 9,600 -
Total 1,23,600 1,08,000
2.
Adjusted GTI = Gross taxable income – Long term capital gain – STCG being covered by sec. 111A - All
deduction under 80’s other than sec. 80GG – Income u/s 115A, etc.
= ₹ 5,83,140 – ₹ (35,000 + 5,000 + 10,000 + 2,40,000) = ₹ 2,93,140
Quick MCQs:-
1. Deduction u/s 80C in respect of LIP, contribution to provident fund etc., is allowed to –
(a) any assessee
(b) an individual
(c) an individual or HUF
(d) an individual of HUF who is resident in India
2. Deduction u/s 80CCC is allowed to the extent of –
(a) D 20,000
(b) D 1, 50, 000
(c) D 40,000
(d) None of the above
3. Deduction u/s 80D is allowed if the premium is paid to –
(a) Life Insurance Corporation
(b) General Insurance Corporation or any other Insurer approved by the IRDA
(c) Life Insurance or General Insurance Corporation
(d) None of above.
4. Maximum deduction u/s 80GG shall be limited to –
(a) D 1,000 p.m
(b) D 5000 p.m
(c) D 3000 p.m
(d) None of the above.
5. Deduction u/s 80E in respect of repayment of loan taken for higher education shall be allowed to –
(a) an individual assessee only
(b) an individual who is resident in India

The Institute of Cost Accountants of India 453


Direct Taxation

(c) an individual or HUF


(d) an individual or HUF who is resident in India.

6. Deduction under Section 80-IB in respect of specified or other hotel shall be allowed to
(a) an Indian Company
(b) any assessee
(b) an Indian Company or a person other than Company resident in India.
(c) an Indian Company with a paid up capital of a t lest D 5,00,000

7. Deduction u/s 80JJAA in respect of employment of new workers shall be allowed to-
(a) any assessee to who Sec.44AB applies
(b) an Indian company
(c) an Indian Company or a person other than Company resident in India
(d) None of above

8. For an individual deriving income, investment in PPF for him and his Spouse for the Financial Year 2023-
2024 is D 85,000. Eligible amount for deduction U/s 80C is –
(a) D 85,000
(b) D 70,000
(c) D 1,00,000
(d) None of the above.

9. Where an individual has repaid in the 2nd year, D 20,000 towards principal and D 60,000 totally towards
eligible education loan from an approved bank, the deduction available u/s 80E is –
(a) Nil
(b) D 60,000
(c) D 40,000
(d) D 20,000

10. The maximum amount which can be donated in cash for claiming deduction under section 80G for the
P.Y. 2023-24 is –
(a) D 5,000
(b) D 10,000
(c) D 10,000
(d) D 2,000

11. Rebate in Tax u/s 87A, is applicable to


(a) Companies
(b) Individuals
(c) HUF
(d) Resident Individual

454 The Institute of Cost Accountants of India


Total Income and Tax Liability of Individuals & HUF

Taxation of Individual (including AMT but


3.4
excluding Non-resident) & HUF
3.4.1 Computation of Total Income & Tax Liablity of Individual
Computation of Total Income requires the complete knowledge of all heads of income and deduction under
Chapter VIA.
Computation of Total Income of……….. for the A.Y. 2024-25

Particulars Amount
Salaries ***
Income from house property ***
Profits and gains of business or profession ***
Capital gains ***
Income from other sources ***
Gross Total Income ****
Less: Deductions u/s 80C to 80U (subject to Max. of GTI before casual income, STCG covered u/s ***
111A & LTCG)
Total Income / Taxable Income (Rounded off u/s 288A) ****

Computation of Tax Liability of A.Y. 2024-25

Particulars Rate Details Amount


Total Income other than STCG u/s 111A, LTCG & casual income ***
[as per regime]
Add: Tax on casual income 30% ***
Add: Tax on LTCG 20%/10%/0% ***
Add: Tax on STCG covered u/s 111A 15% ***
Total tax liability before rebate and cess PQR
Less: Rebate u/s 87A **
STY
Add: Surcharge **
ABC
Add: Health and Education cess @ 4% **
Tax liability after cess ***
Less: Relief u/s 86, 89, 90 & 91 **

The Institute of Cost Accountants of India 455


Direct Taxation

Particulars Rate Details Amount


Tax liability ***
Less: Advance Tax ***
Less: Tax Deducted / Collected at Source ***
Less: Any other prepaid tax *** ***
Tax Payable (if + ve) or Refundable (if – ve) (Rounded of u/s 288B) ***

Illustration 36:
Determine the tax liability of Mr. Ratan, who is a handicapped, for the A.Y.2024-25:

Particulars Amount
Net salary 2,85,000
Annual value of let-out house property 60,000
Lottery income 50,000
Taxable business & profession income 50,000
He paid LIC premium 10,000
Mr. Sunil deposited in LIC annuity plan ₹ 12,000 & paid medical insurance premium ₹ 5,000.

Solution :
Computation of Total Income of Mr. Ratan for the A.Y. 2024-25
Particulars Details Amount
Salaries
Net salary 2,85,000
Income from house property
Annual value of let-out house property 60,000
Less: Standard deduction u/s 24(a) 18,000 42,000
Profits & gains of business or profession 50,000
Income from other sources
Lottery income 50,000
Gross Total Income 4,27,000
Less: Deduction u/s
80C (LIC Premium) 10,000
80CCC (Deposit in LIC annuity plan) 12,000
80D (Medical insurance premium paid) 5,000
80U (Assessee is handicapped) 75,000 1,02,000
Total Income 3,25,000
Computation of Tax liability of Mr. Ratan for the A.Y. 2024-25

Particulars Rate On Details Amount


Casual income (Note) ₹ 50,000 30% 15,000

456 The Institute of Cost Accountants of India


Total Income and Tax Liability of Individuals & HUF

Particulars Rate On Details Amount


Other Income (Note)
Upto ₹ 2,50,000 - -
From ₹ 2,50,001 to ₹ 2,75,000 5% 25,000 1,250 1,250
Tax Liability 16,250
Less: Rebate u/s 87A 12,500
3,750
Add: Health & Education cess 4% of ₹ 3,750 150
Tax and cess payable 3,900

Note: Taxable income can be divided as follows:


–– Casual income ₹ 50,000 on which tax @ 30% is applicable, and
–– Income other than casual income ₹ 2,75,000 (being ₹ 3,25,000 – ₹ 50,000) on which slab is applicable.

Illustration 37:
Compute taxable income and tax liability of Sri Dev from the following information:
a. He is working with X Ltd. for a basic of ₹ 24,000 p.m. and dearness allowance ₹ 2,000 p.m. Bonus ₹ 50,000
b. He and his employer contribute to RPF @ 6% of salary.
c. His income from business & profession is ₹ 52,000
d. During the year, interest on fixed deposit ₹ 5,000
e. LIC premium paid ₹ 4,000

Solution :
Computation of Total Income of Sri Dev for the A.Y. 2024-25

Particulars Details Amount


Salaries
Basic 2,88,000
Bonus 50,000
Dearness allowance 24,000
Employer’s contribution to RPF (as less than 12% of salary) Nil
Gross Salary 3,62,000
Less: Standard Deduction u/s 16(ia) 50,000 3,12,000
Profits & gains of business or profession 52,000
Income from other sources
Interest on Fixed deposit 5,000
Gross Total Income 3,69,000
Less: Deduction u/s 80C1 22,720
Total Income 3,46,280

The Institute of Cost Accountants of India 457


Direct Taxation

Computation of Tax Liability for the A.Y. 2024-25

Income Rate On Tax


Upto ₹ 2,50,000 - -
From ₹ 2,50,001 to ₹ 3,46,280 5% 96,280 4,814
Tax liability 4,814
Less: Rebate u/s 87A 4,814
Nil
Add: Health and Education cess Nil
Tax and cess payable Nil
1. Computation of deduction u/s 80C

Particulars Amount
LIC premium paid 4,000
Employees contribution to RPF (6% of basic and DA) 18,720
Total 22,720

Illustration 38:
Anil (aged 67 years) is an employee of a Government organisation at Burdwan. During the year ending 31.3.2024,
he received the following income from his employer:

Salary 20,000 p.m.
Dearness allowance 4,000 p.m.
Entertainment allowance 6,500 p.m.
Annual bonus 20,000
Rent-free unfurnished accommodation of which licence fee is ₹ 30,000 whereas fair rental value is ₹ 1,00,000.
Find his taxable income and tax liability for the assessment year 2024-25 on the basis of the following further
information:
a. His own contribution to a Provident Fund is ₹ 30,000 and his employer’s contributed the same amount.
b. He pays life insurance premium ₹ 30,000 p.a.
Solution :
Computation of Total Income of Anil for the A.Y. 2024-25
Particulars Details Amount Amount
Salaries
Basic salary 2,40,000
Bonus 20,000
Allowances:
Dearness allowance 48,000
Entertainment allowance 78,000 1,26,000
Perquisite u/s 17(2):
Rent free accommodation Licence fee 30,000
Employer’s contribution to PF Contribution to SPF Nil
Gross Taxable Salary 4,16,000

458 The Institute of Cost Accountants of India


Total Income and Tax Liability of Individuals & HUF

Particulars Details Amount Amount


Less: Deduction u/s
16(ia) Standard Deduction 50,000
16(ii) Entertainment allowance (Least of the followings):

- 20% of basic 48,000


- Maximum limit 5,000
He is a Government
- Actual amount 78,000 employee
5,000 45,000
Gross Total Income 3,61,000
Less: Deduction u/s
80C Note 1 60,000
Total Income 3,01,000
Computation of Tax Liability of Mr. Anil for the A.Y.2024-25

Income Rate On Tax


Upto ₹ 3,00,000 - -
From ₹ 3,00,001 to ₹ 3,01,000 5% 50 50
Tax liability before cess 50
Less: Rebate u/s 87A 50
Tax and cess payable Nil
1. Computation of Deduction u/s 80C

Particulars Amount
Insurance premium 30,000
Employee’s own contribution to SPF 30,000
Total 60,000

Illustration 39:
Mrs. Ria (age 46 years), a professor in a college furnishes the following particulars of her income for the previous
year 2023–24:

a. Income from salary 3,51,000
b. Income from House Property
i. Rent Received 24,000
ii. Municipal Tax 2,000
c. Income from other sources
iii. Bank interest on fixed deposit 12,000
iv. Dividend from Indian Company 10,000
v. Examiner’s remuneration from Calcutta University 5,700
c. Deposited in P.P.F. 40,000
d. Own Contribution to R.P.F. 12,000
e. Paid premium on insurance on own health 16,000
f. Donated ₹ 10,000 to National Relief Fund and ₹ 15,000 to a trust registered u/s 80G(5)

The Institute of Cost Accountants of India 459


Direct Taxation

From the above information, compute total income and tax liability of Mrs. Ria for the assessment year 2024–25.
Solution :
Computation of total income of Mrs. Ria for the A.Y. 2024-25

Particulars Amount Amount Amount


Salaries 3,51,000
Income from house property
Gross Annual Value being Rent Received 24,000
Less: Municipal Tax paid 2,000
Net Annual Value (NAV) 22,000
Less: Standard Deduction u/s 24(a) [30% of NAV] 6,600 15,400
Income from other sources
Bank Interest on fixed deposit 12,000
Dividend from Indian companies 10,000
Examiner’s Remuneration 5,700 27,700
Gross Total Income 3,94,100
Less: Deductions
- u/s 80C
Own Contribution to RPF 12,000
Deposit in PPF 40,000 52,000
- u/s 80D (Mediclaim) 16,000
- u/s 80G
Donation to National Defence Fund 10,000
Donation to Trust (50%#) 7,500 85,500
Total Income 3,08,600
Tax on above [(₹ 2,50,000 x 0%) + (58,600 x 5%)] 2,930
Less: Rebate u/s 87A 2,930
Tax Payable Nil
# As payment is within limit

Illustration 40:
Sri Chandan Saha furnished the following information for the previous year 2023-24:


Income from salary (net) 6,00,000
Income from house property 1,40,000
Short-term capital loss 15,000
Long-term capital gain 40,000
Interest on bank fixed deposit 25,000
Dividend from Indian Company 10,000
He made the following payments:
i. Life insurance premium on own life ₹ 25,000 (sum assured ₹ 2,00,000 taken on 15.10.2023)

460 The Institute of Cost Accountants of India


Total Income and Tax Liability of Individuals & HUF

ii. Deposit in Public Provident Fund ₹ 1,40,000


iii. Own contribution to Recognised Provident Fund ₹ 20,000
iv. Medical insurance premium on own health ₹ 10,000 and on the health of spouse ₹ 8,000 paid by cheque.
Compute his income and tax payable for the assessment year 2024-25.
Solution :
Computation of total income of Sri Chandan Saha for the A.Y. 2024-25

Particulars Amount Amount Amount


Income from salary 6,00,000
Income from house property 1,40,000
Capital Gains
Long term capital gain 40,000
Less: Short term capital loss 15,000 25,000
Income from other sources
Interest on bank fixed deposit 25,000
Dividend from Indian companies 10,000 35,000
Gross Total Income 8,00,000
Less: Deductions
- u/s 80C
LIC (Max. upto 10% of sum assured) 20,000
Deposit in PPF 1,40,000
Own contribution to RPF 20,000
Max. Limit 1,80,000 1,50,000
- u/s 80D (Mediclaim) 18,000 1,68,000
Total Income 6,32,000
Computation of Tax liability of Sri Chandan Saha for the A.Y. 2024-25

Particulars Rate On Details Amount


LTCG of ₹ 25,000 20% 5,000
Other Income (Note)
Upto ₹ 2,50,000 - -
From ₹ 2,50,001 to ₹ 5,00,000 5% 2,50,000 12,500
From ₹ 5,00,001 to ₹ 6,07,000 20% 1,07,000 21,400 33,900
Tax Liability 38,900
Less: Rebate u/s 87A -
38,900
Add: Health and Education cess 4% of ₹ 38,900 1,556
Tax and cess payable 40,456
Rounded off u/s 288B 40,460

The Institute of Cost Accountants of India 461


Direct Taxation

3.4.2 Alternate Minimum Tax (AMT) [Sec. 115JC]Amended


Applicable to
All assessee (other than company) who has claimed any deduction under:
 Sec. 80H to Sec. 80RRB (other than sec. 80P); or
 Sec.10AA
 Sec.35AD
Exception:
The provisions shall not apply to an individual or a HUF or an AOP or a BOI, whether incorporated or not, or an
artificial juridical person, if the adjusted total income of such person does not exceed ₹ 20 lakh.
Taxpoint:
 The exception is not applicable in case of Firm and Limited Liability Partnership. That means, AMT is
applicable on LLP / Firm (claiming deduction under aforesaid section) even though adjusted total income does
not exceed ₹ 20 lakh.
 The provisions of this section shall not apply to a person who has exercised the option referred to in sec.
115BAC or 115BAD [alternative tax regime]
 The provision is not applicable to the specified fund referred to in clause (c) of the Explanation to sec. 10(4D).
Scheme of Alternate Minimum Tax (AMT)

Step 1 Compute regular income tax liability (before Cess) of the assessee covered A ****
under these provisions
Step 2 Compute Adjusted Total income of the assessee i.e. *****
Total income of the assessee **** B
Add:
 Deduction claimed u/s 80H to sec. 80RRB (other than sec. 80P) *** C
 Deduction claimed u/s 35AD less Depreciation u/s 32 *** D
 Deduction u/s 10AA *** E
Adjusted Total Income **** F
Note:
i. If ‘C’, ‘D’ and ‘E’ is zero, then these provisions are not applicable to any assessee.
ii. if ‘F’ does not exceed Rs.20 lakh, then these provisions are not applicable in case of an
Individual / HUF / AOP / BOI / Artificial juridical person. However, the provision is
applicable on LLP / Firm.
Step 3 Compute Alternate Minimum Tax (AMT) [Being 18.5% of Adjusted Total G=F* ****
Income] 18.5%
Step 4 Income Tax liability Higher of A ****
&G
Add: Health & Education Cess **
Tax liability after Cess ****

462 The Institute of Cost Accountants of India


Total Income and Tax Liability of Individuals & HUF

Impact where AMT is applicable i.e., case where value of Step 3 is higher than value of Step 1
a. Adjusted total income (as computed in step 2) shall be deemed as total income of the assessee.
b. Tax liability of the assessee shall be 18.5% (+ surcharge + cess) of adjusted total income of the assessee.
Exceptions
¾¾ In case of a unit located in an International Financial Services Centre and derives its income solely in
convertible foreign exchange, AMT shall be computed considering 9% (instead of 18.5%) of adjusted
total income.
¾¾ In case of co-operative society, AMT shall be computed considering 15% (instead of 18.5%) of adjusted
total income.
Taxpoint: Rate of AMT are as under:

Assessee Rate of AMT


Unit located in an International Financial Services Centre and derives its income solely in
9%
convertible foreign exchange
Co-operative society 15%
Other 18.5%
a. A report in Form 29C from a chartered accountant is required to be upload one month prior to the due date of
furnishing of return of income u/s 139(1).
b. All other provisions of the Act, like advance tax, interest, etc. is applicable to such assessee.

Provision Illustrated
Compute tax of the following assessee:

Particulars Mr. W Mr. X Mr. Y A LLP B LLP


Gross Total Income being Business Income 15,00,000 25,00,000 27,00,000 32,00,000 8,00,000
Deduction u/s 80C 1,00,000 1,00,000 1,00,000 Nil Nil
Deduction u/s G 25,000 1,00,000 Nil 1,00,000 1,00,000
Deduction u/s 80IE 7,75,000 Nil 8,00,000 Nil 2,00,000
Total Income 6,00,000 23,00,000 18,00,000 31,00,000 5,00,000
Regular Tax 32,500 5,02,500 3,52,500 9,30,000 1,50,000
Adjusted Total Income 13,75,000 23,00,000 26,00,000 31,00,000 7,00,000
Whether sec. 115JC is applicable or not No1 No2 Yes No2 Yes
1.
As adjusted total income does not exceed `20
lakh
2.
As no deduction is claimed u/s 80H to 80RRB
(other than sec. 80P) or u/s 10AA
Alternate Minimum Tax (AMT) u/s 115JC NA NA 4,81,000 NA 1,29,500
[18.5% of adjusted total income]
Tax (Higher of Regular Tax and AMT) 32,500 5,02,500 4,81,000 9,30,000 1,50,000
Add: Health & Education Cess 1,300 20,100 19,240 37,200 6,000
Tax and Cess Liability (Rounded off) 33,800 5,22,600 5,00,240 9,67,200 1,56,000

The Institute of Cost Accountants of India 463


Direct Taxation

Tax credit for alternate minimum tax [Sec. 115JD]


 The excess of alternate minimum tax paid over the regular income-tax payable of that year shall be allowed
as tax credit.
 Mathematically, tax credit available = Tax paid u/s 115JC – Regular Tax payable
 However, no interest shall be payable on the tax credit allowed.
 The amount of tax credit determined shall be carried forward and set off but such carry forward shall not be
allowed beyond the 15th assessment year immediately succeeding the assessment year in which tax credit
becomes allowable.
 The tax credit shall be allowed set-off in a year when regular tax becomes payable by the assessee.
 Set off in respect of brought forward tax credit shall be allowed for any assessment year to the extent of the
difference between the alternate minimum tax payable u/s 115JC for that assessment year and the balance of
the tax credit, if any, shall be carried forward. In other words, after setting off of AMT credit, tax liability of
the year cannot be less than AMT for that year.
 The amount of tax credit in respect of any income-tax paid in any country or specified territory outside India
u/s 90 or 90A or 91, allowed against the alternate minimum tax payable, exceeds the amount of the tax credit
admissible against the regular income-tax payable by the assessee, then, while computing the amount of credit
u/s 115JD, such excess amount shall be ignored.
 If the amount of regular income-tax or the AMT is reduced or increased as a result of any order passed under
this Act, the amount of tax credit allowed under this section shall also be varied accordingly.
Examples:
(a) Tax liability u/s 115JC (AMT) 500
Regular Tax liability 1,000
Difference 500
₹ 500 cannot be treated as credit because liability u/s 115JC (AMT) is not greater than regular tax liability.
(b) Year 1
Liability u/s 115JC (AMT) 2,000
Regular Tax Liability 1,000
1,000 It can be carried forward
Year 2
Liability u/s 115JC (AMT) 2,500
Regular Tax Liability 5,000
Difference 2,500
Now regular liability is more than liability u/s 115JC (AMT), the credit carried forward can be set off to the
extent of the difference i.e., ₹ 2,500
In year 2, tax payable shall be as under:
Regular Tax Liability 5,000
Less: Set off of AMT credit 1,000
Tax payable (before surcharge and cess) 4,000
In the above example, if the credit carried forward was ₹ 3,500, then tax payable in year 2 would be calculated
as under:
Regular Tax Liability 5,000
Less: Set off of AMT credit 2,500
Tax payable (Credit available ₹ 3,500 but restricted to ₹ 2,500) 2,500
Here ₹ 1,000 (being balance credit left i.e., ₹ 3,500 – ₹ 2,500) is carried forward to the next year.
In other words, the excess of regular tax over AMT u/s 115JC is the amount of maximum set off permissible.

464 The Institute of Cost Accountants of India


Total Income and Tax Liability of Individuals & HUF

3.4.3 Alternative Tax Regime for Individual / HUF [Sec. 115BAC(IA)]


Applicable to
Individual / HUF/AOP/BOI/AJP
Conditions
a. Total income of the assessee shall be computed:
i. Without any exemption or deduction under following provisions

Deduction not available under


Details
following section
10(5) Leave Travel Concession
10(13A) House Rent Allowance
10(14) Special Allowances
Exception: Few prescribed allowances
10(17) Allowance to MPs/MLAs
10(32) Exemption in respect of clubbing of minor child
10AA Special Economic Zone
16 Deduction under the head Salaries - Deduction for Entertainment allowance
and Deduction for professional tax. However, standard deduction is allowed
24(b) in respect of self occupied Interest on borrowed capital
property Taxpoint: Deduction is available in respect of other properties like let out,
deemed to be let out
32(1)(iia) Additional Depreciation
32AD Investment Allowance
33AB Tea / Coffee / Rubber Development Allowance
33ABA Site Restoration Fund
35(2AA) or 35(1)(ii) / (iia) / (iii) Scientific Research through outside institution
35AD Capital Expenditure in respect of specified business
35CCC Agriculture Extension Project
Deduction under chapter VIA Exception: Deduction in respect of contribution to NPS u/s 80CCD(2);
contribution to ACF u/s 80CCH; deduction u/s 80JJAA and deduction u/s
80LA is avaialble
ii. without set off of any loss:
a. carried forward or depreciation from any earlier assessment year, if such loss or depreciation is
attributable to any of the deductions referred above;
b. under the head “Income from house property” with any other head of income;
iii. by claiming the depreciation, if any, u/s 32 [except additional depreciation], determined in prescribed manner;
and
iv. without any exemption or deduction for allowances or perquisite, by whatever name called, provided under
any other law for the time being in force.
b. The assessee is required to exercise the option (in prescribed manner) to avail the benefit of this section.

The Institute of Cost Accountants of India 465


Direct Taxation

Rate of Tax
Under this tax regime, income tax shall be computed at the option of the assessee considering the following rate:

Total income Rate of tax


Upto ₹ 3,00,000 Nil
From ₹ 3,00,001 to ₹ 6,00,000 5%
From ₹ 6,00,001 to ₹ 9,00,000 10%
From ₹ 9,00,001 to ₹ 12,00,000 15%
From ₹ 12,00,001 to ₹ 15,00,000 20%
Above ₹ 15,00,000 30%
Taxpoint

 If a person opts for this regime, ₹ 3,00,000 shall be considered as basic exemption limit irrespective of his
age. In other words, for all category of individual i.e, senior citizen, super senior citizen and others, basic
exemption limit is ₹ 3,00,000

 Rebate u/s 87A is available. Threshold limit ₹ 7,00,000 with marginal rebate.

 Computed tax is further increased by applicable surcharge (not exceeding 25%), if any, and health and
education cess

 If any income is taxable at special rate u/s 110 to sec. 115BBG [except sec. 115BAC(IA)], such income shall
be taxable at that special rate of tax.

Other Points

 Full effect of loss and depreciation: The loss and depreciation referred above shall be deemed to have been
given full effect to and no further deduction for such loss or depreciation shall be allowed for any subsequent
year. Where there is a depreciation allowance in respect of a block of assets which has not been given full
effect to prior to the assessment year 2021-22, corresponding adjustment shall be made to the written down
value of such block of assets as on 01-04-2020 in the prescribed manner (if the option is exercised for a
previous year relevant to the assessment year 2021-22).

 Exercise of option: The provision of this section shall not apply unless option is exercised in the prescribed
manner by the person:

Where the person has income Within the due date specified u/s 139(1) for furnishing the returns of
from business or profession income for any previous year relevant to the assessment year and such
option once exercised shall apply to subsequent assessment years
Where the person not having Alongwith the return of income to be furnished u/s 139(1) for a previous
aforesaid income year relevant to the assessment year

 Withdrawal of option: In case person having income from business or profession, option once exercised for
any previous year can be withdrawn only once for a previous year other than the year in which it was exercised
and thereafter, the person shall never be eligible to exercise option. However, if such person ceases to have
any income from business or profession in which case, he may exercise the option for that assessment year.

466 The Institute of Cost Accountants of India


Total Income and Tax Liability of Individuals & HUF

Where the person fails to satisfy the conditions in any previous year, the option shall become invalid in respect
of the assessment year relevant to that previous year and other provisions of this Act shall apply, as if the option
had not been exercised for the assessment year relevant to that previous year.
Further where the option was exercised by a person having income from business or profession, in the event
of failure to satisfy the conditions, it shall become invalid for subsequent assessment years also and other
provisions of this Act shall apply for those years accordingly.
 Alternate Minimum Tax: In case, the person has opted for this scheme, the provision of alternate minimum
tax (AMT) u/s 115JC is not applicable. Consequently, any credit of AMT cannot be adjusted against tax
liability computed u/s 115BAC(IA).

Example 8:
Akhil, 35 years, has provided following details relating to his income for the previous year 2022-23:
a. Income from business ₹ 15,00,000
b. Income from saving bank interest ₹ 12,000
c. Interest on PPF ₹ 36,000
d. Investment in PPF ₹ 1,50,000
You are requested to compute his tax liability and advise him whether he should opt for alternative tax regime u/s
115BAC(IA).
Solution :
Computation of Income and tax liability of Mr. Akhil for A.Y. 2024-25

Particulars Regular Tax Regime Alternative Tax Regime


Business Income 15,00,000 15,00,000
Interest on saving bank deposit 12,000 12,000
Interest on PPF Exempt Exempt
Gross Total Income 15,12,000 15,12,000
Less: Deduction
U/s 80C (PPF Contribution) 1,50,000 NA
U/s 80TTA (Interest on Saving Bank Interest) 10,000 NA
Total Income 13,52,000 15,12,000
Tax on above 2,18,100 1,53,600
Less: Rebate u/s 87A (As income exceeds ₹ 5,00,000) NA NA
Tax after rebate 2,18,100 1,53,600
Add: Surcharge Nil Nil
Tax and surcharge 2,18,100 1,53,600
Add: Health & Education cess 8,724 6,144
Tax liability (Rounded off u/s 288B) 2,26,820 1,59,740
In the instant case, tax liability under alternative tax regime u/s 115BAC(IA) is lower, hence it is advisable to opt
for provision of sec. 115BAC(IA).

The Institute of Cost Accountants of India 467


Direct Taxation

HINDU UNDIVIDED FAMILY

3.4.4 Meaning of HUF


The term Hindu Undivided Family (or Joint Hindu Family) is not defined in the Income-tax Act. The term HUF has
the same meaning as in Hindu law. A Hindu Undivided Family (HUF) consists of all persons lineally descended
from a common ancestor including their wives and unmarried daughters.
Taxpoint: Only those undivided families are covered here, to which Hindu law applies.
Notes
1. An HUF is not the creation of a contract. Its membership arises from status.
2. Jain families are also treated as Hindu undivided family for the purpose of Income-tax Act. However, Muslim
undivided family cannot be treated as HUF.
3. Once a family is assessed as Hindu undivided family, it will continue to be assessed as such till its partition.

3.4.5 Essential requirements of an HUF


An assessee shall be assessed as HUF if the following conditions are satisfied:
1. Joint family property: There should be a joint family property, which consists of either ancestral property or
any property acquired with the aid of ancestral property.
Ancestral property: It is a property, which a man inherits from any of the three immediate male ancestors
i.e. -
¾¾ His father;
¾¾ His grand father; and
¾¾ His great grand father
Notes
a. Property inherited from any other person (e.g. father in law, uncle, etc.) is not an ancestral property.
b. In the following cases income of ancestral property is taxable as income of HUF -
 Family of widow-mother and sons (minor or major)
 Family of husband and wife having no child.
 Family of two or more brothers.
 Family of uncle and nephew.
 Family of mother, son and son’s wife.
 Family of a male and his late brother’s wife.
2. Coparcener: There should be a coparcener. Coparcener means any person who satisfies two tests:
¾¾ He/she acquires by birth an interest in joint family property; and
¾¾ He/she has right to enforce partition.
Member of HUF vs Coparcener
i. Coparcener must be a member of the family but a member may or may not be the coparcener of the
family.

468 The Institute of Cost Accountants of India


Total Income and Tax Liability of Individuals & HUF

Taxpoint: All coparcener must be the member of the family but all members need not be coparcener of
the family.
ii. Coparcener acquires interest in the property of the HUF by birth. A person may acquire membership in
HUF otherwise than by birth.
iii. Member of the family who is not a coparcener of the family has no right to claim partition.
Taxpoint: A child in the womb of his mother is entitled to share of HUF property, on partition.

3.4.6 School of Hindu Law


There are two schools of Hindu law -
1. Dayabhaga School of Hindu Law; and
2. Mitakshara School of Hindu Law

Law Dayabhaga School of Hindu Law Mitakshara School of Hindu Law


Applicable to Rules of Dayabhaga school prevails in the Rules of Mitakshara school prevail over rest of
State of West Bengal and Assam India.
Features
Interest in As per this thought, a son acquires interest in As per this thought, a son acquires interest in
property ancestral property only after the death of his ancestral property as soon as he is born.
father. Taxpoint: A son acquires an equal interest in
Taxpoint: A son does not acquire interest in the ancestral property with his father mere by
the ancestral property mere by birth. birth.
Right to claim Since son has no interest in ancestral property, Since son acquires interest in ancestral property
partition he cannot claim partition till his father is alive. by his birth, therefore he can claim partition at
Taxpoint: Father enjoys an absolute right to any time.
dispose of the property of the family.
Fluctuation in The coparcenary interest does not fluctuate The coparcenary interest fluctuates on every
coparcenary on every birth or death of a male member. It birth or death of a male member.
interest fluctuates only on death of the father.
Note: After Hindu Succession (Amendment) Act 2005, the daughter (in the same manner as the son) of a
coparcener shall by birth become a coparcener in her own right. The female heir can demand the partition of
coparcenary property.

The Institute of Cost Accountants of India 469


Direct Taxation

3.4.7 Computation of total income and tax liability of HUF


There is no specific provision in Income-tax Act for computation of total income of HUF. Total income and tax
liability of HUF shall be computed in same manner as in case of an individual.
Taxpoint:
Tax rate, health & education cess : As in case of an individual
Residential Status : Refer chapter “Residential Status”
Computation of income under various : As usual, however, HUF cannot have any income under the head
heads ‘Salaries’.
Clubbing of income : As usual.
Sec. 64(1) & (1A) are not applicable in case of an HUF as it is
specifically applicable to individual. Specially refer sec. 64(2).
Setoff & Carry forward of losses : As usual
Deductions : Refer chapter “Deductions & Relief”
Return and Assessment : As usual
Advance tax : As in case of an individual
Alternative Tax Regime u/s 115BAC : Available
Following points shall be considered –
1. Remuneration to member of HUF due to investment of HUF fund: Where joint fund is invested in a
company or a firm, fees or remuneration received by any member of HUF as a director or partner from such
company or firm by virtue of such investment shall be treated as income of the HUF. On the other hand, where
such remuneration or fees is received by virtue of service rendered by such member (in his personal capacity)
then such amount shall be taxable in hands of such member
2. Income from ancestral property may not be taxed in the hands of HUF in certain cases: As per Dayabhaga
school, son has no right in the property during the life time of father, hence income arising from ancestral
property to father, having no brother as coparcener, shall be treated as income of father and not of HUF.
3. Remuneration to Karta: Any genuine (not excessive) remuneration paid to the Karta for conducting business
of the HUF is allowed expenditure in the hands of the HUF provided such remuneration is paid under a
bonafide agreement and is in the interest of the family business.
4. Personal income of the members: Income of the member of HUF acquired in his personal capacity shall not
be taxable in the hands of HUF.
Taxpoint: ‘Stridhan’ is an absolute property of a woman, income therefrom is not taxable in the hands of HUF.
5. Income from impartible estate: Though the impartible estate belongs to the family, income arising therefrom
is taxable in the hands of the holder of the ‘estate’ and not in the hands of the HUF.

3.4.8 Partition of HUF [Sec. 171]


Partition means—
Case Partition
A physical division of the property.
Where the property admits of a physical Taxpoint: Mere physical division of the income without a physical
division division of the property producing the income shall not be deemed to
be a partition.
Such division as the property admits of.
Where the property does not admit of a
physical division Taxpoint: Mere severance of status shall not be deemed to be a
partition.

470 The Institute of Cost Accountants of India


Total Income and Tax Liability of Individuals & HUF

Partial Partition means a partition, which is partial as regards the persons constituting the Hindu undivided
family, or the properties belonging to the Hindu undivided family, or both.
Person entitled to share on partition
Following persons can claim share on partition:

Case Persons who can claim share on partition


Any Coparceners
Any A child in the womb of his mother at the time of partition
Partition between sons after the death of father Mother – gets an equal share to that of son
Wife - gets an equal share to that of a son (apart from that of
Partition between father and sons
husband)

3.4.9 Assessment after partition of a Hindu undivided family


An HUF assessed as such shall be deemed (for the purposes of this Act) to continue to be an HUF, except where a
finding of partition has been given under this section [Sec. 171(1)]
1. Inquiry by Assessing Officer: Where, at the time of making assessment, it is claimed by or on behalf of any
member of an HUF that a partition (whether total or partial), has taken place among the members of such
family, the Assessing Officer shall make an inquiry after giving notice of the inquiry to all the members of the
family [Sec. 171(2)]
2. Record of findings: On the completion of the inquiry, the Assessing Officer shall record his findings as to
whether there has been a total or partial partition of the HUF and if there has been such a partition, the date on
which it has taken place [Sec. 171(3)]
3. Assessment upto the period of partition: Where a finding of total or partial partition has been recorded by
the Assessing Officer under this section, and the partition has taken place during the previous year —
¾¾ the total income of the HUF in respect of the period up to the date of partition shall be assessed as if no
partition had taken place; and
¾¾ each member shall be jointly and severally# liable for tax on the income so assessed. [notwithstanding
anything contained in sec.10(2)] [Sec. 171(4)]
#
The liability of any member or group of members shall be computed according to the portion of the joint
family property allotted to him or it at the partition [Sec. 171(7)]
4. Partition after the expiry of previous year: Where the finding of total or partial partition has been recorded
by the Assessing Officer and the partition took place after the expiry of the previous year, the total income of
the previous year of the joint family shall be assessed as if no partition had taken place. [Sec. 171(5)]
5. Notice of partition after assessment: If the Assessing Officer finds after completion of the assessment of
HUF, that the family has already effected a partition (whether total or partial) the Assessing Officer shall
proceed to recover tax from every person who was a member of the family before the partition, and every such
person shall be jointly and severally liable for tax on the income so assessed. [Sec. 171(6)]
6. Collection of penalty, interest, fine, etc.: The provisions of this section shall apply in relation to the levy and
collection of penalty, interest, fine or other sum (if any) in respect of any period up to the date of the partition,
whether total or partial, as they apply in relation to the levy and collection of tax. [Sec.171(8)]

The Institute of Cost Accountants of India 471


Direct Taxation

Treatment in case of partial partition taken place after 31-12-1978 [Sec. 171(9)]
Where a partial partition has taken place among the members of an HUF after 31-12-1978, then -
 no claim that such partial partition has taken place shall be inquired into u/s 171(2)
 no finding regarding partition shall be recorded u/s 171(3).
 such family shall continue to be liable to be assessed under this Act as if no such partial partition had taken
place;
 each member or group of members of such family immediately before such partial partition and the family
shall be jointly and severally1 liable for any tax, penalty, interest, fine or other sum payable under this Act by
the family.
1
. Liability of any member or group of members aforesaid shall be computed according to the portion of the
joint family property allotted to him or it at such partial partition.
Taxpoint: Even though such tax, penalty etc. relates to the period, before or after such partial partition.

472 The Institute of Cost Accountants of India


Total Income and Tax Liability of Individuals & HUF

Advance Tax 3.5

G
enerally, tax on the income earned in the previous year is paid in the respective assessment year, but in
certain cases, an assessee may be required to pay tax during the previous year itself, as Advance tax.
The scheme of advance tax is based on the concept “Pay as you earn”. Under this scheme assessee needs
to estimate its income and tax liability of the previous year and pay tax on basis of such estimation in
the previous year itself. For instance, income earned during the previous year 2021-22 is normally taxable in the
assessment year 2022-23, however under the scheme of Advance tax, assessee is required to pay tax on estimated
income of previous year 2021-22 in the previous year itself.
Applicable to
All assessee irrespective of his residential status and citizenship
Scheme of Advance tax [Sec.208]
Where the advance tax liability# of the assessee is ₹ 10,000 or more, the assessee should pay such tax in the
previous year itself within the due date$.
# Advance tax liability [Sec. 209]

Particulars Amount
Estimated Gross Total Income ****
Less: Deduction under chapter VIA ****
Estimated Total Income ****

Gross tax liability on Estimated Total Income ****


Less: Rebate u/s 87A **
Tax liability after Rebate ****
Add: Surcharge (if applicable) ****
Tax and surcharge payable ****
Add: Health & Education cess ****
Tax liability after cess ****
Less: Tax deducted or collected at source / other Rebate & Relief ****
Advance tax liability ****

The Institute of Cost Accountants of India 473


Direct Taxation

$
Due date for payment of advance tax [Sec. 211]

Due date of installment


Assessee Minimum amount payable
(of previous year)

An eligible assessee in
respect of an eligible
business referred to in On or before March 15 100% of advance tax liability

sec. 44AD or 44ADA

On or before June 15 Upto 15% of advance tax liability


On or before September 15 Upto 45% of advance tax liability
Other Assessee
On or before December 15 Upto 75% of advance tax liability
On or before March 15 Upto 100% of advance tax liability
Notes
a. Any amount paid u/s 211 on or before 31st March of the previous year, shall be treated as advance tax paid
during the financial year.
b. Where an assessee is a senior citizen (or super senior citizen) and does not have any income chargeable under
the head “Profits and gains of business or profession”, provision of advance tax is not applicable. In other
words, senior citizen not having business income is not liable to pay advance tax.
c. Every income including capital gain, winning from lotteries, dividend, etc. is subject to advance tax. However,
it is not possible to estimate capital gain, casual gain or dividend or where income under the head “Profits and
gains of business or profession” accrues or arises for the first time, therefore, where the assessee has paid the
whole of the amount of tax payable in respect of such income -
i. As part of the remaining installments of advance tax which were due; or
ii. Where no installments were due, by March 31 of the financial year immediately preceding the assessment
year,
- then it is deemed that all the provisions are complied.
d. While calculating advance tax, net agricultural income shall also be taken into consideration for computing
tax liability.
e. If any assessee does not pay any installment within due date he shall be deemed to be an assessee in default in
respect of such installment [Sec. 218]
f. Any sum, other than a penalty or interest, paid by an assessee as advance tax shall be treated as a payment of
tax and credit for such shall be given to the assessee in the regular assessment [Sec. 219]

3.5.1 Procedure to pay Advance Tax


a. On assessee’s own motion [Sec. 210(1)]
Procedure for 1st installment
1. Make an estimate of current year’s income, considering brought-forward losses, after deducting all
allowable deductions under chapter VIA.
Note: The estimate is not required to be filed with the tax authorities.

474 The Institute of Cost Accountants of India


Total Income and Tax Liability of Individuals & HUF

2. Compute the tax liability on above estimated income at the rates in force during the financial year and
reduce rebate, If any.
3. Add surcharge (if applicable).
4. Add Health and Education cess.
5. Deduct tax deducted or collected at source.
6. The amount so derived is the advance tax payable.
Where the advance tax payable is ₹ 10,000 or more, an appropriate percentage thereof should be deposited.
Procedure for subsequent installments
i. Check if estimate of income made earlier requires revision.
ii. If not, deposit appropriate amount of second, third or fourth installment of advance tax.
iii. If estimate of income needs revision, then make a revised estimate and compute tax liability thereon.
iv. Determine advance tax payable in subsequent installments after deducting amount paid in earlier
installments.
v. Deposit such advance tax.
b. On receipt of order from the Assessing Officer [Sec. 210(3) or (4)]
The A.O. may pass an order and issue a notice of demand u/s 156 requiring the assessee to pay advance tax.
Conditions to be satisfied for issuing such order
¾¾ The assessee has already been assessed by way of a regular assessment in any previous year.
¾¾ The Assessing Officer is of opinion that such person is liable to pay advance tax.
¾¾ Such order can be passed at any time during the financial year but not after last day of February.
¾¾ Such order must be made in writing.
¾¾ Such order also specifies the amount of advance tax and the installments thereof to be paid by the assessee.
Note: Such order can be issued even if assessee has paid any installment of advance tax during the year,
which is, in the opinion of the Assessing Officer, not as per provision of sec. 211.
Determination of advance tax by the Assessing Officer
The amount determined by the Assessing Officer shall be the higher of the following –
¾¾ Tax on latest assessed income as per regular assessment; or
¾¾ Tax on income declared by the assessee in the return relating to the previous year subsequent to the
previous year for which regular assessment has been made.
Procedure to be followed by assessee on receipt of such order
Advance tax to be paid by Whether intimation to AO is
Case
the assessee required
Where income estimated by the Assessing On the basis of the order of
No
Officer is correct the Assessing Officer
Where assessee estimates his current
On the basis of his own
income to be higher than that estimated No
estimate
by the Assessing Officer

The Institute of Cost Accountants of India 475


Direct Taxation

Advance tax to be paid by Whether intimation to AO is


Case
the assessee required
Where assessee estimates his current Yes, assessee shall submit his own
On the basis of his own
income to be lower than that estimated estimate of current income to the
estimate
by the Assessing Officer Assessing Officer (in Form 28A)
Note: As per sec. 210(4), Assessing Officer can revise his order to pay advance tax at any time before 1st
March of the relevant previous year.
Interest on non-payment of advance tax
Where an assessee fails to pay advance tax or defers the payment of advance tax on specified date, he shall be
liable to pay interest u/s 234B & 234C.

Illustration 41:
Find out the amount of advance tax payable by Mr. A on specified dates under the Income tax Act, 1961 for the
financial year 2023-24:

Business income ₹ 4,85,000


Long term capital gain on 31-5-2023 ₹ 60,000
Winning from lotteries on 12-6-2023 ₹ 50,000
Interest on loan ₹ 10,000
Other income ₹ 5,000
Investment in PPF ₹ 10,000
Tax deducted at source:
Case 1 ₹ 38,000
Case 2 ₹ 15,000
Solution :
Computation of total income of Mr. A for the previous year 2023-24
Particulars Details Amount
Profits and gains of business or profession 4,85,000
Capital gains: Long term capital gains 60,000
Income from other sources
Winning from lotteries 50,000
Interest on loan 10,000
Other income 5,000 65,000
Gross Total Income 6,10,000
Less: Deduction u/s 80C 10,000
Total Income 6,00,000
Computation of tax liability of Mr. A for the previous year 2023-24

Income Case 1 Case 2


Long term capital gain (₹ 60,000 @ 20%) 12,000 12,000
Winning from lotteries (₹ 50,000 @ 30%) 15,000 15,000
Balance income (₹ 4,90,000) 12,000 12,000

476 The Institute of Cost Accountants of India


Total Income and Tax Liability of Individuals & HUF

Income Case 1 Case 2


Tax 39,000 39,000
Less: Rebate u/s 87A Nil Nil
39,000 39,000
Add: Health & Education cess 1,560 1,560
40,560 40,560
Less: Tax Deducted at Source 38,000 15,000
Total Advance tax payable 2,560 25,560

Advance tax to be paid on specified dates –

Case 1: Since amount of advance tax payable is less than ₹ 10,000, assessee is not liable to pay advance tax.
Case 2:
Alternate 1 Alternate 2
Date
Working Amount Working Amount
15-06-2023 15% of ₹ 25,560 3,834 15% of ₹ 25,560 3,834
15-09-2023 30% of ₹ 25,560 7,668 [(45% of ₹ 25,560) – ₹ 3,834] 7,668
15-12-2023 30% of ₹ 25,560 7,668 [(75% of ₹ 25,560) – (₹ 3,834 + ₹ 7,668)] 7,668
15-03-2024 25% of ₹ 25,560 6,390 [100% of ₹ 25,560 – (₹ 3,834 + ₹ 7,668 + ₹ 7,668)] 6,390
Total 25,560 Total 25,560

Illustration 42:
Find out the amount of advance tax payable by ABC Ltd. on specified dates for the F.Y. 2023-24:

Business income ₹ 1,75,000


Long term capital gain on 31-7-2023 ₹ 2,50,000
Bank interest ₹ 10,000
TDS on business income ₹ 20,550

Solution :
Computation of total income of ABC Ltd. for the previous year 2023-24

Particulars Amount
Profits and gains of business or profession 1,75,000
Capital gains: Long term capital gains 2,50,000
Income from other sources: Bank Interest 10,000
Total Income 4,35,000

The Institute of Cost Accountants of India 477


Direct Taxation

Computation of tax liability of ABC Ltd. for the previous year 2023-24

Particulars Long term capital gain Other income


Income 2,50,000 1,85,000
Tax rate 20% 30%
Tax on above 50,000 55,500
Add: Surcharge Nil Nil
Tax and surcharge payable 50,000 55,500
Add: Health & Education cess 2,000 2,220
Tax and cess payable 52,000 57,720
Less: TDS - 20,550
Advance tax payable 52,000 37,170
Advance tax to be paid on specified dates – Alternate 1

Advance tax on LTCG Advance tax on income other than LTCG


Total
Date Amount
Working Working Amount (b) (a + b)
(a)
15-06-2023 As LTCG occurred on
Nil 15% of ₹ 37,170 5,576 5,576
31/7/2021
15-09-2023 45% of ₹ 52,000 23,400 30% of ₹ 37,170 11,151 34,551
15-12-2023 30% of ₹ 52,000 15,600 30% of ₹ 37,170 11,151 26,751
15-03-2024 25% of ₹ 52,000 13,000 25% of ₹ 37,170 9,292 22,292
Total 52,000 37,170 89,170
Advance tax to be paid on specified dates – Alternate 2

Advance tax on LTCG Advance tax on income other than LTCG Total
Date
Working Amount (a) Working Amount (b) (a + b)
15-06-2023 As LTCG occurred on
Nil 15% of ₹ 37,170 5,576 5,576
31/7/2021
15-09-2023 (45% of ₹ 37,170) –
(45% of ₹ 52,000) – Nil 23,400 11,151 34,551
(₹5,576)
15-12-2023 (75% of ₹ 52,000) – (₹ (75% of ₹ 37,170) – (₹
15,600 11,151 26,751
23,400) 5,576 + ₹ 11,151)
15-03-2024 (100% of ₹ 37,170) – (₹
(100% of ₹ 52,000) – (₹
13,000 5,576 + ₹ 11,151 + ₹ 9,292 22,292
23,400 + ₹ 15,600)
11,151)
Total 52,000 37,170 89,170

Illustration 43:
Find out the amount of advance tax payable by A on specified dates for the financial year 2023-24:

Business income (Tax of ₹ 4,704 is deducted at source) ₹ 5,11,000


Agricultural income ₹ 86,000

478 The Institute of Cost Accountants of India


Total Income and Tax Liability of Individuals & HUF

Solution :
Computation of tax liability of A for the previous year 2023-24

Particulars ₹
Tax on ₹ 5,97,000 (i.e. agricultural income ₹ 86,000 + non-agricultural income ₹ 5,11,000) 31,900
Less: Tax on ₹ 3,36,000 (i.e. agro income ₹ 86,000 + maximum exempted limit ₹ 2,50,000) 4,300
Tax liability 27,600
Less: Rebate u/s 87A Nil
27,600
Add: Health & Education cess 1,104
Tax and cess payable 28,704
Less: TDS 4,704
Advance tax payable 24,000
Advance tax to be paid on specified dates –

Alternate 1 Alternate 2
Date
Working Amount Working Amount
15-06-2023 15% of ₹ 24,000 3,600 15% of ₹ 24,000 3,600
15-09-2023 30% of ₹ 24,000 7,200 [(45% of ₹ 24,000) – ₹ 3,600] 7,200
15-12-2023 30% of ₹ 24,000 7,200 [(75% of ₹ 24,000) – (₹ 3,600 + ₹ 7,200)] 7,200

15-03-2024 25% of ₹ 24,000 6,000 [100% of ₹ 24,000 – (₹ 3,600 + ₹ 7,200 + ₹ 7,200)] 6,000

Total 24,000 Total 24,000

Quick MCQs:-

1. Advance Tax shall be payable by any person in respect of current income is


(a) D 50,000 or more
(b) D 10,000 or more
(c) D 1,00,000 or more
(d) None of the above

2. The benefit of payment of advance tax in one installment of or before 15th March is available to assessees
computing profits on presumptive basis-
(a) only under section 44AD
(b) under section 44AD and 44ADA
(c) under section 44AD and 44AE
(d) under section 44AD, 44ADA and 44AE

The Institute of Cost Accountants of India 479


Direct Taxation

Tax Deducted at Source & Tax Collected


3.6
at Source
TAX DEDUCTED AT SOURCE (TDS)

I
3.6.1 Meaning
t is a measure, in which person who are making payment of income are responsible to deduct tax from such
income (at specified rates) and pay only net amount. Tax so deducted (called TDS) shall be deposited with the
Government’s treasury within the stipulated time. The payer will issue a certificate in Form 16 or 16A9 to the
payee and the payee will get credit for TDS and his tax liability shall be reduced to that extent. In nutshell,
the provisions are merely a mode of collection of income tax and a check on tax evasion through proper control
and information.
Objects
 Quicker realisation of tax.
 Effective realisation of tax.
There are several provisions in the Act for TDS10, being discussed infra:

3.6.2 TDS on salary [Sec.192]


Who is responsible to deduct tax:
Any person responsible for paying any income chargeable under the head “Salaries” (i.e. employer) is required to
deduct tax at source.
When tax shall be deducted:
Tax shall be deducted at the time of payment of such income.
Rate of TDS:
Tax shall be deducted at the average rate of tax, computed on basis of prescribed rates in force for the financial year
in which payment to employee is made.
Other Points:
Following points shall be kept in mind by the person responsible to deduct tax at source u/s 192 -
1. Maximum exempted limit: Tax shall not be deducted if taxable salary is less than basic exemption limit.
2. Payment of tax by employer: The employer paying any income in the nature of non-monetary perquisite may
pay, at his option, tax on such income without making any deduction therefrom at the time when such tax was

9
All deductors (including Government deductors who deposit TDS in the Central Government Account through book entry) shall issue TDS certificate
generated through TIN central system and which is downloaded from the TIN website with a unique TDS certificate number.
10
As per provision of sec.206AA, if resident payee fails to provide his PAN, tax is required to be deducted at the rate mentioned in respective section or
20%, whichever is higher. [The sec.206AA is discussed later in the module]

480 The Institute of Cost Accountants of India


Total Income and Tax Liability of Individuals & HUF

otherwise deductible. For this purpose, tax shall be determined at the average of income-tax computed on the
basis of the rates in force for the financial year, on the income chargeable under the head “Salaries”. It is to
be noted that tax paid on non-monetary perquisites by the employer shall not be considered as income of the
employee.
3. Particulars of perquisites: The employer shall furnish a statement to the employee (whose salary exceeds
₹ 1,50,000) giving correct and complete particu­lars of perquisites or profits in lieu of salary provided to
employee and the value thereof in Form 12BA provided salary of such employee exceeds ₹ 2,00,000.
4. Salary from more than one source: As per Sec. 192(2), where assessee is working simultaneously with more
than one employer, he may furnish to any one employer at his choice, details (in Form No. 12B) of income
taxable under the head “Salaries” due to or received by him from other employer(s) and such employer shall
deduct tax on aggregate salary.
5. Treatment of other income: As per Sec. 192(2B), where an assessee who receives any income chargeable
under the head “Salaries” has, in addition, any income chargeable under any other head of income for the same
financial year, he may (or may not) furnish to employer the particulars (in plain paper) of—
¾¾ such other income (only income and not loss)
¾¾ tax deducted on such other income as per provision of this chapter;
¾¾ the loss, if any, under the head “Income from house property” (Losses under any other head are not to be
considered)
Tax deducted u/s 192 shall be higher of the following:
¾¾ Tax deductible from income, that would be so deductible, if loss under the head ‘Income from House
Property’, other income (only income) and the tax deducted thereon had been taken into account.
¾¾ Tax deductible from income under the head “Salaries”, that would be so deductible (after adjusting loss
under the head Income from house property), if the other income (or loss) and the tax deducted thereon
had not been taken into account.
6. Evidence of claim: The responsible person shall, for the purposes of estimating income of the assessee or
computing tax deductible, obtain the evidence or proof or particulars of prescribed claims (including claim for
set-off of loss) from the assessee in such form and manner as may be prescribed.
7. TDS on ESOP: ESOPs have been a significant component of the compensation for the employees of start-ups,
as it allows the founders and start-ups to employ highly talented employees at a relatively low salary amount
with balance being made up via ESOPs. ESOPs are taxed as perquisites u/s 17(2). The taxation of ESOPs is
split into two components:
i. Tax on perquisite as income from salary at the time of exercise.
ii. Tax on income from capital gain at the time of sale.
The tax on perquisite is required to be paid at the time of exercising of option which may lead to cash flow
problem as this benefit of ESOP is in kind. In order to ease the burden of payment of taxes by the employees
of the eligible start-ups or TDS by the start-up employer, it is provided that for the purpose of deducting or
paying tax, a person, being an eligible start-up [u/s 80-IAC], responsible for paying any income to the assessee
being such perquisite, deduct or pay, as the case may be, tax on such income within 14 days:
a. after the expiry of 48 months from the end of the relevant assessment year; or
b. from the date of the sale of such specified security or sweat equity share by the assessee; or

The Institute of Cost Accountants of India 481


Direct Taxation

c. from the date of which the assessee ceases to be the employee of the person;
–– whichever is the earlier.
The tax shall be deducted or paid at the rates in force of the financial year in which the said security or sweat
equity share is allotted or transferred.
Similar provision is applicable, in case, employee is paying tax directly (in case tax is not deducted).

Provision Illustrated
Mr. X working in Y Ltd. furnishes the following, compute the tax to be deducted at source by Y Ltd.

Taxable salary ₹ 5,00,000


Loss from House Property ₹ 5,000
Loss from Business ₹ 10,000
Gross Interest Income (TDS ₹ 5,000) ₹ 60,000
Investment in PPF ₹ 10,000

Solution
Computation of tax to be deducted at source by Y Ltd

Tax on income Tax on income


Particulars including other excluding other
income income
Salary income 5,00,000 5,00,000
Loss from House Property (5,000) (5,000)
Interest Income 60,000 Nil
Gross Total Income 5,55,000 4,95,000
Less: Deduction u/s 80C [Investment in PPF] 10,000 10,000
Total Income 5,45,000 4,85,000
Tax on above 21,500 11,750
Less: Rebate u/s 87A Nil 11,750
Tax after rebate 21,500 Nil
Add: Health & Education cess @ 4% 860 Nil
Tax and cess payable 22,360 Nil
Less: Tax deducted at source on other income 5,000 Nil
Balance 17,360 Nil
Tax to be deducted by Y Ltd. (Higher of ₹ 17,360 or ₹ Nil) 17,360
Exemption or relaxation from the provision
When the recipient applies to the Assessing Officer in Form 13 and gets a certificate authorizing the payer to deduct
no tax or deduct tax at lower rate. [Refer sec.197]

3.6.3 TDS on Payment from Employees Provident Fund [Sec. 192A]


Who is responsible to deduct tax: The trustees of the Employees’ Provident Fund Scheme, 1952, framed u/s 5
of the Employees’ Provident Funds and Miscellaneous Provisions Act, 1952 or any person authorised under the
scheme to make payment of accumulated balance due to employees.

482 The Institute of Cost Accountants of India


Total Income and Tax Liability of Individuals & HUF

When tax shall be deducted: At the time of making of payment.


Tax is deducted on: The accumulated balance due to an employee participating in a recognised provident fund
includible in his total income owing to the not applicability of the provisions of rule 8 of Part A of the Fourth
Schedule.

Rule 8 of Part A of the Fourth Schedule provides the accumulated balance due and becoming payable to an
employee participating in a recognised provident fund shall be exempted:
a. if he has rendered continuous service with his employer for a period of 5 years or more; or
b. if, though he has not rendered such continuous service, the service has been terminated by reason of the
employee’s ill-health, or by the contraction or discontinuance of the employer’s business or other cause
beyond the control of the employee; or
c. if, on the cessation of his employment, the employee obtains employment with any other employer, to the
extent the accumulated balance due and becoming payable to him is transferred to his individual account in
any recognised provident fund maintained by such other employer.
Taxpoint: If the accumulated balance paid to the assessee is exempt then tax is not required to be deducted.
Rate of TDS: 10%
 Any person entitled to receive any amount on which tax is deductible shall furnish his Permanent Account
Number (PAN) to the person responsible for deducting such tax, failing which tax shall be deducted at 20%.
When TDS is not applicable: Aggregate amount of such payment to the payee is less than ₹ 50,000.

3.6.4 TDS on Interest on Securities [Sec 193]


Who is responsible to deduct tax: Any person responsible for payment of interest on securities (other than
interest on Government securities and certain specified securities) to any resident person.
Note: In the following cases tax is not required to be deducted:
1. Interest payable to a resident-individual or a resident HUF on debentures is not subject to TDS provided
following conditions are satisfied –
a. Such debentures are issued by a company in which the public are substantially interested;
b. Such debentures may be listed or unlisted
c. The interest is paid by the company by an account payee cheque; and
d. The amount of such interest payable during the financial year to such individual does not exceed ₹ 5,000.
2. Any interest payable on any security of the Central or State Government. (However, tax is required to be
deducted on interest payable on 8% Savings (Taxable) Bonds, 2003 or 7.75% Savings (Taxable) Bonds, 2018,
if amount of interest in a financial year exceeds ₹ 10,000).
3. Any interest payable on any security issued by a company, where such security in dematerialised form and is
listed on a recognised stock exchange.
4. Any interest payable on securities beneficially held by Life Insurance Corporation of India or General Insurance
Corporation of India or any of the 4 companies formed by virtue of the scheme framed u/s 16(1) of the General
Insurance Business (Nationalisation) Act, 1972 or any other insurer.
5. Any interest payable to Regimental Fund or non-Public Fund established by Armed Force [income of whose
is exempt u/s 10(23AA).

The Institute of Cost Accountants of India 483


Direct Taxation

When tax shall be deducted: At the time of payment or crediting the payee, whichever is earlier.
Taxpoint: Where any amount is credited to any account (for e.g. “Interest payable account” or “Suspense account”)
instead of Payee account, such crediting shall be deemed to be credit of such income to the account of the payee.
Rate of TDS: 10% (No surcharge, health and education cess)
Exemption or relaxation from the provision:
 When the recipient applies to the Assessing Officer in Form 13 and gets a certificate authorizing the payer to
deduct tax at lower rate or deduct no tax; [Refer sec.197]
 When a declaration in Form 15G (in duplicate) is furnished by the assessee to the payer [Refer sec.197A]

3.6.5 TDS on Dividends [Sec. 194]


Who is responsible to deduct tax: The principal officer of a domestic company paying dividend u/s 2(22) to any
resident shareholder.
Note: In the following cases, tax is not required to be deducted:
A. Dividend is paid to an insurance company
B. Dividend is paid to a business trust [as defined u/s 2(13A)], by a special purpose vehicle referred to in the
Explanation to sec. 10(23FC);
C. Dividend paid to an individual shareholder, if:
i. the dividend is paid by the company by any mode other than cash; and
ii. the amount of such dividend or the aggregate of the amounts of such dividend distributed or paid or
likely to be distributed or paid during the financial year by the company to the shareholder, does not
exceed ₹ 5,000:
D. Dividend paid to any other notified person
When tax shall be deducted: At the time of payment
Rate of TDS: 10% (No surcharge, health and education cess) on dividend considered u/s 2(22)
Exemption or relaxation from the provision:
 When the recipient applies to the Assessing Officer in Form No. 13 and gets a certificate authorizing the payer
to deduct tax at lower rate or deduct no tax [Refer sec.197]
 When a declaration in Form 15G (in duplicate) is furnished by the assessee to the payer [Refer sec.197A]

3.6.6 TDS on Interest other than interest on securities [Sec. 194A]


Who is responsible to deduct tax
Following person are responsible to deduct tax at source on interest other than interest on securities to a resident
person -
 Any person, other than Individual or HUF; or
 An individual or a HUF, whose total sales, gross receipts or turnover from the business or profession carried
on by him exceed ₹ 1 crore in case of business or ₹ 50 lakh in case of profession during the financial year
immediately preceding the financial year in which such interest is credited or paid.

484 The Institute of Cost Accountants of India


Total Income and Tax Liability of Individuals & HUF

Note: No tax shall be deducted at source if during the financial year, interest payable by the payer to payee does not
exceed ₹ 5,000.
However, following persons are responsible to deduct tax at source on interest other than interest on securities paid
or payable to a resident person if during the financial year, interest paid/payable exceeds ₹ 40,000 (₹ 50,000 in case
payee is a senior citizen):
 A banking company;
 Co-operative society engaged in carrying on the business of banking;
 Post office in respect of notified scheme

Taxpoint: In case of interest payable by banking company or co-operative society or housing company, such limit
shall be computed with reference to branch. However, where payer has adopted Core Banking Solution (CBS), the
aforesaid limit shall be computed with reference to entity and not branch.

When tax shall be deducted: At the time of payment or crediting the payee, whichever is earlier.

Rate of TDS: 10% (No surcharge, health and education cess)


Taxpoint: The Central Government may notify that the deduction of tax shall not be made or shall be made at such
lower rate, from payment made to specified person.
Other points
Following interests are not subject to TDS -
 Interest payable to -
a. Banking company; or
b. Co-operative society engaged in carrying on the business of banking (including a co-operative land
mortgage bank); or
c. Financial corporation established by or under a Central, State or Provincial Act; or
d. The Life Insurance Corporation of India; or
e. The Unit Trust of India; or
f. Any company or co-operative society carrying on the business of insurance; or
g. Such other notified institution, association or body [No notification shall be issued after 31-03-2020].
 Interest credited or paid by a firm to a partner of the firm;
 Interest credited or paid by a co-operative society (other than a co-operative bank) to a member thereof or to
such income credited or paid by a co-operative society
 Interest on deposits with a primary agricultural credit society or a primary credit society or a co-operative land
mortgage bank or a co-operative land development bank;
 Interest on deposits (other than time deposits) with a co-operative society, other than a co-operative society or
bank referred above, engaged in carrying on the business of banking
Exception [in case of co-operative society]
Aforesaid co-operative society shall be liable to deduct tax, if:

The Institute of Cost Accountants of India 485


Direct Taxation

a. the total sales, gross receipts or turnover of the co-operative society exceeds ₹ 50 crore during the
financial year immediately preceding the financial year in which the interest is credited or paid; and
b. the aggregate of the amounts of such interest, credited or paid, during the financial year is more than
₹ 50,000 in case of payee being a senior citizen (₹ 40,000 in other cases).
 Interest credited or paid in respect of deposits under any notified scheme like NSC, Indira Vikas Patra, Kisan
Vikas Patra and Post office Monthly Income Account.
 Interest credited or paid in respect of deposits (other than time deposits and recurring deposit) with a banking
company.
¾¾ TDS is also required to be deducted on Recurring Deposits.
 Interest credited on the compensation amount awarded by the Motor Accidents Claims Tribunal;
 Interest paid on the compensation amount awarded by the Motor Accidents Claims Tribunal where the amount
of such income or, as the case may be, the aggregate of the amounts of such income paid during the financial
year does not exceed ₹ 50,000
 Interest credited or paid by the Central Government under any provision of the Income Tax Act or the Wealth-
tax Act, 1957
 Income which is paid or payable by an infrastructure capital company or infrastructure capital fund or
infrastructure debt fund or a public sector company or scheduled bank in relation to a Zero Coupon Bond
issued on or after 1-6-2005 by such company or fund.
 Income by way of interest referred to in sec. 10(23FC)
 Interest on FDRs, made in the name of the Registrar General of the Court or the depositor of the fund on the
directions of the Court, will not be subject to TDS till the matter is decided by the Court. However, once the
Court decides the ownership of the money lying in the fixed deposit, the provisions of section 194A will apply
to the recipient of the income
Exemption or relaxation from the provision
 When the recipient applies to the assessing officer in Form No. 13 and gets a certificate authorizing the payer
to deduct tax at lower rate or deduct no tax. [Refer sec.197]
 When a declaration in Form 15G is furnished by the assessee to the payer [Refer sec.197A]
Adjustment for short-deduction
The person responsible to deduct tax may at the time of making any deduction, increase or reduce the amount to
be deducted for the purpose of adjusting any excess or deficiency arising out of any previous deduction or failure
to deduct during the financial year.

3.6.7 TDS on Winning from lotteries or cross word puzzles, etc. [Sec. 194B]
Who is responsible to deduct tax: Any person responsible for paying to any person any income by way of winning
from any lottery or crossword puzzle or card game and other game of any sort, exceeding ₹ 10,000 in aggregate.
Taxpoint: If income is more than ₹ 10,000 in aggregate (say ₹ 12,000), then tax shall be deducted on the whole
amount (i.e. ₹ 12,000).
Rate of TDS: 30% (in case of non-resident payee, applicable surcharge, health and education cess shall also be
considered)

486 The Institute of Cost Accountants of India


Total Income and Tax Liability of Individuals & HUF

Payee Rate of TDS as a % of amount paid or payable


Resident Person 30%
Non-Resident 30% + Surcharge + Health and Education cess
When tax shall be deducted: At the time of payment.
Other Points
 If prize is given partly in cash and partly in kind then tax on whole prize (i.e. aggregate of cash and value of
prize in kind) shall be deducted from the cash prize.
 If prize is given in kind only (or cash prize is not sufficient), then payer should ensure that tax has been paid
on such income before releasing such prize.
 Where a certain percentage has to be forgone either in favour of Government or an agency conducting lotteries,
then such portion is not subject to deduction of tax at source.
 Where an agent receives the prize money on unsold ticket or becomes entitled to an unclaimed prize, it shall
form part of his business income and therefore not liable for tax deduction u/s 194B
 If prize money is paid in instalments, then tax shall be deducted at the time of payment of each instalment.
 Tax shall be deducted on payment of commission, etc. to the lottery agent u/s 194G and not u/s 194B.

3.6.8 TDS on winning from Horse races [Sec. 194BB]


Who is responsible to deduct tax: Any person1 responsible for paying to any person any income by way of
winning from horse races, exceeding ₹ 10,000 in aggregate.
Rate of TDS: See sec. 194B (TDS on Winning from lotteries or cross word puzzles or card games, etc.)
When tax shall be deducted: At the time of payment.
Other Points
1. Any person, here means a book-maker or a person to whom a licence has been granted by the Government for
horse racing or arranging for wagering, betting in any race course.
2. Race-income other than horse races like camel races etc is not covered by this section.
3. Note: Sec 194BA, TDS on winning from on-line gaming is @30% without any threshold limit (i.e. ₹ 10,000).

3.6.9 TDS on payment to Contractor [Sec. 194C]


Who is responsible to deduct tax :
Any specified person1 responsible for paying any sum to any resident-contractor for carrying out any work
(including supply of labour for carrying out any work) in pursuance of a contract.
1.
Following are the specified person -
a. The Central Government or any State Government; or
b. Any local authority; or
c. Any corporation established by or under a Central, State or Provincial Act; or
d. Any company; or
e. Any co-operative society;

The Institute of Cost Accountants of India 487


Direct Taxation

f. Any authority, constituted in India by or under any law, engaged either for the purpose of dealing with and
satisfying the need for housing accommodation or for the purpose of planning, development or improvement
of cities, towns and villages, or for both; or
g. Any society registered under the Societies Registration Act, 1860 or under any law corresponding to that Act
in force in any part of India; or
h. Any trust; or
i. Any University established or incorporated by or under a Central, State or Provincial Act and an institution
declared to be a University u/s 3 of the University Grants Commission Act, 1956; or
j. Any Government of a foreign State or a foreign enter­prise or any association or body established outside India;
or
k. Any firm; or
l. Individual or a HUF or an association of persons or a body of individuals (if not covered by aforesaid cases),
has total sales, gross receipts or turnover from business or profession carried on by him exceeding ₹ 1 crore in
case of business or ₹ 50 lakh in case of profession during the financial year immediately preceding the financial
year in which such sum is credited or paid to the account of the contractor.
Note: However, no individual or a HUF shall be liable to deduct income-tax where amount is credited or paid
exclusively for personal purposes of such individual or any member of HUF.
When tax cannot be deducted:
 Case 1
When following conditions are satisfied then tax cannot be deducted:
i. Any sum credited or paid in pursuance of any contract, the consideration for which does not exceed ₹
30,000; and
Taxpoint: The limit of ₹ 30,000 is on individual contract.
ii. Where the aggregate of the amounts of such sums credited or paid or likely to be credited or paid during
the financial year does not exceed ₹ 1,00,000
 Case 2
When following conditions are satisfied then tax cannot be deducted:
a. Amount is paid or payable to a resident contractor during the course of plying, hiring or leasing goods
carriage (here-in-after referred to as transport operator).
b. Such operator furnishes his Permanent Account Number (PAN) to the payer.
c. Where such contractor owns 10 or less goods carriages at any time during the previous year and furnishes
a declaration to that effect along with his PAN to the person paying or crediting such sum.
Taxpoint: Tax is not required to be deducted even if amount of payment exceeds Rs. 1,00,000/-

When tax shall be deducted: At the time of payment or crediting the party, whichever is earlier.
Rate of TDS:

Payee Rate as a % of amount paid or payable


Individual or HUF 1%
Other Payee 2%

488 The Institute of Cost Accountants of India


Total Income and Tax Liability of Individuals & HUF

Exemption or relaxation from the provision


When the recipient applies to the Assessing Officer in Form 13 and gets a certificate authorising the payer to deduct
tax at lower rate or deduct no tax. [Refer sec.197]
Other Points
 Contract shall include sub-contract.
 “Work” shall include —
a. advertising;
b. broadcasting and telecasting including production of programmes for such broadcasting or telecasting;
c. carriage of goods and passengers by any mode of transport other than by railways;
d. catering.
e. manufacturing or supplying a product according to the requirement or specification of a customer by
using material purchased from such customer or its associate, being a person placed similarly in relation
to such customer as is the person placed in relation to the assessee u/s 40A(2)(b),
but does not include manufacturing or supplying a product accord­ing to the requirement or specification
of a customer by using material purchased from a person, other than such customer or associate of such
customer.
 Where any sum is paid or credited for carrying out any work mentioned in (e), tax shall be deducted at source:
(i) on the invoice value excluding the value of material, if such value is mentioned separately in the invoice;
or
(ii) on the whole of the invoice value, if the value of material is not mentioned separately in the invoice.

3.6.10 TDS on Insurance Commission [194D]


Who is responsible to deduct tax: Any person responsible for paying to a resident person any income by way
of remuneration or reward (i.e. commission etc.) for soliciting or procuring insurance business including business
relating to the continuance, renewal or revival of policies of insurance.
Note: Tax shall not be deducted if the aggregate amounts of remuneration or reward credited or paid during the
financial year to the payee does not exceed ₹ 15,000.
When tax shall be deducted: At the time of payment or crediting the party whichever is earlier.
Rate of TDS:

Payee Rate of TDS


Domestic Company 10%
Other Payee 5%
Exemption or relaxation from the provision
 When the recipient applies to the Assessing Officer in Form 13 and gets a certificate authorizing the payer to
deduct tax at lower rate or deduct no tax; [Refer sec.197]
 When a declaration in Form 15G (in duplicate) is furnished by the assessee to the payer [Refer sec.197A]

The Institute of Cost Accountants of India 489


Direct Taxation

3.6.11 TDS on Payment in respect of Life Insurance Policy [194DA]


Who is responsible to deduct tax: Any person responsible for paying to a resident any sum under a life insurance
policy, including the sum allocated by way of bonus on such policy other than the amount not includible in the total
income u/s 10(10D).
 TDS is not required to be deducted on any payment which is exempt u/s 10(10D).
 Tax shall not be deducted if the aggregate amounts of payments to the payee during the financial year is less
than ₹ 1,00,000.
When tax shall be deducted: At the time of payment
Rate of TDS: 5% (No surcharge, health and education cess) on the income comprised therein.

3.6.12 TDS on payment to non-resident sportsman or sports associations [Sec. 194E]


Who is responsible to deduct tax: Any person who is responsible to pay the following income –

Payee Income by way of


Sportsman (including an 1. Participation in India in any game (excluding card game or gambling) or sport
athlete) or an entertainer 2. Advertising
being non-resident foreign 3. Contribution of articles relating to any game or sports in any newspaper,
citizen magazine or journal.
Sports association being non-
Any game (other than card game) or sports organised in India
resident
Rate of TDS: 20% (+ Surcharge + health and education cess)
When tax shall be deducted: At the time of payment or crediting the party whichever is earlier.

3.6.13 Payments in respect of deposits under National Savings Scheme, etc.[Sec. 194EE]
Who is responsible to deduct tax: Any person [i.e. post office] responsible for paying an amount (either principal
or interest) referred to in sec. 80CCA(2)(a) [i.e. National Saving Scheme, 1987]
Note: Tax shall not be deducted at source if the aggregate amount of such payments to the payee during the
financial year is less than ₹ 2,500.
When tax shall be deducted: At the time of payment.
Rate of TDS:

Payee Rate of TDS as a % of amount paid or payable


Resident 10%
Non-Resident 10% (+ Surcharge + Health and Education cess)
Exemption or relaxation from the provision
a. Where the payment is made to the heirs of the deceased assessee.
b. When a declaration in Form 15-G (in duplicate) is furnished by the assessee to the payer.

3.6.14 TDS on repurchase of units of Mutual Fund or Unit Trust of India [Sec. 194F]
Who is responsible to deduct tax: The person responsible for paying to any person any amount referred to in
section 80CCB(2) [i.e. mutual fund or UTI]

490 The Institute of Cost Accountants of India


Total Income and Tax Liability of Individuals & HUF

When tax shall be deducted: At the time of payment


Rate of TDS: 20%

3.6.15 TDS on Commission on sale of lottery tickets [Sec. 194G]


Who is responsible to deduct tax: Any person responsible for paying any income by way of commission,
remuneration or prize (by whatever name called) on lottery tickets to any person, who is stocking, distributing,
purchasing or selling such tickets.
When tax shall be deducted: At the time of payment or crediting the payee, whichever is earlier.
Rate of TDS:

Payee Rate of TDS as a % of amount paid or payable


Resident 5%
Non-Resident 5% (+ Surcharge + Health & Education cess)
When TDS cannot be made
Where amount of commission, etc. on such tickets does not exceed ₹ 15,000.
Exemption or relaxation from the provision
When the recipient applies to the assessing officer in Form 13 and gets a certificate authorising the payer to deduct
tax at lower rate or deduct no tax. [Refer sec.197]

3.6.16 TDS on commission, etc. other than insurance commission [Sec. 194H]
Who is responsible to deduct tax: Following persons are responsible to deduct tax at source on commission or
brokerage (other than commission on insurance) to a resident person -
 Any person, other than individual or HUF; &
 Individual or HUF whose total sales, gross receipts or turnover from business or profession carried on by him
exceeding ₹ 1 crore in case of business or ₹ 50 lakh in case of profession during the financial year immediately
preceding the financial year in which such sum is credited or paid.
Note:
1. No tax shall be deducted if the aggregate amounts of commission or brokerage credited or paid during the
financial year to the payee does not exceed ₹ 15,000.
2. No deduction shall be made on any commission or brokerage payable by Bharat Sanchar Nigam Limited or
Mahanagar Telephone Nigam Limited to their public call office franchisees.
When tax shall be deducted: At the time of payment or crediting the payee, whichever is earlier.
Rate of TDS: 5%
Exemption or relaxation from the provision
When the recipient applies to the Assessing Officer in Form No. 13 and gets a certificate authorising the payer to
deduct tax at lower rate or deduct no tax [Refer sec.197]
When commission is retained by an agent
Where commission or brokerage is retained by the consignee/agent while remitting the sale consideration, the
consignor/principal will have to deposit the tax deductible on the amount of such retained commission - [Circular
No.619]

The Institute of Cost Accountants of India 491


Direct Taxation

Other Points: Commission or brokerage includes any payment received or receivable, directly or indirectly, by a
person acting on behalf of another person for services rendered (not being professional services) or for any services
in the course of buying or selling of goods or in relation to any transaction relating to any asset, valuable article or
thing, not being securities.

3.6.17 TDS on Rent [Sec. 194-I]


Who is responsible to deduct tax: Following persons are responsible to deduct tax at source on rent to a resident
person -
 Any person, other than individual or HUF; &
 Individual or HUF whose total sales, gross receipts or turnover from business or profession carried on by him
exceeding ₹ 1 crore in case of business or ₹ 50 lakh in case of profession during the financial year immediately
preceding the financial year in which such sum is credited or paid
Note: Tax shall not be deducted if the aggregate amounts of rent credited or paid during the financial year to the
payee does not exceed ₹ 2,40,000.
Taxpoint: Where the share of each co-owner is known, the limit of ₹ 2,40,000 is applicable to each co-owner
separately.
“Rent” means any payment, by whatever name called, under any lease, sub-lease, tenancy or any other agreement
or arrangement for the use of (either separately or together) any, -

a) Land b) Building (including factory building) c) Land appurtenant to a building (including


d) Machinery e) Plant factory building)
f) Furniture g) Fittings h) Equipment
- whether or not any or all of the above are owned by the payee.
When tax shall be deducted: At the time of payment or crediting the account of payee, whichever is earlier.
Rate of TDS:

Nature of Assets Rate


Machinery or plant or equipment 2%
Land or building (including factory building) or land appurtenant to a building (including factory 10%
building) or furniture or fittings

When TDS cannot be made


 Where the payee is the Government, local authorities u/s 10(20) and statutory authorities u/s 10(20A), then
TDS cannot be made - [Circular No. 699, dated 30-1-1995].
 Where rent is credited or paid to a business trust, being a real estate investment trust, in respect of any real
estate asset, referred to in sec. 10(23FCA), owned directly by such business trust
Exemption or relaxation from the provision
 When a declaration in Form 15G (in duplicate) is furnished by the assessee to the payer [Refer sec.197A]
 When the recipient applies to the Assessing Officer in Form 13 and gets a certificate authorising the payer to
deduct tax at lower rate or deduct no tax. [Refer sec.197]

492 The Institute of Cost Accountants of India


Total Income and Tax Liability of Individuals & HUF

Other Points
 Payment by an individual reimbursed by the company: Where an employee or an individual representing
a company (like a consultant, auditor, etc.) makes a payment for hotel accommodation directly to the hotel,
the question of tax deduction at source would not normally arise (except where he is covered u/s 44AB). This
is because, it is the employee or such individual who makes the payment and the company merely reimburses
the expenditure. [Circular No. 5/2002, dated 30-7-2002]
 Lump sum lease premium or one-time upfront lease charges, which are not adjustable against periodic
rent, paid or payable for acquisition of long-term leasehold rights over land or any other property are not
payments in the nature of rent within the meaning of sec. 194-I. Therefore, such payments are not liable for
TDS u/s 194-I [Circular 35/2016 dated 13-10-2016]

3.6.18 TDS on transfer of certain immovable property other than agricultural land [Sec.
194-IA] Amended
Who is responsible to deduct tax: Any person, being a transferee, responsible for paying (other than the person
referred to in sec. 194LA) to a resident transferor any sum by way of consideration for transfer of any immovable
property (other than rural agricultural land)
 “Immovable property” means any land (other than rural agricultural land) or any building or part of a building.
 “Rural agricultural land” – as discussed in Chapter ‘Capital Gain’
When tax shall be deducted: At the time of payment or crediting the payee, whichever is earlier.
Rate of TDS: 1% of such consideration or stamp duty value of such property, whichever is higher.
When TDS cannot be made: Where both the consideration for the transfer of an immovable property and stamp
duty value of the property are less than ₹ 50 lakh.
 Consideration for transfer of any immovable property shall include all charges of the nature of club membership
fee, car parking fee, electricity or water facility fee, maintenance fee, advance fee or any other charges of
similar nature, which are incidental to transfer of the immovable property;
Other Points:
The deductor is not required to obtain Tax Deduction Account Number as required u/s 203A.

3.6.19 TDS on Payment of rent by certain individual / HUF [Sec. 194-IB]Amended


Who is responsible to deduct tax: Any person, being an individual or a Hindu undivided family (other than those
referred to in sec. 194-I), responsible for paying to a resident any rent.
 “Rent” means any payment, by whatever name called, under any lease, sub-lease, tenancy or any other
agreement or arrangement for the use of any land or building or both
When tax shall be deducted: At the time of credit of rent for the last month of the previous year (or the last month
of tenancy, if the property is vacated during the year) to the account of the payee or at the time of payment thereof
in cash or by issue of a cheque or draft or by any other mode, whichever is earlier
Rate of TDS: 5%

When TDS cannot be made: Where rent for a month or part thereof does not exceed ₹ 50,000

The Institute of Cost Accountants of India 493


Direct Taxation

Other Points:

 The deductor is not required to obtain Tax Deduction Account Number as required u/s 203A.

 As per provision of sec. 206AA, if payee fails to provide his PAN, TDS is required to be deducted @ 20%.
However, deduction under this section shall not exceed the amount of rent payable for the last month of the
previous year (or the last month of the tenancy).

3.6.20 TDS on Payment under Joint Development Agreement [Sec. 194-IC]


Who is responsible to deduct tax: Any person, responsible for paying any consideration referred to in sec. 45(5A)
[i.e., Joint Development Agreement] to a resident person.

When tax shall be deducted: At the time of payment or crediting the payee, whichever is earlier.

Rate of TDS: 10%

3.6.21 Fees for Professional or Technical Services [Sec. 194J]


Who is responsible to deduct tax: Following persons are responsible to deduct tax at source for paying fees for
professional or for technical service or royalty or any sum referred to in sec.28(va) (i.e., fees for non-competence)
or director fees (which is not covered u/s 192) to a resident person -

 Any person, other than individual or HUF; &

 Individual or HUF, whose total sales, gross receipts or turnover from business or profession carried on by him
exceeding ₹ 1 crore in case of business or ₹ 50 lakh in case of profession during the financial year immediately
preceding the financial year in which such sum is credited or paid.

Note: Tax cannot be deducted if the aggregate amount of such fees credited or paid during the financial year to the
payee does not exceed:

Nature of Payment Amount


Professional fees ₹ 30,000
Fees for technical service ₹ 30,000
Royalty ₹ 30,000
Any sum referred to in sec.28(va) ₹ 30,000
Director fees by whatever name called, (which is not covered u/s 192) Nil

Note: ₹ 30,000 limit is not on individual transaction. If several professional bills in a year together exceed the
amount of ₹ 30,000, tax shall be deducted, even though any individual bill amount does not exceed ₹ 30,000.
Taxpoint: U/s 194C limit of ₹ 30000 was on individual transaction, whereas u/s 194J limit of ₹ 30,000 is applicable
on all transactions together made during the year.
No threshold limit is provided for director fees. i.e., TDS is applicable even amount is less than ₹ 30,000.
When tax shall be deducted: At the time of payment or crediting the payee, whichever is earlier.

494 The Institute of Cost Accountants of India


Total Income and Tax Liability of Individuals & HUF

Rate of TDS:

Case Rate of Tax


Payee is engaged only in the business of operation of call centre
Fees for technical services (other than professional services)
2%
Royalty where such royalty is in the nature of consideration for sale, distribution
or exhibition of cinematographic films
other 10%
When TDS cannot be made
1. Individual or HUF shall not be liable to deduct income-tax on fees which is credited or paid exclusively for
personal purposes of such individual or any member of HUF.
2. Fees paid by non-resident: Any fees paid through regular banking channels to any professional who is
resident in India by the non-resident who does not have any agent or business connection or permanent
business establishment in India, may not be subject to this provision [Circular No.726 dated 16-10-1995].
Exemption or relaxation from the provision
When the recipient applies to the Assessing Officer in Form 13 and gets a certificate authorising the payer to deduct
tax at lower rate or deduct no tax. [Refer sec. 197]
Other Points
“Professional services” means services rendered by a person in the course of carrying on legal, medical, engineering
or architectural profession or the profession of accountancy or technical consultancy or interior decoration or
advertising (i.e. Model, photographer, artist providing service to advertising agency) or such other profession as is
notified by the Board for the purposes of section 44AA or of this section;
“Fees for technical services” means any consideration (including any lump sum consideration) for rendering
of any managerial, technical or consultancy services (including the provision of services of technical or other
personnel) but does not include consideration for any construction, assembly, mining or like project undertaken by
the recipient or consideration which would be income of the recipient chargeable under the head “Salaries”.

3.6.22 TDS on income in respect of units [Sec. 194K]


Who is responsible to deduct tax: Any person responsible for paying to a resident any income in respect of:
a. units of a Mutual Fund specified u/s 10(23D); or
b. units from the Administrator of the specified undertaking; or
c. units from the specified company
When tax shall be deducted: At the time of payment or crediting the payee, whichever is earlier.
Rate of TDS: 10%
When TDS cannot be made: In the following cases tax shall not be deducted:
a. If the aggregate amounts of income credited or paid during the financial year to the payee does not exceed ₹
5,000
b. If the income is of the nature of capital gains.

The Institute of Cost Accountants of India 495


Direct Taxation

Exemption or relaxation from the provision


 When the recipient applies to the Assessing Officer in Form 13 and gets a certificate authorizing the payer to
deduct tax at lower rate or deduct no tax; [Refer sec.197]
 When a declaration in Form 15G (in duplicate) is furnished by the assessee to the payer [Refer sec.197A]

3.6.23 TDS on payment of compensation on acquisition of certain immovable property


[Sec. 194LA]
Who is responsible to deduct tax: Any person responsible for paying to a resident any sum, being compensation
or the enchanced compensation or the consideration or the enhanced consideration on account of compulsory
acquisition of any immovable property (other than agricultural land)
When tax shall be deducted: At the time of payment of such sum in cash or by issue of a cheque or draft or by
any other mode, whichever is earlier.
Rate of TDS: 10%
When TDS cannot be made
Where the amount of such payment or the aggregate amount of such payments to a resident during the financial year
does not exceed ₹ 2,50,000.
Exemption or relaxation from the provision
When the recipient applies to the Assessing Officer in Form 13 and gets a certificate authorising the payer to deduct
tax at lower rate or deduct no tax [Refer sec. 197]
Other points
1. Agricultural land means agricultural land in India.
2. Immovable property means any land (other than agricultural land) or any building or part of a building.
3. Deduction shall not be made where payment is made in respect of any award or agreement which has been
exempted from levy of income-tax u/s 96 of the Right to Fair Compensation and Transparency in Land
Acquisition, Rehabilitation and Resettlement Act, 2013

3.6.24 Payment of certain sums by certain individuals or Hindu undivided family [Sec.
194M]
Who is responsible to deduct tax: An individual or a Hindu undivided family (other than those who are required
to deduct income-tax as per the provisions of sec. 194C, 194H or 194J) responsible for paying following sum during
the financial year:
i. any sum to any resident for carrying out any work (including supply of labour for carrying out any work) in
pursuance of a contract,
ii. any sum to any resident by way of commission (not being insurance commission referred to in section 194D)
or brokerage or
iii. any sum to any resident by way of fees for professional services
Note: Tax cannot be deducted if the aggregate amount of such sum credited or paid to a resident during the financial
year does not exceed ₹ 50 lakh
When tax shall be deducted: At the time of payment or crediting the payee, whichever is earlier.

496 The Institute of Cost Accountants of India


Total Income and Tax Liability of Individuals & HUF

Rate of TDS: 5%
Other Point: The payer is not required to obtain TAN. He shall deposit the tax deducted under this section by
using his PAN.
Exemption or relaxation from the provision
When the recipient applies to the Assessing Officer in Form 13 and gets a certificate authorising the payer to deduct
tax at lower rate or deduct no tax [Refer sec. 197]

3.6.25 Payment of certain amounts in cash [Sec. 194N]


Who is responsible to deduct tax: Every person, being,—
i. a banking company to which the Banking Regulation Act, 1949 applies (including any bank or banking
institution referred to in section 51 of that Act);
ii. a co-operative society engaged in carrying on the business of banking; or
iii. a post office,
who is responsible for paying cash (in aggregate) in excess of ₹ 1 crore (₹ 20 lakh in case of defaulter) during the
previous year, to any person from one or more accounts maintained by the recipient with it.
When tax shall be deducted: At the time of payment
Rate of TDS:

Case Rate
In case of defaulter
- Aggregate payment exceeds ₹ 20 lakh but does not exceed ₹ 1 crore 2%
- Aggregate payment exceeds ₹ 1 crore 5%
In any other case 2%
Defaulter means the recepient who has not filed the returns of income for all of the 3 assessment years relevant to
the 3 previous years, for which the time limit to file return of income u/s 139(1) has expired, immediately preceding
the previous year in which the payment of the sum is made to him
Note: In case of co-operative society ₹ 3 crore is considered instead of ₹ 1 crore.
Exception
The provision is not applicable if payment is made to:
a. the Government;
b. any banking company or co-operative society engaged in carrying on the business of banking or a post office;
c. any business correspondent of a banking company or co-operative society engaged in carrying on the business
of banking, in accordance with the guidelines issued in this regard by the Reserve Bank of India under the
Reserve Bank of India Act, 1934;
d. any white label automated teller machine operator of a banking company or co-operative society engaged in
carrying on the business of banking, in accordance with the authorisation issued by the Reserve Bank of India
under the Payment and Settlement Systems Act, 2007;
e. such other person or class of persons, which the Central Government may, by notification in the Official
Gazette, specify in consultation with the Reserve Bank of India.
f. Other notified person

The Institute of Cost Accountants of India 497


Direct Taxation

Other Point: Tax deducted u/s 194N is not considered as deemed receipt of income.

3.6.26 TDS on payment of certain sums by e-commerce operator to e-commerce participant


[194-O]
Who is responsible to deduct tax: Where sale of goods or provision of services of an e-commerce participant
is facilitated by an e-commerce operator through its digital or electronic facility or platform (by whatever name
called), such e-commerce operator shall deduct tax.
Note: If following conditions are satisfied, TDS shall not be deducted:
a. e-commerce participant is an individual or Hindu undivided family.
b. The gross amount of such sale or services or both during the previous year does not exceed ₹ 5,00,000
c. Such e-commerce participant has furnished his Permanent Account Number or Aadhaar number to the
e-commerce operator.
When tax shall be deducted: At the time of payment or crediting the payee, whichever is earlier.
Rate of TDS: 1% of the gross amount of such sales or services or both [if e-commerce participant does not intimate
his PAN to eCommerce operator, then rate of TDS is 5%]
Taxpoint: Any payment made by a purchaser of goods or recipient of services directly to an e-commerce participant
for the sale of goods or provision of services or both, facilitated by an e-commerce operator, shall be deemed to be
the amount credited or paid by the e-commerce operator to the e-commerce participant and shall be included in the
gross amount of such sale or services for the purpose of deduction of tax
Other Points
 A transaction in respect of which tax has been deducted by the e-commerce operator (or which is not liable to
deduction due to threshold limit), shall not be liable to TDS under any other provisions. However, any amount
received is or receivable by an e-commerce operator for hosting advertisements or providing any other services
which are not in connection with the sale or services, other provision relating to TDS, if any, is applicable.
 “Electronic commerce” means the supply of goods or services or both, including digital products, over digital
or electronic network;
 “e-Commerce operator” means a person who owns, operates or manages digital or electronic facility or
platform for electronic commerce;
 “e-Commerce participant” means a person resident in India selling goods or providing services or both,
including digital products, through digital or electronic facility or platform for electronic commerce;
 “Services” includes “fees for technical services” and fees for “professional services”, as defined in sec. 194J
 E-auction services carried out through an electronic portal [Circular No. 20/2021 dated 25-11-2021]
 In case of purchase of goods through the digital or electronic facility or platform, every e-commerce operator,
facilitating the sale of goods or provision of services of an e-commerce participant through its digital or
electronic facility or platform, is required to deduct tax at source u/s 194-O.
 In an e-auction, the e-auctioneer is only responsible for the price discovery, and the transaction of purchase/
sale is carried out directly by the purchase & seller. Further, the price so discovered can be negotiated between
parties without the knowledge of the e-auctioneer. Thus, the CBDT has clarified that section 194-O shall not
apply in relation to e-action activities carried out by e-auctioneers if the prescribed conditions have been
satisfied.

498 The Institute of Cost Accountants of India


Total Income and Tax Liability of Individuals & HUF

Exemption or relaxation from the provision


When the recipient applies to the Assessing Officer in Form 13 and gets a certificate authorising the payer to
deduct tax at lower rate or deduct no tax [Refer sec. 197]

3.6.27 TDS in respect of certain specified senior citizen [Sec. 194P]


Who is responsible to deduct tax: Specified Bank
When tax shall be deducted: In case of a specified senior citizen, the specified bank shall, after giving effect to the
deduction allowable under Chapter VI-A and rebate allowable u/s 87A, compute the total income of such specified
senior citizen for the relevant assessment year and deduct income-tax on such total income on the basis of the rates
in force (i.e. slab rates).
Taxpoint:
 Such specified senior citizen is not required to file his return of income for the assessment year relevant to the
previous year in which the tax has been deducted.
 Specified bank means notified banking company
 Specified senior citizen means an individual, being a resident in India:
a. who is of the age of 75 years or more at any time during the previous year;
b. who is having income of the nature of pension and no other income except the interest income received
or receivable from any account maintained by such individual in the same specified bank in which he is
receiving his pension income; and
c. has furnished a declaration to the specified bank containing such particulars, in such form and verified
in such manner, as may be prescribed.

3.6.28 TDS on certain sums for purchase of goods [Sec. 194Q]


Who is responsible to deduct tax: Any person, being a buyer who is responsible for paying any sum to any
resident seller for purchase of any goods of the value or aggregate of such value exceeding ₹ 50 lakhs in any
previous year.
 “Buyer” means a person whose total sales, gross receipts or turnover from the business carried on by him
exceed ₹ 10 crore during the financial year immediately preceding the financial year in which the purchase of
goods is carried out. However, buyer does not include certain notified person provided they satisfied specified
conditions.
When tax shall be deducted: At the time of payment or crediting the payee, whichever is earlier.
Taxpoint: Where any amount is credited to any account (for e.g. “Suspense account” or by any other name) instead
of seller account, such crediting shall be deemed to be credit of such sum to the account of the seller.
Rate of TDS: 0.1% of such sum exceeding ₹ 50 lakhs. In case where seller do not have PAN, then rate of TDS
shall be 5%
Taxpoint
 TDS u/s 194Q shall be deducted on the taxable value i.e. exclusive of GST component (or other indirect tax
component). However, on the amount paid as advance, TDS shall be deducted on entire amount since GST
component cannot be separately identified. [Circular 13/2021 dated 30-06-2021]
 The provision is not applicable where seller is Central or State Government. The exemption is not applicable
where seller is public sector undertaking or corporation. [Circular 20/2021 dated 25-11-2021]

The Institute of Cost Accountants of India 499


Direct Taxation

 Purchase Return: The tax is required to be deducted at the time of payment or credit, whichever is earlier. Thus,
before purchase return happens, the tax must have already been deducted u/s 194Q on that purchase. If that
is the case and against this purchase return the money is refunded by the seller, then this tax deducted may be
adjusted against the next purchase against the same seller. No adjustment is required if the purchase return is
replaced by the goods by the seller as in that case the purchase on which tax was deducted u/s 194Q has been
completed with goods replaced.
 The provisions of this section shall not apply to a transaction on which:
 tax is deductible under any of the provisions of this Act; and
 tax is collectible u/s 206C other than a transaction to which sec. 206C(1H) applies.
 194O -vs.- 194Q: If tax has been deducted by the e-commerce operator on a transaction u/s 194-O [including
transactions on which tax is not deducted on account of 194-O(2)], that transaction shall not be subjected to
tax deduction u/s 194Q.
 Though sec. 206C(IH) provides exemption from TCS if the buyer has deducted tax at source on goods
purchased by him, to remove difficulties it is clarified that this exemption would also cover a situation where
instead of the buyer the e-commerce operator has deducted tax at source on that transaction of sale of goods by
seller to buyer through e-commerce operator.
 194O -vs.- 206C(1H): Similarly, if a transaction is both within the purview of section 194-O as well as sec.
206C(1H), tax is required to be deducted u/s 194-O. The transaction shall come out of the purview of sec.
206C(1H) after tax has been deducted by the e-commerce operator on that transaction. Once the e-commerce
operator has deducted the tax on a transaction, the seller is not required to collect the tax u/s 206C(1H) on the
same transaction. It is clarified that here primary responsibility is on e-commerce operator to deduct the tax u/s
194-O and that responsibility cannot be condoned if the seller has collected the tax u/s 206C(1H). This is for
the reason that the rate of TDS u/s 194-O is higher than rate of TCS u/s 206(1H).
 194Q -vs.- 206C(1H): If a transaction is both within the purview of sec. 194-Q as well as sec. 206C(1H), the
tax is required to be deducted u/s 194-Q. The transaction shall come out of the purview of sec. 206C(1H) after
tax has been deducted by the buyer on that transaction. Once the buyer has deducted the tax on a transaction,
the seller is not required to collect the tax u/s 206C(1H) on the same transaction. However, if, for any reason,
tax has been collected by the seller before the buyer could deduct tax u/s 194-Q on the same transaction, such
transaction would not be subjected to tax deduction again by the buyer. This concession is provided to remove
difficulty, since tax rate of deduction and collection are same in section 194Q and sec. 206C(1H).
 The provision is not applicable in case of following transactions:
a. Transaction in securities and commodities traded through recognised stock exchange
b. Transaction in electricity, renewable energy certificate and energy saving certificate through power
exchanges [Circular 13/2021 dated 30-06-2021]

3.6.29 TDS on benefit or perquisite in respect of business or profession [Sec. 194R]New


Who is responsible to deduct tax: Any person responsible for providing to a resident, any benefit or perquisite,
whether convertible into money or not, arising from business or the exercise of a profession, by such resident.
 However, tax is not required to be deducted where provider of such facility is an individual or a Hindu undivided
family, whose total sales, gross receipts or turnover does not exceed ₹ 1 crore in case of business or ₹ 50 lakhs
in case of profession, during the financial year immediately preceding the financial year in which such benefit
or perquisite, as the case may be, is provided by such person

500 The Institute of Cost Accountants of India


Total Income and Tax Liability of Individuals & HUF

 Person responsible for providing means the person providing such benefit or perquisite, or in case of a company,
the company itself including the principal officer thereof
When tax shall be deducted: Before providing such benefit or perquisite to a resident.
Rate of TDS: 10% of the value or aggregate of value of such benefit or perquisite
When TDS is not applicable: Where the value or aggregate of value of the benefit or perquisite provided or likely
to be provided to such resident during the financial year does not exceed ₹ 20,000
Taxpoint
Where the benefit or perquisite, as the case may be, is wholly in kind or partly in cash and partly in kind but such
part in cash is not sufficient to meet the liability of deduction of tax in respect of whole of such benefit or perquisite,
the person responsible for providing such benefit or perquisite shall, before releasing the benefit or perquisite,
ensure that tax required to be deducted has been paid in respect of the benefit or perquisite

3.6.30 TDS on Payment on transfer of virtual digital asset [Sec. 194S] New
Who is responsible to deduct tax: Any person responsible for paying to any resident any sum by way of
consideration for transfer of a virtual digital asset.
When tax shall be deducted: At the time of credit of such sum to the account of the resident or at the time of
payment of such sum by any mode, whichever is earlier
Where any sum is credited to any account, whether called “Suspense Account” or by any other name, in the books
of account of the person liable to pay such sum, such credit of the sum shall be deemed to be the credit of such sum
to the account of the payee
Rate of TDS: 1%
When TDS is not applicable: No tax shall be deducted in a case, where:

Consideration is payable by Value


A specified person The value or aggregate value of such consideration does not
exceed ₹50,000 during the financial year
Any person other than a specified person The value or aggregate value of such consideration does not
exceed ₹ 10,000 during the financial year
 Specified person means a person,
a. being an individual or a HUF, whose total sales, gross receipts or turnover from the business carried on
by him or profession exercised by him does not exceed ₹ 1 crore in case of business or ₹ 50 lakh in case
of profession, during the financial year immediately preceding the financial year in which such virtual
digital asset is transferred;
b. being an individual or a Hindu undivided family, not having any income under the head “Profits and
gains of business or profession
Taxpoint
 The provisions of sections 203A and 206AB shall not apply to a specified person.
 In case of a transaction to which sec. 194O are also applicable along with the provisions of this section, then,
tax shall be deducted u/s 194S.
 Where the consideration for transfer of virtual digital asset is:

The Institute of Cost Accountants of India 501


Direct Taxation

a. wholly in kind or in exchange of another virtual digital asset, where there is no part in cash; or
b. partly in cash and partly in kind but the part in cash is not sufficient to meet the liability of deduction of
tax in respect of whole of such transfer,
the person responsible for paying such consideration shall, before releasing the consideration, ensure that
tax required to be deducted has been paid in respect of such consideration for the transfer of virtual digital
asset.

3.6.31 Other Provisions


TDS on sums payable to Government, Reserve Bank or certain corporations [Sec. 196]
No deduction of tax shall be made from any sums payable to following person:
i. the Government, or
ii. the Reserve Bank of India, or
iii. a corporation established by or under a Central Act which is, under any law for the time being in force, exempt
from income-tax on its income, or
iv. a Mutual Fund specified u/s 10(23D),
Tax deducted is income received [Sec. 198]
 All sums deducted shall, for the purpose of computing the income of an assessee, be deemed to be income
received. E.g., If Mr. X received interest of ₹ 18,000 (after deduction of tax ₹ 2,000), then interest income of
Mr. shall be considered as ₹ 20,000/-.
Exception: The sum being the tax paid, u/s 192(1A) [i.e., tax paid by employer on non-monetary perquisite]
and u/s 194N for the purpose of computing the income of an assessee, shall not be deemed to be income
received.
 Where any income is paid ‘net of tax’ (i.e., tax chargeable on such income shall be borne by the payer), then
for the purpose of deducting tax, such income is required to be grossed up [Sec. 195A]. However, grossing up
is not required in case of tax on non-monetary perquisite paid by the employer.
Credit for tax deducted [Sec. 199]
 Any deduction made in accordance with the foregoing provisions of this Chapter and paid to the Central
Government shall be treated as a payment of tax on behalf of the person from whose income the deduction
was made. If such income is assessable in the hands of other person, credit shall be given to such other person.
 Any sum referred to in sec. 192(1A) and paid to the Central Government shall be treated as the tax paid on
behalf of such employee.
 The Board may, for the purposes of giving credit in respect of tax deducted or tax paid in terms of the provisions
of this Chapter, make such rules as may be necessary, including the rules for the purposes of giving credit to
a person other than those referred above and also the assessment year for which such credit may be given.
Certificate for deduction at lower rate [Sec. 197]
 Where the Assessing Officer, on an application (Form 13) made by a person, is satisfied that existing and
estimated tax liability of a person justifies the deduction of tax at lower rate or no deduction of tax, as the case
may be, the Assessing Officer shall issue a certificate for deduction of tax at such lower rate or no deduction
of tax.

502 The Institute of Cost Accountants of India


Total Income and Tax Liability of Individuals & HUF

 The certificate shall be valid for such period of the previous year as may be specified in the certificate, unless
it is cancelled by the Assessing Officer at any time before the expiry of the specified period.
 The certificate shall be valid only with regard to the person responsible for deducting the tax and named
therein.
 The certificate shall be issued direct to the person responsible for deducting the tax under advice to the person
who made an application for issue of such certificate
Computation of lower rate [Rule 28AA]
The existing and estimated liability shall be determined by the Assessing Officer after taking into consideration the
following:
i. tax payable on estimated income of the previous year relevant to the assessment year;
ii. tax payable on the assessed or returned income, as the case may be, of the last 3 previous years;
iii. existing liability under the Income-tax Act,1961 and Wealth-tax Act,1957;
iv. advance tax payment for the assessment year relevant to the previous year till the date of making application;
v. tax deducted / collected at source for the assessment year relevant to the previous year till the date of making
application
In nutshell, the Assessing Officer may issue a certificate for deduction of tax at source at lower rate calculated in
the manner specified below:

Calculate estimated total income of the current year A


Calculate tax payable thereon B
Examine the advance tax already paid or tax already deducted for the current year C
Reduce advance tax already paid or tax already deducted from the current year estimated D=B-C
tax liability
Calculate average rate of tax E = D / A * 100
Calculate average of the average rate of tax of preceding 3 years F
Rate of TDS for Certificate Higher of E and F

No deduction to be made in certain cases [Sec. 197A]


 No deduction shall be made under any section from any payment made to New Pension System Trust referred
to in sec. 10(44).
 No deduction of tax shall be made u/s 194 (Dividend) and 194EE (NSS),
a. The payee is a resident individual;
b. The aggregate amount of income paid or credited does not exceed basic exemption limit
c. Such individual furnishes to the payer, a declaration in writing in duplicate in the Form 15G (Form 15H
in case of senior citizen).
d. Such declaration states that the tax on his estimated total income of the previous year in which such
income is to be included in computing his total income will be nil.
 No deduction of tax shall be made u/s 192A (payment of accumulated balance of provident fund) or 193
(interest on securities) or 194A (other interest) or 194D (insurance commission) or 194DA (payment in respect
of life insurance policy) or 194-I (rent),

The Institute of Cost Accountants of India 503


Direct Taxation

a. The payee is a person other than company or a firm.


b. The aggregate amount of income paid or credited does not exceed basic exemption limit.
c. Such person furnishes to the payer a declaration in writing in duplicate in the form 15G (Form 15H in
case of senior citizen).
d. Such declaration states that the tax on his estimated total income of the previous year in which such
income is to be included in computing his total income will be nil.
 No deduction of tax shall be made, or deduction of tax shall be made at such lower rate, from such payment
to such notified person or class of persons, including institution, association or body or class of institutions,
associations or bodies.

3.6.32 Duty of person responsible for deducting tax at source


Deposit of TDS within Time-Limit [Sec. 200 read with Rule 30]
Tax deducted by the deductor shall be deposited electronically through internet banking facility to the credit of the
Central Government within following time limit –

TDS Time limit


a) Tax is deducted on behalf of the Government
¾¾ Where the tax is paid without production
On the same day
of an income-tax challan
¾¾ Where the tax is paid with production of On or before 7 days from the end of the month in which
an income-tax challan tax is deducted
b. When the Assessing Officer (after obtaining prior approval from Joint Commissioner) permits quarterly
payment of tax:
Within July 7 (for Quarter ending on June 30), October
¾¾ Where deduction is made u/s 192, 194A, 7 (for Quarter ending on September 30), January 7 (for
194D or 194H Quarter ending on December 31) & April 30 (for Quarter
ending on March 31)
c. In any other case
¾¾ For the month of March Within forthcoming 30th April
¾¾ For the months other than month of March Within 7 days from the end of the month in which tax is
deducted at source.
Exception: Where tax is deducted u/s 194-IA or 194-IB, tax shall be paid to the credit of the Central Government
within a period of 30 days from the end of the month in which the deduction is made and shall be accompanied
by a challan-cum-statement in Form No. 26QB (for sec. 194-IA) / 26QC (for sec. 194-IB)
Issuance of certificate to payee
Any person responsible for deducting tax, shall require to issue a certificate (electronically generated) to the payee -

Nature of payment Certificate Time-limit


Salary 16 Annual certificate shall be issued on or before June 15 of the financial year
following the financial year in which tax is deducted.
Others 16A Quarterly certificate shall be issued within 15 days from the due date of
furnishing quarterly TDS return

504 The Institute of Cost Accountants of India


Total Income and Tax Liability of Individuals & HUF

Furnishing of statement in respect of payment of any income to residents without deduction of tax [Sec.
206A]
 Any banking company or co-operative society or public company referred to in sec. 194A(3) responsible
for paying to a resident any income not exceeding ₹ 40,000 , where the payer is a banking company or a co-
operative society, and ₹ 5,000 in any other case by way of interest (other than interest on securities), shall
prepare such statement in such form, containing such particulars, for such period, verified in such manner and
within such time, as may be prescribed, and deliver or cause to be delivered the said statement to the prescribed
income-tax authority or to the person authorised by such authority.
 The Board may require any person, other than above, responsible for paying to a resident any income liable
for deduction of tax at source under Chapter XVII, to prepare such statement in such form, containing such
particulars, for such period, verified in such manner and within such time, as may be prescribed, and deliver
or cause to be delivered the said statement to the income-tax authority or the authorised person referred above.
 The person responsible for paying to a resident any of the aforesaid income may also deliver to the income-tax
authority, a correction statement for rectification of any mistake or to add, delete or update the information
furnished in the statement delivered in such form and verified in such manner, as may be prescribed.
Duty to furnish statement
Any person after paying the tax collected to the credit of the Central Government prepare following quarterly
statements:

Particulars Form No.


Tax is deducted from salary u/s 192 24Q
Tax is deducted from payment made to non-resident, foreign company or not-ordinarily resident 27Q
In other cases 26Q
Note: The person may also deliver to the prescribed authority a correction statement for rectification of any mistake
or to add, delete or update the information furnished in the statement delivered under this sub-section in such form
and verified in such manner as may be specified by the authority.
Due date of furnishing quarterly statement

Particulars Due date of submission of quarterly return


For quarter ending on 30 June July 31 of the financial year
For quarter ending on 30 September October 31 of the financial year
For quarter ending on 31 December January 31 of the financial year
For quarter ending on 31 March May 31 of the forthcoming financial year
Processing of statements of tax deducted at source [Sec. 200A]
 The aforesaid statement (including correction statement) shall be processed in the following manner:
a. the sums deductible shall be comput­ed after making the following adjustments:
i. any arithmetical error in the statement; or
ii. an incorrect claim, apparent from any information in the statement#;
b. the interest, if any, shall be computed on the basis of the sums deductible as computed in the statement.
c. the fee, if any, shall be computed in accordance with the provisions of sec. 234E;
d. the sum payable by, or the amount of refund due to, the deductor shall be determined after adjustment

The Institute of Cost Accountants of India 505


Direct Taxation

of the amount computed under clause (b) and clause (c) against any amount paid u/s 200 or 201 or 234E
and any amount paid otherwise by way of tax or interest or fee;
e. an intimation shall be prepared or generated and sent to the deductor specifying the sum determined to
be payable by, or the amount of refund due to, him under clause (d); and
f. the amount of refund due to the deductor in pursuance of the determination under clause (d) shall be
granted to the deductor
 Time Limit: Such intimation shall not be sent after the expiry of 1 year from the end of the financial year in
which the statement is filed.
 For the purposes of processing of statements, the Board may make a scheme for centralised processing of
statements of tax deducted at source to expeditiously deter­mine the tax payable by or refundable to the deductor.
#
An incorrect claim apparent from any information in the statement shall mean a claim, on the basis of an entry,
in the statement:
i. of an item, which is inconsistent with another entry of the same or some other item in such statement;
ii. in respect of rate of deduction of tax at source, where such rate is not in accordance with the provisions
of this Act;

3.6.33 Tax deduction and collection account number [Sec. 203A]


Every person, deducting tax or collecting tax, who has not been allotted a tax-deduction account number or tax-
collection account number, shall within specified time, apply to the Assessing Officer for the allotment of a “tax-
deduction and collection-account number” in Form 49B. Where such number has been allotted to a person, such
person shall quote such number—
a. in all challans for the payment of tax deducted or collected;
b. in all certificates furnished u/s 203 or sec. 206C(5);
c. in all the statements prepared and delivered u/s 200(3) or 206C(3);
d. in all the returns delivered u/s 206 or 206C(5A) or (5B) to any income-tax authority; and
e. in all other documents pertaining to such transactions as may be prescribed in the interests of revenue.
Note: The section is not applicable to the person deducting tax u/s 194-IA or 194-IB

3.6.34 Requirement to furnish Permanent Account Number [Sec. 206AA]


 Any person entitled to receive any sum or income or amount, on which tax is deductible (hereafter referred to
as deductee) shall furnish his Permanent Account Number (PAN) to the person responsible for deducting such
tax (hereaf­ter referred to as deductor), failing which tax shall be deducted at the higher of the following rates:
i. at the rate specified in the relevant provision of this Act; or
ii. at the rate or rates in force; or
iii. at the rate of 20%
 In case of non-PAN case covered u/s 194-O or 194-Q, rate of TDS shall be 5%.
 No declaration u/s 197A shall be valid unless the person furnishes his Permanent Account Number in such
declaration.

506 The Institute of Cost Accountants of India


Total Income and Tax Liability of Individuals & HUF

 In case any declaration becomes invalid, the deductor shall deduct the tax at source accordingly.
 No certificate u/s 197 shall be granted unless the application made for the purpose contains the Permanent
Ac­count Number of the applicant.
 The deductee shall furnish his Permanent Account Number to the deductor and both shall indicate the same in
all the corre­spondence, bills, vouchers and other documents which are sent to each other.
 Where the Permanent Account Number provided to the deductor is invalid or does not belong to the deductee,
it shall be deemed that the deductee has not furnished his Permanent Account Number to the deductor.
 In the case of a non-resident, not being a company, or a foreign company and not having PAN, the provisions
of sec. 206AA shall not apply in respect of payments in the nature of interest, royalty, fees for technical
services and payments on transfer of any capital asset, if the deductee furnishes the specified details and the
documents to the deductor [Notification 53 dated 24-06-2016]

3.6.35 TDS for non-filers of ITR [Sec. 206AB] Amended


Where tax is required to be deducted at source under aforesaid provisions, other than sec. 192, 192A, 194B,
194BB, 194-IA, 194-IB, 194LBC, 194M or 194N on any sum or income or amount paid, or payable or credited,
by a person (hereafter referred to as deductee) to a specified person, the tax shall be deducted at the higher of the
following rates, namely:—
a. at twice the rate specified in the relevant provision of the Act; or
b. at twice the rate or rates in force; or
c. at the rate of 5%.
Taxpoint
 Specified person means
¾¾ a person who has not furnished the return of income for the assessment year relevant to the previous year
immediately preceding the financial year in which tax is required to be deducted, for which the time limit
for furnishing the return of income u/s 139(1) has expired; and
¾¾ the aggregate of TDS and TCS in his case is ₹ 50,000 or more in the said previous year
However, specified person shall not include a non-resident who does not have a permanent establishment
in India. Permanent establishment includes a fixed place of business through which the business of the
enterprise is wholly or partly carried on.
 If the provisions of sec. 206AA is applicable to a specified person, in addition to the provision of this section,
the tax shall be deducted at higher of the two rates provided in this section and in sec. 206AA.

3.6.36 Electronic-payment of tax [Rule 125]


The following persons shall pay tax electronically:
i. a company; and
ii. a person (other than a company), to whom provisions of section 44AB (tax audit) are applicable.
Taxpoint:
 Pay tax electronically shall mean, payment of tax by way of:
(i) internet banking facility of the authority bank; or
(ii) credit or debit cards;

The Institute of Cost Accountants of India 507


Direct Taxation

 For the purpose of this rule, tax includes interest, penalty and payment of any amount by a deductor by way of
Tax Deducted at Source (TDS) or Tax Collected at Source (TCS).
 An assessee can make electronic payment of taxes also from the account of any other person. However, the
challan for making such payment must clearly indicate the Permanent Account Number (PAN) of the assessee
on whose behalf the payment is made. It is not necessary for the assessee to make payment of taxes from his
own account in an authorized bank.

3.6.37 Direct payment [Sec. 191]


 In the case of income in respect of which:
¾¾ Provision is not made for deducting income-tax at the time of payment;
¾¾ Income-tax has not been deducted in accordance with the provisions of this Chapter,
tax shall be payable by the assessee direct.
 If any person, including the principal officer of a company,—
who is required to deduct any sum in accordance with the provisions of this Act; or
a. referred to in sec. 192(1A), being an employer,
b. does not deduct, or after so deducting fails to pay, or does not pay, the whole or any part of the tax,
as required by or under this Act, then, such person shall, without prejudice to any other consequences
which he may incur, be deemed to be an assessee in default. [Sec. 201(1)].
Exception
First proviso to sec. 201(1) states that the payer is not deemed as an assessee in default:
i. Such resident recipient has furnished his return of income u/s 139
ii. Such resident recipient has taken into account such sum for computing income in such return of income;
and
iii. Such resident recipient has paid the tax due on the income declared by him in such return of income,
iv. The payer furnishes a prescribed certificate [Form No. 26A] to this effect from a chartered accountant
 Penalty: No penalty shall be charged u/s 221 from such person, principal officer or company unless the
Assessing Officer is satisfied that such person or principal officer or company, as the case may be, has without
good and sufficient reasons failed to deduct and pay the tax. [Second Proviso to Sec. 201(1)]
 Other Point: Relaxation in case of tax on perquisite in respect of allotment of ESOP shares [Refer sec. 192]

3.6.38 Deduction only one mode of recovery [Sec. 202]


The power to recover tax by deduction under the foregoing provisions of this Chapter is only one mode for recovery
of tax. However, the Assessing Office can use any other prescribed methods of recovery in addition to TDS.

3.6.39 TDS is to be deducted on amount excluding GST component


Wherever in terms of the agreement/contract between the payer and the payee, the GST component comprised in
the amount payable to a resident is indicated separately, tax shall be deducted at source under Chapter XVII-B of
the Act on the amount paid/payable without including such GST component

508 The Institute of Cost Accountants of India


Total Income and Tax Liability of Individuals & HUF

TAX collected AT SOURCE (TCS)


Apart from TDS, another device applied for quicker collection of tax is Tax collection at source (TCS) u/s 206C.

3.6.40 Applicability of Sec. 206C


1. Every seller1, shall collect tax from the buyer2 of any specified goods3, at the time of -
¾¾ Debiting the amount payable by the buyer to the account of the buyer; or
¾¾ Receipt of such amount from the buyer,
- whichever is earlier.
2. Every person, who grants a lease or a licence or enters into a contract or otherwise transfers any right or
interest in -
¾¾ any parking lot; or
¾¾ toll plaza; or
¾¾ mine or quarry excluding mines or quarrying of mineral oil (mineral oil includes Petroleum and Natural
gas),
¾¾ to another person (other than a public sector company) for the use of such parking lot or toll plaza or mine
or quarry for the purpose of business shall collect tax from the licensee or lessee at the time of:
¾¾ Debiting the amount payable by the licensee or lessee to the account of the licensee or lessee; or
¾¾ Receipt of such amount from the licensee or lessee,
- whichever is earlier.
3. Every person -
a. being an authorised dealer, who receives an amount, for remittance out of India from a buyer, being
a person remitting such amount out of India under the Liberalised Remittance Scheme of the Reserve
Bank of India;
b. being a seller of an overseas tour program package, who receives any amount from a buyer, being the
person who purchases such package
shall collect from the buyer at the time of
¾¾ Debiting the amount payable by the buyer; or
¾¾ Receipt of such amount from the said buyer
- whichever is earlier,
Taxpoint
¾¾ Authorised dealer means a person authorised by the Reserve Bank of India u/s 10(1) of the Foreign
Exchange Management Act, 1999 to deal in foreign exchange or foreign security;
¾¾ Overseas tour programme package means any tour package which offers visit to a country or countries or
territory or territories outside India and includes expenses for travel or hotel stay or boarding or lodging
or any other expenditure of similar nature or in relation thereto.

The Institute of Cost Accountants of India 509


Direct Taxation

Exception
i. The authorised dealer shall not collect the sum, if aggregate of the amounts being remitted by a buyer is
less than ₹ 7,00,000 in a financial year and is for a purpose other than purchase of overseas tour program
package.
Taxpoint: If remittance is more than ₹ 7,00,000 (say ₹ 8,00,000), then tax shall be collected on excess
amount (i.e. ₹ 1,00,000).
ii. The authorised dealer shall not collect the sum on an amount in respect of which the sum has been
collected by the seller.
iii. The provision is not applicable, if the buyer is:
a. liable to deduct tax at source under any other provision of this Act and has deducted such amount;
b. the Central Government, a State Government, an embassy, a High Commission, a legation, a
commission, a consulate, the trade representation of a foreign State, a local authority as defined in
the Explanation to sec. 10(20) or any other notified person
4. Every person, being a seller, who receives any amount as consideration for sale of any goods of the value or
aggregate of such value exceeding ₹ 50,00,000 in any previous year shall at the time of receipt of such amount,
collect from the buyer, a sum equal to 0.1% of the sale consideration exceeding ₹ 50,00,000 [Sec. 206C(1H)]
Exception
¾¾ The provision is not applicable in case of goods being exported out of India or motor vehicle or any goods
covered in point 3 above.
¾¾ If the buyer has not provided the Permanent Account Number or the Aadhaar number to the seller, tax
shall be collected @ 1%
¾¾ The provisions shall not apply, if the buyer is liable to deduct tax at source under any other provision of
this Act on the goods purchased by him from the seller and has deducted such amount.
Taxpoint: For the purposes of this:
a. Buyer means a person who purchases any goods, but does not include:
i. the Central Government, a State Government, an embassy, a High Commission, legation, commission,
consulate and the trade representation of a foreign State; or
ii. a local authority; or
iii. a person importing goods into India or any other notified person
b. Seller means a person whose total sales, gross receipts or turnover from the business carried on by him
exceed ₹ 10 crore during the financial year immediately preceding the financial year in which the sale of
goods is carried out, but does not include notified person
Meaning of important terms [for points 1 to 3]
1. “Seller” means -
a. The Central Government; or
b. State Government; or
c. Local authority; or
d. Statutory corporation; or
e. Authority established by or under a Central, State or Provincial Act; or
f. Company; or
g. Firm; or
h. Co-operative society; or

510 The Institute of Cost Accountants of India


Total Income and Tax Liability of Individuals & HUF

i. An individual or a HUF, whose total sales, gross receipts or turnover from the business or profession
carried on by him exceed ₹ 1 crore in case of business or ₹ 50 lakh in case of profession during the
financial year immediately preceding the financial year in which such goods are sold.
2. “Buyer” (for specified goods other than motor car) means a person who obtains in any sale (by way of auction,
tender or any other mode) specified goods or the right to receive any such goods but does not include, —
i. A public sector company, the Central Government, a State Government and an embassy, a High
Commission, Legation, Commission, consulate and the trade representation, of a foreign state and a
club; or
ii. A buyer in the retail sale of such goods purchased by him for personal consumption.
 Buyer in case of motor car means a person who obtains in any sale, motor care, but does not include:
a. the Central Government, a State Government and an embassy, a High Commission, legation,
commission, consulate and the trade representation of a foreign State; or
b. a local authority; or
c. a public sector company which is engaged in the business of carrying passengers
3. “Specified goods” includes:
i. Alcoholic Liquor for human consumption
ii. Tendu leaves;
iii. Timber;
iv. Any forest-produce;
v. Scrap.
vi. Specified minerals i.e., coal, lignite and iron-ore
vii. Motor car value of which exceeds ₹ 10 lakhs
Rate of TCS
Rate as a % of the amount payable by
Particulars
the buyer or licensee or lessee*
1. Alcoholic liquor for human consumption 1%
2. Tendu leaves 5%
3. Timber obtained under a forest lease 2.5%
4. Timber obtained by any mode other than under a forest lease 2.5%
5. Any other forest produce (not being timber or tendu leaves) 2.5%
6. Scrap 1%
7. Specified minerals 1%
8. Motor car value of which exceeds ₹ 10 lakh 1%
9. Parking lot, toll plaza, mining and quarrying 2%
10. In case of point 3
a. if the amount being remitted out is a loan obtained from
any financial institution as defined in sec. 80E, for the 0.5%
purpose of pursuing any education
b. In other case 5%
11. In case of point 4 0.1%

The Institute of Cost Accountants of India 511


Direct Taxation

* However, where the purchaser or licensee or lessee is a non-resident non-corporate assessee or a non-domestic
company, then surcharge (if any applicable), health and education cess is also required to be deducted alongwith
aforesaid rates.
Note: “Scrap” means waste and scrap from the manufacture or mechanical working of materials which is definitely
not usable as such because of breakage, cutting up, wear and other reasons.

Requirement to furnish PAN by collectee [Sec. 206CC]


 Any person paying any sum, on which tax is collectible at source shall furnish his PAN to the person responsible
for collecting such tax, failing which tax shall be collected at the higher of the following rates:
i. at twice of the specified TCS rate; or
ii. at the rate of 5%.
 Where the PAN provided is invalid or does not belong to the collectee, it shall be deemed that the collectee has
not furnished his PAN to the collector.
Exception
 The provisions of higher rate shall not be applicable to a non-resident who does not have permanent
establishment in India.
 In case of point 4, if the buyer has not provided the Permanent Account Number or the Aadhaar number to the
seller, tax shall be collected @ 1%

Special provision for TCS for non-filers of ITR [Sec. 206CCA] Amended
Where tax is required to be collected at source under the aforesaid provisions, on any sum or amount received by
a person from a specified person, the tax shall be collected at the higher of the following rates:
 at twice the rate specified in the relevant provision of the Act; or
 at the rate of 5%.
Taxpoint
 If the provisions of sec. 206CC is applicable to a specified person, in addition to the provisions of this section,
the tax shall be collected at higher of the two rates provided in this section and in sec. 206CC.
 Specified person means
a. a person who has not furnished the return of income for the assessment year relevant to the previous year
immediately preceding the financial year in which tax is required to be collected, for which the time limit
for furnishing the return of income u/s 139(1) has expired; and
b. the aggregate of TDS and TCS in his case is ₹ 50,000 or more in the said previous year
However, the specified person shall not include a non-resident who does not have a permanent establishment in
India. Permanent establishment includes a fixed place of business through which the business of the enterprise
is wholly or partly carried on.

Quick MCQs:-

1. Deduction of tax from Salary u/s. 192 shall be –


(a) @ 10% of Salary
(b) At the average rate of income tax computed on the basis of rates in force for the financial year in
which the payment is made.

512 The Institute of Cost Accountants of India


Total Income and Tax Liability of Individuals & HUF

(c) At the maximum rate of 30%


(d) None of the above

2. Liability to deduct tax at source in case on income from interest on securities arises at the time of –
(a) Payment of interest
(b) Accrual of interest
(c) Credit of interest to the account of the payee / interest payable account or payment thereof whichever
is earlier.
(d) None of the above

3. The rate of TDS in case of listed debentures for the financial year 2023-24 is –
(a) 20%
(b) 21%
(c) 10%
(d) 10% but where income exceeds D 8,50,000, 10% plus surcharge @ 10%.

4. Mothi has won a state government lottery of D 1,00,000 on 11-10-2023. The state government should
deduct tax on such winning amounting to –
(a) D 30,900
(b) D 33,000
(c) D 29,070
(d) D 30,000

5. No tax is to be deducted at source if the amount credited / paid to the contractors during the relevant
previous year does not exceed –
(a) D 1,00,000
(b) D 30,000
(c) D 30,000 at one time or D 1,00,000 in aggregate in a FY.
(d) None of above

6. Mr.X a resident Indian, wins D 10,000 in a lottery. Which of the statement is true?
(a) Tax is deductible u/s. 194B @ 30%.
(b) Tax is deductible u/s. 194B @ 30.9%
(c) No tax is deductible at source
(d) None of the above

The Institute of Cost Accountants of India 513


Direct Taxation

Filing of Return of Income 3.7


3.7.1 Filing of Return
As per provisions of sec. 139(1), following persons need to file a return of income in the prescribed form and within
the prescribed time -

Section Assessee Size of income


139(1)(a) A company or a firm Irrespective of size of income (even where there is a
loss)
139(4A) Trust
139(4B) Political party
Discussed later in this chapter
139(4C) Scientific research association; News
agency; etc.
139(4D) Any University / College / other institution
referred to on Sec. 35(1)(ii) or (iii) Irrespective of size of income (even where there is a
139(4E) Business Trust loss)
139(4F) Investment Fund referred to in sec. 115UB
139(1)(b) Any other person Where income before giving effect to sec. 54, 54B,
54D, 54EC, 54F, 54G, 54GA, 54GB and chapter VIA
(i.e., deduction u/s 80C to 80U) exceeds the maximum
amount which is not chargeable to income tax.
Compulsory filing of return: Any person, being resident other than not ordinarily resident, shall furnish, a
return, within due date, in respect of his income or loss for the previous year irrespective of the fact that his total
income does not exceed basic exemption limit or does not have any taxable income, if he:
(i) holds, as a beneficial owner or otherwise, any asset (including any financial interest in any entity) located
outside India or has signing authority in any account located outside India; or
(ii) is a beneficiary of any asset (including any financial interest in any entity) located outside India.
Exception: An individual, being a beneficiary of any asset (including any financial interest in any entity) located
outside India where, income, if any, arising from such asset is includible in the income of the person referred
above in accordance with the provisions of this Act.
 “Beneficial owner” in respect of an asset means an individual who has provided, directly or indirectly,
consideration for the asset for the immediate or future benefit, direct or indirect, of himself or any other
person.
 “Beneficiary” in respect of an asset means an individual who derives benefit from the asset during the previous
year and the consideration for such asset has been provided by any person other than such beneficiary.

514 The Institute of Cost Accountants of India


Total Income and Tax Liability of Individuals & HUF

Mandatory furnishing of return in case of high value transactions [7th proviso to sec. 139(1)]
A person (other than firm and company), who is not required to furnish a return as per aforesaid provision, and who
during the previous year:
a. has deposited an aggregate amount exceeding ₹ 1 crore in one or more current accounts maintained with a
banking company or a co-operative bank; or
b. has incurred expenditure of an aggregate amounts exceeding ₹ 2 lakh for himself or any other person for travel
to a foreign country; or
c. has incurred expenditure of an aggregate amount exceeding ₹ 1 lakh towards consumption of electricity; or
d. fulfils such other conditions as may be prescribed,

The following are prescribed [Rule 12AB]:


 If his total sales, turnover or gross receipts, as the case may be, in the business exceeds ₹60 lakhs during the
previous year; or
 If his total gross receipts in profession exceeds ₹ 10 lakh during the previous year; or
 If the aggregate of TDS and TCS during the previous year, in the case of the person, is ₹ 25,000 or more (in
case of senior citizen ₹ 50,000); or
 The deposit in one or more savings bank account of the person, in aggregate, is ₹ 50 lakh or more during the
previous year
shall furnish a return of his income on or before the due date in such form and verified in such manner and setting
forth such other particulars, as may be prescribed.

Illustration 44:
Whether following assessee is compulsorily required to file return of income–

Assessee Taxable Income before deduction Required to file return or not


Mr. X 5,00,000 Yes
Mr. Y 30,000 No*
A Ltd. 10,000 Yes
B Ltd (-) 20,000 Yes
* Assessee can file a return of income voluntarily irrespective of it’s size of income.

3.7.2 Scheme for submission of return through Tax Return Preparers (TRP) [Sec. 139B]
A TRP is an individual who has been authorized to enable any specified class(es) of person1 to prepare and furnish
their returns of income. The scheme framed under the above provision shall specify:
 The manner in which the TRP shall assist the person furnishing the return of income
 The educational and other qualifications to be possessed,
 The training and other conditions required to be fulfilled, by a person to act as a TRP,
 The code of conduct for the TRP,
 Duties and obligation of the TRP

The Institute of Cost Accountants of India 515


Direct Taxation

 The manner in which authorization may be withdrawn; and


 Any other matter.
Note: The TRP shall also affix his signature on such return.
1
Specified class(es)of person means resident individual and resident HUF other than person whose accounts are
required to be audited.
Person not eligible to become TRP
 Chartered Accountant
 Any legal practitioner who is entitled to practice in any civil court in India.
 Any officer of a scheduled bank cannot be the TRP of the assessee who maintains a Current account or has
other regular dealing with such bank.
Educational Qualification of TRP
An individual, who holds a bachelor degree from a recognised Indian University or institution, or has passed the
intermediate level examination conducted by the Institute of Chartered Accountants of India or the Institute of
Company Secretaries of India or the Institute of Cost Accountants of India, shall be eligible to act as Tax Return
Preparer

3.7.3 Forms – Return of income


Rule 12 provides following Form for filing return of income for different assessee:

For Individuals having Income from Salaries, one house property (does not have any brought forward
loss), other sources [Interest (does not have any loss under the head) etc. but except winnings from
lottery or income from race horses] and having total income upto ₹ 50 lakh
However, the form is not to be used by an individual who:
a. has any brought forward / carry forward loss under the head ‘Income from House Property’;
b. has assets (including financial interest in any entity) located outside India;
c. has signing authority in any account located outside India;
d. has income from any source outside India;

ITR - 1 e. has income to be apportioned in accordance with provisions of section 5A

(Sahaj) f. has claimed deduction u/s 57, other than deduction from family pension;
g. is a director in any company;
h. has held any unlisted equity share at any time during the previous year;
i. is assessable for the whole or any part of the income on which tax has been deducted at source in
the hands of a person other than the assessee;
j. has claimed any relief of tax u/s 90 or 90A or 91;
k. has agricultural income, exceeding ₹ 5,000;
l. has total income, exceeding ₹ 50 lakh;
m. has income of the nature referred to in section 115BBE;

516 The Institute of Cost Accountants of India


Total Income and Tax Liability of Individuals & HUF

n. is a person in whose case tax has been deducted u/s 194N; or


o. is a person in whose case payment or deduction of tax has been deferred u/s 191(2) or 192(1C)
p. has to furnish return under 7th proviso to sec. 139(1)
q. has income under the head “Capital Gains” and / or “Profits and Gains of Business or Profession”;
or
r. has loss under the head “Income from Other Sources”.
ITR - 2 For Individuals and HUFs not carrying out business or profession under any proprietorship
ITR - 3 For individuals and HUFs having income from a proprietary business or profession
In the case of a person being an individual (ordinarily resident) or a HUF (ordinarily resident) or
a resident firm (other than LLP), deriving income under the head “Profits or gains of business or
profession” and such income is computed in accordance sec. 44AD, 44ADA and 44AE
Taxpoint: The form is applicable to an assessee computing his business or profession income u/s
44AD, 44ADA and 44E.
The form is not applicable to a person who:
a. has assets (including financial interest in any entity) located outside India;
b. has signing authority in any account located outside India;
c. has income from any source outside India;
d. has income to be apportioned in accordance with provisions of sec. 5A;
e. is a director in any company;
ITR- 4
f. has held any unlisted equity share at any time during the previous year;
(Sugam)
g. has total income, exceeding ₹ 50 lakh;
h. owns more than one house property, the income of which is chargeable under the head “Income
from house property”;
i. has any brought forward loss or loss to be carried forward under any head of income;
j. is assessable for the whole or any part of the income on which tax has been deducted at source in
the hands of a person other than the assessee;
k. has claimed any relief u/s 90 or 90A or deduction of tax u/s 91;
l. has agricultural income, exceeding ₹ 5,000;
m. has income of the nature referred to in sec. 115BBE;
n. has income of the nature specified in sec. 17(2)(vi) on which tax is payable or deductible, as the
case may be, u/s 191(2) or 192(1C).
ITR - 5 For person other than (i) Individual; (ii) HUF; (iii) Company; & (iv) Person filing Form ITR-7
ITR - 6 For Companies other than companies claiming exemption u/s 11
For persons including companies required to furnish return u/s 139(4A) or 139(4B) or 139(4C) or
ITR - 7
139(4D) or 139(4F)
Income Tax Return Verification Form [Where the data of the aforesaid Return of Income has
ITR - V
transmitted electronically without digital signature]

The Institute of Cost Accountants of India 517


Direct Taxation

3.7.4 Mode of furnishing Income-tax Return


a. Paper Return
b. Electronic Return with Digital Signature
c. Transmitting the data in the return electronically and thereafter submitting the verification of the return in
Form ITR-V
d. Transmitting the data electronically in the return under electronic verification code
Notes
1. Compulsory E-Return:
Person Condition Mode
Company -
Political Party - Electronically with
Firm or LLP or Individual Audit u/s 44AB required digital sign
or HUF
Individual Where total income assessable during the previous year Any of the given
of a person, being an individual of the age of 80 years mode
or more at any time during the previous year, and who
furnishes the return in Form number SAHAJ (ITR-1) or
Form number SUGAM (ITR-4)
Any other person Any mode other than
paper mode
2. A resident Individual (other than not-ordinarily resident) or a resident HUF (other than not-ordinarily resident)
must file the return of income electronically (with or without digital sign) if he/it has:
a. assets (including financial interest in any entity) located outside India; or
b. signing authority in any account located outside India.
3. Further, any person who has claimed any relief u/s 90 or 90A or 91, is required to file return electronically.
4. Where an assessee is required to furnish a report of audit specified u/s 10(23C)(iv), (v), (vi) or (via), sec. 10A,
sec. 12A(1)(b), sec. 44AB, sec. 80-IA, sec. 80-IB, sec. 80-IC, sec. 80-ID, sec. 80JJAA, sec. 80LA, sec. 92E or
sec. 115JB, he shall furnish the same electronically. Apart from said report, the return shall paper less.

3.7.5 Time limit for filing return of income [Explanation 2 to Sec. 139(1)]
A return should be filed on or before the following due date (of respective assessment year):

Assessee Due date


¾¾ Where the assessee (including the partners of the firm) is required to furnish a report in 30 November
th

Form 3CEB u/s 92E pertaining to international transaction(s)


¾¾ Where the assessee is a partner11 in a firm and the said firm is required to furnish report 30th November
in Form 3CEB u/s 92E pertaining to international transaction(s)
¾¾ Where the assessee is a company not having international transaction(s) 31st October
¾¾ Any other assessee

11
Also spouse of such partner if the provisions of section 5A applies to such spouse

518 The Institute of Cost Accountants of India


Total Income and Tax Liability of Individuals & HUF

Assessee Due date


− Where accounts of the assessee are required to be audited under any law 31st October
− Where the assessee is a partner1 in a firm and the accounts of the firm are required 31st October
to be audited under any law
− In any other case 31st July

3.7.6 Fee for default in furnishing return of income [Sec. 234F]


Where a person required to furnish a return of income u/s 139, fails to do so within the due date, he shall pay fee of:

Case Fee
Total income does not exceed ₹ 5 lakh ₹ 1,000
Total income exceeds ₹ 5 lakh ₹ 5,000
3.7.7 When a return of loss should be filed [Sec. 139(3)]
An assessee, other than few, is not compulsorily required to furnish return of loss. However, the following losses
cannot be carried forward if the return of loss is not submitted within the time allowed u/s 139(1) -
a. Business loss (speculative or otherwise);
b. Capital loss;
c. Loss from the activity of owning and maintaining race horses
d. Loss from business specified u/s 35AD
Notes
a. Loss declared in belated return cannot be carried forward. However, set-off of losses of current year is not
prohibited while computing the total income, even if the return of loss is filed after the due date.
b. Delay in filing the return of loss may be condoned in certain cases
c. Unabsorbed depreciation u/s 32 and loss under the head “Income from house property” can be carried forward
even if the loss return is filed after the due date u/s 139(1).
d. Although the loss of the current year cannot be carried forward unless the return of loss is submitted before
the due date but the loss of earlier years can be carried forward if the return of loss of that year was submitted
within the due date.

3.7.8 Belated Return [Sec. 139(4)]


If an assessee fails to file return within the time limit allowed u/s 139(1) or within the time allowed under a notice
issued u/s 142(1), he can file a belated return.
Time limit: Assessee may file such return -
 On or before 31st December of the relevant assessment year; or
 before the completion of assessment (u/s 144),
- whichever is earlier.
However, if an assessee files a belated return, he would be liable to fee u/s 234F and interest u/s 234A.

The Institute of Cost Accountants of India 519


Direct Taxation

Illustration 45:
State the time allowed u/s 139(4) to submit a belated return for the P.Y. 2023-24 in the following cases -
a. No assessment is made u/s 144.
b. Assessee failed to respond to notice u/s 142(1), served on 5/11/2024 (time allowed in such notice to submit
return is 5/12/2024) but no assessment is made u/s 144.
c. Assessee failed to respond to notice u/s 142(1), served on 5/10/2024 (time allowed in such notice to submit
return is 5/11/2024) and assessment u/s 144 is completed on 7/12/2024.
d. Assessee failed to respond to notice u/s 142(1), served on 5/10/2024 (time allowed in such notice to submit
return is 5/11/2024) and assessment u/s 144 is completed on 7/12/2024. However, assessee has received the
assessment order on 5/1/2025.
Solution :

a. 31/12/2024 b. 31/12/2024 c. 7/12/2024 d. 7/12/2024


3.7.9 Return of income of Charitable Trust [Sec. 139(4A)]
Every person who is in receipt of –
 income from property held under the trust or other legal obligation wholly or partly for charitable or religious
purpose; or
 income by way of voluntary contribution on behalf of such trust or institution,
and if such income before allowing exemption u/s 11 or 12 exceeds the maximum amount which is not chargeable
to tax, must file a return before the due date as per sec.139(1).
Penalty: Where an assessee fails to file return of income under this section, within the time limit, it shall be liable
to pay a penalty of ₹ 100 per day during which such failure continues [Sec. 272A(2)].

3.7.10 Return of income of Political Party [Sec. 139(4B)]


The chief executive officer (whether such chief executive officer is known as Secretary or by any other designation)
of any political party is required to furnish a return in respect of income of such political party, if the amount of
gross total income before allowing exemption u/s 13A exceeds the maximum amount not chargeable to tax.

3.7.11 Return of income of scientific research association, etc. [Sec. 139(4C)]


Every -
 Research Association referred to in sec. 10(21);
 News agency referred to in sec. 10(22B);
 Association or institution referred to in sec. 10(23A) or sec. 10(23B);
 Specified Employee Welfare Fund referred to in sec. 10(23AAA);
 Any university or other educational institution referred to in sec. 10(23C)(iiiad) or (iiiab);
 Any hospital or other medical institution referred to in sec. 10(23C)(iiiae) or (iiiac);
 Fund or institution referred to in sec. 10(23C)(iv);
 Trust or institution referred to in sec. 10(23C)(v);
 Any university or other educational institution referred to in sec. 10(23C)(vi);

520 The Institute of Cost Accountants of India


Total Income and Tax Liability of Individuals & HUF

 Any hospital or other medical institution referred to in sec. 10(23C)(via);


 Mutual Fund referred to in sec. 10(23D);
 Securitisation trust referred to in sec. 10(23DA);
 Investor Protection Fund referred to in sec. 10(23EC) or sec. 10(23ED);
 Core Settlement Guarantee Fund referred to in sec. 10(23EE);
 Venture Capital Company or Venture Capital Fund referred to in sec. 10(23FB);
 Trade union or an association of such union referred to in sec. 10(24);
 Body or authority or Board or Trust or Commission referred to in sec. 10(46) or 10(29A);
 Infrastructure debt fund referred to in sec. 10(47),
must file a return, if the total income without giving effect to the provisions of sec. 10, exceeds the maximum
amount which is not chargeable to income-tax.
Penalty: Where an assessee fails to file return of income under this section, within the time limit, it shall be liable
to pay a penalty of ₹ 100 per day during which such failure continues [Sec. 272A(2)].

3.7.12 Return of income by a University/ College etc. [Sec. 139(4D)]


Every University, college or other institutions referred to in sec. 35(1)(ii) or (iii) is required to furnish a return in
respect of income or loss irrespective of size of income or loss.

3.7.13 Return of income of a Business Trust [Sec. 139(4E)]


Every business trust, which is not required to furnish return of income or loss under any other provisions of this
section, shall furnish the return of its income in respect of its income or loss in every previous year and all the
provisions of this Act shall, so far as may be, apply if it were a return required to be furnished u/s 139(1).

3.7.14 Return of income of Investment Fund [Sec. 139(4F)]


Every investment fund referred to in sec. 115UB, which is not required to furnish return of income or loss under
any other provisions of this section, shall furnish the return of income in respect of its income or loss in every
previous year and all the provisions of this Act shall, so far as may be, apply as if it were a return required to be
furnished u/s 139(1)

3.7.15 Revised Return [Sec. 139(5)]


If an assessee discovers any omission or wrong statement (bonafide in nature) in the return filed, he can revise his
return u/s 139(5).
Time limit: Assessee may file the revised return -
 On or before 31st December of the relevant assessment year; or
 before completion of regular assessment,
- whichever is earlier.
Notes
a. Replacement of original return: Once a revised return is filed, it replaces the earlier return. This signifies that
the revised return should be complete in itself and not merely an accessory to the original return.
b. Revision of revised return: A revised return can again be revised i.e. a second revised return can be filed u/s
139(5) for correcting any omission or wrong statement made in the first revised return within specified time.

The Institute of Cost Accountants of India 521


Direct Taxation

c. Revision of belated return: A belated return u/s 139(4) can be revised.


d. Revision of loss return: A loss return can be revised
e. Return filed pursuant to notice u/s 142(1) cannot be revised.

Note: Updated return of income [Sec 139(8A)] can be filed within 24 months from the end of relevant assessment
year whether original return of income is filed or not.

3.7.16 Defective Return [Sec. 139(9)]


When a return is termed defective - A return of income is said to be defective where all the following conditions
are not fulfilled:
 The return is furnished without paying self-assessment tax along with interest, if any.
 The annexure, statements and columns in the return of income have been duly filled in.
 The return is accompanied by the following documents -
a. a statement showing the computation of tax liability;
b. the audit report u/s 44AB (where the report has been submitted prior to the furnishing of return, a copy
of audit report together with proof of furnishing the report);
c. the proof of tax deducted or collected at source, advance tax paid and tax paid on self-assessment;
d. where regular books of account are maintained by the assessee:
i. copies of Manufacturing A/c, Trading A/c, Profit and Loss A/c or Income and Expenditure A/c or
any other similar account and Balance Sheet;
ii. in the case of –
¾¾ A proprietary business or profession - the personal account of the proprietor;
¾¾ A firm, AOP or BOI - personal account of the partners or members; or
¾¾ A partner or member of the firm, AOP or BOI - his personal account in the firm, association
of persons or body of individuals;
where regular books of account are not maintained by the assessee –
e. where regular books of account are not maintained by the assessee:
i. a statement indicating the amount of turnover or gross receipts, gross profit, expenses and net profit
of the business or profession and the basis on which such amount have been computed; and
ii. the amount of sundry debtors, sundry creditors, stock and cash balance as at the end of the previous
year.
f. where the accounts of the assessee have been audited, copies of the audited Profit and Loss A/c, Balance
Sheet and a copy of the Auditor’s report;
g. Cost audit report u/s 233B of the Companies Act, 1956 (if any).
h. Where an updated return is furnished u/s 139(8A) then the return is accompanied by the proof of payment
of tax as required u/s 140B
Effect: Where the Assessing Officer considers that the return of income furnished by the taxpayer is defective, he
may intimate the defect to the taxpayer and give him an opportunity to rectify the defect(s).
Time limit for rectification: The assessee must rectify the error within a period of 15 days from the date of
intimation (served on the assessee) or within such extended time as allowed by the Assessing Officer. Where

522 The Institute of Cost Accountants of India


Total Income and Tax Liability of Individuals & HUF

the taxpayer rectifies the defect after the expiry of the period of 15 days or such extended period but before the
assessment is completed, the Assessing Officer can condone such delay.
Consequence when defect is not rectified: If defect is not rectified within the time limit, the Assessing Officer
will treat the return as an invalid return and provisions of the Act will apply as if the taxpayer had failed to furnish
the return at all.
Note: Currently, the assessee is required to furnish paper-less return. i.e., no documents, proof or report (other than
some specified report required to be furnished electronically) is required to be attached with return of income. In
this regard, return of income shall not be considered as defective return. However, the assessee should retain these
documents, proof or report with himself. If called for by the income-tax authority during any proceeding, it shall
be incumbent upon the assessee to furnish/produce the same.

3.7.17 Verification of Return [Sec. 140]


The return of income is required to be verified:

Assessee Case Verified by


Individual In general Individual himself
Where the individual concerned is absent Individual himself or by the duly authorized
from India person of such individual
Where the individual is mentally Guardian of such individual or any other person
incapacitated competent to act on his behalf
Where by any other reason it is not possible Any person duly authorised by him
for the individual to verify the return.
Note: When return is verified by any authorised person in that case the return should be
accompanied with power of attorney.
HUF In general Karta
Where the ‘karta’ is absent from India or is Any adult member of the family.
mentally incapacitated
Firm In general Managing partner
If due to any reason it is not possible for Any adult partner
managing partner to verify or where there is
no managing partner
Limited In general Designated partner
liability If due to any unavoidable reason such Any partner or any other prescribed person
partnership designated partner is not able to verify
the return, or where there is no designated
partner as such
Principal Officer
Local authority
Political party Chief Executive Officer

The Institute of Cost Accountants of India 523


Direct Taxation

Assessee Case Verified by


Company In general Managing Director (MD)
If due to any reason it is not possible for MD Any director or any other prescribed person
to verify or where there is no MD
Where an application for corporate insolvency Insolvency professional appointed by such
resolution process has been admitted by the Adjudicating Authority
Adjudicating Authority under Insolvency
and Bankruptcy Code, 2016
Non-resident company A person holding a valid power of attorney.
Copy of such power of attorney must be
attached with the return.
Company in process of winding up Liquidator of the company
Where the management of the company Principal officer
has been taken over by the Central or State
Government.
Any other Any member or principal officer
association
Any other Such person or any other person competent to act on its behalf.
person

3.7.18 Updated Return [Sec. 139(8A)]New


Who can file updated return: Any person, whether or not he has furnished a return u/s 139(1) or (4) or (5), for
an assessment year, may furnish an updated return of his income or the income of any other person in respect of
which he is assessable under this Act, for the previous year relevant to such assessment year, in the prescribed form,
verified in such manner and setting forth such particulars as may be prescribed.
Taxpoint
 An updated return cannot be filed, if the updated return:
a. is a return of a loss; or
b. has the effect of decreasing the total tax liability determined on the basis of return already furnished
u/s 139(1) or (4) or (5) [in nutshell, a person cannot reduce his tax liability by filing updated return]; or
c. results in refund or increases the refund due on the basis of return furnished u/s 139(1) or (4) or (5), of
such person for the relevant assessment year.
 A person shall not be eligible to furnish an updated return, where:
a. a search has been initiated u/s 132 or books of account or other documents or any assets are requisitioned
u/s 132A in the case of such person; or
b. a survey has been conducted u/s 133A [other than sec. 133A(2A)], in the case such person; or
c. a notice has been issued to the effect that any money, bullion, jewellery or valuable article or thing,
seized or requisitioned u/s 132 or 132A in the case of any other person belongs to such person; or
d. a notice has been issued to the effect that any books of account or documents, seized or requisitioned u/s
132 or 132A in the case of any other person, pertain or pertains to, or any other information contained
therein, relate to, such person, for the assessment year relevant to the previous year in which such
search is initiated or survey is conducted or requisition is made and any assessment year preceding such
assessment year.

524 The Institute of Cost Accountants of India


Total Income and Tax Liability of Individuals & HUF

 No updated return shall be furnished by any person for the relevant assessment year, where:
a. an updated return has already been furnished by him for the relevant assessment year; or
b. any proceeding for assessment or reassessment or recomputation or revision of income is pending or has
been completed for the relevant assessment year in his case; or
c. the Assessing Officer has information in respect of such person for the relevant assessment year in his
possession under the Smugglers and Foreign Exchange Manipulators (Forfeiture of Property) Act, 1976
or the Prohibition of Benami Property Transactions Act, 1988 or the Prevention of Money-laundering
Act, 2002 or the Black Money (Undisclosed Foreign Income and Assets) and Imposition of Tax Act,
2015 and the same has been communicated to him, prior to the date of furnishing of return under this
sub-section; or
d. information for the relevant assessment year has been received under an agreement referred to in sec.
90 or 90A in respect of such person and the same has been communicated to him, prior to the date of
furnishing of return under this sub-section; or
e. any prosecution proceedings under the Chapter XXII have been initiated for the relevant assessment
year in respect of such person, prior to the date of furnishing of return; or
f. he is such person or belongs to such class of persons, as may be notified by the Board in this regard.
 If any person has sustained a loss in any previous year and has furnished a return of loss in the prescribed from
within the time allowed u/s 139(1) and verified in the prescribed manner and containing such other particulars
as may be prescribed, he shall be allowed to furnish an updated return where such updated return is a return
of income.
zz If the loss or any part thereof carried forward or unabsorbed depreciation carried forward or tax credit
carried forward u/s 115JAA or u/s 115JD is to be reduced for any subsequent previous year as a result
of furnishing of updated return for a previous year, an updated return shall be furnished for each such
subsequent previous year.
Time limit for filing updated return: At any time within 24 months from the end of the relevant assessment year.
Taxpoint: The updated return is accompanied by the proof of payment of tax as required u/s 140B

3.7.19 Tax on updated return [Sec. 140B] New


A. Where assessee has not furnished return earlier
Where no return of income u/s 139(1) or (4) has been furnished by an assessee and tax is payable, on the basis
of updated return to be furnished by such assessee, after taking into account:
i. the amount of tax, if any, already paid as advance tax;
ii. TDS or TCS;
iii. any relief u/s 89;
iv. any relief u/s 90 or 91 on account of tax paid in a country outside India;
v. any relief u/s 90A on account of tax paid in any specified territory outside India; and
vi. any tax credit claimed to be set off in accordance with the provisions of sec. 115JAA or 115JD, the
assessee shall be liable to pay such tax together with interest and fee payable for any delay in furnishing
the return or any default or delay in payment of advance tax, along with the payment of additional
income-tax, before furnishing the return and the return shall be accompanied by proof of payment of
such tax, additional income-tax, interest and fee.

The Institute of Cost Accountants of India 525


Direct Taxation

B. Where assessee has furnished return earlier


Where, return of income u/s 139(1) or (4) or (5) (referred to as earlier return) has been furnished by an assessee
and tax is payable on the basis of updated return to be furnished by such assessee
a. after taking into account:
i. the amount of relief or tax referred to in sec. 140A(1), the credit for which has been taken in the
earlier return;
ii. TDS or TCS on any income which is taken into account in computing total income and which has
not been included in the earlier return;
iii. any relief claimed u/s 90 or 91 on account of tax paid in a country outside India on such income
which has not been included in the earlier return;
iv. any relief claimed u/s 90A on account of tax paid in any specified territory outside India on such
income which has not been included in the earlier return;
v. any tax credit claimed, to be set off in accordance with the provisions of sec. 115JAA or 115JD,
which has not been claimed in the earlier return; and
b. as increased by the amount of refund, if any, issued in respect of such earlier return, the assessee shall
be liable to pay such tax together with interest for any default or delay in payment of advance tax along
with the payment of additional income-tax as reduced by the amount of interest paid in the earlier return,
before furnishing the return and the return shall be accompanied by proof of payment of such tax,
additional income-tax, interest and fee.
Computation of Additional Income Tax
The additional income-tax payable at the time of furnishing the updated return shall be:

if such return is furnished after expiry of the time available u/s 139(4) 25% of aggregate of tax (+ surcharge
or (5) and before completion of the period of 12 months from the end + cess) and interest payable on tax
of the relevant assessment year calculated above
if such return is furnished after the expiry of 12 months from the end of 50% of aggregate of tax (+ surcharge
the relevant assessment year but before completion of the period of 24 + cess) and interest payable on tax
months from the end of the relevant assessment year calculated above

Quick MCQs:-
1. Mr. Z, a salaried individual, has a total income of D 8 lakhs for A.Y 2024-25. He furnishes his return of
income for A.Y 2024-25 on 28th August, 2024. He is liable to pay fee of –
(a) upto D 1,000 under section 234F
(b) D 5,000 under section 234F
(c) D 10,000 under section 234F
(d) Not liable to pay any fee
2. Which of the following returns can be revised under Section 139(5)?
(i) A return of income filed u/s. 139(1)
(ii) A belated return of income filed u/s. 139(4)
(iii) A return of loss filed u/s. 139(3)
(a) Only (i)
(b) Only (i) and (ii)
(c) Only (i) and (iii)
(d) (i), (ii) and (iii)

526 The Institute of Cost Accountants of India


Total Income and Tax Liability of Individuals & HUF

PAN 3.8
Permanent Account Number (PAN) is an alpha-numeric (ten characters) code given to a person by income tax
department for the purpose of identification of the assessee. A person can have only one PAN.

3.8.1 Allotment of PAN


A. On Application
Compulsory application for allotment of PAN
As per sec. 139A & rule 114, following persons are under statutory obligation to apply for PAN within the time
limit stated as under:

Who is to apply for PAN When to apply for PAN


Any person whose total income exceeds maximum exempted limit.
Any resident person other than an individual, which enters into
a financial transaction of an amount aggregating to ₹ 2,50,000 or
more in a financial year
On or before 31st May of the relevant
Any person who is the managing director, director, partner, trustee, assessment year.
author, founder, karta, chief executive officer, principal officer or
office bearer of the person referred above or any person competent
to act on behalf of the person referred above or who intends to enter
into prescribed transactions
Any person whose sales or turnover or gross receipts are likely to
exceed ₹ 5,00,000 in any previous year
Any person who is required to furnish return u/s 139(4A) i.e. trust On or before the end of the relevant
and charitable institution financial year
Any person who is entitled to receive any sum or income, on which
tax is deductible under in any financial year
Any person who requires export-import code Before making any export or import.
Before making application for
Assessee under the GST
registration under GST.
7 days before entering into such
Any person intends to enter into specified transactions12
transactions

12
Following are specified transactions (further PAN or Aadhar number is required to be quote on any document pertaining to such transactions)
1. Cash deposit or deposits aggregating to ₹ 20 lakh or more in a financial year, in one or more account of a person with a banking company or a co-
operative bank to which the Banking Regulation Act, 1949 applies or a Post Office;
2. Cash withdrawal or withdrawals aggregating to ₹ 20 lakh or more in a financial year, in one or more account of a person with a banking company
or a co-operative bank to which the Banking Regulation Act, 1949 applies or a Post Office;
3. Opening of a current account or cash credit account by a person with a banking company or a co-operative bank to which the Banking Regulation
Act, 1949 applies or a Post Office

The Institute of Cost Accountants of India 527


Direct Taxation

Penalty for failure to apply for PAN


Failure to apply for PAN or to quote PAN in prescribed documents (discussed later in this chapter) attracts penalty
of ₹ 10,000 u/s 272B.
Voluntary application for allotment of PAN [Sec. 139A(3)]
The section empowers a person to apply for a PAN, even though, the person does not fall under any of the categories
as mentioned above. However, a person, who has been already allotted a PAN under new series, shall not apply
for another PAN.
Interchangeability of PAN and Aadhar [Sec. 139A(5E)]
Every person who is required to furnish or intimate or quote his PAN, and who,––
a. has not been allotted PAN but possesses the Aadhaar number, may furnish or intimate or quote his Aadhaar
number in lieu of the PAN, and such person shall be allotted a PAN in such manner as may be prescribed;
b. has been allotted a PAN, and who has intimated his Aadhaar number in accordance with provisions of sec.
139AA, may furnish or intimate or quote his Aadhaar number in lieu of the PAN.

Whom to apply (either compulsory or voluntary)


The application is to be made to the Assessing Officer or any other person who has been assigned the function of
allotment of PAN. However, where no such Assessing Officer or other person has been assigned such function then
application shall be made to the Assessing Officer having jurisdiction over the person.
Form of application13 – The prescribed forms are as under:

Case Form
For Indian Citizen / Indian Company / Entities incorporated in India / Unincorporated entities formed 49A
in India
In other cases 49AA
b. Suo-moto allotment of PAN
¾¾ As per sec 139A(1B) for the purpose of collecting any information which may be useful for the purposes
of the Act, the Central Government may, by way of notification, specify any class of person to apply
within prescribed time to the AO for allotment of PAN.
¾¾ Section 139A(2) empowers the Assessing Officer to allot a PAN to any person other than the person
falling under the categories mentioned above.

3.8.2 Importance of PAN


A. PAN or Aadhar Number must be quoted in all documents and challans [Sec. 139A(5)]
a. A person to whom a PAN or Aadhar is allotted, is required to quote that number in -
 All his returns to; or
 Any correspondence with; or
 Any other documents to,
Income-tax authority.
b. A person to whom a PAN or Aadhar is allotted, is required to quote that number in challans for payment
of any sum due under this Act.

13
Specified applicant may apply for allotment of PAN through a common application form notified by the Central Government

528 The Institute of Cost Accountants of India


Total Income and Tax Liability of Individuals & HUF

B. PAN or Aadhar Number must be quoted in documents pertaining to certain prescribed transactions
[Sec. 139A(5)(c) & Rule 114B]
Every person shall quote its PAN or Aadhar in all documents pertaining to following transactions entered into
by him –
1. *Transactions relating to sale or purchase of a motor vehicle (other than two wheeled vehicles), which
requires registration.
2. *Opening an account [other than a time-deposit and a Basic Savings Bank Deposit Account] with a
banking company or a co-operative bank
3. Making application for issue of a credit card or debit card.
4. *Opening of a demat account
5. Payment in cash exceeding ₹ 50,000 to a hotel or restaurant against a bill or bills at any one time
6. Payment in cash exceeding ₹ 50,000 in connection with travel to any foreign country or payment for
purchase of any foreign currency at any one time
7. *Payment exceeding ₹ 50,000 to any mutual fund for purchase of its units.
8. *Payment exceeding ₹ 50,000 to a company or an institution for acquiring debentures or bonds issued
by it
9. Payment exceeding ₹ 50,000 to RBI for acquiring bonds issued by it.
10. *Deposit in cash exceeding ₹ 50,000 during any one day with a banking company or a co-operative bank
 As per Rule 114BB, PAN or aadhar is required to quote in case of cash withdrawal or withdrawals
aggregating to ₹ 20 lakh or more in a financial year, in one or more account of a person with a
banking company or a co-operative bank to which the Banking Regulation Act, 1949 applies or a
Post Office
11. Purchase of bank drafts or pay orders or banker’s cheques from a banking company or a co-operative
bank in cash for an amount exceeding ₹ 50,000 during any one day
12. *A time deposit of an amount exceeding ₹ 50,000 or aggregating to more than ₹ 5 lakh during a financial
year with: (i) a banking company or a co-operative bank; or (ii) a Post Office; or (iii) a Nidhi referred to
in section 406 of the Companies Act, 2013; or (iv) a non-banking financial company
13. Payment in cash or by way of a bank draft or pay order or banker’s cheque of an amount aggregating to
more than ₹ 50,000 in a financial year for one or more pre-paid payment instruments, as defined in the
policy guidelines for issuance and operation of pre-paid payment instruments issued by Reserve Bank of
India u/s 18 of the Payment and Settlement Systems Act, 2007, to a banking company or a co-operative
bank
14. *Payment aggregating to more than ₹ 50,000 in a financial year as life insurance premium to an insurer
15. *A contract for sale or purchase of securities (other than shares) where transaction value exceeds ₹ 1
lakh
16. *Sale or purchase, by any person, of shares of a company not listed in a recognised stock exchange
where transaction value exceeds ₹ 1 lakh
17. *Sale or purchase of any immovable property where amount exceeds ₹ 10 lakh or stamp value exceeds
₹ 10 lakh

The Institute of Cost Accountants of India 529


Direct Taxation

18. Sale or purchase, by any person, of goods or services of any nature other than those specified above
where transaction value exceeds ₹ 2 lakh
Note
i. Where a person, entering into any of the aforesaid transaction, is a minor and who does not have any income
chargeable to tax, he shall quote the PAN of his father or mother or guardian, as the case may be, in the
document pertaining to the said transaction
ii. Any person who does not have a PAN and who enters into any of the aforesaid transaction, he shall make a
declaration in Form No.60 giving therein the particulars of such transaction.
iii. The provisions of this rule shall not apply to the following class or classes of persons, namely:
–– the Central Government, the State Governments and the Consular Offices;
–– the non-residents in respect of the transactions other than a transaction referred above with *.
C. PAN or Aadhar Number on TDS & TCS Certificate
a. On TDS: Every person is required to intimate his PAN to the person who has deducted the tax at source
[Sec. 139A(5A)].
Exceptions
This provision is not applicable to an assessee who furnishes to the payer a declaration in writing in the
prescribed form (Form 15G or 15H) and manner to the effect that the tax on his estimated total income is
nil.
The person deducting tax at source is required to quote –
 PAN of the payee;
 PAN of himself;
 Tax Deduction Account Number (TAN) of himself,
–– in all statements, certificates furnished u/s 200(3) or 203 or 206 [Sec. 139A(5B)].
b. On TCS: Every person responsible for collecting tax in accordance with the provisions of sec.206C
shall quote PAN of every buyer or licensee or lessee in all certificates furnished in accordance with the
provisions of sec. 206C(3) or 206C(5) and in all returns prepared u/s 206C(5A)/(5B) to the Income-tax
authority [Sec. 139A(5D)]
Every buyer or licensee or lessee is required to intimate his PAN to the seller who has collected the tax
at source [Sec. 139A(5C)]
Other Point
 Every person entering into such transaction shall quote his PAN or Aadhaar number in the documents pertaining
to such transactions and also authenticate such PAN or Aadhaar number [Sec. 139(6A)]
 Every person receiving any document relating to such transactions shall ensure that PAN or Aadhaar number
has been duly quoted in such document and also ensure that such PAN or Aadhaar number is so authenticated
[Sec. 139(6B)]
–– Authentication means the process by which the PAN or Aadhaar number alongwith demographic
information or biometric information of an individual is submitted to the income-tax authority or such
other authority or agency as may be prescribed for its verification and such authority or agency verifies
the correctness, or the lack thereof, on the basis of information available with it.

530 The Institute of Cost Accountants of India


Total Income and Tax Liability of Individuals & HUF

3.8.3 Intimation for any change


Every person shall intimate the Assessing Officer, in the prescribed manner, any change in his address or in the
name & nature of his business on the basis of which PAN was allotted to him [Sec. 139A(5)(d)]

3.8.4 Quoting of Aadhaar number [Sec. 139AA]


Every person who is eligible to obtain Aadhaar number shall quote Aadhaar number:
a. in the application form for allotment of permanent account number;
b. in the return of income:
Note:
 Where the person does not possess the Aadhaar Number, the Enrolment ID of Aadhaar application form issued
to him at the time of enrolment shall be quoted.
 Every person who has been allotted PAN before 01-07-2017 and who is eligible to obtain Aadhaar number,
shall intimate his Aadhaar number to such authority on or before specified date (31-03-2022). In case of failure
to intimate the Aadhaar number, the PAN allotted to the person shall be made inoperative after the notified date
in such manner as may be prescribed.
 The provisions of this section shall not apply to notified persons or State
 Fee for default relating to intimation of Aadhaar number [Sec. 234H]: Where a person fails to intimate
his aadhaar number on or before prescribed date, he shall be liable to pay such fee, as may be prescribed, not
exceeding ₹ 1,000, at the time of making intimation after the said date.

Quick MCQs:-

1. An individual client has consulted you on the matter of PAN. He is carrying on the business of sale &
purchase of electronic appliances. His turnover is D 3,00,000 and the profit is D 75,000 for the P.Y 2023-
24. He has asked you to provide him threshold of turnover, if any, exceeding which he has to apply for
PAN.
(a) More than D 2,00,000
(b) More than D 2,50,000
(c) More than D 3,00,000
(d) More than D 5,00,000

The Institute of Cost Accountants of India 531


Direct Taxation

Self-Assessment & Intimation 3.9

A
ssessment means to assess the income of the assessee i.e. to decide the income and tax liability of the
assessee on the basis of return filed, information gathered or to the best of judgment of income tax
department. It begins with self-assessment i.e. assessment by the assessee himself.

3.9.1 Self-Assessment [Sec. 140A]


In self-assessment, assessee itself is responsible to determine its taxable income, tax liability and to pay tax
accordingly. Provision of sec. 140A is as follows -
a. Where any tax is payable (after deducting relief, rebate, advance payment of tax or tax deducted or collected at
source or MAT or AMT credit, if any, or any tax or interest payable u/s 191(2)) on the basis of return furnished
the assessee is required to pay such tax before filing the return.
Taxpoint: A return furnished without paying self-assessment tax & interest, if any, shall be treated as defective
return.
b. If any interest is payable for delayed filing of return (u/s 234A) or default in payment of advance tax (u/s 234B)
or for deferment of advance tax (u/s 234C) or fee (u/s 234F) is payable for filing return after due date, then
such interest or fee should be paid along with self-assessment tax.
Note: While calculating above interest for the purpose of self-assessment, tax on the total income declared in
the return shall be considered.
c. Where the amount paid by the assessee falls short of the aggregate of tax, interest and fee, the amount so paid
shall first be adjusted towards fee and thereafter towards interest payable and the balance, if any, shall be
adjusted towards tax payable.
d. After assessment, any amount paid under this section shall be deemed to have been paid towards such
assessment.
e. If an assessee fails to pay whole or any part of such tax or interest or both in accordance with the provisions of
sec. 140A, he shall be deemed to be an assessee in default.
Illustration 46:
Mr. Gambhir Mitra paid ₹ 42,000 together with return. Show the treatment u/s 140A from following data:

Tax liability ₹ 82,000


Interest payable u/s 234A (as per returned income) ₹ 750
Interest payable u/s 234B (as per returned income) ₹ 3,730
Interest payable u/s 234C ₹ 1,660
Fee u/s 234F ₹ 5,000
Advance tax paid ₹ 1,500
TDS ₹ 6,000

532 The Institute of Cost Accountants of India


Total Income and Tax Liability of Individuals & HUF

Solution :
Amount payable together with filing return for the A.Y.2024-25

Particulars Amount Amount


Tax liability 82,000
Add: Fee payable u/s 234F 5,000
Interest payable u/s 234A 750
Interest payable u/s 234B 3,730
Interest payable u/s 234C 1,660 11,140
Total 93,140
Less: Advance tax paid 1,500
TDS 6,000 7,500
Amount payable 85,640
Treatment of amount paid: Amount paid i.e. ₹ 42,000 shall be first adjusted with fee
payable i.e., ₹ 5,000 and then with interest payable i.e. ₹ 6,140 and the balance amount i.e.
₹ 30,860 shall be adjusted towards tax liability. Finally, tax yet to be paid is ₹ 43,640 (i.e.
₹ 85,640 – ₹ 42,000).

3.9.2 Intimation or Assessment by Income tax department


After submission of return or on non-submission of return by the assessee, assessment is made by the Income tax
department. The Assessing Officer can assess the income of the assessee in any of the following manner:
1. Intimation u/s 143(1);
2. Scrutiny Assessment u/s 143(3);
3. Best Judgment Assessment u/s 144;
4. Income Escaping Assessment u/s 147
For making assessment, the Assessing Officer can make an inquiry. Provision relating to inquiry before assessment
are as under:

3.9.3 Inquiry before assessment


1. Issue of notice to the assessee [Sec. 142(1)]
- to submit a return [Sec. 142(1)(i)]
- to produce accounts, documents etc. [Sec. 142(1)(ii) & (iii)]
2. Making inquiry [Sec. 142(2)]
3. Giving direction to get books of account audited [Sec. 142(2A) to (2D)]
4. Opportunity of being heard [Sec. 142(3)]
5. Estimate by Valuation Officer in certain cases [Sec. 142A]

Issue of notice to the assessee [Sec. 142(1)]


For the purpose of making assessment, the Assessing Officer (or any prescribed income-tax authority) may serve
a notice on any person -

The Institute of Cost Accountants of India 533


Direct Taxation

 who has submitted a return u/s 139; or


 in whose case the time allowed u/s 139(1) for furnishing the return has expired.
Such notice may relate to any of the following matter -
1. Notice to submit a return [Sec. 142(1)(i)]: If the assessee has not submitted a return of income within
specified time, the Assessing Officer (or prescribed income tax authority) may require him to submit a return
in the prescribed form on or before the date specified in the notice.
Taxpoint: In case assessee has not furnished the return of income, it is not mandatory for the Assessing Officer
to issue notice u/s 142(1)(i) if he wishes to make best judgment assessment.
2. Notice to produce accounts, documents etc. [Sec. 142(1)(ii)]: The Assessing Officer may ask the assessee to
produce such documents or accounts as he may require.
Exception: Assessing Officer shall not require the production of any accounts pertaining to a period more than
3 years prior to the previous year.
3. Notice to furnish information [Sec. 142(1)(iii)]: Assessing Officer may require the assessee to furnish in
writing information in such form and on such points or matters as he may require (including a statement of
all assets and liabilities of the assessee, whether included in the accounts or not). However, prior approval of
the Joint Commissioner shall be obtained before requiring the assessee to furnish a statement of all assets and
liabilities not included in the accounts.
Taxpoint: Notice u/s 142(1)(i) can be served only if return has not been submitted where as notice u/s 142(1)(ii)
& (iii) can be served whether return has been furnished or not.

Making inquiry [Sec. 142(2)]


For the purpose of obtaining full information in respect of the income (or loss) of any person, the Assessing Officer
may make such inquiry, as he considers necessary.
Taxpoint: U/s 142(1) Assessing Officer collects information from the assessee, however u/s 142(2) Assessing
Officer has the power to collect information from any source.

Giving direction to get books of account audited [Sec. 142(2A) to (2D)]


The Assessing Officer (after giving reasonable opportunity to the assessee) may direct the assessee to get his
accounts audited if he is of the opinion that it is necessary to do so having regard to the -
 nature and complexity of the accounts, volume of the accounts, doubts about the correctness of the accounts,
multiplicity of transactions in the accounts or specialised nature of business activity of the assessee; and
 interest of revenue.
Such direction can be issued even if the accounts of the assessee have already been audited u/s 44AB or any other
law for the time being in force
Notes
a. Such direction can be issued only with the prior approval of the Principle Chief Commissioner / Principle
Commissioner / Chief Commissioner / Commissioner.
b. The Principle Chief Commissioner / Principle Commissioner / Chief Commissioner / Commissioner nominates
such auditor.
c. Such order can be issued at any stage of the proceedings before the Assessing Officer. However, no such order
shall be issued after the completion of assessment/reassessment.

534 The Institute of Cost Accountants of India


Total Income and Tax Liability of Individuals & HUF

Time Limit for audit report: The audit report shall be furnished by the assessee within the period specified by the
Assessing Officer. The Assessing Officer has power to extend such period on an application made by the assessee
or suomotu. However, the aggregate period (fixed originally and extended) shall not exceed 180 days from the date
on which such direction is received by the assessee.
Form of audit report: The chartered accountant shall submit the report in Form 6B to the assessee. Thereafter
such report is to be submitted by the assessee to the Assessing Officer within such period as allowed by the
Assessing Officer.
Audit fees: The audit fees and audit expenditure shall be determined by the Principle Chief Commissioner /
Principle Commissioner / Chief Commissioner / Commissioner (which shall be final) and paid by the Central
Government.
Consequences of failure to get books of account audited: In case assessee fails to get books of account audited,
it -
 will be liable to Best Judgment Assessment u/s 144; and
 attracts penalty and prosecution.
Note: Penalty etc. are attracted only if there is a default by the assessee. If accountant nominated by the Commissioner
refuses to audit the accounts, the assessee cannot be held responsible

Opportunity of being heard [Sec. 142(3)]


The assessee must be given an opportunity of being heard in respect of any material gathered on the basis of any
inquiry u/s 142(2) or any audit u/s 142(2A) and is proposed to be utilised for the purpose of the assessment.
Note: Sec. 142(3) shall not be applicable in case of assessment u/s 144.

Estimation by Valuation Officer in certain cases [Sec. 142A]


 The Assessing Officer may, for the purposes of assessment or reassessment, make a reference to a Valuation
Officer to estimate the value, including fair market value, of any asset, property or investment and submit a
copy of report to him.
¾¾ “Valuation Officer” has the same meaning as in clause (r) of section 2 of the Wealth-tax Act, 1957
 The Assessing Officer may make a reference to the Valuation Officer whether or not he is satisfied about the
correctness or completeness of the accounts of the assessee.
 The Valuation Officer, on a reference made, shall, for the purpose of estimating the value of the asset, property
or investment, have all the powers that he has under section 38A of the Wealth-tax Act, 1957.
 The Valuation Officer shall, estimate the value of the asset, property or investment after taking into account
such evidence as the assessee may produce and any other evidence in his possession gathered, after giving an
opportunity of being heard to the assessee.
 The Valuation Officer may estimate the value of the asset, property or investment to the best of his judgment,
if the assessee does not co-operate or comply with his directions.
 The Valuation Officer shall send a copy of the report of the estimate made to the Assessing Officer and the
assessee, within a period of 6 months from the end of the month in which a reference is made.
 The Assessing Officer may, on receipt of the report from the Valuation Officer, and after giving the assessee an
opportunity of being heard, take into account such report in making the assessment or reassessment.

The Institute of Cost Accountants of India 535


Direct Taxation

3.9.4 Faceless inquiry or Valuation [Sec. 142B]


 The Central Government may notify a scheme for the purposes of issuing notice u/s 142(1) or making inquiry
before assessment u/s 142(2), or directing the assessee to get his accounts audited u/s 142(2A) or estimating the
value of any asset, property or investment by a Valuation Officer u/s 142A, so as to impart greater efficiency,
transparency and accountability by:
a. eliminating the interface between the income-tax authority or Valuation Officer and the assessee or any
person to the extent technologically feasible;
b. optimising utilisation of the resources through economies of scale and functional specialisation;
c. introducing a team-based issuance of notice or making of enquiries or issuance of directions or valuation
with dynamic jurisdiction.
 The Central Government may direct (upto 31-03-2022) that any of the provisions of this Act shall not apply or
shall apply with such exceptions, modifications and adaptations as may be specified in the notification.
 Every notification issued under sub-section (1) and sub-section (2) shall, as soon as may be after the notification
is issued, be laid before each House of Parliament.

3.9.5 Intimation / Assessment by Assessing Officer


The Assessing Officer makes the following order:
Intimation [Sec. 143(1)]
On the basis of return filed; or
Regular assessment
 On the basis of further evidence gathered by him [Scrutiny Assessment u/s 143(3)]
 On the basis of best of his judgement [Best Judgement Assessment u/s 144]

3.9.6 Intimation [Sec. 143(1)]


Where a return has been made u/s 139 or in response to a notice u/s 142(1), such return shall be processed in the
following manner, namely:—
a. the total income or loss shall be computed after making the following adjustment:
i. any arithmetical error in the return;
ii. an incorrect claim, if such incorrect claim is apparent from any information in the return;
iii. disallowance of loss claimed, if return of the previous year for which set off of loss is claimed was
furnished after the due date;
iv. disallowance of expenditure or increase in income indicated in the audit report but not taken into account
in computing the total income in the return;
v. disallowance of deduction claimed u/s 10AA or under any of the provisions of Chapter VI-A under the
heading “C.—Deductions in respect of certain incomes”, if the return is furnished after the due date;
b. the tax, interest and fee, if any, shall be computed on the total income computed above;

536 The Institute of Cost Accountants of India


Total Income and Tax Liability of Individuals & HUF

c. the sum payable by (or the amount of refund due to), the assessee shall be determined after adjustment of the
tax, interest and fee, if any, by any TDS, TCS, advance tax paid, any relief, tax paid on self-assessment and
any amount paid otherwise by way of tax, interest or fee;
d. an intimation shall be prepared or generated and sent to the assessee specifying the sum determined to be
payable by, or the amount of refund due to, the assessee; and
e. the amount of refund due to the assessee in pursuance of the determination shall be granted to the assessee.
f. An intimation shall also be sent to the assessee in a case where the loss declared in the return by the assessee
is adjusted but no tax or interest or fee is payable by, or no refund is due to, him.
Time limit for intimation: No intimation shall be sent after the expiry of 9 months from the end of the financial
year in which the return is made. The period of limitation will run from the date of filing of latest revised return.
Notes
 An incorrect claim apparent from any information in the return shall mean a claim, on the basis of an entry,
in the return,—
a. of an item, which is inconsistent with another entry of the same or some other item in such return;
b. in respect of which the information required to be furnished under this Act to substantiate such entry has
not been so furnished; or
c. in respect of a deduction, where such deduction exceeds specified statutory limit which may have been
expressed as monetary amount or percentage or ratio or fraction;
 The acknowledgment of the return shall be deemed to be intimation where either no sum is payable by the
assessee or no refund is due to him.
 In case, where refund becomes due to the assessee u/s 143(1) and the Assessing Officer is of the opinion,
having regard to the fact that a notice has been issued u/s 143(2) in respect of such return, that the grant of
the refund is likely to adversely affect the revenue, he may, for reasons to be recorded in writing and with the
previous approval of the Principal Commissioner or Commissioner, as the case may be, withhold the refund
up to the date on which the assessment is made [Sec. 241A]

The Institute of Cost Accountants of India 537


Direct Taxation

Exercise

A. Theoretical Questions:
¾¾ Multiple Choice Questions:
1. Mr. X’s minor daughter earned ₹ 50,000 from his special talent. This income will be clubbed with –
a. The income of Mr. X
b. The income of Mrs. X
c. Mr. X or Mrs. X, whoever’s income is higher
d. It will not be clubbed

2. Income arising to a minor married daughter shall be –


a. assessed in the hands of minor married daughter
b. clubbed with the income of that parent whose total income is higher
c. Exempt from tax
d. clubbed with the income of her spouse

3. Unabsorbed business losses cannot be carried for more than -


a. 7 assessment years
b. 8 assessment years
c. 10 assessment years
d. 12 assessment years

4. Deduction u/s. 80JJA is available if the assessee


a. Is engaged in scientific research
b. Sets up an industrial unit in a backward area
c. Is engaged in agriculture business
d. Is engaged in the business of collecting and processing biodegradable waste.

5. Maximum limit for deduction u/s 80TTA is:


a. ₹ 50,000
b. ₹ 10,000
c. ₹ 40,000
d. ₹ 25,000

6. Advance tax is required to be paid by all assessee only if estimated advance tax liability is
a. ₹ 5,000 or more
b. ₹ 10,000 or more

538 The Institute of Cost Accountants of India


Total Income and Tax Liability of Individuals & HUF

c. More than zero


d. ₹ 50,000 or more

7. TDS is not required to be deducted u/s 194A if the amount of interest on loan does not exceed:
a. ₹ 5,000
b. ₹ 2,500
c. ₹ 7,500
d. ₹ 20,000

8. On salary, tax is required to be deducted at the time of:


a. Payment or crediting the employee, whichever is earlier
b. Crediting the employee
c. Payment
d. Retirement of employee

9. Where assessment has not been completed, belated income tax return for assessment year 2024-25
can be filed upto:
a. 31.12.2024
b. 31.01.2025
c. 31.03.2025
d. 31.12.2025

10. When assessment has not been completed, revised return can be filed within ______ from the end
of the relevant previous year.
a. 9 months
b. 6 months
c. 12 months
d. 2 years
[Answer : 1-d; 2-b; 3-b; 4-d; 5-b; 6-b; 7-a; 8-c; 9-a; 10-a]
¾¾ Short Essay Type Questions
1. Discuss the provisions under IT Act in respect of clubbing of remuneration of spouse
2. Discuss whether long term capital loss can be adjusted against STCG
3. State the provision relating to deduction u/s 80D
4. Who is liable to pay advance tax?
5. Who is liable to file return of income mandatorily?

The Institute of Cost Accountants of India 539


Direct Taxation

B. Numerical Questions
¾¾ Multiple Choice Questions
1. Mr. A gifted debenture of ₹ 1,00,000 to his wife. She received ₹ 10,000 interest which she reinvests
and earns ₹ 1,000. This ₹ 1,000 will be taxable in the hands of –
a. Mr. A
b. Mrs. A
c. Not Taxable
d. Mr. A or Mrs. A, at the choice of the Assessing Officer

2. Mr. A has three minor children deriving interest from bank deposits to the tune of ₹ 2,000, ₹ 1,300,
₹ 1,600 respectively. Exemption available under section 10(32) of the Income Tax is –
a. ₹ 4,900
b. ₹ 4,300
c. ₹ 4,500
d. ₹ 5,000
[Answer : 1-b; 2-b]

¾¾ Comprehensive Numerical Problems

1. From the following incomes and losses for the Previous Year 2023-24, compute Gross Total
Incomes of Mr. Sen:

Income from Business A 2,00,000
Loss from Business B 1,20,000
Long term Capital Gain 50,000
Short term Capital Loss 35,000
Bank Interest on fixed deposit 10,000
Brought forward loss of the year 2020-21 are as follows:
Loss from Business B 40,000
Long term Capital Loss 20,000
[Hints: ₹ 50,000]

2. Mrs. R Mukherjee furnished the following information for the previous year 2023-24:
Particulars Amount
Donation to Prime Minster’s National Relief Fund 15,000
Donation to Jawaharlal Nehru Memorial Fund 10,000
Donation to a notified temple 30,000

540 The Institute of Cost Accountants of India


Total Income and Tax Liability of Individuals & HUF

Particulars Amount
Donation of cloth worth ₹ 10,000 to Bharat Sevashram Sangha to be distributed among Ayla
victims
Donation to an approved University 2,000
Gross Total Income 3,00,000
Payment of life insurance premium on the life of her daughter ₹ 20,000 (Policy value ₹ 2,00,000)
Compute total income
[Hints: ₹ 2,44,000]

¾¾ Unsolved Case
Shri Mayur has two minor children named Lakshmi (age 10) and Sarath (age 14). Following details
pertain to the minor children for the year ended 31.03.2024:
i. Minor Lakshmi won Carnatic music competition in TV channel and was awarded cash prize of
₹ 2,00,000.
ii. Minor Lakshmi received cash gifts from friends of Mayur ₹ 43,000. No single gift exceeded
₹ 10,000.
iii. Minor Sarath received gift of gold chain whose fair market value was ₹ 80,000 from his maternal
uncle on the occasion of his 14th birthday. He also received cash gift of ₹ 12,000 from friends of
Mayur on his birthday.
iv. Out of accumulated savings of daughter Lakshmi, one vacant land was acquired. The stamp duty
value of the land ₹ 3,60,000. The documented value of the land ₹ 3,20,000.
On the basis of aforesaid information, you are requested to answer the following:
a. What will be the income liable to be clubbed in hands of Shri Mayur on account of Lakshmi?
b. What will be the income liable to be clubbed in hands of Shri Mayur on account of Sarath?
c. What will be the income liable to be taxed in the hands of Lakshmi?
[Hints: (a) ₹ 41500; (b) ₹ 10,500; (c) ₹ 2,00,000]

¾¾ References:
https://www.incometaxindia.gov.in/
https://www.incometax.gov.in/
https://www.indiabudget.gov.in/

The Institute of Cost Accountants of India 541


Direct Taxation

542 The Institute of Cost Accountants of India

You might also like